You are on page 1of 701

CCXIIISEET

SETS, RELATIONS AND BINARY OPERATIONS

SETS
SET
A set is a collection of well defined objects which are distinct from each other. Set are generally denoted by
capital letters A, B, C, ........ etc. and the elements of the set by small letters a, b, c ....... etc.
If a is an element of a set A, then we write a  A and say a belongs to A.
If a does not belong to A then we write a  A,
e.g. the collection of first five prime natural numbers is a set containing the elements 2, 3, 5, 7, 11.

METHODS TO WRITE A SET :

(i) Roster Method or Tabular Method : In this method a set is described by listing elements, separated
by commas and enclose then by curly brackets. Note that while writing the set in roster form, an element
is not generally repeated e.g. the set of letters of word SCHOOL may be written as {S, C, H, O, L}.

(ii) Set builder form (Property Method) : In this we write down a property or rule which gives us all the
element of the set.
A = {x : P(x)} where P(x) is the property by which x  A and colon ( : ) stands for ‘such that’
Example # 1 : Express set A = {x : x  N and x = 2n for n  N} in roster form
Solution : A = {2, 4, 6,.........}

Example # 2 : Express set B = {x2 : x < 4, x  W} in roster form


Solution : B = {0, 1, 4, 9}

Example # 3 : Express set A = {2, 5, 10, 17, 26} in set builder form
Solution : A = {x : x = n2 + 1, nN, 1  n  5}

TYPES OF SETS

Null set or empty set : A set having no element in it is called an empty set or a null set or void set, it is
denoted by  or { }. A set consisting of at least one element is called a non-empty set or a non-void set.

Singleton set : A set consisting of a single element is called a singleton set.

Finite set : A set which has only finite number of elements is called a finite set.

Order of a finite set : The number of elements in a finite set A is called the order of this set and
denoted by O(A) or n(A). It is also called cardinal number of the set.
e.g. A = {a, b, c, d}  n(A) = 4
Infinite set : A set which has an infinite number of elements is called an infinite set.
Equal sets : Two sets A and B are said to be equal if every element of A is member of B, and every
element of B is a member of A. If sets A and B are equal, we write A = B and if A and B are not equal
then A  B
Equivalent sets : Two finite sets A and B are equivalent if their number of elements are same
i.e. n(A) = n(B)

e.g. A = {1, 3, 5, 7}, B = {a, b, c, d}  n(A) = 4 and n(B) = 4


 A and B are equivalent sets

RESONANCE Sets, Relations and Binary Operations - 1


CCXIIISEET

Note - Equal sets are always equivalent but equivalent sets may not be equal
Example # 4 : Identify the type of set :
(i) A = {x  N : 5 < x < 6}
(ii) A = {a, b, c}
(iii) A = {1, 2, 3, 4, .......}
(iv) A = {1, 2, 6, 7} and B = {6, 1, 2, 7, 7}
(v) A = {0}
Solution : (i) Null set
(ii) finite set
(iii) infinite set
(iv) equal sets
(v) singleton set

Self Practice Problem :


(1) Write the set of all integers 'x' such that |x – 3| < 8.
(2) Write the set {1, 2, 3, 6} in set builder form.
(3) If A = {x : |x| < 2, x  Z} and B = {–1, 1} then find whether sets A and B are equal or not.

Answers (1) [–4, –3, –2, –1, 0, 1, 2, 3, 4, 5, 6, 7, 8, 9, 10]


(2) {x : x is a natural number and a divisor of 6}
(3) Not equal sets

SUBSET AND SUPERSET :


Let A and B be two sets. If every element of A is an element B then A is called a subset of B and B is called
superset of A. We write it as A  B.
e.g. A = {1, 2, 3, 4} and B = {1, 2, 3, 4, 5, 6, 7}  A  B
If A is not a subset of B then we write A / B

PROPER SUBSET :
If A is a subset of B but A  B then A is a proper subset of B and we write A  B. Set A is not proper subset of
A so this is improper subset of A

Note : (i) Every set is a subset of itself

(ii) Empty set  is a subset of every set

(iii) A  B and B  A  A = B

(iv) The total number of subsets of a finite set containing n elements is 2n.
(v) Number of proper subsets of a set having n elements is 2n – 1.
(vi) Empty set  is proper subset of every set except itself.

POWER SET :
Let A be any set. The set of all subsets of A is called power set of A and is denoted by P(A)
Example # 5 : Examine whether the following statements are true or false :
(i) {a, b} / {b, c, a}
(ii) {a, e} / {x : x is a vowel in the English alphabet}
(iii) {1, 2, 3}  {1, 3, 5}
(iv) {a} {a, b, c}
Solution : (i) False as {a, b} is subset of {b, c, a}
(ii) True as a, e are vowels
(iii) False as element 2 is not in the set {1, 3, 5}
(iv) False as a {a, b, c} and {a}  {a, b, c}
RESONANCE Sets, Relations and Binary Operations - 2
CCXIIISEET

Example # 6 : Find power set of set A = {1, 2}


Solution : P(A) = {, {1}, {2}, {1, 2}}

Example # 7 : If  denotes null set then find P(P(P()))


Solution : Let P() = {}
P(P()) = {,{}}
P(P(P())) = {, {}, {{}}, {, {}}}

Self Practice Problem :


(4) State true/false :A = {1, 3, 4, 5}, B = {1, 3, 5} then A  B.

(5) State true/false :A = {1, 3, 7, 5}, B = {1, 3, 5, 7} then A  B.

(6) State true/false :[3, 7]  (2, 10)

Answers (4) False (5) False (6) True

UNIVERSAL SET :
A set consisting of all possible elements which occur in the discussion is called a universal set and is
denoted by U.
e.g. if A = {1, 2, 3}, B = {2, 4, 5, 6}, C = {1, 3, 5, 7} then U = {1, 2, 3, 4, 5, 6, 7} can be taken as the
universal set.

SOME OPERATION ON SETS :


(i) Union of two sets : A  B = {x : x  A or x  B}
e.g. A = {1, 2, 3}, B = {2, 3, 4} then A  B = {1, 2, 3, 4}

(ii) Intersection of two sets : A  B = {x : x  A and x  B}


e.g. A = {1, 2, 3}, B = {2, 3, 4} then A  B = {2, 3}

(iii) Difference of two sets : A – B = {x : x  A and x  B}. It is also written as AB'.


Similarly B – A = B  A' e.g. A = {1, 2, 3}, B = {2, 3, 4} ; A – B = {1}

(iv) Symmetric difference of sets : It is denoted by A  B and A  B = (A – B)  (B – A)

(v) Complement of a set : A' = {x : x  A but x  U} = U – A


e.g. U = {1, 2,........, 10}, A = {1, 2, 3, 4, 5} then A' = {6, 7, 8, 9, 10}
(vi) Disjoint sets : If A  B = , then A, B are disjoint
e.g. If A = {1, 2, 3}, B = {7, 8, 9} then A  B = 

VENN DIAGRAM :
Most of the relationships between sets can be represented by means of diagrams which are known as venn
diagrams.These diagrams consist of a rectangle for universal set and circles in the rectangle for subsets of
universal set. The elements of the sets are written in respective circles.
For example If A = {1, 2, 3}, B = {3, 4, 5}, U = {1, 2, 3, 4, 5, 6, 7, 8} then their venn diagram is

RESONANCE Sets, Relations and Binary Operations - 3


CCXIIISEET

AB AB A–B B–A

A' (A B) = (A – B)  (B – A) Disjoint

LAWS OF ALGEBRA OF SETS (PROPERTIES OF SETS):


(i) Commutative law : (A  B) = B  A ; A  B = B  A
(ii) Associative law : (A  B)  C = A  (B  C) ; (A  B)  C = A  (B  C)
(iii) Distributive law : A (B  C) = (A  B)  (A  C) ; A  (B  C) = (A  B)  (A  C)
(iv) De-morgan law : (A  B)' = A'  B' ; (A  B)' = A'  B'
(v) Identity law : A  U = A ; A   = A
(vi) Complement law : A  A' = U, A  A' = , (A')' = A
(vii) Idempotent law : A  A = A, A  A = A
NOTE : (i) A – (B  C) = (A – B)  (A – C) ; A – (B  C) = (A – B)  (A – C)
(ii) A   = , A  U = U

Example # 8 : Let A = {2, 4, 6, 8} and B = {6, 8, 10, 12} then find A  B


Solution : A  B = {2, 4, 6, 8, 10, 12}

Example # 9 : Let A = {1, 2, 3, 4, 5, 6}, B = {2, 4, 6, 8}. Find A – B and B – A.


Solution : A – B = {x : x  A and x  B} = {1, 3, 5}
similarly B – A = {8}

Example # 10 : State true or false :


(i) A  A =  (ii)  A = A
Solution : (i) false because A  A' = U
(ii) true as  A = U A = A

Example # 11 : Use Venn diagram to prove that A  B  B  A.

Solution :

From venn diagram we can conclude that B  A.

Example # 12 : Prove that if A  B = C and A  B =  then A = C – B.


Solution : Let x A xAB xC ( AB = C)
Now AB =  xB ( x  A)
 x C – B ( x  C and x  B)
 AC–B Let xC–B  x  C and x  B
 xAB and x  B xA  C – B A
 A=C–B

RESONANCE Sets, Relations and Binary Operations - 4


CCXIIISEET

Self Practice Problem :


(7) Find A  B if A = {x : x = 2n + 1, n  5, n  N} and B = {x : x = 3n – 2, n  4, n  N}.

(8) Find A – (A – B) if A = {5, 9, 13, 17, 21} and B = {3, 6, 9, 12, 15, 18, 21, 24}

Answers (7) {1, 3, 4, 5, 7, 9, 10, 11} (8) {9, 21}

SOME IMPORTANT RESULTS ON NUMBER OF ELEMENTS IN SETS :


If A, B, C are finite sets and U be the finite universal set then

(i) n(A  B) = n(A) + n(B) – n(A  B)

(iii) n(A – B) = n(A) – n(A  B)

(v) n(A  B  C) = n(A) + n(B) + n(C) – n(A  B) – n(B  C) – n(A  C) + n(A  B  C)

(vi) Number of elements in exactly two of the sets A, B, C


= n(A  B) + n(B  C) + n(C  A) – 3n(A  B  C)

(vii) Number of elements in exactly one of the sets A, B, C


= n(A) + n(B) + n(C) – 2n(A  B) – 2n(A  B) – 2n(B  C) – 2n(A  C) + 3n(A  B  C)

Example # 13 : In a group of 40 students, 26 take tea, 18 take coffee and 8 take neither of the two. How many
take both tea and coffee ?
Solution n(U) = 40, n(T) = 26, n(C) = 18
n(T  C) = 8  n(T  C) = 8
 n(U) – n(T  C) = 8
 n(T  C) = 32
 n(T) + n(C) – n(T  C) = 32
 n(T  C) = 12

Example # 14 : In a group of 50 persons, 14 drink tea but not coffee and 30 drink tea. Find
(i) How many drink tea and coffee both ? (ii) How many drink coffee but not tea ?
Solution T : people drinking tea
C : people drinking coffee
(i) n(T) = n(T – C) + n(T  C)  30 = 14 + n(T  C)  n(T  C) = 16

(ii) n(C – T) = n(T  C) – n(T) = 50 – 30 = 20

Self Practice Problem :


(9) Let A and B be two finite sets such that n(A – B) = 15, n(A  B) = 90, n(A  B) = 30. Find n(B)

(10) A market research group conducted a survey of 1000 consumers and reported that 720
consumers liked product A and 450 consumers liked product B. What is the least number that
must have liked both products ?

Answers (9) 75 (10) 170

RESONANCE Sets, Relations and Binary Operations - 5


CCXIIISEET

RELATIONS

ORDERED PAIR :
A pair of objects listed in a specific order is called an ordered pair. It is written by listing the two objects in
specific order separating them by a comma abd enclosing the pair in parantheses.
In the ordered pair (a, b), a is called the first element and b is called the second element.
Two ordered pairs are set to be equal if their corresponding elements are equal. i.e.
(a, b) = (c, d) if a = c and b = d.

CARTESIAN PRODUCT :
The set of all possible ordered pairs (a, b), where a  A and b  B i.e. {(a, b) ; a  A and b  B} is called
the cartesian product of A to B and is denoted by A × B. Usually A × B  B × A.
Similarly A × B × C = {(a, b, c) : a  A, b  B, c  C} is called ordered triplet.

RELATION:
Let A and B be two sets. Then a relation R from A to B is a subset of A × B. Thus, R is a relation from A to B
 R  A × B. The subsets is derived by describing a relationship between the first element and the second
element of ordered pairs in A × B e.g. if A = {1, 2, 3, 4, 5, 6, 7, 8} and B = {1, 2, 3, 4, 5} and
R = {(a, b) : a = b2, a  A, b  B} then R = {(1, 1), (4, 2), (9, 3)}. Here a R b  1 R 1, 4 R 2, 9 R 3.

NOTE :
(i) Let A and B be two non-empty finite sets consisting of m and n elements respectively. Then A × B
consists of mn ordered pairs. So total number of subsets of A × B i.e. number of relations from A to
B is 2mn.
(ii) A relation R from A to A is called a relation on A.

DOMAIN AND RANGE OF A RELATION :


Let R be a relation from a set A to a set B. Then the set of all first components of coordinates of the ordered
pairs belonging to R is called to domain of R, while the set of all second components of corrdinates of the
ordered pairs in R is called the range of R.
Thus, Dom (R) = {a : (a, b)  R} and Range (R) = {b : (a, b)  R}
It is evident from the definition that the domain of a relation from A to B is a subset of A and its range is a
subset of B.

Example # 1 : If A = {1, 2} and B = {3, 4}, then find A × B.


Solution : A × B = {(1, 3), (1, 4), (2, 3), (2, 4)}

Example # 2 : Let A = {1, 3, 5, 7} and B = {2, 4, 6, 8} be two sets and let R be a relation from A to B defined by
the phrase "(x, y)  R  x > y". Find relation R and its domain and range.
Solution : Under relation R, we have 3R2, 5R2, 5R4, 7R4 and 7R6
i.e. R = {(3, 2), (5, 2), (5, 4), (7, 2), (7, 4), (7, 6)}
 Dom (R) = {3, 5, 7} and range (R) = {2, 4, 6}

Example # 3 : Let A = {2, 3, 4, 5, 6, 7, 8, 9}. Let R be the relation on A defined by


{(x, y) : x  A, y  A and x divides y}.
Find domain and range of R.
Solution: The relation R is
R = {(2, 2), (2, 4), (2, 6), (2, 8), (3, 3), (3, 6), (3, 9), (4, 4), (4, 8), (5, 5), (6, 6), (7, 7), (8, 8), (9, 9)}
Domain of R = {2, 3, 4, 5, 6, 7, 8, 9} = A
Range of R = {2, 3, 4, 5, 6, 7, 8, 9} = A

RESONANCE Sets, Relations and Binary Operations - 6


CCXIIISEET

Self Practice Problem :


(1) If (2x + y, 7) = (5, y – 3) then find x and y.

(2) If A × B = {(1, 2), (1, 3), (1, 6), (7, 2), (7, 3), (7, 6)} then find sets A and B.

(3) If A = {x, y, z} and B = {1, 2} then find number of relations from A to B.

(4) Write R = {(4x + 3, 1 – x) : x  2, x  N}

5
Answers (1) x = – , y = 10 (2) A = {1, 7}, B = {2, 3, 6}
2
(3) 64 (4) {(7, 0), (11, –1)}

TYPES OF RELATIONS :

In this section we intend to define various types of relations on a given set A.


(i) Void relation : Let A be a set. Then   A × A and so it is a relation on A. This relation is called
the void or empty relation on A.

(ii) Universal relation : Let A be a set. Then A × A  A × A and so it is a relation on A. This relation
is called the universal relation on A.

(iii) Identity relation : Let A be a set. Then the relation IA = {(a, a) : a  A} on A is called the identity
relation on A. In other words, a relation IA on A is called the identity relation if every element of A
is related to itself only.

(iv) Reflexive relation : A relation R on a set A is said to be reflexive if every element of A is related
to itself. Thus, R on a set A is not reflexive if there exists an element a  A such that (a, a)  R.

Note : Every identity relation is reflexive but every reflexive relation in not identity.
(v) Symmetric relation : A relation R on a set A is said to be a symmetric relation
iff (a, b)  R  (b ,a)  R for all a, b  A. i.e. a R b  b R a for all a, b  A.

(vi) Transitive relation : Let A be any set. A relation R on A is said to be a transitive relation
iff (a, b)  R and (b, c)  R  (a, c)  R for all a, b, c  A i.e. a R b and b R c  a R c
for all a, b, c  A

(vii) Equivalence relation : A relation R on a set A is said to be an equivalence relation on A iff


(i) it is reflexive i.e. (a, a)  R for all a  A
(ii) it is symmetric i.e. (a, b)  R  (b, a)  R for all a, b  A
(iii) it is transitive i.e. (a, b)  R and (b, c)  R  (a, c)  R for all a, b  A

Example # 4 : Which of the following are identity relations on set A = {1, 2, 3}.
R1 = {(1, 1), (2, 2)}, R2 = {(1, 1), (2, 2), (3, 3), (1, 3)}, R3 = {(1, 1), (2, 2), (3, 3)}.
Solution: The relation R3 is idenity relation on set A.
R1 is not identity relation on set A as (3, 3)  R1.
R2 is not identity relation on set A as (1, 3)  R2

Example # 5 : Which of the following are reflexive relations on set A = {1, 2, 3}.
R1 = {(1, 1), (2, 2), (3, 3), (1, 3), (2, 1)}, R2 = {(1, 1), (3, 3), (2, 1), (3, 2)}..
Solution : R1 is a reflexive relation on set A.
R2 is not a reflexive relation on A because 2  A but (2, 2)  R2.

Example # 6 : Prove that on the set N of natural numbers, the relation R defined by x R y  x is less than y is
transitive.
Solution : Because for any x, y, z  N x < y and y < z  x < z  x R y and y R z  x R z. so R is
transitive.
RESONANCE Sets, Relations and Binary Operations - 7
CCXIIISEET
Example # 7 : Let T be the set of all triangles in a plane with R a relation in T given by R = {(T1 , T2) : T1 is congruent
to T2}. Show that R is an equivalence relation.
Solution : Since a relation R in T is said to be an equivalenece relation if R is reflexive, symmetric and
transitive.
(i) Since every triangle is congruent to itself
 R is reflexive
(ii) (T1 , T2)  R  T1 is congruent to T2
 T2 is congruent to T1
 (T2, T1)  R
Hence R is symmetric
(iii) Let (T1, T2)  R and (T2, T3)  R
 T1 is congruent to T2
and T2 is congruent to T3
 T1 is congruent to T3
 (T1, T3)  R
 R is transitive
Hence R is an equivalence relation.

Example # 8 : Show that the relation R in R defined as R = {(a, b) : a  b} is transitive.


Solution : Let (a, b)  R and (b, c)  R
 (a  b) and b  c  a  c
 (a, c)  R
Hence R is transitive.

Example # 9 : Show that the relation R in the set {1, 2, 3} given by R = {(1, 2), (2, 1)} is symmetric.
Solution : Let (a, b)  R [ (1, 2)  R]
 (b, a)  R [ (2, 1)  R]
Hence R is symmetric.

Self Practice Problem :


(5) Let L be the set of all lines in a plane and let R be a relation defined on L by the rule (x ,y)  R
 x is perpendicular to y. Then prove that R is a symmetric relation on L.

(6) Let R be a relation on the set of all lines in a plane defined by (1, 2)  R  line 1 is parallel to
line 2. Prove that R is an equivalence relation.

BINARY OPERATIONS
BINARY OPERATION

A binary operation * on a set A is a function * : A × A  A. We denote * (a, b) by a * b.

Example # 1 : Show that addition, subtraction and multiplication are binary operations on R, but division is not a
binary operation on R. Further, show that division is a binary operation on the set R* of non zero real
numbers.
Solution : + : R × R  R is given by
(a, b)  a + b
– : R × R  R is given by
(a, b)  a – b
× : R × R  R is given by
(a, b)  ab
Since ‘+’, ‘–’ and ‘×’ are functions, they are binary operations on R.
a
But  : R × R  R, given by (a, b)  , is not a function and hence not a binary operation, as for
b
a
b = 0, is not defined.
b
a
However  : R* × R*  R*, given by (a, b)  is a function and hence a binary operation on R*.
b

RESONANCE Sets, Relations and Binary Operations - 8


CCXIIISEET
Example # 2 : Show that subtraction and division are not binary operations on N.
Solution : – : N × N  N, given by (a, b)  a – b is not binary operation as the image of (3, 5) under ‘–’ is
3 – 5 = –2 
 N. Similarly  : N × N  N given by (a, b)  a  b is not a binary operation as the image
3
of (3, 5) under  is 3  5 =  N.

5

Example # 3 : Show that * : R × R  R given by (a, b)  a + 4b2 is a binary operation.


Solution : Since * carries each pair (a, b) to a unique element a + 4b2 in R, * is a binary operation on R.

Example # 4 : Show that the  : R × R  given by (a, b)  max {a, b} and  : R × R  R given by (a, b)  min
{a, b} are binary operations.
Solution : Since  carries each pair (a, b) in R × R to a unique element namely maximum of a and b lying in
R,  is a binary operation. Using the similar argument, one can say that  is also a binary
operation.

OPERATION TABLE
When number of elements in a set A is small, we can express a binary operation * on the set A through a
table called the operation table for the operation *. For example consider A = {1, 2, 3}. Then the operation *
on A defined as * : R × R  given by (a, b)  max {a, b} can be expressed by the following operation table.
Here *(1, 3) = 3, *(2, 3) = 3, *(1, 2) = 2.
* 1 2 3
1 1 2 3
2 2 2 3
3 3 3 3

LAWS OF BINARY OPERATIONS :


(i) Commutative Law : A binary operation * on the set X is called commutative if a * b = b * a for every
a, b  X.

(ii) Associative Law : A binary operation * : A × A  A is said to be associative if (a * b) * c = a * (b * c) 


a, b, c  A.

Example # 5 : : Show that + : R × R  R and × : R × R  R are commutative binary operations but – : R × R  R


and  : R × R  R are not commutative.
Solution : Since a + b = b + a and a × b = b × a  a b  R so '+' and '×' are commutative binary operations.
However '–' is not commutative since 3 – 4  4 – 3.
Similarly 3  4  4  3 shows that  is not commutative.

Example # 6 : Show that * : R × R  R defined by a * b = a + 2b is not commutative.


Solution : Since 3 * 4 = 3 + 8 = 11 and 4 * 3 = 4 + 6 = 10, showing that the operation * is not commutative.

Example # 7 : Show that addition and multiplication are associative binary operations on R. But subtraction is not
associative on R. Division is not associative on R.
Solution : Addition and multiplication are associative since (a + b) + c = a + (b + c) and (a × b) × c
= a × (b × c) 
a, b, c  R. However subtraction and division are not associative as (8 – 5) – 3  8 – (5 – 3) and
(8  5) 3  8  (5  3).

Example # 8 : Show that * : R × R  R given by a * b  a + 2b is not associative.


Solution : The operation * is not associative since
(8 * 5) * 3 = (8 + 10) * 3 = (8 + 10) + 6 = 24.
while 8 * (5 * 3) = 8 * (5 + 6) = 8 * 11 = 8 + 22 = 30.

Self Practice Problem :


(1) Find whether the operation * on Z defined by a * b = ab is binary or not.

(2) Let * be a binary operation on Q. Find 2 * 3 if a * b = 3a – 5b

RESONANCE Sets, Relations and Binary Operations - 9


CCXIIISEET

(3) Show that the binary operations * on Q defined by a * b = a + 12b + ab is not associative.
Answers
(1) No (2) –9

EXISTENCE OF IDENTITY ELEMENT :


Given a binary operation * : A × A  A an element e  A if it exists is called identity for the operation *
if a * e = a = e * a  a  A.

Example # 9 : Show that zero is the identity for addition on R but 1 is the identity for multiplicaton on R. But there
is no identity element for the operations – : R × R  R and  : R* × R*  R*.
Solution. a + 0 = 0 + a = a and a × 1 = a = 1 × a  a  R implies that 0 and 1 are identity elements for the
operations '+' and '×' respectively. Further there is no element e in R with a – e = e – a  a. Similarly
we can not find any element e in R* such tht a  e = e  a  a in R*. Hence, '–' and '' do not have
identity element.

EXISTENCE OF INVERSE :
Given a binary operation * : A × A  A with the identity element e in A, an element a  A is said to be invertible
with respect to the operation * if there exists an element b in A such that a * b = e = b * a and b is called the
inverse of a and is denoted by a–1.

Remark :Zero is identity for the addition operation on R but it is not identity for the addition operation on N as 0 
 N.
In fact the addition operation on N does not have any identity.
One further notices that for the addition operation + : R × R  R, given any a  R, there exists – a in R such
that a + (–a) = 0 (identity for '+') = (–a) + a.
Similarly for the multiplication operation on R given any a  0 in R, we can choose 1/a in R such that
a × 1/a = 1(identity for '×') = 1/a × a. This leads to the existence of inverse.

Example # 10 : Show that –a is the inverse of a for the addition operation '+' on R and 1/a is the inverse of a  0 for
the multiplication operation '×' on R.
Solution : As a + (–a) = a – a = 0 and (–a) + a = 0, –a is the inverse of a for addition.
1 1 1
Similarly for a  0 a × =1= × a implies that is the inverse of a for multiplication.
a a a
1
Example # 11 : Show that –a is not the inverse of a  N for the addition operation + on N and is not the inverse
a
of a  N for multiplication operation × on N for a  1.
Solution : Since –a  N, –a can not be inverse of a for addition operation on N. Althrough –a satisfies
a + (–a) = 0 = (–a) + a.
1
Similarly for a  1 in N,  N which implies that other than 1, no element of N has inverse for

a
multiplication operation on N.

Self Practice Problem :


(4) Let * be a binary operation 'addition' on set of integers find the identity element of (Z, *).
(5) Consider the binary operations * Q × Q  Q defind by a * b = a + b + ab ; a, b  Q. Find inverse
of rational number 5.
Answers
(4) 0 (5) – 5/6

RESONANCE Sets, Relations and Binary Operations - 10


CCXIIISEET

OBJECTIVE QUESTIONS

* Marked Questions are having more than one correct option.


SECTION (A) : Representation of set, Types of sets, Subset, Power Set.
A-1. Which of the following is the empty set
(A) {x : x is a real number and x2 – 1 = 0} (B) {x : x is a real number and x2 + 1 = 0}
(C) {x : x is a real number and x2 – 9 = 0} (D) {x : x is a real number and x2 = x + 2}

A-2. The set A = {x : x R, x2 = 16 and 2x = 6} is


(A) Null set (B) Singleton set (C) Infinite set (D) None of these

A-3. If A = {x :|x| < 3, x  Z} then the number of subsets of A is -


(A) 120 (B) 30 (C) 31 (D) 32

A-4. The number of subsets of the power set of set A = {7, 10, 11} is
(A) 32 (B) 16 (C) 64 (D) 256

SECTION (B) : Venn diagrams, Algebra of sets.


B-1. Sets A and B have 3 and 6 elements respectively. What can be the minimum number of elements in AB ?
(A) 3 (B) 6 (C) 9 (D) 18

B-2. If the sets A and B are defined as

1
A = {(x, y) : y = , x  R, x  0}
x
B = {(x, y) : y = – x, x  R}, then
(A) A B = A (B) A  B = B (C) A  B =  (D) None of these

B-3. Let A = {x : x  R, |x| < 1} : B = {x : x  R, |x – 1|  1} and A B = R – D, then the set D is


(A) {x : 1 < x  2} (B) {x : 1  x < 2} (C) {x : 1  x  2} (D) None of these

B-4. Let A and B be two sets. Then


(A) A  B  A  B (B) A  B  A  B (C) A  B = A  B (D) None of these

B-5. The shaded region in the given figure is

(A) A  (B  C) (B) A  (B  C) (C) A  (B – C) (D) A – (B  C)

B-6. Let n(U) = 700, n(A) = 200, n(B) = 300 and n(A  B) = 100, then n(A'  B') =
(A) 400 (B) 600 (C) 300 (D) 200

B-7. If X = {4n – 3n – 1 : n  N} and Y = {9(n – 1) ; n  N}, then X Y is equal to


(A) X (B) Y (C) N (D) None of these

RESONANCE Sets,Relations and Binary Operations - 11


CCXIIISEET

SECTION (C) : Theorems on cardinal number


C-1. In a college of 300 students, every student reads 5 newspapers and every newspaper is read by 60
students. The number of newspaper is-
(A) at least 30 (B) at most 20 (C) exactly 25 (D) none of these

C-2. In a town of 10,000 families it was found that 40% families buy newspaper A, 20% families buy newspaper B
and 10% families buy newspaper C, 5% families buy A and B, 3 % buy B and C and 4% buy A and C.
If 2% families buy all the three news papers, then number of families which buy newspaper A only is
(A) 3100 (B) 3300 (C) 2900 (D) 1400

C-3. In a city 20 percent of the population travels by car, 50 percent travels by bus and 10 percent travels by
both car and bus. Then persons travelling by car or bus is
(A) 80 percent (B) 40 percent (C) 60 percent (D) 70 percent

SECTION (D) : Cartesian Product of sets, Domain, range and co-domain of Relation.

D-1. If A = {2, 4, 5}, B = {7, 8, 9}, then n(A × B) is equal to


(A) 6 (B) 9 (C) 3 (D) 0

D-2. If A = {x : x2 – 5x + 6 = 0}, B = {2, 4}, C = {4, 5} then A × (B  C) is-


(A) {(2, 4), (3, 4)} (B) {(4, 2), (4, 3)} (C) {(2, 4), (3, 4), (4, 4)} (D) {(2, 2), (3, 3), (4, 4), (5, 5)}

D-3. A and B are two sets having 3 and 4 elements respectively and having 2 elements in common. The number
of relation which can be defined from A to B is
(A) 25 (B) 210 – 1 (C) 212 – 1 (D) none of these

SECTION (E) : Types of Relations


E-1. Let A = {1, 2, 3, 4} and R be a relation in A given
by R = {(1, 1), (2, 2), (3, 3), (4, 4), (1, 2), (2, 1), (3, 1), (1, 3)}, then relation R is
(A) Reflexive (B) Symmetric (C) Equivalence (D) Reflexive and Symmetric

E-2. The relation R = {(1, 1), (2, 2), (3, 3), (1, 2), (2, 3), (1, 3)} on set A = {1, 2, 3} is
(A) Reflexive but not symmetric (B) Reflexive but not transitive
(C) Symmetric and Transitive (D) Neither symmetric nor transitive

E-3. The relation ''less than'' in the set of natural number is


(A) Only symmetric (B) Only transitive (C) Only reflexive (D) Equivalence relation

E-4. The relation R defined in N as aRb b is divisible by a is


(A) Reflexive but not symmetric (B) Symmetric but not transitive
(C) Symmetric and transitive (D) None of these

E-5. In the set A = {1, 2, 3, 4, 5}, a relation R is defined by R = {(x, y)| x, y  A and x < y}. Then R is
(A) Reflexive (B) Symmetric (C) Transitive (D) None of these

E-6. For real numbers x and y, we write x R y  x – y + 2 is an irrational number. Then the relation R is-
(A) Reflexive (B) Symmetric (C) Transitive (D) None of these

E-7. Which one of the following relations on R is equivalence relation-


(A) x R1y  |x| = |y| (B) x R2y  x  y (C) x R3y  x | y (D) x R4y  x < y

RESONANCE Sets,Relations and Binary Operations - 12


CCXIIISEET

E-8. The relation R defined in A = {1, 2, 3} by a R b if |a2 – b2|  5. Which of the following is false-
(A) R = {(1, 2), (2, 2), (3, 3), (2, 1), (2, 3), (3, 2) (B) Co-domain of R = {1, 2, 3}
(C) Domain of R = {1, 2, 3} (D) Range of R = {1, 2, 3}

E-9. Let P = {(x, y) | x2 + y2 = 1, x, y  R}, then P is


(A) reflexive (B) symmetric (C) transitive (D) anti-symmetric

E-10. Let A = {p, q , r}. Which of the following is an equivalence relation on A ?


(A) R1 = {(p, q), (q, r), (p, r) (p, p)} (B) R2 = {(r, q), (r, p), (r, r) (q, q)}
(C) R3 = {(p, p), (q, q), (r, r) (p, q)} (D) none of these

E-11. Let R1 be a relation defined by R1 = {(a, b)| a  b ; a, b  R} . Then R1 is


(A) An equivalence relation on R (B) Reflexive, transitive but not symmetric
(C) Symmetric, Transitive but not reflexive (D) Neither transitive nor reflexive but symmetric

E-12. Let L denote the set of all straight lines in a plane. Let a relation R be defined by R ,, L.
The R is
(A) Reflexive (B) Symmetric (C) Transitive (D) None of these

E-13. Let S be the set of all real numbers. Then the relation R =
{(a, b) : 1 + ab > 0} on S is
(A) Reflexive and symmetric but not transitive (B) Reflexive, transitive but not symmetric
(C) Symmetric, transitive but not reflexive (D) Reflexive, transitive and symmetric

E-14. Let R be a relation on the set N be defined by {(x, y)| x, y  N, 2x + y = 41}. Then R is
(A) Reflexive (B) Symmetric (C) Transitive (D) None of these

SECTION (F) : Binary Operation


F-1. Let A = {1, 2, 3, 4, 5, 6} and * be an operation A defined by a * b = r, where r is the least non-negative
remainder when the product ab is divided by k. Operation * is binary operation if k =
(A) 7 (B) 11 (C) 21 (D) 17

F-2 Let * be a binary operation on Z .If a * b = (a2 + b3)2 where a, b  Z then the value of 5 * 3 is
(A) 1256 (B) 2704 (C) 64 (D) 34

F-3. Let * be a binary operation on Z .If a * b = 2ab where a, b  Z then the value of 19 * 16 is
(A) 608 (B) 70 (C) 68 (D) 304

F-4. Let * be a binary operation on Z defined by x * y = x2 + y2 + xy; x, y  Z. The value of [(1 * 2) + (0 * 3)]2 is
(A) 16 (B) 19 (C) 361 (D) 256

F-5. Let * be a binary operation on N defined by a * b = 25ab ; a, b  N. then the binary operation * is
(A) Only commutative (B) Only associative
(C) Both commutative and associative (D) Neither commutative nor associative

F-6. Let * be a binary operation on Z defined by a * b = a + b – 15 for a, b  Z. The identity element in (Z, ) is
(A) 5 (B) 10 (C) 15 (D) 0

F-7. Let * be a binary operation on Z defined by a * b = a + b – 15 for a, b  Z. The inverse of element 21 in (Z, ) is
(A) 12 (B) –21 (C) 9 (D) 1/21

COMPREHENSIONS
Comprehension # 1
Let R be a relation defined as R = { (x y) : y = |x – 1|, x  Z and | x |  3}
1. Relation R is equal to :
(A) {(1, 0), (1, 2), (3, 2), (4, 3)} (B) {(–3, 4), (–2, 3), (–1, 2), (0, 1), (1, 0), (2, 1), (3, 2)}
(C) {(4, –3), (3, –2), (2 –1), (1, 0), (2, 3)} (D) None of these

2. Domain of R is :
(A) {0, 1, 2, 3, 4} (B) {1, 3, 4}
(C) {– 3, – 2, – 1, 0, 1, 2, 3} (D) {0, 1, 2, 3, 4}

RESONANCE Sets,Relations and Binary Operations - 13


CCXIIISEET

3. Range of R is
(A) {0, 1, 2, 3, 4} (B) {–3, –2, –1, 0, 1, 2, 3}
(C) {–4, –3, –1, –2, 0} (D) {–1, 0, 1, 2, 3, 4}

SUBJECTIVE QUESTIONS

1. Write the set of all vowels in English alphabet which precede letter O.

2. Describe the following set by set property method {0, 3, 8, 15, 24, 35}

3. Which of the following are true ?


(i) If A = {1, 5, 5, 5}, B = {1, 3, 5}, then A  B.
(ii) If A = {x : x3 – 1 = 0, x  N}, B = {x : x2 – 4x + 3 = 0, x  N} then A  B.

4. Assume that P(A) = P(B). Prove that A = B.

5. If A = {x : x = 4n + 1, n  5, n  N} and B {3n : n  8, n  N}, then find A – (A – B).

6. Prove that A  B = A  B iff A = B.

7. Prove that : A – (B  C) = (A – B)  (A – C) without using venn diagram.

8. Prove by using venn diagram


(i) A – (B  C) = (A – B)  (A – C) (ii) A  B  B  A

9. Which of the following venn-diagrams best represents the sets of females, mothers and doctors ?

(A) (B) (C) (D)

10. Of the members of three athletic teams in a school 21 are in the cricket team, 26 are in the hockey team and
29 are in the football team. Among them, 14 play hockey and cricket, 15 play hockey and football, and 12
play football and cricket. Eight play all the three games. Find the total number of members in the three
athletic teams.

11. In a class of 55 students, the number of students studying different subjects are 23 in Mathematics, 24 in
Physics, 19 in Chemistry, 12 in Mathematics and Physics 9 in Mathematics and Chemistry, 7 in Physics
and Chemistry and 4 in all the three subjects. Find the number of students who have taken exactly one
subject.

12. Determine the domain and the range of the relation R defined by
R = {(x + 1, x + 5) : x  {0, 1, 2, 3, 4, 5}}.

13. If A = {3, 4, 6}, B = {1, 3} and C = {1, 2, 6} then find (A – B) × (A – C).

14. Let n be a fixed positive integer. Define a relation R on the set of integers Z, aRb n|(a – b). Then prove
that R is equivalence relation

15. Let R be a relation over the set N × N and it is defined by (a, b) R (c, d)  a + d = b + c. Then prove that R
is equivalence relation

RESONANCE Sets,Relations and Binary Operations - 14


CCXIIISEET

16. Let L be the set of all straight lines in the Euclidean plane. Two lines 1 and 2 are said to be related by the
relation R if 1 is parallel to 2. Then prove that R is equivalence relation.

17. For n, m  N, n | m means that n is a factor of m, then prove that relation | is reflexive, transitive but not
symmetric.

18. Let R = {(x, y) : x, y  A, x + y = 5} where A = {1, 2, 3, 4, 5} then prove that R is neither reflexive nor transitive
but symmetric.

19. Let * be a binary operation on the set R defined by a * b = a + b + ab, a, b  R. Solve the equation
2 * (3 * x) = 7

20. Let * be a binary operation on Z × Z defined by (a, b) * (c, d) = (a + c, b + d); (a, b), (c, d)  Z × Z.
Find (1, 2) * (3, 5) and (4, 3) * (1, 0).

21. Consider the binary operation * on Q defined by a * b = a + 12b + ab for a, b  Q. Show that * is not
commutative.

22. Let A = N × N and '*' be a binary operation on A defined by (a, b) * (c, d) = (ad + bc, bd). Show that (A, ) has
no identity element.

a b
23. If  is a binary operation on R defined by a  b = + for a, b  R, then show that :
4 7
(2  5)  7  2  (5  7).

24. Let A = {1, –1, i, –i}, where i =  1 . Draw the composition table corresponding to binary operation 'multiplication'
on A.

25. Does the composition table :

* a b c
a b c a
b c a b
c a b c
give a commutative binary operation on the set {a, b, c} ?

AIEEE PROBLEMS (PREVIOUS YEARS)


1. If A, B and C are three sets such that A  B = A C and A  B = A C, then [AIEEE - 2009]
(1) A = C (2) B = C (3) A  B =  (4) A = B

2. Let R = {(1, 3), (4, 2), (2, 4), (2, 3), (3, 1)} be a relation on the set A = {1, 2, 3, 4}. The relation R is-
[AIEEE - 2004]
(1) transitive (2) not symmetric (3) reflexive (4) a function

3. Let R = {(3, 3), (6, 6), (9, 9), (12, 12) (6, 12), (3, 9), (3, 12), (3, 6)} be relation on the set A = {3, 6, 9, 12}.
Then the relation R is [AIEEE - 2005]
(1) reflexive and transitive only (2) reflexive only
(3) an equilvalence relation (4) reflexive and symmetric only

RESONANCE Sets,Relations and Binary Operations - 15


CCXIIISEET

4. Let W denote the words in the english dictionary. Define the relation R by : R = {(x, y)  W × W | the
words x and y have at least one letter in common}. Then R is- [AIEEE - 2006]
(1) reflexive, symmetric and not transitive (2) reflexive, symmetric and transitive
(3) reflexive, not symmetric and transitive (4) not reflexive, symmetric and transitive

5. Let R be the real line. Consider the following subsets of the plane R × R [AIEEE - 2008]
S = {(x, y) : y = x + 1 and 0 < x < 2}
T = {(x, y) : x – y is an integer}
Which one of the following is true ?
(1) T is an equivalence relation on R but S is not (2) Neither S nor T is an equivalence relation on R
(3) Both S and T are equivalence relations on R (4) S is an equivalence relation on R but T is not

6. Consider the following relations : [AIEEE - 2010]


R : {(x, y)| x ,y are real numbers and x = wy for some rational number w}
m p
S = {  ,  | m, n, p and q are integers such that n, q  0 and qm = pn}, then
 n q
(1) neither R nor S is an equivalence relation
(2) S is an equivalence relation but R is not an equivalence relation
(3) R and S both are equivalence relations
(4) R is an equivalence relation but S is not an equivalence relation

7. Let R be the set of real numbers. [AIEEE - 2011, I]


Statement-1 : A = {(x, y)  R × R : y – x is an integer} is an equivalence relation on R.
Statement-2 : B = {(x, y)  R × R : x = y for some rational number } is an equivalence relation on R.
(1) Statement-1 is true, Statement-2 is true; Statement-2 is a correct explanation for Statement-1.
(2) Statement-1 is true, Statement-2 is true; Statement-2 is not a correct explanation for Statement-1.
(3) Statement-1 is true, Statement-2 is false.
(4) Statement-1 is false, Statement-2 is true.

8. Consider the following relation R on the set of real square matrices of order 3. [AIEEE - 2011, II]
R = {(A, B)|A = P–1 BP for some invertible matrix P}.
Statement -1 : R is equivalence relation.
Statement - 2 : For any two invertible 3 × 3 matrices M and N, (MN)–1 = N–1M–1.
(1) Statement-1 is true, statement-2 is a correct explanation for statement-1.
(2) Statement-1 is true, statement-2 is true; statement-2 is not a correct explanation for statement-1.
(3) Statement-1 is true, statement-2 is false.
(4) Statement-1 is false, statement-2 is true.

9. Let X = {1, 2, 3, 4, 5}. The number of different ordered pairs (Y, Z) that can formed such that Y X, Z  X and
Y  Z is empty, is : [AIEEE- 2012]
(1) 52 (2) 35 (3) 25 (4) 53

RESONANCE Sets,Relations and Binary Operations - 16


CCXIIISEET

EXERCISE # 1 EXERCISE # 2
SECTION (A) : 1. {a, e, i} 2. {x : x = n2 – 1, n  N, n  6}
A-1. B A-2. A A-3. D A-4. D 3. (i) False (ii) True 5. {9, 21} 9. D
10. 43 11. 22
SECTION (B) : 12. Domain = {1, 2, 3, 4, 5, 6}, Range = {5, 6, 7, 8, 9, 10}
B-1. B B-2. C B-3. B B-4. B
B-5. D B-6. C B-7. B 13. {(4, 3), (4, 4), (6, 3), (6, 4)} 20. (4, 7) , (5, 3)

SECTION (C) : 25. Yes


C-1. C C-2. B C-3. C
EXERCISE # 3
SECTION (D) : 1. (2) 2. (2) 3. (1) 4. (1) 5. (1) 6. (2) 7.(3)
D-1. B D-2. A D-3. D
8. (2) 9. (2)
SECTION (E) :
E-1. D E-2. A E-3. B E-4. A
E-5. C E-6. A E-7. A E-8. A
E-9. B E-10. D E-11. B E-12. B
E-13. A E-14. D

SECTION (F) :
F-1. A F-2 B F-3. A F-4. D
F-5. A F-6. C F-7. C

COMPREHENSIONS
1. B 2. C 3. A

RESONANCE Sets,Relations and Binary Operations - 17


CCXIIISEET

SUBJECTIVE QUESTIONS

1. If A = {(x, y) : x2 + y2 = 25} and B = {(x, y) : x2 + 9y2 = 144} then find n(A  B).

2. In a battle, 70% of the combatants lost one eye, 80% an ear, 75% an arm, 85% a leg, x% lost all the four
limbs. Find the minimum value of x.

3. A survey shows that 63% of the Americans like cheese whereas 76% like apples. If x% of the Americans like
both cheese and apples, find the value of x.

4. Prove that (A B  C) (A  B C)  C = B C

5. Let B be a subset of A and let P(A : B) = {S : B  S  A}. If A = {a, b, c, d} and B = {a, b}, then list all the
elements of P(A : B).

6. Prove that the relation R = {(x, y) : x2 = xy} is reflexive and transitive but not symmetric

7. Consider a relation R defined on set of involutory square matrix of order 2. If A, B are two such matrices then
relation R is defined as (A, B)  R  (AB)T = ATBT. Prove that relation R is reflexive and symmetric.

8. Let A = {a, b}. Find the number of binary operations on A.

9. Let A be the set of all subsets of a non-empty set S. Show that the binary operation 'union' on A is left
distributive over the binary operation 'intersection' on A. Also show that 'intersection' is left distributive over
'union'.

1. 4 2. 10 3. x  [39, 63] 5. {a, b}, {a, b, c}, {a, b, d}, {a, b, c, d}

RESONANCE Sets,Relations and Binary Operations - 18


Functions

A. Definition :
Function is a special case of relation, from a non empty set A to
a non empty set B, that associates each member of A to a unique
member of B. Symbolically, we write f: A  B. We read it as "f is
a function from A to B".
Set 'A' is called domain of f and set 'B' is called co-domain of
f.

Thecalculusisthegreatest For example, let A  {–1, 0, 1} and B  {0, 1, 2}. Then


A × B  {(–1, 0), (–1, 1), (–1, 2), (0, 0), (0, 1), (0, 2), (1, 0), (1, 1),
aid we have to the (1, 2)}
appreciationofphysicaltruth Now, " f : A  B defined by f(x) = x 2 " is the function such that
f  {(–1, 1), (0, 0), (1, 1)}
in the broadest sense of the f can also be show diagramatically by following picture.
word.

.......Wil l i ams F .Osgood

Every function say f : A  B satisfies the following conditions:


That flower of modern (a) f  A x B,
(b)  a  A  (a, f(a))  f and
mathematical thought, the (c) (a, b)  f & (a, c)  f  b = c

notion of a function. Illustration # 1

.....Thomas McCormack (i) Which of the following correspondences can be called a function
?
(A) f(x) = x 3 ; {–1, 0, 1}  {0, 1, 2, 3}
(B) f(x) = ± x ; {0, 1, 4}  {–2, –1, 0, 1, 2}
(C) f(x) = x ; {0, 1, 4}  {–2, –1, 0, 1, 2}

(D) f(x) = – x ; {0, 1, 4}  {–2, –1, 0, 1, 2}

Solution
f(x) in (C) & (D) are functions as definition of function is satisfied.
while in case of (A) the given relation is not a function, as
f(–1)  codomain. Hence definition of function is not satisfied.
While in case of (B), the given relation is not a function, as
f(1) = ± 1 and f(4) = ± 2 i.e. element 1 as well as 4 in domain is
related with two elements of codomain. Hence definition of function
is not satisfied.

TM
RESONANCE 1
(ii) Which of the following pictorial diagrams represent the function

(A) (B)

(C) (D)

Solution
B & D. In (A) one element of domain has no image, while in (C) one element of domain has two images
in codomain

Assignment:

1. Let g(x) be a function defined on [1, 1]. If the area of the equilateral triangle with two of its vertices at
(0,0) & (x,g(x)) is 3 / 4 sq. units, then the function g(x) may be.
2
(A) g(x)=  (1  x ) (B*) g(x) = (1  x 2 ) (C*) g(x) =  (1  x 2 ) (D) g(x) = (1  x 2 )

2. Represent all possible functions defined from {} to {1, 2}


Answer (1) B

(2) (i) (ii) (iii) (iv)

B. Domain, Co-domain & Range of a Function :

Let f: A  B, then the set A is known as the domain of f & the set B is known as codomain of f. If a
member 'a' of A is associated to the member 'b' of B, then 'b' is called the f-image of 'a' and we write
b = f (a). Further 'a' is called a pre-image of 'b'. The set {f(a):  a A} is called the range of f and is
denoted by f(A). Clearly f(A)  B.

Sometimes if only definition of f (x) is given (domain and codomain are not mentioned), then domain is
set of those values of ' x' for which f (x) is defined, while codomain is considered to be (– , )

A function whose domain and range both are sets of real numbers is called a real function. Conven-
tionally the word "FUNCTION” is used only as the meaning of real function.

Illustration # 2
Find the domain of following functions :

(i) f(x) = x2  5 (ii) sin–1 (2x – 1)


Solution

(i) f(x) = 2
x 2  5 is real iff x – 5  0  |x|  5  x  – 5 or x  5

 the domain of f is (–, – 5 ]  [ 5 , )

(ii) –1  2x – 1  + 1
 domain is x  [0, 1]

TM
RESONANCE 2
Algebraic Operations on Functions :

If f & g are real valued functions of x with domain set A and B respectively, then both f & g are defined
in A  B. Now we define f + g, f  g, (f . g) & (f /g) as follows:

f f( x )
(iii)   (x) =
g
  g( x ) domain is {x  x  A  B such that g(x)  0}.

Note :  For domain of (x) = {f(x)}g(x) , conventionally, the conditions are f(x) > 0 and g(x) must be
defined.
 For domain of (x) = f(x) Cg(x) or (x) = f(x) Pg(x) conditions of domain are f(x)  g(x) and f(x)  N
and g(x)  W

Illustration # 3
Find the domain of following functions :
3 1
(i) f(x) = sin x  16  x 2 (ii) f(x) = 2
log(x 3  x) (iii) f(x) = x cos x

4x

Solution
(i) sin x is real iff sin x  0  x[2n, 2n + ], nI.
2
16  x 2 is real iff 16  x  0   4  x  4.
Thus the domain of the given function is {x : x[2n, 2n + ], nI }[4, 4] =
[4, ]  [0, ].

(ii) Domain of 4  x 2 is [2, 2] but 4  x 2 = 0 for x = ± 2  x  (–2, 2)


log(x 3  x) is defined for x 3  x > 0 i.e. x(x  1)(x + 1) > 0.
 domain of log(x 3  x) is (1, 0 )  (1, ).
Hence the domain of the given function is {(1, 0 )  (1, )} (2, 2) =
(1, 0 )  (1, 2).

(iii) x > 0 and –1  x  1


 domain is (0, 1]

Assignment :

3. Find the domain of following functions.


1
(i) f(x) = log( 2  x ) + x  1

–1 2 x  1
(ii) f(x) = 1 x – sin
3
Ans. (i) [–1, 1)  (1, 2) (ii) [–1, 1]

Methods of determining range :

(i) Representing x in terms of y


Definition of the function is usually represented as y (i.e. f(x) which is dependent variable) in terms of
an expression of x (which is independent variable). To find range rewrite given definition so as to
represent x in terms of an expression of y and thus obtain range (possible values of y).
If y = f(x)  x = g(y), then domain of g(y) represents possible values of y, i.e. range of f(x).

TM
RESONANCE 3
Illustration # 4
x2  x  1
Find the range of f(x) =
x2  x  1
Solution
x2  x  1
f(x) = {x 2 + x + 1 and x 2 + x – 1 have no common factor}
x2  x  1
2
x  x 1
y=  yx 2 + yx – y = x 2 + x + 1
x2  x  1
 (y – 1) x 2 + (y – 1) x – y – 1 = 0
If y = 1, then the above equation reduces to –2 = 0. Which is not true.
Further if y  1, then (y – 1) x 2 + (y – 1) x – y – 1 = 0 is a quadratic and has real roots if
(y – 1)2 – 4 (y – 1) (–y – 1)  0 i.e. if y  –3/5 or y  1 but y  1
Thus the range is (–, –3/5]  (1, )

(ii) Graphical Method :


Values covered on y-axis by the graph of function is range

Illustration # 5

x2  4
Find the range of f(x) =
x2
Solution

x2  4
f(x) = = x + 2; x  2
x2

 graph of f(x) would be

Thus the range of f(x) is R – {4}

(iii) Using Monotonocity/Maxima-Minima


(a) Continuous function
If y = f(x) is continuous in its domain then range of f(x) is y  [min f(x), max. f(x)]
(b) Sectionally continuous function
In case of sectionally continuous functions, range will be union of [min f(x), max. f(x)] over all
those intervals where f(x) is continuous, as shown by following example.
Let graph of function y = f(x) is

Then range of above sectionally continuous function is [y2, y3]  (y4, y5]  (y6, y7]

Note : In case of monotonic functions minimum and maximum values lie at end points of interval.

TM
RESONANCE 4
Illustration # 6

Find the range of following functions :

(i) y = n (2x – x 2) (ii) y = sec –1 (x 2 + 3x + 1)

Solution :
(i) Step – 1
Using maxima-minima, we have
2x – x 2  (–, 1]
Step – 2
For log to be defined accepted values are 2x – x 2  (0, 1] {i.e. domain (0, 1]}
Now, using monotonocity
n (2x – x 2)  (–, 0]
 range is (– , 0] Ans.
(ii) y = sec –1 (x 2 + 3x + 1)
Let t = x 2 + 3x + 1 for x  R

 5 
then t   ,  
 4 

 5 
but y = sec –1 (t)  t   ,  1  [1, )
 4 

   1 5  
from graph range is y  0,   sec   4 , 
 2     
Assignment:

4. Find domain and range of following functions.

x 2  2x  5
(i) y = x3 (ii) y=
x 2  2x  5
3  5 3  5 
Answer (i) domain R; range R (ii) domain R ; range  2 , 2 
 
1
(iii) y=
x2  x
Answer domain R – [0, 1] ; range (0, )

(iv) y = cot –1 (2x – x 2)

 
Answer domain R ; range  ,  
4 

TM
RESONANCE 5
 2 3
(v) y = n sin–1  x  x  
 4

 2  5  2  5    
Answer domain x   ,  ; range n 6 , n 2 
 4 4   

C. Classification of Functions :
Functions can be classified as :
(i) One  One Function (Injective Mapping) and Many  One Function:
One  One Function :
A function f : A  B is said to be a one-one function or injective mapping if different elements of
A have different f images in B.
Thus for x 1, x2  A & f(x 1), f(x 2)  B, f(x 1) = f(x 2)  x1 = x 2 or x1  x2  f(x 1)  f(x 2).
Diagrammatically an injective mapping can be shown as

OR

Many  One function :

A function f : A  B is said to be a many one function if two or more elements of A have the
same f image in B.
Thus f : A  B is many one iff there exists atleast two elements x 1, x 2  A, such that
f(x 1) = f(x 2) but x 1  x 2.

Diagrammatically a many one mapping can be shown as

OR

Note :  If a function is oneone, it cannot be manyone and vice versa.

Methods of determining whether function is ONE-ONE or MANY-ONE :


(a) If x 1, x 2  A & f(x 1), f(x 2)  B, f(x 1) = f(x 2)  x 1 = x 2 or x 1  x 2  f(x 1)  f(x 2), then function is
ONE-ONE otherwise MANY-ONE.
(b) If there exists a straight line parallel to x-axis, which cuts the graph of the function atleast at
two points, then the function is MANY-ONE, otherwise ONE-ONE.
(c) If either f(x)  0,  x  complete domain or f(x)  0  x  complete domain, where equality can
hold at discrete point(s) only, then function is ONE-ONE, otherwise MANY-ONE.

(ii) Onto function (Surjective mapping) and Into function :


Onto function :
If the function f : A  B is such that each element in B (codomain) must have atleast one pre
image in A, then we say that f is a function of A 'onto' B. Thus f : A  B is surjective iff  b  B,
there exists some a  A such that f (a) = b.
Diagrammatically surjective mapping can be shown as

OR

TM
RESONANCE 6
Method of determining whether function is ONTO or INTO :
Find the range of given function. If range  codomain, then f(x) is onto, otherwise into
Into function :

If f : A  B is such that there exists atleast one element in codomain which is not the image
of any element in domain, then f(x) is into.

Diagrammatically into function can be shown as

OR

Note :  If a function is onto, it cannot be into and vice versa.

Thus a function can be one of these four types:

(a) oneone onto (injective & surjective)

(b) oneone into (injective but not surjective)

(c) manyone onto (surjective but not injective)

(d) manyone into (neither surjective nor injective)

Note :  If f is both injective & surjective, then it is called a bijective mapping. The bijective functions
are also named as invertible, non singular or biuniform functions.
 If a set A contains 'n' distinct elements then the number of different functions defined from
A  A is nn and out of which n! are one one.
Illustration # 7
(i) Find whether f(x) = x + cos x is one-one.
Solution
The domain of f(x) is R. f (x) = 1  sin x.
 f (x)  0  x  complete domain and equality holds at discrete points only
 f(x) is strictly increasing on R. Hence f(x) is one-one.
3 2
(ii) Identify whether the function f(x) = –x + 3x – 2x + 4 ; R  R is ONTO or INTO

Solution
As codomain  range, therefore given function is ONTO

(iii) f(x) = x 2 – 2x + 3; [0, 3]  A. Find whether f(x) is injective or not. Also find the set A, if f(x) is
surjective.
Solution
f(x) = 2(x – 1); 0  x  3

 ve ; 0  x  1
 f(x) = 
 ve ; 1  x  3

 f(x) is not monotonic. Hence it is not injective.


For f(x) to be surjective, A should be equal to its range. By graph range is [2, 6]
 A  [2, 6]
TM
RESONANCE 7
Assignment:
5. For each of the following functions find whether it is one-one or many-one and also into or onto
(i) f(x) = 2 tan x; (/2, 3/2)  R

Answer one-one onto

1
(ii) f(x) = ; (–, 0)  R
1 x2

Answer one-one into

(iii) f(x) = x 2 + n x

Answer one-one onto

D. Various Types of Functions :


(i) Polynomial Function :
If a function f is defined by f (x) = a0 x n + a1 x n1 + a2 x n2 +... + an1 x + an where n is a non
negative integer and a0, a1, a2,........., an are real numbers and a0  0, then f is called a
polynomial function of degree n.

Note :  There are two polynomial functions, satisfying the relation; f(x).f(1/x) = f(x) + f(1/x), which are
f(x) = 1  x n
(ii) Algebraic Function :
y is an algebraic function of x, if it is a function that satisfies an algebraic equation of the form,
P0 (x) yn + P1 (x) yn1 +....... + Pn1 (x) y + Pn (x) = 0 where n is a positive integer and P0 (x), P1
(x)....... are polynomials in x. e.g. y = x is an algebraic function, since it satisfies the
equation y²  x² = 0.

Note :  All polynomial functions are algebraic but not the converse.
 A function that is not algebraic is called Transcendental Function.

(iii) Fractional / Rational Function :


g( x )
A rational function is a function of the form, y = f (x) = , where g (x) & h (x) are polynomials
h( x )
and h (x)  0.

(iv) Exponential Function :


A function f(x) = ax = ex In a (a > 0, a  1, x  R) is called an exponential function. Graph of
exponential function can be as follows :

Case -  Case - 
For a > 1 For 0 < a < 1

TM
RESONANCE 8
(v) Logarithmic Function : f(x) = logax is called logarithmic function where a > 0 and a  1 and
x > 0. Its graph can be as follows

Case-  Case- 
For a > 1 For 0 < a < 1

(vi) Absolute Value Function / Modulus Function :


 x if x0
The symbol of modulus function is f (x) = x and is defined as: y = x  
 x if x0

(vi) Signum Function :


A function f (x) = sgn (x) is defined as follows :

 1 for x  0

f (x) = sgn (x) =  0 for x  0
 1 for x  0

| x |
 ; x0
It is also written as sgn x =  x
 0 ; x  0

| f ( x ) |
 ; f ( x)  0
Note : sgn f(x) =  f ( x )
 0 ; f (x)  0

(vii) Greatest Integer Function or Step Up Function :


The function y = f (x) = [x] is called the greatest integer function where [x] equals to the
greatest integer less than or equal to x. For example :
for 1  x < 0 ; [x] =  1 ; for 0  x < 1 ; [x] = 0
for 1  x < 2 ; [x] = 1 ; for 2  x < 3 ; [x] = 2 and so on.

TM
RESONANCE 9
Alternate Definition :
The greatest integer occur on real number line while moving L.H.S. of x (starting from x) is [x]

Properties of greatest integer function :


(a) x  1 < [x]  x
(b) [x ± m] = [x] ± m iff m is an integer.
(c) [x] + [y]  [x + y]  [x] + [y] + 1

 0 ; if x is an int eger
(d) [x] + [ x] = 
  1 otherwise
(viii) Fractional Part Function:
It is defined as, y = {x} = x  [x].
e.g. the fractional part of the number 2.1 is 2.1  2 = 0.1 and the fractional part of  3.7 is 0.3.
The period of this function is 1 and graph of this function is as shown.

(ix) Identity function :


The function f : A  A defined by, f(x) = x  x  A is called the identity function on
A and is denoted by  A. It is easy to observe that identity function is a bijection.
(x) Constant function :
A function f : A  B is said to be a constant function, if every element of A has the same f image
in B. Thus f : A  B; f(x) = c,  x  A, c  B is a constant function.

Illustration # 8
(i) Let {x} & [x] denote the fractional and integral part of a real number x respectively. Solve
4{x} = x + [x]
Solution
As x = [x] + {x}
 Given equation  4{x} = [x] + {x} + [x]
2 [x]
 {x} =
3
As [x] is always an integer and {x}  [0, 1), possible values are
[x] {x} x = [x] + {x}
0 0 0
2 5
1
3 3
5
 There are two solution of given equation x = 0 and x =
3
TM
RESONANCE 10
(ii) Draw graph of f(x) = sgn (n x)
Solution

Assignment:

6. If f : R  R satisfying the conditions f(0) = 1, f(1) = 2 and f(x + 2) = 2f (x) + f(x + 1), then find f (6).
Answer 64

7. Draw the graph of following functions where [.] denotes greatest integer function
(i) y= [2 x] + 1 (ii) y = x [x] , 1  x  3 (iii) y = sgn (x 2 – x)

Answer (i) (ii) (iii)

E. Odd & Even Functions :


(i) If f (x) = f (x) for all x in the domain of ‘f’ then f is said to be an even function.
If f (x)  f (x) = 0  f (x) is even.
e.g. f (x) = cos x; g (x) = x² + 3.
(ii) If f (x) = f (x) for all x in the domain of ‘f’ then f is said to be an odd function.
If f (x) + f (x) = 0  f (x) is odd.
e.g. f (x) = sin x; g (x) = x 3 + x.
Note :  A function may neither be odd nor even. (e.g. f(x) = ex , cos –1x)
 If an odd function is defined at x = 0, then f(0) = 0
Properties of Even/Odd Function
(a) Every even function is symmetric about the yaxis & every odd function is symmetric about the
origin.
For example graph of y = x 2 is symmetric about y-axis, while graph of y = x 3 is symmetric
about origin

(b) All functions (whose domain is symmetrical about origin) can be expressed as the sum of an
even & an odd function, as follows

f ( x )  f ( x ) f ( x )  f (  x)
f(x) = 
2 2
Even Odd
TM
RESONANCE 11
(c) The only function which is defined on the entire number line and is even & odd at the same time
is f(x) = 0.

(d) If f and g both are even or both are odd then the function f.g will be even but if any one of them
is odd then f.g will be odd.

(e) If f(x) is even then f(x) is odd but converse need not be true.

Illustration # 9
 2 
Show that log  x  x  1  is an odd function.
 
Solution
 2 
Let f(x) = log  x  x  1  .
 
 2 
Then f(–x) = log   x  (  x )  1 
 

 x 2  1  x  x 2  1  x 
   1
    2 
= log = log 2 – log  x  x  1  = –f(x)
2 x  1 x  
x 1 x
Hence f(x) is an odd function.

Illustration # 10
Show that ax +a–x is an even function.

Solution
Let f(x) = ax + a–x
Then f(–x) = a–x + a–(–x) = a–x +ax = f(x).
Hence f(x) is an even function

Illustration # 11

Show that cos –1 x is neither odd nor even.


Solution
Let f(x) = cos –1x. Then f(–x) = cos –1 (–x) =  – cos –1 x which is neither equal to f(x) nor equal to f(–x).
Hence cos –1 x is neither odd nor even

Assignment:
8. Determine whether following functions are even or odd?

e x  ex
(i) Answer Odd
e x  ex

 2 
(ii) log  x  1  x  Answer Odd
 

 2 
(iii) x log  x  x  1  Answer Even
 

(iv) sin–1 2x 1 x 2 Answer Odd

Even extension / Odd extension :


Let the defincition of the function f(x) is given only for x  0. Even extension of this function implies to
define the function for x < 0 assuming it to be even. In order to get even extension replace x by –x in the
given defincition
Similarly, odd extension implies to define the function for x < 0 assuming it to be odd. In order to get
odd extension, multiply the definition of even extension by –1
TM
RESONANCE 12
Illustration # 12
What is even and odd extension of f(x) = x 3 – 6x 2 + 5x – 11 ; x  0
Solution
Even extension
f(x) = –x 3 – 6x 2 + 5x – 11 ;x<0
Odd extension
f(x) = x 3 + 6x 2 + 5x + 11 ;x<0

F. Periodic Function :
A function f(x) is called periodic with a period T if there exists a real number T > 0 such that for each x
in the domain of f the numbers x – T and x + T are also in the domain of f and f(x) = f(x + T) for all x in
the domain of 'f'. Domain of a periodic function is always unbounded. Graph of a periodic function with
period T is repeated after every interval of 'T'.

e.g. The function sin x & cos x both are periodic over 2 & tan x is periodic over 
The least positive period is called the principal or fundamental period of f or simply the period of
f.

Note :  f (T) = f (0) = f (T), where ‘T’ is the period.


 Inverse of a periodic function does not exist.
 Every constant function is always periodic, with no fundamental period.

Properties of Periodic Function


1
(a) If f(x) has a period T, then and f (x ) also have a period T.
f (x)

T
(b) If f(x) has a period T then f (ax + b) has a period | a | .

f (x)
(c) If f (x) has a period T1 & g (x) also has a period T 2 then period of f(x) ± g(x) or f(x) . g(x) or
g( x)
is L.C.M. of T1 & T 2 provided their L.C.M. exists. However that L.C.M. (if exists) need not to be
f (x)
fundamental period. If L.C.M. does not exists f(x) ± g(x) or f(x) . g(x) or is aperiodic.
g( x)
e.g. |sinx| has the period , | cosx | also has the period 

 |sinx| + |cosx| also has a period . But the fundamental period of |sinx| + |cosx| is .
2
Illustration # 13
Find period of following functions
x x
(i) f(x) = sin + cos (ii) f(x) = {x} + sin x
2 3

3x x 2x
(iii) f(x) = cos x . cos 3x (iv) f(x) = sin – cos – tan
2 3 3
Solution
(i)
x x x x
Period of sin is 4 while period of cos is 6 . Hence period of sin + cos is 12 
2 3 2 3
{L.C.M. of 4 & 6 is 12}

TM
RESONANCE 13
(ii)
Period of sin x = 2
Period of {x} = 1
but L.C.M. of 2 & 1 is not possible
 it is aperiodic

(iii)
f(x) = cos x . cos 3x

 2 
period of f(x) is L.C.M. of  2 ,  = 2
 3 

2
but 2 may or may not be fundamental periodic, but fundamental period = , where n  N. Hence
n
cross-checking for n = 1, 2, 3, ....we find  to be fundamental period f( + x) = (– cos x) (– cos 3x) = f(x)
(iv)
2 2 
Period of f(x) is L.C.M. of , ,
3 / 2 1/ 3 3 / 2

4 2
= L.C.M. of , 6 ,
3 3
= 12

 a p   L.C.M.(a, p,  )
NOTE : L.C.M. of  , ,  
 b q m  H.C.F.(b, q, m)

Assignment:

9. Find the period of following function.


(i) f(x) = sin x + | sin x |
Answer 2
x
(ii) f(x) = 3 cos x – sin 3

Answer 6
2x 3x
(iii) sin – cos
5 7
Answer 70 
(iv) f(x) = sin2x + cos 4x
Answer 

G. Composite Function :
Let f: XY1 and g: Y2 Z be two functions and the set D = {x X: f(x) Y2}. If D  , then the function h
defined on D by h(x) = g{f(x)} is called composite function of g and f and is denoted by gof. It is also called
function of a function.

Note :  Domain of gof is D which is a subset of X (the domain of f ). Range of gof is a subset of the range of
g. If D = X, then f(x) Y2.
Properties of Composite Functions :
(a) In general gof  fog (i.e. not commutative)
(b) The composite of functions are associative i.e. if three functions f, g, h are such that
fo (goh) & (fog) oh are defined, then fo (goh) = (fog) oh.
(c) If f and g both are one-one, then gof and fog would also be one-one.

TM
RESONANCE 14
(d) If f and g both are onto, then gof or fog may or may not be onto.
(e) The composite of two bijections is a bijection iff f & g are two bijections such that gof is
defined, then gof is also a bijection only when co-domain of f is equal to the domain of g.
(f) If g is a function such that gof is defined on the domain of f and f is periodic with T, then gof is
also periodic with T as one of its periods. Further if
# g is one-one, then T is the period of gof
# g is also periodic with T as the period and the range of f is a sub-set of [0, T ], then
T is the period of gof
Illustration # 14
Describe fog and gof wherever is possible for the following functions
(i) f(x) = x  3 , g(x) = 1 + x 2 (ii) f(x) = 2
x , g(x) = x  1.
Solution
(i) Domain of f is [3, ), range of f is [0, ).
Domain of g is R, range of g is [1, ).
Since range of f is a subset of domain of g,
 domain of gof is [3, ) {equal to the domain of f }
gof (x) = g{f(x)} = g ( x  3 ) = 1 + (x+3) = x + 4. Range of gof is [1, ).
Further since range of g is a subset of domain of f,
 domain of fog is R {equal to the domain of g}
fog (x) = f{g(x)}= f(1+ x 2 ) = x 2  4 Range of fog is [2, ).

2
(ii) f(x) = x , g(x) = x  1.
Domain of f is [0, ), range of f is [0, ).
Domain of g is R, range of g is [1, ).
Since range of f is a subset of the domain of g,
 domain of gof is [0, ) and g{f(x)}= g(x) = x  1. Range of gof is [1, )
Further since range of g is not a subset of the domain of f
i.e. [1, )  [0, )
 fog is not defined on whole of the domain of g.
Domain of fog is {xR, the domain of g : g(x) [0, ), the domain of f}.
Thus the domain of fog is D = {xR: 0  g(x) < }
i.e. D = { xR: 0  x 2  1}= { xR: x  1 or x  1 }= (, 1]  [1, )
fog (x) = f{g(x)} = f(x 21) = x 2  1 Its range is [0, ).
  
(iii) Let f(x) = ex ; R+  R and g(x) = sin–1 x; [–1, 1]    ,  . Find domain and range of fog (x)
 2 2
Solution

  
Domain of f(x) : (0, ) Range of g(x) :   , 
 2 2

 
values in range of g(x) which are accepted by f(x) are  0, 
 2

 
 0 < g(x)  0 < sin–1x  0<x1
2 2
Hence domain of fog(x) is x  (0, 1]

Therefore Domain : (0, 1]


Range : (1, e/2]
Example of composite function of non-uniformly defined functions :

TM
RESONANCE 15
Illustration # 15
If f(x) = | |x – 3| – 2 | 0x4
g(x) = 4 – |2 – x| –1x3
then find fog(x) and draw rough sketch of fog(x).
Solution
f(x) = | | x – 3| – 2| 0  x  4

| x  1 | 0  x  3
= 
| x  5 | 3  x  4
1  x 0  x  1

= x 1 1 x  3
5  x 3  x  4

g(x) = 4 – |2 – x| 1  x  3

4  ( 2  x )  1  x  2
= 
4  ( x  2) 2  x  3

2  x  1  x  2
= 
6  x 2  x  3

1  g( x ) 0  g( x )  1

g( x )  1 1  g( x )  3
 fog (x) = 
5  g( x ) 3  g( x )  4

1  ( 2  x ) 0  2  x  1 and  1  x  2

 2  x 1 1  2  x  3 and  1  x  2

= 5  (2  x ) 3  2  x  4 and  1  x  2

 1 6  x 0  6  x  1 and 2  x  3
 6  x 1 1  6  x  3 and 2x3

 5  6  x 3  6  x  4 and 2x3

 1  x 2  x 1 and  1  x  2

 1 x 1 x  1 and  1  x  x
 3  x 1 x  2 and  1  x  2
= 
 x  5  6   x  5 and 2  x  3
 5  x  5   x  3 and 2x3

 x  1  3   x  2 and 2x3

 1  x  2  x  1 and  1  x  2

 1 x 1 x  1 and  1  x  2
 3  x 1 x  2 and  1  x  2
= 
 x5 5x6 and 2  x  3
5x 3x5 and 2x3

 x  1 2x3 and 2x3
1  x 1 x  1

= 3  x 1 x  2
x 1 2x3

TM
RESONANCE 16
Alternate method for finding fog

2  x  1  x  2
g(x) = 
6  x 2  x  3
graph of g(x) is

1  g( x ) 0  g( x )  1

g( x )  1 1  g( x )  3
 fog(x) = 
5  g( x ) 3  g( x )  4

1  g( x ) for no value

g( x )  1  1  x  1
= 
5  g( x ) 1 x  3

 2  x 1 1 x  1

5  (2  x ) 1  x  2
= 
5  (6  x ) 2  x  3

x 1 1 x  1

3  x 1 x  2
= 
x 1 2  x  3

Assignment:

10. Define fog(x) and gof(x). Also their Domain & Range.
(i) f(x) = [x], g(x) = sin x (ii) f(x) = tan x, x  (–/2, /2); g(x) = 1 x 2
Answer (i) gof = sin [x]
domain : R range { sin a : a  }
fog = [ sin x] domain : R range : {–1, 0, 1}

Answer (ii) gof = 1  tan 2 x

  
domain :   ,  range : [0, 1]
 4 4

fog = tan 1 x 2 domain : [–1, 1] range [0, tan 1]

11. Let f(x) = ex : R+  R and g(x) = x2 – x : R  R. Find domain and range of fog (x) & gof (x)
Answer fog (x) gof f(x)
Domain : (–, 0)  (1, ) Domain : (0, )

 1 
Range : [1, ) Range :  4 ,  
 

TM
RESONANCE 17
H. Inverse of a Function :
Let f : A  B be a function. Then f is invertible iff there is a function g : B  A such that go f is an
identity function on A and fog is an identity function on B. Then g is called inverse of f and is denoted
by f 1.
For a function to be invertible it must be bijective
Note :  The inverse of a bijection is unique.
 Inverse of an even function is not defined.
Properties of Inverse Function :
(a) The graphs of f & g are the mirror images of each other in the line y = x. For example f(x) = ax
and g(x) = loga x are inverse of each other, and their graphs are mirror images of each other on
the line y = x as shown below.

(b) Normally points of intersection of f and f–1 lie on the straight line y = x. However it must be noted
–1
that f(x) and f (x) may intersect otherwise also.
(c) In general fog(x) and gof(x) are not equal but if they are equal then in majority of cases either f
and g are inverse of each other or atleast one of f and g is an identity function.
(d) If f & g are two bijections f : A  B, g : B  C then the inverse of gof exists and
(gof)1 = f 1 o g1.
1
(e) If f(x) and g are inverse function of each other then f(g(x)) = g( x )

Illustration # 16
2x  3
(i) Determine whether f(x) = ; R  R, is invertible or not? If so find it.
4
Solution
As given function is one-one and onto, therefore it is invertible.
2x  3
y=
4
4y  3 –1 4x  3
 x=  f (x) =
2 2
–1  2x 1  x 2 
(ii) Is the function f(x) = sin   invertible?
 

Solution
Domain of f is [–1, 1] and f is continuous

 2 1 1

2 1  2x 2  
 1 x 2
if
2
x
2
f( x ) = 1  2x 2 1  x 2 =   2 1 1
 if x  or x 
 1  x 2 2 2
 1 1 
 f(x) is increasing in  ,  and is decreasing in each of the intervals
 2 2
 1   1 
  1,  and  , 1
 2  2 
 f(x) is not one-one, so is not invertible.
TM
RESONANCE 18
(iii) Let f(x) = x 2 + 2x; x  –1. Draw graph of f –1(x) also find the number of solutions of the equation,
f(x) = f –1(x)

Solution

f(x) = f–1(x) is equilavent to solving y = f(x) and y = x


 x 2 + 2x = x  x(x + 1) = 0  x = 0, –1
Hence two solution for f(x) = f –1(x)

(iv) If y = f(x) = x 2 – 3x + 1, x  2. Find the value of g(1) where g is inverse of f

Solution
y=1  x 2 – 3x + 1 = 1  x (x – 3) = 0  x = 0, 3
But x2  x=3
Now g(f(x)) = x
Differentiating both sides w.r.t. x
1
 g(f(x)). f(x) = 1  g(f(x)) =
f ( x )

1 1
 g(f(3)) =  g (1) = = (As f(x) = 2x – 3)
f (3) 63

1
=
3
Alternate Method
y = x 2 – 3x + 1
x 2 – 3x +1 – y = 0

3  9  4(1  y )
x=
2
3  5  4y
=
2
x2

3  5  4y
x=
2

3  5  4y
g(x) =
2

1
g(x) = 0 + x
x 5  4x

1 1 1
g(1) = = =
54 9 3

TM
RESONANCE 19
Assignment:
–1
12. Determine f (x), if given function is invertible
2
(i) f : (–, –1)  (–, –2) defined f(x) = –(x + 1) – 2

  7   
(ii) f:  ,   [–1, 1] defined by f(x) = sin  x  3 
6 6   

2 –1
Answer (i) – 1 + x2 (ii) – sin x
3
I. Equal or Identical Function :
Two functions f & g are said to be identical (or equal) iff :
(i) The domain of f  the domain of g.
(ii) The range of f  the range of g and

1 x
(iii) f(x) = g(x), for every x belonging to their common domain. e.g. f(x) = & g(x) = 2 are identical
x x

x2
functions. But f(x) = x and g(x) = are not identical functions.
x

Illustration # 17
Examine whether following pair of functions are identical or not

x2
(i) f(x) = & g(x) = x
x
Answer No, as domain of f(x) is R – {0}
while domain of g(x) is R

(ii) f(x) = sin2x + cos 2x & g(x) = sec 2x – tan2x

  
Answer No, as domain are not same. Domain of f(x) is R while that of g(x) is R – 2n  1 ; n  I
 2 

Assignment:

13. Examine whether following pair of functions are identical or not

 x
 x0
(i) f(x) = sgn (x) & g(x) =  | x |
 0 x0


(ii) f(x) = sin–1x + cos –1x & g(x) =
2
Answer (i) Yes (ii) No

J. General :
If x, y are independent variables, then:
(i) f (xy) = f (x) + f (y)  f (x) = k ln x or f (x) = 0.
(ii) f (xy) = f (x). f (y)  f (x) = x n, n  R
(iii) f (x + y) = f (x). f (y)  f (x) = akx.
(iv) f (x + y) = f (x) + f (y)  f(x) = kx, where k is a constant.

 1  1
(v) f(x) . f   = f(x) + f    f(x) = 1 ± x n where n  N
x x
TM
RESONANCE 20
Illustration # 18

 1  1
If f(x) is a polynomial function satisfying f(x) . f   = f(x) + f    x  R – {0} and
x
  x
f(2) = 9, then find f (3)
Solution
f(x) = 1 ± x n
As f(2) = 9  f(x) = 1 + x 3
3
Hence f(3) = 1 + 3 = 28

Assignment:

 1  1
14. If f(x) is a polynomial function satisfying f(x) . f   = f(x) + f    x  R – {0} and f(3) = –8, then find
x
  x
f(4)
Answer – 15

f (x)
15. If f(x + y) = f(x) . f(y) for all real x, y and f(0)  0 then prove that the function, g(x) = is an even
1  f 2 (x)
function

TM
RESONANCE 21
Part : (A) Only one correct option

 log0.3 ( x  1)
1. The domain of the function f(x) = is
x 2  2x  8
(A) (1, 4) (B) (– 2, 4) (C) (2, 4) (D) [2, )

2. The function f(x) = cot1 ( x 3) x + cos 1 x 2 3 x 1 is defined on the set S, where S is equal to:
(A) {0, 3} (B) (0, 3) (C) {0,  3} (D) [ 3, 0]

 2 1  2 1
3. The range of the function f (x) = sin1  x   + cos1  x   , where [ ] is the greatest integer
 2  2
function, is:

     
(A)  ,   (B) 0,  (C) {  } (D)  0, 
2   2  2

4. Range of f(x) = log 5


{ 2 (sinx – cosx) + 3} is

 3
(A) [0, 1] (B) [0, 2] (C) 0,  (D) none of these
 2

5. Range of f(x) = 4x + 2x + 1 is
(A) (0, ) (B) (1, ) (C) (2, ) (D) (3, )

6. If x and y satisfy the equation y = 2 [x] + 3 and y = 3 [x – 2] simultaneously, the [x + y] is


(A) 21 (B) 9 (C) 30 (D) 12

7. The function f : [2, )  Y defined by f(x) = x2  4x + 5 is both oneone & onto if


(A) Y = R (B) Y = [1, ) (C) Y = [4, ) (D) Y = [5, )

8. Let S be the set of all triangles and R+ be the set of positive real numbers. Then the function,
f : S  R+, f () = area of the , where  S is :
(A) injective but not surjective (B) surjective but not injective
(C) injective as well as surjective (D) neither injective nor surjective

9. Let f(x) be a function whose domain is [– 5, 7]. Let g(x) = |2x + 5|. Then domain of (fog) (x) is
(A) [– 4, 1] (B) [– 5, 1] (C) [– 6, 1] (D) none of these

e x  e x
10. The inverse of the function y = is
e x  e x

1 1 x 1 2 x 1 1 x
(A) log (B) log (C) log (D) 2 log (1 + x)
2 1 x 2 2x 2 1 x

11. The fundamental period of the function,


f(x) = x + a  [x + b] + sin x + cos 2x + sin 3x + cos 4x +...... + sin (2n  1) x
+ cos 2 nx for every a, b  R is: (where [ ] denotes the greatest integer function)
(A) 2 (B) 4 (C) 1 (D) 0

4
x  x  [ x ]  cos x
12. The period of e cos is ______(where [ ] denotes the greatest integer function)
(A) 1 (B) 2 (C) 3 (D) 4
TM
RESONANCE 22
1
13. 
If y = f(x) satisfies the condition f x  x1  =x 2 +
x2
(x  0) then f(x) =

(A)  x2  2 (B)  x 2  2 (C) x 2  2 (D) x 2  2

a x  ax
14. Given the function f(x) = (a > 0). If f(x + y) + f(x  y) = k f(x). f(y) then k has the value equal to:
2
(A) 1 (B) 2 (C) 4 (D) 1/2

15. A function f : R  R satisfies the condition, x2 f(x) + f(1  x) = 2x  x 4. Then f(x) is:
(A) – x 2 – 1 (B) – x 2 + 1 (C) x 2  1 (D) – x 4  1

1
16. The domain of the function, f (x) = x 
 1 cos 1
(2 x  1) tan 3 x is:

 
(A) ( 1, 0) (B) ( 1, 0)    
 6

     
(C) ( 1, 0]    ,   (D)   , 0 
 6 2  6 

17. If f (x) = 2 [x] + cos x, then f: R  R is: (where [ . ] denotes greatest integer function)
(A) oneone and onto (B) oneone and into
(C) manyone and into (D) manyone and onto

18. If q 2  4 p r = 0, p > 0, then the domain of the function, f (x) = log (p x 3 + (p + q) x 2 + (q + r) x + r) is:

 q    q 
(A) R     (B) R   (   ,  1]   
 2p    2 p  

  q 
(C) R   (   ,  1)    (D) none of these
  2 p  

19. If [ 2 cos x ] + [ sin x ] =  3, then the range of the function, f (x) = sin x + 3 cos x in [0, 2 ] is:
(where [. ] denotes greatest integer function)

(A) [ 2,  1) (B) ( 2,  1] (C) ( 2,  1) (D) [–2, – 3 )

  1  
20. The domain of the function f (x) = log1/2   log 2 1    1 is:
  4 
  x 
(A) 0 < x < 1 (B) 0 < x  1 (C) x  1 (D) null set

21. The range of the functions f (x) = log


2
2  log 16sin x  1 is
2
2

(A) ( 1) (B) ( 2) (C) ( 1] (D) ( 2]

 1  x3 
22. The domain of the function, f (x) = sin1   + sin (sin x ) + log(3{x} + 1) (x2 + 1),
 2 x3 / 2 
 
where {x} represents fractional part function is:
(A) x  {1} (B) x  R  {1,  1} (C) x > 3, x  I (D) none of these

TM
RESONANCE 23
23. The minimum value of f(x) = a tan2 x + b cot2 x equals the maximum value of g(x) = a sin2x + b cos2x where
a > b > 0, when
(A) 4a = b (B) 3a = b (C) a = 3b (D) a = 4b

x
24. Let f (2, 4)  (1, 3) be a function defined by f (x) = x    (where [. ] denotes the greatest integer
2
function), then f 1 (x) is equal to :

x
(A) 2x (B) x +   (C) x + 1 (D) x  1
2

25. The image of the interval R when the mapping f: R  R given by f(x) = cot–1 (x2 – 4x + 3) is

  3     3 
(A)  ,  (B)  ,  (C) (0, ) (D)  0, 
4 4  4   4 

ax 1
26. If the graph of the function f (x) = is symmetric about y-axis, then n is equal to:
x n (a x  1)
(A) 2 (B) 2 / 3 (C) 1 / 4 (D) – 1 / 3

 
27. If f(x) = cot–1x : R+   0, 
 2
and g(x) = 2x – x2 : R  R. Then the range of the function f(g(x)) wherever define is

        
(A)  0,  (B)  0,  (C)  ,  (D)  
 2  4 4 2  4

28. Let f: (e2, )  R be defined by f(x) =n (n(n x)), then


(A) f is one one but not onto (B) f is on to but not one - one
(C) f is one-one and onto (D) f is neither one-one nor onto

29. Let f: (e, )  R be defined by f(x) =n (n(n x)), then


(A) f is one one but not onto (B) f is on to but not one - one
(C) f is one-one and onto (D) f is neither one-one nor onto

30. Let f(x) = sin x and g(x) = |n x| if composite functions fog(x) and gof (x) are defined and have ranges
R1 & R2 respectively then.
(A) R1 = {u: – 1 < u < 1} R2 = {v: 0 < v < }
(B) R1 = {u: –  < u < 0} R2 = {v: –1< v < }
(C) R1 = {u: 0 < u < } R2 = {v:– 1 < v < 1; v  0}
(D) R1 = {u: –1 < u < } R2 = {v:0 < v < }
2
( x 3 x  2 )
31. Function f : (– , 1)  (0, e5] defined by f(x) = e is
(A) many one and onto (B) many one and into (C) one one and onto (D) one one and into

32. The number of solutions of the equation [sin–1 x] = x – [x], where [ . ] denotes the greatest integer function is
(A) 0 (B) 1 (C) 2 (D) infinitely many

x x
33. The function f(x) = x + + 1 is
e 1 2
(A) an odd function (B) an even function
(C) neither an odd nor an even function (D) a periodic function
TM
RESONANCE 24
Part : (B) May have more than one options correct

34. For the function f(x) = n (sin 1 og2 x),

1   
(A) Domain is  , 2 (B) Range is    , n 
2    2

(C) Domain is (1, 2] (D) Range is R

35. A function ' f ' from the set of natural numbers to integers defined by,

n  1
 , when n is odd
f (n) =  2n is:
 , when n is even
 2
(A) one-one (B) many-one (C) onto (D) into

36. Domain of f(x) = sin 1 [2  4x 2] where [x] denotes greatest integer function is:

 3 3    3 3  3 3 
3 3
(A)   ,
  {0} (B)  ,   {0} (C)  2 , 2  (D)   ,
 2 2   2 2     2 2 

sin  [ x ]
37. If F (x) = , then F (x) is:
{x}
(A) periodic with fundamental period 1
(B) even
(C) range is singleton
 { x } 
(D) identical to sgn  sgn  1, where {x} denotes fractional part function and [. ] denotes
 { x } 

greatest integer function and sgn (x) is a signum function.

38. D  [ 1, 1] is the domain of the following functions, state which of them are injective.
(A) f(x) = x2 (B) g(x) = x 3 (C) h(x) = sin 2x (D) k(x) = sin (x/2)

1
1. Find the domain of the function f(x) = + x2
log10 (1  x )

 3x  1 
1 2 x + 3 sin  2 
–1
2. Find the domain of the function f(x) =

3. Find the inverse of the following functions. f(x) = n (x + 1 x 2 )

  
4. Let f :  ,   B defined by f (x) = 2 cos2x + 3 sin2x + 1. Find the B such that f –1 exists. Also find
 3 6
–1
f (x).

5. Find for what values of x, the following functions would be identical.

 x 1 
f (x) = log (x - 1) - log (x - 2) and g (x) = log  x  2  .
 
TM
RESONANCE 25
4x
6. If f(x) = x , then show that f(x) + f(1 – x) = 1
4 2

 1  1
7. Let f(x) be a polynomial function satisfying the relation f(x). f   = f(x) + f    x  R – {0} and
x
  x

f(3) = –26. Determine f(1).

8. Find the domain of definitions of the following functions.

x 2 1 x
(i) f (x) = 3  2 x  21  x (ii) f (x) = +
x2 1 x

(iii) f (x) = og10 (1 – og10(x2 – 5x + 16))

9. Find the range of the following functions.

 4  x2 
x 2  2x  4  
(i) f (x) = (ii) f (x) = sin og  1  x 
x 2  2x  4  

(iii) f (x)= x 4  2 x2 + 5 (iv) f (x) = sin2 x + cos4x

10. Solve the following equation for x (where [x] & {x} denotes integral and fractional part of x)
2x + 3 [x] – 4 {–x} = 4

11. Draw the graph of following functions where [.] denotes greatest integer function and { .} denotes fractional
part function.
(i) y = {sin x } (ii) y = [x] + {x }

2
12. Draw the graph of the function f(x) = x – 4 | x |  3 and also find the set of values of ‘a’ for which the

equation f(x) = a has exactly four distinct real roots.

13. Examine whether the following functions are even or odd or none.

 x | x |, x  1
(1 2 ) x 7 
[1  x ]  [1  x ],  1  x  1
(i) f (x) = (ii) f (x) = 
2x   x | x |,
 x 1

2x (sinx  tanx )
(iii) f (x) = , where [ ] denotes greatest integer function.
 x  2 
2   3
 

TM
RESONANCE 26
14. Find the period of the following functions.
sin2 x cos 2 x
(i) f (x) = 1  1  cot x  1  tanx

(ii) f (x) = tan [ x ] , where [.] denotes greatest integer function.
2

1  |sin x | sinx 
(iii) f (x) =   
2  cos x | cosx | 

sin x  sin 3 x
(iv) f (x) =
cos x  cos 3 x

1  x 2 x 1
15. If f(x) =  and g(x) = 1 – x ; – 2 < x < 1 then define the function fog(x).
 x  1 1  x  2

1
16. Find the set of real x for which the function, f (x) = is not defined, where [x]
[| x  1 |]  [| 12  x | ] 11

denotes the greatest integer not greater than x.

 
cos 1 sin x     4  2cosx 
17. Given the functions f(x) = e 3 , g(x) = cosec1   & the function
 3 

h(x) = f(x) defined only for those values of x, which are common to the domains of the functions f(x) and
g(x). Calculate the range of the function h(x).

18. Let ‘f’ be a real valued function defined for all real numbers x such that for some positive constant ‘a’ the
1 2
equation f ( x  a)   f ( x )  f ( x ) holds for all x. Prove that the function f is periodic.
2

19. If f (x) =1 + x  2, 0  x  4


g (x) = 2  x ,  1  x  3
Then find fog (x), gof (x), fof(x) & gog(x). Draw rough sketch of the graphs of fog (x) & gof (x).

20. Find the integral solutions to the equation [x] [y] = x + y. Show that all the non-integral solutions lie on
exactly two lines. Determine these lines. Here [ .] denotes greatest integer function.

TM
RESONANCE 27
Exercise # 1
1. D 2. C 3. C 4. B 5. B 6. C 7. B
12. a  (1, 3)  {0}
8. B 9. C 10. A 11. A 12. B 13. D 14. B

15. B 16. D 17. C 18. B 19. D 20. D 21. D


13. (i) neither even nor odd (ii) even (iii) odd
22. D 23. D 24. C 25. D 26. D 27. C 28. A
14. (i)  (ii) 2 (iii) 2  (iv) 
29. C 30. D 31. D 32. B 33. B 34. BC

35. AC 36. B 37. ABCD 38. BD 2  2x  x 2 0  x 1


15. f(g(x)) = 
 2  x 1 x  0
Exercise # 2
  
 1 1 16. (0, 1) U {1, 2,......., 12} U (12, 13) 17. e 6 , e  
1. [–2, 0)  (0, 1) 2.  3 , 2   

18. Period 2 a
e x  e x
3. f –1 =
2
 (1  x ) ,  1  x  0
19. fog (x) =  ;
1  1 x  2   
 sin   x 1 , 0  x  2
4. B = [0, 4] ; f –1
(x) =   
2   2  6
x 1 , 0  x 1

3  x , 1 x  2
5. (2, ) 7. – 3 8. (i) [0, 1] (ii)  (iii) (2, 3) gof(x) = 
x 1 , 2x3
5  x , 3x4
1  3 
9. (i)  3 , 3 (ii) [ 1, 1] (iii) [4, ) (iv)  , 1  x , 0x2
  4 
fof (x) =  ;
 4  x , 2x2

3    x , 1 x  0
10.   
2 gog(x) =  x , 0x2
4  x , 2  x  3

20. Integral solution (0, 0); (2, 2). x + y = 6, x + y = 0


11. (i)

(ii)

TM
RESONANCE 28
Limit of Functions

1. Limit of a function f(x) is said to exist as,


x  a when,
Limit f (a  h) = Limit f (a + h) = some finite value M.
h 0 h 0
Even as the finite (Left hand limit) (Right hand limit)
Note that we are not interested in knowing about what happens
enclosesaninfinite at x = a. Also note that if L.H.L. & R.H.L. are both tending towards
'  ' or ‘–’ then it is said to be infinite limit.
series and in the
Remember, Limit
x a  x  a
unlimited limits Solved Example # 1

appear, So the soul

of immensity dwells

in minutia And in Find xLimit


 / 2 f(x)

the narrowest
Solution.
limits no limit in Here Limit f(x) = 1
x / 2

here. What joy to Solved Example # 2

discern the minute

in infinity! The

vast to perceive in Find Limit


x1 f(x)

the small, what


Solution.
divinity!
Left handed limit = 1 Right handed limit = 2
Ber noul l i Jacob, Hence Limit
x1 f(x) = does not exist.

Ar s Conj ect andi Solved Example # 3


(i) Find lim it f(x)
x 0
(ii) Find lim it f(x)
x 1

(iii) Find lim it f(x)


x 3

TM
RESONANCE 29
Solution.
(i) limit f(x) = does not exists
x 0
because left handed limit  right handed limit

(ii) limit f(x) = 0 (iii) limit f(x) = 1


x 1 x 3

2. Indeterminant Forms:
0 
, , 0 , º, 0º,and 1 .
0 
Solved Example # 4
Which of the following limits are forming indeterminant from also indicate the form

lim 1 lim 1 x
(i) (ii)
x 0x x 0 1 x 2
1 1 
(iii) lim x n x (iv) lim   
x 0 x 0 x x2 
(v) lim (sin x)x (vi) lim (n x)x
x 0 x 0
1
lim x lim (1)1/x
x 0 1 sin x 
(vii) (viii) x 0

Solution
0
(i) No (ii) Yes from
0
(iii) Yes 0 ×  form (iv) Yes ( – ) form
(v) Yes, (0)º form (vi) Yes ()º form
(vii) Yes (1) form (viii)
NOTE :
(i) ' 0 ' doesn't means exact zero but represent a value approaching towards zero similary to ' 1 ' and
infinity.
(ii) +=
(iii)  x  = 
(iv) (a/) = 0 if a is finite
a
(v) is not defined for any a  R.
0
(vi) a b = 0, if & only if a = 0 or b = 0 and a & b are finite.

3. Method of Removing Indeterminancy


To evaluate a limit, we must always put the value where ' x ' is approaching to in the function. If we get
a determinate form, then that value becomes the limit otherwise if an indeterminant form comes. Then
apply one of the following methods:
(i) Factorisation (ii) Rationalisation or double rationalisation
(iii) Substitution (iv) Using standard limits
(v) Expansions of functions.

1. Factorization method :-
We can cancel out the factors which are leading to indeterminancy and find the limit of the remaing
expression.

TM
RESONANCE 30
Solved Example # 5
6
limit x  24 x  16
x 2
x 3  2 x  12
Solution.
6 ( x  2)( x 5  2x 4  4 x 3  8 x 2  16 x  8)
limit x  24 x  16 = limit
x 2
x 3  2 x  12 x 2 ( x 2  2x  6)( x  2)

168
= = 12
14

2. Rationalization /Double Rationalization.

We can rationalize the irrational expression by multiplying with their conjugates to remove the
indeterminancy.

Solved Example # 6

4  5x  1
limit .
x 1 2  3x  1
Solution.

4  5x  1
limit
x 1 2  3x  1

( 4  5 x  1)(2  3 x  1)( 4  5 x  1)
= lim it
x 1 (2  3 x  1)( 4  5 x  1)(2  3 x  1)

(15  5 x ) 2  3x  1
= lim it ×
x 1 (3  3 x ) 4  5x  1
5
=
6
Solved Example # 7
Evaluate :
 1 2 ( 2 x  3)  1 x  1 x
(i) lim   3  (ii) lim
x 2 x  2 x  3 x 2  2x  x 0
x

(iii)

lim (2 x  3) x  1 
x 1
2x 2  x  3
Solution
(i) We have
 1 2 ( 2 x  3)   1 2 ( 2 x  3) 
lim   3  = xlim   
x 2 x  2 x  3 x 2  2x  2
 x  2 x ( x  1)( x  2) 

 x ( x  1)  2(2 x  3) 
= xlim  
2
 x ( x  1)( x  2) 

 x 2  5x  6 
lim
= x 2  x( x  1)( x  2) 
 

 ( x  2)( x  3) 
= xlim  
2
 x ( x  1)( x  2) 

 x 3  1
= xlim   =–
2
 x ( x  1)  2
TM
RESONANCE 31
0
(ii) The given limit taken the form when x  0. Rationalising the numerator, we get
0

1 x  1 x  1 x  1 x 1 x  1 x 
lim = xlim   
x 0
x 0 
 x 1  x  1  x 

 (1  x )  (1  x ) 
= xlim  
0  x
 
1  x  1  x  
 2x 
= xlim  
0  x
  1  x  1  x  
 2  2
= xlim
0
  = =1
 1  x  1  x  2
(iii) We have

lim 
 
 ( 2 x  3) x  1 
 = lim 

 ( 2 x  3) x  1 


2
x 1  2 x  x  3  x 1  ( 2 x  3)( x  1) 
   

 
( 2 x  3) x  1  
= xlim  
1  (2 x  3)
  
x  1 x  1  
 2x  3 
= xlim  
1  ( 2 x  3)
 
x  1 
1 1
= =
(5)(2) 10

4. Fundamental Theorems on Limits:


Let Limit Limit
x a f (x) =  & x a g (x) = m. If  & m exists then:

Limit { f (x) ± g (x) } = ± m


(i) x a 

Limit { f(x). g(x) } = . m


(ii) x a 

(iii) Limit f ( x ) =  , provided m 0


x a
g(x) m

(iv) Limit k f(x) = k Limit f(x) ; where k is a constant.


x a x a

 
(v) Limit f [g(x)] = f  Limit g (x )  = f (m); provided f is continuous at g (x) = m.
x a  
 x a 

Solved Example # 8

Evaluate
(i) lim (x + 2) (ii) lim x(x – 1)
x 2 x 2

2
(iii) lim x  4 (iv) lim cos (sin x)
x 2 x 0
x2
2 2
(v) lim x  3 x  2 (vi) lim x  3 x  2
x 1
x2 1 x 1
x2 1

TM
RESONANCE 32
Solution
(i) x + 2 being a polynomial in x, its limit as x  2 is given by xlim
2
(x + 2) = 2 + 2 = 4

(ii) Again x(x – 1) being a polynomial in x, its limit as x  2 is given by


lim x(x – 1) = 2 (2 – 1) = 2
x 2

2 2
(iii) By (II) above, we have lim x  4 = (2)  4 = 2
x 2
x2 22

(iv) lim cos (sin x) = cos  lim sin x  = cos 0 = 1


x 0  x 0 
(v) Note that for x = 1 both the numerator and the denominator of the given fraction vanish. Therefore
x 2  3x  2 ( x  1)( x  2) x2 1
by (III) above, we have xlim
1 2 = xlim
1 ( x  1)( x  1)
= xlim
1 x  1
=–
x 1 2
(vi) Note that for x = 1, the numerator of the given expression is a non-zero constant 6 and the
6
denominator is zero. Therefore, the given limit is of the form . Hence, by (IV) above, we
0
2
conclude that lim x  3 x  2 does not exist
x 1
x2 1

5. Standard Limits:
Limit sinx = 1 = Limit tanx = Limit tan1x Limit sin1x
(a) x 0 x 0 x 0 = x 0
x x x x
[ Where x is measured in radians ]
x

(b) Limit (1 + x)1/x = e ; Limit 1  1  = e


x 0 x  x

x x
(c) Limit e  1 = 1; Limit a  1 = log a, a > 0
x 0 x 0 e
x x
n n
(d) Limit n(1  x ) = 1 (e) Limit x  a = nan – 1.
x 0 x a
x x a

Solved Example # 9

sin 2 x
Find Limit
x 0
x
Solution.

Limit sin 2 x  Limit sin 2 x . 2 =2


x 0 x 0
x 2x

Solved Example # 10
3x
Limit e  1
x 0
x/2
Solution.
3x
Limit e  1
x 0
x/2
3x
Limit 2 × 3 e  1
x 0
3x
= – 6.
TM
RESONANCE 33
Solved Example # 11

Limit tan x  sin x


x 0
x3
Solution.

Limit tan x  sin x


x 0
x3
tan x (1  cos x )
= Limit
x 0
x3
x
tan x . 2 sin2
= Limit
x 0
2
x3
2
 x 
 sin 
tan x  2 
= Limit
x 0 .  x  = 1.
x  
 2 
Solved Example # 12
sin 2 x
Compute xlim
0 sin 3 x

Solution
We have
 sin 2 x 2 x 3x 
lim sin 2 x = xlim . .
x 0sin 3 x 0 
 2 x 3 x sin 3 x 

 sin 2 x  2  3x 
= 2lim  . .  lim ,x  0
 x 0 2 x  3 3 x 0 sin 2 x 

2  sin 3 x  2 2
=1. + 3lim  = ×1=
3  x 0 3 x  3 3

Solved Example # 13
x
 2
Evaluate xlim 1  

 x
Solution
x 2
lim 1  2  = e xlim
 x
.x
= e 2.
x  x

Solved Example # 14
x 3 x
Compute (i) lim e  e (ii) lim x(e  1)
x 3 x 3 x 0 1  cos x
Solution

(i) Put y = x – 3. So, as x  3, y  0. Thus


x
lim e  e
3
lim e3  y  e 3
x 3
= y 0
x 3 y

e 3 . e y  e3
= ylim
0 y

ey 1
= e 3 ylim
0 = e3 . 1 = e3
y
TM
RESONANCE 34
(ii) We have
x
lim x(e  1) x(e x  1)
(ii) x 0 1  cos x
= xlim
0 x
2 sin2
2

 
ex  1 x2 
1 .
= . lim  x x
 = 2.
2 x 0  sin2 
 2 
Solved Example # 15
x3  8
Evaluate xlim
2
x2  4
Solution
(First Method)
The given expression is of the form

x 3  ( 2) 3 x 3  ( 2) 3 x 2  ( 2) 2
2 2 = 
x  ( 2) x 2 x2
3
lim x  8 x 3  ( 2) 3 x 2  ( 2) 2
 x 2 2 = xlim
2
 xlim
2
x 4 x 2 x 2
n n
= 3(22)  2(21) lim x  a = nan–1 )
(using x a x a
= 12  4 = 3
(Second Method)
The numerator and denominator have a common factor (x – 2). Cancelling this factor, we obtain
x3  8 x 2  2x  4
2 =
x 4 x2
3
lim x  8 x 2  2x  4
 x 2 2 = xlim
2
x 4 x2

(2)2  2(2)  4 12
= = =3
22 4
Note : Since x  2, x – 2 is not zero, so the cancellation of the factor x – 2 in the above example is
carried out.

6. Use of Subsitution in Solving Limit Problems


lim f(x) by subtituting x = a + h or x = a – h as
Sometimes in solving limit problem we convert x a

lim f(a + h) or lim f(a – h) according as need of the problem.


h 0 h 0

Solved Example # 16
1  tan x
limit
x  / 4 1  2 sin x

Solution. Put x= +h
4

 x  h0
4

limit
h0

TM
RESONANCE 35
1  tan h
1
1  tan h
= lim
h0
it =
1  sin h  cos h

2 tan h
1  tan h
= lim
h0
it
h h h
2 sin2  2 sin cos
2 2 2

2 tan h 1
= lim it
h0 h h h (1  tanh)
2 sin2 2 sin  cos 
2 2 2

tanh
2
h 1
= lim it
h
h0
sin (1  tanh)
2 sin h  cos h 
h  2 2 
2

2
= = 2.
1

7. Limit When x  
1
Since x     0 hence in this type of problem we express most of the part of expression
x
1 1
in terms of and apply  0. We can see this approch in the given solve examples.
x x

Solved Example # 17

limit x sin 1
x 
x
Solution.

limit x sin 1
x 
x

= lim it sin1/ x
x 
1/ x
=1

Solved Example # 18

limit x 2
x  2x  3
Solution.

limit x 2
x  2x  3

limit 1  2 / x
x  2  3 / x

1
= .
2

TM
RESONANCE 36
Solved Example # 19

limit x 2  4x  5
x 
3x 2  x 3  2
Solution.

limit x 2  4x  5
x 
3x 2  x 3  2

1 4 5
 
x x2 x3
= lim
x 
it 3 2
 1 3
x x
=0
Solved Example # 20

lim 3x 2  2
x  
x 2
Solution.

lim 3x 2  2
x  
x 2
1
Put x = x  – t  0+
t

1
3  2t 2 .
t2
= xlim
 0 1  2t
t

3  2t 2 t
= xlim
 0
 (1  2t ) | t |

3
= =– 3.
1

8. Limits Using Expansion


x ln a x 2 ln2 a x 3 ln3 a x x2 x3
(i) a x  1    .........a  0 (ii) e x 1     ......
1! 2! 3! 1! 2! 3!

x2 x3 x4 x3 x5 x7
(iii) ln (1+x) = x     .........for  1  x  1 (iv) sin x x     .....
2 3 4 3! 5! 7!

x2 x4 x6 x 3 2x 5
(v) cosx 1     ..... (vi) tan x = x    ......
2! 4! 6! 3 15

x3 x5 x7 12 3 12.3 2 5 12.3 2.5 2 7


(vii) tan-1x = x     .... (viii) sin-1x = x  x  x  x  .....
3 5 7 3! 5! 7!

x 2 5 x 4 61x 6
(ix) sec -1x = 1     ......
2! 4! 6!
n(n  1) n(n  1)(n  2)
(x) for |x| < 1, n  R (1 + x)n = 1 + nx + 1. 2 x +
2
1. 2 . 3 x3 + ............

TM
RESONANCE 37
Solved Example # 21

lim e x  1 x
x 0
x2
Solution.

lim e x  1 x
x 0
x2

 2 
1  x  x .......   1  x
 2! 
= xlim  
0 2
x

Solved Example # 22
tan x  sin x
x3
Solution.

lim tan x  sin x


x 0
x3

 3   3 
 x  x ........   x  x .......
   
= xlim
0  3   3! 
x3

1 1 1
= + = .
3 6 2

Solved Example # 23

lim 7  x )1/ 3  2
x 0
x 1
Solution.
Put x  1 + h

lim (8  h)1/ 3  2
h0
h
1/ 3
 h
2 . 1   2
lim  8
h0
h

 2 
1  1  h 
   1  
 1 h 3  3  8  
2 1  .   .......  1
 3 8 1. 2 
 
= hlim
0
 
h

1 1
= hlim
0 2 × =
24 12

TM
RESONANCE 38
Solved Example # 24

x2
n(1  x )  sin x 
lim 2
x 0
x tan x sin x

x2
n(1  x )  sin x 
lim 2
x 0
x tan x sin x

 2 3   3 5  2
 x  x  x .....   x  x  x  .....  x
 2 3   3! 5!  2
   
= xlim
0 tan x sin x
x3 . .
x x

1 1 1
= + =
3 6 2

9. Limits of form 1 , 0 0 ,  0

0
All these forms can be convered into form in the following ways
0
(i) If x  1, y  , then z = (x)y
 n z = y n x
nx
 n z =
(1/ y )

1
Since y   hence  0 and x  1 hence nx  0
y
(ii) If x  0, y  0, then z = x y
 n z = y n x
y 0
 n z = = form
1/ ny 0
(iii) If x   , y  0, then z = xy
 n z = y n x
y 0
 n z = = form
1/ nx 0
also for (1) type of problems we can use following rules.
(i) lim (1 + x)1/x = e
x 0

(ii) lim [f(x)]g(x)


xa
where f(x)  1 ; g(x) as x  a
1
{ f ( x ) 1} . g ( x )

= xlim
f ( x ) 1

a 1  f ( x )  1

= e xlim
a
[ f ( x ) 1] g ( x )

TM
RESONANCE 39
Solved Example # 25
4 x  2
 2 
lim  2 x  1 
x   2 x  3 
2
 
Solution.
Since it is in the form of 1 
4 x  2
 2 
lim  2 x  1 
x   2 x 2  3 
 

 2x 2  1  2x 2  3 
= e xlim
  (4x2 + 2) = e–8
  2 
 2 x  3 

Solved Example # 26
lim
 (tan x)tan 2x
x
4
Solution
lim (tan x 1) tan 2 x

x
= e 4

2 tan x
lim (tan x 1)
x
 1 tan2 x
= e 4

tan  / 4
2
1(1 tan  / 4 )
= e
1
= e–1 =
e

Solved Example # 27
x
tan
lim  2  a 
Evaluate x
2a
.
a
 x
Solution.
x
tan
lim  2  a 
2a
xa x

put x=a+h
  h 
tan  
 lim 1  h   2 2a 
h 0  
 (a  h ) 
 h 
cot 
 lim 1  h   2a 
h 0
 ah
h  h 
lim cot .  1  1
 h0 2a  a  h 
e
 h  2a
 
lim   2a . 
h 0  h  ah
 tan 
  2a  = e–2/
e

TM
RESONANCE 40
Solved Example # 28
lim x x
x 0 
Solution.
y = xlim
0
xx

n y = xlim
0
x n x

1
n 1
= xlim – x =0 
0 1 x
x
y=1

1 0 . Sandwich Theorem or Squeeze Play Theorem:


If f(x)  g(x)  h(x)  x & Limit Limit Limit
x a f(x) =  = x a h(x) then x  a g(x) =  .

Solved Example # 29
[ x ]  [2x ]  [3 x ]  ....  [nx ]
Evaluate nlim

n2
Where [ ] denotes the greatest integer function.
Solution.
We know that, x – 1 < |x|  x
 2x – 1 < [2x]  2x
 3x – 1 < [3x]  3x
....................
....................
 nx – 1 < [nx]  nx
 (x + 2x + 3x + .... + nx) – n < [x] + [2x] + ..... +[nx]  (x + 2x + .... + nx)
n
xn(n  1) x.n(n  1)

2
–n<  [r x] 
r 1
2

Thus, lim [ x ]  [2x ]  ....  [nx ]


n  n2

lim x 1  1  1 [ x ]  [2x ]  ....  [nx ] x 1  1 


   – < nlim 2  nlim  
n  2  n  n   n   2  n

x [ x ]  [2x ]  ....  [nx ] x


 < nlim 
2  n2 2

 lim [ x ]  [2x ]  ....  [nx ] = x


n  n2 2

TM
RESONANCE 41
Aliter
We know that [x] = x – {x}
n

r x
r 1
= [x] + [2x] + .... + nx – [nx]
= (x + 2x + 3x + ... + nx) – ({x} + {2x} + .... + {nx})
xn(n  1)
= – ({x} + {2x} + .. + {nx})
2
n
1 x 1  1  { x }  {2x }  ....  {nx }

n 2
 [r x] =
r 1

2  n
 –
 n2
n

Since, 0  {rx} < 1,  0  [r x] < n


r 1

lim
 [rx]
 n 
r 1 =0
n2
n n

lim
 [rx ]
x 1  1   {rx}
 r 1 = nlim   – lim r 1
n   2  n n 
n2 n2
n

lim
 [rx] x
n 
r 1 =
2
n2
Solved Example # 30
lim it x sin 1
x 0 x
Solution.
lim it x sin 1
x 0 x
= 0 × (some value in [– 1, 1]) = 0

1 1 . Some Important Notes :


lim nx = 0
x
(i) (ii) lim =0
x  x x   ex
As x  , n x increnes much slower than any (+ve) power of x where ex increases much faster than
(+ve) power of x
Limit (1  h)n = 0 & Limit
n (1 + h)  where h > 0.
n
(iii) n 

(iv) If Limit Limit


x  a f(x) = A > 0 & x  a (x) = B (a finite quantity) then;
Limit [f(x)](x) = ez where z = Limit (x). ln[f(x)] = eBlnA = AB
x a x a

Solved Example # 31
1000
lim x
x  ex
Solution.
1000
lim x =0
x 
ex

TM
RESONANCE 42
n 1
Limit 5  3n  22 n
1. n  n n 2n  3
=
5 2 3
(A) 5 (B) 3 (C) 1 (D) zero

cos2  cos2x
2. Limit =
x  1 x2  | x |
(A) 2 cos 2 (B)  2 cos 2 (C) 2 sin 2 (D)  2 sin 2

1 1 cos2x
3. The value of Limit
x 0 is:
x 2
(A) 1 (B)  1 (C) 0 (D) none

4. Limit sin1 (sec x).


x 0

(A) is equal to /2 (B) is equal to 1 (C) is equal to zero (D) none of these

2
5. Limit x  9 x  20 where [x] is the greatest integer not greater than x:
x 5 x  [x]
(A) is equal to 1 (B) 0 (C) 4 (D) none

| x|
6. Limit
x   sinx :
(A) is equal to  1 (B) is equal to 1 (C) is equal to  (D) does not exist

7. Limit ( x  27 ) ln ( x  2) =
x3 2 ( x 9)
(A)  8 (B) 8 (C) 9 (D)  9

100
k
x 100
8. Limit k 1
=
x 1
x 1
(A) 0 (B) 5050 (C) 4550 (D)  5050

9. Limit
x   ( x  a)(x  b)  x  =
ab
(A) ab (B) (C) ab (D) none
2

x 3 . sin x1  x  1
10. Limit =
x 
x2  x  1
(A) 0 (B) 1/2 (C) 1 (D) none

11. Limit ( n  2) !  ( n  1) ! , n N=


n 
( n  3) !
(A) 0 (B) 1 (C) 2 (D)  1
TM
RESONANCE 43
12. Limit | x |sinx =
x 0

(A) 0 (B) 1 (C)  1 (D) none of these

x
 2 
13. Limit  x  2x  1  =
x   2 
 x  4x  2 
(A) 1 (B) 2 (C) e2 (D) e

x (1 a cos x )  b sin x


14. The values of a and b such that xLim = 1 are
0
x3
5 3 5 3 5 3 5 3
(A) , (B) ,– (C) – ,– (D) – ,
2 2 2 2 2 2 2 2

    
2  3 sin  x   cos  x  
Limit  6  6 
15. =
x 0

x 3 3 cos x  sin x 
(A)  1/3 (B) 2/3 (C) 4/3 (D)  4/3

 x 1 , x 1  x 1 , x0
16. If f(x) =  2 , g(x) =  2 and h(x) = |x|
 2 x  2 , x 1   x  1 , x0

then find Lim


x 0
f(g(h(x)))
(A) 1 (B) 0 (C) –1 (D) does not exists

17. Limit (1  x + [x  1] + [1  x]) = where [x] denotes greatest integer function.


x 1

(A) 0 (B) 1 (C)  1 (D) does not exist

 sin  x  3
18. Lt   , where [ . ] denotes greatest integer function is :
x0
 [ x  3] 
(A) 0 (B) 1 (C) does not exist (D) sin 1

  1  1 
x sin   sin  2  x 0
19. Let f (x) =  x x  , then xlim
 f(x) equals
 0 x0

(A) 0 (B)  1/2 (C) 1 (D) none of these.

 3 3
| x| x
20. lim      (a > 0), where [x] denotes the greatest integer less than or equal to x is
xa  a
  a  
(A) a2 1 (B) a 2  1 (C) a2 (D) – a2

Lim ax 2  bx  c
21. Let ,  be the roots of ax2 + bx + c = 0, where 1 <  < . Then x  x = 1 then which of the
0
ax 2  bx  c
following statements is incorrect
(A) a > 0 and x0 < 1 (B) a > 0 and x0 > 
(C) a < 0 and  < x0 <  (D) a < 0 and x0 < 1

TM
RESONANCE 44
22. Limit 1.n  2 ( n  1 )  3 ( n  2 )  ......  n .1 has the value :
n 
1 2  2 2  3 2  ......  n 2

1 1 1
(A) (B) (C) (D) 1
2 3 4

 sin x 
23. x0 

Lim  1  e x 
x  is (where [] represents greatest integral part function)
(A) – 1 (B) 1 (C) 0 (D) does not exist

24. If  = xlim lim


  (sin x  1 – sin x ) and m = x    [sin x  1 – sin x ] where [.] denotes the greatest integer

function then :
(A)  = m = 0 (B) = 0 ; m is undefined
(C) , m both do not exist (D)  = 0, m  0 (although m exist)

n
 1
25. If f(x) =   x     x   1 1 then lim
n f(0) is.
 1
(A) 1 (B) –1 (C) 2 (D) None

  n sin    n tan   
26. The limit lim 
0      , where [x] is the greatest integer function and n , is
       
(A) 2n (B) 2n + 1 (C) 2n – 1 (D) does not exist

 1
27. The limit Limit
x  x  x2 n  1   is equal to :
x
(A) 1/2 (B) 3/2 (C) 1/3 (D) 1

 
x  2 
28. lim  
x / 2  cos x  is : (where [.] represents greatest integer function.
 
(A) – 1 (B) 0 (C) – 2 (D) does not exist

29. If f(x) = sin x , x  n, n = 0 , ± 1 , ± 2 , ± 3, ,....


= 2 , otherwise and
g(x) = x² + 1 , x0,2
= 4 , x=0
= 5 , x=2
Limit
then x 0 g [f(x)] is :
(A) 1 (B) 0 (C) 4 (D) does not exists

 2x x
30. The graph of the function f(x) = tLim  cot 1 2  , is
0   t 

(A) (B) (C) (D)

TM
RESONANCE 45
cos (sin x )  cosx
31. The value of Limit
x0 is equal to:
x4
(A) 1/5 (B) 1/6 (C) 1/4 (D) 1/2

 1 1 
x  xxn n x 
e   2  e   3 x  e 
   
32. Limit   , n  N is equal to :
x 
xn
(A) 0 (B) ln(2/3) (C) ln(3/2) (D) none

 ay   by   
 exp x ln 1  x    exp x ln 1  x   
        
33. Limit  Limit
x  y =
y 0  
 
 
(A) a + b (B) a  b (C) b  a (D)  (a + b)

1. Evaluate the following limits, where [ . ] represents greatest integer function and { . } represents
fractional part function
x
(i) lim [sin x] (ii) lim   (iii) lim sgn [tan x]
 x 2  2  x 
x
2

 x 2  2 , x  2  2x , x  1
2. If f(x) =  and g(x) =  , evaluate xlim f (g(x)).
 1  x , x  2 3  x , x  1 1

3. Evaluate each of the following limits, if exists

x2  4 a  2x  3 x
(i) lim (ii) lim ,a0
x 2 x  2  3x  2 x a 3a  x  2 x

4. Evaluate the following limits, if exists


5 5
tan 3 x  2x
(i) lim (ii) lim ( x  2) 2  (a  2) 2
x 0 3x  sin 2 x x a
xa

(iii) lim x e
2 x

 e2 
x 0 1  cos x

5. Evaluate the following limits, if exist :


 1 2 x 
(i) lim x2  x  1 – x (ii) lim   2  ....  2 
x  x   x 2
x x 

(iii) Limit
x  cos  
x 1  cos x   (iv) lim
x 
x 2  8x + x

6. Evaluate the following limits using expansions :

tan2 x
e x  1  sin x 
(i) lim 2
x 0
x3

a  b sin x  cos x  ce x
(ii) If xlim exists, then find values of a, b, c. Also find the limit
0
x3
TM
RESONANCE 46
[1 . 2x ]  [2 . 3 x ]  .....  [n . (n  1) x ]
7. Evaluate xlim

where [ . ] denotes greatest integer function
n3

x 2n  1
8. If f(x) = nLim
  2n , find range of f(x).
x 1

9. Evaluate the following limits

(ln (1  x )  ln 2)(3.4 x 1  3 x ) x x
(i) Limit (ii) Limit (cos ) (sin ) cos2 ,  0 ,  
x 1
1 1
x4 x 4  
[(7  x )  (1  3 x ) ]. sin( x  1)
3 2
 2

10. Evaluate the following limits

 
x 5 tan  12   3 | x |2  7
(i) Limit x 3  x 2  1  x 4  x 2  (ii) Limit  x 
x    x 
  | x |3 7 | x |  8

11. Evaluate the following limits

 
sec2   
 2  b x
(i) Limit  sin 2    
x 0   
  2  a x 

nx
 1/ x  a 1/ x  a 1/ x  .......  a 1/ x 
(ii) Limit  a 1 2 3 n 
x   n  , where a1, a2, a3,......., an > 0.
 

12. Find the values of a & b so that:

(i)
(
Limit 1  a x sin x )  (b cos x) may find to a definite limit.
x 0
x4

(ii) lim  x 4  ax 3  3 x 2  bx  2  x 4  2x 3  cx 2  3 x  d  = 4
x   

 n 1  x (1 x ) 1 
13. Limit
Find the limits using expansion : x 0   
 x2 x 

sin 1(1  { x }) . cos 1(1  { x })


14. Let f(x) = then find Limit Limit
x0  f(x) and x0  f(x), where {.} denotes the fractional
2{ x } . (1  { x })

part function.

15. Let f (x) = Limit


 2m
 

Limit cos (n!x)  where x  R. Prove that
m  n 

1 if x is rational
f (x) = 0 if x is irrational .

TM
RESONANCE 47

  [12 (sinx ) x ] [22 (sinx ) x ] ....... [n2 (sinx ) x ] 
Limit Limit  
16. 
Evaluate x 0  n  3  ,
  n 
where [. ] denotes the greatest integer function.

17. Evaluate the following limits

(i) Limit cos x cos x cos x ........ cos x


n  n
2 4 8 2

(ii) Limit 1 tan x + 1 tan x + 1 tan x +...... + 1 tan x .


n  2 2 3 3 n n
2 2 2 2 2 2 2 2
(iii) Limit logx 1( x ).logx ( x  1).logx 1( x  2).logx  2 ( x  3).....logk ( x 5 ) ; where k = x5  1.
x 

3 3 2
23  1 3  1 4  1 n3  1
(iv) Let Pn = . 3 . 3 .......... . Prove that Limit
n  P n = .
3
2 1 3 1 4 1 3
n 1 3

Exercise # 1 x
7. 8. {–1, 0, 1}
3
1. D 2. C 3. D 4. D 5. D 6. D 7. C
9 4
8. B 9. B 10. C 11. A 12. B 13. C 14. C 9. (i)  ln
4 e
15. C 16. B 17. C 18. C 19. C 20. C 21. D (ii) cos 4  ln (cos )  sin4  ln (sin )
22. A 23. A 24. B 25. A 26. C 27. A 28. C 1 1
10. (i) (ii) 
29. A 30. C 31. B 32. B 33. B 4 2 
2
 a
Exercise # 2 e b2
11. (i) (ii) (a1 a2 a3 ....... an)

1. (i) 0 (ii) Limit does not exists


(iii) Limit does not exists 1
12. (i) a =  ,b = 1 (ii) a = 2, b  R, c = 5, d  R
2
2
2. 6 3. (i) (– 8) (ii)
3 3 1   1
13. 14. , 16.
5 2 2 2 2 3
4. (i) 1/3 (ii) (a + 2)3/2 (iii) 2e2
2
sin x 1
5. (i) 1/2 (ii) 1/2 (iii) zero (iv)  17. (i) (ii)  cot x (iii) 5
x x
1 1
6. (i) (ii) a = 2, b = 1, c = –1 and value = –
3 3

TM
RESONANCE 48
Today I am going to give two examinations one in trigonometry and one in honesty. I hope you will pass
them both, but if you must fail one let it be trigonometry, for there are many good (people) in this world
today who cannot pass an examination in trigonometry, but there are no good (people) in the world who
cannot pass an examination in honesty.
–Madison Sarrat

1. Principal Values & Domains of Inverse Trigonometric/Circular Functions:


Function Domain Range
 
(i) y = sin1 x where 1x1  y
2 2

(ii) y = cos 1 x where 1x1 0y


 
(iii) y = tan1 x where xR  y
2 2
 
(iv) y = cosec 1 x where x   1 or x  1  y ,y0
2 2

(v) y = sec 1 x where x  1 or x  1 0  y  ; y 
2
(vi) y = cot1 x where xR 0<y<

NOTE:
(a) 1st quadrant is common to the range of all the inverse functions.
(b) 3rd quadrant is not used in inverse functions.

(c) 4th quadrant is used in the clockwise direction i.e. 
 y  0.
2
(d) No inverse function is periodic. (See the graphs on page 17)

Solved Example # 1
 1  1  1  1 
Find the value of tan cos    tan    .
 2  3 
Solution
 1  1  1  1 
Let y = tan cos    tan    .

 2  3 
    
 = tan  3    6 
  


= tan  
6
1
y= Ans.
3

TM
RESONANCE 49
Self practice problems:
Find the value of the followings :
 1  1 
(1) sin  3  sin   2  Ans. 1
  
(2) cosec [sec–1 (– 2 ) + cot–1 ( –1)] Ans. –1

Solved Example # 2
Find domain of sin–1 (2x2 – 1)
Solution.
Let y = sin–1 (2x2 – 1)
For y to be defined
– 1  (2x2 – 1)  1
 0  2x2  2
 0  x2  1
 x  [–1, 1]

Self practice problems:


Find the domain of followings :
(3) y = sec–1 (x2 + 3x + 1)
 x2 
 
(4) y = cos–1  2 
 1 x 
(5) y = tan–1 ( x 2  1)
Answers (3) (– , – 3]  [ – 2, – 1]  [0, )
(4) R
(5) (– , –1]  [1, )

2. Properties of Inverse Trigonometric Functions:


Property - 2(A)
(i) sin (sin1 x) = x, 1  x  1 (ii) cos (cos 1 x) = x, 1  x  1

(iii) tan (tan1 x) = x, xR (iv) cot (cot1 x) = x, xR

(v) sec (sec 1 x) = x, x  1, x  1 (vi) cosec (cosec 1 x) = x, x  1, x  1


These functions are equal to identity function in their whole domain which may or may not be R.(See the
graphs on page 18)

Solved Example # 3
  3  
Find the value of cosec cot  cot 1  .
  4 
Solution.
  3 
Let y = cosec cot  cot 1  .......(i)
  4 
 cot (cot–1 x) = x,  x  R
 1 3   3
 cot  cot  =
 4  4
 from equation (i), we get
 3 
y = cosec  
 4 
y= 2 Ans.
TM
RESONANCE 50
Self practice problems:
Find the value of each of the following :
  1     1 3  
(6) cos sin  sin  (7) sin cos  cos 
  6    4 

3
Answers (6) (7) not defined
2

Property - 2(B)
 
(i) sin1 (sin x) = x,  x (ii) cos 1 (cos x) = x; 0  x  
2 2
 
(iii) tan1 (tan x) = x;  x (iv) cot1 (cot x) = x; 0<x<
2 2
  
(v) sec 1 (sec x) = x; 0  x  , x  (vi) cosec 1 (cosec x) = x; x  0,  x
2 2 2

These are equal to identity function for a short interval of x only.


(See the graphs on page 19-20)

Solved Example # 4
 3 
Find the value of tan–1  tan 
 4 
Solution.
 3 
Let y = tan–1  tan 
 4 
  
Note  tan–1 (tan x) = x if x    , 
 2 2
3   
   , 
4  2 2

 3  3
 tan–1  tan  
 4  4

3   3 
   , 
4 2 2 

 graph of y = tan–1 (tan x) is as :

TM
RESONANCE 51
 3
 from the graph we can see that if < x< ,
2 2
then y = tan–1 (tan x) can be written as
y=x–

 3 
 y = tan–1  tan 
 4 

3
= –
4


 y=–
4

Solved Example # 5
Find the value of sin–1 (sin7)

Solution.
Let y = sin–1 (sin 7)

  
Note : sin–1 (sin 7)  7 as 7   , 
 2 2

5
 2 < 7 <
2
 graph of y = sin–1 (sin x) is as :

5
From the graph we can see that if 2  x  then
2
y = sin–1(sin x ) can be written as :
y = x – 2
 sin–1 (sin 7) = 7 – 2
Similarly if we have to find sin–1 (sin(–5)) then
3
 – 2 < – 5 < –
2
 from the graph of sin–1 (sin x), we can say that
sin–1 (sin(–5)) = 2 + (–5)
= 2 – 5
TM
RESONANCE 52
Self practice problems:
(8) Find the value of cos–1 (cos 13)
 5 
(9) Find sin–1 (sin ), cos–1(cos), tan–1 (tan ), cot–1(cot) for    , 3 
 2 
Ans. (8) 13 – 4
(9) sin-1 (sin) = 3 –  ; cos–1 (cos  ) = – 2 ;
–1
tan (tan ) =  – 3 ; cot–1 (cot ) =  – 2

Property - 2(C)
(i) sin1 (x) =  sin1 x, 1  x  1 (ii) tan1 (x) =  tan1 x, xR
(iii) cos 1 (x) =   cos 1 x, 1  x  1 (iv) cot1 (x) =   cot1 x, xR

The functions sin1 x, tan1 x and cosec 1 x are odd functions and rest are neither even nor odd.

Solved Example # 6
Find the value of cos–1 {sin( – 5)}
Solution.
Let y = cos–1 {sin(–5)}
= cos–1 (– sin 5)  cos–1 (– x) =  – cos–1x, |x|  1
=  – cos–1 (sin 5)
  
=  – cos–1 cos  5  ..........(i)
  2 

 
 – 2 <   5  < – 
 2 

 graph of cos–1 (cos x ) is as :

 from the graph we can see that if – 2 x  – 


then y = cos–1 (cosx) can be written as y = x + 2
       5 
 from the graph cos–1 cos  5  =   5  + 2 =   5
  2    2   2 
 from equation (i), we get
 5 
 y=–   5
 2 
3
 y=5– Ans.
2

TM
RESONANCE 53
Self practice problems:
Find the value of the following

  7  
(10) cos–1 (– cos 4) (11) tan–1 tan  
  8 

 1 
(12) tan–1 cot   
  4 

  1 
Answers. (10) 4– (11) (12)   
8 4 2
Property - 2(D)
1 1
(i) cosec 1 x = sin1 ;x  1, x  1 (ii) sec 1 x = cos 1 ;x  1, x  1
x x

 1 1
1  tan x ; x  0
(iii) cot x  
1
  tan 1 ; x  0
 x

Solved Example # 7
 1   2  
Find the value of tan cot  
  3 
Solution
   2 
Let y = tan cot 1  ........(i)
  3 
 cot–1 (–x) =  – cot–1x, x  R
 equation (i) can be written as
  2 
y = tan   cot 1 
  3 

 2
y = – tan  cot 1 
 3

1
 cot–1 x = tan–1 if x>0
x

3 3
 y = – tan  tan 1   y=–
 2 2

Self practice problems:


Find the value of the followings

  2    1 
(13) sec  cos 1   (14) cosec  sin1  

 3   3
   

3
Answers. (13) (14) – 3
2

TM
RESONANCE 54
Property - 2(E)
 
(i) sin1 x + cos 1 x = , 1  x  1 (ii) tan1 x + cot1 x = , xR
2 2

(iii) cosec 1 x + sec 1 x = , x  1
2

Solved Example # 8
1
Find the value of sin (2cos–1x + sin–1x) when x =
5
Solution.
Let y = sin [2cos–1x + sin–1x]

 sin–1x + cos–1x = , |x|  1
2
  
 y = sin 2 cos 1 x   cos 1 x 
 2 
 1 
= sin   cos x 
2 
1
= cos (cos–1x)  x=
5

 1
 y = cos  cos 1  ........(i)
 5
 cos (cos–1x) = x if x  [–1, 1]
1
  [–1, 1]
5

 1 1
 cos  cos 1  =
 5 5
 from equation (i), we get
1
 y= .
5

Self practice problems:

Solve the following equations

(15) 5 tan–1x + 3 cot–1x = 2


(16) 4 sin–1x =  – cos–1x
1
Answers. (15) x=1 (16) x=
2
Property - 2(F)
2
(i) sin (cos 1 x) = cos (sin1 x) = 1  x , 1  x  1

1
(ii) tan (cot1 x) = cot (tan1 x) = , x  R, x  0
x
x
(iii) cosec (sec 1 x) = sec (cosec 1 x) = , x > 1
x2  1

TM
RESONANCE 55
Solved Example # 9
 1 3 
Find the value of sin  tan .
 4
Solution.
 3
Let y = sin  tan 1  ..........(i)
 4

Note : To find y we use sin(sin–1 x) = x, – 1  x  1


For this we convert tan–1 x in sin–1 x

3 3  
Let  = tan–1  tan = and    0, 
4 4  2

3
 sin  =
5
3
 sin–1 (sin ) = sin–1   ..........(ii)
5
 
   0,   sin–1 (sin ) = 
 2

 equation (ii) can be written as :

3 3
  = sin–1     = tan–1  
5 4

3 3
 tan–1   = sin–1  
4 5
 from equation (i), we get
 3
 y = sin  sin 1 
 5

3
y=
5

Solved Example # 10
1 1 5 

Find the value of tan  cos
2 3 
Solution.
1 1 5 

Let y = tan  cos ..............(i)
2 3 

5   5
Let cos–1 =     0,  and cos  =
3  2  3
 equation (i) becomes

y = tan   ............(ii)
2

TM
RESONANCE 56
5
1
 1  cos  3 3 5 (3  5 ) 2
 tan2 = = = =
2 1  cos  5 3 5 4
1
3

 3 5 
tan =±   .........(iii)
2  2 
 

    
    0,     0, 
 2 2  4


 tan >0
2
 from equation (iii), we get

 3 5 
 
tan =  2 
2  
 from equation (ii), we get
3 5 
 y =  
 Ans.
 2 

Solved Example # 11
1
Find the value of cos (2cos–1x + sin–1x) when x =
5

Solution.
Let y = cos [2cos–1x + sin–1x]

 sin–1x + cos–1x = , |x|  1
2
  
 y = cos 2 cos 1 x   cos 1 x 
 2 

 1 
= cos   cos x 
2 

1
= – sin (cos–1x)  x=
5

 1
 y= – sin  cos 1  ........(i)
 5

 sin (cos–1x) = 1 x 2 , | x |  1

 1 1  1 24
 sin  cos  = 1 =
 5 25 5
 from equation (i), we get
24
y=–
5

TM
RESONANCE 57
1 1  
Aliter : Let cos 1 =  cos  = and    0, 
5 5  2
24
 sin =
5
 24 
 
 sin–1 (sin ) = sin–1  5  ..........(ii)
 
 
   0, 2   sin–1 (sin ) = 
 
 equation (ii) can be written as
 24   1
 = sin–1  
   = cos–1  
 5  5
 1  24 
 cos–1   = sin–1  

5  5 
Now equation (i) can be written as
  24 
1
y = – sin sin  
 ........(iii)
  5 

24   24  24
1
  [–1, 1]  sin sin   =

5   5  5
 from equation (iii), we get
24
y=–
5
Self practice problems:

Find the value of the followings :


 
1 41   1 16 
(17) tan  cos ec (18) sec  cot 
 4   63 

1 1  3    1 
(19) sin  cot   (20) tan 2 tan 1   
2  4    5  4

4 65 2 5 7
Answers : (17) (18) (19) (20)
5 16 5 17

3. Identities of Addition and Substraction:


A.
 2 2
(i) sin1 x + sin1 y = sin1 x 1  y  y 1  x  , x  0, y  0 & (x 2 + y2)  1
 
 2 2
=   sin1 x 1  y  y 1  x  , x  0, y  0 & x 2 + y2 > 1
 

Note that: x 2 + y2  1  0  sin1 x + sin1 y 
2

x 2 + y2  1   sin1 x + sin1 y  
2
TM
RESONANCE 58
 2
1  y 2  , x  0, y  0
(ii) cos 1 x + cos 1 y = cos 1 x y  1  x
 

xy
(iii) tan1 x + tan1 y = tan1 , x > 0, y > 0 & xy < 1
1  xy

xy
=  + tan1 , x > 0, y > 0 & xy > 1
1  xy


= , x > 0, y > 0 & xy = 1
2
 
Note that : xy < 1  0 < tan1 x + tan1 y < ;xy > 1  < tan1 x + tan1 y < 
2 2

B.
 2 2
(i) sin1 x  sin1 y = sin1  x 1  y  y 1  x  , x  0, y  0
 

 2
1  y 2  , x  0, y  0, x  y
(ii) cos 1 x  cos 1 y = cos 1  x y  1  x 

xy
(iii) tan1 x  tan1y = tan1 1  x y , x  0, y  0

Note: For x < 0 and y < 0 these identities can be used with the help of properties 2(C)
i.e. change x and y to  x and  y which are positive .

Solved Example # 12
3 15 84
Show that sin–1 + sin–1 =  – sin–1
5 17 85
Solution.
2 2
3 15 3  15  8226
 > 0, > 0 and   +   = >1
5 17 5
  17
  7225

3 15 3 225 15 9 
 1  1
 sin–1 + sin–1 =  – sin–1 5 289 17 25 
5 17 

 3 8 15 4 
=  – sin–1  .  . 
 5 17 17 5 

 84 
=  – sin–1  
 85 

Solved Example # 13
Evaluate:
12 4 63
cos–1 + sin–1 – tan–1
13 5 16
Solution.
12 4 63
Let z = cos–1 + sin–1 – tan–1
13 5 16
TM
RESONANCE 59
4  4
 sin–1 = – cos–1
5 2 5

12  1 4  63
 z = cos–1 +   cos  – tan–1 .
13  2 5  16

  1 4 12  63
z= –  cos  cos 1  – tan–1 .........(i)
2  5 13  16

4 12 4 12
 > 0, > 0 and <
5 13 5 13

4 12  4 12 16 144   63 
cos–1 – cos–1 = cos–1    1 1  –1  

5 13  5 13 25 169  = cos  65 

 equation (i) can be written as

  63   63 
z= – cos–1   – tan–1  
2  65   16 

 63   63 
z = sin–1   – tan–1   .........(ii)
 65   16 

 63   63 
 sin–1   = tan–1  
 65   16 
 from equation (ii), we get

 63   63 
 z = tan–1   – tan–1    z=0 Ans.
 16   16 

Solved Example # 14
5
Evaluate tan–1 9 + tan–1
4
Solution.

5 5
 9 > 0, > 0 and 9   > 1
4 4

 5 
5  9 
4 
 tan–1 9 + tan–1 =  + tan–1 
4  5
 1  9. 
 4
=  + tan–1 (– 1)

=– .
4

5 3
tan–1 9 + tan–1 = .
4 4

TM
RESONANCE 60
Self practice problems:

4 5 16
(21) Evaluate sin–1 + sin–1 + sin–1
5 13 65
(22) If tan–14 + tan–1 5 = cot–1 then find ‘’
3 16 1 7
(23) Prove that 2 cos–1 + cot–1 + cos–1 =
13 63 2 25
Solve the following equations
 2
(24) tan–1 (2x) + tan–1 (3x) = (25) sin–1x + sin–1 2x =
4 3

 19 1 1
Answers. (21) (22) =– (24) x= (25) x=
2 9 6 2

C.


 2 sin x
1
if | x |  1

2
(i) sin  2 x 1  x 2 
1 1
=    2 sin x if x 1
   2

   2 sin 1 x

 if x 1
2

 2 cos 1 x if 0  x 1
(ii) cos 1 (2 x 2  1) =  1
 2   2 cos x if  1 x0

 2 tan 1x if | x |  1
2x  1
(iii) tan 1
=    2 tan x if x  1
1 x2


   2 tan 1x  if x 1

 2 tan 1x if | x | 1
2x 
(iv) sin1 =    2 tan 1x if x 1
1  x2


   2 tan 1x  if x  1

1 x 2  2 tan 1x if x  0
(v) cos1 =  1
1x 2  2 tan x if x  0

(See the graphs on page 20)

Solved Example # 15
Define y = cos–1 (4x3 – 3x) in terms of cos–1 x and also draw its graph.

Solution.
Let y = cos–1 (4x3 – 3x)
Note  Domain : [–1, 1] and range : [0, ]
Let cos–1 x =    [0, ] and x = cos 
 y = cos–1 (4 cos3 – 3 cos  )
y = cos–1 (cos 3) ...........(i)
TM
RESONANCE 61
Fig.: Graph of cos–1 (cos x)
  [0, ]
 3 [0, 3]
 to define y = cos–1 (cos 3), we consider the graph of cos–1 (cos x)
in the interval [0, 3]. Now, from the above graph we can see that
(i) if 0  3     cos–1 (cos 3) = 3
 from equation (i), we get
y = 3 if  3

 y = 3 if 0
3
1
 y = 3 cos–1x if x1
2
(ii) if   3   2  cos–1 (cos 3) = 2– 3
 from equation (i), we get
y = 2 – 3 if   3   2
 2
 y = 2 – 3 if <
3 3
1 1
y = 2 – 3cos–1 x if – x<
2 2
(iii)   3   3  cos–1 (cos 3) = – 2 + 3
 from equation (i), we get
 y = – 2 + 3 if   3   3
2
 y = – 2 + 3 if    
3
1
 y = – 2 + 3 cos–1 x if – 1  x –
2
 from (i), (ii) & (iii), we get

 1 1
 3 cos x ;
2
 x 1
 1 1
y = cos–1 (4x3 – 3x) =  2  3 cos 1 x ;   x 
 2 2
 2  3 cos 1 x ;  1  x   1
 2

Graph :
For y = cos–1 (4x3 – 3x)
domain : [–1, 1]
range : [0, ]
TM
RESONANCE 62
1
(i) if  x  1 , y = 3 cos–1x.
2
dy 3
 = = – 3(1 – x2)–1/2 ...........(i)
dx 1 x2
dy 1 
 <0 if x  , 1
dx 2 

1 
 decreasing if x   , 1
2 
again if we differentiate equation (i) w.r.t. ‘x’, we get
d2 y 3x
=–
dx 2 (1  x 2 )3 / 2

d2 y 1  1 
 <0 if x   , 1  concavity downwards if x   , 1
dx 2 2  2 
1 1
(ii) if –  x < , y = 2 – 3cos–1 x.
2 2
dy 3 dy 1 1
 =  >0 if x   , 
dx 1 x 2 dx  2 2

 1 1 d2 y 3x
 increasing if x   ,  and =
 2 2 dx 2
(1  x 2 )3 / 2

 1  d2 y
(a) if x   , 0  then <0
 2  dx 2

 1 
 concavity downwards if x   , 0 
 2 

 1 d2 y
(b) if x   0,  then >0
 2 dx 2

 concavity upwards if x   0, 1 
 2

1 dy d2 y
(iii) Similarly if – 1  x < – then < 0 and > 0.
2 dx dx 2
 the graph of y = cos–1 (4x3 – 3x) is as

TM
RESONANCE 63
Self practice problems:
(26) Define y = sin–1 (3x – 4x3) in terms of sin–1x and also draw its graph.
 3x  x 3 
(27) Define y = tan–1   in terms of tan–1 x and also draw its graph.
 1  3x 2 
 
Answers

 1 1 1
 3 sin x ; 
2
x
2
 1
1
(26) y = sin–1 (3x – 4x3) =   3 sin x ;  x 1
 2
    3 sin 1 x ; 1
1 x  
 2

 graph of y = sin–1 (3x – 4x3)

 1 1 1
 3 tan x ;  x
3  3 3
 3x  x 
   1 1
(27) y = tan–1  1  3 x 2  =   3 tan x ;   x  
   3
 1 1
    3 tan x ; x
 3

 3x  x 3 
 
Fig.: Graph of y = tan–1  2 
 1  3x 
D.
 x  y  z  x yz 
If tan1 x + tan1 y + tan1 z = tan1  1  xy  yz  zx  if, x > 0, y > 0, z > 0 & (xy + yz + zx) < 1
 

NOTE:
(i) If tan1 x + tan1 y + tan1 z =  then x + y + z = xyz

(ii) If tan1 x + tan1 y + tan1 z = then xy + yz + zx = 1
2
1 1 
(iii) tan1 1 + tan1 2 + tan1 3 =  (iv) tan1 1 + tan1 + tan1 =
2 3 2
TM
RESONANCE 64
Inverse Trigonometric Functions
Some Useful Graphs
1.
  
(i) y = sin 1 x, x  1, y   ,  (ii) y = cos 1 x, x  1, y  [0, ]
 2 2
y y

 
2 2

O

1 1 x 1 O 1 x



2

  
(iii) y = tan 1 x, x  R, y    2, 2  , (iv) y = cot 1 x, x  R, y  (0, )
 
y
y

2

 
2


O x
 O x




2

        
(v) y = sec 1 x, x  1, y  0,  U  ,  (vi) y = cosec 1 x, x  1, y   2 , 0  U  0 , 2 
 2 2     

y y

 
2

 2


1 O 1 x  1 O 1 x



2

TM
RESONANCE 65
Part - 2(A)
(i) y = sin (sin 1 x) = cos (cos 1 x) = x, x  [ 1, 1], y  [ 1, 1]; y is aperiodic

y
1
x
=
y

)45º
1 O +1 x

1

(ii) y = tan (tan -1 x) = cot (cot -1 x) = x, x  R, y  R; y is aperiodic

y




x
 =
 y

) 45º

O x







(iii) y = cosec (cosec 1 x) = sec (sec 1 x) = x, x  1, y  1 ; y is aperiodic

y

=x

 y

1 O 1 x

1



x
=
 y

TM
RESONANCE 66
Part -2(B)
y = sin 1 (sin x), x  R, y   ,  , is periodic with period 2 
 
(i)
 2 2
y


y
x


= 2 y

2
=



(
2

x
+ 

y
x

=
=

 3
x)

y

y

2 ) 45º 2

 2 3  O   2 x

2 2



2

(ii) y = cos 1 (cos x), x  R, y  [0, ], is periodic with period 2 


y


y=

y=

x

y=
2

2
x+

2
x


=

x
y

x
 2   O   2

2 2

     
(iii) y = tan 1 (tan x), x  R  (2 n  1) , n     y    ,  is periodic with period 
 2   2 2
y

2




x+

x
x+

y=
y=

y=
y=

2
 
 2 
3  
 O  3 x
2 2 2 2

x
=
y



2
      
(iv) y = sec 1 (sec x), y is periodic with period 2; x  R  ( 2n  1) , n   , y  0, 2  U  2 , 
 2     
y


y=

y=


x

y=
2

2
x+

2
x


=

x
y

x
 2 3   O   3 2
 
2 2 2 2

TM
RESONANCE 67
   
(v) y = cot–1 (cot x), y is periodic with period ; x  R – {n, n  }, y   0,    ,  
 2 2 

  
(vi) y = cosec–1 (cosec x), y is periodic with period 2; x  R – {n, n  }, y   ,  – {0}
 2 2

TM
RESONANCE 68
Part - 3(C)

(i) graph of y = sin1  2 x 1  x 2 


 

(ii) graph of y = cos1 (2 x2  1)

Note : In this graph it is advisable not to check its derivability just by the inspection of the graph
because it is difficult to judge from the graph that at x = 0 there is a shapr corner or not.

2x
(iii) graph of y = tan1
1 x 2

2x
(iv) graph of y = sin1
1  x2

1 x2
1
(v) graph of y = cos
1x 2

TM
RESONANCE 69
Part : (A) Only one correct option

1. If cos –1  + cos–1 + cos –1 v = 3 then  + v + v is equal to


(A) – 3 (B) 0 (C) 3 (D) – 1

2. Range of f(x) = sin–1 x + tan–1 x + sec –1 x is

  3    3    3 
(A)  ,  (B)  ,  (C)  ,  (D) none of these
4 4  4 4  4 4 

   3 
3. The solution of the equation sin1  tan   sin1    = 0 is
 4  x 6
 
(A) x = 2 (B) x =  4 (C) x = 4 (D) none of these

 
4. The value of sin–1 [cos{cos –1 (cosx) + sin–1 (sin x)}], where x   ,   is
2 

   
(A) (B) (C) – (D) –
2 4 4 2

5. The set of values of k for which x2  kx + sin–1 (sin 4) > 0 for all real x is
(A) { 0 } (B) (2, 2) (C) R (D) none of these

6. sin–1 (cos(sin–1x)) + cos –1 (sin (cos–1x)) is equal to


  3
(A) 0 (B) (C) (D)
4 2 4

 1 x2  x
2 2
7. cos 1  x  1  x . 1   = cos–1 2  cos–1x holds for
 2 4

(A) | x |  1 (B) x  R (C) 0  x  1 (D) 1  x  0

8. tan–1 a + tan–1 b, where a > 0, b > 0, ab > 1, is equal to

 ab   ab 
(A) tan–1   (B) tan–1  1  ab  – 
 1  ab   

 ab   ab 
(C)  + tan–1   (D)  – tan–1  
 1  ab   1  ab 

9. The set of values of ‘x’ for which the formula 2 sin–1x = sin–1 (2x 1 x 2 ) is true, is

 3 3  1 1 
(A) (– 1, 0) (B) [0, 1] (C)  ,  (D)  , 
 2 2   2 2

10. The set of values of ‘a’ for which x2 + ax + sin–1 (x 2 – 4x + 5) + cos –1 (x 2 – 4x + 5) = 0 has at least one
solution is

(A) (– , – 2 ]  [ 2 , ) (B) (– , – 2 )  ( 2 , )
(C) R (D) none of these
TM
RESONANCE 70
3
11. All possible values of p and q for which cos–1 p + cos –1 1 p + cos –1 1 q = holds, is
4

1 1 1
(A) p = 1, q = (B) q > 1, p = (C) 0  p  1, q = (D) none of these
2 2 2

12. If [cot–1x] + [cos–1x] = 0, where [.] denotes the greatest integer function, then complete set of values of
‘x’ is
(A) (cos1, 1] (B) (cot 1, cos 1) (C) (cot1, 1] (D) none of these

13. The complete solution set of the inequality [cot–1x]2 – 6 [cot–1 x] + 9  0, where [.] denotes greatest
integer function, is
(A) (– , cot 3] (B) [cot 3, cot 2] (C) [cot 3, ) (D) none of these

 1 1   1 1 
14. tan   cos x  + tan   cos x  , x  0 is equal to
4 2  4 2 

2 x
(A) x (B) 2x (C) (D)
x 2

1 1  3 sin 2  
15. If sin   = , then tan  is equal to
2  5  4 cos 2  4
(A) 1/3 (B) 3 (C) 1 (D)  1

 u
16. If u = cot1 tan   tan1 tan  , then tan    is equal to
4 2

(A) tan  (B) cot  (C) tan  (D) cot 

 1  sin x  1  sin x  
17. The value of cot1  , < x < , is:
 1  sin x  1  sin x  2

x  x x x
(A)  (B) + (C) (D) 2 
2 2 2 2 2

18. The number of solution(s) of the equation, sin1x + cos 1 (1  x) = sin1 ( x), is/are
(A) 0 (B) 1 (C) 2 (D) more than 2

 1   1   2 
19. The number of solutions of the equation tan 1   + tan 1   = tan 1  2  is
 2x  1   4x  1  x 
(A) 0 (B) 1 (C) 2 (D) 3

1 1 1 1
20. If tan1 + tan1 + tan1 + .......+ tan1 = tan1 , then  is equal to
1 2 1  2.3 1  3 .4 1  n(n  1)

n n n 1 1
(A) n  2 (B) n  1 (C) (D)
n n

TM
RESONANCE 71
n 
21. If cot1 > , n  N, then the maximum value of ‘ n ‘ is:
 6
(A) 1 (B) 5 (C) 9 (D) none of these

22. The number of real solutions of (x, y) where, y = sin x, y = cos –1 (cos x), 2  x  2is:
(A) 2 (B) 1 (C) 3 (D) 4

1 1
23. The value of cos  cos 1  is equal to
2 8
(A) 3/4 (B) – 3/4 (C) 1/16 (D) 1/4

Part : (B) May have more than one options correct

24.  and  are three angles given by

1  1 1
 = 2tan–1 ( 2  1),  = 3sin–1 + sin–1    and  = cos–1 . Then
2  2 3
(A) > (B) > (C) < (D) >

25. cos1x = tan1x then


 5  1  5  1
(A) x2 =  
 (B) x 2 =  

 2   2 
 5  1  5  1
(C) sin (cos1x) =   (D) tan (cos 1x) =  

 2 
   2 

26. For the equation 2x = tan (2 tan 1 a) + 2 tan (tan 1 a + tan 1 a3), which of the following is invalid?
(A) a2 x + 2a = x (B) a2 + 2 ax + 1 = 0 (C) a  0 (D) a  1, 1

 4n
27. The sum  tan 1
n  2n 2  2
4 is equal to:
n 1

(A) tan 1 2 + tan 1 3 (B) 4 tan 1 1 (C) /2 


(D) sec 1  2 
28. If the numerical value of tan (cos–1 (4/5) + tan–1 (2/3)) is a/b then
(A) a + b = 23 (B) a – b = 11 (C) 3b = a + 1 (D) 2a = 3b

29. If  satisfies the inequation x2 – x – 2 > 0, then a value exists for


(A) sin–1  (B) cos –1  (C) sec –1  (D) cosec –1 

x 1 
30. If f (x) = cos 1x + cos1   3  3x 2  then:
2 2 

2  2 2 
(A) f   = (B) f   = 2 cos 1 –
3 3 3 3 3

 1   1 1 
(C) f   = (D) f   = 2 cos 1 – m
3 3 3 3 3

TM
RESONANCE 72
1. Find the value of the following :

 1  1    1  1 
(i) sin   sin    (ii) tan cos 1  tan 1   
3  2   2  3 

   3 
1
(iii) sin–1 cos sin   
   2 
  


2. Solve the equation : cot–1 x + tan–1 3 =
2

 x 1  x 1 
3. Solve the equation : tan–1   + tan–1   =
x 2  x  2 4

4. Solve the following equations :

 1 x  1
(i) tan–1   = tan–1x , (x > 0)
 1  x  2

 1  1 1
(ii) 3tan–1   – tan–1   = tan–1  

2 3  x 3

 1 1  2 x  1 1 
2 
5. Find the value of tan  sin    cos  1 y  , if x > y > 1.
 2  1 x2  2  1 y2 
 

1 1 4 
6. If x = sin (2 tan–12) and y = sin  tan  then find the relation between x and y .
 2 3

7. If arc sinx + arc siny + arc sinz =  then prove that:(x, y, z > 0)

(i) x 1  x 2  y 1  y 2  z 1  z 2  2xyz
(ii) x 4 + y4 + z4 + 4 x2y2z2 = 2 (x2 y2 + y2 z2 + z2x 2)

8. Solve the following equations :


x x
(i) sec 1  sec 1 = sec1b  sec 1a a  1; b  1, a  b .
a b

x x 1 1
(ii) sin 1  sin 1  sin 1
1 x x 1 1 x

5 2
(iii) Solve for x, if (tan–1x)2 + (cot–1 x)2 =
8

TM
RESONANCE 73
 1 x   1 x2 
9. If  = 2 tan–1   &  = sin–1   

 1 x   1  x 2  for 0 < x < 1, then prove that  + =. What the value of  +
 
 will be if x > 1 ?
10. If X = cosec tan1 cos cot1 sec sin1 a & Y = sec cot1 sin tan1 cosec cos1 a; where 0  a  1. Find the
relation between X & Y. Express them in terms of 'a'.

11. Solve the following inequalities:


(i) cos 1 x > cos 1 x2 (ii) sin–1 x > cos–1 x
(iii) tan–1 x > cot–1 x. (iv) sin–1 (sin 5) > x2 – 4x.
(v) tan2 (arc sin x) > 1 (vi) arccot2 x  5 arccot x + 6 > 0
(vii) tan 1 2 x  2 tan 1 x

12. Find the sum of each of the following series :

31 139 319  2 5 
(i) cot–1 + cos –1 + cot–1 + ... + cot–1  3n  .
12 12 12  12 

1 2 2n 1
(ii) tan1 + tan1 + ..... + tan1 + ..................... 
3 9 1  2 2n1

1
13. Prove that the equation, (sin1x)3 + (cos1x)3 = 3 has no roots for < .
32

14. (i) Find all positive integral solutions of the equation, tan1 x + cot1 y = tan1 3.
(ii) If 'k' be a positive integer, then show that the equation:
tan1 x + tan1 y = tan1 k has no nonzero integral solution.

Exercise # 1 1  xy
5. xy 6. x = 4y2
1. C 2. C 3. C 4. D 5. D 6. C 7. C
8. (i) x = ab (ii) x  0 (iii) x = – 1
8. C 9. D 10. D 11. C 12. C 13. A 14. C

15. B 16. A 17. B 18. B 19. B 20. A 21. B 9.  10. X=Y= 3a 2

22. C 23. A 24. BC 25. AC 26. BC 27. AD  2 


11. (i) [ 1, 0) (ii)  , 1 (iii) x > 1
 2 
28. ABC 29. CD 30. AD 
(iv) 2 – 9  2  x  2  9  2
Exercise # 2 
 2  2 
(v)  ,1 U   1, 
 2 
   2 
1 
1. (i) 1 (ii) (iii) 2. x = 3. (vi) (  cot 3) U (cot 2, ) (vii) x  0
3 6

1 1 18n  13 
3. ± 4. (i) x = (ii) x = 2 12. (i) cot–1 (ii)
2 3 12n 4

14. Two solutions (1, 2) (2, 7)

TM
RESONANCE 74
Continuity of Functions

1. A function f(x) is said to be continuous at x = c,

if Limit
x c f(x) = f(c). Symbolically f is continuous at

x = c if Limit
h0 f(c  h) = Limit
h0 f(c+h) = f(c).

i.e. LHL at x = c = RHL at x = c equals value of ‘f’ at x = c.


Calculus required
If a function f (x) is continuous at x = c, the graph of f (x) at the
continuity, and corresponding point { c , f (c) } will not be broken. But if f (x) is
discontinuous at x = c , the graph will be broken at the
continuity was corresponding point.
supposed to require
theinfinitely little;
But nobody could
discover what the
infinitelylittlemight
be. (i) (ii)
. . Ral ei gh
NC

A quantity which is
increased or decreased
by an infinitely small
quantity is neither (iii) (iv)
((i), (ii) and (iii) are discontinuous at x = c)
increased nor ((iv) is continuous at x = c)
decreased.
D. Mac Hal e
A function f can be discontinuous due to any of the following
three reasons:
Death should not be (i) Limit f(x) does not exist i.e. Limit f(x)  Limit
x c x c x c  f (x)

fearead, its just the [figure (i)]

continuity of life (ii) f (x) is not defined at x = c


[figure (ii)]
Met t r i e L.

(iii) Limit f(x)  f (c)


x c

[figure (iii)]
Geometrically, the graph of the function will exhibit a break
at x= c.

TM
RESONANCE 1
Solved Example # 1
Find whether f(x) is continuous or not at x = 1
x
f(x) = sin ; x<1
2
= [x] x  1

Solution

 x
sin  x 1
f(x) =  2 for continuity at x = 1, we determine, f(1), xlim
1–
f(x) and xlim
1
f(x).
 [ x ]  x 1
Now, f(1) = [1] = 1

lim f(x) = lim– sin x = sin  = 1


x 1– x 1 2 2

and lim f(x) = lim [x] = 1


x 1 x 1

so f(1) = xlim
1–
f(x) = xlim
1
f(x)
 f(x) is continuous at x = 1

Self practice problems :


1. If possible find value of  for which f(x) is continuous at x =
2
1  sin x 
f(x) = , x
1  cos 2x 2

=  x
2

2x   
= x
4  2x    2 2
Answer discontinuous

2. Find the values of a and b such that the function



f(x) = x + a 2 sin x ; 0x
4
 
= 2x cot x + b x
4 2
  
= a cos 2x – b sin x x is continuous at and
2 4 2
 
Answer a= ,b=
6 12

3. If f(x) = (1  ax ) x x<0
= b x=0
1
(x  c)3  1
= x>0
x
The find the values of a, b, c, f(x) is continuous at x = 0
1
Answer a = –In 3, b = ,c=1
3
TM
RESONANCE 2
2. Types of Discontinuity :
(a) Removable Discontinuity

In case Limit
x c f(x) exists but is not equal to f(c) then the function is said to have a removable

discontinuity. In this case we can redefine the function such that Limit
x c f(x) = f(c) & make it
continuous at x = c.

Removable type of discontinuity can be further classified as :


(i) Missing Point Discontinuity :
Where Limit f(x) exists finitely but f(a) is not defined.
x a

(1  x ) (9  x 2 )
e.g. f(x) = has a missing point discontinuity at x = 1. [figure (ii) Page 1]
(1  x )

(ii) Isolated Point Discontinuity:

Where Limit
x a f(x) exists & f(a) also exists but;

2
Limit f(x)  f(a). e.g. f(x) = x  16 , x  4 & f (4) = 9 has a break at x = 4. [figure (iii) Page 1]
x a
x4
(b) Irremovable Discontinuity

In case Limit
x c f(x) does not exist then it is not possible to make the function continuous by

redefining it. However if both the limits (i.e. L.H. L. & R.H.L.) are finite, then discontinuity is
said to be of first kind otherwise it is nonremovable discontinuity of second kind.
[figure (i) Page 1]

Irremovable type of discontinuity can be further classified as:


(i) Finite discontinuity e.g. f(x) = x  [x] at all integral x.

1 1
(ii) Infinite discontinuity e.g. f(x) = or g(x) = at x = 4.
x4 ( x  4) 2

1
(iii) Oscillatory discontinuity e.g. f(x) = sin at x = 0.
x
In all these cases the value of f (a) of the function at x = a (point of discontinuity) may or may not exist

but Limit
x a does not exist.

(c) Discontinuity of st kind


If L.H.L. and R.H.L both exist finitely then discontinuity is said to be of st kind

(d) Discontinuity of  nd kind


If either L.H.L. or R.H.L does not exist then discontinuity is said to be of nd kind

(e) Point functions defined at single point only are to be treated as discontinuous.
eg. f(x) = 1  x  x  1 is not continuous at x = 1.

Solved Example # 2

If f(x) = x x<1
= x2 x>1
then check if f(x) is continuous at x = 1 or not if not, then comment on the type of discontinuity.
Solution
x  x 1
f(x) =  2
x  x 1

TM
RESONANCE 3
lim f(x) = lim– x = 1
x 1– x 1

and lim f(x) = lim x 2 = 1


x 1 x 1

 lim f(x) = lim f(x) = finite


x 1 x 1
and f(1) is not defined.
So f(x) is discontinuous at x = 1 and this discontinuity is removable missing point discontinuity

Self practice problems :

4. f(x) = x, x<1
= x2 x>1
=2 x = 1 which type of discontinuity is there
Answer isolated point discontinuity

5. f(x) = x ; x<1
= 2x 1 x Find which type of discontinuity it is.
Answer non removable of Ist kind

3. Jump of discontinuity
ln case of non removable discontinuity of the first kind the non negative difference between the value
of the RHL at x = c & LHL at x = c is called, the Jump of discontinuity.
Jump of discontinuity =  RHL – LHL 

NOTE : A function having a finite number of jumps in a given interval is called a Piece Wise Continuous
or Sectionally Continuous function in this interval. For e.g. {x} , [x]

Solved Example # 3

f(x) = cos–1 {cot x} x<
2

 [x] – 1 x Find jump of discontinuity.
2

Ans.  –1
2

 
cos cot x if
1
x
Sol. f(x) =  2

  [x]  1 if x
 2

lim  lim 
x
 f(x) = x
 cos –1 {cot x}
2 2
= cos –1 {0+}

= cot–1 0 =
2
lim lim
x
 f(x) = x
 [x] – 1 =  – 1
2 2

TM
RESONANCE 4

 jump of discontinuity =–1–
2

= –1
2

4. Continuity in an Interval :
(a) A function f is said to be continuous in (a, b) if f is continuous at each & every point (a, b).
(b) A function f is said to be continuous in a closed interval [ a, b ] if:
(i) f is continuous in the open interval (a, b) &

(ii) f is right continuous at ‘a’ i.e. Limit


x a 
f(x) = f(a) = a finite quantity..

(iii) f is left continuous at ‘b’ i.e. Limit


x b 
f(x) = f(b) = a finite quantity..

(c) All Polynomials, Trigonometrical functions, Exponential and Logarithmic functions are continuous
in their domains.
(d) Continuity of {f(x)} and [f(x)] should be checked at all points where f(x) becomes integer.
(e) Continuity of sgn (f(x)) should be checked at the points where f(x) = 0 (if f(x) is constanly equal
to 0 when x  a then x = a is not a point of discontinuity)
(f) Continuity of a function should be checked at the points where definition of a function changes.

Solved Example # 5

If f(x) = [sin x] 0  x 1


 5  2
= Sgn  x   x   1  x  2 , where { . } represents fractional function
 4  3
then comment on the continuity of function in the interval [0, 2].

Solution
(i) Continuity should be checked at the end-points of intervals of each definition i.e. x = 0, 1, 2
(ii) For [sin x], continuity should be checked at all values of x at which sin x  
1
i.e. x = 0,
2
 5  2 5
(iii) For sgn  x   x   , continuity should be checked when x – = 0 (as sgn (x) is
 4  3 4

5 2
discontinuous at x = 0) i.e. x= and when x – 
4 3
5
i.e. x= (as {x} is discontinuous when x )
3

1 5 5
 overall discontinuity should be checked at x = 0, , 1, , and 2
2 4 3
check the discontinuity your self.
1 5 5
Answer discontinuous at x = ,1 ,
2 4 3

TM
RESONANCE 5
Self practice problems :

 1
6. If f(x) = sgn  x    [In x] 1 < x  3
 2  
= {x 2} 3 < x  3.5
Find the point where the continuity of f(x) should be checked.
3 5
Ans. { 1, , , e, 3 , 10 , 11 , 12 , 3.5 }
2 2

5. If f & g are two functions which are continuous at x = c then the functions defined by:
F 1(x) = f(x)  g(x) ; F2(x) = K f(x), K any real number ; F3(x) = f(x).g(x) are also continuous at x = c.
f (x)
Further, if g (c) is not zero, then F4(x) = g( x ) is also continuous at x = c.

Note :
(i) If f(x) is continuous & g(x) is discontinuous at x = a then the product function
 (x) = f(x). g(x) may be continuous but sum or difference function  (x) = f(x)  g(x) will
necessarily be discontinuous at x = a. e.g.
sin  x  0
x
f (x) = x & g(x) = 
 0 x0
(ii) If f (x) and g(x) both are discontinuous at x = a then the product function(x) = f(x). g(x) is not
necessarily be discontinuous at x = a. e.g.
 1 x0
f (x) = g(x) = 
 1 x  0

Solved Example # 6

If f(x) = [sin(x–1)] – {sin(x–1)}



Comment on continuity of f(x) at x = 1
2

Solution
f(x) = [sin (x – 1)] – {sin (x – 1)}
Let g(x) = [ sin (x – 1)] + {sin (x – 1} = sin (x – 1)

which is continuous at x = +1
2

as [sin (x – 1)] and { sin (x – 1)} both are discontinuous at x = +1
2

 At most one of f(x) or g(x) can be continuous at x = +1
2

As g(x) is continuous at x = + 1, there fore, f(x) must be discontinuous
2
lim  
Alternatively, check the continuity of f(x) by evaluling x
 f(x) and f   1
2 2 

TM
RESONANCE 6
6. Continuity of Composite Function :
If f is continuous at x = c & g is continuous at x = f(c) then the composite g[f(x)] is continuous at
xsinx x sin x
x = c. eg. f(x) = 2 & g(x) = x are continuous at x = 0, hence the composite (gof) (x) = 2
x 2 x 2
will also be continuous at x = 0.

Solved Example # 7
x 1 1
If f(x) = and g(x) = , then discuss the continuity of f(x), g(x) and fog (x).
x 1 x2
x 1
Sol. f(x) =
x 1
f(x) is a rational function it must be continuous in its domain
and f is not defined at x = 1
 f is discontinuous at x = 1
1
g(x) =
x2
g(x) is also a rational function. It must be continuous in its domain and fog is not defined at x = 2
 g is discontinuous at x = 2
Now fog (x) will be discontinuous at
(i) x=2 (point of discontinuity of g(x))
(ii) g(x) = 1 (when g(x) = point of discontinuity of f(x))
if g(x) = 1
1
 =1
x2
 x=3
 discontinuity of fog(x) should be checked at x = 2 and x = 3
at x = 2
1
1
x2
fog (x) = 1
1
x2
fog (2) is not defined
1
1
lim fog (x) = lim x2 1 x  2
x 2 x 2 1 = xlim
2 1  x  2
=1
1
x2
 fog (x) is discontinuous at x = 2 and it is removable discontinuity at x = 3
fog (3) = not defined
1
1
lim fog (x) = lim x2
x 3  x 3 1 =
1
x2

1
1
lim fog (x) = lim x2
x 3  x 3 1 = –
1
x2
 fog (x) i s discontinuous at x = 3 and it is non removable discontinuity of nd kind.

TM
RESONANCE 7
Self practice problems :

 1
1  x 3 , x  0 ( x  1) 3 , x0
f ( x)   2 g( x )  
7. 1
 x  1 , x  0  2
( x  1) , x0
Then defined fog (x) and comment the continuity of gof(x) at x = 1
Ans. [fog(x) = x, x  R gof(x) is discontinous at x = 0, 1]

7. Intermediate Value Theorem :


A function f which is continuous in a , b possesses the following properties:
(i) If f(a) & f(b) possess opposite signs, then there exists at least one solution of the equation
f(x) = 0 in the open interval (a, b).

(ii) If K is any real number between f(a) & f(b), then there exists at least one solution of the
equation f(x) = K in the open inetrval (a, b).

Solved Example # 7

Given that a > b > c > d then prove that the equation (x – a) (x – c) + 2(x – b) (x – d) = 0 will have real
and distinct roots.
Solution
(x – a) (x – c) + 2 (x – b) (x – d) = 0
f(x) = (x – a) (x – c) + 2 (x – b) (x – d)
f(a) = (a – a) (a – c) + 2 (a – b) (a – d) = + ve
f(b) = (b – a) (b – c) + 0 = –ve
f(c) = 0 + 2 (c – b) (c – d) = –ve
f(d) = (d – a) (d – c) + 0 = +ve

hence (x – a) (x – c) + 2(x – b) (x – d) = 0
have real and distinct root

Self practice problems :

8. f(x) = xex – 2 then show that f(x) = 0 has exactly one root in the interval (0, 1).

Solved Example # 8
1 
Let f(x) = Lim 2, then find f   and also comment on the continuity at x = 0
n  1  n sin x 4
Ans. [Discontinous, removable discontinuity of Isolated type]

1
Sol. Let f(x) = nlim
 1  n sin 2 x

 1
f   = nlim
4 
1  n . sin2

4

TM
RESONANCE 8
1
= nlim

=0
 1
1 n  
2
Now
1
f(0) = nlim
 n . sin 2 (0)  1

1
= =1
1 0

 1 
lim f(x) = lim  lim 
x 0 x 0 n 1  n sin 2 x 
 

 1 
=  
1   
{here sin2x is very small quantity but not zero and very small quantity when multiplied
with  becomes }
 f(x) is not continuous at x = 0

Self practice problems :

9. f(x) = nLim

(1 + x)n
1
Comment on the continuity of f(x) at 0 and explain Lim
x 0
(1  x ) x  e
Ans. Discontinous (non-removable)

Solved Example # 9

f(x) = maximum (sin t, 0  t  x), 0  x  2discuss the continuity of this function at x=
2

Solution
f(x) = maximum (sin t, 0  t  x), 0  x  2

 
if x  0,  , sin t is increasing function
 2
Hence if t  [0, x], sin t will attain its maximum value at t = x.
 
 f(x) = sin x if x  0, 
 2

 
if x   , 2 and t  [0, x]
 2 


then sin t will attain its maximum value when t =
2

  
 f(x) = sin = 1 if x   , 2
2 2 

TM
RESONANCE 9
  
sin x , if x  0, 
  2
 f(x) = 
  
 1 , if x   , 2
  2 


Now f   =1
2

lim lim
x
 f(x) = x
 sin x = 1
2 2

lim lim
x
 f(x) = x
 1=1
2 2
as f(x) = L.H.S. = R.H.S.

 f(x) is continuous at x =
2

TM
RESONANCE 10
Part : (A) Only one correct option

(a 2  ax  x 2 )  (a 2  ax  x 2 )
1. The value of f (0), so that the function, f (x) = (a > 0) becomes
(a  x )  (a  x )

continuous for all x, is given by :


(A) a a (B) a (C)  a (D)  a a

sin (1 / x) , x  0
2. The value of R which makes f (x) =  continuous at x = 0 is:
R , x0
(A) 8 (B) 1 (C)  1 (D) None of these

cos(sin x )  cos x
3. A function f(x) is defined as below f(x) = , x  0 and f(0) = a
x2
f(x) is continuous at x = 0 if a equals
(A) 0 (B) 4 (C) 5 (D) 6

1   
4. Let f(x) = sin x   2 x , x  . If f(x) is continuous at x = then f   is
2 2 2
(A) e (B) 1 (C) 0 (D) none of these

 (1  px )  (1  px )
 x , 1  x  0
5. f (x) =  is continuous in the interval [  1, 1 ], then ' p' is equal to:
 2x  1 , 0  x 1
 x2

(A)  1 (B)  1/2 (C) 1/2 (D) 1

 1
6. Let f(x) =  x  2  [ x] when – 2  x  2. where [ . ] represents greatest integer function. Then
 
(A) f(x) is continuous at x = 2 (B) f(x) is continuous at x = 1
(C) f(x) is continuous at x = – 1 (D) f(x) is discontinuous at x = 0

7. The set of all points for which


| x 3| 1
f(x) = + where [.] represents greatest integer function is continuous is
| x2| [1  x]
(A) R (B) R – [– 1, 0]
(C) R –  {2}  [– 1, 0] (D) R – {(–1, 0)  n, n }

 ( 2x  1) 
8. The function f (x) = [ x ] cos   , ( [.] denotes the greatest integer function) is dicontinuous at:
 2 
(A) all x (B) x = n/2, n {1}
(C) no x (D) x which is not an integer

9. Let [x] denote the integral part of x  R and g(x) = x  [x]. Let f(x) be any continuous function with
f(0) = f(1) then the function h(x) = f(g(x)) :
(A) has finitely many discontinuities (B) is continuous on R
(C) is discontinuous at some x = c (D) is a constant function.

TM
RESONANCE 11

log( 4 x 3 ) x 2  2x  5  if 34  x  1 & x  1
10. The function f(x) is defined by f(x) = 
 4 if x  1
(A) is continuous at x = 1
(B) is discontinuous at x = 1 since f(1+) does not exist though f(1) exists
(C) is discontinuous at x = 1 since f(1) does not exist though f(1+) exists
(D) is discontinuous since neither f(1) nor f(1+) exists.

1  sinx n (sin x)  


11. Let f(x) = . x . The value of f   so that the function is continuous
2
(   2x )2 
n 1   2  4  x  4 x 2  2

at x = /2 is:
(A) 1/16 (B) 1/32 (C)  1/64 (D) 1/128

x 2 if x is irrational
12. Let f(x) =  then:
1 if x is rational
(A) f(x) is discontinuous for all x
(B) discontinuous for all x except at x = 0
(C) discontinuous for all x except at x = 1 or  1
(D) none of these

13. Let f(x) = [x2 ] – [x]2 , where [ . ] denotes the greatest integer function. Then
(A) f(x) is discontinuous for all integeral values of x
(B) f(x) is discontinuous only at x = 0, 1
(C) f(x) is continuous only at x = 1
(D) none of these

14. Let f(x) be a continuous function defined for 1  x  3. If f(x) takes rational values for all x and f(2) = 10
then the value of f(1.5) is
(A) 7.5 (B) 10 (C) 8 (D) none of these

15. Let f(x) = Sgn(x) and g(x) = x (x2 – 5x + 6). The function f(g(x)) is discontinuous at
(A) infinitely many points (B) exactly one point
(C) exactly three points (D) no point

 2  1
16. The function f(x) =  x  2   , x  0, is [ . ] represents the greatest integer less than or equal to x
 x  
(A) continuous at x = 1 (B) continuous at x = – 1
(C) discontinuous at infinitely many points (D) continuous at x = – 1

 (1  sin x ) t  1
17. lim
The function f defined by f(x) = t   .   is
 (1  sin x ) t  1
(A) everywhere continuous (B) discontinuous at all integer values of x
(C) continuous at x = 0 (D) none of these
a 2|x|  { x }  1
18. If [x] and {x} represent integral and fractional parts of a real number x, and f(x) = , x  0,
2 [ x]  { x }
f(0) = loge a, where a > 0, a  1, then
(A) f(x) is continuous at x = 0 (B) f(x) has a removable discontinuity at x = 0

(C) xlim
0
f(x) does not exist (D) none of these

Part : (B) May have more than one options correct

19. If f(x) = x and g(x) = x – 1, then


(A) fog is continuous on [0, ) (B) gof is continuous on [0, )
(C) fog is continuous on [1, ) (D) none of these
TM
RESONANCE 12
 m 1
 x sin , x  0
20. The function f(x) =  x is continuous at x = 0 if
0 , x0

(A) m  0 (B) m > 0 (C) m < 1 (D) m  1

1
21. Let f(x) = [sin x ] ( [ . ] denotes the greatest integer function) then

(A) domain of f(x) is (2n  + , 2n  + 2)  {2n  + /2}


(B) f(x) is continuous when x  (2n  + , 2n  + 2)
(C) f(x) is continuous at x = 2n + /2
(D) f(x) has the period 2

22. Let f(x) = [x] + x  [ x ] , where [x] denotes the greatest integer function. Then
(A) f(x) is continuous on R+ (B) f(x) is continuous on R
(C) f(x) is continuous on R –  (D) discontinuous at x = 1

0 , x
23. Let f(x) and g(x) be defined by f(x) = [x] and g(x) =  2 (where [ . ] denotes the greatest
x , x  R  
integer function) then

(A) lim g(x) exists, but g is not continuous at x = 1


x 1

(B) lim f(x) does not exist and f is not continuous at x = 1


x 1

(C) gof is continuous for all x


(D) fog is continuous for all x

24. Which of the following function(s) defined below has/have single point continuity.

1 if x  Q  x if x  Q
(A) f(x) =  (B) g(x) = 
0 if x  Q 1  x if x  Q

x if x  Q  x if x  Q
(C) h(x) =  (D) k(x) = 
0 if x  Q  x if x  Q

1. Discuss the continuity of the function, f (x) at x = 3, if


 x [x] , if 0  x  3
f (x) =  where [.] denotes greatest integer function.
( x 1) [x] , if 3  x  4
 1  sin3 x
 3 cos2 x ,x   / 2

2. Find the values of ' a ' & ' b ' so that the function, f (x) =  a , x   / 2 is continuous at
 b (1  sinx )
 (   2x )2 , x   / 2

x = /2.
ex 1
 ,x  0
 (1  2x )
n
3. Discuss the continuity of the function, f (x) =  at x = 0. If discontinuous, find the
 7 ,x  0
nature of discontinuity ?

4. If f(x) = x + { x} + [x], where [x] is the integral part & {x} is the fractional part of x. Discuss the
continuity of f in [  2, 2 ]. Also find nature of each discontinuity.
TM
RESONANCE 13
 1  x ,0  x  2
5. Let f(x) =  . Determine the form of g(x) = f (f(x)) & hence find the point of discontinuity
3  x ,2  x  3
of g, if any.

6. Examine the continuity at x = 0 of the sum function of the infinite series:


x x x
   .......... .... .
x 1 ( x  1)(2 x  1) (2x  1)(3 x  1)

sin3 x  A sin 2x  B sin x


7. If f(x) = (x  0) is continuous at x = 0. Find A & B. Also find f (0).
x5

8. Let [x] denote the greatest integer function & f(x) be defined in a neighbourhood of 2 by

 
 1 

 exp  ( x  2) 4 [x  1] ln 4  16
  
,x  2
f(x) =  4 x 16 .

 1  cos( x  2) , x2
A
( x  2) tan ( x  2)

Find the values of A & f(2) in order that f(x) may be continuous at x = 2.

(1  sinx )n  lnx
9. Discuss the continuity of the function f (x) = Limit
n  .
2  (1  sinx )n
10. Let f(x + y) = f(x) + f(y) for all x , y and if the function f(x) is continuous at x = 0, then show that f(x) is
continuous at all x.

11. If f(x  y) = f(x) f(y) for all x, y and f(x) is continuous at x = 1. Prove that f(x) is continuous for all x except
at x = 0. Given f(1)  0.

 x 2y  f( x ) 2f( y )
12. If f   =  x, y  R and f(x) is continuous at x = 0. Prove that f (x) is continuous for all
 3  3
x  R.

max m {f ( t ) ; 0  t  x , 0  x  2
13. If f(x) = sin x and g(x) =  , then discuss the continuity of g(x)  x > 0 .
3x  4 ; x2

Exercise # 1 3. Removable isolated point

4. discontinuous at all integral values in [ 2, 2]


1. C 2. D 3. A 4. B 5. B 6. D 7. D
5. g(x)= 2 + x ; 0  x  1,
8. B 9. B 10. D 11. C 12. C 13. D 14. B = 2  x ; 1 < x  2,
= 4  x ; 2 < x  3,
15. C 16. C 17. B 18. C 19. BC 20. BD g is discontinuous at x = 1 & x = 2
21. ABD 22. ABC 23. ABC 24. BCD 6. Discontinuous 7. A =  4, B = 5, f(0) = 1

Exercise # 2 8. A = 1; f(2) = 1/2

1. continuous at x = 3 9. f (x) is discontinuous at natural multiples of 

1 13. continuous for all x  0 except at x = 2


2. a = ,b=4
2
TM
RESONANCE 14
Differentiation (MOD)

A. First Principle Of Differentiation


1. The derivative of a given function f at a point x = a on its domain is
defined as:

Limit f (a  h)f (a ) , provided the limit exists & is denoted by f (a).


h0
h

f ( x )f (a)
i.e. f (a) = Limit
x a , provided the limit exists.
xa
Mathematics is very much
2. If x and x + h belong to the domain of a function f defined by
like poetry ... what makes
y = f(x), then
a good poem — a great
Limit f ( x  h)f ( x ) if it exists, is called the Derivative of f at x & is
poem — is that there is a h0
h
large amount of thought dy f ( x  h)f ( x )
denoted by f (x) or . i.e., f (x) = Limit
h 0
expressedinveryfewwords. dx h
This method of differentiation is also called ab-initio method or
In this sense formulas
first principle.
likee i  1  0
Solved Example # 1
or
Find derivative of following functions by first principle
 (i) f(x) = x2 (ii) f(x) = tan x
x2
e

dx   are poems. (iii) f(x) = esinx

Solution
D Al ber s , G Al exander son,
( x  h) 2  x 2
C Rei d, Mor e Mat hemat i cal
(i) f(x) = hlim
0
h
Peopl e
2xh  h2
= hlim
0 = 2x.
h

tan( x  h)  tan x
(ii) f(x) = hlim
0
h

tan( x  h  x )[1  tan x tan( x  h)]


= hlim
0
h

tan h
= hlim
0 . (1 + tan2x) = sec 2x.
h

TM
RESONANCE 33
e sin ( x  h)  e sin x
(iii) f(x) = hlim
0
h

= hlim sin x
 

e sin ( x  h ) sin x  1  sin( x  h)  sin x 

0 e h
sin( x  h)  sin x  

= esin x lim sin( x  h)  sin x


h0
h
= esin x cos x

3. Differentiation of some elementary functions

f(x) f(x)

1. xn nx n – 1 (x  R, n  R)

2. ax ax n a

1
3. n |x|
x

1
4. logax x n a

5. sin x cos x

6. cos x – sin x

7. sec x sec x tan x

8. cosec x – cosec x cot x

9. tan x sec 2 x

10. cot x – cosec x

4. Basic Theorems

d
1. (f ± g) = f(x) ± g(x)
dx

d d
2. (k f(x)) = k f(x)
dx dx

d
3. (f(x) . g(x)) = f(x) g(x) + g(x) f(x)
dx

d  f ( x)  g( x ) f ( x )  f ( x ) g( x )
4.   =
dx  g( x )  g2 ( x )

d
5. (f(g(x))) = f(g(x)) g(x)
dx

This rule is also called the chain rule of differentiation and can be written as
dy dy dz
= .
dx dz dx

TM
RESONANCE 34
Note that an important inference obtained from the chain rule is that

dy dy dx
=1= .
dy dx dy

dy 1
 =
dx dx / dy
another way of expressing the same concept is by considering y = f(x) and x = g(y) as inverse functions
of each other.

dy dx
= f(x) and dy = g(y)
dx

1
 g(y) =
f ( x )

Solved Example # 2

Find the differential of the following functions with respect to x.


(i) f(x) = esin x (ii) f(x) = sin( 2x  3)

x
(iii) f(x) = (iv) f(x) = x . sin x
1 x2
Solution.
(i) f(x) = esin x
d
f(x) = esin x (sin x)
dx
= esin x cos x
(ii) f(x) = sin ( 2 x  3 )

1 d
= . (sin (2x + 3))
2 sin (2x  3) dx

cos(2x  3)
=
sin (2x  3)

x
(iii) f(x) =
1 x2

(1  x 2 )  x(2x )
f(x) =
(1  x 2 )2

1 x2
=
(1  x 2 )2
(iv) f(x) = x sin x
f(x) = x. cos x + sin x

TM
RESONANCE 35
Solved Example # 3
If f(x) = sin (x + tanx) then find value of f(0).
Solution.
f(x) = cos (x + tanx) (1 + sec 2x)
f(0) = 2

Self Practice Problems :

1. Find the derviative of following functions using first principle.


(i) f(x) = x sin x
(ii) f(x) = sin2 x
Ans. (i) x cosx + sinx (ii) 2sin x cos x

2. Evaluate if f(5) = 7, then

lim f (5  t )  f (5  t )
t0 2t
Ans. 7.

3. Differentiate the following functions


( x  1)
(i) (1 + 3x2) (2x3 – 1) (ii)
( x  2)( x  3)

1 x
(iii) 1 x 2 (iv)
1 x
(v) cos 3 x sin x (vi) x ex sin x
sin x
(vii) (viii) n (sin x – cos x)
1  cos x

 x 2  2x  1
Ans. (i) 6x (5x3 + x – 1) (ii)
( x  2) 2 ( x  3 ) 2

x 1
(iii) (iv) 1/ 2
1 x 2 (1  x ) (1  x )3 / 2
(v) cos4 x – 3 cos2x sin2x (vi) ex ((sin x + cos x) x + sin x)
1 x cos x  sin x
(vii) sec 2 (viii)
2 2 sin x  cos x

B. Derivative Of Inverse Trigonometric Functions.


 
y = sin–1 x  – y
2 2
 x = sin y
dx
= cos y
dy
dy 1 1
= =
dx cos y 1  sin 2 y

dy 1
= – 1 < x < 1.
dx 1 x2
TM
RESONANCE 36
  
Note here that cos y  1  sin 2 y , rather cos y = ± 1  sin 2 y but for values of y    ,  , cos y is
 2 2
always positive and hence the result. similarly let us find derivative of other inverse trigonometric
functions.
Let y = tan–1 x
x = tan y
dx
= sec 2y = 1 + tan2 y
dy

dx
2
 dy = 1 + x

dy 1
= (x  R)
dx 1 x2


Also if y = sec–1 x y  [0, ] –  
2
 x = secy
dx
dy = sec y tan y

dy 1
 =
dx x. tan y

dy 1
 =
dx  x sec 2 y  1

 1
 sec y  1
dy  2
=  x x 1
dx 1
 sec y  1
 x x 2  1

dy 1
 = x  (– , – 1)  (1, )
dx | x | x2  1
results for the derivative of inverse trigonometric functions can be summarized as :
f(x) f(x)
1
sin–1x ; |x| < 1
1 x 2
1
cos –1x ; |x| < 1
1 x 2
1
tan–1x ; xR
1 x2
1
cot–1x ; xR
1 x2

1
sec –1 x ; |x| > 1
| x | x2  1

1
cosec -1 x ; |x| > 1
| x | x2  1
TM
RESONANCE 37
Solved Example # 4
If f(x) = n (sin–1 x 2) find f(x)
Solution.
1 1
f(x) = . . 2x
(sin 1 x 2 ) 1  ( x 2 )2
2x
=
(sin 1 x 2 ) 1  x 4

Solved Example # 5

If f(x) = 2x sec–1 x – cosec–1 (x) then find f(–2)


Solution.

2x 1
f(x) = 2 sec –1(x) – +
| x | x2  1 | x | x2  1

2 1
f(–2) = 2.sec –1 (– 2) + +
3 2 3

4 5
f(–2) = + .
3 2 3

C. Methods Of Differentiation
1. Logrithmic Differentiation
The process of taking logarithm of the function first and then differentiate is called Logarithmic
Differentiation. It is useful if
(i) a function is the product or quotient of a number of functions OR
(ii) a function is of the form [f(x)]g(x) where f & g are both derivable,
Solved Example # 6
dy
If y = x x find
dx
Solution.
n y = x n x
1 dy 1
 y . dx = x . x + n x

dy
 = x x (1 + n x)
dx

Solved Example # 7
dy
If y = (sin x)n x, find
dx
Solution.
n y = n x . n (sin x)
1 dy 1 cos x
y dx = n (sin x) + n x.
x sin x

dy  n sin x  
 = (sin x)n x   cot x n x 
dx  x 

TM
RESONANCE 38
Solved Example # 8

x1/ 2 (1  2x )2 / 3 dy
If y= 3/4 4/5 find
(2  3x ) (3  4 x ) dx
Solution.
1 2 3 4
n y = n x + n (1 – 2x) – n (2 – 3x) – n (3 – 4x)
2 3 4 5

1 dy 1 4 9 16
 y dx = – + 4 (2  3 x ) + 5 (3  4 x )
2x 3(1  2x )

dy  1 4 9 16 
= y  2x  3 (1  2 x )  4(2  3 x )  5 (3  4 x ) 
dx  

2. Implicit differentiation

If f(x, y) = 0, is an implicit function then in order to find dy/dx, we differentiate each term w.r.t. x
regarding y as a functions of x & then collect terms in dy/dx.

Solved Example # 9
dy
If x3 + y3 = 3xy find
dx

Solution.
Differentiation both sides w.r.t.x, we get
dy dy
3x2 + 3y2 = 3x + 3y
dx dx

dy y  x2
= 2
dx y x
Note that above result holds only for points where y2 – x  0

Solved Example # 10
dy
If xy = ex – y, then find
dx

Solution.
Taking log on both sides
y n x = (x – y) .........(i)
differentiating w.r.t x, we get
y dy dy
+ lnx =1–
x dx dx

y
1
dy x
 =
dx 1  n x

dy xy
 = x(1  n x )
dx

TM
RESONANCE 39
Solved Example # 11
dy
If xy + yx = 2 then find
dx
Solution.
u+v=2
du dv
 + =0
dx dx
where u = xy & v = yx
 n u = y n x & n v = x n y

1 du y dy 1 dv x dy
 = + n x & = n y + y
u dx x dx v dx dx

du y dy  dv  x dy 
 = x y   n x  & = yx  n y  y dx 
dx x dx  dx  

y dy   x dy 
x y   n x  + yx  n y  

x dx   y dx  = 0.

 x y y
 y n y  x . 
dy  x
 =–
dx  y x
 x n x  y x . 
 y

Self Practice Problems

1. Differentiate the following functions :


 1 x 
(i) y = sec –1 (x 2) (ii) y = tan–1  
 1 x 
x
 1 x
(iii) y = 1   (iv) y = ex
 x
(v) y = (ln x)x + (x)sin x

dy
2. Find if
dx
(i) y = cos (x + y) (ii) x2/3 + y2/3 = a2/3
(iii) x = y n (x – y)

dy n x
3. If xy = ex – y, then prove that = .
dx (1  n x )2

x a dy x
4. If = log , prove that =2– .
xy xy dx y

x   1 1 
2 1  1
Ans. 1. (i) (ii) (iii) 1   n 1    
1 x2   x  1 x 
x x4  1  x

  1   sin x 
(v)  n (nx )   nx   (n x)x + x sinx   cos x nx 
x
(iv) x x. e x (nx + 1)
    x 

1/ 3
 sin( x  y ) y y( x  y )
2. (i) 1  sin( x  y ) (ii) –   (iii) x( x  y )
x
TM
RESONANCE 40
3. Differentiation using substitution

Following substitutions are normally used to sumplify these expression.


(i) x 2  a2  x = a tan  or a cot 

(ii) a2  x 2  x = a sin  or a cos 

(iii) x 2  a2  x = a sec  or a cosec 

xa
(iv)  x = a cos 
ax

Solved Example # 12

 1  x 2  1
 
Differentiate y = tan–1  .
x
 
Solution.
Let x = tan 

  
  = tan–1x ;    , 
 2 2

 | sec  | 1    
y = tan–1   [ |sec| = sec   ,  ]
 tan    2 2

 1  cos  
 y = tan–1  
 sin  

 
 y = tan–1  tan 
 2

   
 y= [tan–1 (tanx) = x for x    ,  ]
2  2 2

1
 y= tan–1 x
2

dy 1
 =
dx 2(1  x 2 )

Solved Example # 13

dy  1 x  1 x 
Find where y = tan–1  
dx  1 x  1 x 
 
Solution.
x = cos
 = cos –1 (x) ;  [0, ]

 1  cos   1  cos  
 y = tan–1  

 1  cos   1  cos  

  
 2 cos  2 sin 
 2 2 
 y = tan–1
  
 2 cos  2 sin 
 2 2
TM
RESONANCE 41
 
 1  tan 
 2
 y = tan–1  
 1  tan 
 2

 
 y= –
4 2

 1
 y= – cos –1 x
4 2

dy 1
 =
dx 2 1 x2

     
Note that 1  cos  = 2 cos but for   0,  , 2 cos = 2 cos
2 2  2 2 2

  
Also tan–1 (tan x) = x for x    ,  .
 2 2

Solved Example # 14

 2x 
If f(x) = sin–1   then find
 1 x2 

 1
(i) f(2) (ii) f   (iii) f(1)
2
Solution.
x = tan 
 
  = tan–1 (x) ; – <<
2 2
 y = sin–1 (sin 2)

 
   2 2
 2      2 tan 1 x x 1
    1
2 tan x 1 x  1
y =  2   2 
2 2  f(x) = 
  (   2 tan 1 x ) x  1
 (   2)    2    
 2

 2
 2
x 1
 1 x
 2
 2
1 x  1
 f(x) =  1  x
  2
x  1
 1  x 2

2  1 8
(i) f(2) = – (ii) f   =
5 2 5
(iii) f(1+) = – 1 & f(1–) = + 1
 f(1) does not exist.

TM
RESONANCE 42
Aliter
Above problem can also be solved without any substution also, but in a little tedious way.
 2x 
f(x) = sin–1  
 1 x2 

1 2{(1  x 2 )  2x 2 }
f(x) = .
4x 2 (1  x 2 )2
1
(1  x 2 )2

(1  x 2 ) 2(1  x 2 )
= .
(1  x 2 )2 (1  x 2 )2

2 (1  x 2 )
f(x) = 2 .
(1  x ) | 1 x2 |

 2
 |x|1
 1 x2
thus f(x) =   2
|x|1
1  x 2

Solved Example # 15
dy 1 y2
If 1 x 2 + 1 y 2 = a(x – y), then prove that = .
dx 1 x2
Solution.
Put x = sin    = sin–1 (x)
y = sin    = sin–1 (y)
 cos + cos  = a (sin – sin)

       


 2cos   cos   = 2a cos   sin  
 2   2   2   2 

 
 cot   =a
 2 
  –  = 2 cot–1 (a)
 sin–1 x – sin–1 y = 2 cot–1 (a)
differentiating w.r.t to x.

1 1 dy
– =0
2 2 dx
1 x 1 y

dy 1 y2
 =
dx 1 x2
Aliter
Using implicit differention.

x y dy  dy 
2 – 2 = a 1  
1 x 1 y dx  dx 

x
  a
a  y  dy x dy 1 x2
   =a+  =
 1 y2  dx 1 x 2 dx
a
y
1 y2

TM
RESONANCE 43
1 x 2  1 y 2 x

dy xy 1 x 2
 =
dx 1 x2  1 y2 y

xy 1 y2

dy (1  x 2 )  (1  x 2 )(1  y 2 )  x 2  xy 1 y2 1  (1  x 2 )(1  y 2 )  xy 1 y2
= . = .
dx (1  x 2 )(1  y 2 )  (1  y 2 )  xy  y 2 1 x2 1  (1  x 2 )(1  y 2 )  xy 1 x2

dy 1 y2
 = Hence proved
dx 1 x2

4. Parametric Differentiation
dy dy / d
If y = f() & x = g() where is a parameter, then dx  dx / d .

Solved Example # 16
dy
If x= a cos3t and y = a sin3t. Find
dx

dy dy / dt  3a sin 2 t cos t
= = = – tan t
dx dx / dt 3a cos 2 t sin t

Solved Example # 17
dy 
If y = a cos t and x = a (t – sint) find the value of at t = .
dx 2

dy a sin t )
= a(1  cos t )
dx

dy
dx 
= – 1.
t
2

5. Derivative of one function with respect to another

dy dy / dx f '(x)
Let y = f(x); z = g(x) then   .
dz dz / dx g' (x)
Solved Example # 18
Find derivative of y = n x with respect to z = e x.
dy dy / dx 1
= =
dz dz / dx xex

Self Practice Problems :

dy
1. Find when
dx
(i) x = a (cos t + t sin t) & y = a (sin t – t cos t)
 1 t2   2t 
(ii)

x=a 
 & y=b.  
2 
 1 t   1 t 2 

( t 2  1)b
Ans. (i) tan t (ii)
2at

TM
RESONANCE 44
 x2  2 xa 2
  dy
2. If y = sin–1  4 4  then prove that dx = x 4  a 4 .
 x a 

 2x  dy 2
3. If y = tan–1  2
 then prove that = (| x |  1)
 1 x  dx 1 x2

du 1  u2
4. If u = sin (m cos–1 x) and v = cos (m sin–1 x) then prove that = .
dv 1 v 2

D. Derivatives of Higher Order


Let a function y = f(x) be defined on an open interval (a, b). It’s derivative, if it exists on (a, b) is a certain
function f (x) [or (dy/dx) or y ] & is called the first derivative of y w. r. t. x.
If it happens that the first derivative has a derivative on (a, b) then this derivative is called the second
derivative of y w. r. t. x & is denoted by f (x) or (d2y/dx 2) or y .

d3 y d  d2 y 
Similarly, the 3rd order derivative of y w. r. t. x, if it exists, is defined by 
dx 3 dx  dx 2  It is also denoted

by f (x) or y .

Solved Example # 19

If y = x3 n x then yand y


Solution.
1
y = 3x2 n x + x 3
x
y = 3x2 n x + x2
1
y = 6x n x + 3x 2 . + 2x
x
y = 6x n x + 5x
y = 6 n x + 11

Solved Example # 20
x
 1
If y =   then find y(1)
x
Solution.
n y = – x n x when x=1  y=1
y
 y = – (1 + n x)  y = – y (1 + n x) ......(i)

again diff. w.r.t. to x,


1 y
y = – y(1 + n x) – y .  y = y (1 + ln x)2 – (using (i))
x x
 y(1) = 0

It must be carefully noted that in case of parametric functions

dy dy / dt d2 y d2 y / dt 2 d2 y d  dy / dt 
although = but  rather =  
dx dx / dt 2 2 2 2 dx  dx / dt 
dx dx / dt dx

TM
RESONANCE 45
which on applying chain rule can be resolved as

 dx d2 y dy d2 x 
 .  . 
 dt dt 2 dt dt 2 
d2 y d  dy / dt  dt d2 y   dt
2 =  .  = 2 .
dx dt  dx / dt  dx dx 2  dx  dx
 
 dt 

 dx d2 y dy d2 x 
 . 2  . 
d2 y  dt dt dt dt 2 
= 3
dx 2  dx 
 
 dt 

Solved Example # 21

d2 y
If x = t + 1 and y = t2 + t3 then find .
dx 2
Solution.
dy dx
= 2t + 3t2 ; =1
dt dt

dy
 = 2t + 3t2
dx

d2 y d dt
 2 = (2t + 3t2) .
dx dt dx

d2 y
= 2 + 6t.
dx 2

Solved Example # 22

d2 y 
If x = 2 cos t – cos 2t and y = 2 sin t – sin 2t then find value of 2 at t = .
dx 2
Solution.
dy dx
= 2 cos t – 2 cos 2t = 2 sin 2t – 2 sin t
dt dt

3t t
2 sin . sin
dy cos t  cos 2t 2 2
= = .
dx sin 2t  sin t 3t t
2 cos . sin
2 2

dy 3t
 = tan
dx 2

d2 y d  3t  d2 y d  3 t  dt
 2 =  tan   2 =  tan  .
dx dx  2 dx dt  2  dx

3 3t
. sec 2 d2 y
d2 y 2 2 3
=  =–
dx 2
2 (sin 2t  sin t ) dx 2 t
 2
2

TM
RESONANCE 46
Solved Example # 23

Find second order derivative of y= sin x with respect to z = ex.


Solution.

dy dy / dx cos x
 = =
dz dz / dx ex

d2 y d  cos x  d2 y d  cos x  dx
 =    =  .
dz 2 dz  e x  dx 2 dx  e x  dz

 e x sin x  cos xe x 1
= .
(e x )2 ex

d2 y (sin x  cos x )
=–
dz 2
e2x
Solved Example # 24

y = f(x) and x =g(y) are inverse functions of each other than express g(y) and g(y) in terms of
derivative of f(x).
Solution.
dy dx
= f(x) and dy = g(y)
dx
1
 g(y) = ...........(i)
f ( x )
again differentiating w.r.t. to y

d  1 
g(y) = dy  
 f ( x ) 

d  1  dx
=   .
dx  f ( x )  dy

f ( x )
=– . g(y)
f ( x )2

f ( x )
 g(y) = – .........(ii)
f ( x )3
which can also be remembered as

d2 y
2
d x dx 2
2 = – 3 .
dy  dy 
 
 dx 

Solved Example # 25

y = sin (sinx) then prove that y + (tanx) y + y cos 2x = 0

Solution.
Such expression can be easily proved using implict differention.
 y = cos (sin x) cos x
 sec x.y = cos (sin x)
again differentiating w.r.t x, we can get
secx y + y sec x tan x = – sin (sin x) cos x
 tanx y = – y . cos2 x
 y +(tanx) y + y cos 2x = 0
TM
RESONANCE 47
Self Practice Problems :

n x d2 y
1. If y = then find
x dx 2

2n x  3
Ans.
x3

2. Prove that y = x + tan x satisfies the differentiation equation

d2 y
cos 2 x – 2y + 2x = 0.
dx 2

d2 y
3. If x = a (cos +  sin ) and y = a(sin  –  cos) then find .
dx 2

sec 3 
Ans.
a
4. Find second derivative of nx with respect to sin x.
x sin x  cos x
Ans.
x 2 cos3 x

5. if y = e– x (A cos x + B sin x), prove that

d2 y dy
2 +2. + 2y = 0.
dx dx

Solved Example # 26

d2 y dy
If y = (tan–1 x)2 then prove that (1 + x 2)2 2 + 2x (1 + x 2) = 2.
dx dx
Solution.

dy 2 tan 1 x
=
dx 1 x2

dy
 (1 + x 2) = 2tan–1 (x)
dx

d2 y dy 2
 (1 + x 2) + 2x =
dx 2 dx (1  x 2 )

d2 y dy
 (1 + x 2) 2 + 2x (1 + x2) =2
dx dx

f ( x ) g( x ) h( x )
l( x ) m( x ) n( x )
11. If F(x) = , where f, g, h, l, m, n, u, v, w are differentiable functions of x then F (x)
u( x ) v( x ) w( x)

f ' ( x ) g' ( x ) h' ( x ) f ( x ) g( x ) h( x ) f ( x) g( x ) h( x )


l( x ) m( x ) n( x ) l' ( x ) m' ( x ) n' ( x ) l( x ) m( x ) n( x )
= + +
u( x ) v( x ) w( x) u( x ) v( x ) w( x ) u' ( x ) v' ( x ) w ' ( x )

TM
RESONANCE 48
12. L’ Hospital’s Rule:
If f(x) & g(x) are functions of x such that:

(i) Limit f(x) = 0 = Limit g(x) OR Limit f(x) =  = Limit


x a x a x a x a g(x) &

(ii) Both f(x) & g(x) are continuous at x = a &

(iii) Both f(x) & g(x) are differentiable at x = a &

(iv) Both f (x) & g (x) are continuous at x = a, Then

f (x) f ' (x) f " (x)


Limit = Limit = Limit
x a g( x ) x a g' ( x ) x a g" ( x ) & so on till indeterminant form vanishes

TM
RESONANCE 49
Part : (A) Only one correct option

2
dy
1. If f(x) = 2 x 2  1 and y = f(x ) then dx at x = 1 is
(A) 2 (B) 1 (C) – 2 (D) –1

x2 dy
2. If y = x then =
dx
2 2 2  1 2
(A) 2 n x. xx (B) (2 n x + 1). xx (C) (2 n x + 1). xx (D) x x  1 . n ex 2

 x
tan 1  sin 
 2
3. If f(x) = e , then f(0).

1 1
(A) (B) – (C) 1 (D) –1
2 2

x dy
4. If y = then =
x dx
a
x
b
a .......... .....

a b a b
(A) (B) (C) (D)
ab  2 ay ab  2 by ab  2 by ab  2 ay

d2 F
5. Let f(x) = sin x; g(x) = x 2 & h(x) = loge x & F(x) = h[g(f(x))] then is equal to:
dx 2
(A) 2 cosec 3 x (B) 2 cot (x2)  4x 2 cosec 2 (x 2)
(C) 2x cot x 2 (D)  2 cosec2 x

dy
6. If y = (1 + x) (1 + x 2) (1 + x4) .....(1 + x2n), then at x = 0 is
dx
(A) –1 (B) 1 (C) 0 (D) 2n

dy 1
7. If y = sin1  x 1  x  x 1  x 2  and dx = + p, then p =
 2 x (1  x)

1 1
(A) 0 (B) (C) sin1 x (D)
1 x 1 x 2

 
 y  dy
8. If x 2  y 2 = e t where t = sin–1   then dx :
 x y2
2
 

xy xy 2x  y xy


(A) (B) (C) (D) 2x  y
xy xy xy

TM
RESONANCE 50
x2  1 x2  1 dy
9. If y = sin1 2 + sec 1
2 , x > 1 then is equal to:
x 1 x 1 dx

x x2
(A) 4 (B) 4 (C) 0 (D) 1
x 1 x 1

t 1
10. The differential coefficient of sin1 w.r.t. cos 1 is:
1  t2 1  t2

1
(A) 1 (B) t (C) (D) none
1  t2

 tan  1 x 
11. Differentiation of   1
 1  w.r.t. tan x is:
 1  tan x 

 1  1 1
(A)   (B)  1 (C) (D)
1
(1  tan 1 x )2
 1  tan x  1  tan x 1 2

12. Let f(x) be a polynomial in x. Then the second derivative of f(ex), is:
(A) f  (ex). ex + f  (ex) (B) f  (ex). e2x + f  (ex). e2x
x 2x
(C) f  (e ) e (D) f  (ex). e2x + f  (ex). ex

f g h
f  g h
13. If f(x), g(x), h(x) are polynomials in x of degree 2 and F(x) = , then F(x) is equal to
f  g h

(A) 1 (B) 0 (C) –1 (D) f(x) . g(x) . h(x)

y y1 y2
14. If y = sin mx then the value of y 3 y4 y 5 (where settings of y shows the order of derivative) is:
y6 y7 y8

(A) independent of x but dependent on m (B) dependent of x but independent of m


(C) dependent on both m & x (D) independent of m & x.

f (5  t )  f (5  t )
15. If f  (5) = 7 then Limit
t 0 =
2t
(A) 0 (B) 3.5 (C) 7 (D) 14

16. Let ef(x) = ln x. If g(x) is the inverse function of f(x) then g (x) equals to:
x
(A) ex (B) ex + x (C) exe (D) ex + ln x

dn
17. If u = ax + b then n [f(ax + b)] is equal to:
dx

dn dn dn dn
(A) [f(u)] (B) a [f(u)] (C) an [f(u)] (D) an [f(u)]
du n du n du n dx n
TM
RESONANCE 51
 2x  1 dy
18. If y = f  2  & f  (x) = sin x then =
 x  1 dx

(A)
1  x  x2
sin 
 2x  1
 (B)

2 1  x  x2  2x  1
sin  
2
1  x 2  x 2  1 1  x 
2
2
 x 2  1

1  x  x2  2x  1
(C) 2 sin   (D) none

1  x2   x 2  1

 d  3 d 2y 
19. If y = P(x), is a polynomial of degree 3, then 2  
2  y . 2  equals:
 dx   dx 
(A) P  (x) + P  (x) (B) P  (x). P  (x) (C) P (x). P  (x) (D) a constant

Part : (B) May have more than one options correct

20. Two functions f & g have first & second derivatives at x = 0 & satisfy the relations,
2
f(0) = , f  (0) = 2 g  (0) = 4g (0), g  (0) = 5 f  (0) = 6 f(0) = 3 then:
g(0)

f (x) 15
(A) if h(x) = then h  (0) = (B) if k(x) = f(x). g(x) sin x then k  (0) = 2
g(x) 4

g (x) 1
(C) Limit
x 0 = (D) none
f  (x) 2

fn  1 ( x ) d
21. If f n (x) = e for all n  N and f o (x) = x, then {f (x)} is equal to:
dx n

d
(A) f n (x). {f (x)} (B) f n (x). f n  1 (x)
dx n  1
(C) f n (x). f n  1 (x)........ f 2 (x). f 1 (x) (D) none of these

22. If f is twice differentiable such that f(x) = –f(x) and f(x) = g(x). If h(x) is a twice differentiable function
such that h(x) = [f(x)]2 + [g(x)]2 . If h(0) = 2, h(1) = 4, then the equation y = h(x) represents:
(A) a curve of degree 2
(B) a curve passing through the origin
(C) a straight line with slope 2
(D) a straight line with y intercept equal to 2.

x3
23. Given f(x) =  + x 2 sin 1.5 a  x sin a. sin 2a  5 sin–1 (a2  8a + 17) then:
3
(A) f(x) = – x2 + 2x sin6 – sin4 sin8 (B) f  (sin 8) > 0
(C) f  (x) is not defined at x = sin 8 (D) f  (sin 8) < 0

24. If f(x) = x3 + x2f(1) + xf(3) for all x  R then


(A) f(0) + f(2) = f(1) (B) f(0) + f(3) = 0 (C) f(1) + f(3) = f(2) (D) none of these

25. If f(x) = (ax + b) sin x + (cx + d) cos x, then the values of a, b, c and d such that f(x) = x cos x for all
x are
(A) a = d = 1 (B) b = 0 (C) c = 0 (D) b = c

TM
RESONANCE 52
d2 y dy
1. If y = A e kt cos (p t + c) then prove that 2 +2k + n2 y = 0, where n2 = p2 + k 2.
dt dt

Evaluate the following limits using L hospitale rule as otherwise

2. Limit log tan 2 x (tan2 2 x)


x 0

( xa) 4 ( xa)3 1 ( xa) 4 ( xa)2 1


( xb) 4 ( xb)3 1 ( xb) 4 ( xb)2 1
3. If f (x) = then f  (x) =  . . Find the value of  .
( xc ) 4 ( xc )3 1 ( xc ) 4 ( xc )2 1

d3 y 8.b
4. If x = a t3 &y=b t 2, where t is a parameter, then prove that =
dx 3
27a 3 .t 7

dy sina
5. If sin y = x sin (a + y), show that = 2 .
dx 1  2xcosax

F" f " g" 2c F f  g


6. If F(x) = f(x). g(x) & f (x). g (x) = c, prove that F  f  g  f g & F  f  g .

7. If  be a repeated root of a quadratic equation f(x) = 0 & A(x), B(x), C(x) be the polynomials of degree

A( x ) B( x ) C( x )
A( ) B( ) C( )
3, 4 & 5 respectively, then show that is divisible by f(x), where dash denotes the
A ' ( ) B' () C' ( )
derivative.
3/2
 2
1   dy  
  dx   1 1
8. Show that R = 2 can be reduced to the form R2/3 = 2
 2 .
d y 2 2
d y 3
d x 3
dx 2  
 dx 2 
 
 dy 2 
   
Also show that, if x = a sin 2  (1 + cos 2 ) & y = a cos 2  (1  cos 2 ) then the value of R equals to
4 a cos 3.

9. Differentiate the following functions with respect to x.


sinx  xcosx  1 1  cos x

(i) x 2. n x. ex (i) xsinx  cosx (iii) tan  tan 
 1  cos x 
 

Exercise # 1 Exercise # 2
2. 1 3. 3
1. A 2. C 3. A 4. D 5. D 6. B 7. D
9. (i) ex x (2 n x + 1 + x n x)
8. B 9. C 10. A 11. C 12. D 13. B 14. D
x2 1 x
15. C 16. C 17. C 18. B 19. C 20. ABC (ii) (iii) sec 2
( xsinx  cosx) 2 2 2
21. AC 22. CD 23. AD 24. ABC 25. ABC

TM
RESONANCE 53
Derivability

1. Differentiability of a function at a point:

(i) The right hand derivative of f (x) at x = a denoted by


f (a+) is defined by:
I recoil with
f (a+) = Limit
f(a  h)f(a)
, provided the limit exists.
h 0 
h
dismay and horror
at this lamentable
(ii) The left hand derivative of f(x) at x = a denoted by
plague of functions f (a–) is defined by:
which do not have
f(a  h)f(a )
derivatives. f  (a– ) = Limit
h 0  , Provided the limit exists.
h
A function f(x) is said to be differentiable(finitely) at
x = a if f (a+) = f  (a–) = finite
Charl es Hermit e
f(a  h)f(a)
By definition f (a) = Limit
h0
h
Irecognizethelion Solved Example # 1

by his paw. x , x 1
Comment on the differentiability of f(x) = f(x) =  2
x , x 1
.
at x = 1.
...Jacob Ber noul l i Solution.

f (1  h)  f (1)
R.H.D. = f (1+) = hlim
0  h
After reading an
(1  h)2  1
anonymous solution = hlim
0 
h
to a problem that
1  h 2  2h  1
he realized was = hlim
0 
h
Newton’s solution.
= hlim
0 
(h + 2)
=2
f (1  h)  f (1)
L.H.D. = f(1–) = hlim
0  h

1 h  1
= hlim
0  h
=1
As L.H.D.  R.H.D. Hence f(x) is not differentiable at x = 1.

TM

RESONANCE 15
Solved Example # 2
If f(x) = A + Bx 2 x<1
= 3Ax – B + 2 x1
then find A and B so that f(x) become differentiable at x = 1.
Solution.

f (1  h)  f (1)
f(1+) = hlim
0  h

3 A (1  h)  B  2  3 A  B  2
= hlim
0  h

3 Ah
= hlim
0  h
= 3A

f (1  h)  f (1)
f(1–) = hlim
0  h

A  B(1  h)2  3A  B  2
= hlim
0  h

( 2A  2B  2)  Bh2  2Bh


= hlim
0 
h
hence for this limit to be defined
– 2A + 2B – 2 = 0
B=A+1

f(1–) = hlim
 0 (Bh – 2B)

= 2B
 f(1–) = f(1–)
3A = 2B = 2(A + 1)
A = 2, B = 3 Ans.

Solved Example # 3

[cos x ] x  1
f(x) =  comment on the derivability at x = 4.
 2{ x }  1 x  1
Solution.
f (1  h)  f (1)
f(i) = h lim
 0
h

cos(   h)  3
= h lim
 0
h

1  cos h
= h lim
 0
h

h
2 sin2
lim  2
= h0 h
=0

f (1  h)  f (1)
f(1+) = h lim
 0
h

TM

RESONANCE 16
2{1  h}  1  1
= h lim
 0
h

= h lim
 0  2h

=2
f(x) is not differetiable at x = 3.

Self Practice Problem :

[2x]  x , x 1
1. If f(x) = 
 {x}  1 , x 1
comment on the continuity and differentiable at x = 1.
Ans. Discontinuous and non-differentiable at x = 1

x tan 1 x, x  0
2. f(x) = 
 0 , x0

comment on the derivative of f(x) at x = 0.


Ans. non-differentiable at x = 0

2. Concept of Tangent and its Association with Derivability:

Tangent :- The tangent is defined as the limiting case of a chord or a secant

f (b )  f (a )
slope of chord joining (a, f(a)) and (b, (f(b)) =
ba

f (a  h )  f ( a )
slope of the line joining a and (a,f(a)) and (a + h, f(a + h)) =
h

f (a  h )  f ( a )
R.H.D. = f(a+) = h lim
 0
h

f (a  h )  f (a )
L.H.D. = f(a–) = h lim
 0
h
A function will have a tangent at point x = a if f(a+) = f(a–) (may or may be finite)
and equation of tangent at (a, f(a)) is given by y – f(a) = f(a) (x – a)
Note :-
(i) y = x 3 has x-axis as tangent at origin.
(ii) tangent is also defined as the line joining two infinitly small close points on a curve.
(iii) y = sgn (x) will have a vertical tangent at x = 0
(iv) y = |x| does not have tangent at x = 0 as L.H.D.  R.H.D.
(v) Discontinuous function can also have vertical tangent e.g. y = sgn (x) at x = 0.
(vi) A function is said to derivable at x = a if their exist a tangent of finite slope at that point.
f(a+) = f(a–) = finite value
TM

RESONANCE 17
Note: -
If a function f (x) is differentiable at x = a, the graph of f (x) will be such that there is a unique tangent to
the graph at the corresponding point. But if f (x) is non-differentiable at x = a, there will not be unique
tangent at the corresponding point of the graph.

Solved Example # 4

Find the equation of tangent of y = (x)1/3 at x = 1 and x = 0

Solution.
At x = 1 Here f(x) = (x)1/3

f (1  h)  f (1)
L.H.D = f(1–) = hlim
0
h

(1  h)1/ 3  1
= hlim
0
h

1 1  2
  1 h
h 33 
   ........ 
3 2!
= hlim
0 h

1
=
3

f (1  h)  f (1) (1  h)1/ 3  1
R.H.D. f(1+) = hlim
0 = hlim
0
h h

1
=
3

1
As R.H.D. = L.H.D. =
3

1
 slope of tangent =
3

1
 y – f(1) = (x – 1)
3

1
y–1= (x – 1)
3
 3y – x = 2 is tangent to y = x 1/3at (1, 1)
At x = 0

(0  h)1/ 3  0
L.H.D. = hlim
0 =+
h

(0  h)1/ 3  0
R.H.D = hlim
0 =+
h
as L.H.D. = R.H.D - infinite.
 y = f(x) will have a vertical tangent at origin.
 x = 0 is the tangent to y = x 1/3 at origin.

TM

RESONANCE 18
Self Practice Problems :

1. If possible find equation of tangent to the following curves at the origin points.
(i) y = x 3 + 3x 2 + 28x +1 at x = 0 Ans. y = 28x + 1
(ii) y = sgn (x 2) at x = 0 Ans. tangent does not exist.
(iii) y = sgn (x 2 – 3x + 2)at x = 2 Ans. x = 2
2/3
(iv) y = (x – 8) at x = 8 Ans. tangent does not exist

3. Relation between Derivability & Continuity:

(i) If f (a) exists then f(x) is continuous at x = a.

(ii) If f(x) is derivable for every point of its domain of definition, then it is continuous in that domain.

NOTE: The Converse of the above result is not true i.e."If 'f' is continuous at x, then 'f' is derivable at
x" is not true.
e.g. the functions f(x) =  x  2  is continuous at x = 2 but not derivable at x = 2.
(iii) If a function f is not differentiable but is continuous at x = a it geometrically implies a sharp
corner or kink at x = a.

If f(x) is a function such that R.H.D = f(a+) =  and L.H.D. = f(a–) = m


Case - 
If  = m = some finite value, then the function f(x) is differentiable as well as continuous
Case - 
if   m = but both have some finite value, then the function f(x) will not be differentiable but it will be
continuous.
Case - 
If at least one of the  or m is infinite, then the function is non differentiable but we can not say about
continuity of f(x).

(i) (ii) (iii)


continuous and differentiable continuous but not differentiable neither continuous nordifferentiable

Solved Example # 5
If f(x) is differentiable at x = a prove that it will be continuous at x = a.
Solution.

lim f (a  h)  f (a) = 
h0  h

lim [f(a+h) – f(a)] = h


h0 

as h  0+ and  is finite then hlim


0 
f(a + h) – f(a) = 0

 lim f (a + h) = f(a).
h0 

TM

RESONANCE 19
Similarly hlim
0 
[f(a – h)– f(a)] = – h

 lim f(a – h) = f(a)


h0 

 lim f(a + h) = f(a) = lim f(a – h)


h0  h0

Hence, f(x) is continuous.

Solved Example # 6

A  Bx 2 , x 1
If f(x) =  find the value of A and B so that f(x) is differentiable at x = 1.
 3Ax  B  2 , x  1

Solution.
Since f(x) will be differentaible at x = 1 then it must be continuous
 3A – B + 2 = A + B
 2A – 2B + 2 = 0
A–B+1=0 ..........(i)

2Bx , x  1
Now, f(x) = 
3 , x  1
 3A = 2B .........(ii)
from (i) an (ii)
2B
–B+1=0
3

B
 – =–1
3
 B=3
 A=2
A = 2, B = 3 Ans.

Solved Example # 7

 x 2 sgn[ x]  { x } , 0  x  2
If f(x) =  , comment on the continuity and differentiablilty of f(x) at x = 1, 2.
sin x  | x  3 | , 2  x  4

Solution.
Continuity at x = 1
lim f(x) = lim  (x 2 sgn[x] + {x})
x  1 x 1

=1+0=1
lim f(x) = lim  (x 2 sgn [x] + {x})
x  1 x 1

=1 sgn (0) + 1
=1
 f(1) = 1
 L.H.L = R.H.L = f(1). Hence f(x) is continuous at x = 1.
Now for differentiability,
f (1  h)  f (1)
f(1+) = h lim
 0
h

(1  h)2 sgn[1  h]  {1  h}  1
= h lim
 0
h
TM

RESONANCE 20
(1  h)2  h  1
= h lim
 0
h

1  h 2  2h  h  1
= h lim
0 
h

h2  3h
= h lim
0 
h
=3
f (1  h)  f (1)
and f(1–) = h lim
 0
h

(1  h)2 sgn[1  h]  1  h  1
= h lim
 0
h
=1
f(1 )  f(1–).
+

Hence f(x) is non differentiable at x = 1.


Now at x=2
lim f(x) = lim x2 sgn [x] + {x}
x 2  x 2

=4.0+1
=1
lim f(x) = lim (sinx + |x – 3|)
x 2  x 2

= 1 + sin 2
Hence L.H.L  R.H.L
Hence f(x) is discontinuous at x = 2 and then f(x) also be non differentaible at x = 2.

Self Practice Problem :

 e [ x ]  | x |  1 
  x0
1. If f(x) =  [ x ]  {2x }  , comment on the continuity at x = 0 differentiability of at x = 0.
 1/ 2 x0

Ans. discontinuous hence non-differentiable at x = 0

4. Derivability Over an Interval:

f (x) is said to be derivable over an open interval if it is derivable at each & every point of the interval and
f(x) is said to be derivable over a closed interval [a, b] if:
(i) for the points a and b, f (a+) & f (b–) exist finiitely
(ii) for any point c such that a < c < b, f (c+) & f(c –) exist finitely & are equal.

For checking the derivability in an interval following points should be checked


(i) All those point where discontinuity may arise
(ii) Modulus functions are also non differentiable and hence should be checked at their critical
points.
Important : -
All polynomial, exponantial, logrithimic and trigonometric (inverse trigonomettric not included) are
differentiabl in their domain.

2x
Graph of y = sin–1 Graph of y = sin–1 x.
1 x2
TM

RESONANCE 21
Non differentiable at x = 1 & x = – 1 Non differentiable at x = 1 & x = – 1

Solved Example # 8

  4
[2x ] sgn x   1  x  2
  3
f(x) =  Find that points at which continuity and differentiability should be
 1 
 x  [sin x] 0  x  2
  3

checked. Also check the continuity and differentaibility of f(x) at x = 1.


Solution.

  4
[2x] sgn x   1  x  2
  3
f(x) =  1 the point, where we should check the continuity and differentiability
 
 x  [sin x] 0  x  1
  3

1 2 4 3
are x = 0, , , 1, , , 2 line charge now continuity at x = 3.
2 3 3 2

 1
 lim f(x) = lim x   [sin x]
x 1 x 1  3
=0

 1
lim f(x) = lim [2x] sgn x  
x 1 x 1  3 
= 2 {– 1}
=–2
 L.H.L  R.H.L. hence f(x) is discontinuous at x = 1 and hence also it is non diffferentiable at x = 1.
Self Practice Problem:

1. If f(x) = [x] + [1 – x] = – 1  x  3, Draw its graph and comment on the continuity and differentiability of
f(x).
Ans. + is discontinuous at x = 0, 1, 2, 3 hence non-differentiable

 | 1  4 x 2 | 0  x  1
2. f(x) =  2 draw the graph of f(x) and coment on the differentiability and continuity of
 [ x  2x] 1  x  2

f(x).
1
Ans. + is discontinuous at x = 1, 2 & non differentiable at x = , 1, 2
2
TM

RESONANCE 22
5. Differentiabilty of sum, product & composition of functions

(i) If f(x) & g(x) are derivable at x = a then the functions f(x)  g(x), f(x). g(x) will also be derivable
at x = a & if g (a)  0 then the function f(x)/g(x) will also be derivable at x = a.

(ii) If f(x) is not differentiable at x = a & g(x) is differentiable at x = a, then the product function
F(x) = f(x). g(x) can still be differentiable at x = a
e.g. f(x) = x and g(x) = x 2.

(iii) If f(x) & g(x) both are not differentiable at x = a then the product function;
F(x) = f(x). g(x) can still be differentiable at x = a e.g. f(x) = x & g(x) = x.

(iv) If f(x) & g(x) both are non-derivable at x = a then the sum function F(x) = f(x) + g(x) may be a
differentiable function. e.g. f(x) = x & g(x) = x.

f (a  g(h))  f (a  p(h))
(v) If f is differentiable at x = a, then hlim
0 g(h)  p(h) = f(a), where ,

lim P(h) = lim g(x) = 0


h0 h0

Solved Example # 9
Discuss the differentiability of f(x) = x + |x|.
Solution.

Non-differentiable at x = 0.

Solved Example # 10
Discuss the differentiability of f(x) = x|x|
Solution.

 x 2 , x  0
 f(x) =  2
 x , x  0

Differentiable at x = 0
Solved Example # 11
If f(x) is differentiable and g(x) is differentiable then prove that f(x) . g(x) will be differentiable.
Solution.
Given, f(x) is differentiable

TM

RESONANCE 23
lim f (a  h )  f ( a )
i.e. h0 = f(a)
h
g(x) is differentiable

lim g(a  h)  g(a )


i.e. h0 = g(a)
h
let p(x) = f(x) . g(x)

lim p(a  h)  p(a )


Now, h0
h

f (a  h).g(a  h)  f (a).g(a)
= hlim
0
h

f (a  h)g(a  h)  f (a  h).g(a )  f (a  h).g(a)  f (a ).g(a)


= hlim
0
h

 f (a  h) (g (a  h)  g(a)) pg(a)( f (a  h)  f (a)) 


= hlim
0 
 
 h h 

 g(a  h)  g(a ) f ( a  h)  f ( a ) 
= hlim
0 
f (a  h).  g(a). 
 h h 
= f(a) . g(a) + g(a) f(a) = p(a)
= p(a)
Hence p(x) is differentiable.

Solved Example # 12

x  3 x0
If f(x) =  2
 x  3x  2 x0
g(x) = f(|x|) + |f(x)| then comment on the continuity and differentiability of g(x) by drawing the graph of
f(|x|) and, |f(x)|.
Solution.

Graph of f(|x|) and |f(x)|

TM

RESONANCE 24
Continuity of g(x) : g(x) is continuous at the points where f(|x|) and |f(x)| both are continuous at x = 0
f(|x|) is continuous but |f(x)| is discontinuous.
 g(x) is discontinuous at x = 0 similarly g(x) is non differntiable at x = 0, 1, 2, (find the reason
yourself).

Solved Example # 13

lim f (a  2h)  f (a  3h) , evaluate the limit f(a) = 3


h0 h
Solution.

 lim f (a  2h)  f (a  3h)


h0 h

f (a  2h)  f (a  3h)
= hlim
0
.5
5h
= f(a) × 5
= 3 × 5 = 15

Self Practice Problem :

1. If f(x) and g(x) are differentiable then prove that f(x) ± g(x) will be differentiable.

f (2  h)  f (2  sinh)
2. If f(2) = 4 then find the value of hlim
0 h. sinh . tanh
Ans. 2/3

6. Finding Function Satisfying a given Condition:

Solved Example # 14
If f(x) is a function saitsfies the relation for all x, y  R, f(x + y) = f(x) + f(y) and if f(0) = 2 and function
is differentiable every where then find f(x).
Solution.

f ( x  h)  f ( x )
f(x) = hlim
0 h

f ( x )  f (h )  f ( x )  f ( 0 )
= hlim
0 h

TM

RESONANCE 25
f (h)  f (0)
= hlim
0 h
= f(0)
f = 2

  f(x) dx   2 dx
f(x) = 2x + c
 f(0) = 2.0 + c
as f(0) = 0
 c=0
 f(x) = 2x

Second Method :
Since f(x + y) = f(x) + f(y) is true for all values of x and y is independent of differentiating both sides w.r.t
x (here y is constant with respect to x).
f(x + y) = f(x)
put x = 0
f(y) = f(0)

  f (y) dy   2 dy
f(y) = 2y + c
 f(x) = 0 + c = 0
 c=0
 f(y) = 2y
 f(x) = 2x.

Solved Example # 15
f(x + y) = f(x) . f(y)  x, y  R and f(x) is a differentiable function and f(0) = 1 f(x)  0 for any x.
Solution.
f(x) is differentiable function
f ( x  h)  f ( x )
f(x) = hlim
0 h

f ( x ).f (h)  f ( x ).f (0)


= hlim
0 h

f ( x ).f (h)  f (0)


= hlim
0 h
= f(x) . f(0)
= f(x)
 f(x) = f(x)

f ( x )
  f ( x) dx = 1 dx
 n f(x) = x + c
 n 1 = 0 + c
 c=0
 n f(x) = x
 f(x) = ex Ans.

TM

RESONANCE 26
Solved Example # 16

xy f(x)  f (y)


f   =  x, y  R and f(0) = 1 and f(0) = – 1 then function is differentiable for are x then
 2  2
find f(x).
Solution.

 2x  2h   2x  0 
f f  
f(x) = hlim
0
 2   2 
h

f (2x )  f (2h) f (2x )  f (0)



= hlim
0
2 2
2h

f (2h)  f (0)
= hlim
0 2h
= f(0)
=–1
f(x) = – 1
interigrating,
f(x) = – x + c
 c = + 1 (as f(0) = 1)
 f(x) = – x + 1
=1–x Ans.

Self Practice Problem:

x
1. f  y  = f(x) – f(y)  x , y  R + and f(1) = 1 then show that f(x) = nx.
 

TM

RESONANCE 27
Part : (A) Only one correct option

e 1/ x 2  x  0
1. If f(x) =  then the value of f  (0) is:
 0 x0

(A) e (B) 1 (C) 0 (D) undefined

 x 2 e 2( x1) for 0  x  1
2. Given f(x) =  f(x) is differentiable at x = 1 provided:
a sgn ( x  1) cos (2x  2)  bx 2 for 1  x  2

(A) a =  1, b = 2 (B) a = 1, b =  2 (C) a =  3, b = 4 (D) a = 3, b =  4

3. If f(x) = p sinx + q.ex + r x3 and f(x) is differentiable at x = 0, then


(A) p = q = r = 0 (B) p = 0, q = 0, r  R
(C) q = 0, r = 0, p  R (D) p + q = 0, r  R


4. Let f(x) = sin x, g(x) = [x + 1] and g(f(x)) = h(x), where [.] is the greatest integer function. Then h  
2
is
(A) nonexistent (B) 1 (C) – 1 (D) none of these

5. A function f(x) = x [1 + (1/3) sin (lnx²)], x  0. [x] denotes the greatest integer less than or equal to x and
f(0) = 0. Then the function:
(A) is continuous at x = 0 (B) is monotonic
(C) is derivable at x = 0 (D) can not be defined for x < –1

6. The number of points at which the function f(x) = max. {a  x, a + x, b},   < x < , 0 < a < b cannot be
differentiable is:
(A) 1 (B) 2 (C) 3 (D) none

7. For what triplets of real numbers (a, b, c) with a 0 the function

x , x 1
f(x) =  2 is differentiable for all real x?
 ax  bx  c , otherwise

(A) {(a, 12a, a)  a  R, a 0 } (B) {(a, 12a, c)  a, c  R, a 0 }


(C) {(a, b, c)  a, b, c  R, a + b + c =1 } (D) {(a, 12a, 0)  a  R, a 0}

8. The functions defined by f(x) = max {x2, (x  1)2, 2x (1  x)}, 0  x  1


(A) is differentiable for all x
(B) is differentiable for all x except at one point
(C) is differentiable for all x except at two points
(D) is not differentiable at more than two points.


 
 2 sinx  sin 3 x  sinx  sin3 x 
 
9. Consider f(x) = 
 3
 3 , x 
 2 sinx  sin x  sinx  sin x  2
for x  (0, )

f(/2) = 3 where [ ] denotes the greatest integer function then,


(A) f is continuous & differentiable at x = /2
(B) f is continuous but not differentiable at x = /2
(C) f is neither continuous nor differentiable at x = /2
(D) none of these
TM
RESONANCE 28
   2  5 
   [x]  [x]  
  x  
 x a
 

10. 
Given f(x) = log a a [ x ]  [  x ]  
 1
 for x  0 ; a  1


  3 a x 
 
  
 0 for x  0

where [ ] represents the integral part function, then:


(A) f is continuous but not differentiable at x = 0
(B) f is continuous & differentiable at x = 0
(C) the differentiability of 'f' at x = 0 depends on the value of a
(D) f is continuous & differentiable at x = 0 and for a = e only.

maxf( t ) for 0  t  x for 0  x  1


11. Let f(x) = x3  x 2 + x + 1 and g(x) =  then:
 3  xx 2 for 1  x  2

(A) g(x) is continuous & derivable at x = 1


(B) g(x) is continuous but not derivable at x = 1
(C) g(x) is neither continuous nor derivable at x = 1
(D) g(x) is derivable but not continuous at x = 1

  4  x2 ,
max  1  x 2  ,  2  x  0
12. Let f (x) be defined in [ 2 , 2] by f (x) =   
min  4  x2 , 2
1 x  , 0  x  2
 
 
then f (x) :
(A) is continuous at all points
(B) is not continuous at more than one point .
(C) is not differentiable only at one point
(D) is not differentiable at more than one point

13. Suppose that f is a differentiable function with the property that f(x + y) = f(x) + f(y) + xy and

lim 1 f(h) = 3 then


h0 h
(A) f is a linear function (B) f(x) = 3x + x 2

x2
(C) f(x) = 3x + (D) none of these
2
 max f ( t ), 0  t  x, 0  x  1
14. Let f(x) = x – x2 and g(x) 
 sin x , x  1
Then in the interval [0, ]
(A) g(x) is everywhere continuous except at two points
(B) g(x) is everywhere differentiable except at two points
(C) g(x) is everywhere differentiable except at x = 1
(D) none of these

15. If f: R  R be a differentiable function, such that f(x + 2y) = f(x) + f(2y) + 4xy  x, y  R. then
(A) f(1) = f(0) + 1 (B) f(1) = f(0) – 1 (C) f(0) = f(1) + 2 (D) f(0) = f(1) – 2
TM
RESONANCE 29
16. Let f(x + y) = f(x) f(y) for all x and y. Suppose that f(3) = 3 and f(0) = 11 then f(3) is given by
(A) 22 (B) 44 (C) 28 (D) none of these

2f ( x )  3f (2x )  f ( 4 x )
17. Let f(x) be continuous x = 0 and f(0) = 4 the value of xlim is
0 x2
(A) 11 (B) 2 (C) 12 (D) none of these

18. If f (x) is differentiable everywhere, then:


2
(A) f  is differentiable everywhere (B) f is differentiable everywhere

(C) f f is not differentiable at some point (D) f + f is differentiable everywhere

1
19. Let f: R  R be any function and g (x) = . Then g is
f ( x)
(A) onto if f is onto (B) one-one if f is one-one
(C) continuous if f is continuous (D) differentiable if f is differentiable

20. Let f(x) = [n + p sin x], x  (0, p), n  Z, p is a prime number and [x] = then greatest integer less than
or equal to x. The number of points at which f(x) is not differentiable is
(A) p (B) p – 1 (C) 2p + 1 (D) 2p – 1

Part : (B) May have more than one options correct

21. If f(x) = a k | x |k , where a s are real constants, then f(x) is


i
k 0

(A) continuous at x = 0 for all a i (B) differentiable at x = 0 for all ai  R


(C) differentiable at x = 0 for all a2k + 1 = 0 (D) none of these

 x e1/ x e 1/ x 


  
for x  0
1. Let f(x) =  e1/ x  e1/ x . Check differentiability of f at x = 0.
 for x  0
 0

2. Examine the differentiability of f (x) = 1  e x 2 at x = 0.

 x 3 ; x 2  1
3. A function is defined as follows: f(x) =  . Draw the graph of the function & discuss continuity
x ; x 2  1

& differentiability at x = 1.

( x 1)2 sin 1  | x | , if x  1


4. Let f (x) =  x 1 be a real valued function. Find the points where f (x) is not
 1 , if x  1

differentiable.

TM
RESONANCE 30
 x m sin 1  ; x 0
5. Show that the function f (x) =  x is,
 0 ; x0

(i) differentiable at x = 0, if m > 1.


(ii) continuous but not differentiable at x = 0, if 0 < m < 1.
(iii) neither continuous nor differentiable, if m  0.

6. Draw a graph of the function, y = [x] + |1  x| 1  x  3. Determine the points, if any, where this
function is not differentiable, where [. ] denotes the greatest integer function.

f (1  cos x )  f (2)
7. If f(2) = 4 then Evaluate xlim .
0 tan 2 x

8. Let a function f : R  R be given by f(x + y) = f(x) f(y) for all x, y  R


and f(x)  0 for any x  R. If the function f (x) is differentiable at x = 0, show that f(x) = f(0) f(x) for all
x  R. Also, determine f(x).

9. Discuss the continuity & differentiability of the function f (x) = |sin x| + sin |x|, x R. Draw a rough
sketch of the graph of f (x) . Also comment on periodicity of function f(x).

  2 [ x]  
10. Given f(x) = cos1  sgn  3x  [ x ]   where sgn ( ) denotes the signum function & [ ] denotes the greatest
  

integer function. Discuss the continuity & differentiability of f (x) at x = ± 1.

11. If f (x) = x2  2 |x| then test the derivability of g (x) in the interval [ 2, 3], where

min f ( t );  2  t  x   2  x  0
g (x) = 
 max f( t );0  t  x 0x3

12. Discuss the continuity on 0  x  1 & differentiability at x = 0 for the function.

1 1 1
f(x) = x sin sin
1 where x  0, x  r & f(0) = f (1/r) = 0, r = 1, 2, 3,.......
x
x sin
x

13. Let R be the set of real numbers and f: RR be such that for all x & y in R
f(x)  f(y)  x  y3. Prove that f(x) is constant.

 xy 
14. Let f: R  (, ) be a derivable function such that f(x) + f(y) = f   , xy < 1.
 1  xy 

If f(1) =

& Limit f( x ) = 2, find f(x).
x 0
2 x
15. The function f is defined by y = f(x). Where x = 2t  | t | y = t² + t | t | t  R. Draw the graph of f for
the interval 1  x  1. Also discuss its continuity & differentiability at x = 0.

16. Discuss the continuity and differentiability of f(x) = [x] + {x} 2 and also draw its graph. Where
[. ] and {.} denotes the greatest integer function and fractional part function respectively.

17. If f(x) = 1 + x  1, 1  x


 3; g(x) = 2  x + 1,  2  x  2, then calculate (fog) (x) & (gof) (x).
Draw their graph. Discuss the continuity of (fog) (x) at x = 1 & the differentiability of (gof) (x) at x = 1.

TM
RESONANCE 31
10. f is discontinuous at x = 2 and continuous at all other
Exercise # 1
point f is not differentiable at x = 1, 3/2 & 2 and
1. C 2. A 3. D 4. A 5. A 6. B 7. A differentiable at all other points.

8. C 9. A 10. B 11. C 12. D 13. C 14. C 11. not derivable at x = 0 and 2

15. D 16. D 17. C 18. B 19. B 20. D 12. continuous in 0  x  1 & not differentiable at x = 0

21. AC 14. f(x) = 2 tan-1 x

15. f(x) = 2x² for 0  x  1 & f(x) = 0


Exercise # 2
for – 1  x < 0, f is differentiable & hence

1. not differentiable at x = 0 2. not diff. at x = 0 continuous at x = 0

3. f is continuous but not differentiable at x = 1 16. Continuous everywhere but not differentiable at
integral points.
4. f (x) is differentiable except at x = 0
17. (fog)(x) = x+1 for  2  x   1,  (x + 1)
6. f is not derivable at all integral values in 1 < x  3 for  1 < x  0 & = x  1 for 0 < x  2.
(fog)(x) is continuous at x = 1,
7. – 2 8. f(x) = exf(0)  x  R
(gof)(x) = x+1 for1  x  1 & 3  x for
9. f (x) is continuous but not differentiable at x = 0, f(x) 1< x  3. (gof) (x) is not differentiable at x = 1
is not periodic.

TM
RESONANCE 32
Tangent & Normal

A. Derivative as rate of change


If the quantity y varies with respect to another quantity x

[The universe] dy
satisfying some relation y = f(x), then f(x) or represents
dx
cannot be read until
rate of change of y with respect to x.
we have learnt the
language and Example : The volume of a cube is increasing at rate of 7 cm 3/sec.
become familiar with How fast is the surface area increasing when the length
of an edge is 4 cm?
the characters in Solution. Let at some time t, the length of edge is x cm.
which it is written. dv dx dv
v = x3  = 3x2 (but = 7)
It is written in dt dt dt
mathematical dx 7
language, and the  = cm/sec.
dt 3x 2
lettersaretriangles, Now s = 6x 2
circles and other ds dx ds 7 28
= 12x  = 12x. 2 =
geometrical figures, dt dt dt 3x x
without which means ds
when x = 4 cm = 7 cm 2/sec.
it is humanly dt
impossible to Example : Sand is pouring from pipe at the rate of 12 cm 3/s. The
comprehend a single falling sand forms a cone on the ground in such a way
that the height of the cone is always one - sixth of radius
word. of base. How fast is the height of the sand cone
increasing when height is 4 cm?
1
– Gal i l eoGal i l ei Solution. v= r 2 h
3
r
but h=
6
1
 v=  (6h)2 h
3
 v = 12 h3
dv dh
= 36 h2.
dt dt

dv
when, = 12 cm 3/s and h = 4 cm
dt

dh 12 1
= 2 = cm/sec.
dt 36.( 4) 48 
TM
RESONANCE 1
Self practice problem :

1. Radius of a circle is increasing at rate of 3 cm/sec. Find the rate at which the area of circle is increasing
at the instant when radius is 10 cm.
Ans. 60 cm 2/sec

2. A ladder of length 5 m is leaning against a wall. The bottom of ladder is being pulled along the ground
away from wall at rate of 2cm/sec. How fast is the top part of ladder sliding on the wall when foot of
ladder is 4 m away from wall.
8
Ans. cm/sec
3
3. Water is dripping out of a conical funnel of semi-vertical angle 45° at rate of 2cm 3/s. Find the rate at
which slant height of water is decreasing when the height of water is 2 cm.

1
Ans. cm/sec.
2

4. A hot air balloon rising straight up from a level field is tracked by a range finder 500 ft from the lift-off
point. At the moment the range finder's elevation angle is /4, the angle is increasing at the rate of 0.14
rad/min. How fast is the balloon rising at that moment.
Ans. 140 ft/min.

B Equation of Tangent and Normal


dy
= f(x 1) denotes the slope of tangent at point (x1, y1) on the curve y = f(x). Hence the equation
dx ( x1, y1 )

of tangent at (x1, y1) is given by


(y – y1) = f(x 1) (x – x 1)
Also, since normal is a line perpendicular to tangent at (x1 , y1) so its equation is given by
1
(y – y1) = – f ( x ) (x – x1)
1

Example: Find equation of tangent to y = ex at x = 0.

Solution At x = 0  y = e0 = 1
Hence point of tangent is (0, 1)

dy dy
= ex  dx =1
dx x 0

Hence equation of tangent is


1 (x – 0) = (y – 1)
 y=x+1
1
Example : Find the equation of all straight lines which are tangent to curve y = and which are
x 1
parallel to the line x + y = 0.
Solution : Suppose the tangent is at (x 1, y1) and it has slope – 1.
dy
 = – 1.
dx ( x1 , y1 )

1
 – = – 1.
( x1  1)2
 x1 = 0 or 2
TM
RESONANCE 2
 y1 = – 1 or 1
Hence tangent at (0, – 1) and (2, 1) are the required lines with equations
– 1(x – 0) = (y + 1) and – 1 (x – 2) = (y – 1)
 x+y+1=0 and y+x=3

Example : Find equation of normal to the curve y = |x 2 – | x | | at x = – 2.


Solution In the neighborhood of x = – 2, y = x 2 + x.
Hence the point of contact is (– 2, 2)
dy dy
= 2x + 1  = – 3.
dx dx x  2

1
So the slope of normal at (– 2, 2) is .
3
Hence equation of normal is
1
(x + 2) = y – 2.
3
 3y = x + 8.

Example : Prove that sum of intercepts of the tangent at any point to the curve x + y = a on the
coordinate is constant.
Solution : Let P(x 1, y1) be a variable point on the curve x + y = a

y1
 equation of tangent at point p is – (x – x 1) = (y – y1)
x1

x y
 – x1 + x1 = y1 – y1

x y
 x1 + y1 = x1 + y1

x y
 x1 + y1 = a ( x1  y 1  a )

Hence point A is  ax , 0 and coordinates of point B is (0,


1
ay 1 ) . Sum of interceptss

= a ( x1 + y1 ) = a . a = a.

C. Tangent from an External Point


Given a point P(a, b) which does not lie on the curve y = f(x), then the equation of possible tangents to
the curve y = f(x), passing through (a, b) can be found by solving for the point of contact Q.

TM
RESONANCE 3
Example : Find the equation of all possible normal to the parabola x 2 = 4y drawn from point (1, 2).
 h2 
Solution Let point Q be  h, 
 4 

Now, m PQ = slope of normal at Q.


dx 2
Slope of normal = – =–
dy x h
h
2
h
2
4 2
 =–
h 1 h

h3
 – 2h = – 2h + 2
4
 h3 = 8  h=2
Hence coordinates of point Q is (2, 1) and so equation of required normal becomes x + y = 3.

Note : The equation gives only one real value of h, hence there is only one point of contact implying
that only one real normal is possible from point (1, 2).

Example : Find value of c such that line joining points (0, 3) and (5, – 2) becomes tangent to curve
c
y= .
x 1
Solution. Equation of line joining A & B is x + y = 3

Solving this line and curve we get


c
3–x=  x2 – 2x + (c – 3) = 0 ......(i)
x 1
For tangency, roots of this equation must be coincident. Hence D = 0
 4 = 4 (c – 3)  c=4

Note : If a line touches a curve then on solving the equation of line and tangent we get at least two
repeated roots corresponding to point of contact.
Putting c = 4, equation (i) becomes
x 2 – 2x + 1 = 0  x=1
Hence point of contact becomes (1, 2).

Example : Tangent at P(2, 8) on the curve y = x 3 meets the curve again at Q. Find coordinates of Q.

Solution. Equation of tangent at (2, 8) is


y = 12x – 16
Solving this with y = x 3
x 3 – 12x + 16 = 0

this cubic must give all points of intersection of line and curve y = x 3 i.e., point P and Q.
But, since line is tangent at P so x = 2 will be a repeated root of equation x 3 – 12x + 16 = 0 and
another root will be x = h. Using theory of equations
sum of roots  2+2+h=0  h=–4
Hence coordinates of Q are (– 4, – 64)

TM
RESONANCE 4
Self Practice Problems :

1. Find the slope of the normal to the curve x = 1 – a sin , y = b cos 2  at  = .
2
a
Ans. –
2b

2. Find the equation of the tangent and normal to the given curves at the given points.
(i) y = x 4 – 6x 3 + 13x 2 – 10x + 5 at (1, 3)

x3
(ii) y2 = at (2, – 2).
4x
Ans. (i) Tangent : y = 2x + 1, Normal :x + 2y = 7
(ii) Tangent : 2x + y = 2, Normal :x – 2y = 6

3. Prove that area of the triangle formed by any tangent to the curve xy = c 2 and coordinate axes is
constant.

4. How many tangents are possible from origin on the curve y = (x + 1)3. Also find the equation of these
tangents.
Ans. y = 0, 4y = 27x.

x9
5. Find the equation of tangent to the hyperbola y = which passes through (0, 0) origin
x5
Ans. x + y = 0; 25y + x= 0

D. Length of Tangent, Normal


Let P (h, k) be any point on curve y = f(x). Let tangent drawn at point P meets x-axis at T & normal at
point P meets x-axis at N. Then the length PT is called the length of tangent and PN is called length of
normal.

Projection of segment PT on x-axis, TM, is called the subtangent and similarly projection of line segment
PN on x axis is called sub normal.

dy
Let m = dx = slope of tangent.
h, k

Hence equation of tangent is m (x – h) = (y – k)


k
putting y = 0 we get x - intercept of tangent x = h –
m
similarly the x-intercept of normal is x = h + km
Now, length PT, PN etc can be easily evaluated using distance formula

TM
RESONANCE 5
1
(i) PT = k 1  = Length of Tangent
m2

2
(ii) PN = k 1  m = Length of Normal

k
(iii) TM = = Length of subtangent
m
(iv) MN = |km| = Length of subnormal

Example: Find the length of tangent for the curve y = x 3 + 3x2 + 4x – 1 at point x = 0.

dy
Solution. Here m= & k = y(0)  k=–1
dx x 0

dy
= 3x 2 + 6x + 4  m=4
dx

1 1 17
= k 1  2  =  1 1 =
m 16 4

Example: Prove that for the curve y = bex/a, the length of subtangent at any point is always constant.
Solution y = bex/a Let the point be (x1, y1)

dy b.e x1 / a y1
 m= = 
dx x1 a a

y1 y1
Now, length of subtangent = = =a
m y1 / a
Hence proved.

Example : For the curve y = a n (x 2 – a2) show that sum of lengths of tangent & subtangent at any point
is proportional to coordinates of point of tangency.
Solution. Let point of tangency be (x 1, y1)
dy 2ax1
m= = 2
dx x1 x 1  a2

1 y1
tangent + subtangent = y1 1  +
m2 m

( x 21  a 2 ) 2 2
y1( x1  a 2 )
= y1 1 2 +
4a 2 x 1 2ax1

4 2 2
x1  a 4  2a 2 x1 y1( x1  a 2 )
= y1 +
2ax1 2ax1

2 2
y1( x1  a 2 ) y1( x1  a 2 )
= +
2ax1 2ax1

2
y1( x1 ) x1 y1
= = Hence proved.
2ax1 2a
TM
RESONANCE 6
E Angle between the curves
Angle between two intersecting curves is defined as the acute angle between their tangents or the normals
at the point of intersection of two curves.

m1  m 2
tan  =
1  m1m 2

where m1 & m2 are the slopes of tangents at the intersection point (x1, y1). Note carefully that

(i) The curves must intersect for the angle between them to be defined. This can be ensured by finding their
point of intersection or graphically.

(ii) If the curves intersect at more than one point then angle between curves is written with references to the point
of intersection.

(iii) Two curves are said to be orthogonal if angle between them at each point of intersection is right angle.
i.e. m1 m2 = – 1.

Example : Find angle between y2 = 4x and x2 = 4y. Are these two curves orthogonal?
Solution. y2 = 4x and x2 = 4y intersect at point (0, 0) and (4, 4)

C1 : y2 = 4x C2 : x2 = 4y

dy 2 dy x
= y =
dx dx 2

dy dy
dx = dx =0
0, 0 0, 0

Hence tan  = 90° at point (0, 0)

dy 1 dy
dx = dx =2
( 4, 4 ) 2 ( 4, 4 )

1
2
2 3
tan = 1 =
1  2. 4
2
Two curves are not orthogonal because angle at (4, 4) is not 90°.

Example: Find the angle between curves y2 = 4x and y = e–x/2


Solution. Let the curves intersect at point (x1, y1)

dy 2
for y2 = 4x dx = y
( x1, y1 ) 1

dy 1 x / 2 y1
and for y = e–x/2 =– =–
2 e
1
dx ( x1, y1 ) 2
 m1 m2 = – 1 Hence  = 90°

Note : here that we have not actually found the intersection point but geometrically we can see
that the curves intersect.
TM
RESONANCE 7
Example : Find possible values of p such that the equation px2 = nx has exactly one solution.

Solution. Two curves must intersect at only one point. Hence

(i) (ii)

I. if p  0 then only one solution (see graph)


II. if p > 0
then the two curves must only touch each other
i.e. tangent at y = px2 and y = nx must have same slope at point (x1, y1)
1
 2px1 = x
1

1
 x 12 = .........(i)
2p

 1 
also y1 = px12  y1 = p  
 2p 

1
 y1 = ........(ii)
2

1
and y1 = nx1  = nx1
2
 x1 = e1/2 .........(iii)
1 1 1
Hence x12 =  e=  p=
2p 2p 2e

 1
Hence possible values of p are (– , 0]   
 2e 

Self Practice Problems :


1. For the curve x m + n = am – n y2n, where a is a positive constant and m, n are positive integers, prove that
the m th power of subtangent varies as nth power of subnormal.

a a a 2  x 2
2. Prove that the segment of the tangent to the curve y = n – a 2  x 2 contained
2 a a 2  x 2

between the y-axis & the point of tangency has a constant length .
3. A curve is given by the equations x = at2 & y = at3 . A variable pair of perpendicular lines through the
origin 'O' meet the curve at P & Q . Show that the locus of the point of intersection of the tangents at
P & Q is 4y2 = 3ax - a2 .
4. Find the length of the subnormal to the curve y2 = x3 at the point (4, 8).
Ans. 24
5. Find the angle of intersection of the following curves:
(i) y = x2 & 6y = 7  x3 at (1, 1)

x2 y2
(ii) x 2 – y2 = 5 &  = 1.
18 8
Ans. (i) /2 (ii) /2
TM
RESONANCE 8
F. Shortest distance between two curves
Shortest distance between two non-intersecting curves always along the common normal.
(Wherever defined)

Example: Find the shortest distance between the line y = x – 2 and the parabola y = x2 + 3x + 2.

Solution. Let P(x1, y1) be a point closest to the line y = x – 2

dy
then dx = slope of line
( x1 , y1 )

 2x1 + 3 = 1
 x1 = – 1
 y1 = 0
Hence point (– 1, 0) is the closest and its perpendicular distance from the line y = x – 2 will give the
shortest distance
3
 p= .
2

TM
RESONANCE 9
PART - (A) Only one correct option

1. Water is poured into an inverted conical vessel of which the radius of the base is 2 m and height 4 m, at
the rate of 77 litre/minute. The rate at which the water level is rising at the instant when the depth is
70 cm is: (use  = 22/7)
(A) 10 cm/min (B) 20 cm/min (C) 40 cm/min (D) none

2. The area of the triangle formed by the positive x-axis and the normal and the tangent to the circle
x 2 + y2 = 4 at (1, 3 ) is

(A) 3 3 sq. units (B) 2 3 sq. units (C) 4 3 sq. units (D) 3 sq. units


3. The line which is parallel to x-axis and crosses the curve y = x at an angle of is
4
(A) y =  1/2 (B) x = 1/2 (C) y = 1/4 (D) y = 1/2

4. If at any point on a curve the subtangent and subnormal are equal, then the tangent is equal to

(A) oridinate (B) 2 ordinate (C) 2 (ordinate ) (D) none of these

5. If curve y = 1 – ax2 and y = x2 intersect orthogonally then the value of a is


(A) 1/2 (B) 1/3 (C) 2 (D) 3

(length of normal)2
6. For a curve is equal to
(length of tan gent )2
(A) (subnormal) / (subtangent) (B) (subtangent) / (subnormal)
(C) subnormal/(subtangent)2 (D) none of these

2 3
7. If the tangent at each point of the curve y = x – 2ax 2 + 2x + 5 makes an acute angle with the positive
3
direction of x-axis, then
(A) a  1 (B) – 1  a  1 (C) a  – 1 (D) none of these

sinx 2
8. Equation of normal drawn to the graph of the function defined as f(x) = , x  0 and f(0) = 0 at the
x
origin is:
(A) x + y = 0 (B) x  y = 0 (C) y = 0 (D) x = 0

 x
9. All points on the curve y2 = 4a  x  a sin  at which the tangents are parallel to the axis of x, lie on a
 a 
(A) circle (B) parabola (C) line (D) none of these

10. The point(s) of intersection of the tangents drawn to the curve x 2y = 1  y at the points where it is
intersected by the curve xy = 1  y is/are given by:
(A) (0,  1) (B) (0, 1) (C) (1, 1) (D) none of these

11. The ordinate of y = (a/2) (ex/a + e-x/a) is the geometric mean of the length of the normal and the quantity:
(A) a/2 (B) a (C) e (D) none of these

TM
RESONANCE 10
12. The curves x3 + p xy2 =  2 and 3 x 2y  y3 = 2 are orthogonal for:
(A) p = 3 (B) p =  3 (C) no value of p (D) p = ± 3

13. If the area of the triangle included between the axes and any tangent to the curve xn y = an is constant,
then n is equal to

3 1
(A) 1 (B) 2 (C) (D)
2 2

14. A curve with equation of the form y = ax4 + bx 3 + cx + d has zero gradient at the point (0, 1) and also
touches the x  axis at the point ( 1, 0) then the values of x for which the curve has a negative gradient
are:
(A) x >  1 (B) x < 1 (C) x <  1 (D)  1  x  1

15. If the tangent at P of the curve y2 = x 3 intersects the curve again at Q and the straight lines OP, OQ
make angles ,  with the x-axis, where 'O' is the origin, then tan /tan  has the value equal to:
(A)  1 (B)  2 (C) 2 (D) 2

PART - (B) One or more than one correct options

16. Consider the curve f(x) = x1/3 , then


(A) the equation of tangent at (0, 0) is x = 0
(B) the equation of normal at (0, 0) is y = 0
(C) normal to the curve does not exist at (0, 0)
(D) f(x) and its inverse meet at exactly 3 points.

n n
x y
17. The equation of normal to the curve     = 2 (n  N) at the point with abscissa equal to 'a' can
a b
be:
(A) ax + by = a2  b2 (B) ax + by = a2 + b2
(C) ax  by = a2  b2 (D) bx  ay = a2  b2

18. If the line, ax + by + c = 0 is a normal to the curve xy = 2, then:


(A) a < 0, b > 0 (B) a > 0, b < 0 (C) a > 0, b > 0 (D) a < 0, b < 0

19. In the curve x = t2 + 3t – 8, y = 2t2 – 2t – 5, at point (2, – 1)


(A) length of subtangent is 7/6. (B) slope of tangent = 6/7

(C) length of tangent = (85 ) / 6 (D) none of these

20. If y = f(x) be the equation of a parabola which is touched by the line y = x at the point where x = 1. Then
(A) f(1) = 1 (B) f(0) = f(1)
(C) 2f(0) = 1 – f(0) (D) f(0) + f(0) + f(0) = 1

21. If the tangent to the curve 2y3 = ax 2 + x 3 at the point (a, a) cuts off intercepts ,  on co-ordinate axes,
where 2 +  2 = 61, then the value of 'a' is equal to:
(A) 20 (B) 25 (C) 30 (D)  30

22. The curves ax 2 + by2 = 1 and Ax 2 + By2 = 1 intersect orthogonally, then

1 1 1 1 1 1 1 1 1 1 1 1 1 1 1 1
(A)    (B)    (C)    (D)   
a A b B a A b B a b B A a b A B
TM
RESONANCE 11
1. Find the parameters a, b, c if the curve y = a x2 + b x + c is to pass through the point (1, 2) and is to be
tangent to the line y = x at the origin.

2. If the tangent at (1, 1) on y2 = x(2 – x)2 meets the curve again at P, then find coordinates of P

3. If the relation between subnormal SN and subtangent ST at any point S on the curve by2 = (x + a)3 is

p
p(SN) = q (ST)2, then find value of in terms of b and a.
q

4.  
In the curve x = a cos t  log tan 1 t , y = a sin t, show that the portion of the tangent between the
2
point of contact and the x  axis is of constant length.

5. Find the angle of intersection of the following curves:


(i) 2 y2 = x 3 & y2 = 32 x (ii) y = 2sin2x and y = cos2x at x = /6
(iii) y = 4  x2 & y = x2

6. The length x of rectangle is decreasing at a rate of 3 cm/min and the width y is increasing at the rate
of 2 cm/min. when x = 10 cm and y = 6 cm, find the rates of changes of (i) the perimeter, and (ii) the
area of the ractangle.

7. A particle moves along the curve 6 y = x 3 + 2 . Find the points on the curve at which the
y coordinate is changing 8 times as fast as the x coordinate.

x2 y2
8. Prove that the straight line, x cos  + y sin  = p will be a tangent to the curve + 2 = 1,
a2 b
if p2 = a2 cos 2  + b2 sin2 .

9. Show that the normal to any point of the curve x = a (cos t + t sin t), y = a (sin t  t cos t) is at a
constant distance from the origin.

2/3 2/3 2/3


x2 y2
10. Show that the condition, that the curves x +y =c and + 2 = 1 may touch,
a2 b
if c = a + b.

11. Find the equation of axes of the conic 5x2 + 4xy + 2y2 = 1.

12. Find the abscissa of the point on the curve, x y = (c + x)2 the normal at which cuts off numerically equal
intercepts from the axes of co-ordinates.

13. In the curve x a yb = Ka+b, prove that the portion of the tangent intercepted between the coordinate axes
is divided at its point of contact into segments which are in a constant ratio. (All the constants being
positive).

TM
RESONANCE 12
14. The tangent to curve y = x – x3 at point P meets the curve again at Q. Prove that one point of trisection
of PQ lies on y-axis. Find locus of other point of trisection

15. A straight line is drawn through the origin and parallel to the tangent to a curve

 a  a2  y2 
x a2  y 2  
= n  y  at an arbitary point M. Show that the locus of the point P of
a  

intersection of the straight line & the straight line parallel to the x-axis & passing through the point M
is x2 + y2 = a2 .

16. Find the possible values of a such that the inequality 3 – x2 > |x – a| has atleast one negative solution.

17. Consider the family of circles x2 + y2 = r2, 2 < r < 5. In the first quadrant, the common tangents to a
circle of this family and the ellipse 4x2 + 25y2 = 100 meets the co-ordinate axes at A and B, then find
the equation of the locus of the mid-point of AB. [IIT – 1999]

18. Let T 1, T 2 be two tangents drawn from (–2, 0) onto the circle C : x 2 + y2 = 1. Determine the circles
touching C and having T1, T2 as their pair of tangents. Further; find the equations of all possible common
tangents to these circles, when taken two at a time. [IIT – 1999]

19. An inverted cone of height H and radius R is pointed at bottom. It is filled with a volatile liquid completely.
If the rate of evaporation is directly proportional to the surface area of the liquid in contact with air
(constant of proportionality k > 0). Find the time in which whole liquid evaporates. [IIT – 2003, 4]

20. If |f(x1) – f(x 2)| < (x 1 – x2)2 , for all x 1, x 2  R. Find the equation of tengent to the curve y = f(x) at the point
(1, 2). [IIT – 2005, 2]

EXERCISE # 1
1. B 2. B 3. D 4. B (ii) /3

4 2
5. B 6. A 7. B 8. A (iii) tan1  
 7  at  
2 ,2 ,  2 ,2 
 
9. B 10. B 11. B 12. B

13. A 14. C 15. B 16. ABD 6. (i) – 2 cm/min (ii) 2 cm 2/min.

17. AC 18. AB 19. ABC 20. AC 7. (4 , 11) & ( 4,  31/3)

21. CD 22. BD c
11. 2x + y = 0, x = 2y 12. ±
2
EXERCISE # 2
 13 
14. y = x – 5x 3 16. a    , 3
1. a = 1, b = 1, c = 0 2. (9/4, 3/8)  4 
8b H
3. 17. 25y2 + 4x 2 = 4x 2y2 19. t =
27 k
  1
5. (i) at (0, 0); tan 1   at (8, 16), (8,  16) 20. y = 2
2  2
TM
RESONANCE 13
Monotonocity

A. Monotonocity about a point

1. A function f(x) is called an increasing function at point x = a.


If in a sufficiently small neighbourhood around x = a.
f(a – h) < f(a) < f(a + h)
It may be
said that
conceptions of
differential
quotient and
integral which
in their origin
2. A function f(x) is called a decreasing function at point x = a if in
certainly go a sufficiently small neighbourhood around x = a.
f(a – h) > f(a) > f(a + h)
back to
archimedes,
were intro-
duced into the
science by the
investigations
of Kepler,
Descartes, Note :
If x = a is a boundary point then use the appropriate one sided
Fermats, Wallis
inequality to test monotonocity of f(x).

.........SophusLi e

TM
RESONANCE 14
Example : Which of the following functions is increasing, decreasing or neither increasing nor decreasing
at x = a.

(i) (ii)

(iii) (iv)

3. Test for increasing and decreasing functions at a point


(i) If f(a) > 0 then f(x) is increasing at x = a.
(ii) If f(a) < 0 then f(x) is decreasing at x = a.
(iii) If f(a) = 0 then examine the sign of f(a+) and f(a–).
(a) If f(a+) > 0 and f(a–) > 0 then increasing
(b) If f(a+) < 0 and f(a–) < 0 then decreasing
(c) otherwise neither increasing nor decreasing.

Example : Let f(x) = x 3 – 3x + 2. Examine the nature of function at points x = 0, 1, 2.

Solution : f(x) = x 3 – 3x + 2
f(x) = 3(x2 – 1)
(i) f(0) = – 3  decreasing at x = 0
(ii) f(1) = 0
also, f(1+) = positive and f(1–) = negative
 neither increasing nor decreasing at x = 1.
(iii) f(2) = 9  increasing at x = 2

Note : Above rule is applicable only for functions that are differentiable at x = a.

B. Monotonocity over an interval


1. A function f(x) is said to be monotonically increasing for all such interval (a, b) where f(x)  0 and
equality may hold only for discreet values of x. i.e. f(x) does not identically become zero for x  (a, b)
or any sub interval.

2. f(x) is said to be monotonically decreasing for all such interval (a, b) where f(x)  0 and equality may
hold only for discrete values of x.

Note : By discrete, points, we mean that points where f(x) = 0 don’t form an interval

TM
RESONANCE 15
For example.
Let f(x) = x 3
f(x) = 3x2
f(x) > 0 every where except at x = 0. Hence f(x) will be considered monotonically increasing function for
x  R. also,

Let f(x) = x – sinx


f(x) = 1 – cosx
Now, f(x) > 0 every where except at x = 0, ± 2, ± 4 etc. but all these points are discrete and donot
form an interval hence we can conclude that f(x) is monotonically increasing for x  R. In fact we can
also see it graphically.

Let us consider another function whose graph is shown for x  (a, b).

Here also f(x)  0 for all x  (a, b) but note that in this case equality of f(x) = 0 holds for all x  (c, d)
and (e, b). Here f(x) become identically zero and hence the given function cannot be assumed to be
monotonically increasing for x  (a, b).

Example : Find the interval where f(x) = x 3 – 3x + 2 is monotonically increasing.

Solution. f(x) = x 3 – 3x + 2
f(x) = 3(x2 – 1)
f(x) = 3(x – 1) (x + 1)

for M.. f(x)  0  3(x – 1) (x + 1) 0


 x  [ – , – 1]  [1, )
Note :
(i) A function is said to be monotonic if it's either increasing or decreasing.
(ii) The points for which f (x) is equal to zero or doesn't exist are called critical points. Here it
should also be noted that critical points are the interior points of an interval.
(iii) The stationary points are the points where f (x) = 0 in the domain.

TM
RESONANCE 16
Example : Find the intervals of monotonicity of following functions.
(i) f(x) = x 2 (x – 2)2
(ii) f(x) = x n x
(iii) f(x) = sinx + cosx ; x  [0, 2]
Solution. (i) f(x) = x 2 (x – 2)2
f(x) = 4x (x – 1) (x – 2)
observing the sign change of f(x)

Hence M.. for x  [0, 1]  [2, )


and M.D. for x  (– , 0]  [1, 2]
Note : Closed bracket can be used for both M.. as well as M.D. In above example x = 1 is
boundary point for x  [0, 1] and since f(1) > f(1 – h). So we can say f(x) is M.. at
x = 1 for x  [0, 1]. However also note that for the interval x  [1, 2] again x = 1
becomes a boundary point and f(1) > f(1 + h). Hence f(x) is M.D. at x = 1 for x  [1, 2]
(ii) f(x) = x n x
f(x) = 1 + n x
1
f(x)  0  n x  – 1  x
e

1   1
 M.. for x   ,   and M.D for x   0 ,  .
e   e
(iii) f(x) = sinx + cosx
f(x) = cosx – sinx
for M.. f(x)  0  cosx  sinx

   5 
 x  0 ,    , 2
 4  4 

  5 
therefore M.D. for x   , 
4 4 
Exercise

1. For each of the following graph comment whether f(x) is increasing or decreasing or neither increasing
nor decreasing at x = a.

(i) (ii) (iii)

(iv) (vi)

Ans. (i) neither M.. nor M.D. (ii) M.D.


(iii) M.D (iv) M..

TM
RESONANCE 17
2. Let f(x) = x 3 – 3x 2 + 3x + 4, comment on the monotonic behaviour of f(x) at (i) x = 0 (ii) x = 1.
Ans. M.. both at x = 0 and x = 1.

 x 0  x 1
3. Draw the graph of function f(x) =  . Graphically comment on the monotonic behaviour of
[ x ] 1  x  2
f(x) at x = 0, 1, 2. Is f(x) M.. for x  [0, 2] ?
Ans. M.. at x = 0, 2 ; neither M.. nor M.D. at x = 1. No, f(x) is not M.. for x  [0, 2].

4. Find the intervals of monotonicity of the following functions.


(i) f(x) = – x 3 + 6x 2 – 9x – 2 Ans.  in [1, 3] ; D in (– , 1]  (3, )
1
(ii) f(x) = x + Ans.  in (– , – 2]  [0, ) ; D in [– 2, – 1)  (–1, 0]
x 1

2  1   1
(iii) f(x) = x . e x  x Ans.  in  , 1 ; D in   ,    [1, )
 2   2
(iv) f(x) = x – cosx Ans. I for x  R

C. Classification of functions
Depending on the monotonic behaviour, functions can be classified into following cases.

1. Increasing functions 2. Non decreasing functions

3. Decreasing functions 4. Non-increasing functions

However note that this classification is not complete and there may be function which cannot be
classified into any of the above cases for some interval (a, b).

Example : f(x) = [x] is a step up function. Is it a monotonically increasing function for x  R.

Solution. No, f(x) = [x] is not M.I. for x  R rather, it is a non-decreasing function as illustrated by its
graph.

TM
RESONANCE 18
Example : If f(x) = sin4x + cos 4x + bx + c, then find possible values of b and c such that f(x) is monotonic
for all x  R
Solution. f(x) = sin4x + cos 4x + bx + c
f(x) = 4 sin3x cosx – 4cos 3x sinx + b = – sin4x + b.
(i) for M.I. f(x)  0 for all x  R
b  sin4x for all x  R  b1
(ii) for M.D. f(x)  0 for all x  R
b sin4x for all x  R  b–1
Hence for f(x) to be monotonic b  (– , – 1]  (1, ) and c  R.

Example : Find possible values of a such that f(x) = e 2x – (a + 1) e x + 2x is monotonically increasing for
xR

Solution. f(x) = e2x – (a + 1) ex + 2x


f(x) = 2e2x – (a + 1) ex + 2
Now, 2e2x – (a + 1) ex + 2  0 for all xR
 x 1 
 2  e  x  – (a + 1)  0 for all xR
 e 

 x 1 
(a + 1) < 2  e  x  for all xR
 e 
 1 
 a+14  ex  has min imum value 2 
 ex 
 a3
Aliter
2e2x – (a + 1) ex + 2  0 for all xR
putting ex = t ; t  (0, )
2t2 – (a + 1) t + 2  0 for all t  (0, )
Hence either
(i) D0
 (a + 1)2 – 4  0
 (a + 5) (a – 3) 0
 a  [– 5, 3]
or
(ii) both roots are negative

b
D0 & – <0 & f(0)  0
2a
a 1
 a  (– , – 5]  [3, ) & <0 & 20
4
 a  (– , – 5]  [3, ) & a<–1 & aR
 a  (– , – 5]
Taking union of (i) and (ii), we get a  (– , 3].
Exercise
1. Let f(x) = x – tan–1x. Prove that f(x) is monotonically increasing for x  R.

2. If f(x) = 2ex – ae–x + (2a + 1) x – 3 monotonically increases for  x  R, then find range of values of a
Ans. a  0
3. Let f(x) = e2x – aex + 1. Prove that f(x) cannot be monotonically decreasing for  x  R for any value of
'a'.
4. Find range of values of 'a' such that f(x) = sin2x – 8(a + 1) sinx + (40 – 10)x is monotonically decreasing
xR
Ans. a  [ – 4, 0]

5. If f(x) = x3 + (a + 2)x2 + 5ax + 5 is a one-one function then find values of a.


Ans. a  [1, 4]
TM
RESONANCE 19
D. Proving Inequalities
Comparision of two functions f(x) and g(x) can be done by analysing their monotonic behavior or graph.

 
Example : For x   0,  prove that sin x < x < tan x
 2
Solution. Let f(x) = x – sin x  f(x) = 1 – cos x
 
f(x) > 0 for x   0, 
 2
 f(x) is M.I.  f(x) > f(0)
 x – sin x > 0  x > sin x
Similarly consider another function g(x) = x – tan x  g(x) = 1 – sec 2x
 
g(x) < 0 for x   0,   g(x) is M.D.
 2
Hence g(x) < g(0)
x – tan x < 0  x < tan x
sin x < x < tan x Hence proved

x3 x3  tan 1 x 
Example : For x  (0, 1) prove that x – –1
< tan x < x – lim
hence or otherwise find x 0  x 
3 6  

x3
Solution. Let f(x) = x – – tan–1x
3
1
f(x) = 1 – x2 –
1 x2

x4
f(x) = –
1 x2
f(x) < 0 for x  (0, 1)  f(x) is M.D.
 f(x) < f(0)
x3
 x– – tan–1x < 0
3
x3
 x– < tan–1x ...........(i)
3
x3
Similarly g(x) = x – – tan–1x
6
x2 1
g(x) = 1 – –
2 1 x2

x 2 (1  x 2 )
g(x) =
2(1  x 2 )
g(x) > 0 for x  (0, 1)  g(x) is M.I.
 g(x) > g(0)
x3
x– – tan–1x > 0
6

x3
x– > tan–1x ........(ii)
6
from (i) and (ii), we get
x3 x3
x– < tan–1x < x – Hence Proved
3 6
TM
RESONANCE 20
x2 tan 1 x x2
Also, 1– < <1–
3 x 6
tan 1 x
Hence by sandwich theorem we can prove that xlim
0
= 1 but it must also be noted that
x
tan 1 x tan 1 x
as x  0, value of  1 from left hand side i.e. <1
x x
 tan1 x 
 lim   =0
 x 
x 0 

NOTE : In proving inequalities, we must always check when does the equality takes place because the point of
equality is very important in this method. Normally point of equality will occur at the end point of
intervals or will be easy to be predicated by hit and trial

  x3
Example : For x   0,  , prove that sin x > x –
 2 6

x3
Solution Let f(x) = sin x – x +
6

x2
f(x) = cos x – 1 +
2
we cannot decide at this point wether f(x) is positive or negative, hence let us check for
monotonic nature of f(x)
f(x) = x – sinx
 
Since f(x) > 0  f(x) is M.I. for x   0, 
 2
 f(x) > f(0)
 f(x) > 0  f(x) is M.I.
 f(x) > f(0)
x3
 sin x – x + >0
6

x3
 sin x > x – Hence proved
6
 sin x tan x   
Example : Examine which is greater sin x tan x or x 2. Hence evaluate xlim
0  2  , where x   0, 2 
 x   
Solution Let f(x) = sinx. tanx – x2
f(x) = cos x . tan x + sin x . sec 2x – 2x
 f(x) = sin x + sin x sec 2x – 2x
 f(x) = cos x + cos x sec 2x + 2sec 2x sin x tan x – 2
 f(x) = (cos x + sec x – 2) + 2 sec 2x sin x tan x
 
Now  2

cos x + sec x – 2 = cos x  sec x and 2 sec 2x tan x . sin x > 0 because x   0, 
 2
 f(x) > 0  f(x) is M.I.
Hence f(x) > f(0)
 f(x) > 0  f(x) is M.I.
 f(x) > 0
 sin x tan x – x2 > 0
Hence sin x tan x > x2
sin x tan x
 >1
x2

 sin x tan x 
 lim   =1
x 0 x2 
TM
RESONANCE 21
x
 1
Example : Prove that f(x) = 1   is monotonically increasing in its domain. Hence or otherwise draw
 x
graph of f(x) and find its range
x
 1 1
Solution. f(x) = 1   , for Domain of f(x) 1 + >0
 x x

x 1
 >0  (–, –1)  (0, )
x

 
x  
 1 1 x  1
Consider f(x) = 1   n1    
x 1 2
 x   1 x 
 x 

x
 1   1 1 
 f(x) = 1   n1    
 x   x  x  1
x
 1  1 1
Now 1   is always positive, hence the sign of f(x) depends on sign of n 1   –
 x  x  1  x

 1 1
i.e. we have to compare n 1   and
 x  1 x

 1 1
So lets assume g(x) = n 1   –
 x  x 1

1 1 1
g(x) = +
1 x 2 ( x  1)2
1
x
1
 g(x) =
x( x  1)2
(i) for x  (0, ), g(x) < 0
 g(x) is M.D. for x  (0, )
g(x) > xlim

g(x)
g(x) > 0.
and since g(x) > 0  f(x) > 0
(ii) for x  (– , – 1), g(x) > 0
 g(x) is M.I. for x  (– , –1)
 g(x) > xlim
 
g(x)
 g(x) > 0  f(x) > 0
Hence from (i) and (ii) we get f(x) > 0 for all x  (– , –1)  (0, )
 f(x) is M.I. in its Domain
For drawing the graph of f(x), its important to find the value of f(x) at boundary points
i.e. ± , 0, –1
x
lim  1
1   = e
x  x

x x
lim 1  1  = 1 and lim  1
1   = 
x 0   x x 1
 x
so the graph of f(x) is

Range is y  (1, ) – {e}

TM
RESONANCE 22
E. Proving inequalities using graph
Generally these inequalities involve comparison between values of two functions at some particular
points.
2 x1  x 2
e 2 x1  e x 2 3
Example : Prove that for any two numbers x 1 & x2 , > e
3

Solution. Assume f(x) = e x and let x1 & x 2 be two points on the curve y = ex.
Let R be another point which divides P and Q in ratio 1 : 2.

2 x1  x 2
2 x1 x2
y coordinate of point R is e  e and y coordinate of point S is e 3 . Since f(x) = e x is
3
always concave up, hence point R will always be above point S.
2x  x
e 2 x1  e x 2 1 2

 < e 3
3
(above inequality could also be easily proved using AM and GM.)

 x1  x 2  x 3  sin x 1  sin x 2  sin x 3


Example : If 0 < x 1 < x 2 < x3 <  then prove that sin   > . Hence or
 3  3
otherwise prove that if A, B, C are angles of a triangle then maximum value of
3 3
sinA + sinB + sinC is .
2

Solution.

TM
RESONANCE 23
sin x 1  sin x 2  sin x 3
Let point A, B, C form a triangle y coordinate of centroid G is and y
3

 x1  x 2  x 3 
coordinate of point F is sin   .
 3 

 x1  x 2  x 3  sin x 1  sin x 2  sin x 3


Hence sin   > .
 3  3
if A + B + C = , then
 A B C sin A  sin B  sin C  sin A  sin B  sin C
sin   >  sin >
 3  3 3 3

3 3
 > sin A  sin B  sin C
2

3 3
 maximum value of (sinA + sinB + sinC) =
2
Example : Compare which of the two is greater (100) 1/100 or (101) 1/101.
Solution. Assume f(x) = x 1/x and let us examine monotonic nature of f(x)
 1  nx 
f(x) = x1/x .  
 x2 
f(x) > 0  x  (0,e)
and f(x) < 0  x  (e,)
Hence f(x) is M.D. for x e
and since 100 < 101
 f(100) > f(101)
 (100)1/100 > (101)1/101
Exercise
1. Prove the following inequalities
(i) x < – n(1 – x) for x  (0, 1)
(ii) x > tan–1(x) for x  (0, )
(iii) ex > x + 1 for x  (0, )
x
(iv)  n (1 + x)  x for x  (0, )
1 x

2 sin x  
(v) < <1 for x   0, 
 x  2

1 e2 1  2
2. Identify which is greater or
e 
1 e2
Ans.
e

3. If 0 < x1 < x 2 < x 3 < , then prove that


 2 x1  x 2  x 3  2 sin x1  sin x 2  sin x 3
sin   >
 4  4
4. If f(x) is monotonically decreasing function and f(x) > 0. Assuming f –1(x) exists prove that

f  1( x1 )  f  1( x 2 )  x1  x 2 
> f –1  .
2  2 
5. Using f(x) = x1/x, identify which is larger e or e.
Ans. e
TM
RESONANCE 24
F. Mean Value of Theorems
(a) Rolle's Theorem:
Let f(x) be a function of x subject to the following conditions:
(i) f(x) is a continuous function of x in the closed interval of a  x  b.
(ii) f  (x) exists for every point in the open interval a < x < b.
(iii) f(a) = f(b).
Then there exists at least one point x = c such that f(c) = 0  c  (a,b).

(b) LMVT Theorem:


Let f(x) be a function of x subject to the following conditions:
(i) f(x) is a continuous function of x in the closed interval of a  x  b.
(ii) f  (x) exists for every point in the open interval a < x < b.
(iii) f(a)  f(b).
f ( b)  f (a )
Then there exists at least one point x = c such that a < c < b where f  (c) =
ba
Geometrically, the slope of the secant line joining the curve at x = a & x = b is equal to the slope of the
tangent line drawn to the curve at x = c. Note the following:
* Rolle's theorem is a special case of LMVT since
f (b )  f (a )
f(a) = f(b)  f(c) = = 0.
ba
(c) Application Of Rolles Theorem For Isolating The Real Roots Of An Equation f(x) = 0
Suppose a & b are two real numbers such that;
(i) f(x) & its first derivative f  (x) are continuous for a  x  b.
(ii) f(a) & f(b) have opposite signs.
(iii) f  (x) is different from zero for all values of x between a & b.
Then there is one & only one real root of the equation f(x) = 0 between a & b.

Example : If 2a + 3b + 6c = 0 then prove that the equation ax 2 + bx + c = 0 has atleast one real root
between 0 and 1.

ax 3 bx 2
Solution. Let f(x) = + + cx
3 2
f(0) = 0
a b
and f(1) = + + c = 2a + 3b + 6c = 0
3 2
If f(0) = f(1) then f(x) = 0 for some value of x  (0, 1)
 ax2 + bx + c = 0 for atleast one x  (0, 1)

Example : Verify Rolles throrem for f(x) = (x – a)n (x – b)m where m, n are natrual numbers for x  [a, b].
Solution. Being a polynomial function f(x) is continuous as well as differentiable, f(a) = 0 and f(b) = 0
 f(x) = 0 for some x  (a , b)
n(x – a)n–1 (x – b)m + m(x – a)n (x – b)m–1 = 0
 (x – a)n–1 (x – b)m–1 [(m + n) x – (nb + ma)] = 0
nb  ma
 x= , which lies in the interval (a, b)
mn

Example : Verify LMVT for f(x) = – x 2 + 4x – 5 and x  [–1, 1]


Solution. f(1) = –2 ; f(–1) = –10
f (1)  f ( 1)
 f(c) =
1  ( 1)
 –2c + 4 = 4  c=0

TM
RESONANCE 25
ba ba
Example : Using mean value theorem, prove that if b > a > 0, then 2 < tan–1 b – tan–1 a <
1 b 1 a2
Solution. Let f(x) = tan–1 x ; x  [a, b] applying LMVT
tan 1 b  tan 1 a 
f(c) = for a < c < b and f(x) = ,
ba 1 x 2
Now f(x) is a monotonically decreasing function
Hence if a < c < b  f(b) < f(c) < f(a)
1 tan 1 b  tan 1 a 1
 2 < < Hence proved
1 b ba 1 a2

TM
RESONANCE 26
Only one correct options

 x  1|
1. The function is monotonically decreasing for
x2
(A) (2, ) (B) (0, 1) (C) (0, 1)  (2, ) (D) (– , )

 p4 
2. The values of p for which the function f(x) =   1 x5 – 3x + ln 5 decreases for all real x is

 1 p 

 3  21 
(A) (– , ) (B)  4,   (1, )
 2 
 5  27 
(C)  3,   (2, ) (D) [1, )
 2 

3. The set of all x for which n (1 + x)  x is equal to


(A) x > 0 (B) x > – 1 (C) – 1 < x < 0 (D) null set

4. Let f(x) = x 3 + ax 2 + bx + 5 sin2 x be an increasing function in the set of real numbers R. Then a & b
satisfy the condition:
(A) a2  3b  15 > 0 (B) a2  3b + 15 < 0 (C) a2  3b  15 < 0 (D) a > 0 & b > 0

5. If f(x) = a
a x sgnx ; g(x) = aa x sgnx for a > 1, a  1and x  R, where { } & [ ] denote the fractional part and
integral part functions respectively, then which of the following statements holds good for the function
h(x), where (n a) h(x) = (n f(x) + n g(x)).
(A) ‘h’ is even and increasing (B) ‘h’ is odd and decreasing
(C) ‘h’ is even and decreasing (D) ‘h’ is odd and increasing

6. If f : [1, 10]  [1, 10] is an non-decreasing function and g : [1, 10]  [1, 10] is a non-increasing
function. Let h(x) = f(g(x)) with h(1) = 1, then h(2)
(A) lies in (1, 2) (B) is more than 2 (C) is equal to 1 (D) is not defined

7. Let f be a differentiable function of x,  x  R. If f(1) = – 4 and f(x)  2  x  [1, 6], then minimum value
of f(6) is
(A) 6 (B) 2 (C) 4 (D) none of these

8. For what values of a does the curve f(x) = x(a2 – 2a – 2) + cosx is always strictly monotonic  x  R.

(A) a  R (B) |a| < 2

(C) 1 – 2 <a<1+ 2 (D) |a| < 2 – 1

x2 x2
9. If f(x) = ; g(x) = where 0 < x < 1, then
2  2 cos x 6 x  6 sin x
(A) both 'f' and 'g' are increasing functions
(B) 'f' is decreasing & 'g' is increasing function
(C) 'f' is increasing & 'g' is decreasing function
(D) both 'f' & 'g' are decreasing function

RESONANCE 27
TM
One or more than one correct options :

10. The set of values of a for which the function f(x) = x 2 + ax + 1 is an increasing function on [1, 2] is 1 and
decreasing in [1, 2] is 2, then :
(A) 1 : a  (2, ) (B) 2 : a  (– , – 4)
(C) 2 : a  (– , – 4] (D) 1 ; a  [– 2, )

11. If f is an even function then


(A) f 2 increases on (a, b) (B) f cannot be monotonic
(C) f 2 need not increases on (a, b) (D) f has inverse

12. Let g(x) = 2f(x/2) + f(1 – x) and f(x) < 0 in 0  x  1 then g(x) :

 2 2 
(A) decreases in 0,  (B) decreases  , 1
 3 3 

 2 2 
(C) increases in 0,  (D) increases in  , 1
 3 3 

13. On which of the following intervals, the function x 100 + sinx – 1 is strictly increasing
(A) (– 1, 1) (B) [0, 1] (C) [/2, ] (D) [0, /2]

2x  1
14. The function y = (x  2) :
x2
(A) is its own inverse (B) decreases for all values of x
(C) has a graph entirely above x-axis (D) is bound for all x.

15. Let f and g be two functions defined on an interval  such that f(x)  0 and g(x)  0 for all x and f is
strictly decreasing on  while g is strictly increasing on  then
(A) the product function fg is strictly increasing on 
(B) the product function fg is strictly decreasing on I
(C) fog(x) is monotonically increasing on 
(D) fog (x) is monotonically decreasing on 

 max ( x, x 2 ) x  0

1. Let f(x) =  . Draw the graph of f(x) and hence comment on the nature of monotonic
min ( x, x 2  2) x  0

behaviour at x = – 1, 0, 1.

 x2 x0

2. Let f(x) =  . Find possible values of a such that f(x) is monotonically increasing at x = 0.
x | a | x  0

3. Find the relation between the constants a, b, c & d so that the function,
f(x) = [a sinx + b cosx] [c sin x + d cos x] is always increasing.

4. Find the intervals of monotonocity for the following functions :

1 x  x2
f(x) =
1 x  x2

RESONANCE 28
TM
x  p2 pq pr
5. If p, q, r be real, locate the intervals in which, f(x) = pq x  q2 qr ,
pr qr x  r2

(a) increase (b) decrease

6. Find the values of ‘a’ for which the function f(x) = (a + 2) x3 – 3ax2 + 9ax – 1 decreases for all real values
of x.

7. Prove that for 0 < x < 1, the inequality, x < – n (1 – x) < x (1 – x)–1.

   3x 
8. For x   0,  identify which is greater (2sinx + tanx) Or (3x) hence find xlim  2 sin x  tan x  .
 2   0  

sin x  
9. Prove that f(x) = is monotonically decreasing function for x   0,  . Hence prove that
x  2

   sin x sin(sin x )
(i) for x   0,  , x cosec x < (ii) <
 6  3 x sin x

 
10. A (0, 1), B  , 1 are two points on the graph given by y = 2 sinx + cos2x. Prove that there exists a
2 
point P on the curve between A & B such that tangent at P is parallel to AB. Find the co-ordinates of P.

11. Using Rolle’s theorem prove that the equation 3x 2 + px – 1 = 0 has at least one real root in the interval
x  (– 1, 1).

12. Show that xex = 2 has one & only one root between 0 & 1.

13. Find the interval in which the following function is increasing or decreasing :
4 sin x  2x  x cos x
f(x) = in [0, ].
2  cos x

 
x sin for x  0
14. Show that the derivative of the function f(x) =  x vanishes on an infinite set of points of
 0 for x  0

the interval (0, 1).

f (a ) f (b )
15. Assume that f is continuous on [a, b] a > 0 and differentiable in (a, b) such that = . Prove that
a b

f(x0 )
there exists x0  (a, b) such that f(x 0) = .
x0

x  1 
16. Find the greatest & least value of f(x) = sin–1 – n x in  , 3 .
x2  1  3 

 
Show that, 1 + x n  x  x  1  1 x 2 for all x  0.
2
17.
 

RESONANCE 29
TM
tan x 2 x2 
18. Prove the inequality, > x for 0 < x 1 < x 2 < .
tan x1 1 2

f ( x)
19. A function f is differentiable in the interval 0  x  5 such that f(0) = 4 & f(5) = – 1. If g(x) = , then
x 1
5
prove that there exists some c  (0, 5) such that g(x) = – .
6

20. Prove that for all x  R ex + 1 e 2 x  (1 + x) + 2  2x  x 2 .

 
21. Let f (sinx) < 0 and f (sin x) > 0,  x   0,  and g(x) = f(sin x) + f(cos x), then find the interval in
 2

which g(x) is increasing and decreasing.

22. If ax 2 + (b/x)  c for all positive x where a > 0 and b > 0 then show that 27ab2  4c 3.

23. Prove that for 0  p  1 & for any a > 0, b > 0 the inequality (a + b)p  ap + bp.

24. Find the greatest and the least values of the function f(x) defined as below.
f(x) = minimum of {3t4 – 8t3 – 6t2 + 24t ; 1  t  x}, 1  x < 2.

 1 
maximum of 3t  sin 2 t  2 ; 2  t  x  , 2  x  4.
 4 

25. If a > b > 0, with the aid of Largrange's formula prove the validity of the inequailities
nbn – 1 (a – b) < an – bn < nan – 1 (a – b), if n > 1. Also prove that the inequalities of the opposite sense if
0 < n < 1.

Exercise # 1 6. (– , – 3]

1. C 2. B 3. B 4. C 8. 2sinx + tanx > 3x, limit = 0


5. D 6. C 7. A 8. C
 3
9. C 10. CD 11. BC 12. BC 10.  , 
6 2
13. ABD 14. AB 15. ABD
13. increasing on [0, /2] and decreasing on [/2, ]
Exercise # 2 16. (/6) + (1/2) n 3, (/3) – (1/2) n 3
1. Neither increasing nor decreasing at x = – 1,  
21. increasing when x   ,  ,
increasing at x = 0, 1. 4 2
2. a  0 3. ad > bc
 
decreasing when x   0,  .
4.  in (– , – 1]  [1, ) & D in [– 1, 1]  4
2 23. Prove that for 0  p  1 & for any a > 0, b > 0 the
5. (a) x < – (p2 + q2 + r 2), x > 0
3 inequality (a + b)p  ap + bp.
2
(b) (– (p2 + q2 + r2), 0)
3 24. greatest = 14, least = 8

RESONANCE 30
TM
Maxima - Minima

A.  st Fundamental Theorem

1. A function f(x) is said to have a local maximum at


x = a if f(a) > f(x)  x  (a – h, a + h). Where h is a
Do not worry about very small positive arbitrary number.

your problems with


mathematics, I
assure you mine are
fargreater.
Note : The local maximum of a function is the largest value
only in neighbourhood of point x = a.
—Al bert Ei nst ei n
2. A function f(x) is said to have local minimum at x = a
if f(a) < f(x)  x  (a – h, a + h).

Life is good only


for to things, Dis-
covering Mathemat- First fundamental theorem is applicable to all functions
continuous, discontinuous, differentiable or non-
ics & Teaching differentiable at x = a.
Mathematics
| x | 0  | x |  2
Example : Let f(x) =  . Examine the behaviour of
 1 x0
—Si mPoi ssi on f(x) at x = 0.

Solution. f(x) has local maxima at x = 0.

RESONANCE TM
31
 (b 3  b 2  b  1)
 – x3  0  x 1
Example : Let f(x) =  (b 2  3b  2)
2x  3 1 x  3

Find all possible values of b such that f(x) has the smallest value at x = 1.
Solution. Such problems can easily solved using graphical approach.

Hence the limit value of f(x) from left of x = 1 should either be greater than or equal to the value
of function at x = 1.
lim f(x)  f(1)
x 1

(b 3  b 2  b  1)
 –1+ >–1
(b 2  3b  2)

(b 2  1)(b  1)
 0
(b  1) (b  2)
 b  (– 2, 1)  [1, – )

Note : If x = a happens to be a boundary point of the function, then compare the value of f(a) with appropriate
values in either the left or right neighbourhood of x = a.

From these figure we can see that boundary points are almost always points of local maxima/
minima.

B. Global Maxima/Minima
Global maximum or minimum value of f(x), x  [a, b] basically refers to the greatest value and least
value of f(x) over that interval mathematically
(i) If f(c)  f(x) for  x  [a, b] then f(c) is called global maximum or absolute maximum value of
f(x).
(ii) Similarly if f(d)  f(x)  x  [a, b] then f(d) is called global minimum or absolute minimum value.
For example consider the graph of function

f(x) has local maxima at x = c, e, b and local minima at x = a, d, f. It can also be easily seen that f(b)
is the greatest value and hence global maximum and similarly f(d) is global minimum.
Also be careful about the fact that a function has global maximum or minimum value when it actually
achieves these values.

RESONANCE TM
32
2 x  1 1  x  2
Let us take graph of function as f(x) = 
4 x 2  x  4

This function has local minima at x = 1, 4 and at x = 2, it is a monotonically decreasing function and
hence neither maximum nor minimum.
f(4) = 0, which the global minimum value but global maximum value is not defined. The value of function
can be made as close to 3 as we may please.

 3  2x 0  x 1

Also consider graph of another function as shown f(x) 1 1 x  2
 x 1 2x3

f(x) has local maxima at x = 0, 3 and f(0) = 3 value 1 over this interval which is global minimum although
note that f(x) does not has local minima at x = 1, 2.
Self Practice Problems
1. In each of following case identify if x = a is point of local maxima, minima or neither of them

(i) (ii)

(iii) (iv)

Ans. (i) Maxima (ii) Neither maxima nor minima


(iii) Minima (iv) Neither maxima nor minima

( x   )2 x0
2. If f(x) =  , find possible values of  such that f(x) has local maxima at x = 0.
 cos x x0

Ans.  [ –1, 1)

RESONANCE TM
33
3. Draw the graph of function f(x) = 2 |x – 2| + 5 |x – 3| (x  R). Also identify points of local Maxima/Minima
and also global Maximum/Minimum values
Ans. Local minima at x = 3, Global minimum value 2 at x = 3, No point of local maximum, Global
maximum value is not defined.

4. Examine the graph of following functions in each case identify the points of global maximum/minimum
and local maximum / minimum.

(i) (ii) (iii)

Ans. (i) Local maxima at x = 2, Local minima at x = 3, Global maxima at x = 2


(ii) Local minima at x = – 1, No point of Global minima, no point of local or Global maxima
(iii) Local & Global maxima at x = 1, Local & Global minima at x = 0.

C.  nd Fundamental Theorem
Following points should be examined for maxima/minima in an interval.
1. Points where f(x) = 0
2. Points where f(x) does not exists
3. Boundary points of interval (only when the interval is closed).

Example : Find the possible points of Maxima/Minima for f(x) = |x 2 – 2x| (x  R)

x 2  2x x  2

f(x) = 2x  x xx2
2
Solution.
x  2x x  0
2

 2( x  1) x  2

f(x) = 2(1  x ) 0  x  2
2

 2( x  1) x  0

f(x) = 0 at x = 1 and f(x) does not exist at x = 0, 2. Thus these are the possible critical points.

f(x) = |x2 – 2x|

from graph we can see that x = 1 is a point of local mixima where as x = 0, 2 are points of local
minima.
Example : If f(x) = x3 + ax 2 + bx + c has extreme values at x = – 1 and x = 3. Find a, b, c.
Solution. Extreme values basically mean maximum or minimum values, since f(x) is differentiable function
so
f(– 1) = 0 = f(3)
f(x) = 3x2 + 2ax + b
f(3) = 27 + 6a + b = 0
f(– 1) = 3 – 2a + b = 0
 a = – 3, b = – 9, c  R

RESONANCE TM
34
D Critical Points
All those points in the interior of an interval where f(x) is either equal to zero or does not exist are
called critical points.

Example: Find the critical points of the function f(x) = 4x 3 – 6x2 – 24x + 9 if (i) x  [0, 3] (ii) x  [–3, 3]
(iii) x  [– 1, 2].
Solution. f(x) = 12(x2 – x – 2)
= 12(x – 2) (x + 1)
f(x) = 0  x = – 1 or 2
(i) if x  [0, 3] , x = 2 is the critical point.
(ii) if x  [– 3, 3], then we have two critical points x = – 1, 2.
(iii) If x  [– 1, 2], then no critical point as both x = 1 and x = 2 become boundary points.
Note : Critical points are always interior points of an interval.

Example : Find the number of critical points for f(x) = max (sinx, cosx) f, x  (0, 2).
Solution.

  5
f(x) has three critical points x = , , .
4 2 4

D. Test for Maxima/Minima

Upto now we have been able to identity exactly which points should be examined for finding the extreme
values of a function. Let as now consider the various tests by which we can separate the critical points
into points of local maxima or minima.

1.  st derivative Test

(i) If f(x) changes sign from negative to positive while passing through x = a from left to
right then x = a is a point of local maxima
(ii) If f(x) changes sign from positive to negative while passing through x = a from left to
right then x = a is a point of local minima.
(iii) If f(x) does not changes its sign about x = a then x= a is neither a point of maxima nor
minima.
Note : This test is applicable only for continuous functions. If f(x) is discontinuous at x = a, then use
of st fundamental theorem is advisable for investigating maxima/minima.

Example : Find the points of maxima or minima of f(x) = x2 (x – 2)2.


Solution. f(x) = x2 (x – 2)2
f(x) = 4x (x – 1) (x – 2)
f(x) = 0  x = 0, 1, 2
examining the sign change of f(x)

Hence x = 1 is point of maxima, x = 0, 2 are points of minima.


Note : In case of continuous functions points of maxima and minima are alternate.

RESONANCE TM
35
Example : Find the points of Maxima/Minima of f(x) = x 3 – 12x also draw the graph of this functions.

Solution. f(x) = x 3 – 12x


f(x) = 3(x 2 – 4) = 3(x – 2) (x + 2)
f(x) = 0  x=±2

For tracing the graph let us find maximum and minimum values of f(x).

x f(x)
2  16
 2  16

Example : Find the greatest and least values of f(x) = x 3 – 12x x  [– 1, 3]

Solution. By graph of the function f(x) = x3 – 12x we can easily see that minimum value of f(x) is – 16 and
maximum value is 11.
Aliter
We can use nd fundamental theorem. The possible points of maxima/minima are critical points
and the boundary points.
for x  [– 1, 3] and f(x) = x 3 – 12x
x = 2 is the only critical points.
Hence points of local maxima/minima are x = – 1, 2, 3. Examining the value of f(x) at these
points we can find greatest and least values.

x f(x)
 1 11
2  16
3 9

 Minima f(x) = – 16 & Maxima f(x) = 11.

Example: Show that f(x) = (x 3 – 6x 2 + 12x – 8) does not have any point of local maxima or minima.
Solution. f(x) = x 3 – 6x2 + 12x – 8
f(x) = 3(x 2 – 4x + 4)
f(x) = 3(x – 2)2
f(x) = 0  x=2
but clearly f(x) does not change sign about x = 2. f(2+) > 0 and f(2–) > 0. So f(x) has no point
of maxima or minima. In fact f(x) is a monotonically increasing function for x  R.

RESONANCE TM
36
 x 3  x 2  10 x x0
Example : Let f(x) =  . Examine the behaviour of f(x) at x = 0.
3 sin x x0

Solution. f(x) is continuous at x = 0.

 3 x 2  2x  10 x0
f(x) = 
3 cos x x0

f(0+) = 3 and f(0–) = – 10 thus f(x) is non-diff. at x = 0  x = 0 is a critical point.


Also derivative changes sign from negative to positive. So x = 0 is a point of local minima.

Example : Let f(x) = x 3 + 3(a – 7)x 2 + 3(a2 – 9) x – 1. If f(x) has positive point of maxima, then find possible
value of 'a'.
Solution. f(x) = 3 [x2 + 2(a – 7)x + (a2 – 9)] = 0
Let ,  be roots of f(x) = 0 and let  be the smaller root. Examining sign change of f(x).

Maxima occurs at smaller root  which has to be positive. This basically implies that both of
roots f(x) = 0 must be positive.
Applying location of roots
29
(i) D>0  a<
7
b
(ii) – >0  a<7
2a
(iii) f(0) > 0  a  (– , – 3) (3, )

 29 
from (i), (ii) and (iii)  a  (– , – 3)   3, 
 7 
Self Practice Problems :

1. Let f(x) = 2x 3 – 9x2 + 12x + 6


(i) Find the possible points of Maxima/Minima of f(x) for x  R.
(ii) Find the number of critical points of f(x) for x  [0, 2].
(iii) Discuss absoluble Maxima/Minima value of f(x) for x  [0, 2]
(iv) Prove that for x  (1, 3), the function does not has a Global maximum.
Ans. (i) x = 1, 2
(ii) 1 (x = 1)
(iii) f(0) = 6 is the global minimum, f(1) = 11 is global maximum

2. Let f(x) = sinx (1 + cosx) ; x  (0, 2). Find the number of critical points of f(x). Also identify which of
these critical points are points of Maxima/Minima.
 5
Ans. 3 critical point x = , ,
3 3

 5
Local maxima at x = , Local minima at x = .
3 3

x 2
3. Let f(x) = + . Find local maximum and local minimum value of f(x). Can you explain this discrepancy
2 x
of locally minimum value being greater than locally maximum value.
Ans. Local maxima at x = – 2 f(–2) = – 2
Local minima at x = 2 f(2) = 2.

RESONANCE TM
37
4. Find the points of local Maxima or Minima of following functions
(i) f(x) = (x – 1)3 (x + 2) 2 (ii) f(x) = sin 2x – x
(iiI) f(x) = x 3 + x 2 + x + 1.
Ans. (i) Maxima at x = – 2, Minima at x = 0
 
(ii) Maxima at x = n + ; Minima at x = n –
6 6
(iii) No point of local maxima or minima.

2.  nd derivative Test
If f(x) is continuous function in the neighbourhood of x = 0 such that f(x) = 0 and f(a) exists
then we can predict maxima or minima at x = 0 by examining the sign of f(a)
(i) If f(a) > 0 then x = a is a point of local minima.
(ii) If f(a) < 0 then x = a is a point of local maxima.
(iii) If f(a) = 0 then second derivative test does not gives use conclusive results.

Example : Find the points of local maxima or minima for f(x) = sin2x – x, x  (0, ).
Solution. f(x) = sin2x – x
f(x) = 2cos2x – 1
1  5
f(x) = 0  cos 2x =  x= ,
2 6 6
f(x) = – 4 sin 2x

 
f   < 0  Maxima at x =
6 6

 5  5
f   >0  Minima at x =
 6  6
3. n th derivative test

Let f(x) be function such that f(a) = f(a) = f(a) = ........= f n – 1 (a) = 0 & f n (a) = 0, then

n = even, n = odd
(i) f n (a) > 0  Minima Neither Maxima nor Minima at x = a
(ii) f n(a) < 0  Maxima

Example : Find points of local maxima or minima of f(x) = x 5 – 5x 2 + 5x3 – 1


Solution. f(x) = x 5 – 5x 2 + 5x3 – 1
f(x) = 5x 2 (x – 1) (x – 3)
f(x) = 0  x = 0, 1, 3
f(x) = 10x (2x – 6x + 3)
2

Now, f(1) < 0  Maxima at x = 1


f(3) > 0  Minima at x = 3
and, f(0) = 0  nd derivative test fails
so, f(x) = 30 (2x 2 – 4x + 1)
f(0) = 30
 Neither maxima nor minima at x = 0.
Note : It was very convenient to check maxima/minima at first step by examining the sign
change of f(x) no sign change of f(x) at x = 0
f(x) = 5x 2 (x – 1) (x – 3)

RESONANCE TM
38
E. Application of Maxima/Minima to Problems
Example : Find two positive numbers x and y such that x + y = 60 and xy3 is maximum.
Solution. x + y = 60
 x = 60 – y  xy3 = (60 – y)y3
Let f(y) = (60 – y) y3
; y  (0, 60)
for maximizing f(y) let us find critical points
f(y) = 3y2 (60 – y) – y3 = 0
f(y) = y2 (180 – 4y) = 0
 y = 45
f(45 ) < 0 and f(45–) > 0. Hence local maxima at y = 45.
+

So x = 15 and y = 45.

Example : Rectangles are inscribe inside a semi-circle of radius r. Find the rectangle with maximum area.
Solution. Let sides of rectangle be x and y.
 A = xy.
Here x and y are not independent variables and are related by pythogoreas theorem with r.
x2 x2
+ y2 = r2  y= r2 
4 4

x2
 A(x) = x r2 
4

x4
 A(x) = x 2r 2 
4
x4
Let f(x) = r2x 2 – ; x  (0, r)
4
A(x) is maximum when f(x) is maximum
Hence f(x) = x(2r 2 – x 2) = 0
 x=r 2

also f(r 2  ) < 0 and f(r 2  ) > 0

r
confirming at f(x) is maximum when x = r 2 & y = .
2
Aliter Let use choose coordinate system with origin as centre of circle
A = xy

 A = 2 (rcos) (rsin)
 
 A = r2 sin2   0, 
 2

Clearly A is maximum when  =
4

r
 x=r 2 and y= .
2

RESONANCE TM
39
Example: A sheet of area 40 m 2 in used to make an open tank with square base. Find the dimensions of
the base such that volume of this tank is maximum.
Solution. Let length of base be xm and height be ym.

v = x 2y

again x and y are related to surface area of this tank which is equal to 40 m 2.
 x 2 + 4xy = 40

40  x 2
y= x  (0, 40 )
4x
 40  x 2 


V(x) = x  4x 
2 
 
( 40 x  x 3 )
V(x) =
4
maximizing volume,

( 40  3x 2 ) 40
V(x) = =0  x= m
4 3

 40 
3x  
and V(x) = –  V  3  < 0.
2  

40
confirming that volume is maximum at x = m.
3

Example : If a right circular cylinder is inscribed in a given cone. Find the dimensions of the cylinder such
that its volume is maximum.
Solution. Let x be the radius of cylinder and y be its height
v = x 2y
x, y can be related by sing similar triangles
y h
=
rx r

h
 y= (r – x)
r

h
 v(x) = x 2 (r – x) x  (0, r)
r

h
 v(x) = (rx 2 – x 3)
r

h
v(x) = x (2r – 3x)
r
 2r 
v(x) = 0 and v   > 0
3
 2r  h
Thus volume is maximum at x =   and y = .
3 3
Note : Following formulae of volume, surface area of important solids are very useful in problems of
maxima & minima.

RESONANCE TM
40
6. Useful Formulae of Measuration to Remember :
1. Volume of a cuboid = bh.

2. Surface area of cuboid = 2(b + bh + h).

3. Volume of cube = a3

4. Surface area of cube = 6a2

1 2
5. Volume of a cone = r h.
3

6. Curved surface area of cone = r ( = slant height)

7. Curved surface of a cylinder = 2rh.

8. Total surface of a cylinder = 2rh + 2r2.

4 3
9. Volume of a sphere = r .
3

10. Surface area of a sphere = 4r2.

1 2
11. Area of a circular sector = r , when  is in radians.
2

12. Volume of a prism = (area of the base) × (height).

13. Lateral surface of a prism = (perimeter of the base) × (height).

14. Total surface of a prism = (lateral surface) + 2 (area of the base)


(Note that lateral surfaces of a prism are all rectangle).

1
15. Volume of a pyramid = (area of the base) × (height).
3

1
16. Curved surface of a pyramid = (perimeter of the base) × (slant height).
2
(Note that slant surfaces of a pyramid are triangles).

Example : Among all regular square pyramids of volume 36 2 cm 3. Find dimensions of the pyramid having
least lateral surface area.
Solution. Let the length of a side of base be x cm and y be the perpendicular height of the pyramid
1
V= area of base x height
3
1 2
 V= x y = 36 2
3

108 2
 y=
x2
1
and S= perimeter of base x slant height
2
1
= (Ax). 
2
x2
but =  y2
4

RESONANCE TM
41
x2
 S = 2x  y2 = x 4  4x 2 y 2
4

 2 
2
108
 S= x 
4
4x 2 
 x 
 
2

8.(108 )2
S(x) = x4 
x2
8.(108 )2
Let f(x) = x4 + for minimizing f(x)
x2
16(108 )2
f(x) = 4x 3 – =0
x3
(x 6  66 )
 f(x) = 4 =0
x3
 x = 6, which a point of minima
Hence x = 6 cm and y = 3 2 .

Example : Let A(1, 2) and B(– 2, – 4) be two fixed points. A variable point P is chosen on the straight line
y = x such that perimeter of PAB is minimum. Find coordinates of P.
Solution. Since distance AB is fixed so for minimizing the perimeter of PAB, we basically have to
minimize (PA + PB)
Let A be the mirror image of A in the line y = x.
F(P) = PA + PB
F(P) = PA + PB
But for PAB

PA + PB  AB and equality hold when P, A and B comes collinear. Thus for minimum path
length point P is that special point for which PA and PB be come incident and reflected rays
with respect to the mirror y = x.
Equation of line joining A and B is y = 2x intersection of this line with y = x is the point P.
Hence P  (0, 0).

Note : Above concept is very useful because such problems become very lengthily by making perimeter as a
function of position of P and then minimizing it.

Self Practice Problems :

1. Find the two positive numbers x and y whose sum is 35 and the product x2 y5 maximum.
Ans. x = 25, y = 10.

2. A square piece of tin of side 18 cm is to be made into a box without top by cutting a square from each
corner and folding up the slops to form a box. What should be the side of the square to be cut off such
that volume of the box is maximum possible.
Ans. 3 cm

RESONANCE TM
42
3. Prove that a fight circular cylinder of given surface area and maximum volume is such that the height is
equal to the diameter of the base.

x2 y2
4. A normal is drawn to the ellipse + = 1. Find the maximum distance of this normal from the
25 16
centre.
Ans. 1 unit

5. A line is drawn passing through point P(1, 2) to cut positive coordinates axes at A and B. Find minimum
area of PAB.
Ans. 4 units

6. Two towns A and B are situated on the same side of a straight road at distances a and b respectively
perpendiculars drawn from A and B meet the road at point c and d respectively. The distance between
C and D is C. A hospital is to be built at a point P on the road such that the distance APB is minimum.
Find position of P.
ac
Ans. P is at distance of from c.
ab

F. Points of Inflection
For continuous function f(x), If f(x 0) = 0 or doesnot exist at points where f(x 0) exists and if f(x)
changes sign when passing through x = x0 then x0 is called a point of inflection. At the point of inflection,
the curve changes its concavity i.e.
(i) If f(x) < 0, x  (a, b) then the curve y = f (x) is convex in (a, b)

(ii) If f(x) > 0, x  (a, b) then the curve y = f (x) is concave in (a, b)

Example : Find the points of inflection of the function f(x) = sin2x x  [0, 2]
Solution. f(x) = sin2x
f(x) = sin2x
f(x) = 2 cos2x
 3
f(0) = 0  x= ,
4 4
both these points are inflection points as sing of f(x) change but f(x) does not changes about
these points.

RESONANCE TM
43
Example : Find the inflection point of f(x) = 3x4 – 4x 3. Also draw the graph of f(x) giving due importance to
maxima, minima and concavity.
Solution. f(x) = 3x4 – 4x3
f(x) = 12x3 – 12x2
f(x) = 12x 2 (x – 1)
f(x) = 0  x = 0, 1

examining sign change of f(x)


thus x = 1 is a point of local minima
f(x) = 12(3x2 – 2x)
f(x) = 12x(3x – 2)
2
f(x) = 0  x = 0, .
3
Again examining sign of f(x)
2
thus x = 0, are the inflection points
3
Hence the graph of f(x) is

RESONANCE TM
44
Only one correct option

1. The greatest value of f(x) = (x + 1) 1/3  (x  1) 1/3 on [0, 1] is:


(A) 1 (B) 2 (C) 3 (D) 21/3

2. The function 'f' is defined by f(x) = xp (1  x) q for all x  R, where p,q are positive integers, has a
maximum value, for x equal to:
pq p
(A) pq (B) 1 (C) 0 (D) pq

3. The coordinates of the point on the curve x 2 = 4y, which is at least distance from the line y = x  4 is
(A) (2, 1) (B) (  2, 1) (C) (  2, 1) (D) none

4. Tangents are drawn to x2 + y2 = 16 from the point P(0, h). These tangents meet the x-axis at A and B.
If the area of triangle PAB is minimum, then
(A) h = 12 2 (B) h = 6 2 (C) h = 8 2 (D) h= 4 2

5. A function f is such that f(2) = f(2) = 0 and f has a local maximum of – 17 at x = 2, then f(x) may be
(A) f(x) = – 17 – (x – 2)n n  N n  4 (B) f(x) = – 17 – (x – 2)n n  3
(C) f(x) = – 17 + (x – 2)n n  3 (D) f(x) – 171 (x – 2)n n  4

 
 tan x, | x | 
1
4
6. f(x) =    , then
  | x |, | x | 
2 4
(A) f(x) has no point of local maxima (B) f(x) has only one point of local maxima
(C) f(x) has exactly two points of local maxima (D) f(x) has exactly two points of local minimas

 
 x 3  x 2 10x 5 , x1
7. Let f(x) =  the set of values of b for which f(x) have greatest value at x = 1 is
2x log2 b 2 2 , x 1
given by :
(A) 1  b  2 (B) b = {1, 2}
   
(C) b  ( ,  1) (D)   130 , 2  U  2 , 130 
   

8. A tangent to the curve y = 1  x2 is drawn so that the abscissa x0 of the point of tangency belongs to the
interval (0, 1]. The tangent at x0 meets the xaxis and yaxis at A & B respectively. The minimum area
of the triangle OAB, where O is the origin is

2 3 4 3 2 2
(A) (B) (C) (D) none
9 9 9

9. The lower corner of a leaf in a book is folded over so as to just reach the inner edge of the page. The
fraction of width folded over if the area of the folded part is minimum is:
(A) 5/8 (B) 2/3 (C) 3/4 (D) 4/5

n2
10. {a1, a2,....., a4,......} is a progression where an = . The largest term of this progression is:
n 200
3

(A) a6 (B) a7 (C) a8 (D) none

RESONANCE 45
TM
(sin1 x  tan 1 x )
11. If f(x) = + 2 x then the range of f(x) is

3 5  11 3 7   7 11
(A)  ,  (B) 0,  (C)  ,  (D)  , 
4 4  4 4 4 4 4 

{x} {x}
12. Let f(x) = sin + cos where a > 0 and { . } denotes the fractional part function. Then the set of
a a
values of a for which f can attain its maximum values is
 4 4 
(A)  0,  (B)  ,   (C) (0, ) (D) none of these
   

13. A and B are the points (2, 0) and (0, 2) respectively. The coordinates of the point P on the line
2x + 3y + 1 = 0 are
 1 1
(A) (7, –5) if |PA – PB| is maximum (B)  ,  if |PA – PB| is maximum
5 5

 1 1
(C) (7, –5) if |PA – PB| is minimum (D)  ,  if |PA – PB| is minimum
5 5

14. The maximum area of the rectangle whose sides pass through the angular points of a given rectangle
of sides a and b is

1 1 2
(A) 2 (ab) (B) (a + b)2 (C) (a + b2) (D) none of these
2 2

15. Number of solution(s) satisfying the equation, 3x 2  2x 3 = log2 (x 2 + 1)  log2 x is:


(A) 1 (B) 2 (C) 3 (D) none

2
Least value of the function, f(x) = 2 x  1 +
2
16. is:
x2
2 1
(A) 0 (B) 3/2 (C) 2/3 (D) 1

17. A straight line through the point (h, k) where h > 0 and k > 0, makes positive intercepts on the coordinate
axes. Then the minimum length of the line intercepted between the coordinate axes is
(A) (h2/3 + k 2/3)3/2 (B) (h3/2 + k 3/2)2/3 (C) (h2/3 – k2/3)3/2 (D) (h3/2 – k3/2)2/3
x x
18. The value of a for which the function f(x) = (4a – 3) (x + log 5) + 2(a – 7) cot sin2 does not posses
2 2
critical points is
(A) (– , – 4/3) (B) (– , – 1) (C) [1, ) (D) (2, )

 1   1 
19. The minimum value of 1   1   is
 sinn    cos n  
(A) 1 (B) 2 (C) (1 + 2n/2)2 (D) None of these

20. The altitude of a right circular cone of minimum volume circumscribed about a sphere of radius r is
(A) 2 r (B) 3 r (C) 5 r (D) none of these

One or more than one correct options

21. Let f(x) = 40/(3x4 + 8x 3 – 18x2 + 60), consider the following statement about f(x).
(A) f(x) has local minima at x = 0
(B) f(x) has local maxima at x = 0
(C) absolute maximum value of f(x) is not defined
(D) f(x) is local maxima at x = – 3, x = 1

RESONANCE 46
TM
22. Maximum and minimum values of the function,
2x 1
f(x) = cos  (x + 3) + 2 sin  (x + 3) 0 < x < 4 occur at :
 
(A) x = 1 (B) x = 2 (C) x = 3 (D) x = 

23. If xlim
a
f(x) = xlim
a
[f(x)] ( [ . ] denotes the greater integer function) and f(x) is non-constant continuous
function, then

(A) xlim
a
f(x) is integer (B) xlim
a
f(x) is non-integer
(C) f (x) has local maximum at x = a (D) f (x) has local minima at x = a

24. If the derivative of an odd cubic polynomial vanishes at two different values of ‘x’ then
(A) coefficient of x 3 & x in the polynomial must be same in sign
(B) coefficient of x 3 & x in the polynomial must be different in sign
(C) the values of ‘x’ where derivative vanishes are closer to origin as compared to the respective
roots on either side of origin.
(D) the values of ‘x’ where derivative vanishes are far from origin as compared to the respective
roots on either side of origin.

x
25. Let f(x) = ln (2x – x2) + sin . Then
2
(A) graph of f is symmetrical about the line x = 1
(B) graph of f is symmetrical about the line x = 2
(C) maximum value of f is 1
(D) minimum value of f does not exist

x1
26. The curve y = has:
x2  1

(A) x = 1, the point of inflection (B) x =  2 + 3 , the point of inflection

(C) x =  1, the point of minimum (D) x =  2  3 , the point of inflection

27. If the function y = f (x) is represented as,


x =  (t) = t3  5 t2  20 t + 7
y =  (t) = 4 t3  3 t2  18 t + 3 ( 2 < t < 2), then:
(A) ymax = 12 (B) ymax = 14 (C) ymin =  67/4 (D) ymin =  69/4

ax 2  2bx  c
28. The maximum and minimum values of y = are those for which
Ax 2  2Bx  C
(A) ax2 + 2bx + c – y (Ax 2 + 2Bx + C) is equal to zero
(B) ax2 + 2 bx + c – y (Ax 2 + 2Bx + C) is a perfect square
dy d2 y
(C) = 0 and 0
dx dx 2
(D) ax 2 + 2bx + c – y (Ax2 + 2 Bx + C) is not a perfect square

29. f(x) is cubic polynomial which has local maximum at x = – 1, If f(2) = 18, f(1) = – 1 and f(x) has local minima
at x = 0, then [IIT – 2006, (5, –1)]
(A) the distance between point of maxima and minima is 2 5 .
(B) f(x) is increasing for x  [1, 2 5 )
(C) f(x) has local minima at x = 1
(D) the value of f(0) = 5
RESONANCE 47
TM
1. Find the area of the largest rectangle with lower base on the x-axis & upper vertices on the curve
y = 12  x² .

2. Find the cosine of the angle at the vertex of an isosceles triangle having the greatest area for the given
constant length  of the median drawn to its lateral side .

ax 3
3. Find the set of value(s) of 'a' for which the function f (x) = + (a + 2) x2 + (a  1) x + 2 possess a
3
negative point of inflection.

4. The fuel charges for running a train are proportional to the square of the speed generated in m.p.h. &
costs Rs. 48/- per hour at 16 mph. What is the most economical speed if the fixed charges i.e. salaries
etc. amount to Rs. 300/- per hour.

5. The three sides of a trapezium are equal each being 6 cms long, find the area of the trapezium when it
is maximum.

6. What are the dimensions of the rectangular plot of the greatest area which can be laid out within a
triangle of base 36 ft. & altitude 12 ft? Assume that one side of the rectangle lies on the base of the
triangle.

7. A closed rectangular box with a square base is to be made to contain 1000 cubic feet. The cost of the
material per square foot for the bottom is 15 paise, for the top 25 paise and for the sides 20 paise. The
labour charges for making the box are Rs. 3/-. Find the dimensions of the box when the cost is
minimum.

8. Find the point on the curve, 4 x2 + a2 y2 = 4 a2, 4 < a2 < 8, that is farthest from the point (0, 2).

9. A cone is circumscribed about a sphere of radius ' r '. Show that the volume of the cone is minimum
 1
when its semi  vertical angle is, sin 1   .
 3
a 3
10. Find the values of 'a' for which the function f(x) = x + (a + 2) x 2 + (a  1) x + 2 possess a negative
3
point of minimum.

11. A figure is bounded by the curves, y = x 2 + 1, y = 0, x = 0 & x = 1. At what point (a, b), a tangent should
be drawn to the curve, y = x2 + 1 for it to cut off a trapezium of the greatest area from the figure.

12. Prove that the least perimeter of an isosceles triangle in which a circle of radius ' r ' can be inscribed is

6r 3.

 f ( x) 
1/ x
13. Find the polynomial f (x) of degree 6, which satisfies Limit
x0 
1  = e2 and has local maximum
 x3 
at x = 1 and local minimum at x = 0 & 2.

14. Two towns located on the same side of the river agree to construct a pumping station and filteration
plant at the river’s edge, to be used jointly to supply the towns with water. If the distance of the two
towns from the river are ‘a’ & ‘b’ and the distance between them is ‘c’, show that the pipe lines joining
them to the pumping station is atleast as great as c 2  4ab .

RESONANCE 48
TM
15. Find the co-ordinates of all the points P on the ellipse (x2/a2) + (y2/b2) = 1 for which the area of the
triangle PON is maximum, where O denotes the origin and N the foot of the perpendicular from O to the
tangent at P. [IIT – 1999, 10]

16. If p(x) be a polynomial of degree 3 satisfying p(–1) = 10, p(1) = – 6 and p(x) has maxima at
x = – 1 and p(x) has minima at x = 1. Find the distance between the local maxima and local minima of
the curve. [IIT – 2005]

Exercise # 1 Exercise # 2
1. B 2. D 3. A 4. D 1. 32 sq. units 2. cos A = 0.8

5. A 6. C 7. D 8. B 3. (,  2)  (0, ) 4. 40 mph

5. 27 3 sq. cms 6. 6   18 
9. B 10. B 11. B 12. A
7. side 10', height 10' 8. (0, 2)
13. A 14. B 15. A 16. D
 1 5
10. (1, ) 11.  , 
17. A 18. A 19. C 20. D
 2 4

12 5 2 6
21. ACD 22. AC 23. AD 24. BC
13. f (x) = 2 x4  x + x
5 3
25. ACD 26. ABD 27. BD 28. BC
a2 b2
29. BC 15. ± ,±
a 2  b2 a 2  b2

16. 4 65

RESONANCE 49
TM
Indefinite Integration

1. If f & g are functions of x such that g(x) = f(x) then,


d
f(x) dx = g(x) + c  {g(x)+c} = f(x), where c is called
dx
The mov i ng the constant of integration.

power of 2. Standard Formula:


ax  bn 1

mat hemat i cal
 1
a  n  1
(i) (ax + b) n
dx = + c, n
i nvent i on i s


not r easoni ng dx 1
(ii) = ln (ax + b) + c
ax  b a
but


1 ax+b
i magi nat i on. (iii) eax+b dx = e +c
a

(iv)  apx+q dx =
1 a pxq
p n a
+ c; a > 0


August us
1
(v) sin (ax + b) dx =  cos (ax + b) + c
De Mo r g a n a


Nat ur e l aughs
1
(vi) cos (ax + b) dx = sin (ax + b) + c
at t he a


di f f i cul t i es of 1
(vii) tan(ax + b) dx = ln sec (ax + b) + c
a
i nt egr at i on.


1
(viii) cot(ax + b) dx = ln sin(ax + b)+ c
a


Pi er r e-Si mon 1
(ix) sec² (ax + b) dx = tan(ax + b) + c
a
Lapl ace

(x)  cosec²(ax + b) dx = 
1 cot(ax + b)+ c
a


1
(xi) sec (ax + b). tan (ax + b) dx = sec (ax + b) + c
a

(xii)  cosec (ax + b). cot (ax + b) dx = 


1 cosec (ax + b) + c
a

RESONANCE 1
TM
(xiii)  secx dx = ln (secx + tanx) + c OR ln tan
   x  + c
 4 2


x
(xiv) cosec x dx = ln (cosecx  cotx) + c OR ln tan + c OR  ln (cosecx + cotx) + c
2


dx x
(xv) = sin1 +c
a x2 2
a


dx 1 x
(xvi) = tan1 +c
a x22
a a


dx 1 x
(xvii) = sec 1 +c
x x a 2 2
a a

(xviii)  x a
dx
2 2
= ln x  x2  a2  OR sinh1
x
a
+c

(xix)  x a
dx
2 2
= ln x  x2  a 2  OR cosh1
x
a
+c


dx 1 ax
(xx) 2 = ln ax + c
a x2
2a


dx 1 xa
(xxi) 2 = ln xa + c
x a2
2a


x a2 x
(xxii) a  x dx =
2 2
a x
2 2
+ sin1 +c
2 2 a

 x  x2  a2 

x a2  
(xxiii) x  a dx =
2 2
x a
2 2
+ n  a + c
2 2  

 x  x 2  a2 

x a2  
(xxiv) x 2  a 2 dx = x2  a 2  n  a  +c
2 2  


e ax
(xxv) e . sin bx dx = 2 (a sin bx  b cos bx) + c
a  b2
ax


e ax
(xxvi) eax. cos bx dx = (a cos bx + b sin bx) + c
a 2  b2

RESONANCE 2
TM
3. Theorems on integration
(i)  c f ( x).dx =c  f (x).dx
(ii)  (f (x)  g(x)) dx =  f ( x)dx  g( x) dx
 f ( x)dx  g( x)  c  f (ax  b)dx =
g(ax  b)
(iii)  +c
a

Note : (i) every contineous function is integrable


(ii) the integral of a function reffered only by a constant.

 f (x).dx = g(x) + c
= h(x) + c
g(x) = f(x) & h(x) = f(x)
g(x) – h(x) = 0
means, g(x) – h(x) = c

 4x dx
5
Example : Evaluate :

 4x
4 6 2 6
Solution.
5
dx = x +C= x + C.
6 3

  x
 7 2 
Example : Evaluate :
3
 5x 2  4    dx
x x

  x
 7 2 
Solution.
3
 5x 2  4    dx
x x

x  5x  4dx +  
7 2
=
3
dx + 2
dx – dx + dx
x x

x x   x
1 1/ 2
=
3
dx + 5 . 2
dx – 4 . 1 . dx + 7 . dx + 2 . dx
x

x3  x 1/ 2 
=
x4
+5.
 
– 4x + 7 log | x | + 2  1/ 2  + C
4 3  

x4 5
= + x 3 – 4x + 7 log | x | + 4 x +C
4 3

Example : Evaluate :
e  e a log x  e a log a dx
x log a

Solution. We have,

e  e a log x  e a log a dx
x log a

=
e  elog x  elog a dx
log a x a a

 (a  x a  a a ) dx
x
=

a dx + x dx + a dx
x a a
=

ax x a 1
= + + aa . x + C.
loga a 1

RESONANCE 3
TM

2x  3x
Example : Evaluate : dx
5x


2x  3x
Solution. dx
5x


 2x 3x 
  
=  5 x 5 x  dx
 


  2 x  3 x 
       dx = (2 / 5) (3 / 5) x
x
= + +C
  5   5   loge 2 / 5 loge 3 / 5

Example: Evaluate :  sin x cos3 x dx


3

 (2 sin x cos x)
1 3
Solution. = dx
8

 sin
1
= 3
2x dx
8


1 3 sin 2x  sin 6 x
= dx
8 4

 (3 sin 2x  sin 6x) dx


1
=
32
1  3 1 
=  2 cos 2x  6 cos 6 x  + C
32  

x
x4
Example : Evaluate : dx
2
1

x
x4
Solution. dx
2
1

 x
x4  1 1 x4  1 1
= dx = + dx
x 12 2
1 x2  1

 x
1 x3
= ( x 2  1) dx + dx = – x + tan–1 x + C
2
1 3

 4  9x
1
Example: Evaluate : 2 dx

Solution. We have

 4  9x
1
2

 4x
1 1
= dx
9 2
9

 ( 2 / 3)
1 1
= dx
9 2
 x2

1 1  x  1  3x 
= . tan–1   +C= tan–1   +C
9 ( 2 / 3)  2/3  6  2 

RESONANCE 4
TM
Example :  cos x cos 2x dx
Solution.  cos x cos 2x dx
 2 cos x cos 2x dx
1
=
2

 (cos 3x  cos x) dx
1
=
2

1  sin 3 x  sin x 
=   +c
2  3 1 

Self Practice Problems

 tan x dx
2
1. Evaluate : Ans. tanx – x + C

 1  sin x
1
2. Evaluate : dx Ans. tanx – sec x + C

4. Integration by Subsitutions
If we subsitute x = (t) in a integral then
(i) everywhere x will be replaced in terms of t.
(ii) dx also gets converted in terms of dt.
(iii) (t) should be able to take all possible value that x can take.

x sin x 4 dx
3
Example : Evaluate :
Solution. We have

= x sin x 4 dx
3

1
Let x4 = t  d(x4) = dt  4x3 dx = dt  dx = dt
4x 3


(n x )2
Example : dx
x


(n x )2
Solution. dx
x
Put nx = t
1
 dx = dt
x


 dx 
= t 2.  
 x 

 t dt
2
=

t3
= +c
3
(  n x )3
= +c
3

RESONANCE 5
TM
 (1  sin x ) cos x dx
2
Example : Evaluate
Solution. Put sinx = t
cosx dx = dt


t3
(1  t 2 ) dt = t + +c
3

sin3 x
= sin x + +c
3

x
x
Example : Evaluate : dx
4
 x2  1
Solution. We have,

x  (x
x x
= dx = dx
4
 x 1 2 )  x2  1
2 2

dt
Let x 2 = t, then, d (x 2) = dt  2x dx = dt  dx =
2x


x dt
= .
t  t 1
2
2x


1 1
= dt
2 t  t 1
2


1 1
= dt
2  3
2
 1
2
 t     

 2  2 

 
t 1
1 1  2 +C
= . tan–1
2 3  3 
 
2  2 

1  2t  1  1  2x 2  1 
tan–1   +C=   + C.
=  tan–1  3 
3  3  3 


( f ( x ))n1
Note: (i) [ f(x)]n f (x) dx =
n 1


f  (x) ( f ( x ))1n
 f (x)n
(ii) dx =
1 n


dx
(iii) nN Take x n common & put 1 + xn = t.
x ( x n 1)


dx
 N, take xn common & put 1+xn = tn
 
(iv) ( n1) n
x x 1
2 n n


dx
 
(v) take x n common as x and put 1 + x n = t.
x 1 x
n n 1/ n

RESONANCE 6
TM
Self Practice Problems


sec 2 x
1. dx Ans. n |1 + tan x| + C
1  tan x


sin(nx )
2. dx Ans. – cos (n x) + C
x

5. Integration by Part :

 f ( x) g(x) dx  g(x) dx –   dx f ( x)  g( x) dx  dx


 d 
= f(x)

(i) when you find integral  g( x) dx then it will not contain arbitarary constant.
(ii)  g( x) dx should be taken as same both terms.
(iii) the choice of f(x) and g(x) is decided by ILATE rule.
the function will come later is taken an integral function.
  Inverse function
L  Logrithimic function
A  Algeberic function
T  Trigonometric function
E  Exponential function

 x tan
1
Example : Evaluate : x dx

 x tan
1
Solution. x dx

 1 x
x2 1 x2
= (tan–1 x) – . dx
2 2
2

 1 x
x2 1 x2  1 1 x2 1 1
= tan–1 x – dx = tan–1 x – dx
2 2 x 1
2
2 2 2
1

x2 1
= tan–1 x – [x – tan–1 x] + C.
2 2

Example : Evaluate :  x log(1  x) dx


Solution.  x log(1  x) dx

x2 1 x2
= log (x + 1) . – . dx
2 x 1 2

 
x2 1 x2 x2 1 x 2  1 1
= log (x + 1) – dx = log (x + 1) – dx
2 2 x 1 2 2 x 1

 x 1
x2 1 x2  1 1
= log (x + 1) – + dx
2 2 x 1


x2 1   ( x  1)  1  dx 
= log (x + 1) –  
2 2   x  1 

1  x  x  log | x  1 |
2
x2   +C
= log (x + 1) –
2 2  2 

RESONANCE 7
TM
Example : Evaluate :
e sin 3x dx
2x

Let  = e sin 3x dx. Then,


2x
Solution.

= e sin 3x dx
2x

 2e
 cos 3 x   cos 3 x 
  = e2x    –   dx
2x
 3   3 

e
1 2x 2
 =– cos 3x dx
2x
e cos 3x +
3 3


1 2x 2  2 x sin 3 x sin 3 x 
 =– e cos 3x + e  2e 2 x dx 
3 3  3 3 

e
1 2x 2 4
 =– sin 3x dx
2x
e cos 3x + e2x sin 3x –
3 9 9

1 2x 2 4
 =– e cos 3x + e2x sin 3x – 
3 9 9

4 e 2x
 + = (2 sin 3x – 3 cos 3x)
9 9

13 e 2x
 = (2 sin 3x – 3 cos 3x)
9 9

e 2x
 = (2 sin 3x – 3 cos 3x) + C
13

Note :

(i)  ex [f(x) + f (x)] dx = ex. f(x) + c

(ii)  [f(x) + xf (x)] dx = x f(x) + c

e
x x
Example : dx
( x  1)2

e
x x  1 1
Solution. dx
( x  1)2


 1 1  ex
ex  
  ( x  1)  ( x  1)2  dx =
( x  1)
+c
 

e
 1  sin x 
  dx
x
Example :
 1  cos x 

 x x
 1  2 sin cos 
e
x  2 2
Solution.  2 x  dx
 2 sin 
 2 

e
1 x x
  cos ec 2  cot  dx
x
= – ex cot +c
2 2 2

RESONANCE 8
TM
 n (nx )  (nx)
 1 
Example :  dx

2

Solution. put x = et

e
 1
  nt  2  dt
t

 t 

e
 1 1 1  1
  nt    2  dt = et  nt   + c
t

 t t t   t

 1 
 x n ( nx )   +c
  nx 

Self Practice Problems

1.  x sin x dx Ans. – x cosx + sin x + C

x e dx
2 x
2. Ans. x 2 ex – 2xex + 2ex + C

6. Integration of Rational Algebraic Functions by using Partial Fractions:

PARTIAL FRACTIONS :

f (x)
If f(x) and g(x) are two polynomials, then g( x ) defines a rational algebraic function of a rational function
of x.
f (x)
If degree of f(x) < degree of g(x), then g( x ) is called a proper rational function.
f (x)
If degree of f(x)  degree of g(x) then g( x ) is called an improper rational function
f (x) f (x)
If g( x ) is an improper rational function, we divide f(x) by g(x) so that the rational function g( x ) is
( x )
expressed in the form (x) + g( x ) where (x) and (x) are polynomials such that the degree of (x) is

f (x)
less than that of g(x). Thus, g( x ) is expressible as the sum of a polynomial and a proper rational

function.
f (x)
Any proper rational function g( x ) can be expressed as the sum of rational functions, each having a

simple factor of g(x). Each such fraction is called a partial fraction and the process of obtained them is
f (x)
called the resolutions or decomposition of g( x ) into partial fractions.

f (x)
The resolution of g( x ) into partial fractions depends mainly upon the nature of the factors of g(x) as

discussed below.

RESONANCE 9
TM
CASE I When denominator is expressible as the product of non-repeating linear factors.
Let g(x) = (x – a1) (x – a2) .....(x – an). Then, we assume that

f (x) A1 A2 An
g( x ) = x  a1 + x  a2 + ..... + x  an

where A1, A2, ...... An are constants and can be determined by equating the numerator on R.H.S. to the
numerator on L.H.S. and then substituting x = a1, a2, ........,an.

3x  2
Example : Resolve into partial fractions.
x  6 x 2  11x  6
3

3x  2 3x  2
Solution. We have, =
x  6 x  11x  6
3 2 ( x  1)( x  2)( x  3)

3x  2 A B B
Let = + + . Then,
( x  1)( x  2)( x  3) x 1 x2 x 3

3x  2 A( x  2)( x  3)  B( x  1)( x  3)  C( x  1)( x  2)


 =
( x  1)( x  2)( x  3) ( x  1)( x  2)( x  3)
 3x + 2 = A(x – 2) (x – 3) + B (x – 1) (x – 3) + C(x – 1) (x – 2) ...........(i)
Putting x – 1 = 0 or x = 1 in (i), we get

5
5 = A(1 – 2) (1 – 3)  A = ,
2
Putting x – 2 = 0 or, x = 2 in (i), we obtain
8 = B (2 – 1) (2 – 3)  B = –8.
Putting x – 3 = 0 or, x = 3 in (i), we obtain

11
11 = C (3 – 1) (3 – 2)  C = .
2

3x  2 3x  2 5 8 11
 = = – +
x  6 x  11x  6
3 2 ( x  1)( x  2)( x  3) 2( x  1) x2 2( x  3)

Note : In order to determine the value of constants in the numerator of the partial fraction corresponding to the
non-repeated linear factor px + q in the denominator of a rational expression, we may proceed as
follows :

q
Replace x = – (obtained by putting px + q = 0) everywhere in the given rational expression except in
p
the factor px + q itself. For example, in the above illustration the value of A is obtained by replacing x

3x  2
by 1 in all factors of except (x – 1) i.e.
( x  1)( x  2)( x  3)

3  1 2 5
A= =
(1  2)(1  3) 2
Similarly, we have

3 2 1 33  2 11
B= = –8 and, C = =
(1  2)(2  3) (3  1)(3  2) 2

RESONANCE 10
TM
x 3  6 x 2  10 x  2
Example : Resolve into partial fractions.
x 2  5x  6
Solution. Here the given function is an improper rational function. On dividing we get

x 3  6 x 2  10 x  2 (  x  4)
=x–1+ ...........(i)
x 2  5x  6 ( x  5 x  6)
2

x  4 x  4
we have, =
x  5x  6
2 ( x  2)( x  3)

x  4 A B
So, let = + – x + 4 = A(x – 3) + B(x – 2) ...........(ii)
( x  2)( x  3) x2 x 3
Putting x – 3 = 0 or, x = 3 in (ii), we get
1 = B(1)  B = 1.
Putting x – 2 = 0 or, x = 2 in (ii), we get
2 = A (2 – 3)  A = – 2
x  4 2 1
 = +
( x  2)( x  3) x2 x 3

x 3  6 x 2  10 x  2 2 2
Hence =x–1– +
x  5x  6
2
x2 x 3
CASE II When the denominator g(x) is expressible as the product of the linear factors such that some
of them are repeating.

1 1
Example g( x ) = this can be expressed as
( x  a) ( x  a1 )( x  a 2 ).......( x  a r )
k

A1 A2 A3 Ak B1 B2 Br
+ 2 + 3 + ....+ k + ( x  a ) + ( x  a ) + ...... + ( x  a )
x a ( x  a) ( x  a) ( x  a) 1 2 r

Now to determine constants we equate numerators on both sides. Some of the constants are determined
by substitution as in case I and remaining are obtained by

The following example illustrate the procedure.

 ( x  1) (x  1)(x  2)
3x  2 (3 x  2)dx
Example : Resolve into partial fractions, and evaluate
( x  1) ( x  1)( x  2)
2 2

3x  2 A1 A2 A3 A4
Solution. Let = + 2 + +
( x  1) ( x  1)( x  2)
2
x 1 ( x  1) x 1 x2
 3x – 2 = A1 (x – 1) (x + 1) (x + 2) + A2 (x + 1) (x + 2)
+ A3 (x – 1)2 (x + 2) + A4 (x – 1)2 (x + 1) .......(i)
Putting x – 1 = 0 or, x = 1 in (i) we get
1
1 = A2 (1 + 1) (1 + 2)  A2 =
6
Putting x + 1 = 0 or, x = –1 in (i) we get
5
– 5 = A3 (–2)2 (–1 + 2)  A3 = –
4
Putting x + 2 = 0 or, x = –2 in (i) we get
8
– 8 = A4 (–3)2 (–1)  A4 =
9
Now equating coefficient of x 3 on both sides, we get 0 = A1 + A3 + A4
5 8 13
 A1 = –A3 – A4 = – =
4 9 36

RESONANCE 11
TM
3x  2 13 1 5 8
 = + 2 – +
( x  1) ( x  1)( x  2)
2
36( x  1) 6( x  1) 4( x  1) 9( x  2)

 ( x  1) (x  1)(x  2)
(3 x  2)dx
and hence 2

13 1 5 8
= n |x – 1| – – n |x + 1| + n |x + 2| + c
36 6( x  1) 4 9

CASE III When some of the factors of denominator g(x) are quadratic but non-repeating. Corresponding
Ax  B
to each quadratic factor ax 2 + bx + c, we assume partial fraction of the type , where A and
ax  bx  c 2

B are constants to be determined by comparing coefficients of similar powers of x in the numerator of


A(2ax  b) B
both sides. In practice it is advisable to assume partial fractions of the type +
ax  bx  c
2
ax  bx  c
2

The following example illustrates the procedure

 (x  1)(x
2x  1 2x  1
Example : Resolve into partial fractions and evaluate dx
( x  1)( x  2)
2 2
 2)

2x  1 A Bx  C
Solution. Let = + 2 . Then,
( x  1)( x  2)
2
x 1 x 2

2x  1 A( x 2  2)  (Bx  C)( x  1)
=
( x  1)( x 2  2) ( x  1)( x 2  2)
 2x – 1 = A (x2 + 2) + (Bx + C) (x + 1) ...(i)
Putting x + 1 = 0 or, x = –1 in (i), we get – 3 = A(3)  A = –1.
Comparing coefficients of the like powers of x on both sides of (i), we get
A + B = 0, C + 2A = –1 and C + B = 2
 –1 + B = 0, C – 2 = –1 (Putting A = –1)
 B = 1, C = 1
2x  1 1 x 1
 =– + 2
( x  1)( x 2  2) x 1 x 2

 (x  1)(x
2x  1
Hence dx
2
 2)

1 1 x
= – n |x + 1| + n |x2 + 1| + tan–1 +c
2 2 2
CASE IV When some of the factors of the denominator g(x) are quadratic and repeating fractions of the

 A (2ax  b) 

   
 A 0 (2ax  b) A1  
A2
   +  2 2
1
form
 ax  bx  c ax  bx  c   ax  bx  c ax  bx  c 
2 2 2 2

 A 
2k 1( 2ax  b )

   
A 2k
+ .......+  2  k
 ax  bx  c ax 2  bx  c 
k

The following example illustrates the procedure.

RESONANCE 12
TM
2x  3
Example: Resolve into partial fractions.
( x  1)( x 2  1)2

2x  3 A Bx  C Dx  E
Solution. Let = + 2 + . Then,
( x  1)( x 2  1)2 x 1 x 1 ( x 2
 1)2

2x – 3 = A(x 2 + 1) 2 + (Bx + C) (x – 1) (x 2 + 1) + (Dx + E) (x – 1) ......(i)

1
Putting x = 1 in (i), we get – 1 = A (1 + 1)2  A = –
4
Equation coefficients of like powers of x, we have
A + B = 0, C – B = 0, 2A + B – C + D = 0, C + E – B – D = 2 and A – C – E = –3.

1
Putting A = – and solving these equations, we get
4
1 1 5
B= = C, D = and E =
4 2 2
2x  3 1 x 1 x5
 = + +
( x  1)( x  1) 2 2
4( x  1) 4( x  1)
2
2( x 2  1)2

2x
Example : Resolve into partial fractions.
x 13

2x 2x
Solution. We have, =
x3  1 ( x  1)( x 2  x  1)

2x A Bx  C
So, let = + 2 . Then,
( x  1)( x 2  x  1) x 1 x  x 1
2x = A (x2 + x + 1) + (Bx + C) (x – 1) .......(i)
2
Putting x – 1 = 0 or, x = 1 in (i), we get 2 = 3 A  A =
3
2
Putting x = 0 in (i), we get A – C = 0  C = A =
3
Putting x = – 1 in (i), we get –2 = A + 2B – 2 C.

2 4 2
 –2= + 2B – B=–
3 3 3

2x 2 1 2 / 3 x  2 / 3 2x 2 1 2 1 x
 = . + or,, 3 = +
x 13 3 x 1 x  x 1
2
x 1 3 x  1 3 x  x 1
2

Self Practice Problems

 (x  2)(x  3) dx
1 x2
1. (i) Ans. +C
x3
n

 ( x  1)(x
dx 1 1 1
(ii) Ans. n |x + 1| – n (x 2 + 1) + tan–1 (x) + C
2
 1) 2 4 2

RESONANCE 13
TM
 ax 
dx

dx
7. Integration of type 2
bx  c
,
ax 2 bx c
, ax 2 bx c dx

Express ax 2 + bx + c in the form of perfect square & then apply the standard results.

Example : Evaluate :  x 2  2x  5 dx

Solution. We have,

 x 2  2x  5

=  x 2  2x  1  4 dx

1 1
= (x + 1) ( x  1)2  2 2 + . (2)2 log |(x + 1) + ( x  1)2  2 2 | + C
2 2
1
= (x + 1) x 2  2x  5 + 2 log |(x + 1) + x 2  2x  5 | + C
2

x
1
Example : Evaluate : dx
2
 x 1

x
1
Solution. dx
2
 x 1

x
1
= dx
1 1
2
 x   1
4 4

 ( x  1/ 2)
1
= dx
2
 3/4

 ( x  1/ 2)   
1 1  x  1/ 2 
= 2 dx = tan–1   +C

3 /2 3 /2  3 /2 
2

2  2x  1 
= tan–1   + C.

3  3 


1
Example : Evaluate : dx
9  8x  x 2


1
Solution. dx
9  8x  x 2


1
= dx
 { x  8 x  9}
2


1
= dx
 { x  8 x  16  25}
2

  {(x  4) 
1 1 x 4
= dx = dx = sin–1   +C
2
5 } 2
5  ( x  4)
2 2
 5 

Self Practice Problems

 2x
1 1 2x  1
1. dx Ans. +C
 x 1 3 2x  2
2 n


1 1  3 3
log  x  4   x  2 x  1 + C
2
2. dx Ans.
2x  3x  2
2 2  

RESONANCE 14
TM
8. Integration of type

 ax   (px  q)
px  q px  q
dx, dx, ax 2  bx  c dx
2
bx  c ax 2 bx c
Express px + q = A (differential coefficient of denominator) + B.


2x  3
Example : Evaluate : dx
x  4x  1
2


2x  3
Solution. dx
x  4x  1
2


( 2x  4)  1
= dx
x 2  4x  1

 
2x  4 1
= dx – dx
x 2  4x  1 x 2  4x  1

 
 3
dt 1
= – dx, where t = x2 + 4x + 1
t ( x  2)  2 2

=2 t – log | (x + 2) + x 2  4x  1 | + C

= 2 x 2  4 x  1 – log | x + 2 + x 2  4x  1 | + C

Example : Evaluate :  ( x  5) x 2  x dx

d
Solution. Let (x – 5) =  . (x 2 + x) + . Then,
dx
x – 5 = (2x + 1) + .
Comparing coefficients of like powers of x, we get
1 11
1 = 2 and  +  = – 5   = and  = –
2 2

 ( x  5) x 2  x dx

  2 (2x  1)  2 
1 11 
= x 2  x dx

 2 (2x  1) 
1 11
= x 2  x dx – x 2  x dx
2

 (2x  1) 
1 11
= x 2  x dx – x 2  x dx
2 2

 
 1  1
2 2
1 11
= t dt – x      dx where t = x2 + x
2 2  2 2

 2
11  1  x  1   x  1    1  
2
1 t3 / 2
= . –  2 2 
2 3/2 2  2  2 


 2 
 x  1    x  1    1  
2
1  1
2

.   log  

2 2 2  2   2   + C
 

RESONANCE 15
TM
1 3/2 11  2x  1 x 2  x  1 n  1 
= t –   x    x2  x  + C
3 2  4 8  2 

1 2 11  2x  1 x 2  x  1 n  1 
 x    x2  x  + C
= (x + x)3/2 – 
3 2  4 8  2 

Self Practice Problems

x
x 1 1 1  2x  1 
1. dx Ans. log |x2 + x + 3| + tan–1   + C
2
x3 2 11  11 


6x  5
2. dx Ans. 2 3x 2  5x  1 + C
3x 2  5x  1

3.  ( x  1) 1  x  x 2 dx

1 3 9
Ans. (x 2 + x + 1)3/2 – (2x + 1) 1  x  x 2 – 16 log (2x +1 + 2 x  x  1 ) + C
2
3 8

9. Integration of trigonometric functions

  
dx dx dx
(i) OR OR
a  b sin x 2 a  b cos 2 x a sin x  b sin x cos x  c cos 2 x
2

Multiply Nr & Dr by sec² x & put tan x = t.

  
dx dx dx
(ii) a  b sinx OR OR
a  bcosx a  b sin x  c cos x
Hint:
Convert sines & cosines into their respective tangents of half the angles and then,
x
put tan =t
2


a.cos x b.sinx c d
(iii) dx. Express Nr  A(Dr) + B (Dr) + c & proceed.
.cos x m.sinx n dx

 1  sin x  cos x
1
Example : Evaluate : dx

 1  sin x  cos x
1
Solution. = dx


1
= 2 tan x / 2 1  tan 2 x / 2 dx
1 
1  tan 2 x / 2 1  tan 2 x / 2

 1  tan 
1  tan 2 x / 2 sec 2 x / 2
= dx = dx
2
x / 2  2 tan x / 2  1  tan 2 x / 2 2  2 tan x / 2

x 1 x
Putting tan = t and sec2 dx = dt, we get
2 2 2

 t 1
1 x
= dt = log | t + 1| + C = log tan 1 + C
2

RESONANCE 16
TM
 3 cos x  2 sin x
3 sin x  2 cos x
Example : Evaluate : dx

 3 cos x  2 sin x
3 sin x  2 cos x
Solution. = dx

d
Let 3 sin x + 2 cos x = . (3 cos x + 2 sin x ) +  (3 cos x + 2 sin x)
dx
 3 sin x + 2 cos x =  (–3 sin x + 2 cos x) +  (3 cos x + 2 sin x )
Comparing the coefficients of sin x and cos x on both sides, we get
12 5
– 3 + 2 = 3 and 2 + 3 = 2  = and  = –
13 13


( 3 sin x  2 cos x )  (3 cos x  2 sin x )
 = dx
3 cos x  2 sin x

  3 cos x  2 sin x
3 sin x  2 cos x
= 1 . dx +  dx

t
dt
=x+ , where t = 3 cos x + 2 sin x

5 12
=  x +  n | t | + C = x+ n | 3 cos x + 2 sin x | + C
13 13

 sin x  2 cos x  3
3 cos x  2
Example : Evaluate : dx

Solution. We have,

 sin x  2 cos x  3
3 cos x  2
= dx

Let 3 cos x + 2 =  (sin x + 2 cos x + 3) +  (cos x – 2 sin x) + 


Comparing the coefficients of sin x, cos x and constant term on both sides, we get
 – 2 = 0, 2 +  = 3, 3 +  = 2
6 3 8
 = , and  = –
5 5 5


(sin x  2 cos x  3)  (cos x  2 sin x )  
 = dx
sin x  2 cos x  3

 dx    sin x  2 cos x  3  sin x  2 cos x  3


cos x  2 sin x 1
 = dx +  dx

  =  x +  log | sin x + 2 cos x + 3 | +  1, where

 sin x  2 cos x  3
1
1 = dx

2 tan x / 2 1  tan 2 x / 2
Putting, sin x = , cos x = we get
1  tan x / 2
2
1  tan 2 x / 2


1
1 = dx
2 tan x / 2 2(1  tan 2 x / 2)
 3
1  tan 2 x / 2 1  tan 2 x / 2

 2 tan x / 2  2  2 tan
1  tan 2 x / 2
= dx
2
x / 2  3(1  tan 2 x / 2)

 tan
sec 2 x / 2
= dx
2
x / 2  2 tan x / 2  5

RESONANCE 17
TM
x 1 x x
Putting tan = t and sec2 = dt or sec 2 dx = 2 dt, we get
2 2 2 2

t
2dt
1 = 2
 2t  5

 x 

 (t  1)
dt  t  1  tan  1 
2 2
=2 = tan–1   = tan–1  
2
 22 2  2   2 
 
 

 x 
 tan  1 
 2  +C
Hence,  = x +  log | sin x + 2 cos x + 3 | +  tan–1
 2 
 
 

6 3 8
where  = ,= and  = –
5 5 5

 1 3 cos
dx
Example : 2
x


sec 2 x dx
Solution. =
tan 2 x  4

1  tan x 
= tan–1   +C
2  2 

Self Practice Problems

 5 sin x  4 cos x
4 sin x  5 cos x 40 9
1. dx Ans. x+ log |5sinx + 4cosx| + C
41 41

1 0 . Integration of type  sin x. cos n x dx


m

Case - 
If m and n are even natural number then converts higher power into higher angles.
Case - 
If at least m or n is odd natural number then if m is odd put cosx = t and vice-versa.
Case -
When m + n is a negative even integer then put tan x = t.

 sin x cos 4 x dx
5
Example:
Solution. put cos x = t  – sinx dx = dt

 (1  t ) . t4 . dt
2 2
=–

 (t  2t 2  1) t4 dt
4
=–

 (t  2t 6  t 4 ) dt
8
=–

t9 2t 7 t5
=– + – +c
9 7 5

cos 9 x cos 7 x cos 5 x


=– +2 – +c Ans.
9 7 5

RESONANCE 18
TM
 (sin x) (cos x ) 7 / 3 dx
1/ 3
Example :

 (sin x) (cos x ) 7 / 3 dx
1/ 3
Solution.


1
= (tan x )1/ 3 dx
cos 2 x
put tanx = t  sec2x dx = dt

t dt
1/ 3
=

3 4/3
= t +c
4
3
= (tanx) 4/3 + c Ans.
4

 sin x cos 4 x dx
2
Example :

 sin
1
Solution.
2
2x(1  cos 2x )dx
8

 sin  sin
1 1
=
2
2x dx + 2
2x cos 2x dx
8 8


1 1  sin3 2x 
(1  cos 4 x ) dx   
=  3 
16 16  

1 sin 4 x sin3 2x
= – + +c
16 64 48

1 1 . Integration of type


x2  1
dx where K is any constant.
x 4  Kx 2  1

1
Divide Nr & Dr by x² & put x  = t.
x


1 x2
Example : dx
1 x2  x4

 1 
1  2  dx


 x  1
Solution. 1 x+ =t
x2  2  1 x
x


dt
 –
t 1
2

1 t 1
– +C
2 t 1
n

1
x 1
1 x
– 1 +C
2 x  1
n
x

RESONANCE 19
TM
x
1
Example : Evaluate : dx
4
1
Solution. We have,


1
= dx
x 1
4

1
 = x 2
x2

1
dx

x2

x
1 x2
 = dx
2 1
2

x2

1 1
1 1 2
1
 x  x
2
 = 1 1 dx
2 x  2 x  2
2 2
x x

1 1
1 1
 
1 x2 1 x2
 = 1 dx – 1 dx
2 x2  2 2 x2  2
x x

1 1
1 1
 
1 x2 1 x2
 = dx – dx
2  1 2  1
2 2
x    2 x    2
 x   x

1 1
Putting x – = u in 1st integral and x + =  in 2nd integral, we get
x x

 u   2     2
1 du 1 d
= 2 – 2
2 2
2 2

1  u  1  2
= tan–1   – 1 log +C
2 2  2 2 2 2  2

1  x  1/ x  1 x  1/ x  2
= tan–1   – log +C
2 2  2  4 2 x  1/ x  2

1  x 2  1 1 x2  2 x  1
 
= tan–1   – log +C
2 2  2x 4 2 x2  x 2  1

Self Practice Problem :

1
x 3


x 1
2
1 x
1. dx Ans. 1 +C
x  7x  1 6 x 3
4 2 n
x


1  y  1 y 2 1
2. tan x dx Ans. tan–1   + + C where y = tan x –
2  2 2 2 y 2 tan x
n

RESONANCE 20
TM
1 2 . Integration of type

  ax
dx

dx
OR ; put px + q = t 2.
( ax  b px  q 2
bx  c px q

)
 ( x  3)
1
Example: Evaluate : dx
x 1

 ( x  3)
1
Solution. Let  = dx
x 1
Here, P and Q both are linear, so we put Q = t2 i.e. x + 1 = t2 and dx = 2t dt

 (t
1 2t
 = dt
2
 1  3) t 2

t
dt 1 t2
 =2 =2. log +C
2
2 2 2( 2) t 2

1 x 1 2
 = log + C.
2 x 1  2

 (x
x2
Example : Evaluate : dx
2
 3 x  3) x  1

 (x
x2
Solution. Let  = dx
2
 3 x  3) x  1

 {(t
( t 2  1) 2t dt
Putting x + 1 = t , and dx = 2t dt, we get  =
2
2
 1)2  3( t 2  1)  3} t 2

1
1
t 
( t 2  1) t2
 =2 dt = 2 1 dt
4
 t2  1 t2  2  1
t

 u   3
du 1
 =2 2 where t – = u.
2
t

 1
t
2  u  2  t 
 = tan–1   +C=  
 tan–1  3  +C
3  3 3
 

2  t 2  1 2  x 
    +C
 = tan  t 3  + C = 3 tan  3 ( x  1) 
–1 –1
3  

1 3 . Integration of type


dx

dx 1 1
, put ax + b = ; , put x =
t ax  b
2
px  q 2
t
ax  b px 2
 qx  r
(

)
(

RESONANCE 21
TM

dx
Example :
x 1 x2  x  1

)
 
 dt  dt
Solution = = 1 1
 1 1 
2
1 t  1
t2     1  t2 t
t t  t

 
 dt  dt
= =
t  t 1
2
 1 3
t   
 2 4

 
 1  1 3
2
= – n  t    t     + C
 2  2 4
 

 (1  x
dx
Example : 2
) 1 x 2

 (t
1 dt
Solution. Put x =  =
t 2
 1) t 2  1
put t2 – 1 = y2

 (y
y dy 1  y 
 =– =– tan–1   + C
2
 2) y 2  2

 1 x2 
1  
=– tan–1   +C
2  2x 

Self Practice Problems :

  
dx
1.
( x  2) x  1
Ans. 2 tan–1 x 1 + C

  
dx  x 1
–1  
x 1  2 
2. Ans. 2 tan–1 2 tan +C
( x 2  5 x  6) x  1 

3 
  1 

dx
3. Ans. sin–1  2 x 1 + C
( x  1) 1  x  x 2  5 
 
 2 


dx 1  1 x2 
 
4. Ans. – tan–1  2  + C
(2 x 2  1) 1  x 2 3  3x 


dx  x 2  2x  4  6 ( x  1) 
1  
5. Ans. – n  2  +C
( x 2  2 x  2) x 2  2 x  4 2 6  x  2x  4  6 ( x  1) 

RESONANCE 22
TM
1 4 . Integration of type

 dx or 
x α
x  α β  x  ; put x =  cos2  +  sin2 
βx

 dx or 
x α
x  α x  β  ; put x =  sec2   tan2 
x β


dx
 = t2 or x  = t2.
x  α x  β 
; put x

1 5 . Reduction formula of  tan x dx ,  cot x dx ,


 sec x dx ,  cos ec x dx
n n n n

n =  tan x dx =  tan x tann2 x dx =  (sec x  1) tann – 2x dx


n 2 2
1.

 n =  sec x tann2 + dx – n – 2
2

tann1 x
 n = – n – 2
n 1

n =  cot  cot . cot n 2 x dx =  (cos ec x  1) cot


n 2
2.
n
x dx = 2 2
x dx

 n =  cos ec x cot n2 x dx – n – 2


2

cot n1 x
 n = – – n – 2
n 1

n =  sec x dx =  sec x sec n 2 x dx


n 2
3.

  n = tanx secn – 2x –  (tan x)(n  2) sec n–3


x. secx tanx dx.
  n = tanx sec n – 2 x dx – (n – 2) (sec2 x – 1) sec n – 2x dx
 (n – 1) n = tanx sec n – 2x + (n – 2) n – 2

tan x sec n2 x n2


n = + 
n 1 n 1 n – 2

n =  cos ec dx =  cos ec x cosec


n 2
4. n–2
x dx

  n = – cotx cosec n – 2x +  (cot x)(n  2) (– cosec n–3


x cosec x cot x) dx

  cot x cos ec n2 x dx


2
– cotx cosecn – 2x – (n – 2)

  n = – cotx cosec n – 2x – (n – 2)  (cos ec x  1) cosecn – 2 x dx


2

 (n – 1) n = – cotx cosec n – 2 x + (n – 2) 2n – 2

cot x cos ec n2 x n2


n = + 
n 1 n 1 n – 2

RESONANCE 23
TM
Example : Obtain reducation formula for n =  sin x dx. Hence evaluate  sin x dx
n 4

Solution. n =  (sin x) (sin x) n –1


dx


= – cos x (sin x)n–1 + (n – 1)  (sin x) n–2


cos2x dx

= – cos x (sin x) n–1 + (n – 1)  (sin x) n–2


(1 – sin2x) dx
n = – cos x (sin x)n–1 + (n – 1) n–2 – (n – 1) n

cos x(sin x )n1 (n  1)


 n = – + n–2 (n  2)
n n

cos x(sin x )3 3   cos x(sin x )  1 x 


Hence 4 = – +   +C
4 4  2 2 

Self Practice Problems :

1. 
x 3
x4
dx Ans. ( x  3)( x  4) + n  x3  x4 + C 


dx  x 1 2  x 

2. Ans. 8  2  x  x 1  + C
[( x  1)(2  x )]3 / 2  


dx  x 1
1/ 7

3. Ans. 7   +C
[( x  2)8 ( x  1)6 ]1/ 7  x  2

 
dx dx
4. Deduce the reduction formula for n = and Hence evaluate 2 =
(1  x 4 )n (1  x 4 )2

x 4n  5
Ans. n = + 
4(n  1)(1  x ) 4 n 1
4(n  1) n–1

  1  1 
x 3  1 x  1 x  2 
2 = + tan 1 x  n  x  + C
4 (1  x )
4
4 2 2  2  4 2  1 
   x  2 
    x 

5. If m,n =  (sin x) m
(cos x) n dx then prove that

(sin x )m1(cos x )n1 n 1


m,n = + .
mn m  n m,n–2

RESONANCE 24
TM
Part : (A) Only one correct option

1.  [f ( x)g(x)  f ( x)g(x)] dx is equal to


f (x)
(A) g( x ) (B) f(x) g(x) – f(x) g(x)

(C) f(x) g(x) – f(x) g(x) (D) f(x) g(x) + f(x) g(x)


1
2. dx is equal to
sin x cos x
3

2 2
(A) +c (B) 2 tan x + c (C) +c (D) – 2 tan x – c
tan x tan x


n | x|
3. dx equals :
x 1  n |x|

2 2
(A) 1  n x (nx 2) + c (B) 1  n x (nx+ 2) + c
3 3
1
(C) 1  n x (nx 2) + c (D) 2 1  n x (3 nx 2) + c
3


x tan 1 x
4. If dx = 1 x 2 f(x) + A n (x + x 2  1 ) + C, then
1 x2
(A) f(x) = tan–1 x, A = –1 (B) f(x) = tan–1 x, A = 1
(C) f(x) = 2 tan–1 x , A = –1 (D) f(x) = 2 tan–1 x, A = 1

 1  2 sin
sin 8 x  cos 8 x
5. dx =
2
x cos 2 x

1 1 1
(A) sin 2x + c (B) – sin 2x + c (C) – sin x + c (D) – sin2x + c
2 2 2


ax ax
6.  dx is equal to
ax ax

(A) – 2 a 2  x 2 + C (B) a2  x 2 + C (C) – x 2  a2 + C (D) none of these

7.  tan(x  ) tan(x  ) tan 2x dx is equal to


sec 2x . sec( x  ) sec 2x
(A) n +C (B) n sec( x   ) sec( x   ) + C
sec( x   )

sec 2x . sec( x  )
(C) n +C (D) none of these
sec( x   )

RESONANCE 25
TM
8.
 sec x  1 dx is equal to

 x 2 x 1   x 2 x 1 
 
(A) 2 n  cos 2  cos 2  2  + C (B) n  cos 2  cos 2  2  + C
   

 x 2 x 1 

(C) – 2 n  cos 2  cos 2  2  + C (D) none of these
 

 cos
dx
9. is equal to
3
x sin 2x

 1   1 
(A) 2 
cos x  tan 5 / 2 x  + C (B) 2 
tan x  tan 5 / 2 x  + C
5  5 

 1 5/2 
(C) 2  tan x  5 tan x + C

(D) none of these

3 x4  1
x 
10. Primitive of w.r.t. x is:
x1
4 2

x x
(A) +c (B)  +c
x x1
4
x x1
4

x1 x1
(C) +c (D)  +c
x x1
4
x x1
4

x4  1
If 
B
 
11. dx = A n x + + c, where c is the constant of integration then:
x x2  1
2
1  x2

(A) A = 1; B =  1 (B) A =  1; B = 1
(C) A = 1; B = 1 (D) A =  1; B =  1


1 x
12. dx equals :
1 x

(A) x 1  x  2 1  x + cos 1  x + c (B) x 1  x + 2 1  x + cos 1  x + c


(C) x 1  x  2 1  x  cos 1  x + c (D) x 1  x + 2 1  x  cos 1  x + c
13.  sin x. cos x. cos 2x. cos 4x. cos 8x. cos 16 x dx equals:

sin 16 x cos 32 x cos 32 x cos 32 x


(A) +c (B)  +c (C) +c (D)  +c
1024 1024 1096 1096

RESONANCE 26
TM

1
14. d x equals :
cos x  sin 6 x
6

(A) tan 1 (tan x + cot x) + c (B)  tan 1 (tan x + cot x) + c


(C) tan 1 (tan x  cot x) + c (D)  tan 1 (tan x  cot x) + c

 ln(1  sin x)  x tan 4  2  dx is equal to:


  x 
15.

(A) x n (1 + sinx) + c (B) n (1 + sin x) + c


(C) – x n (1 + sin x) + c (D) n (1 – sin) + c


dx
16. equals:
cos x . sin 2x
3

2 2
(A) (tan x) 5/2 + 2 tan x + c (B) (tan2 x + 5) tan x + c
5 5

2
(C) (tan2 x + 5) 2 tanx + c (D) none
5

If 
dx
17. = a cot x + b tan 3 x + c where c is an arbitrary constant of integration then the
sin x cos x
3 5

values of ‘a’ and ‘b’ are respectively:


2 2 2
(A)  2 & (B) 2 &  (C) 2 & (D) none
3 3 3

x
x2  1
18. dx is equal to [IIT - 2006, (3, –1)]
3
2 x 4  2x 2  1

2x 4  2x 2  1 2x 4  2x 2  1
(A) +c (B) +c
x2 x3

2x 4  2x 2  1 2x 4  2x 2  1
(C) +c (D) +c
x 2x 2

Part : (B) May have more than one options correct

x  1 dx
If 
f (x )
19. is equal to + c then
x2 2 x 2  2x  1 g (x )
(A) f(x) = 2x2 – 2x + 1 (B) g(x) = x + 1

(C) g(x) = x (D) f(x) = 2x 2  2x

 x

dx
20. =  tan1  m tan  + C then:
5  4 cos x  2
(A) l = 2/3 (B) m = 1/3 (C) l = 1/3 (D) m = 2/3

RESONANCE 27
TM
3 cot 3x  cot x
21. If  dx = p f(x) + q g(x) + c where 'c' is a constant of integration, then
tan x  3 tan 3x

1 3  tan x
(A) p = 1; q = ; f(x) = x; g(x) = n
3 3  tan x
1 3  tan x
(B) p = 1; q =  ; f(x) = x; g(x) = n
3 3  tan x
2 3  tan x
(C) p = 1; q =  ; f(x) = x; g(x) = n
3 3  tan x
1 3  tan x
(D) p = 1; q =  ; f(x) = x; g(x) = n
3 3  tan x


sin 2 x
22. dx is equal to:
sin x  cos4 x
4

(A) cot 1 (cot2 x) + c (B)  cot 1 (tan2 x) + c


(C) tan 1 (tan2 x) + c (D)  tan 1 (cos 2 x) + c

x 1
n  x  1
23.  x2  1
dx equal:

1 2 x 1 1 2 x 1 1 2 x 1 1 2 x 1
(A) +c (B) +c (C) +c (D) +c
2 x 1 4 x 1 2 x 1 4 x 1
n n n n


n (tan x)
24. dx equal:
sin x cos x

1 2 1 2
(A) n (cot x) + c (B) n (sec x) + c
2 2
1 2 1 2
(C) n (sin x sec x) + c (D) n (cos x cosec x) + c
2 2


cos ec 2 x. sin x
1. Integrate with respect . dx
(sin x  cos x )

1 x2
2. Integrate with respect to x
1 x2  x4

1
3. Integrate with respect to x
( x  1) x   2

(x  1)2 2 sin 2   cos 


4.  x 4  x2  1
dx 5.  6  cos 2   4 sin 
d

RESONANCE 28
TM

tan   tan 3 

cos 5x  cos 4x
6. d 7. dx
1  tan  3
1  2 cos 3x


1  cos  cos x

3  4 sin x  2 cos x
8. dx 9. dx
3  2 sin x  cos x cos   cos x

 (x  ) 
x 
dx dx
10. 11.
( x   ) ( x  ) 3
 3 x  3x  1
2
x 2  2x  3

 cos
x  1
x3  x  2 (cos 2x  3)
12.  ex
2
2
dx 13. 4
x 4  cot 2 x
dx

   
 
 x 2 1 ln x 2 1  2ln x
 dx
14. dx 15. , (a > b)


x4 

a  b cos x 2

  7 x  10  x 
cos ec x  cot x sec x x
16. . dx 17. dx
cos ec x  cot x 1  2 sec x
2
3/ 2

 
2  x  x2
18. dx 19. tan 1 x. n (1 + x2) dx.
x2

a  b sin x
 
dx
 b  a sin x x  1
20. 2 dx 21. 2
x 4 3

1  x cos x

 
22. dx
x 1  x 2 e2 sin x

x cos   1

f ( x)
x 
23. dx = + c then find f(x) and g(x)
2
 2 x cos   1
3/ 2
g( x)


n (1  sin2 x )
24. Evaluate dx.
cos 2 x

x3  3x  2
Integrate, 
x  1
25. dx. [IIT - 1999, 7]
(x  1)
2
2

 2 x 
26. For any natural number m, evaluate,

 x
1/ m
3m
 x2 m  x m 2m
 3x m  6 d x, x > 0. [IIT - 2002, 5]

RESONANCE 29
TM
EXERCISE # 1  x1
12. ex  2  +c

1. C 2. A 3. A 4. A 5. B 6. A  x  1

7. B 8. C 9. B 10. B 11. C 12. A 1


13. c – tanx.(2 + tan2x). 4  cot 2 x
3
13. B 14. C 15. A 16. B 17. A 18. D

19. AC 20. AB 21. AD 22. ABCD 23. BD x 2


1  x2 1  1 
. 2  3 ln 1  2  
14.  
9 x3  x
24. ACD

a  a  b 
bsinx 2a
EXERCISE # 2 15. – 2
 b (a  bcosx )
2 +
2 2 3/2

1 ab x
x
 3 arctan .tan + c
x 1 x ab 2
1. n 1 2 tan + c 2. – 1 +c
2 2 3
n
x  x 1 x
x 16. sin1  sec 2  + c
2 2
2 (7 x  20)
1  1  1
2
2 17. c
3. – n  t  3    t  3   9 +c 9 7 x  10  x 2
3
 
2  x  x2 2 4x2 2 2xx 
2
1
where t= 18. – + ln  
x 1 x 4 x
 

 x 2  1 2  2x 2 1 
1      2 x1 
4. tan  x 3   3 tan  3  +c – sin–1  + c
1 1

3      3 
19. x tan 1 x. n (1 + x2) + (tan 1 x)2  2x tan 1 x
5. 2 n sin   4sin   5 + 7 tan 1(sin 2) + c
2

+ n (1 + x2)   n 1  x 


2
1 1 1 +c
2

6.  n1+tan+ ntan2  tan  + 1+


3 6 3
cos x
 2 tan   1 20. – c
tan 1   +c b  a sin x
 3 

 
2 x 3 1 1 1
21.  3  +c
sin 2x 3 3x 3 x3 1
n
x 3
7.  (sin x  )+ c
2
1
2 x  3 arc tan  tan  1  c
x 22. ln (x esinx) - ln (1 - x2 e2 sinx) + c
8.
 2
2 

23. x; x 2 + 2x cos  + 1
 cos 1 (  x) 
 2 
x cos  + sin  n 
1 (  x) 
9. +c
 cos 2 
 24. tan x ln (1 + sin2x) – 2x + 2 tan–1 ( 2 .tan x) + c.

3 1 1 x
2 x 25. tan-1 x - n (1 + x) + n (1 + x2) + +c
10. . c 2 2 4 1  x2
   x

m1
x 2  2x  3 1  2  z m
11. + . cos1   +c
8 (x  1) 2 16  x  1 26.
6 (m  1)
+ c, where z = 2 x3m + 3 x2m + 6 xm

RESONANCE 30
TM
Definite Integrals

PART A :
A Let f(x) be a continuous function defined on [a, b],

  f (x) dx = F(b) – F(a) is called definite


b
f ( x ) dx = F(x) + c. Then
a

integral. This formula is known as Newton-Leibnitz formula.

Note :

But j ust as much as 1. The indefinite integral  f (x) dx is a function of x, where as definite
 f (x) dx is a number..
b
i t i s easy t o f i nd t he
integral
a
di f f er ent i al of a

 f (x) dx we can find  f (x) dx , but given  f (x) dx we can-


b b
gi ven quant i t y, so i t 2. Given
a a

 f (x) dx
i s di f f i cul t t o f i nd t he
not find
i nt egr al of a gi ven


2
di f f er ent i al . dx
Illustration. 1 Evaluate ( x  1)( x  2)
Mor eover , somet i mes 1

1 1 1
we cannot say wi t h Sol. = – (by partial fractions)
( x  1)( x  2) x 1 x2


cer t ai nt y whet her 2

( x  1)( x  2) = loge ( x  1)  loge ( x  2)1


dx
2
t he i nt egr al of a 1

gi ven quant i t y can 9


 
= loge – loge – loge + loge = loge
3 4 2 3 8

be f ound or not . Self Practice Problems


Evaluate the following
– Johann Ber noul l i


 3
2
5x 2 5 5
1. dx Ans. 5–  9 loge 4  log e 2 
1
x  4x  3
2
2  

 2 sec 

4 
2
2.
2
x  x 3  2 dx Ans. + +2
1024 2
0


3
x 2   2 
 
3. dx Ans. – + 2 loge  
1  sec x 18 3 3  3
0

RESONANCE TM 31
PART B :
Properties of definite integral

 f (x ) dx =  f (t) dt
b b

P–1
a a

i.e. definite integral is independent of variable of integration.

 f (x ) dx = –  f (x ) dx
b a

P–2
a b

   f (x ) dx, where c may lie inside or outside the interval [a, b].
b c b

P–3 f ( x ) dx = f ( x ) dx +
a a c

 f (x ) dx
 x3 : x 3 5

Illustration 2 If f(x) =  2 , then find


3 x  1 : x  3
2

   f (x ) dx
5 3 5

Sol. f ( x ) dx = f ( x ) dx +
2 2 3

  (3 x
3 5

= ( x  3 ) dx + 2
 1) dx
2 3

94 211
= + 3 (3 – 2) + 53 – 33 + 5 – 3 =
2 2


8

Illustration 3 Evaluate | x  5 | dx
2

   ( x  5) dx
8 5 8

Sol. | x  5 | dx = (  x  5) dx + =9
2 2 5

   (2x  1)
2 5 2

Illustration 4 Show that (2x  1) dx = ( 2x  1) +


0 0 5

Sol. L.H.S. = x + x ] 0 = 4 + 2 = 6
2 2

R.H.S. = 25 + 5 – 0 + (4 + 2) – (25 + 5) = 6
 L.H.S. R.H.S

Self Practice Problems


Evaluate the following

| x
2

1.
2
 2x  3 | dx Ans. 4
0

 [x] dx , where [x] is integral part of x.


3

2. Ans. 3
0

  t  dt
9

3. Ans. 13
0

RESONANCE TM 32
PART C :

 f (x) dx =  (f ( x )  f ( x)) dx
a a

P–4
a 0

 f (x ) dx
a

=2 if f(–x) = f(x) i.e. f(x) is even


0

=0 if f(–x) = –f (x) i.e. f(x) is odd


e x  ex
1

Illustration 5 Evaluate dx
1
1 ex

 
 e x  e x ex  e x 
1
e x  ex
1
  
Sol. dx =  1 ex 1 ex  dx
1
1 ex 0  

 
 e x  e  x e x (e  x  e x ) 
1
(e 1  1) e2  1
1
   (e x  e  x ) dx = e – 1 +
=  1 ex ex  1  dx = =
0   1 e
0

 cos x dx
2
Illustration 6 Evaluate


2
 

 cos x dx = 2  cos x
2 2
Sol. dx = 2 ( cos x is even function)
 0

2

 log
2x
1

Illustration 7 Evaluate e  dx
1
2 x
2x
Sol. Let f(x) = loge  
2x

2x 2x
 f(–x) = loge   = – log   = – f(x)
 2  x  e
2x
i.e. f(x) is odd function

 log
2x
1

 e  dx = 0
1
2 x

Self Practice Problems


Evaluate the following

 | x | dx
1

1. Ans. 1
1

 sin
2
2. 7
x dx Ans. 0


2


2
cos x
3. dx Ans. 1

1 ex

2

RESONANCE TM 33
PART D :

  f (a  b  x ) dx
b b

P–5 f ( x ) dx =
a a

  f (a  x) dx
a a

Further f ( x ) dx =
0 0

 

 

2 2
g (sin x ) g (cos x )
Illustration 8 Prove that dx = dx =
g (sin x )  g (cos x ) g (sin x )  g (cos x ) 4
0 0


2
g (sin x )
Sol. Let  = dx
g (sin x )  g (cos x )
0

    
g  sin  x  
  g (cosx)  g (sinx)
 2  g (cosx)
2 2
 = = dx
     
0 g  sin  x    g  cos  x   0
 2   2 
on adding, we obtain
 

  g (sin x )  g (cos x )  g (cos x)  g (sin x)  dx 


  
2 2
g (sin x ) g (cos x )
2 = = dx =
4
0 0

Note : 1. The above illustration can be remembered as a formula

2. Other similar formulae are


 

 

2 2
g (tan x ) g (cot x )
dx = dx =
g (tan x )  g (cot x ) g (tan x )  g (cot x ) 4
0 0

 

 

2 2
g (cos ecx ) g (sec x )
dx = dx =
g (cos ecx )  g (sec x ) g (cos ecx )  g (sec x ) 4
0 0


a
g (x) a
g ( x )  g (a  x ) dx = 2
0

Self Practice Problems


Evaluate the following

 1  sin x

x
1. dx Ans. 
0

 

 sin x  cos x
2
x 
2. dx Ans. loge 1 2
2 2
0

RESONANCE TM 34

 sin
2
2
x sin x cos x
3. dx Ans.
0
4
x  cos 4 x 16

 1
3
dx

4. tan x Ans.
 12
6

PART E :

  (f ( x)  f (2a  x)) dx
2a a

P–6 f ( x ) dx =
0 0

 f (x ) dx
a

=2 if f (2a – x) = f(x)
0

=0 if f (2a – x) = –f(x)

 sin

Illustration 9 Evaluate
3
x cos 3 x dx
0

Sol. Let f(x) = sin x cos3x


3
 f( – x) = – f(x)

 sin

 3
x cos 3 x dx = 0
0

 1  2 sin

dx
Illustration 10 Evaluate dx
0
2
x

1
Sol. Let f(x) =
1  2 sin2 x
 f( – x) = f(x)
 

 1  2 sin  1  2 sin 

dx 2
dx
2
sec 2 x dx
 =2 =2
x 1  tan2 x  2 tan 2 x
2
0 0
2
x 0

tan  3 tan x  

 1  3 tan
2
sec 2 x dx 2 
=2 = 1 2
2
x 3 0
0


tan is undefined, we take limit
2

   
 
2  
 Lt  tan
1
3 tan x  tan 1
3 tan 0 
=
3 x 
 2

2  
= =
3 2 3

Note : We can evaluate the integral without using this property

RESONANCE TM 35
 1  2 sin  
  
dx cos ec 2 x cos ec 2 x dx
Alternatively : = dx =
0
2
x 0
cos ec 2 x  2 0
cot 2 x  3
Observe that we are not converting in terms of tan x as it is not continuous in (0, )

  cot x 

1  1 cot x  1 1 cot x 
=– tan   = –
  Lt  tan    Lt tan 1
  3 

3   3  0 3  x    3  x 0  

1    
=–  2  2  =
3   3
Note : If we convert in terms of tan x, then we have to break integral using property P – 3.
  

 log  log  log (sin 2x) dx = –



2 2 2
Illustration 11 Prove that sin x dx = cos x dx = loge2 .
2
e e e
0 0 0

 log
2
Sol. Let  = e sin x dx ..........(i)
0

 log  sin  2  x   dx


2
  
 = e (by property P – 5)
0

 log (cos x ) dx
2
= e ..........(ii)
0
Adding (i) and (ii)
 

 log (sin x . cos x ) dx =  log


 sin 2x 
2 2
2= e  dx
 2 
e
0 0
 

 log  log
2 2
2= e (sin 2x ) dx – dx
2
e
0 0


2  = 1 – log2e ..........(iii)
2

 log
2
where  = e (sin 2x ) dx
0

1
put 2x = t  dx = dt
2
L.L:x=0  t=0

U.L:x=  t=
2

 log (sin t )

1
 1 = dt
e
2
0

 log (sin t ) dt
1
2
= ×2 (by using property P – 6)
2
e
0


 1 =   (iii) gives  = – log2e
2

RESONANCE TM 36
Self Practice Problems
Evaluate the following

  1
 loge  x   


  x
1.  1 x 2  dx : Ans :  loge2
0  
 


1
sin 1 x 
2. x dx : Ans : loge2
0 2



2
3. x log e sin x dx Ans : – loge2
0
2

PART F :

P–7 If f(x) is a periodic function with period T, then

  f (x ) dx, n  z
nT T

(i) f ( x ) dx = n
0 0

  f (x ) dx, n  z, a  R
a nT T

(ii) f ( x ) dx = n
a 0

  f (x ) dx, m, n  z
nT T

(iii) f ( x ) dx = (n – m)
mT 0

  f (x ) dx, n  z, a  R
a nT a

(iv) f ( x ) dx =
nT 0

  f (x ) dx, n  z, a, b  R
b  nT a

(v) f ( x ) dx =
a  nT a

e
2
Illustration 12 Evaluate {x}
dx
1

e e  e
2 1 3 1 1
Sol. {x}
dx = {x}
dx = 3 e {x}
dx = 3 {x}
dx = 3(e – 1)
1 1 0 0

 | cos x | dx ,
n  v

Illustration 13 Evaluate < v <  and n  z
2
0

 | cos x | dx =  | cos x | dx +  | cos x | dx


n  v v n  v

Sol.
0 0 v

   | cos x | dx
2 v 

= cos x – cos x dx + n
0  0

RESONANCE TM 37

 cos x
2
= (1 – 0) – (sin v – 1) + 2n dx
0

= 2 – sin v + 2n (1 – 0) = 2n + 2 – sin v

Self Practice Problem


Evaluate the following

e
2
{3 x }
1. dx Ans. 3 (e – 1)
1


2000 
dx
2. dx Ans. 1000 
0
1  e sin x

5



4
sin 2x
3. dx Ans.

sin 4 x  cos 4 x 4

PART G :

P–8 If (x)  f(x)   (x) for a  x  b, then

   (x) dx
b b b
( x ) dx  f ( x ) dx 
a a a

 f (x ) dx  M (b – a)
b
P–9 If m  f(x)  M for a  x  b, then m (b – a) 
a

 f (x ) dx < f(a) (b – a) and if f(x)


b
Further if f(x) is monotonically decreasing in (a, b) then f(b) (b – a) <
a

 f (x ) dx < f(b) (b – a)
b
is monotonically increasing in (a, b) then f(a) (b – a) <
a

 
b b

P – 10 f ( x ) dx  f ( x ) dx
a a

 f (x) dx  0
b

P – 11 If f(x)  0 on [a, b] then


a

1 1 1
Illustration 14 For x  (0, 1) arrange f1(x) = , f2(x) = and f3(x) = in ascending
4  x2 4  2x 2 4  x2  x3



1
 dx
order and hence prove that < <
6 0 4  x2  x3 4 2
Sol.  0 < x 3 < x2  x2 < x2 + x3 < 2x2  –2x2 < – x2 – x3 < –x2
 4 – 2x < 4 –x – x < 4 – x2
2 2 3

 4  2x 2 < 4  x2  x3 < 4  x2

RESONANCE TM 38
 f1(x) < f3(x) < f2(x) for x  (0, 1)

   f (x) dx
1 1 1

 f1( x ) dx < f3 ( x ) dx < 2


0 0 0


 x  x 
1 1 1
dx 1
sin–1   < < sin–1 
 2  0 0 4  x2  x3 2 2 0



1
 dx
< <
6 0 4x x 2 3 4 2


2
sin x
Illustration 15 Estimate the value of dx
x
0

sin x
Sol. Let f(x) =
x
x cos x  sin x (cos x )( x  tan x )
f(x) = = <0
x 2
x2
 f(x) is monotonically decreasing function.
f(0) is not defined, so we evaluate

Lt Lt  sin x = 1. Take f(0) = Lt  f(x) = 1


f(x) = x 0
x 0  x x 0

 2
f   =
 
2 


2   2
sin x  
.   0 < dx < 1 .   0 
 2  x 2 
0



2
sin x
1< dx <
x 2
0

Note : Here by making the use of graph we can make more appropriate approximation as in next illustration.

e
1
Illustration 16 Estimate the value of x2
dx using (i) rectangle, (ii) triangle
0

Sol. (i) By using rectangle

e
1

Area OAED <


x2
dx < Area OABC
0


1

1< e x dx < 1 . e
2

e
1

1<
x2
dx < e
0

RESONANCE TM 39
(ii) By using triangle

e
1

Area OAED <


x2
dx < Area OAED + Area of triangle DEB
0

 e
1 1

1 < e dx < 1 +
1 x2
. 1. (e – 1) 1<
x2
dx < e  1
0 2 0 2

  e dx
1 1
Illustration 17 Estimate the value of e x dx by using x
2

0 0

Sol. For x  (0, 1), e x < ex


2

e  e dx
1 1
 1×1< x2
dx < x

0 0

e
1
1< x2
dx < e – 1
0

Exercise : Prove the following :

 
1 1

1. e  x cos 2 x dx < e  x cos 2 x dx


2

0 0

 

 sin  sin
2 2
n 1
2. 0< x dx < 2
x dx
0 0

e
1 1
 x2 x
3. e 4 < dx < 1
0


1
1 x 3 cos x 1
4. –  dx <
2 0
2  x2 2



2
5. 1< sin x dx <
2
0

 16  x
2
x dx 1
6. 0< <
6
3
0

RESONANCE TM 40
PART - H

 f (t) dt , then
h( x )
Leibnitz Theorem : If F(x) =
g( x )

dF( x )
= h(x) f(h(x)) – g(x) f(g(x))
dx

Proof : Let P(t) =  f (t) dt

 f (t) dt = P(h(x)) – P(g(x))


h( x )

 F(x) =
g( x )

dF( x )
 = P(h(x)) h(x) – P(g(x)) g(x)
dx
= f(h(x)) h(x) – f(g(x)) g(x)


x2

Illustration 18 If F(x) = sin t dt , then find F(x)


x

Sol. F(x) = 2x . sin x 2 – 1 . sin x


e3 x
t
Illustration 19 If F(x) = dt , then find first and second derivative of F(x) with respect to log xe
log et
e2 x

at x = log2e

 
dF( x ) dx  e3x 
dF( x ) 2x e
2x
Sol. = = 3 . e 3 x .  2 . e  x = e6x – e4x.
d logex dx d (log x )  logee
3x
logee
2x


   
d2F( x ) d d 1
= (e6x – e4x) = (e6x – e4x) × = (6 e6x – 4 e4x) x
d logex
2
d logex dx d log ex
dx
First derivative of F(x) at x = log2e (i.e. ex = 2) is 26 – 24 = 48
Second derivative of F(x) at x = log e (i.e. ex = 2) is (6 . 26 – 4 . 24) . loge2 = 5 . 26 . log2e.
2


x 2 
2
 e t dt 
 
0 
Illustration 20 Evaluate xLt

e
 x
2t 2
dt
0


x 2 
2
 e t dt 
   
Sol. Lt  0   form 

e
x  x 
2t 2
dt
0

Applying L Hospital rule

RESONANCE TM 41

x
2 . e t dt . e x
2 2

= Lt 0
x 
1 . e 2x
2


x
2 . e t dt
2

2 . ex
2

= Lt 0 = Lt =0
x  x  2x . e x
2

ex
2

Modified Leibnitz Theorem :

 f ( x, t ) dt, then
h( x )

If F(x) =
g( x )


 f ( x, t )
h( x )

F(x) = dt + f(x, h(x)) h(x) – f(x, g(x)) . g(x)


x
g( x )


x
dt
Illustration 21 If f(x) = x  t , then find f(x)
log ex

  
1 
x  log 
x
1
x
1 1 1 1 1
= (x  t)  x +
x  t 2
Sol. f(x) = dt + 1 . – –
2x x x
e
 loge 2x x x  log ex
logex

     
1 1 1 1 1 x 1 logex  1
= – x + – = – x =
2x x  log e 2x x x  log ex x x x  log e x x  logex


x


x
dt
Alternatively : f(x) =  log e ( x  t ) (treating ‘t’ as constant)
xt 
log ex  logex

f(x) = loge2x – loge (x + log xe)

   
1 1  1 log ex  1
f(x) = – 1   =
x x  log ex  x x x  log ex

Definite Integrals dependent on parameters :


xb  1
1

Illustration 23 Evaluate , ‘b’ being parameter


0
log ex


xb  1
1

Sol. Let (b) = dx


0
log ex


1
d (b ) x b log ex
= dx + 0 – 0
db logex
0

(using modified Leibnitz Theorem)

RESONANCE TM 42

x b 1 
1
1
1
= x dx =
b
 =
b  1 b 1
0 0

(b) = loge (b + 1) + c
b=0  (0) = 0
 c=0  (b) = loge (b+1)


1
tan 1(ax )
Illustration 24 Evaluate dx , ‘a’ being parameter
0 x 1 x2


1
tan 1(ax )
Sol. Let (a) = dx
0 x 1 x2

 
1 1
d (a) x 1 dx
= dx =
da 0
(1  a 2 x 2 ) x 1  x 2 0 (1  a x ) 1  x 2
2 2

Put x = sin t  dx = cos t dt


L.L. : x = 0  t=0

U.L. : x = 1  t=
2
 

 1 a  1 a
d (a) 2
1 1 2
dt
= cos t dt =
da 2
sin t cos t
2 2
sin 2 t
0 0

 1  (1  a

2
sec 2 t dt 1  1  a 2 tan t  2
= = tan –1
 
2
) tan 2 t 1 a2   0
0

1 
= .
1 a2 2

  2 
 (a) = loge  a  1  a  + c
2  
But (0) = 0  c=0
  2 
 (a) = loge  a  1  a 
2  

Self Practice Problems :


x3
1. If f(x) = cos t dt, find f(x). Ans. 3x2 cos x 3
0

 1 t
x
t 2
2. If f(x) = eg(x) and g(x) = 4 dt then find the value of f(2). Ans.
2 17


y
dt d2 y
3. If x = and = Ry then find R Ans. 4
0 1  4t 2 dx 2

x
x2
4. If f(x) =
2
sin t dt then find f(x). Ans. x2 (2x sin x2 – sin x) + (cos x – cos x2) x
x

RESONANCE TM 43
 ( x  t ) (t) dt, then find the value of (x) + (x). Ans.
x
5. If (x) = cos x – – cos x
0

 (log  2
x
6. Find the value of the function f(x) = 1 + x + e)
t 2
 2 log et dt where f(x) vanishes. Ans. 1 +
e
1

 cos t
x2
2
dt
7. Evaluate xLt . Ans. 1
0
0 x sin x

 1  1  b2 


 
8. Evaluate log e (1  b cos x ) dx, ‘b’ being parameter.. Ans.  loge  2 
0  

PART - 

Definite Integral as a Limit of Sum.


Let f(x) be a continuous real valued function defined on the closed interval [a, b] which is divided into n parts
as shown in figure.

y = f(x)

a a+h a+2h ................ a+(n-1)h a+nh=b


x

ba
The point of division on x-axis are a, a + h, a + 2h ..........a + (n – 1)h, a + nh, where = h.
n
Let Sn denotes the area of these n rectangles.
Then, Sn = hf(a) + hf(a + h) + hf(a + 2h) + ........+hf(a + (n – 1)h)
Clearly, Sn is area very close to the area of the region bounded by curve y = f(x), x–axis and the ordinates
x = a, x = b.

 f (x) dx =
b
Hence Lt Sn
n
a

   
ba 
n 1
b n 1
(b  a ) r 
f ( x ) dx = Lt h f (a  rh) = Lt  f a  
n n r0
n   n 
a r 0

Note :
1. We can also write

  
ba  ba 
b n

Sn = hf(a + h) + hf (a + 2h) + .........+ hf(a + nh) and f ( x ) dx = Lt  f  a    r 


n   r 1 n    n  
a

RESONANCE TM 44
 n f  n 
1 r

1 n 1

2. If a = 0, b = 1, f ( x ) dx = n Lt

r 0
0

Steps to express the limit of sum as definte integral


r 1
Step 1. Replace by x, by dx and n Lt  by 
n n 

r
Step 2. Evaluate n Lt  
   n  by putting least and greatest values of r as lower and upper limits respectively..

  f (x) dx
1 r  r r
pn p

For example n Lt f  ( n Lt   = 0, n Lt  

r 1
n n =  n
r 1
 n
r  np
= p)
0

Illustration 25 : Evaluate
 1 1 1 1
1  n  2  n  3  n  .........  2n 
Lt
n
 

 1 1 1 1
Sol. Lt 1  n  2  n  3  n  .........  2n 
n  

 r n
1
n

= n Lt

r 1

 
1
1
x  1 = log e ( x  1)0 = loge2.
1
n
dx
= n Lt
 n r = 1
r 1   1 0
n

 n 1 n2 n3 3
Illustration 26 : Evaluate n Lt  2  2  ......... 
 
n  1
2 2
n 2 2
n 3 2 5n 

n 
1
nr 1
2n 2n
Lt Lt n
Sol.  r2 = n   n  
2 2
n r
r 1 r 1
1  
n

r
Lt   = 0, when r = 1, lower limit = 0
n
 n

r  2n 
and Lt   = Lt   = 2, when r = 2n, upper limit = 2
n n n  n 

 1 x  1 x  1 x
1 x
2 2 2
1 1 2x
dx = dx + dx
2 2
2 2
0 0 0

2 
2
1
= tan x] –1 2
+ log e (1  x )
0 2 0

1
= tan–1 2 + loge5
2

RESONANCE TM 45
Illustration 27 : Evaluate
1
Lt  n!  n
 n
n 
n

1
 n!  n
Sol. Let y = n Lt
  n 
n 

1  n! 
loge y = n Lt loge  n 

n n 

1  1 . 2 . 3 ........n 
= n Lt loge  

n  nn 

1 log  1   log  2   log  3   .....  log  n 


= n Lt  e 
n n n  n 
e e e

n 

 log  n 
r
n
1
= n Lt e

n r 1



1
1

= log e x dx  x log e x  x 

0 0

= (0 – 1) – Lt x logex + 0

x 0
= – 1 – 0 = –1
1
 y=
e
Self Practice Problems :

Evaluate the following limits

 2  1
 1 1 1 1 
1. Lt     ....   Ans. 2
 n n2  n n 2  2n n 2  n 2 
n 2

 1 1 1 1
1  n  2  n  3  n  .......  5n 
2. Lt Ans. loge5
n
 

1  3  2 3 n  2
Lt sin  2 sin 3  3 sin 3  ........  n sin3
3. 2  4n 
Ans. (52 – 15)
n
n  4n 4n 4n 9 2


n 1
1 
4. Lt Ans.
n
r 0 n  r2
2
2

3 1  n

n

n
 ...... 
n 
5. Lt   Ans. 2
n
n  n3 n6 n9 n  3(n  1) 

RESONANCE TM 46
PART – J

Reduction Formulae in Definite Integrals


 sin
2
 n  1
1. If n =
n
x dx , then show that n =   
 n  n–2
0

 sin
2
Proof : n = x dx
n

 

 (n  1) sin
 2
n =  sinn1 x cos x 2 + n 2
x . cos 2 x dx
0
0

 sin
2
= (n – 1)
n2
x . (1  sin 2 x ) dx
0

 

 sin  sin
2 2
= (n – 1)
n2
x dx  (n  1) n
x dx
0 0

n + (n – 1) n = (n – 1) n–2
 n  1
n =   
 n  n–2
 

 sin  cos
2 2
Note : 1.
n
x dx = n
x dx
0 0

 n  1  n  3   n  5 
2. n =       .....  or 
 n  n2 n 4 0 1


according as n is even or odd. 0 = , =1
2 1

 n  1  n  3  n  5   1 
   ........  . if n is even
 n  n  2  n  4  2 2
Hence n = 
  n  1  n  3  n  5  2
    ........  . 1 if n is odd
  n  n  2  n  4  3

 tan
1
4
2. If n = x dx , then show that n + n–2 =
n
n 1
0

 (tan x)
4
n =
n2
Sol. . tan2x dx
0

 (tan x)
4
n2
= (sec2x – 1) dx
0

RESONANCE TM 47
 

 (tan x)  (tan x)
4 4
n2 n2
= sec2x dx – dx
0 0


 (tan x )n 1  4
=   – n–2
 n  1  0

1
n = – n–2
n 1

1
 n + n–2 =
n 1

 sin
m 1
2
3. If m,n = x . cosn x dx , then show that m,n =  ,n
m
m  n m–2
0

 sin
2
m,n =
m 1
Sol. x (sin x cosn x ) dx
0

 


 sinm 1 x . cosn 1 x  2 2
cos n 1 x
=   + (m – 1) sinm–2 x cos x dx
 n 1  0 n 1
0

 sin
 m  1 2
=   m2
x . cos n x . cos 2 x dx
 n 1
0

 sin 

 m  1 2
=   x . cos n x  sinm x . cos n x dx
m2
 n 1
0

 m  1  m  1
=    –   
 n 1 m–2,n
 n  1  m,n

 m  1  m  1
 1    =   
 n 1  m,n
 n  1  m–2,n

 m 1
m,n =   
 m  n  m–2,n

RESONANCE TM 48
 m 1  m  3   m  5 
Note : 1. m,n =       ........  or  according as m is even or odd.
mn mn2 mn 4 0,n 1,n

 

 cos  sin x . cos


1
2 2
0,n = x dx and 1,n = x dx =
n n
n 1
0 0

2. Walli’s Formula

 (n  1) (n  3) (m  5) .........( n  1) (n  3) (n  5)....... 
 when both m, n are even
(m  n) (m  n  2) (m  n  4)........ 2

m,n = 
 (m  1) (m  3) (m  5) .........( n  1) (n  3) (n  5).......
 otherwise
 (m  n) (m  n  2) (m  n  4)........

 sin
2
Illustration 28 : Evaluate
2
x cos 2 x(sin x  cos x ) dx


2

 

 sin  sin
2 2

Sol. Given integral =


3
x cos 2 x dx +
2
x cos 3 x dx
 
 
2 2

 sin
2
=0+2
2
x cos 3 x dx ( sin3x cos2x is odd and sin2x cos3x is even)
0

1. 2 4
= 2. 5 . 3 . 1 =
15

 x sin

Illustration 29 : Evaluate
5
x cos 6 x dx
0

 x sin

Sol. Let  =
5
x cos 6 x dx
0

 (  x) sin (  x) cos

=
5 6
(   x ) dx
0

 sin  x sin
 

=
5
. cos x dx –
6 5
x . cos 6 x dx
0 0

 sin
2
 2 =  . 2 x . cos 6 x dx
5

4 . 2 . 5 . 3 .1
 =  11 . 9 . 7 . 5 . 3 . 1

8
=
693

RESONANCE TM 49
x
1

Illustration 30 : Evaluate
3
(1  x )5 dx
0

Sol. Put x = sin2  dx = 2 sin  cos  d


L.L :x=0  =0

U.L. :x=1  =
2

x  sin
1 2

3
(1  x )5 dx = 6
(cos 2 )5 2 . sin  . cos  d
0 0

 sin
2
=2.
7
 cos11  d
0

6 . 4 . 2 . 10 . 8 . 6 . 4 . 2 1
= 2 . 18 . 16 . 14 . 12 . 10 . 8 . 6 . 4 . 2 =
504

Self Practice Problems:

Evaluate the following

 sin
2
18
1.
5
x dx Ans.
15
0

 sin
2
8
2.
5
x cos 4 x dx Ans.
315
0

x
1
6
sin 1 x dx  16
3. Ans. –
0 14 245

   a9
a 7
4. x a2  x 2 2 dx Ans.
9
0

x
2
3/2
2  x dx 
5. Ans.
0 2

RESONANCE TM 50
Part : (A) Only one correct option

 f ( x) dx equals
cos ec
 1
1. If f(x) is a function satisfying f   + x2 f(x) = 0 for all non-zero x, then
x sin 

(A) sin + cosec (B) sin2  (C) cosec2  (D) none of these

x
1
dx 
2. The value of the integral , where 0 <  < , is equal to
0
2
 2x cos   1 2

 
(A) sin  (B)  sin  (C) (D) sin 
2 sin  2

 f ( x) dx = a, then  
1 
 f r  1  x  dx 
100 100

3. If  =
0
r 1 0 
(A) 100 a (B) a (C) 0 (D) 10 a

 f (t) dt is
x
 T T
4. If f(x) is an odd function defined on  ,  and has period T, then (x) =
 2 2 a

T
(A) a periodic function with period (B) a periodic function with period T
2
T
(C) not a periodic function (D) a periodic function with period
4

 (f ( x )  f ( x )) sin x dx = 5 then f(0) is equal to, (it is given that f(x) is continuous in [0, ])

5. If f() = 2 and
0

(A) 7 (B) 3 (C) 5 (D) 1

 x . f  (2x) dx is equal to
1
6. If f(0) = 1, f(2) = 3, f(2) = 5 and f(0) is finite, then
0

(A) zero (B) 1 (C) 2 (D) none of these

lim  sin  . sin 2 . sin 3  ....... sin (n  1) 


1/ n

7. is equal to
n 
 2n 2n 2n n 


(A) (B) e4/ (C) e2/ (D) none of these
2

 f ( x) dx is equal to
 
/3

8. f(x) = Minimum {tanx, cot x}  x   0,  . Then


 2
0

 3  3
   
(A) n  2  (B) n  2  (C) n ( 2 ) (D) n ( 3 )
   
RESONANCE 51
TM

 ( x  2) 

cos x 2
sin 2x
9. If A = dx, then dx is equal to
x 1
2
0 0

1 1 1 1 1 1
(A) + –A (B) –A (C) 1 + –A (D) A – –
2 2 2 2 2 2


/2
| x | dx
10. has the value
 / 2
8 cos 2 2x  1

2 2 2
(A) (B) (C) (D) none of these
6 12 24


n
3n

11. Lt
n 
r  2n  1 r  n 2 is equal to
2

2 3 2 3
(A) log (B) log (C) log (D) log
3 2 3 2

 cos t dt = 
y x2
2 sin t dy
12. If dt , then the value of is
t dx
a a

2 sin2 x 2 sin x 2 2 sin x 2


(A) (B) (C) (D) none of these
x cos2 y x cos y 2  y 2 
x 1  2 sin
 2 

 f (x) dx
 n
0 , where x 
2
, n  1, 2, 3.....
13. If f(x) =  n 1 , then the value of
1 , else where 0

(A) 1 (B) 0 (C) 2 (D) 

 (1  x )
1
x dx
14. 3/4 =
0

15 16 3
(A) (B) – (C) – (D) none
16 5 16

 
2 2
dx dx
15. Let 1 = and 2 = x , then
1 1 x2 1

(A) 1 > 2 (B) 2 > 1 (C) 1 = 2 (D) 1 > 22

 (x  [x]) dx is
[ x]

16. The value of


0

1 1
(A) [x] (B) 2[x] (C) 2 [ x] (D) none of these
2

RESONANCE 52
TM

 
 tan 1 x  tan 1 x  1  dx is equal to
3 2
17. The value of the integeral
 x 1 x 
1 
2

(A)  (B) 2 (C) 4 (D) none of these

 log | tan x  cot x | dx is


/2

18. The value of


0

 
(A)  log 2 (B) – log 2 (C) log 2 (D) – log 2
2 2

e
1
x2
19. If (x – ) dx = 0, then
0

(A) 1 <  < 2 (B)  < 0 (C) 0 <  < 1 (D)  = 0

 f (x)dx  n . The value of  f (x)dx is :


n 1 4
2
20. Suppose for every integer n,
n 2

(A) 16 (B) 14 (C) 19 (D) 21


et d t

et
1 a
21. Let A = dt then dt has the value :
0 1 t a 1 t  a 1
(A) Ae–a (B) – Ae–a (C) – ae–a (D) Aea

25  x 

5 2 3

22. dx equals to :
5/2
x4

 2 
(A) (B) (C) (D) none
3 3 6

t
x
dt
23. The function f(x) = satisfies [IIT - 1996]
0
x
(A) f(x + y) = f(x) + f(y) (B) f  y  = f(x) + f(y) (C) f(xy) = f(x) + f(y) (D) none of these
 

The value of 
cos 2 x 

24. dx , a > 0 is
  1 a
x

(A)  (B) a (C) /2 (D) 2

  1  x
1/ 2
25. The integral   [x]   n    d x equals:
 1  x
 1/ 2 

(A)  1/2 (B) 0 (C) 1 (D) 2 ln (1/2)

RESONANCE 53
TM
t
1

26. If (m, n) =
m
(1  t )n dt, then the expression of (m, n) in terms of (m + 1, n – 1) is
0

[IIT - 2003]

2n n n
(A) –  (m + 1, n – 1) (B)  (m + 1, n – 1)
m1 m 1 m1

2n n n
(C) –  (m + 1, n – 1) (D)  (m +1, n – 1)
m 1 m1 m1

t
1
 1 
(f(t)) dt = (1 – sinx), then f   is

2
27. If [IIT - 2005]
sin x
 3

(A) 1/3 (B) 1/ 3 (C) 3 (D) 3

 {x
0
28.
3
 3x 2  3 x  3  ( x  1) cos( x  1)} dx is equal to [IIT - 2005]
2

(A) – 4 (B) 0 (C) 4 (D) 6

Part : (B) May have more than one options correct

 xf (sin x ) dx is

29. The value of integral
0

  f (sin x ) dx
 /2
(A)  f (sin x ) dx (B)  (C) 0 (D) none of these
0 0

 f (x ) dx is equal to
2
30. If f(x) is integrable over [1, 2], then
1

 
r r
n 2n
1 f  1 f 
(A) nlim (B) nlim
 n
r 1 n  n
r n1 n

  f  n 
r n r
n 2n
1 f  1
(C) nlim (D) nlim
 n
r 1  n   n
r 1

 (cos
x
31. If f(x) = 4
t  sin 4 t ) dt, f (x + ) will be equal to
0

 
(A) f(x) + f() (B) f(x) + 2 f() (C) f(x) + f   (D) f(x) + 2f  
2 2

2x 2  3x  3
 
The value of 
1
32. dx is:
0 (x  1) x 2  2x  2

  1 
(A) + 2 ln2  tan1 2 (B) + 2 ln2  tan1 (C) 2 ln2  cot1 3 (D)  + ln4 + cot1 2
4 4 3 4

RESONANCE 54
TM
33. Given f is an odd function defined everywhere, periodic with period 2 and integrable on every interval. Let

 f(t) dt. Then:


x

g(x) =
0

(A) g(2n) = 0 for every integer n (B) g(x) is an even function


(C) g(x) and f(x) have the same period (D) none


/2
dx
34. If  = , then
0 1  sin3 x


(A) 0 <  < 1 (B)  > (C)  < 2 (D)  > 2
2

If In = 
1
dx
1  x 
35. 2
n ; n  N, then which of the following statements hold good?
0

 1
(A) 2n In + 1 = 2 n + (2n  1) In (B) I2 = 
8 4
 1  5
(C) I2 =  (D) I3 = 
8 4 16 48


 I
2
1. e cos x cos3 (2n + 1) x d x, n
0

2. If f, g, h be continuous functions on [0, a] such that f (a  x) = f (x), g (a  x) =  g (x)


a
and 3 h (x)  4 h (a  x) = 5, then prove that,  f (x) g (x) h (x) = 0.
0


3. Assuming  log sin x d x =  log 2, show that,
0

3  2
 

 3 log sin  d  =
2 
 log 2 sin  d .
0 0

 dx  ln a .  f (  ).

a x ln x 
a x dx
4. Show that f (  ).
0 x a x 0 x a x

 
u 

x x

5. Prove that  f (t ) dt du = f (u).(x  u) du.



0 0  0

Prove that  

dx dx
1  x 
1
6.  (n > 1)
0 1  xn 0
n 1/ n

RESONANCE 55
TM
1  2p  2 p 2 2 p 3 2p   pr
=
p

7. Limit
Prove that n    cos  cos  cos  ......  cos 
n  2n 2n 2n 2 r 1 4r



 
x dx
8.
2
0 a 2 cos 2 x  b 2 sin 2 x

 x  t. cos t dt where ‘x’ is any real number


1

9. Evaluate
0

 2x  1  2 x 
1 3 cos  1  2   tan  

 x  1  1  x2 
10. dx
1 3 ex  1

1
11. Evaluate, I =  2 sin (p t) sin (q t) d t, if:
0
(i) p & q are different roots of the equation, tan x = x.
(ii) p & q are equal and either is root of the equation tan x = x.

 x 1
x
sin x
12. Prove that dx  0 for x  0.
0

 f (x)dx , a  R
a

13. Let f(x) be a continuous functions  x  R, except at x = 0 such that +


exists. If
0

  g( x) dx =  f (x) dx
a a a
f (t)
g(x) = t dt, prove that
x 0 0

  f (x) dx
/2 
sin x   
14. If f(x) =  x  (0, ], prove that, f ( x ) f   x  dx =
x 2 0
2  0

, x > 0. If 
d e sin x 4
2 e sin x
2

15. Let F (x)  dx = F (k)  F (1) then one of the possible values of k is ______.
dx x 1
x


1 3
x4  2x 
16. cos1   dx. [IIT - 1995, 5 + 2 + 2 ]
1 3
1 x 4  1 x2 

e

 1  1 
17. Evaluate |cos x|
 2 sin  cos x   3 cos cos x   sinx dx. [IIT - 2005, 2]
0   2   2 

 (1  x
1
)
50 100
dx
0
18. The value of 5050 is [IIT - 2006, (6, 0)]

 (1  x
1
)
50 101
dx
0

RESONANCE 56
TM
EXERCISE # 1 2
9.  cos x for 0 < x < 1 ;
1. D 2. C 3. B 4. B 5. B 6. C 2

7. B 8. D 9. A 10. B 11. B 12. B 2 2


for x  1 &  for x  0
 2
2
13. C 14. D 15. B 16. A 17. B 18. A

19. C 20. C 21. B 22. A 23. C 24. C


 p2
10. 11. (i) 0 (ii)
25. A 26. A 27. C 28. C 29. AB 30. BC 2 3 1  p2

31. AD 32. AD 33. ABC 34. BC 35. AB

 
 2 
EXERCISE # 2 15. 16 16.
4
n 2  3 
12

3

(
 2 a 2 b 2 ) 24 e cos  1   1 e sin  1   1
    
1. 0 8. 17.  18. 5051
4a b3 3
5  2 2 2 

RESONANCE 57
TM
Area Under Curve

1. Curve Tracing :
To find the approximate shape of a curve, the following procedure is
adopted in order:
(a) Symmetry:
(i) Symmetry about x  axis:
If all the powers of ' y ' in the equation are even then the curve
is symmetrical about the x  axis.
The mathematical
sciences particularly
exhibitorder,
symmetry, and
limitation; and these
E.g.: y2 = 4 a x.
are the greatest forms (ii) Symmetry about y  axis:
of the If all the powers of ' x ' in the equation are even then the curve
is symmetrical about the y  axis.
beautiful.
Ari st ot l e (384-322 BC)

Met aphysi ca

E.g.: x2 = 4 a y.
Our notion of
(iii) Symmetry about both axis;
symmetry is derived If all the powers of ' x ' and ' y ' in the equation are even, the
curve is symmetrical about the axis of ' x ' as well as ' y '.
form the human face.
Hence, we demand
symmetry horizontally
and in breadth only, E.g.: x2 + y2 = a2.
not vertically nor in (iv) Symmetry about the line y = x:
If the equation of the curve remains unchanged on
depth.
interchanging ' x ' and ' y ', then the curve is symmetrical
– Bl ai sePascal about the line y = x.

E.g.: x3 + y3 = 3 a x y.

RESONANCE
TM
1
(v) Symmetry in opposite quadrants:
If the equation of the curve remains unaltered when ' x ' and ' y ' are replaced by  x and  y respectively,
then there is symmetry in opposite quadrants.

E.g.: x y = c2.

(b) Find the points where the curve crosses the xaxis and also the yaxis.

dy
(c) Find dx and equate it to zero to find the points on the curve where you have horizontal tangents.

(d) Examine if possible the intervals when f (x) is increasing or decreasing.

(e) Examine what happens to ‘y’ when x   or x   .

(f) Asymptotes :
Asymptoto(s) is (are) line (s) whose distance from the curve tends to zero as point on curve moves towards
infinity along branch of curve.

(i) If x Lt Lt
 a f(x) =  or x  a f(x) = – , then x = a is asymptote of y = f(x)

(ii) If Lt f(x) = k or Lt f(x) = k, then y = k is asymptote of y = f(x)


x   x  

f (x)
(iii) If x Lt = m1, x Lt
  (f(x) – m 1x) = c, then y = m 1x + c1 is an asymptote. (inclined to right)

x

f (x)
(iv) If Lt = m 2, Lt (f(x) – m2x) = c2, then y = m2x + c2 is an asymptote (inclined to left)
x  x x 

Example : Find asymptote of y = e–x

Solution. Lim y = Lim e–x = 0


x  x 

 y = 0 is asymptote.

Example : Find asymptotes of xy = 1 and draw graph.


1
Solution y=
x

Lim y = Lim 1 =   x = 0 is asymptote.


x 0 x 0 x

Lim y = Lim 1 = 0  y = 0 is asymptote.


x  x  x

RESONANCE
TM
2
1
Example : Find asymptotes of y = x + and sketch the curve.
x

 1
Solution Lim y = Lim  x   = + or –
x 0 x 0  x
 x = 0 is asymptote.

 1
Lim y = Lim  x   = 
x 0 x 0  x
 there is no asymptote of the type y = k.

 1 
Lim y = Lim 1   =1
x  x x   x2 

 1 
Lim (y – x) = Lim  x   x  = Lim 1 = 0
x  x   x  x  x

 y = x + 0  y = x is asymptote.
A rough sketch is as follows

2. Quadrature :

 f (x) dx
b
(a) If f(x)  0 for x  [a, b], then area bounded by curve y = f(x), x-axis, x-axis, x = a and x = b is
a

Example : Find area bounded by the curve y = n x + tan–1x and x-axis between ordinates x = 1 and x = 2.

Solution y = n x + tan–1x
dy 1 1
Domain x > 0 = + >0
dx x 1 x2
It is increasing function
Lt y = Lt (n x + tan–1x) = 
x  x 

Lt Lt  (n x + tan–1x) = 


y = x 0
x 0 

A rought sketch is as follows

 (n x  tan
2
 Required area = 1
x ) dx
1

 2 
2
1
=  x n x  x  x tan x  n (1  x )
1
 2 1

1 1
= 2 n 2 – 2 + 2 tan–12 – n 5 – 0 + 1 – tan–1 1 + n 2
2 2

5 1 
= n 2 – n 5 + 2 tan–12 – –1
2 2 4

RESONANCE
TM
3
 f (x) dx
b

(b) If f(x)  0 for x  [a, b], then area bounded by curve y = f(x), x-axis, x = a and x = b is –
a

Example : Find area bounded by y = log 1 x and x-axis between x = 1 and x = 2.


2

Solution. A rought sketch of y = log 1 x is as follows


2

  log
2 2
Area =– log 1 x dx = – e x . log 1 e dx
1 2 1 2

= – log 1 e . [ x loge x  x ]12


2

= – log 1 e . (2 loge2 – 2 – 0 + 1)
2

= – log 1 e . (2 loge 2 – 1)
2

Note : If y = f(x) does not change sign an [a, b], then area bounded by y = f(x), x-axis between

 f ( x) dx
b

ordinates x = a, x = b is .
a

(c) If f(x) > 0 for x  [a,c] and f(x) < 0 for x  [c,b] (a < c < b) then area bounded by curve y = f(x) and x–axis

 
c b
between x = a and x = b is f ( x ) dx  f ( x ) dx .
a c

Example : Find the area bounded by y = x3 and x–axis between ordinates x = – 1 and x = 1.

 
0 1

Solution Required area =  x 3 dx  x 3 dx


1 0

x4  x3 
0 1

=  4  + 4
 1  0
 1 1 1
= 0 –   + –0=
 4 4 2

 | f ( x) | dx .
b
Note : Area bounded by curve y = f(x) and x–axis between ordinates x = a and x = b is
a

(d) If f(x) > g(x) for x[a,b] then area bounded by curves y = f(x) and y = g(x) between ordinates x = a and

 f ( x )  g( x)dx .
b

x = b is
a

RESONANCE
TM
4
Example : Find the area enclosed by curve y = x2 + x + 1 and its tangent at (1,3) between ordinates x = – 1 and
x = 1.
dy
Solution. = 2x + 1
dx
dy
= 3 at x = 1
dx
Equation of tangent is
y – 3 = 3 (x – 1)
y = 3x

 (x
1

Required area =
2
 x  1  3 x ) dx
1



1 1
x3
= ( x  2x  1) dx  3  x  x 
2 2

1
 1

1   1 
=   1  1 –    1  1
 3   3 
2 8
+2= =
3 3
Note : Area bounded by curves y = f(x) and y = g (x) between ordinates x = a and x = b is

 | f ( x)  g(x ) | dx .
b

(e) If g (y)  0 for y  [c,d] then area bounded by curve x = g(y) and y–axis between abscissa y = c and

 g(y)dy
d
y = d is
y c


Example : Find area bounded between y = sin–1x and y–axis between y = 0 and y = .
2
Solution y = sin–1 x
 x = sin y

 sin y dy
2
Required area =
0



=  cos y 2 = – (0 – 1) = 1
0
Note : The area in above example can also evaluated by integration with respect to x.

Area = (area of rectangle formed by x = 0, y = 0 , x = 1, y = ) – (area bounded by y = sin–1x,
2
x–axis between x = 0 and x = 1)

 sin
 
1
1
= ×1– x dx = – (x sin–1x + 1 x 2 )
1
2 
0

   
= –  0  0  1 = 1
2  2 
RESONANCE
TM
5
Some more solved examples
Example : Find the area contained between the two arms of curves (y – x)2 = x3 between x = 0 and x = 1.

Solution (y – x)2 = x3  y = x ± x 3/2

For arm
dy 3 1/2
y = x + x3/2  =1+ x >0 x > 0.
dx 2
y is increasing function.
For arm
dy 3 1/2
y = x – x3/2  =1– x
dx 2
1
dy 4 d2 y 3  4
=0 x= ,   x 2 < 0 at x =
dx 9 dx 2 4 9

4
 at x = y = x – x3/2 has maxima.
9

 (x  x
1

Required are a =
3/2
 x  x 3 / 2 ) dx
0


2 x5 / 2 
1 1

x 3 / 2 dx  4
=2  =
5 / 2  5
0 0

Example : Find area contained by ellipse 2x2 + 6xy + 5y2 = 1


Solution. 5y2 + 6xy + 2x2 – 1 = 0

 6x  36 x 2  20(2x 2  1)
y=
10

 3x  5  x 2
y=
5
 y is real  R.H.S. is also real.
 – 5 <x< 5
If x=– 5 , y=3 5

If x= 5, y = –3 5

1
If x = 0, y=+
5

1
If y = 0, x=+
2


5   3x  5  x 2  3 x  5  x 2 
  
Required area =  5 5  dx
 5 


5
2 5  x 2 dx
=
5  5


5
4
= 5  x 2 dx
5 0

RESONANCE
TM
6
Put x = 5 sin  : dx = 5 cos  d
L.L : x = 0  = 0

U.L : x = 5   =
2


4 2
= 5  5 sin2  5 cosd
5
0

 cos
1 
2
=4
2
d = 4 =
2 2
0

Example : Let A (m) be area bounded by parabola y = x2 + 2x – 3 and the line y = mx + 1. Find the least area
A(m).
Solution. Solving we obtain
x2 + (2 – m) x – 4 = 0
Let  be roots  = m – 2,  = – 4

 (mx  1 x

A (m) =
2
 2x  3) dx

 (x

=
2
 (m  2) x  4) dx


 x3 x2 
  (m  2 )  4x 
=  3 2 
 

 3  3 m  2 2
=  (   2 )  4 (   )
3 2

1 2 (m  2)
= | – |.  (     2 )  (   )  4
3 2

=
1

(m  2)
(m  2)2  16  3 (m  2)  4  2 (m  2)  4
2

1 8
= (m  2)2  16 (m  2 ) 2 
6 3

 
1
6 (m  2)  16
3/2
A(m) = 2

1 32
Leas A(m) = (16)3/2 = .
6 3

Self Practice Problems

1. Find the area between curve y = x2 – 3x + 2 and x–axis


1
(i) bounded between x = 1 and x = 2. Ans.
6
(ii) bound between x = 0 and x = 2. Ans. 1

RESONANCE
TM
7
2. Find the area included between curves y = 2x – x2 and y + 3 = 0.
32
Ans.
3

3. Find area between curves y = x2 and y = 3x – 2 from x = 0 to x = 2.


Ans. 1

4. Curves y = sinx and y = cosx intersect at infinite number of points forming regions of equal area between
them calculate area of one such region.
Ans. 2 2

5. Find the area of the region bounded by the parabola (y – 2)2 = (x – 1) and the tangent to it at ordinate y = 3
and x–axis.
Ans. 9

6. Find the area included between y = tan–1x, y = cot–1x and y–axis.


Ans. n2

7. Find area common to circle x2 + y2 = 2 and the parabola y2 = x.


 3 2
Ans.  
3 2 3

4  x2
8. Find the area included between curves y = and 5y = 3|x| – 6.
4  x2
8
Ans. 2 –
5

1
9. Find the area bounded by the curve |y| + = e–|x|.
2
Ans. 2 (1–n2)

10. Find the area of loop y2 = x (x – 1)2.


8
Ans.
15

11. Find the area enclosed by |x| + |y| < 3 and xy > 2.
Ans. 3–4n2

12. Find are bounded by x2 + y2 < 2ax and y2 > ax, x > 0.
 3  8 
Ans.   a2.
 6 

RESONANCE
TM
8
1. The area bounded by the curve x 2 = 4y, x-axis and the line x = 2 is
2 3
(A) 1 (B) (C) (D) 2
3 2

2. The area bounded by the x-axis and the curve y = 4x – x 2 – 3 is


1 2 4 8
(A) (B) (C) (D)
3 3 3 3

3. The area bounded by the curve y = sin ax with x-axis in one arc of the curve is
4 2 1
(A) (B) (C) (D) 2a
a a a

4. The area contained between the curve xy = a2, the vertical line x = a, x = 4a (a > 0) and x-axis is
(A) a2 log 2 (B) 2a2 log 2 (C) a log 2 (D) 2a log 2

5. The area of the closed figure bounded by the curves y = x , y = 4  3x & y = 0 is:

4 8
(A) (B) (C) 16 (D) none
9 9 9

2 
6. The area of the closed figure bounded by the curves y = cos x; y = 1  x & x = is
 2

4 3 3  4 3  4
(A) (B) (C) (D)
4 4 4 4

7. The area included between the curve xy2 = a2 (a  x) & its asymptote is:

a2
(A) (B) 2  a2 (C)  a2 (D) none
2

x
8. The area bounded by x² + y²  2 x = 0 & y = sin in the upper half of the circle is:
2

 4  2 8
(A)  (B)  (C)   (D) none
2  4  

9. The area of the region enclosed between the curves 7x 2 + 9y + 9 = 0 and 5 x 2 + 9 y + 27 = 0 is:
(A) 2 (B) 4 (C) 8 (D) 16

10. The area bounded by the curves y = x (1  ln x); x = e 1 and a positive Xaxis between
x = e 1 and x = e is :

 e 2  4 e 2   e 2  5 e 2   4 e 2  e 2   5 e 2  e 2 
(A)   (B)   (C)   (D)  
 5   4   5   4 

 1
11. The area enclosed between the curves y = loge(x + e), x = loge  y  and the x-axis is
 
(A) 2 (B) 1 (C) 4 (D) none of these

RESONANCE 9
TM
12. The area bounded by the curves x  y  1 and x + y = 1 is

1 1 1
(A) (B) (C) (D) none of these
3 6 2

13. The area bounded by x-axis, curve y = f(x), and lines x = 1, x = b is equal to (b 2  1) – 2 for all
b > 1, then f(x) is

(A) ( x  1) (B) ( x  1) (C) ( x 2  1) (D) x / (1  x 2 )

14. The area of the region for which 0 < y < 3 – 2x – x 2 and x > 0 is

    (3  2x  x
3 3 1 3

(A) (3  2x  x 2 ) dx (B) (3  2x  x 2 ) dx (C) (3  2x  x ) dx


2
(D)
2
) dx
1 0 0 1

15. The area bounded by y = x2 , y = [x + 1], x  1 and the y-axis is


(A) 1/3 (B) 2/3 (C) 1 (D) 7/3

16. The area bounded by the curve x = acos 3t, y = a sin3 t is

3a 2 3a 2 3a 2


(A) (B) (C) (D) 3a2
8 16 32

17. If A1 is the area enclosed by the curve xy = 1, x -axis and the ordinates x = 1, x = 2; and A2 is the area
enclosed by the curve xy = 1, x -axis and the ordinates x = 2, x = 4, then
(A) A1 = 2 A2 (B) A2 = 2 A1 (C) A2 = 2 A1 (D) A1 = A2

18. The area bounded by the curve y = f(x), x-axis and the or dinates x = 1 and x = b is
(b – 1) sin (3b + 4),  b  R, then f(x) =
(A) (x – 1) cos (3x + 4) (B) sin (3x + 4)
(C) sin (3x + 4) + 3(x – 1) cos (3x + 4) (D) none of these

19. Find the area of the region bounded by the curves y = x2 + 2, y = x, x = 0 and x = 3.
21
(A) sq. unit (B) 22 sq. unit (C) 21 sq. unit (D) none of these
2

20. The areas of the figure into which curve y2 = 6x divides the circle x 2 + y2 = 16 are in the ratio

2 4  3 4  3
(A) (B) (C) (D) none of these
3 8  3 8  3

21. The triangle formed by the tangent to the curve f(x) = x2 + bx – b at the point (1, 1) and the coordinate axes,
lies in the first quadrant . If its area is 2, then the value of b is [IIT - 2001]
(A) – 1 (B) 3 (C) – 3 (D) 1

1. Find the area of the region bounded by the curve y2 = 2y – x and the y-axis.

2. Find the value of c for which the area of the figure bounded by the curves y = sin 2x, the straight lines
x = /6, x = c & the abscissa axis is equal to 1/2.

3. For what value of 'a' is the area bounded by the curve y = a2x 2 + ax + 1 and the straight line
y = 0, x = 0 & x = 1 the least?

RESONANCE 10
TM
4. Find the area of the region bounded in the first quadrant by the curve C: y = tan x, tangent drawn to


C at x = and the x  axis.
4
5. Find the values of m (m > 0) for which the area bounded by the line y = mx + 2 and
x = 2y  y2 is, (i) 9/2 square units & (ii) minimum. Also find the minimum area.
6. Consider the two curves y = 1/x² & y = 1/[4 (x  1)].
(i) At what value of ‘a’ (a > 2) is the reciprocal of the area of the figure bounded by the curves, the
lines x = 2 & x = a equal to ‘a’ itself?
(ii) At what value of ‘b’ (1 < b < 2) the area of the figure bounded by these curves, the lines
x = b & x = 2 equal to 1  1/b.

7. A normal to the curve, x2 +  x  y + 2 = 0 at the point whose abscissa is 1, is parallel to the line
y = x. Find the area in the first quadrant bounded by the curve, this normal and the axis of ' x '.
8. Find the area between the curve y2 (2 a  x) = x 3 & its asymptotes.

9. Draw a neat & clean graph of the function f (x) = cos 1 (4x 3  3x), x  [1, 1] & find the area enclosed
between the graph of the function & the xaxis as x varies from 0 to 1.

10. Find the area of the loop of the curve, a y2 = x 2 (a  x).

11. Let b  0 and for j = 0, 1, 2,......, n, let Sj be the area of the region bounded by the yaxis and the curve
j ( j  1) 
xeay = sin by, y . Show that S0, S1, S2,....., Sn are in geometric progression. Also, find
b b
their sum for a =  1 and b =  [IIT - 2001, 5]
12. Find the area of the region bounded by the curves, y = x 2, y = 2  x2 & y = 2
which lies to the right of the line x = 1. [IIT - 2002, 5]

 4a 2 4a 1  f ( 1) 3a 2  3a


    2 
4b 2 4b 1  f (1)  3b  3b 
13. If    =  , f(x) is a quadratic function and its maximum value occurs at a
 4c 2 4c 1   f (2)  3c 2  3c 
   
point V. A is a point of intersection of y = f(x) with x-axis and point B is such that chord AB subtends a
right angle at V. Find the area enclosed by f(x) and cheord AB. [IIT - 2005, 6]

EXERCISE # 1 1 1
4. ln 2  5. (i) m = 1, (ii) m = ; Amin = 4/3
1. B 2. C 3. B 4. B 5. B 6. D 7. C 2 4

8. A 9. C 10. B 11. A 12. A 13. D 14. C


7
6. a = 1 + e2, b = 1 + e 2 7. 8. 3  a2
15. B 16. A 17. D 18. C 19. A 20. C 21. C 6

EXERCISE # 2
9. 3  
3  1 sq. units 10.
8 a2
15
  3
1. 4/3 sq. units 2. c =  or 3. a = 
6 3 4
20 125
11.  4 2 sq. units 13. square units.
3 3

RESONANCE 11
TM
Differential Equation

1. Introduction :
An equation involving independent and dependent variables and

There being, then,


the derivatives of the dependent variables is called a
differential equation. There are two kinds of differential
equation:

three figures which 1.1 Ordinary Differential Equation : If the dependent


variables depend on one independent variable x, then
of themselves can fill the differential equation is said to be ordinary.

up space round a dy dz
for example + = y + z,
dx dx
point, viz. the
dy d3 y dy
triangle, the square + xy = sin x , 2 + y = ex ,
dx dx 3 dx

and the hexagon, the


  dy  2    dy  2 
3/2
d2 y dy
bees have wisely k = 1     , y= x + k 1   dx  
dx 2   dx   dx    
selected for their
1.2 Partial differential equation : If the dependent
structure thatwhich variables depend on two or more independent variables,
then it is known as partial differential equation
contains most angles,
z  2z
 = ax,  z   z  0
2 2
suspecting indeed for example y y
x y
2

 x2  y2
that it could hold
2. Order and Degree of a Differential Equation:
more honey than
2.1 Order : Order is the highest differential appearing in a
either of the other differential equation.
2.2 Degree :
two.
It is determined by the degree of the highest order
....... Pappus derivative present in it after the differential equation is
cleared of radicals and fractions so far as the derivatives
are concerned.

n1 n2
 dm y   dm1y   dy 
nk
f 1 (x, y)  m  + f 2 (x, y)  m1  + ........ f k(x, y)   =0
 dx   dx   dx 

The above differential equation has the order m and degree n1.

RESONANCE TM 12
Example :
Find the order & degree of following differential equations.

  dy  
 dy d2 y 
1/ 4
6   
d2 y
    
 dx dx 2 
(i) = y (ii) y= e 
dx 2   dx  

 dy d2 y 
(iii)
 
sin  dx  2  = y (iv) ey – xy + y = 0
 dx 

Solution.

 d2 y 
4
 = y   dy 
6

(i) 
 dx 2   dx 
 
 order = 2, degree = 4

d2 y dy
(ii) + = ny
dx 2 dx
 order = 2, degree = 1

d2 y dy
(iii) + = sin–1 y
dx 2 dx
 order = 2, degree = 1

d3 y d2 y
(iv) –x +y=0
e dx dx 2
3

 equation can not be expressed as a polynomial in differential coefficients, so degree is


not applicable but order is 3.

Self Practice Problems :

1. Find order and degree of the following differential equations.

dy 1
(i) +y= Ans. order = 1, degree = 2
dx dy
dx

 d5 t 
 
 dy d3 y 
= n  5  1
  
(ii)  dx dx 3  Ans. order = 5, degree = not applicable.
e  
 dx 

 dy 1 / 2 
2
d2 y
(iii)    y  = Ans. order = 2, degree = 2
 dx   dx 2

3. Formation of Differential Equation:


Differential equation corresponding to a family of curve will have :
(a) Order exactly same as number of essential arbitrary constants in the equation of curve.
(b) no arbitrary constant present in it.
The differential equation corresponding to a family of curve can be obtained by using the following
steps:
(a) Identify the number of essential arbitrary constants in equation of curve.

RESONANCE TM 13
NOTE : If arbitrary constants appear in addition, subtraction, multiplication or division, then we can
club them to reduce into one new arbitrary constant.
(b) Differentiate the equation of curve till the required order.
(c) Eliminate the arbitrary constant from the equation of curve and additional equation obtained in
step (b) above.
Example :
Form a differential equation of family of straight lines passing through origin.
Sol. Family of straight lines passing through origin is y = mx where’m’ is parameter.
Differentiating w.r.t. x
dy
=m
dx
Eliminating ‘m’ from both equations
dy y
=
dx x
which is the required differential equation.

Example :
Form a differential equation of family of circles touching x-axis at the origin ?
Sol. Equation of family of circles touching x-axis at the origin is
x 2 + y2 + y = 0 ..........(i) where  is parameter
dy dy
2x + 2y + =0 .........(ii)
dx dx
Eliminating ‘’ from (i) and (ii)

dy 2xy
= 2
dx x  y2
which is required differential equation.

Self Practice Problems :

1. Obtain a differential equation of the family of curves y = a sin (bx + c) where a and c being arbitrary
constant.

d2 y
Ans. + b 2y = 0
dx 2
2. Show the differential equation of the system of parabolas y2 = 4a(x – b) is given by

 dy 
2
d2 y
y +   =0
dx 2  dx 
3. Form a differential equation of family of parabolas with focus origin and axis of symmetry along the
x-axis.

 dy 
2
dy
Ans. y = y   + 2xy
2 2
 dx  dx

4. Solution of a Differential Equation:


Finding the dependent variable from the differential equation is called solving or integrating it. The
solution or the integral of a differential equation is, therefore, a relation between dependent and
independent variables (free from derivatives) such that it satisfies the given differential equation
NOTE : The solution of the differential equation is also called its primitive, because the differential
equation can be regarded as a relation derived from it.
There can be three types of solution of a differential equation:
RESONANCE TM 14
(i) General solution (or complete integral or complete primitive) : A relation in x and y satisfying
a given differential equation and involving exactly same number of arbitrary constants as order of
differential equation.
(ii) Particular Solution : A solution obtained by assigning values to one or more than one arbitrary
constant of general solution.
(iii) Singular Solution : It is not obtainable from general solution. Geomatrically, General solution
acts as an envelope to singular solution.

5. Differential Equation of First Order and First Degree :


A differential equation of first order and first degree is of the type
dy
+ f(x, y) = 0, which can also be written as :
dx
Mdx + Ndy = 0, where M and N are functions of x and y.

6. Elementa r y Types of First Order and Fir st Deg ree Dif ferential
Equations :
6.1 Variables separable : If the differential equation can be put in the form, f(x) dx =  (y) dy
we say that variables are separable and solution can be obtained by integrating each side
separately.

A general solution of this will be  f (x ) dx =  (y) dy + c, where c is an arbitrary constant

Example : Solve the differential equation


(1 + x) y dx = (y – 1) x dy
Solution. The equation can be written as -

 1 x   y  1
  dx =   dy
 x   y 

  x  1 dx =  1  y  dy


1   1

n x + x = y – ny + c
ny + nx = y – x + c
xy = cey–x
dy
Example : Solve : = (ex + 1) (1 + y2)
dx
Solution. The equation can be written as-
dy
 ( e x  1)dx
1 y2
Integrating both sides,
tan–1 y = ex + x + c.

dy  2 dy 
Example : Solve : y – x = a y  
dx  dx 
Solution. The equation can be written as -
dy
y – ay2 = (x + a)
dx

dx dy

x  a y  ay 2

RESONANCE TM 15
dx 1

x  a y(1  ay ) dy

dx 1 a 
  
x  a  y 1  ay  dy
Integrating both sides,
n (x + a) = n y – n (1 – ay) + n c

 cy 
n (x + a) = n  1  ay 
 
cy = (x + a) (1 – ay)
where 'c' is an arbitrary constant.

6.1.1 Sometimes transformation to the polar co-ordinates facilitates separation of variables. In this
connection it is convenient to remember the following differentials:
If x = r cos  ; y = r sin  then,
(i) x dx + y dy = r dr (ii) dx 2 + dy2 = dr2 + r2d2 (iii) x dy – y dx = r2d
If x = r sec  & y = r tan  then
(i) x dx – y dy = r dr (ii) x dy – y dx = r 2 sec d.

Example : Solve the differential equation xdx + ydy = x (xdy – ydx)

Solution. Taking x = r cos, y = r sin


x2 + y2 = r2
2x dx + 2ydy = 2rdr
xdx + ydy = rdr .........(i)
y
= tan
x

dy
d y d
dx = sec2.
x 2 dx

xdy – y dx = x2 sec2 . d
xdy – ydx = r2 d ........(ii)
Using (i) & (ii) in the given differential equation then it becomes
r dr = r cos. r2 d
dr
= cos d
r2
1
– = sin + 
r

1 y
– = +
x2  y2 x2  y2

y 1
= c where – = c
x2  y2
(y + 1)2 = c(x2 + y2)

6.1.2 Equations Reducible to the Variables Separable form : If a differential equation can be
reduced into a variables separable form by a proper substitution, then it is said to be
dy
“Reducible to the variables separable type”. Its general form is = f(ax + by + c) a, b  0. To
o
dx
solve this, put ax + by + c = t.

RESONANCE TM 16
dy
Example : Solve = (4x + y + 1)2
dx
Solution. Putting 4x + y + 1 = t
dy dt
4+ =
dx dx
dy dt
= –4
dx dx
Given equation becomes
dt
– 4 = t2
dx
dt
= dx (Variables are separated)
t x
2

Integrating both sides,

4t  dx
dt
2 =

1 t 1  4 x  y  1
 tan–1 =x+c  tan–1   =x+c
2 2 2  2 

 dy 
Example : Solve sin–1   = x + y
 dx 

dy
Solution. = sin (x + y)
dx
putting x + y = t
dy dt
= –1
dx dx

dt dt dt
 – 1 = sin t  = 1 + sin t  1  sin t = dx
dx dx
Integrating both sides,

 1  sin t =  dx
dt

 cos
1  sin t
dt = x + c
2
t

 (sec t  sec t tan t ) dt = x + c


2

tan t – sec t = x + c
1 sin t
– =x+c
cos t

t t
 sin
cos
2 2
– t t =x+c
cos  sin
2 2

 t 
– tan    = x + c
4 2

 xy
tan    +x+c=0
4 2 

RESONANCE TM 17
Self Practice Problems :

1. Solve the differential equation


dy 1
x2 y = (x + 1) (y + 1) Ans. y n (y + 1) = nx – +c
dx x

2. Solve the differential equation


xdx  ydy ydx  xdy y
x2  y2
= Ans. x2  y2 + =c
x 2
x

dy 1 x3
3. Solve : = ex + y + x2ey Ans. – = ex + +c
dx e y
3

dy
4. Solve : xy = 1 + x + y + xy Ans. y = x + n |x (1 + y)| + c
dx

dy
5. Solve = 1 + ex – y Ans. ey – x = x + c
dx

dy xy
6. = sin(x + y) + cos (x + y) Ans. log tan 1 = x + c
dx 2

dy
7. = x tan (y – x) + 1 Ans. sin (y – x) = ex + c
dx

6.2 Homogeneous Differential Equations :

dy f ( x, y )
A differential equation of the from = where f and g are homogeneous function of
dx g ( x, y )
x and y, and of the same degree, is called homogeneous differential equaiton and can be
solved easily by putting y = vx.

  y 2 
y   dy
Example : Solve 2 +   x   1
x     dx
Solution. Putting y = vx
dy dv
=v+x
dx dx

 dv 
2v + (v2 – 1)  v  x  =0
 dx 

dv 2v
v+x =– 2
dx v 1

dv  v(1  v 2 )
x =
dx v2  1

 v(1 v 
v2 1 dx
dv = –
2
) x

RESONANCE TM 18
  1  v
 2v 1
  dv = – n x + c
2 v
n (1 + v2) – n v = – n x + c

1 v 2
.x =c
n v

x2  y2
n y =c

x2 + y2 = yc' where c = ec

Example : Solve : (x2 – y2) dx + 2xydy = 0 given that y = 1 when x = 1

dy x2  y2
Solution. =–
dx 2xy
y = vx
dy dv
=v+
dx dx

dv 1 v 2
 v+x =–
dx 2v

 1 v 
2v dx
dv = –
2
x
n (1 + v2) = – nx + c
at x = 1, y = 1  v=1
n 2 = c

 2  
 1  y  . x 
 n  2  = n2
 x  

x2 + y2 = 2x

6.2.1 Equations Reducible to the Homogeneous form


dy ax  by  c
Equations of the form  .........(1)
dx Ax  By  C
can be made homogeneous (in new variables X and Y) by substituting x = X + h and y = Y + k,
dY aX  bY  (ah  bk  c )
where h and k are constants, we get = . .........(2)
dX AX  BY  ( Ah  Bk  C)
Now, h and k are chosen such that ah + bk + c = 0, and Ah + Bk + C = 0; the differential
equation can now be solved by putting Y = vX.

dy x  2y  5
Example : Solve the differential equation = 2x  xy  4
dx
Solution. Let x = Y + h, y=Y+k
dy dy dY dX
= . .
dx dY dX dx

dY
= 1. . 1.
dX

RESONANCE TM 19
dY
=
dX

dY X  h  2( Y  k )  5
 =
dX 2 X  2h  Y  k  4

X  2Y  (h  2k  5)
=
2 X  Y  (2h  k  4)
h & k are such that h + 2k – 5 = 0 & 2h + k – 4 = 0
h = 1, k = 2
dY X  2Y
 = which is homogeneous differential equation.
dX 2x  Y

Now, substituting Y = vX
dY dv
=v+X
dX dX

dv 1  2v
 X = –v
dX 2v

 1 v X
2v dx
2 dv =

  2( v  1)  2(1  v )  dv = n X + c


 1 3 

1 3
n (v + 1) – n (1 – v) = n X + c
2 2

v 1
= nX2 + 2c
(1  v )3
n

(Y  Y) X2
= e2c
( X  Y )3 X2
X + Y = c(X – Y)3 where e2c = c1
x – 1 + y – 2 = c ( – 1– y + 2)3
x + y – 3 = c(x – y + 1)3
Special case :
(A) In equation (1) if aB = Ab, then the substitution ax + by = v will reduce it to the form in which
variables are separable.

dy 2x  3 y  1
Example : Solve = 4 x  6y  5
dx

Solution. Putting u = 2x + 3y
du dy
=2+3.
dx dx

1  du  u 1
  2 =
3  dx  2u  5

du 3u  3  4u  10
=
dx 2u  5

 7u  13  dx
2u  5
dx =

RESONANCE TM 20
1.du – 7  7u  13 . du = x + c
2 9 1

7
2 9 1
 u– . n (7u – 13) = x + c
7 7 7
9
 4x + 6y – n (14x + 21y – 13) = 7x + 7c
7
9
 – 3x + 6y – n (14x + 21y – 13) = c
7
(B) In equation (1), if b + A = 0, then by a simple cross multiplication equation (1) becomes an
exact differential equation.

dy x  2y  5
Example : Solve = 2x  y  1
dx
Solution. Cross multiplying,
2xdy + y dy – dy = xdx – 2ydx + 5dx
2 (xdy + y dx) + ydy – dy = xdx + 5 dx
2 d(xy) + y dy – dy = xdx + 5dx
On integrating,

y2 x2
2xy + –y= + 5x + c
2 2
 x2 – 4xy – y2 + 10x + 2y = c where c = – 2c

(C) If the homogeneous equation is of the form :


yf(xy) dx + xg(xy)dy = 0, the variables can be separated by the substitution xy = v.

Self Practice Problems :

Solve the following differential equations

 dy 
x  y  tan–1 y = x given that y = 0 at x = 1
y 1 y
1. Ans. x 2  y 2 = e x tan
 dx  x x

dy y y
2. x = y – x tan Ans. x sin =C
dx x x

dy x  2y  3
3. = 2x  y  3 Ans. x + y = c (x – y + 6)3
dx

dy x  y 1
4. = 2x  2 y  3 Ans. 3(2y – x) + log (3x + 7y + 4) = C
dx

dy 3 x  2y  5
5. = 3 y  2x  5 Ans. 3x2 + 4xy – 3y2 – 10x – 10y = C
dx

6.3 Exact Differential Equation :


dy
The differential equation M + N =0 ...........(1)
dx
Where M and N are functions of x and y is said to be exact if it can be derived by direct differentiation
(without any subsequent multiplication, elimination etc.) of an equation of the form f(x, y) = c
dx
e.g. y2 dy + x dx + = 0 is an exact differential equation.
x

RESONANCE TM 21
M N
NOTE : (i) The necessary condition for (1) to be exact is  y   x .

(ii) For finding the solution of Exact differential equation, following exact differentials must be
remembered :

xdy  ydx y


(a) xdy + y dx = d(xy) (b) = d  (c) 2(x dx + y dy) = d (x 2 + y2)
x 2
x

xdy  ydx  y xdy  ydx  1 y  xdy  ydx


(d) = d  ln  (e) = d  tan  (f) = d(ln xy)
xy  x x y
2 2
 x xy

xdy  ydx  1
(g) = d   xy 
x y  
2 2

xdy  ydx
Example : Solve : y dx + x dy =
x2  y2

xdy  ydx
Solution. ydx + xdy =
x2  y2
d (xy) = d (tan–1 y/x)
Integrating both sides -
xy = tan–1 y/x + c

 x2 
Example :
 2
Solve : (2x ny) dx +  y  3y dy  0
 
Solution. The given equation can be written as -

 dy 
ny (2x) dx + x2  y  + 3y2 dy = 0
 
 ny d (x2) + x2 d (ny) + d (y3) = 0
 d (x2 ny) + d (y3) = 0
Now integrating each term, we get
x2ny + y3 = c

Self Practice Problems :

1. Solve : xdy + ydx + xy ey dy = 0 Ans. n (xy ) + ey = c

2. Solve : ye–x/y dx – (xe–x/y + y3) dy = 0 Ans. 2e–x/y + y2 = c

6.4 Linear Differential Equation :

When the dependent variable and its derivative occur in the first degree only and are not multiplied
together, the differential equation is called linear
The m th order linear differential equation is of the form.

dm y dm1y dy
P0(x) + P1(x) m1 + .................... + Pm–1 (x) + Pm (x) y = (x),
dx m
dx dx
where P0(x), P1(x) ..................Pm(x) are called the coefficients of the differential equation.
dy
NOTE : + y2 sinx = lnx is not a Linear differential equation.
dx

RESONANCE TM 22
Linear differential equations of first order :
dy
The differential equation + Py = Q , is linear in y..
dx
where P and Q are functions of x.
Integrating Factor (I.F.) : - It is an expression which when multiplied to a differential equation converts
it into an exact form.
I.F for linear differential equation = e 
Pdx
(constant of integration will not be considered)
 after multiplying above equation by .F it becomes;
dy
. e  Pdx + Py . e  = Q. e 
Pdx Pdx

dx

d
 ( y. e  Pdx ) = Q. e  Pdx
dx

 y. e  Pdx =  Q. e  Pdx
C .

NOTE : Some times differential equation becomes linear if x is taken as the dependent variable and y
as independent variable. The differential equation has then the following form :
dx
dy + P1 x = Q1.
where P1 and Q1 are functions of y.

The .F. now is e 


P1 dy

Example : Solve

dy 3x 2 sin2 x
+ y =
dx 1 x3 1 x3
dy
Solution. + Py = Q
dx
3x 2
P=
1 x3

F = e  P.dx =  1 x 3 dx = e n(1 x
3x2
) = 1 + x3
3

e
 General solution is

y(F) =  Q(IF).dx  c
 1 x
sin2 x
y (1 + x3) = 3 (1 + x3) dx + c


1  cos 2 x
y(1 + x3) = dx + c
2
1 sin 2x
y(1 + x3) = x– +c
2 4

dy
Example : Solve : x nx + y = 2 n x
dx

dy 1 2
Solution. + y=
dx xnx x
1 2
P= ,Q=
xnx x

RESONANCE TM 23
IF = e  P.dx =  xnx dx = e n( nx ) = n x
1
e
 General solution is

 x .nx.dx  c
2
y. (n x) =

y (n x) = (n x)2 + c

Example : Solve the differential equation


t (1 + t2) dx = (x + xt2 – t2) dt and it given that x = –/4 at t = 1
Solution. t (1 + t2) dx = [x (1 + t2) – t2] dt
dx x t
= t 
dt (1  t 2 )

dx x t
 
dt t 1 t2

dx
which is linear in
dt

1 t
Here, P = – , Q=–
t 1 t 2

 t dt = e–nt = 1
1

IF = e
t
 General solution is -

 t .  1  t
1 1 t 
x- =  dt + c
t 2

x
= – tan–1 t + c
t
putting x = – /4, t = 1
– /4 = – /4 + c  c=0
 x = – t tan–1 t

Equations reducible to linear form


6.4.1 By change of variable.

dy
Example : Solve : y sinx = cos x (sinx – y2)
dx
Solution. The given differential equation can be reduced to linear form by change of variable by a suitable
subtitution.
Substituting y2 = z
dy dz
2y =
dx dx
differential equation becomes
sin x dz
+ cos x.z = sin x cos x
2 dx

dz dz
+ 2 cot x . z = 2 cos x which is linear in
dx dx

IF = e   e 2n sin x = sin2 x


2 cot x dx

RESONANCE TM 24
 General solution is -

 2 cos x. sin x. dx  c
2
z. sin2 x =

2
y2 sin2x = sin3 x + c
3

6.4.2 Bernoulli’s equation :


dy
Equations of the form + Py = Q.yn, n  0 and n  1
dx
where P and Q are functions of x, is called Bernoulli’s equation and can be made linear in v by
dividing by yn and putting y –n+1 = v. Now its solution can be obtained as in (v).
dy
e.g. : 2 sin x – y cos x = xy3 ex .
dx

dy y y 2
Example : Solve :   (Bernoulli's equation)
dx x x 2
Solution. Dividing both sides by y2
1 dy 1 1
2 dx
  2 ..... (1)
y xy x

1
Putting y = t

1 dy dt
– 
y 2 dx dx
 differential equation (1) becomes,
dt t 1
–  
dx x x 2

dt t 1 dt
  2 which is linear differential equation in
dx x x dx

IF = e  x = enx = x
1
dx

 General solution is -

 x
1
t. x = 2
. x dx + c

tx = – nx + c
x
y = – nx + c

Self Practice Problems :

dy  x 2  1
 
1. Solve : x (x + 1) = y (1 – x2) + x2 nx Ans.  x  y = x n x – x + c
2
dx  

dy
2. Solve : (x + 2y3) =y Ans. x = y (c + y2)
dx

dy
3. Solve : x + y = y2 log x Ans. y (1 + cx + log x) = 1
dx

RESONANCE TM 25
4. Solve the differential equation
 dy 
xy2  dx  – 2y3 = 2x3 given y = 1 at x = 1 Ans. y3 + 2x3 = 3x6
 

7. Clairaut’s Equation :
The differential equation
dy
y = px + f(p), ..............(10), where p =
dx
is known as Clairout’s Equation.
To solve (10), differentiate it w.r.t. x, which gives
dp
either =0p=c ...........(11)
dx
or x + f(p) = 0 ............(12)
NOTE :
(i) If p is eliminated between (10) and (11), the solution obtained is a general solution of (10)
(ii) If p is eliminated between (10) and (12), then solution obtained does not contain any arbitrary
constant and is not particular solution of (10). This solution is called singular solution of (10).
dy
Example : Solve : y = mx + m – m3 where, m =
dx
Solution. y = mx + m – m3 ..... (i)
The given equation is in clairaut's form.
Now, differentiating wrt. x -
dy dm dm dm
mx   3m 2
dx dx dx dx

dm dm dm
m=m+x   3m 2
dx dx dx

dm
(x + 1 – 3m2) = 0
dx

dm
=0  m=c ..... (ii)
dx

x 1
or x + 1 – 3m2 = 0  m2 = ..... (iii)
3
Eliminating 'm' between (i) & (ii) is called the general solution of the given equation.
y = cx + c – c3 where, 'c' is an arbitrary constant.
Again, eliminating 'm' between (i) & (iii) is called singular solution of the given equation.
y = m (x + 1 – m2)

 x  1  x  1
1/ 2

y=    x  1 
 3   3 

 x  1
1/ 2
2
y=   (x + 1)
 3  3

 x  1
3/2

y=2  
 3 

4
y2 = (x + 1)3
27
27y2 = 4 (x + 1)3
RESONANCE TM 26
Self Practice Problems :
1. Solve the differential equation
dy
Y = mx + 2/m where, m =
dx
Ans. General solution : y = cx + 2/c where c is an arbitrary constant
Singular solution : y2 = 8x
dy
2. Solve : sin px cos y = cos px sin y + p where p =
dx
Ans. General solution : y = cx – sin–1 (c) where c is an arbitrary constant.

x2  1
Singular solution : y= x 2  1  sin 1
x2
8 Orthogonal Trajectory :
An orthogonal trajectory of a given system of curves is defined to be a curve which cuts every member of a
given family of curve at right angle.

Steps to find orthogonal trajectory :


(i) Let f (x, y, c) = 0 be the equation of the given family of curves, where 'c' is an arbitrary constant.
(ii) Differentate the given equation w.r.t. x and then eliminate c.
dy dx
(iii) Replace by – in the equation obtained in (ii).
dx dy
(iv) Solve the differential equation obtained in (iii).
Hence solution obtained in (iv) is the required orthogonal trajectory.

Example : Find the orthogonal trajectory of family of straight lines passing through the origin.
Solution. Family of straight lines passing through the origin is -
y = mx ..... (i)
where 'm' is an arbitrary constant.
Differentiating wrt x
dy
m ..... (ii)
dx
Eliminate 'm' from (i) & (ii)
dy
y= x
dx

dy dx
Replacing by – dy , we get
dx

dx
y =  dy x

x dy + y dy = 0
Integrating each term,

x2 y2
 =c
2 2
 x2 + y2 = 2c
which is the required orthogonal trajectory.

RESONANCE TM 27
Example : Find the orthogonal trajectory of y2 = 4ax (a being the parameter).
Solution. y2 = 4ax ..... (i)
dy
2y = 4a ..... (ii)
dx
Eliminating 'a' from (i) & (ii)
dy
y2 = 2y x
dx

dy dx
Replacing by  dy , we get
dx

 dx 
y = 2   dy  x
 
2 x dx + y dy = 0
Integrating each term,

y2
x2 + =c
2
2x2 + y2 = 2c
which is the required orthogonal trajectories.

Self Practice Problems :


1. Find the orthogonal trajectory of family of circles concentric at (a, 0)
Ans. y = c (x – a) where c is an arbitrary constant.

2. Find the orthogonal trajectory of family of circles touching x – axis at the origin.
Ans. x2 + y2 = cx where c is an arbitrary constant.

3. Find the orthogonal trajectory of the family of rectangular hyperbola xy = c2


Ans. x2 – y2 = k where k is an arbitrary constant.

Geometrical application of differential equation :


Example : Find the curves for which the portion of the tangent included between the co-ordinate axes is
bisected at the point of contact.
Solution. Let P (x, y) be any point on the curve.
Equation of tangent at P (x, y) is -

P(
x,
y)

dy
Y – y = m (X – x) where m = is slope of the tangent at P (x, y).
dx

 mx  y 
Co-ordinates of A  , 0  & B (0, y–mx)
 m 
P is the middle point of A & B
mx  y
 = 2x
m
RESONANCE TM 28
 mx – y = 2mx
 mx = – y
dy
 x=–y
dx

dx dy
 + y =0
x
 nx + ny = nc
 xy = c

Example : Show that (4x + 3y + 1) dx + (3x + 2y + 1) dy = 0 represents a hyperbola having as asymptotes


the lines x + y = 0 and 2x + y + 1 = 0
Solution. (4x + 3y + 1) dx + (3x + 2y + 1) dy = 0
4xdx + 3 (y dx + x dy) + dx + 2y dy + dy = 0
Integrating each term,
2x2 + 3 xy + x + y2 + y + c = 0
2x2 + 3xy + y2 + x + y + c = 0
which is the equation of hyperbola when x2 > ab &   0.
Now, combined equation of its asymptotes is -
2x2 + 3xy + y2 + x + y +  = 0
which is pair of straight lines
 =0
1 1 3 1 1 9
 2.1  + 2 . . . –2. –1. – =0
2 2 2 4 4 4
 =0
 2x2 + 3xy + y2 + x + y = 0
(x + y) (2x + y) + (x + y) = 0
(x + y) (2x + y + 1) = 0
x+y=0 or 2x + y + 1 = 0

Example : The perpendicular from the origin to the tangent at any point on a curve is equal to the abscissa of
the point of contact. Find the equation of the curve satisfying the above condition and which
passes through (1, 1)
Solution. Let P (x, y) be any point on the curve
Equation of tangent at 'P' is -
Y – y = m (X – x)
mX – Y + y – mx = 0
Now,

 y  mx 
 
  =x
 1 m 
2

y2 + m2x2 – 2mxy = x2 (1 + m2)

y 2  x 2 dy
 which is homogeneous equation
2xy dx
Putting y = vx
dy dv
=v+x
dx dx

dv v2  1
 v+x =
dx 2v

dv v 2  1  2v 2
x 
dx 2v

RESONANCE TM 29
v 
2v dx
dv = –
1
2
x
n (v + 1) = –n x + n c
2

 y2 
 
x  2  1 = c
x 
Curve is passing through (1, 1)
 c=2
x2 + y2 – 2x = 0

Example : Find the nature of the curve for which the length of the normal at a point 'P' is equal to the radius
vector of the point 'P'.
Solution. Let the equation of the curve be y = f(x). P(x, y) be any point on the curve.
dy
Slope of the tanget at P(x, y) is =m
dx
 Slope of the normal at P is P(x,y)
1
m = –
m
Equation of the normal at 'P'
1 O
Y–y=– (X – x) G(x+my,0)
m
Co-ordinates of G (x + my, 0)
Now, OP2 = PG2
x2 + y2 = m2y2 + y2
x
m=± y

dy x
=± y
dx
Taking as the sign
dy x
= y
dx
y . dy = x . dx

y2 x2
= +
2 2
x2 – y2 = – 2
x2 – y2 = c (Rectangular hyperbola)
Again taking as –ve sign
dy x
=– y
dx
y dy = – x dx

y2 x2
=– + 
2 2
x2 + y2 = 2
x2 + y2 = c (circle)

RESONANCE TM 30
Only one correct option

1. The degree of differential equation satisfying the relation

1 x 2 + 1 y 2 =  (x 1 y 2 – y 1 x 2 ) is :
(A) 1 (B) 2 (C) 3 (D) none of these

dy  d2 y 
2. If p and q are order and degree of differential equation y + x3  
dx  dx 2  + xy = cos x, then
 
(A) p < q (B) p = q (C) p > q (D) none of these

3. The differential equation for all the straight lines which are at a unit distance from the origin is

 dy   dy   dy   dy 
2 2 2 2

(A)  y  x  = 1 –   (B)  y  x  =1+  


 dx   dx   dx   dx 

 dy   dy   dy   dy 
2 2 2 2

(C)  y  x  = 1 +   (D)  y  x  =1–  


 dx   dx   dx   dx 

4. The differential equation obtained on eliminating A and B from = y = A cos (t) + B sin (t) is
(A) y + y = 0 (B) y – 2y = 0 (C) y = – 2y (D) y + y = 0

5. The differential equation whose solution is (x – h)2 + (y – k)2 = a2 is (a is a constant)

  dy  2    dy  2 
3 3
 d2 y 
2
d2 y
(A) 1     =a 2
(B) 1     = a  2 
2
  dx   dx 2   dx    dx 

  dy   d2 y 
3 2

(C) 1    = a2   (D) none of these


 dx 2 
  dx   

6. The differential equation of all circles which pass through the origin and whose centres lie on y-axis is
dy dy
(A) (x 2 – y2) – 2xy = 0 (B) (x 2 – y2) + 2xy = 0
dx dx

dy dy
(C) (x 2 – y2) – xy = 0 (D) (x 2 – y2) + xy = 0
dx dx

dy
7. If = e–2y and y = 0 when x = 5, the value of x for y = 3 is
dx

e6  9
(A) e5 (B) e6 + 1 (C) (D) loge 6
2

8. If (x) = (x) and (1) = 2, then (3) equals


(A) e2 (B) 2 e2 (C) 3 e2 (D) 2 e3

9. If integrating factor of x(1 – x 2) dy + (2x2 y – y – ax3) dx = 0 is e  p . dx , then P is equal to

2x 2  ax 3 2x 2  1 (2x 2  1)
(A) (B) (2x 2 – 1) (C) (D)
x(1  x 2 ) ax 3 x(1  x 2 )

RESONANCE 31
TM
dy
10. If = 1 + x + y + xy and y (– 1) = 0, then function y is
dx
(A) e(1 x ) /2 (B) e(1 x ) /2
1 (C) loge (1 + x) – 1 (D) 1 + x
2 2

x2  y2
11. Integral curve satisfying y = , y(1) = 2, has the slope at the point (1, 2) of the curve, equal to
x2  y2

5 5
(A) – (B) – 1 (C) 1 (D)
3 3

dv k
12. The solution of + v = – g is
dt m
k k
mg
k k
 t mg mg  m t  t mg t
(A) v = ce m
– (B) v = c – e (C) v e m =c– (D) ve m =c–
k k k k
dy
13. The solution of the differential equation a  x dx + xy = 0 is

(A) y = Ae2/3 (2a – x) xa (B) y = Ae–2/3 (a – x) xa

(C) y = Ae2/3 (2a + x) xa (D) y = Ae–2/3 (2a – x) x  a


Where A is an arbitrary constant.
dy
14. If y1(x) and y2(x) are two solutions of + y(x) y = r(x) then y1(x) + y2(x) is solution of :
dx

dy dy
(A) + f(x) y = 0 (B) + 2f(x) y = r(x)
dx dx

dy dy
(C) + f(x) y = 2 r(x) (D) + 2f (x) y = 2r(x)
dx dx

2
15. The differential equation of all 'Simple Harmonic Motions' of given period is
n

d2 x d2 x d2 x d2 x 1
(A) + nx = 0 (B) + n2 x = 0 (C) – n2 x = 0 (D) + x = 0.
dt 2
dt 2
dt 2
dt 2
n2

16. If ( x 2  y 2 ) = ae tan , a > 0. Then y(0), equals


1
(y / x)

a /2 2 –/2 a –/2


(A) e (B) ae/2 (C) – e (D) e
2 a 2

 1  cos  cos x

d dx
17. The function f() = satisfies the differential equation
d
0

df df
(A) + 2f() cot  = 0 (B) – 2f() cot  = 0
d d

df df
(C) + 2f() = 0 (D) – 2f () = 0
d d

18. The solution of the differential equation y1 y3 = 3y22 is


(A) x = A1y2 + A2 y + A3 (B) x = A1 y + A2
(C) x = A1 y2 + A2 y (D) none of these

RESONANCE 32
TM
19. The solution of y dx – x dy + 3x 2 y2 e x dx = 0 is
3

x x x
(A) y + e x = C (B) y – e x = 0 (C) – y + e x = C (D) none of these
3 3 3

20. The solution of the differential equation


(x 2 sin3 y – y2 cos x) dx + (x 3 cos y sin2 y – 2y sin x) dy = 0 is
(A) x sin3 y = 3y2 sin x + C
3
(B) x3 sin3 y + 3y2 sin x = C
(C) x 2 sin3 y + y3 sin x = C (D) 2x2 sin y + y2 sin x = C

One or more than one options correct

21. The differential equation of the curve for which the initial ordinate of any tangent is equal to the corre-
sponding subnormal
(A) is linear (B) is homogeneous
(C) has separable variables (D) is none of these

22. The solution of x 2 y12 + xy y1 – 6y2 = 0 are


1
(A) y = Cx 2 (B) x 2 y = C (C) log y = C+ log x (D) x3 y = C
2

 dy 
2

23. The orthogonal trajectories of the system of curves   = a/x are


 dx 

2 2
(A) 9 a(y + c) = 4x 3 (B) y + C = x3/2 (C) y + C = x 3/2 (D) none of these
3 a 3 a

 dy 
24. The solution of   (x 2 y3 + xy ) = 1 is
 dx 

(A) 1/x = 2 – y2 + C e  y / 2
2

(B) the solution of an equation which is reducible to linear equation.


(C) 2/x = 1 – y2 + e–y /2
1 2x
(D) = – y2 + Ce  y /2
2

x dx  y dy 1 x2  y2
1. Solve : x dy  y dx 
x2  y2

2. Solve :
(a) x2 dy + y(x + y) dx = 0, given that y = 1, when x = 1

 y  y   y  y  dy
(b)  x cos   y sin  y –  y sin   x cos  x =0
  
x  x    
x  
x dx

dy y 2  2xy  x 2
3. Find the equation of the curve satisfying dx  2 and passing through (1, –1).
y  2xy  x 2

d3 y d2 y 1
4. Find the solution of the differential equation =8 satisfying y(0) = , y (0) = 0 and y2(0) = 1.
dx 3
dx 2 8 1

RESONANCE 33
TM
5. Solve :
dy
(i) (x + 3y2) = y, y > 0 (ii) (1 + y + x2y) dx + (x + x3)dy = 0
dx
dy dy
(iii) = y tanx – 2sinx (iv) (1 + x2) + 2xy = cosx
dx dx

6. Solve :
dy
(i) y(x2y + ex) dx = ex dy (ii) x + y = x2y4
dx
(iii) 2y sinx dy + (y2 cosx + 2x) dx = 0

7. Solve the following differential equations.

dy 2y x 3
3 + =
dx x 1 y2

8. Find the curve y = f(x) where f(x)  0, f(0) = 0, bounding a curvilinear trapezoid with the base [0, x] whose area
is propostinal to (n + 1)th power of f(x). It is known that f(1) = 1
9. Find the nature of the curve for which the length of the normal at the point P is equal to the radius vector of the
point P.
10. A particle, P, starts from origin and moves along positive direction of y-axis. Another particle, Q, follows P i.e.
it’s velocity is always directed towards P, in such a way that the distance between P and Q remains
constant. If Q starts from (2, 0), find the equation of the path traced by Q. Assume that they start moving at
the same instant.

dy x 2  y 2
11. Let c1 and c2 be two integral curves of the differential equation dx  2 . A line passing through origin
x  y2
meets c1 at P(x1, y1) and c2 at Q(x2, y2). If c1 : y = f(x) and c2 : y = g(x) prove that f ’(x1) = g’(x2).

dy
12. Find the integral curve of the differential equation x(1 – xy) + y = 0 which passes through (1, 1/e).
dx

dy
13. Show that the integral curves of the equation (1 – x2) + xy = ax are ellipses and hyperbolas, with the
dx
centres at the point (0, a) and the axes parallel to the co-ordinate axes, each curve having one constant axis
whose length is equal to 2.
dy
14. If y1 & y2 be solutions of the differential equation + Py = Q, where P & Q are functions of x alone,
dx
and y2 = y1 z, then prove that

  yQ dx
z=1+ ae 1 ,'a' being an arbitrary constant.

15. Find the curve for which the sum of the lengths of the tangent and subtangent at any of its point is
proportional to the product of the coordinates of the point of tangency, the proportionality factor is
equal to k.
16. Find all the curves possessing the following property; the segment of the tangent between the point of
tangency & the xaxis is bisected at the point of intersection with the yaxis .
17. A curve passing through (1 , 0) such that the ratio of the square of the intercept cut by any tangent off
the yaxis to the subnormal is equal to the ratio of the product of the coordinates of the point of
tangency to the product of square of the slope of the tangent and the subtangent at the same point.
Determine all such possible curves.

RESONANCE 34
TM
18. A & B are two separate reservoirs of water. Capacity of reservoir A is double the capacity of reservoir B.
Both the reservoirs are filled completely with water , their inlets are closed and then the water is
released simultaneously from both the reservoirs. The rate of flow of water out of each reservoir at any
instant of time is proportional to the quantity of water in the reservoir at that time. One hour after the
water is released , the quantity of water in reservoir A is 1.5 times the quantity of water in reservoir B.
After how many hours do both the reservoirs have the same quantity of water ?

19. A curve y = f(x) passes through the point P (1 ,1). The norm al to the curve at P is ;
a (y  1) + (x  1) = 0. If the slope of the tangent at any point on the curve is proportional to the ordinate
of the point, determine the equation of the curve. Also obtain the area bounded by the yaxis, the curve
& the normal to the curve at P. [IIT - 1996, 5 ]

du dv
20. Let u (x) & v (x) satisfy the differential equations + p (x) u = f (x) & + p (x) v = g (x) where p (x) ,
dx dx
f (x) & g (x) are continuous functions. If u (x1) > v (x1) for some x 1 and f (x) > g (x) for all x > x 1 , prove that
any point (x , y) where x > x1 does not satisfy the equations y = u (x) & y = v (x).

21. A curve passing through the point (1, 1) has the property that the perpendicular distance of the origin
from the normal at any point P of the curve is equal to the distance of P from the x  axis. Determine the
equation of the curve. [IIT - 1999, 10 ]

22. A country has a food deficit of 10 %. Its population grows continuously at a rate of 3 % per year. Its
annual food production every year is 4 % more than that of the last year. Assuming that the average
food requirement per person remains constant, prove that the country will become selfsufficient in
 n 10   n 9
food after ' n ' years , where ' n ' is the smallest integer bigger than or equal to, .
. )  0.03
 n (104
[IIT - 2000 (Mains) 10 ]
23. An inverted cone of height H and radius R is pointed at bottom. It is filled with a volatile liquid completely.
If the rate of evaporation is directly proportional to the surface area of the liquid in contact with air
(constant of proportionality k > 0) , find the time in which whole liquid evaporates.
[IIT - 2003 (Mains) 4]

Exercise # 1 6. (i)
1
e x
= –
x3
+c (ii)
1
= 3x2 + cx3
y 3 y3
1. A 2. C 3. C 4. C (iii) y2 sinx = –x2 + c
5. B 6. A 7. C 8. B
x6 2 5 1 4
9. D 10. B 11. A 12. A 7. y3 (x + 1) 2 = + x + x +c
6 5 4
13. A 14. C 15. B 16. C 8. y = x1/n 9. Rectangular hyperbola or circle.
17. A 18. A 19. A 20. A 10. y = 2n x – 2n (2 – 4  x2 ) – 4  x2

 
21. AB 22. ACD 23. ABC 24. ABD
1
Exercise # 2 12. x(ey + ny + 1) = 1 15. y =  n c k 2x 2  1
k
c( x  y )
x2  y2  1 x2  y2  ; x =e
 2 y/x
17. x = e
2 y/x
1. 16. y² = cx
x y
2 2
18. T = log4/3 2 hrs from the start
y
(b) xy cos   = c
a  1  e  a 
2. (a) 3x2y = 2x + y 1
x
3. x + y = 0 4. 64y = (e8x – 8x) + 7
19. ea(x1)
a  2  , sq. unit
x 21. (c) x2 + y2  2x = 0
5. (i) y = 3y + c (ii) xy = c – arc tanx
(iii) y = cosx + c secx (iv) y (1 + x2) = c + sinx. 23. t = H/k

RESONANCE 35
TM
Sequence & Progression

Sequence :
A sequence is a function whose domain is the set N of natural
numbers. Since the domain for every sequence is the set N of
natural numbers, therefore a sequence is represented by its
The divergent range. If f : N  R, then f(n) = tn n  N is called a sequence and
is denoted by
series are the invention of {f(1), f(2), f(3),...............} =
{t1, t2, t3, ......................} = {tn}
thedevil, andit is ashame
to base on them any Real Sequence :
demonstration whatsoever.
A sequence whose range is a subset of R is called a real
By using them, one may sequence.

draw any conclusion he Examples :


pleases and that is why (i) 2, 5, 8, 11, .......................
(ii) 4, 1, – 2, – 5, ......................
these series have produced (iii) 3, –9, 27, – 81, ........................
so many fallacies and so
Types of Sequence
many paradoxes ..
On the basis of the number of terms there are two types of
Ni el s Abel sequence.
(i) Finite sequences : A sequence is said to be finite if it has finite
number of terms.
(ii) Infinite sequences : A sequenceis said to be infinite if it has
If you disregard infinite number of terms.

the very simplest cases, Solved Example # 1


Write down the sequence whose nth term is
there is in all of
2n 3  ( 1)n
mathematics not a single (i) (ii)
n 3n
infinite series whose sum Solution.
has been rigorously 2n
(i) Let tn =
determined. In other n
put n = 1, 2, 3, 4, .............. we get
words,the most important
8
parts of mathematics stand t1 = 2, t2 = 2, t3 = ,t =4
3 4
without a foundation. so the sequence is
8
Ni el s Abel 2, 2, , 4, ........
3

RESONANCE 1
TM
3  ( 1)n
(ii) Let tn =
3n
put n = 1, 2, 3, 4, ......
so the sequence is
2 4 2 4
, , , ,............
3 9 27 81

S eries
By adding or substracting the terms of a sequence, we get an expression which is called a series. If
a1, a2, a3,........an is a sequence, then the expression a1 + a2 + a3 + ...... + an is a series.
Example. (i) 1 + 2 + 3 + 4 + .................... + n
(ii) 2 + 4 + 8 + 16 + .................

Progression
It is not necessary that the terms of a sequence always follow a certain pattern or they are described
by some explicit formula for the nth term. Those sequences whose terms follow certain patterns are
called progressions.

An arithmetic progression (A .P.)

A.P. is a sequence whose terms increase or decrease by a fixed number. This fixed number is called
the common difference. If a is the first term & d the common difference, then A.P. can be written as a,
a + d, a + 2 d,....... a + (n  1) d,........

Example – 4, – 1, 2, 5 ...........

(i) n th term of an A.P.


Let a be the first term and d be the common difference of an A.P., then
tn = a + (n – 1) d where d = an – an – 1

Solved Example # 2

If t54 of an A.P. is – 61 and t4 = 64, find t10.

Solution.
Let a be the first term and d be the common difference
so t54 = a + 53d = – 61 .........(i)
and t4 = a + 3d = 64 .........(ii)
equation (i) – (ii)
 50d = – 125
5
d=–
2
143
 a=
2

143  5
so t10 = + 9    = 49
2  2

RESONANCE 2
TM
Solved Example # 3

Find the number of terms in the sequence 4, 12, 20, ........108.

Solution.
a = 4, d = 8 so 108 = 4 + (n – 1)8
 n = 14

(ii) The sum of first n terms of are A.P.


If a is first term and d is common difference then
n
Sn = [2a + (n – 1) d]
2
n
= [a + ] = nt  n 1  ,
2  
 2 

where  is the last term and t  n 1  is the middle term.


 
 2 

(iii) rth term of an A.P. when sum of first r terms is given is tr = sr – Sr – 1.

Solved Example # 4

Find the sum of all natural numbers divisible by 5, but less than 100.

Solution.
All those numbers are 5, 10, 15, 20, ........... 95.
Here a = 5 n = 19  = 95
19
so S= (5 + 95) = 950.
2

Solved Example # 5

Find the sum of all the three digit natural numbers which on division by 7 leaves remainder 3.

Solution.
All these numbers are 101, 108, 115, ........ 997, to find n.
997 = 101 + (n – 1) 7
 n = 129
129
so S= [101 + 997] = 70821.
2
Solved Example # 6
7n  1
The sum of n terms of two A.Ps. are in ratio . Find the ratio of their 11
1th terms.
4n  27

Sol. Let a1 and a2 be the first terms and d1 and d2 be the common differences of two A.P.s respectively then

n
[2a1  (n  1)d1 ]
2 7n  1
n =
[2a1  (n  1)d2 ] 4n  27
2

 n  1
a1    d1
 2  7n  1
 =
 n  1 4n  27
a2    d2
 2 

RESONANCE 3
TM
For ratio of 11th terms
n 1
= 10  n = 21
2
7(21)  1
so ratio of 11th terms is
4(21)  27

148
=
111
Solved Example # 7

If sum of n terms of a sequence is given by Sn = 2n2 + 3n, find its 50th term.

Solution.
Let tn is nth term of the sequence so tn = s n – sn – 1.
= 2n2 + 3n – 2(n – 1)2 – 3(n – 1)
= 4n + 1
so t50 = 201.

Self Practice Problems :

1. Which term of the sequence 2005, 2000, 1995, 1990, 1985, ............. contains the first negative term
Ans. 403.

2. For an A.P. show that


tm + t2n + m = 2 tm + n

3. Find the maximum sum of the A.P. 40, 38, 36, 34, 32, ..............
Ans. 420

Properties of A.P.

(i) The common difference can be zero, positive or negative.


(ii) If a, b, c are in A.P.  2 b = a + c & if a, b, c, d are in A.P.  a + d = b + c.
(iii) Three numbers in A.P. can be taken as a  d, a, a + d; four numbers in A.P. can be taken as
a  3d, a  d, a + d, a + 3d; five numbers in A.P. are a  2d, a  d, a, a + d, a + 2d & six terms in
A.P. are a  5d, a  3d, a  d, a + d, a + 3d, a + 5d etc.
(iv) The sum of the terms of an A.P. equidistant from the beginning & end is constant and equal to
the sum of first & last terms.
(v) Any term of an A.P. (except the first) is equal to half the sum of terms which are equidistant
from it. an = 1/2 (ank + an+k), k < n. For k = 1, an = (1/2) (an1+ an+1);
For k = 2, an = (1/2) (an2+ an+2) and so on.
(vi) If each term of an A.P. is increased, decreased, multiplied or divided by the sA.M.e non zero
number, then the resulting sequence is also an A.P..

Solved Example # 8
The sum of three numbers in A.P. is 27 and the sum of their squares is 293, find them
Solution.
Let the numbers be
a – d, a, a + d
so 3a = 27  a=9
Also (a – d) 2 + a2 + (a + d) 2 = 293.
3a2 + 2d2 = 293
d2 = 25  d=±5
therefore numbers are 4, 9, 14.

RESONANCE 4
TM
Solved Example # 9

a
5
If a1, a2, a3, a4, a5 are in A.P. with common difference  0, then find the value of i when a3 = 2.
i1
Solution.
As a1, a2, a3, a4, a5, are in A.P., we have
a1 + a5 = a2 + a4 = 2a3.

a
5

Hence i = 10.
i1

Solved Example # 10
1 1 1
If , , are in A.P. prove that a2, b2, c 2 are also in A.P..
bc c a ab
Solution.
1 1 1
, , are in A.P..
bc c a ab

1 1 1 1
 – = –
ca bc ab ca

bc c a c  aa b
 =
(c  a)(b  c ) (a  b)(c  a)

ba c b
 =
bc ab
 b2 – a2 = c 2 – b2
 a2, b2, c 2 are in A.P.

Solved Example # 11
bc a c ab abc 1 1 1
If , , are in A.P., then , , are also in A.P.
a b c a b c
Solution.
bc a c ab abc
Given , , are in A.P..
a b c
Add 2 to each term
bc a c ab abc
 , , are in A.P..
a b c
1 1 1
divide each by a + b + c  , , are in A.P..
a b c

Arithmetic Mean (Mean or Average) (A.M.):


If three terms are in A.P. then the middle term is called the A.M. between the other two, so if a, b, c are
in A.P., b is A.M. of a & c.
a 1  a 2  a 3  .....  a n
A.M. for any n number a1, a2,..., an is; A = .
n
(a) n  Arithmetic Means Between Two Numbers:
If a, b are any two given numbers & a, A1, A2,...., An, b are in A.P. then A1, A2,... An are the
n A.M.’s between a & b.
ba 2 (b  a ) n (b  a )
A1 = a + , A2 = a + ,......, An = a +
n1 n1 n 1

RESONANCE 5
TM
NOTE :
Sum of n A.M.’s inserted between a & b is equal to n times the single A.M. between a & b


n
i.e. Ar = nA where A is the single A.M. between a & b.
r 1

Solved Example # 12
13
Between two numbers whose sum is , an even number of A.M.s is inserted, the sum of these
6
means exceeds their number by unity. Find the number of means.
Solution.
Let a and b be two numbers and 2n A.M.s are inserted between a and b then
2n
(a + b) = 2n + 1.
2
 13   13 
n   = 2n + 1. given a  b  6 
 6   
 n = 6.
 Number of means = 12.

Solved Example # 13
Insert 20 A.M. between 2 and 86.
Solution.
Here 2 is the first term and 86 is the 22nd term of A.P. so 86 = 2 + (21)d
 d=4
so the series is
2, 6, 10, 14,......., 82, 86
 required means are 6, 10, 14,...82.

Self Practice Problems :


4. If A.M. between pth and qth terms of an A.P. be equal to the A.M. between r th and s th term of the A.P. then
prove that p + q = r + s.
5. If n A.M.s are inserted between 20 and 80 such that first means :
last mean = 1 : 3, find n.
Ans. n = 11
a n1  b n 1
6. For what value of n, , a  b is the A.M. of a and b.
an  bn
Ans. n=0

Geometric Progression (G.P.)


G.P. is a sequence of numbers whose first term is non zero & each of the succeeding terms is equal to
the proceeding terms multiplied by a constant. Thus in a G.P. the ratio of successive terms is constant.
This constant factor is called the common ratio of the series & is obtained by dividing any term by
that which immediately proceeds it. Therefore a, ar, ar 2, ar3, ar4,...... is a G.P. with a as the first term
& r as common ratio.
Example 2, 4, 8, 16 .......
1 1 1 1
Example , , , .......
3 9 27 81
(i) nth term = a rn1

a rn  1
   , r 1
(ii) Sum of the first n terms i.e. Sn =  r  1
 na , r 1
(iii) Sum of an infinite G.P. when r < 1. When n  r n  0 if r < 1 therefore,

S =
a
1 r
r 1 .
RESONANCE 6
TM
Solved Example # 14
If the first term of G.P. is 7, its nth term is 448 and sum of first n terms is 889, then find the fifth term of
G.P.
Solution.
Given a = 7 the first term
tn = ar n – 1 = 7(r)n – 1= 448.
 7rn = 448 r

a(r n  1) 7(r n  1)
Also Sn = =
r 1 r 1
448r  7
 889 =
r 1
 r=2
Hence T 5 = ar4 = 7(2)4 = 112.

Solved Example # 15

The first term of an infinite G.P. is 1 and any term is equal to the sum of all the succeeding terms. Find
the series.

Solution.
Let the G.P. be 1, r, r2, r 3, .........

r2
given condition  r =
1 r
1
 r= ,
2

1 1 1
Hence series is 1, , , , ..............
2 4 8

Solved Example # 16
1 1 1
Let S = 1 + + + + .......... find the sum of
2 4 8
(i) first 20 terms of the series
(ii) infinite terms of the series.
Solution.

  1  20 
1    
 2 
  2 20  1 1
(i) S20 = 1 = . (ii) S = 1 = 2.
1 2 19
1
2 2

Self Practice Problems :

1. Find the G.P. if the common ratio of G.P. is 3, nth term is 486 and sum of first n terms is 728.
Ans. 2, 6, 18, 54, 162, 486.

2. If the pth, qth, rth terms of a G.P. be a, b, c respectively, prove that aq – r br – p cp – q = 1.

3. A G.P. consist of 2n terms. If the sum of the terms occupying the odd places is S1 and that of the terms
occupying the even places is S2 then find the common ratio of the progression.
S2
Ans. .
S1

RESONANCE 7
TM
4. The sum of infinite number of terms of a G.P. is 4, and the sum of their cubes is 192, find the series.
3
Ans. 6, – 3, ,........
2
Proper ties of G.P.

(i) If a, b, c are in G.P.  b2 = ac, in general if a1, a2, a3, a4,......... an – 1 , an are in G.P.,
then a1an = a1an – 1 = a3 an – 2 = ..........................
a
(ii) Any three consecutive terms of a G.P. can be taken as , a , ar, in general we take
r
a a a
, k 1 , k 2 ,.........a, ar, ar2,.........ar k in case we have to take 2k + 1 terms in a G.P.
.P.
r k
r r
a a
(iii) Any four consecutive terms of a G.P. can be taken as , , ar, ar3, in general we take
r 3
r

a a a
, , ......... , ar, .............ar2k – 1 in case we have to take 2k terms in a G.P.
.P.
r 2k 1
r 2k 3
r

(iv) If each term of a G.P. be multiplied or divided or raised to power by the some nonzero quantity, the
resulting sequence is also a G.P..

(v) If a1, a2, a3,........ and b1, b2, b3,......... are two G.P’s with common ratio r 1 and r2 respectively then the
sequence a1b1, a2b2, a3b3, ..... is also a G.P. with common ratio r1 r2.

(vi) If a1, a2, a3,..........are in G.P. where each ai > 0, then log a1, loga2, loga3,..........are in A.P. and its
converse is also true.

Solved Example # 17

Find three numbers in G.P. having sum 19 and product 216.


Solution.
a
Let the three numbers be , a, ar
r
1 
so a   1  r  = 19 .......(i)
r 
and a3 = 216  a=6
so from (i) 6r2 – 13r + 6 = 0.
3 2
 r= ,
2 3
Hence the three numbers are 4, 6, 9.

Solved Example # 18

Find the product of 11 terms in G.P. whose 6th is 5.

Solution.
Using the property
a1a11 = a2a10 = a3a9 = .............. = a62 = 25
Hence product of terms = 511

Solved Example # 19
p
Using G.P. express 0. 3 and 1.2 3 as form.
q
Solution.
Let x = 0. 3 = 0.3333 .............
= 0.3 + 0.03 + 0.003 + 0.0003 + .............

RESONANCE 8
TM
3 3 3 3
= + + + + ..............
10 100 1000 10000

3
10 3 1
= 1 = 9 = 3.
1
10
Let y = 1.2 3
= 1.233333
= 1.2 + 0.03 + 0.003 + 0.0003 + .............
3 3 3
= 1.2 + + + + ............
10 2
10 3
10 4

3
10 2 1 37
= 1.2 + = 1.2 + = .
1 30 30
1
10
Solved Example # 20

Evaluate 7 + 77 + 777 + ........... upto n terms.


Solution.
Let S = 7 + 77 + 777 + ..........upto n terms.
7
= [9 + 99 + 999 + .......]
9

7
= [(10 – 1) + (102 – 1) + (103 – 1) + ........ + upto n terms]
9

7
= [10 + 102 + 103 + ...........+ 10n – n]
9

7 10 (10 )  1  n
n

=  
9  9 

7
= [10n + 1 – 9n – 10]
81

Geometric Means (Mean Proportional) (G.M.):


If a, b, c are in G.P., b is the G.M. between a & c.
b² = ac, therefore b = a c ; a > 0, c > 0.

(a) nGeometric Means Between a, b:


If a, b are two given numbers & a, G1, G 2,....., Gn, b are in G.P.. Then
G 1, G 2, G 3,...., G n are n G.M.s between a & b.

G 1 = a(b/a)1/n+1, G 2 = a(b/a) 2/n+1,......, G n = a(b/a) n/n+1


NOTE :
The product of n G.M.s between a & b is equal to the nth power of the single G.M. between a & b
n
i.e.  Gr = (G)n where G is the single G.M. between a & b.
r 1

RESONANCE 9
TM
Solved Example # 21
Insert 4 G.M.s between 2 and 486.
Solution.
1
 b  n1
Common ratio of the series is given by r =   = (243)1/5 = 3
a
Hence four G.M.s are
6, 18, 54, 162.
Self Practice Problems :

1. The sum of three numbers in G.P. in 70, if the two extremes be multiplied each by 4 and the mean by
5, the products are in A.P. Find the numbers.
Ans. 10, 20, 40

111 .......... 1
2. If a =  , b = 1 + 10 + 102 + 103 + 104 and c = 1 + 105 + 1010 + ..... + 1050, then prove that
55
(i) ‘a’ is a composite number
(ii) a = bc.
Harmonic Progression (H.P.)
A sequence is said to H.P. if the reciprocals of its terms are in A.P.. If the sequence a1, a2, a3,...., an is
an H.P. then 1/a1, 1/a2,...., 1/an is an A.P. & converse. Here we do not have the formula for the sum
of the n terms of a H.P.. For H.P. whose first term is a and second term is b, the n th term is
ab 2ac a ab
tn = . If a, b, c are in H.P.  b = or = .
b  (n  1)(a  b) ac c bc
NOTE :
ab a ab a
(i) If a, b, c are in A.P.  = (ii) If a, b, c are in G.P.  =
bc a bc b

Harmonic Mean (H.M.):


If a, b, c are in H.P., b is the H.M. between a & c, then b = 2ac/[a + c].
If a1, a2 , ........ an are ‘n’ non-zero numbers then H.M. H of these numbers is given by

1 1  1  1  .......  1 
=  
H n  a1 a 2 an 

Solved Example # 22
If m th term of H.P. is n, while nth term is m, find its (m + n) th term.
Solution.
1
Given T m = n or a  (m  1) d = n; where a is the first term and d is the common difference of the
corresponding A.P.
1
so a + (m – 1)d =
n
1 mn
and a + (n – 1) d =  (m – n)d =
m mn
1
or d=
mn
1 (m  1) 1
so a= – =
n mn mn

1 mn mn
Hence T (m + n) = a  (m  n  d) d = = .
1 m  n  1 mn

RESONANCE 10
TM
Solved Example # 23
2 2
Insert 4 H.M between and .
3 13
Solution.
Let d be the common difference of corresponding A.P..
13 3

so d = 2 2 = 1.
5

1 3 5 2
 H1 = 2 + 1 = 2 or H1 =
5

1 3 7 2
H2 = 2 + 2 = 2 or H2 =
7

1 3 9 2
H3 = 2 + 3 = 2 or H3 =
9

1 3 11 2
H4 = 2 + 4 = 2 or H4 =
11
.

Solved Example # 24

If pth, qth, rth terms of a H.P. be a, b, c respectively, prove that


(q – r)bc + (r – p) ac + (p – q) ab = 0
Solution.
Let x be the first term and d be the common difference of the corresponding A.P..
1
so = x + (p – 1)d ...........(i)
a

1
= x + (q – 1) d ..........(ii)
b

1
= x + (r – 1) d ..........(iii)
c
(i) - (ii)  ab(p – q)d = b – a ..........(iv)
(ii) - (iii)  bc (q – r)d = c – b ..........(v)
(iii) - (i)  ac (r – p) d = a – c ..........(vi)
(iv) + (v) + (vi) gives
bc (q – r) + ac(r – p) + ab (p – q) = 0.

Self Practice Problems :

1. If a, b, c be in H.P., show that a : a – b = a + c : a – c.

2. If the H.M. between two quantities is to their G.M.s as 12 to 13, prove that the quantities are in the ratio
4 to 9.
H H
3. If H be the harmonic mean of a and b then find the value of + – 1.
2a 2b
Ans. 0

4. If a, b, c, d are in H.P., the show that ab + bc + cd = 3ad

RESONANCE 11
TM
Relation between means :
(i) If A, G, H are respectively A.M., G.M., H.M. between a & b both being unequal & positive then,
G² = AH i.e. A, G, H are in G.P.

Solved Example # 25
3 6
The A.M. of two numbers exceeds the G.M. by and the G.M. exceeds the H.M. by ; find the
2 5
numbers.
Solution.
Let the numbers be a and b, now using the relation
G2 = A.H.
 3  6
= G   G  
 2   5
3 9
= G2 + G–
10 5
 G=6
i.e. ab = 36
also a + b = 15
Hence the two numbers are 3 and 12.

(ii) A.M.  G.M.  H.M.


Let a1, a2, a3, .......an be n positive real numbers, then we define their
a1  a 2  a 3  .......  a n
A.M. = , their
n
n
G.M. = (a1 a2 a3 .........an)1/n and their H.M. = 1 1 1 It can be shown that
  ....... 
a1 a 2 an
A.M.  G.M.  H.M. and equality holds at either places iff
a1 = a2 = a3 = ..............= an

Solved Example # 26
a b c
If a, b, c, > 0 prove that + + 3
b c a
Solution.
Using the relation A.M.  G.M. we have
a b c
 
1
 a b c
b c a   . .   3 a b c
 
 3
3 b c a b c a

Solved Example # 27
 1 1 1
For non-zero x, y, z prove that (x + y + z)  x  y  z   9
 
Solution.
Using the relation A.M.  H.M.
xyz 3
 1 1 1
3  
x y z

 1 1 1
 (x + y + z)  x  y  z   9
 

RESONANCE 12
TM
Solved Example # 28

a
n
If ai > 0  i  N such that i  1 , then prove that (1 + a1) (1 + a2) (1 + a3) ........(1 + an)  2n
i 1
Solution.
Using A.M.  G.M.
1 + a1  2 a 1

1 + a2  2 a 2

1 + an  2 an

 (1 + a1) (1 + a2) .........(1 + an)  2n a1a 2 a 3 ......a n 1 / n


As a1 a2 a3 ..... an = 1
Hence (1 + a1) (1 + a2) .......... (1 + an)  2n.

Solved Example # 29
If n > 0 prove that 2n > 1 + n 2n 1
Solution.
Using the relation A.M.  G.M. on the numbers 1, 2, 22, 23........... 2n–1 we have
1  2  22  .......  2n1
> (1.2 22 23 .........2n–1)1/n
n
Equality does not hold as all the numbers are not equal.
1

2 1  (n 1) n  n
 
n
 > n 2 2 
2 1  
(n 1)
 2n – 1 > n 2 2

(n 1)
 2n > 1 + n 2 2

Solved Example # 30

Find the greatest value of xyz for positive value of x, y, z subject to the condition xy + yz + zx = 12.

Solution.
Using the relation A.M.  G.M.
xy  yz  zx
 (x2 y2 z2)1/3 4  (x y z)2/3
3
 xyz  8

Solved Example # 32
If a, b, c are in H.P. and they are distinct and positive then prove that an + cn > 2bn
Solution.
Let an and cn be two numbers
an  c n
then > (an cn)1/2
2
an + cn > 2 (ac)n/2 ...........(i)
Also G.M. > H.M.
i.e. ac > b (ac)n/2 > bn ...........(ii)
hence from (i) and (ii) an + cn > 2bn

RESONANCE 13
TM
Self Practice Problems :

1. If a, b, c are real and distinct then show that a2 (1 + b2) + b2 (1 + c2) + c2 (1 + a2) > 6abc

2. Prove that nn > 1 . 3 . 5 .........(2n – 1)

3. If a, b, c, d be four distinct positive quantities in G.P. then show that


(i) a+d>b+c
1 1  1 1 1 
(ii) + >2    
ab cd  bd ac ad 

4. Prove that ABC is an equilateral triangle iff tan A + tan B + tan C = 3 3

5. If a, b, c > 0 prove that [(1 + a) (1 + b) (1 + c)]7 > 77 a4 b4 c4

Arithmetico-Geometric Series:
A series each term of which is formed by multiplying the corresponding term of an A.P. & G.P. is called
the AritH.M.eticoGeometric Series. e.g. 1 + 3x + 5x2 + 7x3 +.....
Here 1, 3, 5,.... are in A.P. & 1, x, x2, x3..... are in G.P..

Sum of n terms of an ArithmeticoGeometric Series:


Let Sn = a + (a + d) r + (a + 2 d) r² +..... + [a + (n  1)d] r n1

then Sn =
a


d r 1  r n 1
a  (n  1) d r n , r  1.
1 r 1  r 2 1 r
a dr
Sum To Infinity: If r < 1 & n  then Limit r n = 0  S = 
1  r 1  r  2
n
.

Solved Example # 33
Find the sum of the series
4 7 10
1+ + 2 + 3 + ...... to n terms.
5 5 5
Solution.
4 7 10 3n  2
Let S=1+ + 2 + 3 + ...... + ..........(i)
5 5 5 5n1

 1 1 4 7 3n  5 3n  2
  S= + 2 + 3 + ....... + n1 + ..........(ii)
5 5 5 5 5 5n
(i) – (ii) 
4 3 3 3 3 3n  2
S=1+ + 2 + 3 + ....... + n1 – .
5 5 5 5 5 5n

3   1 
n1 
1   
5 5  3n  2
4  
S =1+ 1 –
5 1 5n
5

3 3 1 3n  2
=1+ – × n1 –
4 4 5 5n

7 12n  7 35 (12n  7)
= – 4. 5 n  S = – .
4 16 16 . 5 n1

RESONANCE 14
TM
Solved Example # 35

Evaluate 1 + 2x + 3x 2 + 4x3 + ......... upto infinity where | x | < 1.


Solution.
Let S = 1 + 2x + 3x 2 + 4x3 + ..... ........(i)
xS = x + 2x2 + 3x3 + ......... ........(ii)
(i) - (ii)  (1 – x) S = 1 + x + x2 + x 3 + ..........
1
or S =
(1  x )2

Solved Example # 36

Evaluate 1 + (1 + b) r + (1 + b + b2) r 2 + ......... to infinite terms for | br | < 1.

Solution.
Let S = 1 + (1 + b)r + (1 + b + b2) r 2 +..... ..........(i)
rS = r + (1 + b) r 2 + ......... ..........(ii)
(i) - (ii)
 (1 – r)S = 1 + br + b2r2 + b3r3 + ......
1
 S=
(1  br )(1  r )

Self Practice Problems :

1. Evaluate
1.2 + 2.22 + 3.23 + ...... + 100. 2100
Ans. 99.2101 + 2.

2. Evaluate
1 + 3x + 6x 2 + 10x3 + ...... upto infinite term where | x | < 1.
1
Ans.
(1  x )3

3. Sum to n terms of the series

 1  1
2

1 + 2 1   + 3 1   + ......
 n  n
Ans. n2

Important Results

    
n n n n n
(i) (ar ± br) = ar ± b r. (ii) k ar = k a r.
r 1 r 1 r 1 r 1 r 1


n
(iii) k = k + k + k...............n times = nk; where k is a constant.
r 1


n
n ( n 1)
(iv) r = 1 + 2 + 3 +...........+ n =
r 1
2


n
n (n  1) (2n  1)
(v) r² = 12 + 22 + 32 +...........+ n2 =
r 1 6


n
n 2 (n  1) 2
(vi) r3 = 13 + 23 + 33 +...........+ n3 =
r 1 4

a
n
(vii) 2 i a j = (a1 + a2 + ........+ an ) 2 – (a12 + a22 + ...... + an2)
i j1

RESONANCE 15
TM
Solved Example # 37
Find the sum of the series to n terms whose general term is 2n + 1.
Solution.
Sn = T n = (2n + 1)
= 2n + 1
2(n  1) n
= +n
2
= n2 + 2n or n(n + 2).

Solved Example # 38

T
n
T k = k 2 + 2k then find k .
k 1

Solution.

T k 2
n n n
2 k
k = +
k 1 k 1 k 1

n (n  1) ( 2n  1) 2(2n  1)
= +
6 2 1
n (n  1) ( 2n  1)
= + 2n + 1 – 2.
6

Solved Example # 39


n i j

Find the value of the expression 1


i  1 j  1k  1
Solution.

 
n i j n i

1 = j
i  1 j  1k  1 i 1 j 1


i (i  1)
n
=
i 1
2

1 
 
n n 
= i2  i
2 i  1 i1 
 

1  n (n  1) (2n  1)  n (n  1) 
=  2 
2  6

n (n  1) n (n  1) (n  2)
=
[2n + 1 + 3] = .
12 6
METHOD OF DIFFERENCE

Type – 1
Let u1, u2, u3 ........ be a sequence, such that u2 – u1, u3 – u2, ......... is either an A.P. or a G.P. then nth term
un of this sequence is obtained as follows
S = u1 + u2 + u3 + ........... + un ................(i)
S= u1 + u2 + ........... + un–1 + un ................(ii)
(i) – (ii)  un = u1 + (u2 – u1) + (u3 – u2) + ........... + (un – un–1)

Where the series (u2 – u1) + (u3 – u2) + .......... + (un – un–1) is

u
k

either in A.P. or in G.P. then we can find un and hence sum of this series as S = r
r 1

RESONANCE 16
TM
Solved Example # 40

Find the sum to n-terms 3 + 7 + 13 + 21 + .........

Solution.
Let S = 3 + 7 + 13 + 21 + ......... + Tn ...........(i)
S= 3 + 7 + 13 + ............ + Tn–1 + Tn ...........(ii)
(i) – (ii)  Tn = 3 + 4 + 6 + 8 + .......... + (Tn – Tn–1)
n 1
=3+ [8 + (n – 2)2]
2
= 3 + (n – 1) (n + 2)
= n2 + n + 1
Hence S =  (n2 + n + 1)
= n2 + n + 1
n(n  1)(2n  1) n(n  1)
= + +n
6 2
n 2
= (n + 3n + 5)
3

Solved Example # 41

Find the sum to n-terms 1 + 4 + 10 + 22 + ........

Solution.
Let S = 1 + 4 + 10 + 22 + .........+ Tn ........(i)
S= 1 + 4 + 10 + ......... + Tn–1 + Tn ........(ii)
(i) – (ii)  Tn = 1 + (3 + 6 + 12 + ......... + Tn – Tn–1 )
 2n1  1 

Tn = 1 + 3  2  1 

 
Tn = 3 . 2n–1 – 2
So S =  Tn = 3  2n–1 – 2
 2n  1 
 
= 3 .  2  1  – 2n
 
= 3.2 – 2n – 3
n

Type – 2

If possible express rth term as difference of two terms as tr = f(r) – f(r ± 1). This can be explained with the help
of examples given below.

Solved Example # 42
Find the sum to n-terms of the series 1.2 + 2.3 + 3.4 + ............
Solution.
Let Tr be the general term of the series
So Tr = r(r + 1).
To express tr = f(r) – f(r–1) multiply and divide tr by [(r + 2) – (r – 1)]
r
so Tr = (r + 1) [(r + 2) – (r – 1)]
3
1
= [r (r + 1) (r + 2) – (r – 1) r (r + 1)].
3
1
Let f(r) = r (r + 1) (r + 2)
3
so Tr = [f(r) – f(r – 1)].

RESONANCE 17
TM
T
n

Now S = r = T1 + T2 + T3 + .........+ Tn
r 1

1
T1 = [1 . 2 . 3 – 0]
3
1
T2 = [2 . 3 . 4 – 1 . 2 . 3]
3
1
T3 = [3 . 4 . 5 – 2 . 3 . 4]
3

1
Tn = [n(n+1) (n + 2) – (n – 1)n (n + 1)]
3
1
 S=
n (n + 1) (n + 2)
3
Hence sum of series is f(n) – f(0).

Solved Example # 43
1 1 1
Sum to n terms of the series + + + .........
(1  x )(1  2x ) (1  2x )(1  3 x ) (1  3 x )(1  4x )
Solution.
Let Tr be the general term of the series
1
Tr =
(1  rx )(1  (r  1)x )

1  [1  (r  1)x ]  (1  rx) 
So Tr =  
x  (1  rx )(1  (r  1)x ) 

1  1  1 
=  
x  1  rx 1  (r  1) x 
Tr = f(r) – f(r + 1)
 S =  Tr = T1 + T2 + T3 + .......... + Tn

1  1  1 
=  
x 1  x 1  (n  1)x 

n
=
(1  x )[1  (n  1)x]

Solved Example # 44
4 5 6
Sun to n terms of the series 1 . 2 . 3 + 2 . 3 . 4 + 3 . 4 . 5 + .........

Solution.
r 3
Let Tr =
r(r  1)(r  2)

1 3
= +
(r  1)(r  2) r(r  1)(r  2)

 1

1  3  1  1 
=   +  
 r  1 r  2  2  r (r  1) (r  1)(r  2 ) 

1 1  3 1  1 
 S=    +  
2 n  2 2  2 (n  1)(n  2) 

RESONANCE 18
TM
5 1  3 
= – 1  
4 n2  2( n  1) 

5 1
= – [2n + 5]
4 2(n  1)(n  2)

Note :
It is not always necessary that the series of first order of differences i.e. u2 – u1, u3 – u2 ....... un – un–1, is
always either in A.P. or in G.P. in such case let u1 = T1 , u2 – u1 = T2 , u3 – u2 = T3 ......., un – un–1 = Tn.
So un = T1 + T2 + ..............+ Tn .........(i)
un = T1 + T2 + .......+ Tn–1 + Tn .........(ii)
(i) – (ii) 
Tn = T1 + (T2 – T1) + (T3 – T2) + ..... + (Tn – Tn–1)
Now, the series (T2 – T1) + (T3 – T2) + ..... + (Tn – Tn–1) is series of second order of differences and when it is
either in A.P. or in G.P. , then un = u1 +  Tr
Otherwise in the similar way we find series of higher order of differences and the nth term of the series. With
the help of following example this can be explained.
Solved Example # 45
Find the nth term and the sum of n term of the series
2, 12, 36, 80, 150, 252
Solution.
Let S = 2 + 12 + 36 + 80 + 150 + 252 + ................+Tn ...........(i)
S= 2 + 12 + 36 + 80 + 150 + 252 + .........+Tn–1 + Tn ...........(ii)
(i) – (ii)  Tn = 2 + 10 + 24 + 44 + 70 + 102 + ............... + (Tn – Tn–1) ...........(iii)
Tn = 2 + 10 + 24 + 44 + 70 + 102 + ....... + (Tn–1–Tn–2) + (Tn – Tn–1) ...........(iv)
(iii) – (iv)  Tn – Tn–1 = 2 + 8 + 14 + 20 + 26 + .........
n
=
[4 + (n – 1) 6] = n [3n – 1] = Tn – Tn–1 = 3n2 – n
2
 general term of given series is  Tn – Tn–1 =  3n2 – n = n3 + n2.
Hence sum of this series is
S =  n3 +  n2
n2 (n  1)2 n(n  1)(2n  1)
= +
4 6

n (n  1)
= (3n2 + 7n + 2)
12
1
n (n + 1) (n + 2) (3n + 1)
12

Solved Example # 46
Find the general term and sum of n terms of the series 9, 16, 29, 54, 103
Sol. Let S = 9 + 16 + 29 + 54 + 103 + ................. + Tn ...........(i)
S= 9 + 16 + 29 + 54 + 103 + ......... + Tn–1 +Tn ...........(ii)
(i) – (ii)  Tn = 9 + 7 + 13 + 25 + 49 + ................. + (Tn – Tn–1) ...........(iii)
Tn = 9 + 7 + 13 + 25 + 49 + ........... + (Tn–1–Tn–2) + (Tn – Tn–1) ...........(iv)

(iii) – (iv)  Tn – Tn–1 = 9 + (–2) + 6 12 
24
........
 = 7 + 6 [2 – 1] = 6(2) + 1.
n–2 n–2
 
(n  2 ) terms

 General term is Tn = 6(2)n–1 + n + 2


Also sum S = Tn
= 62n–1 + n + 2
(2n  1) n (n  1)
=6. + + 2n
2 1 2
n(n  5 )
= 6(2n – 1) +
2
RESONANCE 19
TM
Self Practice Problems :

1. Sum to n terms the following series


1 1 2 1 2  3 2n
(i) + + + ......... Ans.
13
1 2
3 3
13  23  3 3 n 1

1 1 1 1 1 1 
(ii) 1 . 3 . 5 + 3 . 5 . 7 + 5 . 7 . 9 + ......... Ans.   
4  3 ( 2n  1)( 2n  3 ) 

n
(iii) 1 . 5 . 9 + 2 . 6 . 10 + 3 . 7. 11 + ......... Ans. (n + 1) (n + 8) (n + 9)
4

(iv) 4 + 14 + 30 + 52 + 82 + 114 + .......... Ans. n(n + 1)2

3n  n2  n  1
(v) 2 + 5 + 12 + 31 + 86 + ............... Ans.
2

RESONANCE 20
TM
Part : (A) Only one correct option

1. If x  R, the numbers 51+x + 51–x, a/2, 25x + 25–x form an A.P. then 'a' must lie in the interval:
(A) [1, 5] (B) [2, 5] (C) [5, 12] (D) [12, )

 1  1  1
a b c

2. If x > 1 and   ,   ,   are in G.P., then a, b, c are in


x x x
(A) A.P. (B) G.P. (C) H.P. (D) none of these

3. If A, G & H are respectively the A.M., G.M. & H.M. of three positive numbers a, b, & c, then the
equation whose roots are a, b, & c is given by:
(A) x 3  3 Ax2 + 3 G 3x  G 3 = 0 (B) x 3  3 Ax2 + 3 (G 3/H)x  G 3 = 0
(C) x 3 + 3 Ax2 + 3 (G 3/H) x  G 3 = 0 (D) x 3  3 Ax2  3 (G 3/H) x + G 3 = 0


 1
4. The sum is equal to:
r2 r 1
2

(A) 1 (B) 3/4 (C) 4/3 (D) none

5. If a, a1, a2, a3,..., a2n , b are in A.P. and a, g1, g2, g3,.....g2n, b are in G.P. and h is the harmonic mean of

a1  a 2n a 2  a 2n  1 an  an  1
a and b, then g1g2n + g2 g2n  1 + ... + gn gn  1 is equal to

2n n
(A) (B) 2nh (C) nh (D)
h h

6. One side of an equilateral triangle is 24 cm. The midpoints of its sides are joined to form another
triangle whose mid  points are in turn joined to form still another triangle. This process continues
indefinitely. Then the sum of the perimeters of all the triangles is
(A) 144 cm (B) 212 cm (C) 288 cm (D) none of these

1 1 p (1  p) 2
7. If p is positive, then the sum to infinity of the series,   -...... is:
1  p (1  p) 2
(1  p) 3
(A) 1/2 (B) 3/4 (C) 1 (D) none of these

8. In a G.P. of positive terms, any term is equal to the sum of the next two terms. The common ratio of the
G.P. is
(A) 2 cos 18° (B) sin 18° (C) cos 18° (D) 2 sin 18°

1 1 1 2 1 1 1
9. If + + +...... upto  = , then 2 + 2 + 2 +...... =
12
2 2
32
6 1 3 5
(A) 2/12 (B) 2/24 (C) 2/8 (D) none of these

10. The sum to 10 terms of the series 2 + 6 + 18 + 54 + ... is

121
(A) 121 ( 6 + 2) (B) ( 3 + 1) (C) 243 ( 3 + 1) (D) 243 ( 3 – 1)
2

RESONANCE 21
TM
11. If a1, a2,... an are in A.P. with common difference d  0, then the sum of the series
(sin d) [cosec a1 cosec a2 + cosec a2 cosec a3 + ... + cosec an–1 cosec an]
(A) sec a1 – sec an (B) cosec a1 – cosec an
(C) cot a1 – cot an (D) tan a1 – tan an

12. Sum of the series


S = 12 – 22 + 32 – 42 + .... – 20022 + 20032 is
(A) 2007006 (B) 1005004 (C) 2000506 (D) none of these

1 1 1 3 5 2n  1
13. If Hn = 1 + + + ...........+ , then value of 1 + + + ......... + is
2 3 n 2 3 n
(A) 2n – Hn (B) 2n + Hn (C) Hn – 2n (D) Hn + n

1 1 1 1
14. The sum of the series log 4 + log 4 + log 4 + ...... + log 4 is
2 4 8 2n

1 1
(A) n (n + 1) (B) n (n + 1) (2n + 1)
2 12
1 1
(C) n (n  1) (D) n (n + 1)
4

15. If S1 , S2, S3 are the sums of first n natural numbers, their squares, their cubes respectively, then
S 3 (1  8S1 )
is equal to
S 22
(A) 1 (B) 3 (C) 9 (D) 10.

16. If p and q are respectively the sum and the sum of the squares of n successive integers beginning with
‘a’, then nq – p2 is
(A) independent of ‘a’ (B) independent of ‘n’
(C) dependent on ‘a’ (D) none of these

x x( x  a1 ) x( x  a1 )( x  a 2 )
17. Sum of n terms of the series 1 + a + a a + a1a 2a 3 + ... is
1 1 2

x( x  a1 ) ...( x  a n 1 ) ( x  a1 )( x  a 2 ).... ( x  a n 1 )
(A) a1a 2 ...a 3 (B) a1a 2 ...an  1

x( x  a1 ).... ( x  a n )
(C) a1a 2 ...a n (D) none of these

18. {an} and {bn} are two sequences given by an = ( x )1/ 2 + ( y )1/ 2 and bn = ( x )1/ 2 – ( y )1/ 2 for all n  N.
n n n n

The value of a1a2a3........an is equal to


xy xy xy
(A) x – y (B) bn (C) b (D) b
n n

19. If a1, a2, a3, ........., an are positive real numbers whose product is a fixed number c, then the minimum
value of a1 + a2 + a3 + .... + an – 1 + 2an is [IIT - 2002, 3]
(A) n(2c)1/n (B) (n + 1) c1/n (C) 2nc1/n (D) (n + 1)(2c)1/n

RESONANCE 22
TM
Part : (B) May have more than one options correct

 r(r  1) (2r + 3) = an
n
20. If 4
+ bn3 + cn2 + dn + e, then
r 1

(A) a + c = b + d (B) e = 0
(C) a, b – 2/3, c – 1 are in A.P. (D) c/a is an integer

21. The sides of a right triangle form a G.P. The tangent of the smallest angle is
5  1 5 1 2 2
(A) (B) (C) (D)
2 2 5 1 5 1

22. Sum to n terms of the series S = 12 + 2(2)2 + 32 + 2(42) + 52 + 2(62) + ... is


1 1 2
(A) n (n + 1)2 when n is even (B) n (n + 1) when n is odd
2 2

1 2 1
(C) n (n + 2) when n is odd (D) n(n + 2)2 when n is even.
4 4

23. If a, b, c are in H.P., then:

a b c 2 1 1
(A) , , are in H.P. (B) = +
bca ca b a bc b ba bc

b b b a b c
(C) a  , ,c are in G.P.
.P. (D) , , are in H.P..
2 2 2 bc ca a b

24. If b1, b2, b3 (bi > 0) are three successive terms of a G.P. with common ratio r, the value of r for which the
inequality b3 > 4b2 – 3b1 holds is given by
(A) r > 3 (B) r < 1 (C) r = 3.5 (D) r = 5.2

1. If a, b, c are in A.P., then show that:


(i) a2 (b + c), b2 (c + a), c 2 (a + b) are also in A.P.
(ii) b + c  a, c + a  b, a + b  c are in A.P.

2. If a, b, c, d are in G.P., prove that :

(i) (a2  b2), (b2  c2), (c 2  d2) are in G.P.

1 1 1
(ii) , , are in G.P..
a b
2 2
b c
2 2
c  d2
2

3. Using the relation A.M.  G.M. prove that



(i) tan  + cot   2 ; if 0 <  <
2
(ii) (x y + y z + z x) (xy + yz + zx 2) > 9x2 y2 z2.
2 2 2 2 2

(iii) (a + b) . (b + c) . (c + a)  abc ; if a, b, c are positive real numbers

RESONANCE 23
TM
4. Find the sum in the nth group of sequence,
(i) 1, (2, 3); (4, 5, 6, 7); (8, 9,........, 15); ............

(ii) (1), (2, 3, 4), (5, 6, 7, 8, 9),........

5. If n is a root of the equation x² (1  ac)  x (a² + c²)  (1 + ac) = 0 & if n HM’s are inserted between
a & c, show that the difference between the first & the last mean is equal to ac(a  c).

6. The sum of the first ten terms of an AP is 155 & the sum of first two terms of a GP is 9. The first term
of the AP is equal to the common ratio of the GP & the first term of the GP is equal to the common
difference of the AP. Find the two progressions.

5 55 555 5555
7. Find the sum of the series + 2 + 3 + + ... up to 
13 (13) (13) (13)4

8. If 0 < x <  and the expression


exp {(1 + cos x + cos 2 x + cos3 x + cos 4 x + ....... upto ) loge 4}
satisfies the quadratic equation y2 – 20y + 64 = 0 the find the value of x.

9. In a circle of radius R a square is inscribed, then a circle is inscribed in the square, a new square in the
circle and so on for n times. Find the limit of the sum of areas of all the circles and the limit of the sum
of areas of all the squares as n .

10. The sum of the squares of three distinct real numbers, which are in GP is S². If their sum is  S, show
that ²  (1/3, 1)  (1, 3).

11. Let S1, S2,...Sp denote the sum of an infinite G.P. with the first terms 1, 2, ...., p and common ratios

1
1/2, 1/3, ...., 1/(p + 1) respectively. Show that S1 + S2 + ... + Sp = p(p + 3)
2

12. Circles are inscribed in the acute angle  so that every neighbouring circles touch each other. If the
radius of the first circle is R then find the sum of the radii of the first n circles in terms of R and .

13. Given that   are roots of the equation, A x 2  4 x + 1 = 0 and ,  the roots of the equation,
B x 2  6 x + 1 = 0, find values of A and B, such that , ,  &  are in H.P.

14. The airthmetic mean between m and n and the geometric mean between a and b are each equal to

ma  nb
: find the m and n in terms of a and b.
mn

15. If a, b, c are positive real numbers then prove that


(i) b2c 2 + c 2a2 + a2b2 > abc (a + b + c).
(ii) (a + b + c)3 > 27abc.
(iii) (a + b + c)3 > 27 (a + b – c) (c + a – b) (b + c – a)

s s s n2
16. If 's' be the sum of 'n' positive unequal quantities a, b, c,......., then    ...  .
sa s b sc n 1

RESONANCE 24
TM
17. Sum the following series to n terms and to infinity:


n
(i) r (r + 1) (r + 2) (r + 3)
r 1

1 2 3
(ii) + + +........
1  12  14 1  22  2 4 1  32  34

1 16 1 24 1 32
(iii) + 2 2 + + 2 2 + + 2 2 +........
3.5 3 .5 5.7 5 .7 7 . 9 7 .9

18. Let a, b, c d be real numbers in G.P. If u, v, w, satisfy the system of equations


u + 2v + 3w = 6
4u + 5v + 6w = 12
6u + 9v = 4 then show that the roots of the equation

1 1 1 
    x 2 + [(b – c)2 + (c – a)2 + (d – b) 2] x + u + v + w = 0 and
u v w 
20x2 + 10 (a – d)2 x – 9 = 0 are reciprocals of each other. [IIT- 1999, 10]

19. The fourth power of the common difference of an arithmetic progression with integer entries added to
the product of any four consecutive terms of it. Prove that the resulting sum is the square of an integer.
[IIT - 2000, 4]

20. If a, b & c are in arithmetic progression and a2, b2 & c2 are in harmonic progression, then prove that
c
either a = b = c or a, b &  are in geometric progression. [IIT – 2003, 4]
2

EXERCISE # 1 65  2 
7. 8. , , 9. 2 R2; 4 R2
36 2 3 3
1. D 2. A 3. B 4. B 5. A 6. A 7. A

8. D 9. C 10. A 11. C 12. A 13. A 14. D


12.
  
R 1  sin 2  1  sin 2 

n

 1 13. A = 3; B = 8

2 sin 2 
 1  sin 2 
  
15. C 16. A 17. B 18. C 19. A 20. ABCD  

21. BC 22. AB 23. ABCD 24. ABCD 2b  a 2a  b


14. m = ,n=
a  b a  b
EXERCISE # 2 17. (i) (1/5) n (n + 1) (n + 2) (n + 3) (n + 4)

4. (i) 2n  2 (2n + 2n  1  1) (ii) (n  1) 3 + n3 n (n  1) 1


(ii) ; s =
2 (n  n  1)
2 2
6. (3 + 6 + 12 +......); (2/3 + 25/3 + 625/6 +......)
n 4 n (n  3)
(iii) +
3 (2 n  3) 9 (2 n  3)2

RESONANCE 25
TM
Determinant

1. Definition:
Let us consider the equations a1x + b1y = 0, a2x + b2y = 0
As the sun  –
a1
=
y
=–
a2
b1 x b2
eclipses the
a1 a2
stars by its  =
b1 b2
brilliancy,so  a 1b 2 – a2b 1 = 0
the man of a1 b1
we express this eliminant as =0
knowledge will a2 b2
eclipse the fame a1 b1
The symbol is called the determinant of order two.
of others in a2 b2
assemblies of Its value is given by: D = a1 b2  a2 b1
the people if he
2. Expansion of Determinant:
proposes
algebraic a1 b1 c1
The symbol a 2 b2 c 2 is called the determinant of order
problems, and
a3 b3 c3
still more if he
three.
solves them.
Its value can be found as:
Brahmagupta, b2 c 2 b1 c1 b1 c1
D = a1  a2 + a3 OR
b3 c 3 b3 c3 b2 c2
I regard it as b2 c 2 a2 c2 a2 b2
aninelegance,or D = a1  b1 + c1 ... & so on.
b3 c 3 a3 c3 a3 b3
imperfection,in In this manner we can expand a determinant in 6 ways using
quaternions, or elements of ; R1, R2, R3 or C1, C2, C3.
rather in the
3. Minors:
state to which
The minor of a given element of a determinant is the
it has been
determinant of the elements which remain after deleting the
hitherto row & the column in which the given element stands. For
unfolded, a1 b1 c 1
b2 c2
whenever it be example, the minor of a1 in a 2 b 2 c 2 is & the
b3 c3
a3 b3 c3
a1 c1
minor of b2 is .
a3 c3
Hence a determinant of order two will have “4 minors” & a
determinant of order three will have “9 minors”.

RESONANCE TM 45
4. Cofactor:
Cofactor of the element aij is Cij = (1)i+j. Mij; Where i & j denotes the row & column in which the
particular element lies.
Note that the value of a determinant of order three in terms of ‘Minor’ & ‘Cofactor’ can be written as:
D = a11M11  a12M12 + a13M13
OR D = a11C11 + a12C12 + a13C13 & so on.

5. Transpose of a Determinant:
The transpose of a determinant is a determinant obtained after interchanging the rows & columns.

a1 b1 c1 a1 a 2 a3
D = a2 b2 c2  D 
T
b1 b 2 b3
a3 b 3 c3 c1 c 2 c3

6. Symmetric, Skew -Symmetric, Asymmetric Determinants:


(i) A determinant is symmetric if it is identical to its transpose. Its ith row is identical to its ith
column i.e. aij = aji for all values of ' i ' and ' j '
(ii) A determinant is skew-symmetric if it is identical to its transpose having sign of each element
inverted i.e. aij = – aji for all values of ' i ' and ' j '. A skew-symmetric determinant has all elements
zero in its principal diagonal.
(iii) A determinant is asymmetric if it is neither symmetric nor skew-symmetric.

7. Properties of Determinants:
(i) The value of a determinant remains unaltered, if the rows & columns are inter changed,

a1 b1 c1 a1 a 2 a3
i.e. D = a2 b2 c 2  b1 b 2 b 3 = D
a3 b3 c3 c1 c 2 c3
(ii) If any two rows (or columns) of a determinant be interchanged, the value of determinant
is changed in sign only. e.g.

a1 b1 c1 a2 b2 c2
Let D = a 2 b2 c2 & D = a1 b1 c 1 Then D =  D.
a3 b3 c3 a 3 b3 c 3

NOTE : A skew-symmetric deteminant of odd order has value zero.


(iii) If a determinant has all the elements zero in any row or column then its value is zero,

0 0 0
i.e. D = a 2 b2 c 2 = 0.
a 3 b3 c 3
(iv) If a determinant has any two rows (or columns) identical, then its value is zero,
a1 b1 c1
i.e. D = a1 b1 c 1 = 0.
a 3 b3 c 3

RESONANCE TM 46
(v) If all the elements of any row (or column) be multiplied by the same number, then the determinant
is multiplied by that number, i.e.

a1 b1 c1 Ka1 Kb1 Kc1


D = a2 b2 c2 and D = a 2 b2 c2 Then D= KD
a3 b3 c3 a3 b3 c3

(vi) If each element of any row (or column) can be expressed as a sum of two terms then the
determinant can be expressed as the sum of two determinants, i.e.

a1x b1y c1z a1 b1 c1 x y z


a2 b2 c2  a2 b2 c 2  a2 b2 c2
a3 b3 c3 a3 b3 c3 a3 b3 c3

(vii) The value of a determinant is not altered by adding to the elements of any row (or column) a
constant multiple of the corresponding elements of any other row (or column),

a1 b1 c 1 a1  ma 2 b1  mb 2 c 1  mc 2
i.e. D = a2 b2 c 2 and D = a2 b2 c2 . Then D= D.
a 3 b3 c 3 a 3  na1 b3  nb1 c 3  nc 1

a b c
Example : Simplify b c a
c a b

Solution. Let R 1  R1 + R 2 + R 3

abc abc abc


 b c a
c a b

1 1 1
= (a + b + c) b c a
c a b
Apply C 1 C 1 – C 2, C 2  C 2 – C 3

0 0 1
= (a + b + c) b  c c  a a
c a a b b

= (a + b + c) ((b – c) (a – b) – (c – a)2)
= (a + b + c) (ab + bc – ca – b2 – c 2 + 2ca – a2)
= (a + b + c) (ab + bc + ca – a2 – b2 – c2)
= 3abc – a3 – b3 – c 3

a b c
Example : Simplify a2 b2 c2
bc ca ab

Solution. Given detereminant is equal to

a2 b2 c2
1
= a 3
b 3
c3
abc
abc abc abc

RESONANCE TM 47
a2 b2 c2
abc
= a3 b3 c3
abc
1 1 1

Apply C1  C1 – C2, C2  C2 – C3

a2  b2 b2  c 2 c2
= a3  b 3 b3  c 3 c3
0 0 1

ab bc c2
= (a – b) (b – c) a  ab  b b  bc  c 2 c3
2 2 2

0 0 1

= (a – b) (b – c) [ab2 + abc + ac2 + b3 + b2C + bc2 – a2b – a2c – ab2 – abc – b3 – b2c]
= (a – b) (b – c) [c(ab + bc + ca) – a(ab + bc + ca)]
= (a – b) (b – c) (c – a) (ab + bc + ca)
Use of factor theorem.

USE OF FACTOR THEOREM TO FIND THE VALUE OF DETERMINANT

If by putting x = a the value of a determinant vanishes then (x  a) is a factor of the determinant.

a b c
Example : Prove that a b c = (a – b) (b – c) (c – a) (ab + bc + ca) by using factor theorem.
2 2 2

bc ca ab

Solution. Let a = b

a b c
 D= a b c2
2 2
=0
bc ac ab

Hence (a – b) is a factor of determinant


Similarly, let b = c, D = 0
c = a, D = 0
Hence, (a – b) (b – c) (c – a) is factor of determinant. But the given determinant is of fifth
order so

a b c
a 2
b 2
c2 = (a – b) (b – c) (c – a) ( (a2 + b2 + c 2) + µ (ab + bc + ca))
bc ca ab

Since this is an identity so in order to find the values of  and µ. Let


a = 0, b = 1, c = – 1
– 2 = (2) (2 – µ)
(2 – µ) = – 1. ........(i)
Let a = 1, b = 2, c = 0

1 2 0
1 4 0 = (–1) 2 (– 1) (5 + 2µ)
0 0 2

 5 + 2µ = 2 .......(ii)

RESONANCE TM 48
from (i) and (ii)  = 0 and µ = 1

a b c
Hence a 2
b 2
c2 = (a – b) (b – c) (c – a) (ab + bc + ca).
bc ca ab

Self Practice Problems

0 ba c a
1. Find the value of  = a  b 0 c b .
ac bc 0

Ans. 0

b 2  ab b  c bc  ac
2. Simplify ab  a a  b b  ab .
2 2

bc  ac c  a ab  a 2

Ans. 0

ab c 2a 2a
2b bc a 2b
3. Prove that = (a + b + c)3.
2c 2c c ab

1 a bc
4. Show that 1 b ca = (a – b) (b – c) (c – a) by using factor theorem .
1 c ab

8. Multiplication Of Two Determinants:

a1 b1 m1 a  b  a1m1b1m 2
 1  1 1 1 2

a2 b2  2 m2 a 2  1b 2  2 a 2 m1b 2 m 2

a1 b1 c1  1 m1 n1 a1 1  b1 2  c1 3 a1m1  b1m 2  c 1m3 a1n1  b1n2  c 1n3


a2 b2 c 2 ×  2 m 2 n2 = a 2  1  b 2  2  c 2  3 a 2m1  b 2m 2  c 2m3 a 2n1  b 2n2  c 2n3
a3 b3 c3  3 m 3 n3 a3  1  b 3  2  c 3  3 a 3m1  b3m 2  c 3m3 a 3n1  b 3n2  c 3n3

We have multiplied here rows by rows but we can also multiply rows by columns, columns by rows and
columns by columns.
If  = |aij| is a detereminant of order n, then the value of the determinant |Aij| = n – 1. This is also known
as power cofactor formula.
1 2 3 0 1 8
Example : Find the value of × and prove that it is equal to .
1 3 1 4  6 12

1 2 3 0
Solution. ×
1 3 1 4

1 3  2  1 1 0  2  4
=
 1 3  3  ( 1)  1 0  3  4

1 8
=
 6 12
= 60
RESONANCE TM 49
a1x1  b1y1 a1x 2  b1y 2 a1x 3  b1y 3
Example : Prove that a 2 x1  b 2 y1 a 2 x 2  b 2 y 2 a2 x 3  b2 y 3 = 0
a 3 x1  b 3 y 1 a 3 x 2  b 3 y 2 a3 x 3  b 3 y 3

Solution. Given determinant can be splitted into product of two determinants

a1x1  b1y1 a1x 2  b1y 2 a1x 3  b1y 3 a1 b1 c 1 x1 x 2 x3


i.e. a 2 x1  b 2 y 1 a 2 x 2  b 2 y 2 a2 x 3  b2 y 3 = a 2 b 2 c 2 × y1 y 2 y3 = 0
a 3 x1  b 3 y 1 a 3 x 2  b 3 y 2 a3 x 3  b 3 y 3 a 3 b3 c 3 0 0 0

(a1  b1 )2 (a1  b 2 )2 (a1  b 3 )2


Example : Prove that (a 2  b1 )2 (a 2  b 2 ) 2 (a 2  b 3 )2
(a 3  b1 )2 (a 3  b 2 ) 2 (a 3  b 3 )2

= 2(a1 – a2) (a2 – a3) (a3 – a1) (b1 – b2) (b2 – b3) (b3 – b1).

(a1  b1 )2 (a1  b 2 )2 (a1  b 3 )2


Solution. (a 2  b1 )2 (a 2  b 2 ) 2 (a 2  b 3 )2
(a 3  b1 )2 (a 3  b 2 ) 2 (a 3  b 3 )2

a1  b1  2a1b1 a1  b 2  2a1b 2 a1  b3  2a1b3


2 2 2 2 2 2

= a 2  b1  2a 2b1 a 2  b 2  2a 2b 2 a 2  b3  2a 2b 3
2 2 2 2 2 2

a3  b1  2a 3b1 a3  b 2  2a 3b 2 a 3  b3  2a 3b 3
2 2 2 2 2 2

a1 1  2a1 1 1 1
2

a2 1  2a 2 b1 b2 b3
2 2 2 2
= ×
a3 1  2a 3 b1 b2 b3
2

1 a1 a1 1 b1 b1
2 2

= 2 1 a2 a2 × 1 b2 b2
2 2

1 a3 a3 1 b3 b3
2 2

= 2(a1 – a2) (a2 – a3) (a3 – a1) (b1 – b2) (b2 – b3) (b3 – b1)

cos( A  P) cos( A  Q) cos( A  R )


Example : Prove that cos(B  P) cos(B  Q) cos(B  R) = 0
cos(C  P) cos(C  Q) cos(C  R)

cos( A  P) cos( A  Q) cos( A  R )


Solution. cos(B  P) cos(B  Q) cos(B  R)
cos(C  P) cos(C  Q) cos(C  R)
cos A cos P  sin A sin P cos A cos Q  sin A sin Q cos A cos R  sin A sin R
= cos B cos P  sin B sinP cos B cos Q  sin B sin Q cos B cos R  sin B sinR
cos C cos P  sin C sin P cos C cos Q  sin C sin Q cos C cos R  sin C sin R

cos A sin A 0 cos P cos Q cos R


cos B sin B 0 sin P sin Q sinR
= ×
cos C sin C 0 0 0 0

= 0 × 0 = 0.

RESONANCE TM 50
Self Practice Problems

1. Find the value of 

2bc  a 2 c2 b2
c2 2ca  b 2 a2
b2 a2 2ab  c 2

Ans. (3abc – a3 – b3 – c 3)2

1 cos(B  A ) cos(C  A )
2. If A, B, C are real numbers then find the value of  = cos( A  B) 1 cos(C  B) .
cos( A  C) cos(B  C) 1

Ans. 0

9. Summation of Determinants

f(r) g(r ) h(r )


Let (r) = a1 a 2 a 3
b1 b 2 b 3

where a1, a2, a3, b1, b2, b3 are constants indepedent of r, then

   h(r )
n n n
f (r ) g(r )

 (r ) =
n r 1 r 1 r 1
a1 a2 a3
r 1 b1 b2 b3

Here function of r can be the elements of only one row or column. None of the elements other then that
row or column should be dependent on r. If more than one column or row have elements dependent on
r then first expand the determinant and then find the summation.

2r  1 n Cr 2r

n

Example : Evaluate x cos 2  y


r 1 n2 2n  1 2n1  2

  2
n n n
(2r  1)
D
n n Cr r

r 1 r 1 r 1
Solution : =
cos 2 
r
r 1 x y
n2 2 1 2  2
n n1

n2 2n  1 2n 1  2
= x cos 2  y
n2 2 1 2  2
n n 1

=0

RESONANCE TM 51
n 2 n2 n2
Cr  2 Cr 1 Cr
Example : Dr = 3 1 1
2 1 0

D
n

evaluate r
r 2


n 2 n2 n2

D
n n Cr  2 Cr 1 Cr
Solution : r = 3 1 1
r 2 r 2
2 1 0

n2
C 0  n  2C1  ....  n 2 Cn 2 n2
C1  n 2 C 2  ....  n 2 Cn 2 n2
C 2  n 2 C3  ....  n 2 Cn 2
= 3 1 1
2 1 0

2n 2 2n  2  1 2n  2  1  n
= 3 1 1
2 1 0

C1  C1 – 2 × C2

2n 2  2n 1  2 2n 2  1 2n 2  1  n
= 1 1 1
0 1 0

2n2  2n1  2 2n2  1  n


= (–1)
1 1
= 2n – 1 – n – 3

r 1

1 0 n
Example : If r = 2 3  r , find
r r
r 1 1  2 r 1

Solution. On expansion of determinent, we get


Dr = (r –1) (3 – r) + 7 + r 2 + 4r = 8r + 4


n
 r = 4n (n + 2)
r 1

Self Practice Problem

r 1

n x 6
1. Evaluate Dr (r  1) 2
y 4n  2
(r  1) z 3n2  3n
r 1 3

Ans. 0

RESONANCE TM 52
1 0 . Integration of a determinant

f ( x ) g( x ) h( x )
a1 b1 c1
Let (x) =
a2 b2 c2

where a1, b1, c1, a2, b2, c 2 are constants independent of x. Hence

   h( x) dx
b b b
f ( x ) dx g( x ) dx

 (x ) dx
b
a a a
= a1 b1 c1
a a2 b2 c2

Note : If more than one row or one column are function of x then first expand the determinant and then
integrate it.

 f ( x ) dx
cos x 1 0 /2

Example : If f(x) = 1 2 cos x 1 , then find


0 1 2 cos x 0

Solution. Here f(x) = cos x (4 cos2x – 1) –2 cos x


= 4 cos 3x – 3 cos x = cos 3x


/2
sin 3 x 
/2
1
3  0
so cos 3 x dx = =–
3
0

 2  1 2  2  2  3

 (x ) dx
1
Example : If  = 6 4 3 , then find
x x 2
x3 0

2  1 2  2 2  3

 (x ) dx
1
Solution. = 6 4 3

 x dx  x x
1 1 1
dx dx
0 2 3

0 0 0

 2  1 2  2  2  3  2  1 2  2  2  3
6 4 3 1 6 4 3
= 1 1 1 = =0
12 6 4 3
2 3 4

RESONANCE TM 53
1 1 . Differentiation of Determinant:
f1( x ) f2 ( x ) f3 ( x )
Let (x) = g1( x ) g2 ( x ) g3 ( x )
h1( x ) h2 ( x ) h3 ( x )

f1( x ) f2 ( x ) f3 ( x ) f1( x ) f2 ( x ) f3 ( x ) f1( x ) f2 ( x ) f3 ( x )


then (x) = g1( x ) g2 ( x ) g3 ( x ) + 1g ( x ) g2 ( x ) g3 ( x ) + 1( x ) g 2 ( x ) g3 ( x )
g
h1( x ) h2 ( x ) h3 ( x ) h1( x ) h2 ( x ) h3 ( x ) h1 ( x ) h2 ( x ) h3 ( x )

3 2 1
If f(x) = 6x 2x x 4 , then find the value of f(a).
2 3
Example :
1 a a2

3 2 1
Solution. f(x) = 12x 6 x 4x 3
2

1 a a2

3 2 1
f(x) = 12 12x 12x
2

1 a a2

3 2 1
 f(a) = 12 1 a a = 0.
2

1 a a2

Example : Let  be a repeated root of quadratic equation f(x) = 0 and A(x), B(x) and C(x) be polynomial of
degree 3, 4 and 5 respectively, then show that

A( x ) B( x ) C( x )
A( ) B( ) C( )
A( ) B( ) C( )

divisible by f(x).

A( x ) B( x ) C( x )
Solution. Let g(x) = A( ) B( ) C( )
A( ) B( ) C( )

A( x ) B( x ) C( x )


 g(x) = A( ) B( ) C( )
A( ) B( ) C( )

Since g() = g() = 0


 g(x) = (x – )2 h(x) i.e.  is the repeated root of g(x) and h(x) is any polynomial
expression of degree 3. Also f(x) = 0 have repeated root . So g(x) is divisible by f(x).

RESONANCE TM 54
Example : Prove that F depends only on x1, x 2 and x3
1 1 1
F= x1  a1 x 2  a1 x 3  a1
x12  b1x1  b 2 x 22  b1x 2  b 2 x 32  b1x 3  b 2
and simplify F.

0 0 0
dF
Solution : = x1  a1 x 2  a1 x 3  a1
da1
x12  b1x1  b 2 x 22  b1x 2  b 2 x 32  b1x 3  b 2

1 1 1 1 1 1
+ 1 1 1 + x1  a1 x 2  a1 x 3  a1 = 0
x12  b1x1  b 2 x 22  b1x 2  b 2 x 32  b1x 3  b 2 0 0 0

Hence F is independent of a1.

dF dF
Similarly = = 0.
db1 db 2
Hence F is independent of b1 and b2 also.
So F is dependent only on x 1, x 2, x 3
Put a1 = 0, b1 = 0, b2 = 0

1 1 1
 x
F= 1 x2 x3
x12 x 22 x 32

= (x 1 – x 2) (x2 – x 3) (x 3 – x1).

ex sin x
Example : If = A + Bx + Cx 2 + ....., then find the value of A and B.
cos x n(1  x )
Solution : Put x = 0 in

ex sin x
= A + Bx + Cx2 + .......
cos x n(1  x )

1 0
 =A
1 0
A = 0.
Differentiating the given determinant w.r.t x, we get

ex cos x ex sin x
+  sin x 1 = B + 2 C x + ......
cos x n(1  x ) 1 x
Put x = 0, we get
1 1 1 0
+ =0
1 0 0 1
 B = –1 + 1 = 0
 A = 0, B = 0

RESONANCE TM 55
Self Practice Problem

x x 1 x
 2x x  1 1
1. If = ax3 + bx 2 + cx + d. Find
x 1 1 x

(i) d Ans. [– 1]
(ii) a+b+c+d Ans. [– 5]
(iii) b Ans. [– 4]

1 2 . Cramer's Rule: System of Linear Equations


(i) Two Variables
(a) Consistent Equations: Definite & unique solution. [ intersecting lines ]
(b) Inconsistent Equation: No solution. [ Parallel line ]
(c) Dependent equation: Infinite solutions. [ Identical lines ]

Let a1x + b1y + c1 = 0 & a2x + b2y + c 2 = 0 then:


a1 b c
 1  1  Given equations are inconsistent &
a2 b2 c2

a1 b c
 1  1  Given equations are dependent
a2 b2 c2
(ii) Three Variables

Let, a1x + b1y + c1z = d1............ (I)


a2x + b2y + c2z = d2............ (II)
a3x + b3y + c3z = d3............ (III)
D1 D2 D3
Then, x= ,Y= ,Z= .
D D D

a1 b1 c 1 d1 b1 c 1 a1 d1 c 1 a1 b1 d1
a2 b2 c 2 d2 b 2 c 2 a2 d2 c 2 a2 b2 d2
Where D = ; D1 = ; D2 = & D3 =
a3 b3 c3 d3 b3 c3 a3 d3 c3 a3 b3 d3

(iii) Consistency of a system of Equations


(a) If D  0 and alteast one of D1, D2, D3  0, then the given system of equations are consistent and
have unique non trivial solution.
(b) If D  0 & D1 = D2 = D3 = 0, then the given system of equations are consistent and have trivial
solution only.
(c) If D = D1 = D2 = D3 = 0, then the given system of equations have either infinite solutions or no
solution.

(Refer Example & Self Practice Problem with*)


(d) If D = 0 but atleast one of D1, D2, D3 is not zero then the equations are inconsistent and have no
solution.

RESONANCE TM 56
(e) If a given system of linear equations have Only Zero Solution for all its variables then the given
equations are said to have TRIVIAL SOLUTION.
(iv) Three equation in two variables :
If x and y are not zero, then condition for a1x + b1y + c 1 = 0 ; a2x + b2y + c 2 = 0 &

a1 b1 c 1
a3x + b3y + c 3 = 0 to be consistent in x and y is a 2 b 2 c 2 = 0.
a 3 b3 c 3

Example: Find the nature of solution for the given system of equations.
x + 2y + 3z = 1
2x + 3y + 4z = 3
3x + 4y + 5z = 0

1 2 3
Solution. Let D = 2 3 4
3 4 5

apply C1  C1 – C2 , C2  C2 – C3
1 1 3
D = 1 1 4 =0 D=0
1 1 5
1 2 3
Now, D1 = 3 3 4
0 4 5

C3  C3 – C2

1 2 1
D1 = 3 3 1
0 4 1

R1  R1 – R2 , R2  R2 – R3

 2 1 0
D1 = 3 1 0 =5
0 4 1

D = 0 But D1  0 Hence no solution

*Example : Solve the following system of equations


x+y+z=1
2x + 2y + 2z = 3
3x + 3y + 3z = 4

1 1 1
Solution.  D= 2 2 2 =0
3 3 3

D1 = 0, D2 = 0, D3 = 0
 Let z = t
x+y=1–t
2x + 2y = 3 – 2t
Since both the lines are parallel hence no value of x and y Hence there is no solution of the
given equation.

RESONANCE TM 57
*Example : Solve the following system of equations
x+y+z=2
2x + 2y + 2z = 4
3x + 3y + 3z = 6

1 1 1
2 2 2
Solution.  D= =0
3 3 3

D1 = 0, D2 = 0, D3 = 0
All the cofactors of D, D1, D2 and D3 are all zeros, hence the system will have infinite solutions.
Let z = t1, y = t2  x = 2 – t1 – t2
where t1, t2  R.

Example : Consider the following system of equations


x+y+z=6
x + 2y + 3z = 10
x + 2y + z = 
Find values of  and  if such that sets of equation have
(i) unique solution (ii) infinite solution
(iii) no solution
Solution. x+y+z=6
x + 2y + 3z = 10
x + 2y + z = 

1 1 1
1 2 3
D=
1 2 

Here for  = 3 second and third rows are identical hence D = 0 for  = 3.

6 1 1
10 2 3
D1 =
 2 

1 6 1
1 10 3
D2 =
1  

1 1 6
1 2 10
D3 =
1 2 

If  = 3 then D1 = D2 = D3 = 0 for µ = 10

(i) For unique solution D  0


i.e. 3

(ii) For infinite solutions


D=0  =3
D1 = D2 = D3 = 0  µ = 10.
(iii) For no solution
D=0  =3
Atleast one of D1, D2 or D3 is non zero  µ  10.

RESONANCE TM 58
Self Practice Problems

*1. Solve the following system of equations


x + 2y + 3z = 1
2x + 3y + 4z = 2
3x + 4y + 5z = 3
Ans. x=1+t y = –2t z=t where t  R

2. Solve the following system of equations


x + 2y + 3z = 0
2x + 3y + 4z = 0
x–y–z=0
Ans. x = 0, y = 0, z = 0

3. Solve: (b + c) (y + z)  ax = b  c, (c + a) (z + x)  by = c  a, (a + b) (x + y)  cz = a  b
where a + b + c  0.
c b ac ba
Ans. x= ,y= ,z=
a bc a bc a bc

4. Let 2x + 3y + 4 = 0 ; 3x + 5y + 6 = 0, 2x 2 + 6xy + 5y2 + 8x + 12y + 1 + t = 0, if the system of equations


in x and y are consistent then find the value of t.
Ans. t=7

1 3 . Application of Determinants:
Following examples of short hand writing large expressions are:
(i) Area of a triangle whose vertices are (x r, yr); r = 1, 2, 3 is:

x1 y1 1
1 x2 y2 1
D= If D = 0 then the three points are collinear.
2 x3 y3 1

x y 1
(ii) Equation of a straight line passing through (x1, y1) & (x2, y2) is x1 y1 1 = 0
x2 y2 1

(iii) The lines: a1x + b1y + c 1 = 0........ (1)


a2x + b2y + c 2 = 0........ (2)
a3x + b3y + c 3 = 0........ (3)

a1 b1 c1
are concurrent if, a2 b2 c 2 = 0.
a3 b3 c3

Condition for the consistency of three simultaneous linear equations in 2 variables.


(iv) ax² + 2 hxy + by² + 2 gx + 2 fy + c = 0 represents a pair of straight lines if:

a h g
abc + 2 fgh  af²  bg²  ch² = 0 = h b f
g f c

RESONANCE TM 59
Part : (A) Only one correct option

a a  a a 
b b  b b 
x 2 x 2
x x
1

c c  c c 
y 2 y 2
1. If a, b, c > 0 & x, y, z  R, then the determinant 1 =
y y

z 2 z 2
z z
1

(A) axbyc z (B) axbyc z (C) a2xb2yc 2z (D) zero

b2c2 bc b  c
2. If a, b & c are non-zero real numbers , then D = c a ca c  a =
2 2

a2b2 ab a  b

(A) abc (B) a2 b2 c 2 (C) bc + ca + ab (D) zero

b1c 1 c 1 a 1 a1b1
3. The determinant b 2 c 2 c 2 a 2 a 2 b 2 =
b 3 c 3 c 3 a 3 a 3 b 3

a1 b1 c1 a1 b1 c1 a1 b1 c1
(A) a 2 b 2 c2 (B) 2 a 2 b 2 c2 (C) 3 a 2 b2 c2 (D) none of these
a 3 b3 c3 a 3 b3 c3 a3 b3 c3

4. The system of linear equations x + y  z = 6, x + 2y  3z = 14 and 2x + 5y  z = 9


(  R) has a unique solution if
(A)  = 8 (B)   8 (C)  = 7 (D)   7

5. If the system of equations x + 2y + 3z = 4, x + py + 2z = 3, x + 4y + z = 3 has an infinite number of


solutions, then:
(A) p = 2,  = 3 (B) p = 2,  = 4 (C) 3 p = 2  (D) none of these

cos 2  cossin sin


 
6. Let f () = cos sin sin 2  cos then f   =
 6
sin cos 0

(A) 0 (B) 1 (C) 2 (D) none

cos() sin() cos2


7. The determinant sin cos sin is:
cos sin cos
(A) 0 (B) independent of 
(C) independent of  (D) independent of  &  both

RESONANCE 60
TM
sin(2 ) sin(  ) sin(   )
8. Value of  = sin(   ) sin(2) sin(   ) is
sin(    ) sin(   ) sin(2 )

(A)  = 0 (B)  = sin2 + sin2 + sin2


(C)  = 3/2 (D) none of these

0 b c
9. If a, b, c are complex number and z = b 0  a is
c a 0

(A) purely real (B) purely imaginary (C) 0 (D) none of these

A B C
sin sin sin
2 2 2
B A
10. If A, B, C are angles of a triangle ABC, then sin( A  B  C) sin cos is less than or
2 2
( A  B  C) C
cos tan( A  B  C) sin
equal to 2 2

3 3 1
(A) (B) (C) 2 2 (D) 2
8 8

4 sinB
1 cos A
b
11. = 2a 8 sin A 1 is (where a, b, c are the sides opposite to angles A, B, C respectively in a
3a 12 sin A cos B

triangle)
1 1 1
(A) cos2A (B) 0 (C) sin2A (D) (cos 2A + cos 2B)
2 2 2

(b  c ) 2 a2 a2
12. If b2 (c  a ) 2 b2 = k abc (a + b + c)3 then the value of k is
c2 c2 (a  b )2

(A) 1 (B) 2 (C) 0 (D) ab + bc + ac

2r1 Cr
m
1
 Dr
m

 
13. Let m be a positive integer & Dr = m 1 2m m1 (0  r  m), then the value of is
2

sin (m) sin (m1)


r0
sin2 m 2 2 2

given by:
(A) 0 (B) m 2  1 (C) 2m (D) 2m sin2 (2m)

a 1  2i 3  5i
14. If a, b, c, are real numbers, and D = 1  2i b  7  3i then D is
3  5i  7  3i c

(A) purely real (B) purely imaginary (C) non real (D) integer

RESONANCE 61
TM
1 x x 1
15. If f(x) = 2x x  x  1 x  1 x then f(100) is equal to: [IIT – 1999, 2]
3x  x  1 x  x  1  x  2  x  1 x x  1
(A) 0 (B) 1 (C) 100 (D)  100

Part : (B) May have more than one options correct

1 1 1
16. Let 1 (x) = x + a1, 2 (x) = x 2 + b1x + b2 and  = 1( x1 ) 1( x 2 ) 1( x 3 ) , then
 2 ( x1 ) 2 ( x 2 )  2 ( x 3 )

(A)  is independent of a1 (B)  is independent of b1 and b2


(C)  is independent of x1, x 2 and x 3 (D) none of these

x 2y  z z
y 2x  z z
17. If  = , then
y 2y  z 2 x  2y  z

(A) x – y is a factor of  (B) (x – y)2 is a factor of 


(C) (x – y) 3 is a factor of  (D)  is independent of z

sin  cos  sin  sin  cos 


18. Let = cos  cos  cos  sin   sin  , then
 sin  sin  sin  cos  0

(A)  is independent of  (B)  is indepedent of 

d 
(C)  is a constant 0
d    / 2
(D)

a 1 0
19. Let  = ax  1 , then
a
ax 2 ax a

(A) x + a is a factor of  (B) (x + a) 2 is a factor of 


(C) (x + a) 3 is a factor of  (D) (x + a)4 is not a factor of 

1 x x2
20. Let  = x 1 x , then
2

x x2 1

(A) 1 – x3 is a factor of  (B) (1 – x 3)2 is factor of 


(C) (x) = 0 has 4 real roots (D) (1) = 0

b c b  c
21. The determinent  = c d c  d is equal to zero if
b  c c  d a  c 3

(A) b, c, d are in A.P. (B) b, c, d are in G.P.


(C) b, c, d are in H.P. (D)  is a root of ax 3 – bx2 – 3cx – d = 0

RESONANCE 62
TM
1. Using the properties of determinants, evalulate:

103 115 114 113 116 104 13  3 2 5 5


(i) 111 108 106 + 108 106 111 . (ii) 15  26 5 10 .
104 113 116 115 114 103 3  65 15 5

ax  b a b
2. Find the non  zero roots of the equation,  = b x  c = 0. b c
ax  b bx  c c

b2  c 2 ab ac
3. Show that  = ab c 2  a2 bc = 4a2b2c 2
ca cb a b
2 2

2     
4. Prove that,        2(  )(    )(   )  (  ) = 0.
   (   )  (  ) 2

S0 S1 S2
5. If Sr = r + r + r then show that S1 S2 S 3 = (  )2 ( )2 (  )2.
S2 S3 S4

6. Find the value of ‘a’ if the three equations,


(a + 1)3 x + (a + 2)3 y = (a + 3) 3; (a + 1) x + (a + 2) y = (a + 3) & x + y = 1 are consistent.

7. Investigate for what values of ,  the simultaneous equations


x + y + z = 6; x + 2 y + 3 z = 10 & x + 2 y +  z =  have;
(a) A unique solution
(b) An infinite number of solutions.
(c) No solution.

8. Find those values of c for which the equations:


2x+3y = 3
(c + 2) x + (c + 4) y = c + 6
(c + 2)² x + (c + 4)² y = (c + 6)² are consistent.
Also solve above equations for these values of c.

 ''  '  '


9. Prove that  =  '  '   ' ' = (' – ') (' – ') (' – ')
 ''  ' '

a 2  ( b 2  c 2 )cos  ab (1  cos ) ac (1  cos )


10. If a + b + c = 1, then prove that
2 2 2 ba (1  cos ) b  (c  a )cos 
2 2 2
bc (1  cos )
ca (1  cos ) cb (1  cos ) c 2  ( a 2  b 2 )cos 

is independent of a, b, c
RESONANCE 63
TM
tan(A  P) tan(B  P ) tan(C  P)
11. Show that the value of the determinant tan(A  Q) tan(B  Q) tan(C  Q) vanishes for all values of
tan(A  R) tan(B  R) tan(C  R)

A, B, C, P, Q & R where A + B + C + P + Q + R = 0.

bc b 2  b c c2  b c
12. Prove that a 2  a c a c c2  a c = (ab + bc + ca)3.
a 2  a b b2  a b a b

(
cos x  x 2 ) ( )
sin x  x 2 (
 cos x  x 2)
13. Show that, sin( x  x )
2
cos( x  x )
2
sin( x  x )
2
= sin (2 x + 2 x 2).
sin2x 0 sin 2x 2

1 1 1
a x bx c x
P
c y  Q
1 1 1
14. If a y by where Q is t he product of t he denom inat ors, prove t hat
1 1 1
az b z c z

P = (a  b) (b  c) (c  a) (x  y) (y  z) (z  x)

15. If A1, B1, C1,................are respectively the cofactors of the elements a1, b1, c1,...........of the determinant

a1 b1 c1
 = a2 b2 c2 then prove that
a3 b3 c3

A1 B1 C1
B2 C2
(i) = a1. (ii) A2 B2 C2 = 
B3 C3
A3 B3 C3

b c  a2 c a  b2 a b  c2 a2 c2 2 a c  b2
Show that, c a  b a b  c2 b c  a 2 = 2 a b  c2
2
16. b2 a2
a b  c2 bc  a2 c a  b2 b2 2 b c  a2 c2

ap a p
17. Using consistancy of equations, prove that if bc + qr = ca + rp = ab + pq =1 then b q b q = 0.
cr c r

sin  cos  1
18. Show that : sin  cos  1 = sin () + sin () + sin ().
sin  cos  1

RESONANCE 64
TM
19. If ax² + 2 hxy + by² + 2 gx + 2 fy + c  (l1x + m 1y + n1) (l2x + m 2y + n2), then prove that

a h g
h b f
= 0.
g f c

20. Find all the values of t for which the system of equations;
(t  1) x + (3 t + 1) y + 2 t z = 0
(t  1) x + (4 t  2) y + (t + 3) z = 0
2 x + (3 t + 1) y + 3 (t  1) z = 0
has non trivial solutions and in this context find the ratios of x: y: z, when t has the smallest of these
values.

21. Let a > 0, d > 0. Find the value of determinant

1 1 1
a a (a  d ) (a  d ) (a  2 d)
1 1 1
(a  d ) ( a  d ) (a  2 d ) ( a  2 d ) (a  3 d ) . [IIT – 1996, 5]
1 1 1
(a  2 d ) (a  2 d ) ( a  3 d ) (a  3 d ) (a  4 d )

22. Let a, b, c be real numbers with a2 + b2 + c 2 = 1. Show that the equation

ax  by  c bx  ay cx  a
bx  ay  ax  by  c cy  b
= 0 represents a straight line [IIT – 2001, 6]
cx  a cy  b  ax  by  c

EXERCISE # 1 EXERCISE # 2
1. D 2. D 3. B 4. B 5. D 6. B 7. B 1. (i) 0 (ii) 5(3 2  5 3 ) 2. x =  2 b/a

8. A 9. B 10. B 11. B 12. B 13. A 14. A


6. a =  2
15. A 16. AB 17. AB 18. BD 19. ABD 20. ABD
7. (a)   3 (b)  = 3,  = 10 (c)  = 3,   10
21. BD
1 4
8. for c = 0, x =  3, y = 3; for c =  10, x =  ,y=
2 3

20. t = 0 or 3; x: y: z = 1: 1: 1

4 d4
21.
a (a  d) 2 (a  2 d) 3 (a  3 d) 2 (a  4 d)

RESONANCE 65
TM
Matrices

Any rectangular arrangement of numbers (real or complex)


(or of real valued or complex valued expressions) is called a
matrix. If a matrix has m rows and n columns then the order of
matrix is said to be m by n (denoted as m × n).
... there is what may The general m × n matrix is
perhaps be called the
 a11 a12 a13 ...... a1j ..... a1n 
 
method of optimism  a 21 a 22 a 23 ...... a 2 j ..... a 2n 
 ..... ..... ..... ..... ..... ..... ..... 
which leads us either A=  
 a i1 a i2 a i3 ...... a ij ...... a in 
wilfullyorinstinctively  ..... ..... ..... ..... ..... ..... ..... 
 
to shut our eyes to a m1 am2 a m3 ..... a mj ..... a mn 
thepossibilityofevil. where aij denote the element of ith row & j th column. The above
Thus the optimist who matrix is usually denoted as [aij]m × n .
Note :
treats a problem in (i) The elements a11, a22, a33,........ are called as diagonal
elements. Their sum is called as trace of A denoted
algebra or analytic as T r(A)
geometry will say, if
(ii) Capital letters of English alphabets are used to denote
he stops to reflect on a matrix.

what he is doing: “I 1. Basic Definitions


know that I have no (i) Row matrix : A matrix having only one row is called as
right to divide by row matrix (or row vector).
General form of row matrix is A = [a11, a12, a13, ...., a1n]
zero; but there are so
(ii) Column matrix : A matrix having only one column is
many other values called as column matrix. (or column vector)
which the expression  a11 
 
by which I am  a 21 
Column matrix is in the form A = 
... 
dividing might have  
a m1 
that I will assume

that the Evil One has (iii) Square matrix : A matrix in which number of rows &
columns are equal is called a square matrix. General
not thrown a zero in form of a square matrix is
my denominator this
 a11 a12 ....... a1n 
 
time.”  a 21 a 22 ........ a 2n 
A= 
....... ....... ....... .......
........Maxi me Bôcher  
 an1 a n2 ....... a nn 
which we denote as A = [aij]n.

RESONANCE 26
TM
(iv) Zero matrix : A = [aij]m × n is called a zero matrix, if aij = 0  i & j.

(v) Upper triangular matrix :


A = [aij]m × n is said to be upper triangular, if aij = 0 for i > j (i.e., all the elements below the
diagonal elements are zero).

(vi) Lower triangular matrix : A = [aij]m × n is said to be a lower triangular matrix, if aij = 0 for
i < j. (i.e., all the elements above the diagonal elements are zero.)

(vii) Diagonal matrix : A square matrix [aij]n is said to be a diagonal matrix if aij = 0 for i  j.
(i.e., all the elements of the square matrix other than diagonal elements are zero)
Note : Diagonal matrix of order n is denoted as Diag (a11, a22, ......ann).

(viii) Scalar matrix :Scalar matrix is a diagonal matrix in which all the diagonal elements are same
A = [aij]n is a scalar matrix, if (i) aij = 0 for i  j and (ii) aij = k for i = j.

(ix) Unit matrix (Identity matrix) :


Unit matrix is a diagonal matrix in which all the diagonal elements are unity. Unit matrix of
order 'n' is denoted by n (or ).
i.e. A = [aij]n is a unit matrix when aij = 0 for i  j & aii = 1

1 0 0
 1 0  
eg. 2 =   , 3 = 0 1 0  .
0 1 0 0 1

(x) Comparable matrices : Two matrices A & B are said to be comparable, if they have
the same order (i.e., number of rows of A & B are same and also the number of columns).

(xi) Equality of matrices : Two matrices A and B are said to be equal if they are comparable and
all the corresponding elements are equal.
Let A = [aij] m × n & B = [bij]p × q
A = B iff (i) m = p, n = q
(ii) aij = bij  i & j.

(xii) Multiplication of matrix by scalar :


Let  be a scalar (real or complex number) & A = [aij]m × n be a matrix. Thus the product A is
defined as
A = [bij]m × n where bij = aij  i & j.
Note : If A is a scalar matrix, then A = , where  is the diagonal element.

(xiii) Addition of matrices : Let A and B be two matrices of same order (i.e. comparable matrices).
Then A + B is defined to be.
A + B = [aij]m × n + [bij]m × n.
= [c ij]m × n where c ij = aij + bij  i & j.

(xiv) Substraction of matrices : Let A & B be two matrices of same order. Then A – B is defined as
A + – B where – B is (– 1) B.

(xv) Properties of addition & scalar multiplication :


Consider all matrices of order m × n, whose elements are from a set F
(F denote Q, R or C).
Let Mm × n (F) denote the set of all such matrices.
Then
(a) A  Mm × n (F) & B  Mm × n (F)  A + B  Mm × n(F)
(b) A+B=B+A
(c) (A + B) + C = A + (B + C)

RESONANCE 27
TM
(d) O = [o]m × n is the additive identity.
(e) For every A  Mm × n(F), – A is the additive inverse.
(f)  (A + B) = A + B
(g) A = A
(h) ( 1 +  2) A =  1A +  2A

(xvi) Multiplication of matrices : Let A and B be two matrices such that the number of columns of
A is same as number of rows of B. i.e., A = [aij]m × p & B = [bij]p × n.

a
p
Then AB = [c ij]m × n where c ij = ik b kj , which is the dot product of ith row vector of A and j th
k 1

column vector of B.

Note - 1: The product AB is defined iff number of columns of A equals number of rows of B. A is
called as premultiplier & B is called as post multiplier. AB is defined 
/ BA is defined.
Note - 2 : In general AB  BA, even when both the products are defined.
Note - 3 : A (BC) = (AB) C, whenever it is defined.
(xvii) Properties of matrix multiplication :
Consider all square matrices of order 'n'. Let Mn (F) denote the set of all square matrices of
order n. (where F is Q, R or C). Then
(a) A, B  Mn (F)  AB  Mn (F)
(b) In general AB  BA
(c) (AB) C = A(BC)
(d) n, the identity matrix of order n, is the multiplicative identity.
An = A = n A  A  Mn (F)
(e) For every non singular matrix A (i.e., |A|  0) of Mn (F) there exist a unique (particular)
matrix B  Mn (F) so that AB = n = BA. In this case we say that A & B are multiplicative
inverse of one another. In notations, we write B = A–1 or A = B–1.
(f) If  is a scalar (A) B = (AB) = A(B).
(g) A(B + C) = AB + AC  A, B, C  Mn (F)
(h) (A + B) C = AC + BC  A, B, C  Mn (F).

Note :
(i) Let A = [aij]m × n. Then An = A & m A = A, where n & m are identity matrices of order
n & m respectively.
(ii) For a square matrix A, A2 denotes AA, A3 denotes AAA etc.
Solved Example # 1

 sin  1/ 2  1/ 2 sin  


   
Let A =  1/ 2 cos   & B = cos  cos  . Find  so that A = B.
 cos  tan   cos   1 
   

Solution.

By definition A & B are equal if they have the same order and all the corresponding elements are equal.
1 1
Thus we have sin  = , cos = – & tan  = – 1
2 2


  = (2n + 1) – .
4

RESONANCE 28
TM
Solved Example # 2
f(x) is a quadratic expression such that

a 2 a 1  f (0)  2a  1
 2     
b b 1  f (1)  = 2b  1 for three unequal numbers a, b, c. Find f(x).
c 2 c 1 f ( 1) 2c  1
 
Solution.
The given matrix equation implies

a 2 f (0)  af (1)  f ( 1) 2a  1


 2   
b f (0)  bf (1)  f ( 1) = 2b  1
c 2 f (0)  cf (1)  f ( 1) 2c  1
 
 x 2 f(0) + xf(1) + f(–1) = 2x + 1 for three unequal numbers a, b, c .....(i)
 (i) is an identity
 f(0) = 0, f(1) = 2 & f(– 1) = 1
 f(x) = x (ax + b)
2 = a + b & – 1 = – a + b.
1 3 3 2 1
 b= &a=  f(x) = x + x.
2 2 2 2

Self Practice Problems :

cos   sin 
1. If A() =   , varify that A() A() = A( + ).
 sin  cos  
Hence show that in this case A(). A() = A() . A().

4 6  1  2 4
   
2. Let A = 3 0 2  , B =  0 1 and C = [3 1 2].
 1  2 5   1 2

Then which of the products ABC, ACB, BAC, BCA, CAB, CBA are defined. Calculate the product
whichever is defined.
Ans. only CAB is defined. CAB = [25 100]

2. Transpose of a Matrix
Let A =[aij]m × n. Then the transpose of A is denoted by A( or AT) and is defined as
A = [bij]n × m where bij = aji  i & j.
i.e. A is obtained by rewriting all the rows of A as columns (or by rewriting all the columns of A as
rows).
(i) For any matrix A = [aij]m × n, (A) = A

(ii) Let  be a scalar & A be a matrix. Then (A) = A

(iii) (A + B) = A + B & (A – B) = A – B for two comparable matrices A and B.

(iv) (A1 ± A2 ± ..... ± An) = A1 ± A2 ± ..... ± An, where Ai are comparable.

(v) Let A = [aij]m × p & B = [bij]p × n , then (AB) = BA

(vi) (A1 A2 .......An)= An. An – 1 ...........A2 . A1, provided the product is defined.

(vii) Symmetric & skew symmetric matrix : A square matrix A is said to be symmetric if A = A
i.e. Let A = [aij]n. A is symmetric iff aij = aji  i & j.
RESONANCE 29
TM
A square matrix A is said to be skew symmetric if A = – A
i.e. Let A = [aij]n. A is skew symmetric iff aij = – aji  i & j.

a h g
 
e.g. A = h b f  is a symmetric matrix.
g f c 

o x y
 
B =  x o z  is a skew symmetric matrix.
 y  z 0
Note-1 In a skew symmetric matrix all the diagonal elements are zero.
( aii = – aii  aii = 0)
Note-2 For any square matrix A, A + A is symmetric & A – A is skew symmetric.
Note- 3 Every square matrix can be uniqualy expressed as sum of two square matrices of
which one is symmetric and other is skew symmetric.
1 1
A = B + C, where B = (A + A) & C = (A – A).
2 2
Solved Example # 3
Show that BAB is symmetric or skew symmetric according as A is symmetric or skew symmetric
(where B is any square matrix whose order is same as that of A).
Solution.
Case -  A is symmetric  A = A
(BAB) = (B)AB = BAB  BAB is symmetric.
Case -  A is skew symmetric  A = – A
(BAB) = (B)AB
= B ( – A) B
= – (BAB)
 BAB is skew symmetric
Self Practice Problems :
1. For any square matrix A, show that AA & AA are symmetric matrices.

2. If A & B are symmetric matrices of same order, than show that AB + BA is symmetric and AB – BA is
skew symmetric.

3. Submatrix, Minors, Cofactors & Determinant of a Matrix


(i) Submatrix : Let A be a given matrix. The matrix obtained by deleting some rows or columns of
A is called as submatrix of A.
a b c d 
 
eg. A = x y z w 
p q r s 

a c  a b c 
  a b d  
Then  x z , p q s  ,  x y z are all submatrices of A.
p r    p q r 

(ii) Determinant of a square matrix :


Let A = [a]1×1 be a 1×1 matrix. Determinant A is defined as |A| = a.
e.g. A = [– 3]1×1 |A| = – 3

a b
Let A =   , then |A| is defined as ad – bc.
 c d
 5 3
e.g. A=   , |A| = 23
 1 4 

RESONANCE 30
TM
(iii) Minors & Cofactors :
Let A = [aij]n be a square matrix. Then minor of element aij, denoted by Mij is defined as the
determinant of the submatrix obtained by deleting ith row & j th column of A. Cofactor of element
aij, denoted by Cij (or Aij) is defined as Cij = (– 1)i + j Mij.

a b
e.g. 1 A=  
 c d
M11 = d = C11
M12 = c, C12 = – c
M21 = b, C21 = – b
M22 = a = C22

a b c 
 
e.g. 2 A = p q r 
 x y z

q r
M11 = = qz – yr = C111.
y z

a b
M23 = = ay – bx, C23 = – (ay – bx) = bx – ay etc.
x y

(iv) Determinant of any order :


Let A = [aij]n be a square matrix (n > 1). Determinant of A is defined as the sum of products of
elements of any one row (or any one column) with corresponding cofactors.

a11 a12 a13 


 
e.g.1 A = a 21 a 22 a 23 
a 31 a 32 a 33 

|A| = a11C11 + a12 C12 + a13C13 (using first row).

a 22 a 23 a 21 a 23 a 21 a 22
= a11 – a12 + a13
a 32 a 33 a 31 a 33 a 31 a 32
|A| = a12 C12 + a22 C22 + a32C32 (using second column).

a 21 a 23 a11 a13 a11 a13


= – a12 + a22 – a32 .
a 31 a 33 a 31 a 33 a 21 a 23

(v) Some properties of determinant


(a) |A| = |A| for any square matrix A.
(b) If two rows are identical (or two columns are identical) then |A| = 0.
(c) Let  be a scalar. Than  |A| is obtained by multiplying any one row (or any one column)
of |A| by 
Note : |A| =  n |A|, when A = [aij]n.
(d) Let A = [aij]n. The sum of the products of elements of any row with corresponding
cofactors of any other row is zero. (Similarly the sum of the products of elements of
any column with corresponding cofactors of any other column is zero).
(e) If A and B are two square matrices of same order, then |AB| = |A| |B|.
Note : As |A| = |A|, we have |A| |B| = |AB| (row - row method)
|A| |B| = |AB| (column - column method)
|A| |B| = |AB| (column - row method)

RESONANCE 31
TM
(vi) Singular & non singular matrix : A square matrix A is said to be singular or non singular
according as |A| is zero or non zero respectively.
(vii) Cofactor matrix & adjoint matrix :Let A = [aij]n be a square matrix. The matrix obtained by
replacing each element of A by corresponding cofactor is called as cofactor matrix of A, denoted
as cofactor A. The transpose of cofactor matrix of A is called as adjoint of A, denoted as adj A.
i.e. if A = [aij]n
then cofactor A = [cij]n when cij is the cofactor of aij  i & j.
Adj A = [dij]n where dij = c ji  i & j.
(viii) Properties of cofactor A and adj A:
(a) A . adj A = |A| n = (adj A) A where A = [aij]n.
(b) |adj A| = |A|n – 1, where n is order of A.
In particular, for 3 × 3 matrix, |adj A| = |A|2
(c) If A is a symmetric matrix, then adj A are also symmetric matrices.
(d) If A is singular, then adj A is also singular.
(ix) Inverse of a matrix (reciprocal matrix) : Let A be a non singular matrix. Then the matrix
1
adj A is the multiplicative inverse of A (we call it inverse of A) and is denoted by A–1.
|A|
We have A (adj A) = |A| n = (adj A) A

 1   1 
 A  adj A  = n =  adj A  A, for A is non singular
 | A |   | A | 
1
 A–1 = adj A.
|A|
Remarks :

1. The necessary and sufficient condition for existence of inverse of A is that A is non singular.

2. A–1 is always non singular.

3. If A = dia (a11, a12, ....., ann) where aii  0  i, then A–1 = diag (a11– 1, a22–1, ...., ann–1).

4. (A–1) = (A )–1 for any non singular matrix A. Also adj (A ) = (adj A) .

5. (A–1)–1 = A if A is non singular.


1 –1
6. Let k be a non zero scalar & A be a non singular matrix. Then (kA)–1 = A .
k
1
7. |A–1| = | A | for |A|  0.
8. Let A be a nonsingular matrix. Then AB = AC  B = C & BA = CA  B= C.

9. A is non-singular and symmetric  A–1 is symmetric.

10. In general AB = 0 does not imply A = 0 or B = 0. But if A is non singular and AB = 0, then B = 0.
Similarly B is non singular and AB = 0  A = 0. Therefore, AB = 0  either both are singular or one of
them is 0.

Solved Example # 4
For a 3×3 skew symmetric matrix A, show that adj A is a symmetric matrix.
Solution.

0 a b  c2  bc ca 
   
A=   a 0 c cof A =  bc b 2  ab 
  b  c 0  ca  ab a 2 
 

RESONANCE 32
TM
 c2  bc ca 
 
adj A = (cof A) =  bc b 2  ab  which is symmetric.
 ca  ab a 2 
 

Solved Example # 5
For two nonsingular matrices A & B, show that adj (AB) = (adj B) (adj A)
Solution.
We have (AB) (adj (AB)) = |AB| n
= |A| |B| n
A (AB)(adj (AB)) = |A| |B| A–1
–1

1
 B adj (AB) = |B| adj A ( A–1 = | A | adj A)

 B–1 B adj (AB) = |B| B–1 adj A


 adj (AB) = (adjB) (adj A)

Self Practice Problems :

1. If A is nonsingular, show that adj (adj A) = |A|n – 2 A.

2. Prove that adj (A–1) = (adj A)–1.

( n 1)
3. For any square matrix A, show that |adj (adj A) | = | A | .
2

4. If A and B are nonsingular matrices, show that (AB)–1 = B–1 A–1.

4. System of Linear Equations & Matrices


Consider the system
a11 x 1 + a12x 2 + .......... + a1nx n = b1
a21x 1 + a22 x2 + ..........+ a2n xn = b2
.................................................
am1x 1 + am2x 2 + ..........+ amnx n = bn.

 b1 
 a11 a12 .......... a1n   x1   

a 21 a 22 .......... a 2n 
  
x b 2 
A =   2
Let ,X= & B =  ...  .
..... ..... .......... .....  ....   
     ... 
a m1 a m2 .......... a mn  xn  b 
 n

Then the above system can be expressed in the matrix form as AX = B.


The system is said to be consistent if it has atleast one solution.
(i) System of linear equations and matrix inverse:
If the above system consist of n equations in n unknowns, then we have AX = B where A is a
square matrix. If A is nonsingular, solution is given by X = A–1B.
If A is singular, (adj A) B = 0 and all the columns of A are not proportional, then the system has
infinite many solution.
If A is singular and (adj A) B  0, then the system has no solution
(we say it is inconsistent).
(ii) Homogeneous system and matrix inverse:
If the above system is homogeneous, n equations in n unknowns, then in the matrix form it is

RESONANCE 33
TM
AX = O. ( in this case b1 = b2 = ....... bn = 0), where A is a square matrix.
If A is nonsingular, the system has only the trivial solution (zero solution) X = 0
If A is singular, then the system has infinitely many solutions (including the trivial solution) and
hence it has non trivial solutions.
(iii) Rank of a matrix :
Let A = [aij]m×n. A natural number  is said to be the rank of A if A has a nonsingular submatrix
of order  and it has no nonsingular submatrix of order more than . Rank of zero matrix is
regarded to be zero.

3  1 2 5
 
eg. A = 0 0 2 0
0 0 5 0

3 2
we have   as a non singular submatrix.
0 2

The square matrices of order 3 are

3  1 2 3  1 5 3 2 5  1 2 5
       
0 0 2 , 0 0 0 , 0 2 0 ,  0 2 0
0 0 5 0 0 0 0 5 0  0 5 0
   
and all these are singular. Hence rank of A is 2.
(iv) Elementary row transformation of matrix :
The following operations on a matrix are called as elementary row transformations.
(a) Interchanging two rows.
(b) Multiplications of all the elements of row by a nonzero scalar.
(c) Addition of constant multiple of a row to another row.

Note : Similar to above we have elementary column transformations also.

Remark :

1. Elementary transformation on a matrix does not affect its rank.


2. Two matrices A & B are said to be equivalent if one is obtained from other using elementary
transformations. We write A  B.
(v) Echelon form of a matrix : A matric is said to be in Echelon form if it satisfy the followings:
(a) The first non-zero element in each row is 1 & all the other elements in the corresponding
column (i.e. the column where 1 appears) are zeroes.
(b) The number of zeroes before the first non zero element in any non zero row is less
than the number of such zeroes in succeeding non zero rows.
Result : Rank of a matrix in Echelon form is the number of non zero rows (i.e. number of rows with
atleast one non zero element.)
Remark :
1. To find the rank of a given matrix we may reduce it to Echelon form using elementary row transformations
and then count the number of non zero rows.
(vi) System of linear equations & rank of matrix:
Let the system be AX = B where A is an m × n matrix, X is the n-column vector & B is the
m-column vector. Let [AB] denote the augmented matrix (i.e. matrix obtained by accepting
elements of B as n + 1th column & first n columns are that of A).
(A) denote rank of A and ([AB]) denote rank of the augmented matrix.
Clearly (A) ([AB]).
(a) If (A) < ([AB]) then the system has no solution (i.e. system is inconsistent).
(b) If (A) = ([AB]) = number of unknowns, then the system has unique solution.
(and hence is consistent)

RESONANCE 34
TM
(c) If (A) = ([AB]) < number of unknowns, then the systems has infinitely many solutions
(and so is consistent).
(vii) Homogeneous system & rank of matrix :
Let the homogenous system be AX = 0, m equations in 'n' unknowns. In this case B = 0 and so
(A) = ([AB]).
Hence if (A) = n, then the system has only the trivial solution. If (A) < n, then the system has
infinitely many solutions.

Solved Example # 6

xyz6
Solve the system x  y  z  2 using matrix inverse.
2x  y  z  1

Solution.

1 1 1  x 6
     
Let A = 1  1 1  , X =  y  & B = 2 .
2 1  1  z   1

Then the system is AX = B.


|A| = 6. Hence A is non singular.

0 3 3 
 
Cofactor A = 2  3 1 
2 0  2

0 2 2
 
adj A = 3  3 0 
3 1  2

0 2 2  0 1/ 3 1/ 3 
1 1    
A–1 = | A | adj A = 3  3 0  =  1 / 2  1 / 2 0 
6 
3 1  2 1/ 2 1/ 6  1/ 3

 0 1/ 3 1/ 3  6 x 1


       
X = A–1 B = 1/ 2  1/ 2 0 
 2 i.e.  y  = 2
1/ 2 1/ 6  1/ 3  1  z  3

 x = 1, y = 2, z = 3.
Solved Example # 7
x – y  2z  1
xy z3
Test the consistancy of the system . Also find the solution, if any..
x – 3y  3z  – 1
2x  4y  z  8.
Solution.

1  1 2 1
  x  
1 1 1   3
A =  X =  y , B =  1
1 3 3
   z   
2 4 1 8

RESONANCE 35
TM
1  1 2 1
 
1 1 1 3
[AB] = 
1  3 3  1
 
2 4 1 8

1  1 2 1
  R 2  R 2  R1
0 2  1 2
   R 3  R 3  R1
0  2 1  2
  R 4  R 4  2R1
0 6  3 6 

1
1  1 2 1 R 2  R 2
  2
0 1  1/ 2 1 1
  R3   R3
0 1  1/ 2 1 2
  1
0 1  1/ 2 1 R 4  R 4
6

1 0 3/2 2
  R1  R1  R 2
0 1  1/ 2 1
  R3  R3  R2
0 0 0 0
  R4  R4  R2
0 0 0 0

This is in Echelon form.


(AB) = 2 = (A) < number of unknowns
Hence there are infinitely many solutions n –  = 1.
Hence we can take one of the variables any value and the rest in terms of it.
Let z = r, where r is any number.
Then x – y = 1 – 2r
x+y=3–r
4  3r 2r
 x= &y=
2 2
 4  3r 2  r 
 Solutions are (x, y, z) =  , ,r  .
 2 2 
Self Practice Problems:

0 1 2
 
1. A =  1 2 3  . Find the inverse of A using |A| and adj A. Also find A–1 by solving a system of equations.
3 1 1

2. Find real values of  and µ so that the following systems has


(i) unique solution (ii) infinite solution (iii) No solution.
x+y+z=6
x + 2y + 3z = 1
x + 2y + z = µ
Ans. (i)  3, µ  R (ii)  = 3, µ = 1 (iii)  = 3, µ  1

3. Find  so that the following homogeneous system have a non zero solution
x + 2y + 3z = x
3x + y + 2z = y
2x + 3y + z = z
Ans. =6

RESONANCE 36
TM
5. More on Matrices
(i) Characteristic polynomial & Characteristic equation :
Let A be a square matrix. Then the polynomial | A – x| is called as characteristic polynomial of
A & the equation | A – x| = 0 is called as characteristic equation of A.
Remark :
Every square matrix A satisfy its characteristic equation (Cayley - Hamilton Theorem).
i.e. a0 x n + a, xn – 1 + ........ + an – 1x + an = 0 is the characteristic equation of A, then
a0An + a1An – 1 + ......... + an – 1 A + an  = 0
(ii) More Definitions on Matrices :
(a) Nilpotent matrix:
A square matrix A is said to be nilpotent ( of order 2) if, A2 = O.
A square matrix is said to be nilpotent of order p, if p is the least positive integer such
that Ap = O.
(b) Idempotent matrix:
A square matrix A is said to be idempotent if, A2 = A.
 1 0
e.g.   is an idempotent matrix.
0 1
(c) Involutory matrix:
A square matrix A is said to be involutory if A2 = , being the identity matrix.
 1 0
e.g. A =   is an involutory matrix.
0 1
(d) Orthogonal matrix:
A square matrix A is said to be an orthogonal matrix if,
A A =  = AA.
(e) Unitary matrix:
A square matrix A is said to be unitary if A( A ) = , where A is the complex conjugate
of A.

Solved Example # 8
1 2 0 
 
If A = 2  1 0  , show that 5A–1 = A2 + A – 5.
0 0  1
Solution.
We have the characteristic equation of A.
| A – x| = 0
1  x 2 0 
 
i.e.  2  1 x 0  =0
 0 0  1  x 
i.e. x 3 + x2 – 5x – 5 = 0.
Using cayley - Hamilton theorem.
A3 + A2 – 5A – 5 = 0
 5 = A3 + A2 – 5A
Multiplying by A–1, we get
5A–1 = A2 + A – 5

Solved Example # 9

Show that a square matrix A is involutory, iff ( – A) ( + A) = 0

Solution.
Let A be involutory
Then A2 = 

RESONANCE 37
TM
( – A) ( + A) =  + A – A – A2
=  + A – A – A2
=  – A2
=0
Conversly, let ( – A) ( + A) = 0
 + A – A – A2 = 0
  + A – A – A2 = 0
  – A2 = 0
 A is involutory

Self Practice Problems

1. If A is idempotent, show that B =  – A is idempotent and that AB = BA = 0.

2. If A is a nilpotent matrix of index 2, show that A ( + A)n = A for all n  N.

3. A is a skew symmetric matrix, such that A2 +  = 0. Show that A is orthogonal and is of even order.

0 c  b
 
4. Let A =   c 0 a  . If A3 + A = 0, find .
 b  a 0 
Ans. a 2 + b 2 + c 2.

RESONANCE 38
TM
Part : (A) Only one correct option

1. Let a, b, c, d, u, v be integers. If the system of equations ax + by = u, cx + dy = v has a unique solution in


integers, then
(A) ad – bc = 1 (B) ad – bc = – 1
(C) ad – bc  0 (D) ad – bc need not be equal to ± 1

2. If AB = O for the matrices

 cos 2  cos  sin   cos 2  cos  sin 


A=   and B =   then  –  is
cos  sin  sin2   cos  sin  sin2  


(A) an odd multiple of (B) an odd multiple of 
2


(C) an even multiple of (D) 0
2

3  4
3. If X =   , then value of Xn is
 1  1

3n  4n 2  n 5  n  3n ( 4)n 


(A)   (B)   (C)  n  (D) none of these
n n   n n  1 ( 1)n 

 0 2  
 
   
4. If the matrix  is orthogonal, then
    

1 1 1
(A)  = ± (B)  = ± (C)  = ± (D) all of these
2 6 3

5. If A, B are two n × n non-singular matrices, then


(A) AB is non-singular (B) AB is singular
(C) (AB)–1 = A–1 B–1 (D) (AB)–1 does not exist

6. If B is a non-singular matrix and A is a square matrix, then det (B–1 AB) is equal to
(A) det (A–1) (B) det (B–1) (C) det (A) (D) det (B)

7. If A is a square matrix of order n × n and k is a scalar, then adj (kA) is equal to


(A) k adj A (B) kn adj A (C) kn – 1 adj A (D) kn + 1 adj A

8. Let A be a matrix of rank r. Then


(A) rank (AT) = r (B) rank (AT) < r (C) rank (AT) > r (D) none of these

9. If A = dig (2, 1, 3), B = dig (1, 3, 2), then A2B =


(A) dig (5, 4, 11) (B) dig (4, 3, 18) (C) dig (3, 1, 8) (D) B

RESONANCE 39
TM
1 1 1
 
10. If  is a cube root of unity and A = 1  2  , then A1 =
1 2  
1  2  1 1 1 1  2  1  2 
 2  1     
 
1
(A)  1  (B) 1  (C) 1   1  
2 2 2
(D)
 3 2
2 1 1  2  1 1 1 1 1 1
     

11. If the system of equations ax + y + z = 0,. x + by + z = 0 and x + y + cz = 0, where

1 1 1
a, b, c  1, has a nontrivial solution, then the value of + + is:
1 a 1 b 1 c
(A) 1 (B) 0 (C) 1 (D) None of these

12. If A is a square matrix of order 3, then the true statement is (where I is unit matrix).
(A) det ( A) =  det A (B) det A = 0
(C) det (A + I) = 1 + det A (D) det 2A = 2 det A

13. Which of the following is incorrect


(A) A2  B2 = (A + B) (A  B) (B) (AT)T = A
(C) (AB)n = AnBn, where A, B commute (D) (A  I) ( I + A) = O  A2 = I

14. The value of a for which system of equations, a3x + (a + 1) 3y + (a + 2)3z = 0,


ax + (a + 1) y + (a + 2) z = 0, x + y + z = 0, has a nonzero solution is:
(A) 1 (B) 0 (C) 1 (D) none of these

3 0 0   a1 a 2 a3 
   
0 3 0  and B =  b1 b 2 b3 
15. If A  then AB is equal to
 0 0 3   c 1 c 2 c 3 

(A) B (B) 3B (C) B3 (D) A + B

a b
16. If A =   satisfies the equation x2 – (a + d) x + k = 0, then
 c d
(A) k = bc (B) k = ad
(C) k = a2 + b2 + c 2 + d2 (D) ad – bc

 x x x 
 
x x x 
17. Let A =  , then A–1 exists if
 x x x   
(A) x  0 (B)  0 (C) 3x +   0,   0 (D) x  0,  0

18. Identity the correct statement


(A) If system of n simultaneous linear equations has a unique solution, then coefficient matrix is
singular
(B) If system of n simultaneous linear equations has a unique solution, then coefficient matrix is
non singular
(C) If A–1 exists, (adj A)–1 may or may not exist

 cos x  sin x 0 
 
sin x cos x 0 
(D) F(x) =  , then F(x) . F(y) = F(x – y)
 0 0 0 

RESONANCE 40
TM
x 3 2
 
1 y 4
19. Matrix A =  , if x y z = 60 and 8x + 4y + 3z = 20, then A(adj A) is equal to
 2 2 z 

 64 0 0   88 0 0   68 0 0   34 0 0 
       
0 64 0 0 88 0 0 68 0 0 34 0
(A)   (B)   (C)   (D)  
 0 0 64   0 0 88   0 0 68   0 0 34 

 3 1 
 
 2 2  1 1
20. If P =  1 3,A=   and Q = PAP
AP1 and x = P1Q2005P, then x is equal to [IIT JEE - 2005 ]
 2 0 1
2 

1 2005  4  2005 3 6015 


(A)   (B)  
0 1   2005 4  2005 3 

1 2  3 1  1  2005 2  3
(C)   (D)  
4   1 2  3  4 2  3 2005 

Comprehension [IIT JEE - 2006 ]

1 0 0  1 2 
     
2 1 0  0  , AU2 =  3
A=   , if U1, U2, and U3 are columns matrices satisfying AU1 =  and
 3 2 1   0   0 

2
 
3
AU3 =   . If U is 3 × 3 matrix whose columns are U1, U2, U3 then answer the following questions
 1 

21. The value of |U| is [IIT JEE - 2006]


(A) 3 (B) –3 (C) 3/2 (D) 2

22. The sum of the elements of U–1 is [IIT JEE - 2006]


(A) –1 (B) 0 (C) 1 (D) 3

3
 
2
23. The value of [3 2 0] U   is [IIT JEE - 2006]
 0 

(A) 5 (B) 5/2 (C) 4 (D) 3/2

Part : (B) May have more than one options correct

 1 2 5 
 
2  4 a  4
24. The rank of the matrix  is
 1  2 a  1

(A) 2 if a = 6 (B) 2 if a = 1 (C) 1 if a = 2 (D) 1 if a = – 6

RESONANCE 41
TM
25. Which of the following statement is always true
(A) Adjoint of a symmetric matrix is a symmetric matrix
(B) Adjoint of a unit matrix is unit matrix
(C) A (adj A) = (adj A) A
(D) Adjoint of a diagonal matrix is diagonal matrix

a b (a  b)
26.
 
Matrix b c ( b  c) is non invertible if
 
2 1 0 

(A)  = 1/2 (B) a, b, c are in A.P. (C) a, b, c are in G.P. (D) a, b, c are in H.P.

 1 a a2 
27. The singularity of matrix cos (p  d) x cos px cos (p  d) x  depends upon which of the following

 
sin (p  d) x sin px sin (p  d) x 
 
parameter
(A) a (B) p (C) x (D) d

28. Which of the following statement is true


(A) Every skew symmetric matrix of odd order is non singular
(B) If determinant of a square matrix is nonzero, then it non singular
(C) Rank of a matrix is equal or higher than the order of the matrix
(D) Adjoint of a singular matrix is always singular

a b
29. If A =   (where bc  0) satisfies the equations x 2 + k = 0, then
c d
(A) a + d = 0 (B) k = – |A| (C) k = |A| (D) none of these

 1 1 0 
 
0 2 1 
30. If A–1 =  , then
 0 0  1 

(A) | A | = 2 (B) A is non-singular

 1/ 2  1/ 2 0 
 
0  1 1 /2
(C) Adj. A =   (D) A is skew symmetric matrix
 0 0  1/ 2 

1 2 3  1
   
1. Find x so that  1 x 1  4 5 6  2 =0
 3 2 5   3 

2. If A and B are two square matrices such that AB = A & BA = B, prove that A & B are idempotent

 3 1
3. If f (x) = x 2  5x + 7, find f (A) where A =  .
 1 2 

RESONANCE 42
TM
4. Prove that the product of matrices

 cos 2  cos  sin   cos 2  cos  sin 


  and   is the null matrix, when  and  differ by an odd
cos  sin  sin   cos  sin  sin2  
2

multiple of /2.

cos x  sin x 0
  
5. Given F (x) =  sin x cos x 0 . If x   ,  . Then for what values of y,,
 2 2
 0 0 1

F (x + y) = F (x) F (y).
0 2y z 
 
x y  z
6. Find the values of x, y, z if the matrix A =  obeys the law At A = I.
 x  y z 

  1 2 5
 
2  3 1
7. Compute A1 for the following matrix A =  . Hence solve the system of equations;
 1 1 1

 x + 2y + 5z = 2; 2x  3y + z = 15 &  x + y + z = 3

1
 1  tan  / 2  1 tan  / 2 cos   sin 
Show that    tan  / 2 =  
1 
8.
tan  / 2 1    sin  cos  

9. Gaurav purchases 3 pens, 2 bags and 1 instrument box and pays Rs. 41. From the same shop Dheeraj
purchases 2 pens, 1 bag and 2 instrument boxes and pays Rs. 29, while Ankur purchases 2 pens, 2
bags and 2 instrument boxes and pays Rs. 44. Translate the problem into a system of equations. Solve
the system of equations by matrix method and hence find the cost of 1 pen, 1 bag and 1 instrument
box.

 1 2 2
 
2 1 2
10. If A =  , then prove that A2  4A  5I = O.
2 2 1
(a) using A1 (b) without using A–1

11. Having given equations x = c y + b z, y = a z + cx, z = bx + a y where x, y, z are not all zero, prove that
a2 + b2 + c2 + 2 abc  1 = 0.

12. Consider the system of linear equations in x, y, z:


(sin 3) x  y + z = 0
(cos 2) x + 4y + 3z = 0
2x + 7y + 7z = 0
Find the values of  for which this system has non  trivial solution.

13. Solve the following systems of linear equations by using the principle of matrix.
(i) 2x  y + 3z = 8 (ii) x+y+z=9
x + 2y + z = 4 2x + 5y + 7z = 52
3x + y  4z = 0 2x + y  z = 0

3  2 3  3 0 3  x  8  2y 
 
14. Compute A1, if A = 2 1  1 Hence solve the system of equations 2 1 0  y    1   z  .
       
4  3 2  4 0 2  z  4 3 y 

RESONANCE 43
TM
15. Find the rank of the following matrices:

1 2  1 3 0 1  3  1
 1 2 3 2    1 3 4 3  
  4 1 2 1   1 0 1 1
(i) 2 3 5 1 (ii)  (iii) 3 9 12 3 (iv) 
3 1 1 2 3 1 0 2
 1 3 4 5   1 3 4 1  
1 2 0 1 1 1 2 0 

 4 4 4   1 1 1 
   
16. Determine the product   7 1 3   1  2  2  and use it to solve the system of equations.
 5  3  1   2 1 3 

x – y + z = 4; x – 2 y – 2 z = 9; 2 x + y + 3 z = 1.

a b c
 
17. If A =  b c a  , where a, b, c are real positive numbers, a b c = 1 and AT A = 1, then find the value of
 c a b
 
a3 + b3 + c3. [IIT JEE - 2003, 2]

18. If M is 3 × 3 matrix M has its det.(M) = 1 and MMT = I. Prove that del (M – ) = 0.
[IIT JEE - 2004, 2 ]

a 1 0 a 1 1 f  a 2  x
       
19. If A  1 b d , B  0 d c  U  g, V   0  , X = y
 
1 b c   f g h h 0  z 
 
and AX = U has infinitely many solution. Prove that BX = V has no unique solution, also prove that if afd  0,
then BX = V has no solution. [IIT JEE - 2004, 4]

EXERCISE # 1  4 3 17 
1  
1. C 2. A 3. D 4. D 5. A 6. C 7. C 7. A1 =   3 4 11 & x = 2, y = 3, z = 2
7  1 1 1
8. A 9. B 10. B 11. C 12. A 13. A 14. A
9. Rs. 2, Rs. 15 & Rs. 5
15. B 16. D 17. C 18. B 19. C 20. A 21. A

22. B 23. A 24. ABD 25. ABCD 26. AB 


12.  = n, n +(1)n ;nI
27. CD 28. BD 29. AC 30. BC 6

EXERCISE # 2 13. (i) x = 2; y = 2; z = 2 (ii) x = 1; y = 3; z = 5

14. x = 1; y = 2; z = 3
9
1. – 3. f(A) = 0 5. y  R
8 15. (i) 2 (ii) 3 (iii) 2 (iv) 2

16. x = 3; y = – 2; z = – 1 17. 4
1 1 1
6. x = ± ,y=± ,z=±
2 6 3

RESONANCE 44
TM
Quadratic Equation

1. Equation v/s Identity:

T
A quadratic equation is satisfied by exactly two values of ' x '
he which may be real or imaginary. The equation,
a x 2 + b x + c = 0 is:
mathematicians have  a quadratic equation if a  0 Two Roots
been very much  a linear equation if a = 0, b  0 One Root
absorbed with finding  a contradiction if a = b = 0, c  0 No Root
the general solution of  an identity if a=b=c=0 Infinite Roots
algebraic equations, If a quadratic equation is satisfied by three distinct values of
' x ', then it is an identity.
and several of them
have tried to prove Solved Example # 1
the impossibility of it. (i) 3x 2 + 2x – 1 = 0 is a quadratic equation here a = 3.
(ii) (x + 1)2 = x2 + 2x + 1 is an identity in x.
However, if I am not
Solution.
mistaken, they have
Here highest power of x in the given relation is 2 and this
not as yet succeeded. relation is satisfied by three different values x= 0, x = 1 and
I therefore dare hope x = – 1 and hence it is an identity because a polynomial
equation of nth degree cannot have more than n distinct roots.
that the
mathematicians will 2. Relation Between Roots & Co-efficients:
receive this memoir
(i) The solutions of quadratic equation, a x 2 + b x + c = 0,
with good will, for its (a  0) is given by
purpose is to fill this  b  b2  4 a c
x=
gap in the theory of 2a
The expression, b2  4 a c  D is called discriminant
algebraic equations. of quadratic equation.

(ii) If ,  are the roots of quadratic equation,


Ni el s Abel a x 2 + b x + c = 0, a  0. Then:

b c D
(a)  +  =  (b)  = (c)  = a
a a
(iii) A quadratic equation whose roots are  & , is
(x ) (x ) = 0 i.e.
x 2  (sum of roots) x + (product of roots) = 0

RESONANCE 1
TM
Solved Example # 2
If  and  are the roots of ax2 + bx + c = 0, find the equation whose roots are +2 and +2.
Solution.
Replacing x by x – 2 in the given equation, the required equation is
a(x – 2)2 + b(x – 2) + c = 0 i.e., ax 2 – (4a – b)x + (4a – 2b + c) = 0.

Solved Example # 3
The coefficient of x in the quadratic equation x2 + px + q = 0 was taken as 17 in place of 13, its roots
were found to be – 2 and – 15. Find the roots of the original equation.
Solution.
Here q = (– 2) × (– 15) = 30, correct value of p = 13. Hence original equation is
x 2 + 13x + 30 = 0 as (x + 10) (x + 3) = 0
 roots are – 10, – 3

Self Practice Problems :

1. If ,  are the roots of the quadratic equation ax 2 + bx + c = 0 then find the quadratic equation whose
roots are
(i) 2, 2 (ii) 2 ,  2 (iii)  + 1,  + 1

1  1    
(iv) , (v) ,
1  1    

(r  1)2 b2
2. If r be the ratio of the roots of the equation ax 2 + bx + c = 0, show that = .
r ac
Ans.(1) (i) ax2 + 2bx + 4c = 0 (ii) a2x 2 + (2ac – b2) x + c2 = 0
(iii) ax2 – (2a – b) x + a + c – b = 0 (iv) (a + b + c)x 2 – 2(a – c) x + a – b + c = 0
(v) ac x 2 – (b2 – 2ac) x + ac = 0

3. Nature of Roots:
Consider the quadratic equation, a x 2 + b x + c = 0 having ,  as its roots; D  b2  4 a c

D=0 D0
Roots are equal =  =  b/2a Roots are unequal

a, b, c  R & D > 0 a, b, c  R & D < 0


Roots are real Roots are imaginary  = p + i q,  = p  i q

a, b, c  Q & a, b, c  Q &
D is a perfect square D is not a perfect square
 Roots are rational  Roots are irrational
 i.e.  = p + q ,  = p  q
a = 1, b, c   & D is a perfect square
 Roots are integral.

RESONANCE 2
TM
Solved Example # 4
For what values of m the equation (1 + m) x 2 – 2(1 + 3m)x + (1 + 8m) = 0 has equal roots.
Solution.
Given equation is (1 + m) x 2 – 2(1 + 3m)x + (1 + 8m) = 0 ........(i)
Let D be the discriminant of equation (i).
Roots of equation (i) will be equal if D = 0.
or, 4(1 + 3m)2 – 4(1 + m) (1 + 8m) = 0
or, 4(1 + 9m 2 + 6m – 1 – 9m – 8m 2) = 0
or, m 2 – 3m = 0 or, m(m – 3) = 0
 m = 0, 3.

Solved Example # 5
Find all the integral values of a for which the quadratic equation (x – a) (x – 10) + 1 = 0 has integral
roots.
Solution.
Here the equation is x 2 – (a + 10)x + 10a + 1 = 0. Since integral roots will always be rational it means
D should be a perfect square.
From (i) D = a2 – 20a + 96.
 D = (a – 10)2 – 4
 4 = (a – 10)2 – D
If D is a perfect square it means we want difference of two perfect square as 4 which is possible only
when (a – 10) 2 = 4 and D = 0.
 (a – 10) = ± 2
 a = 12, 8

Solved Example # 6
If the roots of the equation (x – a) (x – b) – k = 0 be c and d, then prove that the roots of the equation
(x – c) (x – d) + k = 0, are a and b.
Solution.
By given condition
(x – a) (x – b) – k  (x – c) (x – d)
or (x – c) (x – d) + k  (x – a) (x – b)
Above shows that the roots of (x – c) (x – d) + k = 0 are a and b.

Self Practice Problems :


3. Let 4x 2 – 4( – 2)x +  – 2 = 0 (  R) be a quadratic equation. Find the value of  for which
(i) Both roots are real and distinct.
(ii) Both roots are equal.
(iii) Both roots are imaginary
(iv) Both roots are opposite in sign.
(v) Both roots are equal in magnitude but opposite in sign.
4. Find the values of a, if ax2 – 4x + 9 = 0 has integral roots.
5. If P(x) = ax2 + bx + c, and Q(x) = – ax 2 + dx + c, ac  0 then prove that P(x) . Q(x) = 0 has atleast two
real roots.

Ans. (1) (i) (– , 2)  (3, ) (ii)  {2, 3}


(iii) (2, 3) (iv) (– , 2) (v) 
1 1
(2) a= ,–
3 4

RESONANCE 3
TM
4. Common Roots:
Consider two quadratic equations, a1 x 2 + b1 x + c 1 = 0 & a2 x 2 + b2 x + c 2 = 0.
(i) If two quadratic equations have both roots common, then the equation are identical and their
co-efficient are in proportion. i.e.

a1 b c
= 1 = 1 .
a2 b2 c2
(ii) If only one root is common, then the common root '  ' will be:

c a  c 2 a1 b c  b2 c1
= 1 2 = 1 2
a1 b2  a 2 b1 c1 a 2  c2 a1
Hence the condition for one common root is:
2
 c1 a 2  c2 a1   c1 a 2  c2 a1 
a1   + b1   + c1 = 0
 a1 b 2  a 2 b1   a1 b2  a 2 b1 
 c1 a 2  c2 a1  = a 1 b 2  a 2 b1  b1 c 2  b 2 c1 
2

Note : If f(x) = 0 & g(x) = 0 are two polynomial equation having some common root(s) then those common
root(s) is/are also the root(s) of h(x) = a f(x) + bg (x) = 0.

Solved Example # 7
If x2 – ax + b = 0 and x2 – px + q = 0 have a root in common and the second equation has equal roots,
ap
show that b + q = .
2
Solution.
Given equations are : x2 – ax + b= 0 and x 2 – px + q = 0.
Let  be the common root. Then roots of equation (2) will be  and . Let  be the other root of equation
(1). Thus roots of equation (1) are ,  and those of equation (2) are , .
Now +=a ........ (iii)
 = b ........ (iv)
2 = p ........ (v)
2 = q ........ (vi)
L.H.S. = b + q =  +  = ( + )
2
........ (vii)
ap (  ) 2
and R.H.S. = = =  ( + ) ........ (viii)
2 2
from (7) and (8), L.H.S. = R.H.S.

Solved Example # 8
If a, b, c  R and equations ax2 + bx + c = 0 and x2 + 2x + 9 = 0 have a common root, show that
a : b : c = 1 : 2 : 9.

Solution.
Given equations are : x 2 + 2x + 9 = 0 ........(i)
and ax2 + bx + c = 0 ........(ii)
Clearly roots of equation (i) are imaginary since equation (i) and (ii) have a common root, therefore
common root must be imaginary and hence both roots will be common.
Therefore equations (i) and (ii) are identical
a b c
 = =
1 2 9
 a:b:c=1:2:9

RESONANCE 4
TM
Self Practice Problems :

6. If the equation x2 + bx + ac = 0 and x2 + cx + ab = 0 have a common root then prove that the equation
containing other roots will be given by x 2 + ax + bc = 0.

7. If the equations ax 2 + bx + c = 0 and x3 + 3x2 + 3x + 2 = 0 have two common roots then show that
a = b = c.

a b c
8. If ax 2 + 2bx + c = 0 and a1x 2 + 2b1x + c 1 = 0 have a common root and , , are in A.P. show that
a1 b1 c 1
a1, b1, c 1 are in G.P.

5. Factorisation of Quadratic Expressions:


 The condition that a quadratic expression f (x) = a x 2 + b x + c a perfect square of a linear expression,
is D b2  4 a c = 0.
 The condition that a quadratic expressionf (x, y)= ax² +2 hxy + by² +2 gx+ 2 fy + c may be resolved into
two linear factors is that;
a h g
 abc + 2 fgh  af²  bg²  ch² = 0 OR h b f = 0.
g f c
Solved Example # 9
Determine a such that x 2 – 11x + a and x 2 – 14x + 2a may have a common factor.
Solution.
Let x –  be a common factor of x2 – 11x + a and x2 – 14x + 2a.
Then x =  will satisfy the equations x2 – 11x + a = 0 and x 2 – 14x + 2a = 0.
 2 – 11 + a = 0 and 2 – 14 + 2a = 0
Solving (i) and (ii) by cross multiplication method, we get a = 24.

Solved Example # 10
Show that the expression x 2 + 2(a + b + c)x + 3(bc + ca + ab) will be a perfect square if a = b = c.
Solution.
Given quadratic expression will be a perfect square if the discriminant of its corresponding equation is
zero.
i.e. 4(a + b + c) 2 – 4.3 (bc + ca + ab) = 0
or (a + b + c)2 – 3(bc + ca + ab) = 0
1
or ((a – b)2 + (b – c)2 + (c – a)2) = 0
2
which is possible only when a = b = c.

Self Practice Problems :

9. For what values of k the expression (4 – k)x 2 + 2(k + 2)x + 8k + 1 will be a perfect square ?

10. If x –  be a factor common to a1x 2 + b1x + c and a2x 2 + b2x + c prove that (a1 – a2) = b2 – b1.

11. If 3x 2 + 2xy + 2y2 + 2ax – 4y + 1 can be resolved into two linear factors, Prove that  is a root of the
equation x2 + 4ax + 2a2 + 6 = 0.
Ans. (1) 0, 3

RESONANCE 5
TM
6. Graph of Quadratic Expression:
y = f (x) = a x 2 + b x + c

2
 D   b 
y   = a  x  
2 a 
or
 4a  

 the graph between x, y is always a parabola.

 b D 
 the coordinate of vertex are   2 a ,  4 a 
 

 If a > 0 then the shape of the parabola is concave upwards & if a < 0 then the shape of the
parabola is concave downwards.
 the parabola intersect the yaxis at point (0, c).
 the xcoordinate of point of intersection of parabola with xaxis are the real roots of the
quadratic equation f (x) = 0. Hence the parabola may or may not intersect the xaxis at real
points.

7. Range of Quadratic Expression f (x) = a x2 + b x + c.


(i) Absolute Range:

 D 
If a>0  f (x)   4 a ,  
 

 D 
a<0  f (x)     ,  4 a 
 

D
Hence maximum and minimum values of the expression f (x) is  4 a in respective cases and

b
it occurs at x =  2 a (at vertex).

(ii) Range in restricted domain:


Given x  [x 1, x 2]

b
(a) If   [x 1, x 2] then,
2a

 
f (x)  min f ( x1) , f ( x 2 ) ,  
max f ( x1 ) , f ( x 2 ) 
b
(b) If   [x 1, x 2] then,
2a

 D 
 f ( x1 ) , f ( x 2 ) ,   , max  f ( x1) , f ( x 2 ) , 
 D 
f (x)   min 
  4a  4 a  

RESONANCE 6
TM
Solved Example # 11
If c < 0 and ax2 + bx + c = 0 does not have any real roots then prove that
(i) a–b+c<0 (ii) 9a + 3b + c < 0.
Solution.
c < 0 and D < 0  f(x) = ax 2 + bx + c < 0 for all x  R
 f(– 1) = a – b + c < 0
and f(3) = 9a + 3b + c < 0

Solved Example # 12
Find the maximum and minimum values of f(x) = x2 – 5x + 6.
Solution.
D b
minimum of f(x) = – at x = –
4a 2a

 25  24  5 1
=–   at x = =–
 4  2 4
maximum of f(x) = 

 1 
Hence range is  ,   .
 4 

Solved Example # 13
x2  x  1
Find the range of rational expression y = if x is real.
x2  x  1
Solution.
x2  x  1
y=
x2  x  1
 (y – 1)x 2 + (y + 1) x + y – 1 = 0
 x is real
 D0
 (y + 1) 2 – 4(y – 1)2  0
 (y – 3) (3y – 1)  0

1 
 y   , 3 .
 3 

Solved Example # 14
x2
Find the range of y = , if x is real.
2x  3 x  6
2

Solution.
x2
y=
2x  3 x  6
2

 2yx 2 + 3yx + 6y = x + 2
 2yx 2 + (3y – 1) x + 6y – 2 = 0
 x is real
D0
 (3y – 1) 2 – 8y (6y – 2)  0
 (3y – 1) (13y + 1)  0
 1 1
y   , .
 13 3 

RESONANCE 7
TM
Self Practice Problems :

12. If c > 0 and ax 2 + 2bx + 3c = 0 does not have any real roots then prove that
(i) a – 2b + 3c > 0
(ii) a + 4b + 12c > 0

(a  b) 2
13. If f(x) = (x – a) (x – b), then show that f(x)  – .
4

14. For what least integral value of k the quadratic polynomial (k – 2) x 2 + 8x + k + 4 > 0  x  R.

x 2  34 x  71
15. Find the range in which the value of function lies  x  R.
x 2  2x  7

mx 2  3 x  4
16. Find the interval in which 'm' lies so that the function y = can take all real values
 4x 2  3x  m
 x  R.
Ans. (14) k = 5. (15) (– , 5]  [9, ) (16) m  [1, 7]

8. Sign of Quadratic Expressions:


The value of expression, f (x) = a x 2 + b x + c at x = x 0 is equal to ycoordinate of a point on parabola
y = a x 2 + b x + c whose xcoordinate is x 0. Hence if the point lies above the xaxis for some x = x0,
then f (x 0) > 0 and viceversa.
We get six different positions of the graph with respect to xaxis as shown.

NOTE:
(i)  x  R, y > 0 only if a > 0 & D  b²  4ac < 0 (figure 3).
(ii)  x  R, y < 0 only if a < 0 & D  b²  4ac < 0 (figure 6).

9. Solution of Quadratic Inequalities:


The values of ' x ' satisfying the inequality, ax2 + bx + c > 0 (a  0) are:
(i) If D > 0, i.e. the equation ax2 + bx + c = 0 has two different roots  < .
Then a > 0  x  ( )  ( )
a < 0  x  ( )
(ii) If D = 0, i.e. roots are equal, i.e.  = .
Then a > 0  x  (, )  ( )
a < 0  x 

RESONANCE 8
TM
(iii) If D < 0, i.e. the equation ax2 + bx + c = 0 has no real root.
Then a > 0  x  R
a < 0  x  
P ( x ) Q ( x ) R ( x )......... 
(iv) Inequalities of the form A ( x ) B ( x ) C ( x ).........  0 can be quickly solved using the method of

intervals, where A, B, C........, P, Q, R......... are linear functions of ' x '.

Solved Example # 15

x 2  6x  7
Solve 2
x2  1
Solution.
 x 2 + 6x – 7  2x2 + 2
 x 2 – 6x + 9  0
 (x – 3)2  0
 xR

Solved Example # 16

x2  x  1
Solve > 0.
| x  1|
Solution.
 |x + 1| > 0
 x  R – {–1}
 x2 + x + 1 > 0
 D=1–4=–3<0
 x2 + x + 1 > 0  x  R
 x  (– , – 1)  (– 1, )

Solved Example # 17

x 2  3x  1
< 3.
x2  x  1
Solution.

| x 2  3x  1 |
< 3.
x2  x  1
 in x 2 + x + 1
D=1–4=–3<0
 x2 + x + 1 > 0  x  R
 |x2 – 3x – 1| < 3(x2 + x + 1)
 (x 2 – 3x – 1)2 – {3(x2 + x + 1)} 2 < 0
 (4x2 + 2) (– 2x 2 – 6x – 4) < 0
 (2x2 + 1) (x + 2) (x + 1) > 0
 x  (– , – 2)  (– 1, )

Self Practice Problems :

17. (i) |x2 + x | – 5 < 0


(ii) x 2 – 7x + 12 < |x – 4|

2x 1
18. Solve 
x2  9 x2

RESONANCE 9
TM
19. Solve the inequation (x 2 + 3x + 1) (x2 +3x – 3)  5

x 2  x  1
20. Find the value of parameter '' for which the inequality  3 is satisfied  x  R
x2  x  1

x 2  5x  4
21. Solve 1
x2  4

  1  21   21  1  
  ,  
Ans. (17) (i)   2   2  (ii) (2, 4)
    
(18) (– , – 3)  (– 2, 3) (19) (– , – 4]  [–2, –1]  [1, )

 8 5 
(20) (–1, 5) (21) 0, 5    2 ,  
   

1 0 . Location Of Roots:
Let f (x) = ax² + bx + c, where a > 0 & a, b, c  R.

(i) (ii) (iii)

(i) Conditions for both the roots of f (x) = 0 to be greater than a specified number‘x 0’ are
b²  4ac  0; f (x 0) > 0 & ( b/2a) > x 0.
(ii) Conditions for both the roots of f (x) = 0 to be smaller than a specified number ‘x 0’ are
b²  4ac  0; f (x 0) > 0 & ( b/2a) < x 0.
(iii) Conditions for both roots of f (x) = 0 to lie on either side of the number ‘x0’ (in other words the
number ‘x 0’ lies between the roots of f (x) = 0), is f (x 0) < 0.

(iv) (v)

(iv) Conditions that both roots of f (x) = 0 to be confined between the numbers x 1 and
x 2, (x 1 < x 2) are b²  4ac  0; f (x 1) > 0 ; f (x 2) > 0 & x1 < ( b/2a) < x 2.

(v) Conditions for exactly one root of f (x) = 0 to lie in the interval (x1, x 2) i.e.
x 1 < x < x2 is f (x 1). f (x 2) < 0.

Ex.10.1 x 2 – (m – 3) x + m = 0
(a) Find values of m so that both the roots are greater than 2.

Condition -  D0  (m – 3)2 – 4m  0  m 2 – 10m + 9  0


 (m – 1) (m – 9)  0  m  (– , 1]  [9, ) ......(i)

RESONANCE 10
TM
Condition -  f(2) > 0  4 – (m – 3)2 + m > 0 m < 10...(ii),
b m3
Condition -  – >2  2  m > 7.....(iii)
2a 2
Intersection of (i), (ii) and (iii) gives m  [9, 10) Ans.

(b) Find the values of m so that both roots lie in the interval (1, 2)

Condition -  D  0  m  (– , 1]  [9, )


Condition -  f(1) > 0  1 – (m – 3) + m > 0  4>0  mR
Condition -  f(2) > 0  m < 10
b m3
Condition - V 1 < – <2  1< <2  5<m<7
2a 2
intersection gives m  Ans.
(c) One root is greater than 2 and other smaller than 1

Condition -  f(1) < 0  4<0  m  


Condition -  f(2) < 0  m > 10
Intersection gives m   Ans.

(d) Find the value of m for which both roots are positive.

Condition -  D  0  m  (– , 1] [9, )


Condition -  f(0) > 0  m>0
b m3
Condition -   >0  >0  m>3
2a 2
intersection gives m  [9, ) Ans.

(e) Find the values of m for which one root is (positive) and other is (negative).

Condition -  f(0) < 0  m < 0 Ans.

(f) Roots are equal in magnitude and opposite in sign.


sum of roots = 0  m=3
and f(0) < 0  m<0
 m   Ans.

RESONANCE 11
TM
Ex.10.2 Find all the values of 'a' for which both the roots of the equation
(a – 2)x2 + 2ax + (a + 3) = 0 lies in the interval (– 2, 1).

Sol. Case - 

When a–2>0
 a>2
Condition -  f(–2) > 0  (a – 2)4 – 4a + a + 3 > 0  a–5>0a>5
1
Condition -  f(1)> 0  4a + 1 > 0  a>–
4
Condition -  D  0  4a2 – 4(a + 3) (a – 2)  0  a6
b 2(a  1)
Condition - V – <1  >0  a  (– , 1)  (4, )
2a a2

b 2a a4
Condition - V – 2< –  2 (a  2 ) > – 2  >0
2a a2
Intersection gives a  (5, 6]. Ans.
Case- when a – 2 < 0
a<2
Condition -  f(–2) < 0  a<5
1
Condition -  f(1) < 0,  a<–
4

b
Condition -  – 2 < – <1  a  (– , 1)  (4, )
2a
Condition - V D  0  a6

 1
intersection gives a    ,  
 4

 1
complete solution is a    ,    (5, 6] Ans.
 4
Self Practice Problems :

22. Let 4x 2 – 4( – 2)x +  – 2 = 0 (  R) be a quadratic equation find the value of  for which
(a) Both the roots are positive
(b) Both the roots are negative
(c) Both the roots are opposite in sign.
(d) Both the roots are greater than 1/2.
(e) Both the roots are smaller than 1/2.
(f) One root is small than 1/2 and the other root is greater than 1/2.
Ans. (a) [3, ) (b)  (c) (– , 2)
(d)  (e) (– , 2] (f) (3, )

23. Find the values of the parameter a for which the roots of the quadratic equation
x 2 + 2(a – 1)x + a + 5 = 0 are
(i) positive (ii) negative (iii) opposite in sign.
Ans. (i) (–5, – 1] (ii) [4, ) (iii) (– , – 5)

RESONANCE 12
TM
24. Find the values of P for which both the roots of the equation
4x2 – 20px + (25p2 + 15p – 66) = 0 are less than 2.
Ans. (– , –1)

25. Find the values of  for which 6 lies between the roots of the equation x2 + 2( – 3)x + 9 = 0.

 3
Ans.   ,   .
 4
26. Let 4x 2 – 4( – 2)x +  – 2 = 0 ( R) be a quadratic equation find the value of  for which

 1
(i) Exactly one root lies in  0,  .
 2

 1
(ii) Both roots lies in  0,  .
 2

 1
(iii) At least one root lies in  0,  .
 2
(iv) One root is greater than 1/2 and other root is smaller than 0.
Ans. (i) (– , 2)  (3, ) (ii)  (iii) ( – , 2)  (3, )
(iv) 

27. In what interval must the number 'a' vary so that both roots of the equation
x 2 – 2ax + a2 – 1 = 0 lies between – 2 and 4.
Ans. (– 1, 3)

28. Find the values of k, for which the quadratic expression ax 2 + (a – 2) x – 2 is negative for exactly two
integral values of x.
Ans. [1, 2)

1 1 . Theory Of Equations:
If 1, 2, 3,......n are the roots of the equation;

f(x) = a 0 x n + a 1x n-1 + a 2x n-2 +.... + a n-1x + a n = 0 where a 0, a 1,.... a n are all real & a 0  0 then,
a1 a2 a3 an
 1 =  a ,  1 2 = + a , 1 2 3 =  a ,....., 1 2 3........n = (1)n a
0 0 0 0
NOTE :
(i) If  is a root of the equation f(x) = 0, then the polynomial f(x) is exactly divisible by (x ) or
(x ) is a factor of f(x) and conversely.
(ii) Every equation of nth degree (n 1) has exactly n roots & if the equation has more than n roots,
it is an identity.
(iii) If the coefficients of the equation f(x) = 0 are all real and  + i is its root, then   i is also a
root. i.e. imaginary roots occur in conjugate pairs.
(iv) An equation of odd degree will have odd number of real roots and an equation of even degree
will have even numbers of real roots.
(v) If the coefficients in the equation are all rational &  +  is one of its roots, then

  is also a root where ,   Q &  is not a perfect square.


(vi) If there be any two real numbers 'a' & 'b' such that f(a) & f(b) are of opposite signs, then
f(x) = 0 must have odd number of real roots (also atleast one real root) between ' a ' and ' b '.
(vii) Every equation f(x) = 0 of degree odd has atleast one real root of a sign opposite to that of its
last term. (If coefficient of highest degree term is positive).

RESONANCE 13
TM
Ex.11.1 2x3 + 3x 2 + 5x + 6 = 0 has roots , ,  then find  +  + ,  + +  and .
3 5 6
 +  +  = = – +  +  = ,  = – = – 3.
2 2 2

Ex.11.2 Find the roots of 4x 3 + 20x2 – 23x + 6 = 0. If two roots are equal.
Let roots be ,  and 
20
 ++=–
4
 2 +  = – 5 .............(i)
23
  .  +  +  = –
4

23 6
 2 + 2 = – & 2  = –
4 4
from equation (i)
23
2 + 2 (– 5 – 2) = –
4

23
 2 – 10 – 42 = –
4
 12 + 40 – 23 = 0
2

23
  = 1/2, –
6

1
when  =
2
from equation (i)
1 3
2  = (– 5 – 1) = –
4 2

23
when  = –
6

23  23   23  
2  =   5  2x      – 3
36   6  2

1
 = , =–6
2
Hence roots of equation
1 1
= , , – 6 Ans.
2 2

Self Practice Problems :

29. Find the relation between p, q and r if the roots of the cubic equation x3 – px2 + qx – r = 0 are such that
they are in A.P.
Ans. 2p3 – 9pq + 27r = 0

30. If , ,  are the roots of the cubic x3 + qx + r = 0 then find the equation whose roots are
(a)  + ,  + ,  +  Ans. x 3 + qx – r = 0
(b) , ,  Ans. x 3 – qx2 – r2 = 0
(c) ,,
2 2 2
Ans. x 3 + 2qx 2 + q2 x – r 2 = 0
(d) ,,
3 3 3
Ans. x 3 + 3x 2r + (3r2 + q3) x + r3 = 0

RESONANCE 14
TM
Part : (A) Only one correct option

1. The roots of the quadratic equation (a + b – 2c) x 2 – (2a – b – c) x + (a – 2b + c) = 0 are


1
(A) a + b + c and a – b + c (B) and a – 2b + c
2

1
(C) a – 2b + c and (D) none of these
abc

3x
2. The roots of the equation 2x + 2. 3 x 1 = 9 are given by

log 3
(A) 1 – log2 3, 2 (B) log2 (2/3), 1 (C) –2, 2 (D) –2, 1 – log 2

3. Two real numbers  &  are such that  +  = 3 &   = 4, then  &  are the roots of the quadratic
equation:
(A) 4x2  12x  7 = 0 (B) 4x2  12x + 7 = 0
(C) 4x  12x + 25 = 0
2
(D) none of these

4. Let a, b and c be real numbers such that 4a + 2b + c = 0 and ab > 0. Then the equation
ax2 + bx + c = 0 has
(A) real roots (B) imaginary roots (C) exactly one root (D) none of these

     
5. If ecosx – e– cosx = 4, then the value of cos x is
(A) log 2  5 (B) –log 2  5 (C) log  2  5 (D) none of these

6. The number of the integer solutions of x2 + 9 < (x + 3) 2 < 8x + 25 is :


(A) 1 (B) 2 (C) 3 (D) none

7. If (x + 1)2 is greater than 5x  1 & less than 7x  3 then the integral value of x is equal to
(A) 1 (B) 2 (C) 3 (D) 4

8. The set of real ' x ' satisfying, x  1  1  1 is:


(A) [0, 2] (B) [ 1, 3] (C) [ 1, 1] (D) [1, 3]

9. Let f(x) = x 2 + 4x + 1. Then


(A) f(x) > 0 for all x (B) f(x) > 1 when x  0
(C) f(x)  1 when x  – 4 (D) f(x) = f(– x) for all x

x2  x  1
10. If x is real and k = then:
x2  x  1

1
(A) k3 (B) k  5 (C) k  0 (D) none
3

x2  x  c
11. If x is real, then can take all real values if :
x2  x  2c
(A) c  [0, 6] (B) c  [ 6, 0]
(C) c  (  6)  (0, ) (D) c  ( 6, 0)

RESONANCE 15
TM
x 4  3 x 3  2x 2
12. The solution set of the inequality  0 is:
x 2  x  30
(A) ( ,  5)  (1, 2)  (6, )  {0} (B) ( ,  5)  [1, 2]  (6, )  {0}
(C) ( ,  5]  [1, 2]  [6, )  {0} (D) none of these

13. If x – y and y – 2x are two factors of the expression x3 – 3x2y + xy2 + y3, then
11 3
(A)  = 11,  = –3 (B)  = 3,  = –11 (C)  = ,=– (D) none of these
4 4

14. If  are the roots of the equation, x 2  2 m x + m 2  1 = 0 then the range of values of m for which
  ( 2, 4) is:
(A) ( 1, 3) (B) (1, 3) (C) (,  1)  ((3, ) (D) none

15. If the inequality (m  2)x2 + 8x + m + 4 > 0 is satisfied for all x  R then the least integral m is:
(A) 4 (B) 5 (C) 6 (D) none

16. For all x  R, if mx 2 – 9mx + 5m + 1 > 0, then m lies in the interval


(A) – (4/61, 0) (B) [0, 4/61) (C) (4/61, 61/4) (D) (– 61/4, 0]

17. Let a > 0, b > 0 & c > 0. Then both the roots of the equation ax2 + bx + c = 0
(A) are real & negative (B) have negative real parts
(C) are rational numbers (D) none

18. The value of 'a' for which the sum of the squares of the roots of the equation, x 2  (a  2) x  a  1 = 0
assume the least value is:
(A) 0 (B) 1 (C) 2 (D) 3

2x
19. Consider y = , then the range of expression, y2 + y  2 is:
1  x2

(A) [ 1, 1] (B) [0, 1] (C)  9 / 4 , 0  (D)  9 / 4 , 1

20. If both roots of the quadratic equation x 2 + x + p = 0 exceed p where p  R then p must lie in the
interval:
(A) ( , 1) (B) ( ,  2) (C) ( ,  2)  (0, 1/4) (D) ( 2, 1)

21. If a, b, p, q are nonzero real numbers, the two equations, 2 a 2 x 2  2 ab x + b 2 = 0 and


p2 x 2 + 2 pq x + q2 = 0 have:
(A) no common root (B) one common root if 2 a2 + b2 = p2 + q2
(C) two common roots if 3 pq = 2 ab (D) two common roots if 3 qb = 2 ap

1  1  1 
22. If , &  are the roots of the equation, x 3  x  1 = 0 then, + + has the value equal to:
1  1  1 
(A) zero (B)  1 (C)  7 (D) 1

23. The equations x 3 + 5x 2 + px + q = 0 and x3 + 7x 2 + px + r = 0 have two roots in common. If the third root
of each equation is represented by x1 and x 2 respectively, then the ordered pair (x1, x2) is:
(A) ( 5,  7) (B) (1,  1) (C) ( 1, 1) (D) (5, 7)

24. If ,  are roots of the equation ax 2 + bx + c = 0 then the equation whose roots are 2 + 3 and 3 + 2
is
(A) ab x 2 – (a + b) cx + (a + b) 2 = 0 (B) ac x2 – (a + c) bx + (a + c)2 = 0
(C) ac x + (a + c) bx – (a + c) = 0
2 2
(D) none of these

25. If coefficients of the equation ax 2 + bx + c = 0, a  0 are real and roots of the equation are non-real
complex and a + c < b, then
(A) 4a + c > 2b (B) 4a + c < 2b (C) 4a + c = 2b (D) none of these

RESONANCE 16
TM
26. The set of possible values of  for which x2 – ( 2 – 5 + 5)x + (2 2 – 3 – 4) = 0 has roots, whose sum
and product are both less than 1, is

 5  5  5
(A)   1,  (B) (1, 4) (C) 1,  (D) 1, 
 2  2  2

27. Let conditions C1 and C2 be defined as follows : C1 : b2 – 4ac  0, C2 : a, –b, c are of same sign. The
roots of ax2 + bx + c = 0 are real and positive, if
(A) both C1 and C1 are satisfied (B) only C2 is satisfied
(C) only C1 is satisfied (D) none of these

Part : (B) May have more than one options correct

28. If a, b are non-zero real numbers, and ,  the roots of x2 + ax + b = 0, then


(A) 2,  2 are the roots of x 2 – (2b – a2) x + a2 = 0

1 1
(B) , are the roots of bx 2 + ax + 1 = 0
 

 
(C) , are the roots of bx 2 + (2b – a2) x + b = 0
 
(D) –, – are the roots of x 2 + ax – b = 0

29. x 2 + x + 1 is a factor of a x 3 + b x 2 + c x + d = 0, then the real root of above equation is


(a, b, c, d  R)
(A)  d/a (B) d/a (C) (b – a)/a (D) (a – b)/a

30. If (x2 + x + 1) + (x 2 + 2x + 3) + (x 2 + 3x + 5) +...... + (x 2 + 20 x + 39) = 4500, then x is equal to:


(A) 10 (B)  10 (C) 20.5 (D)  20.5

31. cos  is a root of the equation 25x 2 + 5x  12 = 0,  1 < x < 0, then the value of sin 2 is:
(A) 24/25 (B)  12/25 (C)  24/25 (D) 20/25

32. If the quadratic equations, x 2 + abx + c = 0 and x 2 + acx + b = 0 have a common root then the equation
containing their other roots is/are:
(A) x 2 + a (b + c) x  a2bc = 0 (B) x 2  a (b + c) x + a2bc = 0
(C) a (b + c) x  (b + c) x + abc = 0
2
(D) a (b + c) x 2 + (b + c) x  abc = 0

1. Solve the equation, x (x + 1) (x + 2) (x + 3) = 120.

2. Solve the following where x  R.


(a) (x  1)x²  4x + 3+ 2 x² + 3x  5 = 0 (b) (x + 3).x + 2+2x + 3+ 1 = 0

(c) (x + 3). (x + 1) +2x + 5= 0 (d) 2x+2 2x+1  1= 2x+1 + 1

( x  1) ( x  1) ( x  4) ( x  6 )  25
3. If ' x ' is real, show that,  0.
7 x2  8 x  4

x2 2x  3
4. Find the value of x which satisfy inequality > .
x2 4x  1

5. Find the range of the expression f(x) = sin2x – sinx + 1  x  R.

6. Find the range of the quadratic expression f(x) = x 2 – 2x + 3  x  [0, 2].

RESONANCE 17
TM
7. Prove that the function y = (x² + x + 1)/(x² + 1) cannot have values greater than 3/2 and values smaller
than 1/2 for  x  R.

x 2  2x  9 1 
8. If x be real, show that lies in  , 2 .
x  2x  9
2
2 

9. For what values of k the expression 3x 2 + 2xy + y2 + 4x + y + k can be resolved into two linear factors.

10. Show that one of the roots of the equation, a x 2 + b x + c = 0 may be reciprocal of one of the roots of
a1 x 2 + b1 x + c 1 = 0 if (a a1  c c 1)2 = (b c 1  a b1) (b1c  a1b).

11. Let  + i ; ,  R, be a root of the equation x3 + qx + r = 0; q, r  R. Find a real cubic equation,
independent of  and , whose one root is 2.

12. If a, b are the roots of x 2 + px + 1 = 0 and c, d are the roots of x 2 + qx + 1 = 0. Show that
q2  p2 = (a  c) (b  c) (a + d) (b + d).

13. If ,  are the roots of the equation x² - px + q = 0, then find the quadratic equation the roots of which are
(2   2) (3   3) & 3  2 + 2  3.

x 2  kx  1
14. If ' x ' is real, find values of ' k ' for which, < 2 is valid.
x2  x  1

1 4 4 1 1
15. Solve the inequality, – + – < .
x 1 x  2 x  3 x  4 30

16. The equations x2  ax + b = 0 & x3  px 2 + qx = 0, where b  0, q  0 have one common root & the
second equation has two equal roots. Prove that 2 (q + b) = ap.

 x 
2
 x 
   
17. Find the real values of ‘m’ for which the equation,  2   (m  3)  1  x 2  + m = 0 has atleast one
 1  x   
real root ?

18. Let a and b be two roots of the equation x3 +px2 + qx + r = 0 satisfying the relation ab + 1 = 0. Prove that
r2 + pr + q + 1 = 0.

EXERCISE # 1
1  3 
1. D 2. D 3. A 4. A 5. D 6. D 7. C 4. x (– , – 2)   , 1  (4, ) 5.  4 , 3
4   
8. B 9. C 10. A 11. B 12. B 13. C 14. A
15. B 16. B 17. B 18. B 19. C 20. B 21. A 11
6. [2, 3] 9. k = 11. x 3 + qx – r = 0
8
22. C 23. A 24. D 25. B 26. D 27. A

28. BC 29. AD 30. AD 31. AC 32. BD 13. x 2  p(p4  5p2q + 5q2) x + p2q2(p2  4q) (p2  q) = 0

EXERCISE # 2 14. k  (0, 4)

1. {2,  5}
15. (– , – 2)  (– 1, 1)  (2, 3)  (4, 6)  (7, )

2. (a) x = 1 (b) x = ( 7  17 )/2


 7 5
17.  , 
(c) x =  2,  4,  (1+ 3 ) (d) x  1, x =  3  2 6
RESONANCE 18
TM
Straight Line

1. Distance Formula:
The distance between the points A(x 1 ,y1 ) and B(x 2 ,y2 ) is

x1  x 2 2  y1  y 2 2
There is no royal .

Solved Example # 1
road to geometry.
Find the value of x, if the distance between the points
........Eucl i d (x, –1) and (3, 2) is 5

Solution.
Let P(x ,–1) and Q(3, 2) be the given points. Then PQ = 5 (given)

A youth who had ( x  3 ) 2  ( 1  2 ) 2 = 5  (x – 3)2 + 9 = 25


 x = 7 or x = – 1 Ans.
begun to read
Self practice problems :
geometry with Euclid,
1. Show that four points (0, –1), (6, 7) (–2, 3) and (8, 3) are the
when he had learnt vertices of a rectangle.

2. Find the coordinates of the circumcenter of the triangle whose


thefirstproposition, vertices are (8, 6), (8, –2) and (2, –2). Also find its circumradius.
Ans. (5, 2), 5
inquired, “What do I
get by learning these 2. Section Formula :
things?” So Euclid If P(x, y) divides the line joining A(x1, y1) & B(x2, y2) in the ratio
m : n, then;
called a slave and mx 2  nx1 my 2  ny1
x= mn ;y= mn .
said “Give him
NOTE :
threepence, since he m m
(i) If is positive, the division is internal, but if is
must make a gain n n
negative, the division is external.
out of what he
(ii) If P divides AB internally in the ratio m : n & Q divides
learns.” AB externally in the ratio m : n then P & Q are said to be
harmonic conjugate of each other w.r.t. AB.
Mathematically,
2 1 1
  i.e. AP, AB & AQ are in H.P..
AB AP AQ

RESONANCE TM 19
Solved Example# 2
Find the coordinates of the point which divides the line segment joining the points (6, 3) and
(– 4, 5) in the ratio 3 : 2 (i) internally and (ii) externally.
Solution.
Let P (x, y) be the required point.

(i) For internal division :

3  4  2  6 35  23 21
x= and y = or x = 0 and y =
32 32 5

 21 
So the coordinates of P are  0,  Ans.
 5 

(ii) For external division

3  4  2  6 35  23
x= and y =
32 32
or x = –24 and y = 9
So the coordinates of P are (–24, 9) Ans.

Solved Example # 3

Find the coordinates of points which trisect the line segment joining (1, – 2) and (– 3, 4).

Solution.
Let A (1, –2) and B(–3, 4) be the given points. Let the points of trisection be P and Q. Then

AP = PQ = QB =  (say)

 PB = PQ + QB = 2 and AQ = AP + PQ = 2
 AP : PB =  : 2 = 1 : 2 and AQ : QB = 2 :  = 2 : 1
So P divides AB internally in the ratio 1 : 2 while Q divides internally in the ratio 2 : 1
 1  3  2  1 1  4  2  2   1 
 the coordinates of P are  ,  or   , 0 
 1  2 1  2   3 

 2  3  1 1 2  4  1 ( 2)   5 
and the coordinates of Q are  ,  or   , 2 
 2 1 2 1   3 

 1   5 
Hence, the points of trisection are   , 0  and   , 2  Ans.
 3   3 

Self practice problems :

3. In what ratio does the point (–1, –1) divide the line segment joining the points (4, 4) and
(7, 7)?
Ans. 5 : 8 externally

4. The three vertices of a parallelogram taken in order are (–1, 0), (3, 1) and (2, 2) respectively. Find the
coordinates of the fourth vertex.
Ans. (–2, 1)

RESONANCE TM 20
3. Centroid, Incentre & Excentre:
If A (x1, y1), B(x2, y2), C(x3, y3) are the vertices of triangle ABC, whose sides BC, CA, AB are of lengths
a, b, c respectively, then the co-ordinates of the special points of triangle ABC are as follows :
 x 1  x 2  x 3 y1  y 2  y 3 
Centroid G   , 
 3 3 

 ax 1 bx 2  cx 3 ay 1 by 2  cy 3 
Incentre I   a  b  c ,
ab c
 ,and
 

  ax1  bx 2  cx 3  ay1  by 2  cy 3 
Excentre (to A) I1   abc
,
abc
 and so on.
 
NOTE :
(i) Incentre divides the angle bisectors in the ratio, (b + c) : a; (c + a) : b & (a + b) : c.
(ii) Incentre and excentre are harmonic conjugate of each other w.r.t. the angle bisector on which
they lie.
(iii) Orthocenter, Centroid & Circumcenter are always collinear & centroid divides the line joining
orthocentre & circumcenter in the ratio 2 : 1.
(iv) In an isosceles triangle G, O,  & C lie on the same line and in an equilateral triangle, all these
four points coincide.

Solved Example # 4

Find the coordinates of (i) centroid (ii) in-centre of the triangle whose vertices are (0, 6), (8, 12)
and (8, 0).

Solution
(i) We know that the coordinates of the centroid of a triangle whose angular points are (x1, y1), (x2, y2)
(x 3, y3) are
 x 1  x 2  x 3 y1  y 2  y 3 
 , 
 3 3 
So the coordinates of the centroid of a triangle whose vertices are (0, 6), (8, 12) and (8, 0) are
 0  8  8 6  12  0   16 
 ,  or  , 6 Ans.
 3 3   3 

(ii) Let A (0, 6), B (8, 12) and C(8, ) be the vertices of triangle ABC.

Then c = AB = (0  8)2  (6  12)2 = 10, b = CA = (0  8)2  (6  0 )2 = 10

and a = BC = (8  8)2  (12  0)2 = 12.

 ax 1  bx 2  cx 3 ay 1  by 2  cy 3 
The coordinates of the in-centre are  , 
 abc abc 

 12  0  10  8  10  8 12  6  10  12  10  0 
or  , 
 12  10  10 12  10  10 

 160 192 
or  ,  or (5, 6) Ans.
 32 32 

RESONANCE TM 21
Self practice problems :

5. Two vertices of a triangle are (3, –5) and (–7, 4). If the centroid is (2, –1), find the third vertex.
Ans. (10, – 2)

6. Find the coordinates of the centre of the circle inscribed in a triangle whose vertices are
(– 36, 7), (20, 7) and (0, – 8)
Ans. (–1, 0)

4. Area of a Triangle:

If A(x1, y1), B(x2, y2), C(x3, y3) are the vertices of triangle ABC, then its area is equal to

x1 y1 1
1
 ABC = x2 y 2 1 , provided the vertices are considered in the counter clockwise sense. The
2
x3 y3 1

above formula will give a  ve area if the vertices (xi, yi), i = 1, 2, 3 are placed in the clockwise sense.

NOTE : Area of n-sided polygon formed by points (x 1, y1) ; (x 2, y2); ........(x n, yn) is given by

1  x1 x 2 x2 x3 x xn x x1 
   .......... ..... n1  n 
2 y y y2 y3 y n1 y n yn y1 
 1 2

Solved Example # 5
If the coordinates of two points A and B are (3, 4) and (5, –2) respectively. Find the coordinates
of any point P if PA = PB and Area of PAB = 10.
Solution
Let the coordinates of P be (x, y). Then
PA = PB  PA2 = PB2  (x – 3)2 + (y – 4)2 = (x – 5)2 + (y + 2)2
 x – 3y – 1 = 0

x y 1
1 3 4 1
Now, Area of PAB = 10  = ± 10  6x + 2y – 26 = ± 20
2 5 2 1

 6x + 2y – 46 = 0 or 6x + 2y – 6 = 0
 3x + y – 23 = 0 or 3x + y – 3 = 0
Solving x – 3 y – 1 = 0 and 3x + y – 23 = 0 we get x = 7, y = 2. Solving x – 3y – 1 = 0 and
3x + y – 3 = 0, we get x = 1, y = 0. Thus, the coordinates of P are (7, 2) or (1, 0) Ans.

Self practice problems :

7. The area of a triangle is 5. Two of its vertices are (2, 1) and (3, –2). The third vertex lies on
y = x + 3. Find the third vertex.
 7 13   3 3
Ans.  ,  or   , 
 2 2   2 2

8. The vertices of a quadrilateral are (6, 3), (–3, 5), (4, –2) and (x, 3x) and are denoted by A, B, C and
D, respectively. Find the values of x so that the area of triangle ABC is double the area of triangle DBC.
11 3
Ans. x= or –
8 8

RESONANCE TM 22
5. Slope Formula:
If  is the angle at which a straight line is inclined to the positive direction of x  axis, &
0°  < 180°,  90°, then the slope of the line, denoted by m, is defined by m = tan . If  is 90°, m
does not exist, but the line is parallel to the yaxis. If  = 0, then m = 0 & the line is parallel to the
x -axis.
If A (x1, y1) & B (x2, y2), x 1  x 2, are points on a straight line, then the slope m of the line is given by :

 y1  y 2 
m =  x  x  .
 1 2 

Solved Example # 6
What is the slope of a line whose inclination is :
(i) 0º (ii) 90º (iii) 120º (iv) 150º
Solution
(i) Here  = 0º
Slope = tan  = tan 0º = 0 Ans.

(ii) Here  = 90º


 The slope of line is not defined Ans.

(iii) Here = 120º


 Slope = tan  = tan 120º = tan (180º – 60º) = – tan 30º = – 3 Ans.

(iv) Here  = 150º


1
 Slope = tan  = tan 150º = tan (180º – 30º) = – tan 30º = – Ans.
3

Solved Example # 7

Find the slope of the line passing through the points :


(i) (1, 6) and (– 4, 2) (ii) (5, 9) and (2, 9)

Solution
(i) Let A = (1, 6) and B = (– 4, – 2)

26 4 4  y  y1 
 Slope of AB = = = Ans.  U sin g slope  2 
 4 1 5 5  x 2  x1 

(ii) Let A = (5, 9), B = (2, 9)


99 0
 Slope of AB = = = 0 Ans.
25 3

Self practice problems :

9. Find the value of x, if the slope of the line joining (1, 5) and (x, –7) is 4.
Ans. –2

10. What is the inclination of a line whose slope is


(i) 0 (ii) 1 (iii) –1 (iv) –1/ 3
Ans. (i) 0º, (ii) 45º, (iii) 135º, (iv) 150º

RESONANCE TM 23
6. Condition of collinearity of three points:

Points A (x 1, y1), B (x 2, y2), C(x 3, y3) are collinear if

 y  y2   y  y3 
(i) m AB = m BC = m CA i.e.  1  =  2 

 x1  x 2   x2  x3 

x1 y1 1

(ii)  ABC = 0 i.e. x2 y2 1


=0
x3 y3 1

(iii) AC = AB + BC or AB ~ BC
(iv) A divides the line segment BC in some ratio.

Solved Example # 8

Show that the points (1, 1), (2, 3) and (3, 5) are collinear.

Solution.
Let (1, 1) (2, 3) and (3, 5) be the coordinates of the points A, B and C respectively.
3 1 53
Slope of AB = = 2 and Slope of BC = =2
2 1 32
 Slope of AB = slope of AC
 AB & BC are parallel
 A, B, C are collinear because B is on both lines AB and BC.

Self practice problem :


1 1
11. Prove that the points (a, 0), (0, b) and (1, 1) are collinear if + =1
a b

7. Equation of a Straight Line in various forms:

(i) Point - Slope form : y  y1 = m (x  x 1) is the equation of a straight line whose slope is m &
which passes through the point (x 1, y1).

Solved Example # 9

Find the equation of a line passing through (2, –3) and inclined at an angle of 135º with the
positive direction of x-axis.

Solution.
Here, m = slope of the line = tan 135º = tan (90º + 45º) = – cot 45º = –1, x 1 = 2, y1 = –3
So, the equation of the line is y – y1 = m (x – x 1)
i.e. y – (–3) = –1 (x – 2) or y + 3 = –x + 2 or x + y + 1 = 0 Ans.

Self practice problem :

12. Find the equation of the perpendicular bisector of the line segment joining the points A(2, 3) and
B (6, –5).
Ans. x–2y–6=0

(ii) Slope  intercept form : y = mx + c is the equation of a straight line whose slope is m &
which makes an intercept c on the yaxis.

RESONANCE TM 24
Solved Example # 10

Find the equation of a line with slope –1 and cutting off an intercept of 4 units on negative
direction of y-axis.

Solution.
Here m = –1 and c = – 4. So, the equation of the line is y = mx + c i.e. y = – x – 4 or x + y + 4 = 0
Ans.

Self practice problem :

13. Find the equation of a straight line which cuts off an intercept of length 3 on y-axis and is parallel to the
line joining the points (3, –2) and (1, 4).
Ans. 3x + y – 3 = 0
y 2  y1
(iii) Two point form : y  y1 = x  x (x  x1) is the equation of a straight line which passes
2 1

through the points (x 1, y1) & (x2, y2).

Solved Example # 11

Find the equation of the line joining the points (– 1, 3) and (4, – 2)

Solution.
Here the two points are (x 1, y1) = (–1, 3) and (x2, y2) = (4, –2).
So, the equation of the line in two-point form is
3  ( 2 )
y–3= (x + 1)  y – 3 = – x – 1  x + y – 2 = 0 Ans.
 1 4

Self practice problem :

14. Find the equations of the sides of the triangle whose vertices are (–1, 8), (4, –2) and (–5, –3). Also find
the equation of the median through (–1, 8)
Ans. 2x + y – 6 = 0, x – 9y – 22 = 0, 11x – 4y + 43 = 0, 21x + y + 13 = 0

x y 1
(iv) Determinant form : Equation of line passing through (x 1, y1) and (x2, y2) is x1 y1 1  0
x2 y2 1

Solved Example # 12

Find the equation of line passing through (2, 4) & (– 1, 3).


Solution.

x y 1
2 4 1
=0  x – 3y + 10 = 0 Ans.
1 3 1

Self practice problem :

15. Find the equation of the passing through (– 2, 3) & (– 1, – 1).


Ans. 4x + y + 5 = 0

RESONANCE TM 25
x y
(v) Intercept form :  = 1 is the equation of a straight line which makes intercepts a & b on
a b
OX & OY respectively.

Solved Example # 13

Find the equation of the line which passes through the point (3, 4) and the sum of its intercepts
on the axes is 14.
x y
Sol. Let the equation of the line be + =1 ....(i)
a b

3 4
This passes through (3, 4), therefore + =1 ....(ii)
a b

3 4
It is given that a + b = 14  b = 14 – a. Putting b = 14 – a in (ii), we get + =1
a 14  a
 a2 – 13a + 42 = 0
 (a – 7) (a – 6) = 0  a = 7, 6
For a = 7, b = 14 – 7 = 7 and for a = 6, b = 14 – 6 = 8.
Putting the values of a and b in (i), we get the equations of the lines
x y y y
+ = 1 and + =1
7 7 6 8
or x + y = 7 and 4x + 3y = 24 Ans.

Self practice problem :

16. Find the equation of the line through (2, 3) so that the segment of the line intercepted between the axes
is bisected at this point.
Ans. 3x + 2y = 12.

(vi) Perpendicular/Normal form : xcos  + ysin  = p (where p > 0, 0   < 2 ) is the


equation of the straight line where the length of the perpendicular from the origin O on the line
is p and this perpendicular makes an angle  with positive xaxis.

Solved Example # 14

Find the equation of the line which is at a distance 3 from the origin and the perpendicular
from the origin to the line makes an angle of 30º with the positive direction of the x-axis.

Solution.
Here p = 3,  = 30º
 Equation of the line in the normal form is

3 y
x cos 30º + y sin 30º = 3 or x + = 3 or 3x+y=6 Ans.
2 2
Self practice problem :

17. The length of the perpendicular from the origin to a line is 7 and the line makes an angle of 150º with the
positive direction of y-axis. Find the equation of the line.
Ans. 3 x + y – 14 = 0

RESONANCE TM 26
xx yy
, y1 + r sin ) or cos  sin = r is the
1 1
(vii) Parametric form : P (r) = (x, y) = (x 1 + r cos

equation of the line in parametric form, where ‘r’ is the parameter whose absolute value is the
distance of any point (x, y) on the line from the fixed point (x 1, y1) on the line.

Solved Example # 15

Find the equation of the line through the point A(2, 3) and making an angle of 45º with the
x-axis. Also determine the length of intercept on it between A and the line x + y + 1 = 0

Solution.
The equation of a line through A and making an angle of 45º with the x-axis is

x2 y3 x2 y 3


= or =
cos 45 º sin 45 º 1 1
2 2
or x–y+1=0
Suppose this line meets the line x + y + 1 = 0 at P such that AP = r. Then the coordinates of P are given
by
x2 y3
= =r x = 2 + r cos 45º, y = 3 + r sin 45º
cos 45 º sin 45 º

r r
 x=2+ ,y=3+
2 2

 r r 
Thus, the coordinates of P are  2  ,3 
 2 2

r r
Since P lies on x + y + 1 = 0, so 2 + +3+ +1=0
2 2

 2 r = – 6  r = –3 2  length AP = | r | = 3 2

Thus, the length of the intercept = 3 2 Ans.

Self practice problem :

18. A straight line is drawn through the point A  3, 2 making an angle of /6 with positive direction of the
x-axis. If it meets the straight line 3 x – 4y + 8 = 0 in B, find the distance between A and B.
Ans. 6 units

(viii) General Form : ax + by + c = 0 is the equation of a straight line in the general form
a
In this case, slope of line = –
b

c
x - intercept = –
a

c
y - intercept = –
b

RESONANCE TM 27
Solved Example # 16
Find slope, x-intercept & y-intercept of the line 2x – 3y + 5 = 0.
Solution.
Here, a = 2, b = – 3, c = 5
a 2
 slope = – = Ans.
b 3

c 5
x-intercept = – =– Ans.
a 2
5
y-intercept = Ans.
3

Self practice problem :

19. Find the slope, x-intercept & y-intercept of the line 3x – 5y – 8 = 0.


3 8 8
Ans , ,–
5 3 5

8. Angle between two straight lines in terms of their slopes:


If m 1 & m 2 are the slopes of two intersecting straight lines (m 1 m 2  1) &  is the acute angle between

m1  m 2
them, then tan  = .
1  m1m 2
NOTE :
(i) Let m 1, m 2, m 3 are the slopes of three lines L1 = 0;L2 = 0;L3 = 0 where m 1 > m 2 > m 3 then the interior
angles of the  ABC found by these lines are given by,

m 1 m 2 m 2 m 3 m 3 m 1
tan A = ; tan B = & tan C =
1 m 1m 2 1 m 2 m 3 1 m 3 m 1
(ii) The equation of lines passing through point (x 1, y1) and making angle  with the line
y = mx + c are given by :
(y  y1) = tan ( ) (x  x 1) &
(y  y1) = tan (+ ) (x  x1), where tan = m.

Solved Example # 17

The acute angle between two lines is /4 and slope of one of them is 1/2. Find the slope of the
other line.
Solution.
m1  m 2
If  be the acute angle between the lines with slopes m 1 and m 2, then tan  = 1  m m
1 2
 1
Let = and m 1 =
4 2
1
 m2
 2 1  2m 2 1  2m 2
 tan = 1  1 = 2m  2  m 2 = + 1 or – 1
4 1  m2 2
2
1  2m 2 1 1  2m 2
Now 2  m2 = 1  m2 = – and 2  m = – 1  m 2 = 3.
3 2

 The slope of the other line is either – 1/3 or 3 Ans.

RESONANCE TM 28
Solved Example # 18

Find the equation of the straight line which passes through the origin and making angle 60º

with the line x + 3 y + 3 3 = 0.


Solution.

Given line is x + 3 y + 3 3 = 0.
 1  1
 y =    x–3
  Slope of (1) = – .
 3  3

 
Let slope of the required line be m. Also between these lines is given to be 60º.

1  m  1/ 3 
m   1/ 3 3m  1 3m  1
 tan 60º =  3 =  =± 3
3 m 3 m

3m  1 1
=– 3  3m+1=3– 3m m=
3 m 3
Using y = mx + c, the equation of the required line is
1
y= x + 0 i.e. x – 3 y = 0. ( This passes through origin, so c = 0)
3

3m  1
=– 3  3m+1 =–3+ 3m
3 m
 m is not defined
 The slope of the required line is not defined. Thus, the required line is a vertical line. This line
is to pass through the origin.
 The equation of the required line is x = 0 Ans.

Self practice problem :

20. A vertex of an equilateral triangle is (2, 3) and the equation of the opposite side is x + y = 2. Find the
equation of the other sides of the triangle.
Ans. (2 + 3 )x – y + 2 3 – 1 = 0 and (2 + 3 ) x – y – 2 3 – 1 = 0.

9. Parallel Lines:
(i) When two straight lines are parallel their slopes are equal. Thus any line parallel to
y = mx + c is of the type y = mx + d, where k is a parameter.
a b c
(ii) Two lines ax + by + c = 0 and ax + by + c  = 0 are parallel if =  .
a b c
Thus any line parallel to ax + by + c = 0 is of the type ax + by + k = 0, where k is a parameter.
(iii) The distance between two parallel lines with equations ax + by + c 1 = 0 &

c 1 c 2
ax + by + c 2 = 0 is .
a 2 b 2
NOTE: Coefficients of x & y in both the equations must be same.
p1p 2
(iv) The area of the parallelogram = sin , where p1 & p2 are distances

between two pairs of opposite sides &  is the angle between any two
adjacent sides. Note that area of the parallelogram bounded by the
lines y = m 1x + c 1, y = m 1x + c 2 and y = m 2x + d1, y = m 2x + d2 is given
( c 1 c 2 )( d1 d 2 )
by 
m 1 m 2

RESONANCE TM 29
Solved Example # 19

Find the equation of the straight line that has y-intercept 4 and is parallel to the straight line
2x – 3y = 7.

Solution.
Given line is 2x – 3y = 7
2 7
(1)  3y = 2x – 7  y = x–
3 3
 Slope of (1) is 2/3
The required line is parallel to (1), so its slope is also 2/3, y-intercept of required line = 4
 By using y = mx + c form, the equation of the required line is
2
y= x + 4 or 2x – 3y + 12 = 0 Ans.
3
Solved Example # 20
Two sides of a square lie on the lines x + y = 1 and x + y + 2 = 0. What is its area?

Solution.
Clearly the length of the side of the square is equal to the distance between the parallel lines
x + y – 1 = 0 ........(i) and x + y + 2 = 0 ........(ii)
Putting x = 0 in (i), we get y = 1. So (0, 1) is a point on line (i).
Now, Distance between the parallel lines
| 0  1 2 | 3
= length of the  from (0, 1) to x + y + 2 = 0 = =
1 1
2 2 2

 3 
2
3
Thus, the length of the side of the square is and hence its area =   = 9
2  2  2

Solved Example # 21
Find the area of the parallelogram whose sides are x + 2y + 3 = 0, 3x + 4y – 5 = 0,
2x + 4y + 5 = 0 and 3x + 4y – 10 = 0
Solution.

3 5 10 5 1 3
Here, c1 = – , c2 = , d1 = , d2 = – , m 1 = – , m 2 = –
2 2 3 2 2 4

 3 5   10 5 
     
 2 2  3 2 70
 Area = = sq. units Ans.
 1 3 3
  
 2 4

RESONANCE TM 30
Self practice problem :

21. Find the area of parallelogram whose sides are given by 4x – 5y + 1 = 0, x – 3y – 6 = 0,


4x – 5y – 2 = 0 and 2x – 6y + 5 = 0
51
Ans. sq. units
14

1 0 . Perpendicular Lines:

(i) When two lines of slopes m 1 & m 2 are at right angles, the product of their slopes is 1,
i.e. m 1 m 2 = Thus any line perpendicular to y = mx + c is of the form

1
y=  x + d, where d is any parameter..
m
(ii) Two lines ax + by + c = 0 and ax + by + c = 0 are perpendicular if aa + bb = 0. Thus any line
perpendicular to ax + by + c = 0 is of the form bx  ay + k = 0, where k is any parameter.

Solved Example # 22

Find the equation of the straight line that passes through the point (3, 4) and perpendicular to
the line 3x + 2y + 5 = 0

Solution.
The equation of a line perpendicular to 3x + 2y + 5 = 0 is
2x – 3y +  = 0 ...........(i)
This passes through the point (3, 4)
 3×2–3×4+=0=6
Putting = 6 in (i), we get 2x – 3y + 6 = 0, which is the required equation. Ans.

Aliter The slope of the given line is –3/2. Since the required line is perpendicular to the given line. So, the
2
slope of the required line is 2/3. As it passes through (3, 4). So, its equation is y – 4 = (x – 3) or
3
2x – 3y + 6 = 0Ans.

Self practice problem :

22. The vertices of a triangle are A(10, 4), B (–4, 9) and C(–2, –1). Find the equation of its altitudes. Also
find its orthocentre.

 9
Ans. x – 5y + 10 = 0, 12x + 5y + 3 = 0, 14x – 5y + 23 = 0,   1, 
 5

1 1 . Position of the point (x 1 , y 1) relative of the line ax + by + c = 0:

If ax 1 + by1 + c is of the same sign as c, then the point (x 1, y1) lie on the origin side of ax + by + c = 0.
But if the sign of ax 1 + by1 + c is opposite to that of c, the point (x1, y1) will lie on the nonorigin side of
ax + by + c = 0.
In general two points (x1, y1) and (x 2, y2) will lie on same side or opposite side of ax + by + c = 0
according as ax 1 + by1 + c and ax2 + by2 + c are of same or opposite sign respectively.

RESONANCE TM 31
Solved Example # 23

Show that (1, 4) and (0, –3) lie on the opposite sides of the line x + 3y + 7 = 0.

Solution.

At (1, 4), the value of x + 3y + 7 = 1 + 3(4) + 7 = 20 > 0.


At (0, – 3), the value of x + 3y + 7 = 0 + 3(–3) + 7 = –2 < 0
 The points (1, 4) and (0, – 3) are on the opposite sides of the given line. Ans.

Self practice problems :

23. Are the points (3, – 4) and (2, 6) on the same or opposite side of the line 3x – 4y = 8 ?
Ans. Opposite sides

24. Which one of the points (1, 1), (–1, 2) and (2, 3) lies on the side of the line 4x + 3y – 5 = 0 on which the
origin lies?
Ans. (–1, 2)

1 2 . The ratio in which a given line divides the line segment joining two
points:
Let the given line ax + by + c = 0 divide the line segment joining A(x1, y1) & B(x 2, y2) in the ratio m : n,
m ax1by1c
then  . If A & B are on the same side of the given line then m/n is negative but if A &
n ax 2 by 2 c
B are on opposite sides of the given line, then m/n is positive

Solved Example # 24

Find the ratio in which the line joining the points A (1, 2) and B(– 3, 4) is divided by the line
x + y – 5 = 0.

Solution.
Let the line x + y = 5 divides AB in the ratio k : 1 at P
 coordinate of P are

  3k  1 4k  2 
 , 
 k 1 k 1 
Since P lies on x + y – 5 = 0
 3k  1 4k  2
 + – 5 = 0.
k 1 k 1

1
 k=–
2
 Required ratio is 1 : 2 extrenally Ans.

Aliter
Let the ratio is m : n

m (1 1  1 2  5) 1
 =– =–
n 1  ( 3 )  1  4  5 2
 ratio is 1 : 2 externally Ans.

RESONANCE TM 32
Self practice problem :

25. If the line 2x – 3y +  = 0 divides the line joining the points A (– 1, 2) & B(– 3, – 3) internally in the ratio
2 : 3, find .
18
Ans.
5

1 3 . Length of perpendicular from a point on a line:

a x1  b y1  c
The length of perpendicular from P(x1, y1) on ax + by + c = 0 is .
a 2  b2

Solved Example # 25

Find the distance between the line 12x – 5y + 9 = 0 and the point (2, 1)
Solution.

12  2  5  1  9 | 24  5  9 | 28
The required distance = = = Ans.
12  ( 5)
2 2 13 13

Solved Example # 26

Find all points on x + y = 4 that lie at a unit distance from the line 4x + 3y – 10 = 0.

Solution.
Note that the coordinates of an arbitrary point on x + y = 4 can be obtained by putting x = t (or y = t)
and then obtaining y (or x) from the equation of the line, where t is a parameter.
Putting x = t in the equation x + y = 4 of the given line, we obtain y = 4 – t. So, coordinates of an
arbitrary point on the given line are P(t, 4 – t). Let P(t, 4 – t) be the required point. Then, distance of P
from the line 4x + 3y – 10 = 0 is unity i.e.

4t  3( 4  t )  10
 = 1  |t + 2| = 5  t + 2 = ± 5
42  32
 t = –7 or t = 3
Hence, required points are (–7, 11) and (3, 1) Ans.

Self practice problem :

26. Find the length of the altitudes from the vertices of the triangle with vertices :(–1, 1), (5, 2) and (3, –1).
16 8 16
Ans. , ,
13 5 37

1 4 . Reflection of a point about a line:


(i) Foot of the perpendicular from a point on the line is
x  x1 y  y1 ax1 by1c
  
a b a 2 b 2
(ii) The image of a point (x1, y1) about the line ax + by + c = 0 is

xx1 yy1 ax 1  by 1  c
 2 .
a b a 2 b 2

RESONANCE TM 33
Solved Example # 27

Find the foot of perpendicular of the line drawn from P (– 3, 5) on the line x – y + 2 = 0.

Solution.
Slope of PM = – 1

 Equation of PM is
x+y–2=0 .........(i)
solving equation (i) with x – y + 2 = 0, we get coordinates of M (0, 2) Ans.

Aliter

x3 y 5 (1 ( 3)  ( 1)  5  2)


Here, = =–
1 1 (1)2  ( 1)2

x3 y 5
 = =3  x+3=3  x=0
1 1
and y–5=–3  y=2
 M is (0, 2) Ans.

Solved Example # 28

Find the image of the point P(–1, 2) in the line mirror 2x – 3y + 4 = 0.

Solution.
Let image of P is Q.

 PM = MQ & PQ  AB
Let Q is (h, k)

 h 1 k  2 
 M is  , 
 2 2 
It lies on 2x – 3y + 4 = 0.

 h  1 k  2
 2   –3   + 4 = 0.
 2   2 
or 2h – 3k = 0 ...........(i)
k2
slope of PQ =
h 1
PQ  AB
k2 2
 × = – 1.
h 1 3
 3h + 2k – 1 = 0. ........(ii)
soving (i) & (ii), we get
3 2
h= ,k=
13 13

 3 2 
 Image of P(– 1, 2) is Q  ,  Ans.
 13 13 
Aliter
The image of P (– 1, 2) about the line 2x – 3y + 4 = 0 is

x 1 y2 [2( 1)  3(2)  4]


= =–2
2 3 2 2  ( 3 ) 2

RESONANCE TM 34
x 1 y2 8
= =
2  3 13

3
 13x + 13 = 16  x=
13

2
& 13y – 26 = – 24  y=
13

 3 2 
 image is  ,  Ans.
 13 13 

Self practice problems :

27. Find the foot of perpendicular of the line drawn from (– 2, – 3) on the line 3x – 2y – 1 = 0.

  23  41 
Ans.  , 
 13 13 

28. Find the image of the point (1, 2) in y-axis.


Ans. (– 1, 2)

1 5 . Bisectors of the angles between two lines:


Equations of the bisectors of angles between the lines ax + by + c = 0 &

ax  by  c ax  by  c 
ax + by + c  = 0 (ab ab) are : =±
a2  b2 a  2  b 2
NOTE :
Equation of straight lines passing through P(x 1, y1) & equally inclined with the lines a1x + b1y + c 1 = 0
&
a2x + b2y + c2 = 0 are those which are parallel to the bisectors between these two lines & passing
through the point P.

Solved Example # 29
Find the equations of the bisectors of the angle between the straight lines
3x – 4y + 7 = 0 and 12 x – 5y – 8 = 0.
Solution.
The equations of the bisectors of the angles between 3x – 4y + 7 = 0 and 12 x – 5y – 8 = 0 are

3x  4y  7 12 x  5 y  8

3 2  ( 4 ) 2 12 2  ( 5)2

3x  4y  7 12 x  5 y  8
or =±
5 13
or 39x – 52y + 91 = ± (60 x – 25 y – 8)
Taking the positive sign, we get 21 x + 27 y – 131 = 0 as one bisector Ans.
Taking the negative sign, we get 99 x – 77 y + 51 = 0 as the other bisector. Ans.
Self practice problem :
29. Find the equations of the bisectors of the angles between the following pairs of straight lines
3x + 4y + 13 = 0 and 12x – 5y + 32 = 0
Ans. 21x – 77y – 9 = 0 and 99x + 27y + 329 = 0

RESONANCE TM 35
1 6 . Methods to discriminate between the acute angle bisector & the
obtuse angle bisector:
(i) If  be the angle between one of the lines & one of the L1 = 0

bisectors, find tan  P p


  
  q
If tan  < 1, then 2  < 90° so that this bisector is the    

  
 
acute angle bisector.   

  
 
If tan  > 1, then we get the bisector to be the obtuse  


 
angle bisector. u2 = 0

  
u1 = 0

 
(ii) Let L1 = 0 & L2 = 0 are the given lines & u1 = 0 and u2 = 0 are the bisectors between L1 = 0 &
L2 = 0. Take a point P on any one of the lines L1 = 0 or L2 = 0 and drop perpendicular on u1 = 0
& u2 = 0 as shown. If,
p < q  u1 is the acute angle bisector.
p > q  u1 is the obtuse angle bisector.
p = q  the lines L1 & L2 are perpendicular.
(iii) If aa + bb < 0, then the equation of the bisector of this acute angle is

a x + by + c a  x + b  y + c
=+
a 2  b2 a  2  b 2
If, however, aa+ bb > 0, the equation of the bisector of the obtuse angle is :

a x + by + c a  x + b  y + c
=+
a b
2 2
a  2  b 2

Solved Example # 30
For the straight lines 4x + 3y – 6 = 0 and 5x + 12y + 9 = 0, find the equation of the
(i) bisector of the obtuse angle between them;
(ii) bisector of the acute angle between them;
Solution.
(i) The equations of the given straight lines are
4x + 3y – 6 = 0 ........(1)
5x + 12y + 9 = 0 ........(2)
The equation of the bisectors of the angles between lines (1) and (2) are

4x  3y  6 5 x  12y  9 4x  3y  6 5 x  12 y  9
=± or =±
4 3
2 2
5  12
2 2 5 13
Taking the positive sign, we have
4x  3y  6 5 x  12 y  9
=
5 13
or 52x + 39y – 78 = 25x + 60y + 45 or 27x – 21y – 123 = 0
or 9x – 7y – 41 = 0
Taking the negative sign, we have
4x  3y  6 5 x  12 y  9
=–
5 13
or 52x + 39y – 78 = –25x – 60y – 45 or 77x + 99y – 33 = 0
or 7x + 9y – 3 = 0
Hence the equation of the bisectors are
9x – 7y – 41 = 0 ........(3)
and 7x + 9y – 3 = 0 ........(4)
RESONANCE TM 36
4 9
Now slope of line (1) = – and slope of the bisector (3) = .
3 7
If  be the acute angle between the line (1) and the bisector (3), then

9 4
 27  28 55 11
7 3
tan  = = = = >1
9 4 21  36  15 3
1   
7 3
  > 45º
Hence 9x – 7y – 41 = 0 is the bisector of the obtuse angle between the given lines (1) and (2)
Ans.
(ii) Since 9x – 7y – 41 is the bisector of the obtuse angle between the given lines, therefore the other
bisector 7x + 9y – 3 = 0 will be the bisector of the acute angle between the given lines.
2nd Method :
Writing the equation of the lines so that constants become positive we have
– 4x – 3y + 6 = 0 .......(1)
and 5x + 12y + 9 = 0 .......(2)
Here a1 = – 4, a2 = 5, b1 = –3, b2 = 12
Now a1a2 + b1b2 = – 20 – 36 = –56 < 0
 origin does not lie in the obtuse angle between lines (1) and (2) and hence equation of the
bisector of the obtuse angle between lines (1) and (2) will be
4 x  3 y  6 5 x  12y  9
2 = –
( 4)  ( 3)
2
5 2  12 2
or 13(–4x – 3y + 6) = –5(5x + 12y + 9)
or 27x – 21y – 123 = 0 or 9x – 7y – 41 = 0 Ans.
and the equation of the bisector of the acute angle will be (origin lies in the acute angle)
4 x  3 y  6 5 x  12y  9
=
( 4)2  ( 3)2 5 2  12 2
or 77x + 99y – 33 = 0 or 7x + 9y – 3 = 0 Ans.

Self practice problem :

30. Find the equations of the bisectors of the angles between the lines x + y – 3 = 0 and
7x – y + 5 = 0 and state which of them bisects the acute angle between the lines.
Ans. x – 3y + 10 = 0 (bisector of the obtuse angle); 4x + 1 = 0 (bisector of the acute angle)

17. To discriminate between the bisector of the angle containing a


po int:
To discriminate between the bisector of the angle containing the origin & that of the angle not containing
the origin. Rewrite the equations, ax + by + c = 0 & ax + by + c = 0 such that the constant terms c,
a x + by + c a  x + b  y + c
c  are positive. Then ; = + gives the equation of the bisector of the
a b
2 2
a  2  b 2

a x + by + c a  x + b  y + c
angle containing the origin & = gives the equation of the bisector of the
a 2  b2 a  2  b 2
angle not containing the origin. In general equation of the bisector which contains the point (,  ) is,

a x  by  c a  x  b y  c a x  by  c a  x  b y  c
= or =  according as
a 2  b2 a  2  b 2 a 2  b2 a  2  b 2
a + b  + c and a + b + c having same sign or otherwise.

RESONANCE TM 37
Solved Example # 31

For the straight lines 4x + 3y – 6 = 0 and 5x + 12y + 9 = 0, find the equation of the bisector of the
angle which contains the origin.

Solution.
For point O(0, 0), 4x + 3y – 6 = –6 < 0 and 5x + 12y + 9 = 9 > 0
Hence for point O(0, 0) 4x + 3y – 6 and 5x + 12y + 9 are of opposite signs.
Hence equation of the bisector of the angle between the given lines containing the origin will be

4x  3y  6 5 x  12y  9
=–
( 4 )  (3 )
2 2
5 2  122

4x  3y  6 5 x  12 y  9
or =–
5 13
or 52x + 39y – 78 = –25x – 60y – 45.
or 77x + 99y – 33 = 0

or 7x + 9y – 3 = 0 Ans.

Self practice problem :

31. Find the equation of the bisector of the angle between the lines x + 2y – 11 = 0 and
3x – 6y – 5 = 0 which contains the point (1, – 3).
Ans. 3x – 19 = 0

1 8 . Condition of Concurrency:

Three lines a1x + b1y + c 1 = 0, a2x + b2y + c 2 = 0 & a3x + b3y + c 3 = 0 are concurrent if

a1 b1 c1
a2 b2 c2 = 0.
a3 b3 c3

Alternatively : If three constants A, B & C (not all zero) can be found such that
A(a1x + b1y + c1) + B(a2x + b2y + c 2) + C(a3x + b3y + c3)  0, then the three straight lines are concurrent.

Solved Example # 32

Prove that the straight lines 4x + 7y = 9, 5x – 8y + 15 = 0 and 9x – y + 6 = 0 are concurrent.

Solution.
Given lines are
4x + 7y – 9 = 0 ........(1)
5x – 8y + 15 = 0 ........(2)
and 9x – y + 6 = 0 ........(3)

4 7 9
5  8 15
= = 4(–48 + 15) – 7 (30 – 135) – 9 (– 5 + 72) = –132 + 735 – 603 = 0
9 1 6

Hence lines (1), (2) and (3) are concurrent. Proved

RESONANCE TM 38
Self practice problem :

32. Find the value of m so that the lines 3x + y + 2 = 0, 2x – y + 3 = 0 and x + my – 3 = 0 may be


concurrent.
Ans. 4

1 9 . Family Of Straight Lines:


The equation of a family of straight lines passing through the point of intersection of the lines,
L1  a1x + b1y + c 1 = 0 & L2  a2x + b2y + c 2 = 0 is given by L1 + k L2 = 0 i.e.
(a1x + b1y + c1) + k(a2x + b2y + c2) = 0, where k is an arbitrary real number.

NOTE :

(i) If u1 = ax + by + c, u2 = ax + by + d, u3 = ax + by + c,


u4 = ax + by + d
then u1 = 0;u2 = 0; u3 = 0 ; u4 = 0 form a parallelogram.
The diagonal BD can be given by u2u3 – u1u4 = 0.

(ii) The diagonal AC is also given by u1 +  u4 = 0 and


u2 + u3 = 0, if the two equations are identical for some real  and 
[For getting the values of  & compare the coefficients of x, y & the constant terms].

Solved Example # 33

Find the equation of the straight line which passes through the point (2, –3) and the point of
intersection of the lines x + y + 4 = 0 and 3x – y – 8 = 0.

Solution.
Any line through the intersection of the lines x + y + 4 = 0 and 3x – y – 8 =0 has the equation
(x + y + 4) +  (3x – y – 8) = 0 .........(i)
This will pass through (2, –3) if
(2 – 3 + 4) +  (6 + 3 – 8) = 0 or 3 +  = 0   = – 3.
Putting the value of  in (i), the required line is
(x + y + 4) + (–3) (3x – y – 8) = 0
or – 8x + 4y + 28 = 0 or 2x – y – 7 = 0 Ans.
Aliter
Solving the equations x + y + 4 = 0 and 3x – y – 8 = 0 by cross-multiplication, we get x = 1, y = –5
So the two lines intersect at the point (1, –5). Hence the required line passes through (2, –3)
and (1, –5) and so its equation is
53
y+3=– (x – 2) or 2x – y – 7 = 0 Ans.
1 2

Solved Example # 34

Obtain the equations of the lines passing through the intersection of lines 4x – 3y – 1 = 0 and
2x – 5y + 3 = 0 and equally inclined to the axes.

Solution.
The equation of any line through the intersection of the given lines is
(4x – 3y – 1) +  (2x – 5y + 3) = 0
or x (2  + 4) – y (5 + 3) + 3 – 1 = 0 .......(i)
2  4
Let m be the slope of this line. Then m =
5  3

RESONANCE TM 39
As the line is equally inclined with the axes, therefore

2  4 1
m = tan 45º of m = tan 135º  m = ±1, = ± 1   = –1 or , putting the values of  in (i), we
5  3 3
get 2x + 2y – 4 = 0 and 14x – 14y = 0
i.e. x + y – 2 = 0 and x = y as the equations of the required lines. Ans.

Self practice problem :

33. Find the equation of the lines through the point of intersection of the lines x – 3y + 1 = 0 and
2x + 5y – 9 = 0 and whose distance from the origin is 5
Ans. 2x + y – 5 = 0

2 0 . A Pair of straight lines through origin:

(i) A homogeneous equation of degree two,


"ax² + 2hxy + by² = 0" always represents a pair of straight lines passing
through the origin if :
(a) h² > ab  lines are real & distinct .
(b) h² = ab  lines are coincident .
(c) h² < ab  lines are im aginary with real point of
intersection i.e. (0, 0)
(ii) If y = m 1x & y = m 2x be the two equations represented by ax² + 2hxy + by² = 0, then;

2h a
m1 + m2 =  & m1 m2 = .
b b
(iii) If  is the acute angle between the pair of straight lines represented by,

2 h 2  ab
ax² + 2hxy + by² = 0,then ; tan = ab .

(iv) The condition that these lines are :


(a) At right angles to each other is a + b = 0. i.e. coefficient of x² + coefficient of y² = 0.
(b) Coincident is h² = ab .
(c) Equally inclined to the axis of x is h = 0.i.e. coeff. of xy = 0 .
NOTE :
A homogeneous equation of degree n represents n straight lines passing through origin.

(v) The equation to the pair of straight lines bisecting the angle between the straight lines,

x2  y2 xy
ax² + 2hxy + by² = 0 is = .
ab h
Solved Example # 35

Show that the equation 6x2 – 5xy + y2 = 0 represents a pair of distinct straight lines, each
passing through the origin. Find the separate equations of these lines.
Solution.
The given equation is a homogeneous equation of second degree. So, it represents a pair of straight
lines passing through the origin. Comparing the given equation with
ax2 + 2hxy + by2 = 0, we obtain a = 6, b = 1 and 2h = – 5.
25 1
 h2 – ab = –6= > 0  h2 > ab
4 4
RESONANCE TM 40
Hence, the given equation represents a pair of distinct lines passing through the origin.
y y
2

Now, 6x – 5xy + y = 0
2 2
   –5   +6=0
x x

y y y y  y 


2

   –3   –2   +6=0   3    2 = 0
x x x x  x 
y y
 – 3 = 0 or – 2 = 0  y – 3x = 0 or y – 2x = 0
x x
So the given equation represents the straight lines y – 3x = 0 and y – 2x = 0 Ans.

Solved Example # 36

Find the equations to the pair of lines through the origin which are perpendicular to the lines
represented by 2x 2 – 7xy + 2y2 = 0.

Solution.
We have 2x2 – 7xy + 2y2 = 0.
 2x2 – 6xy – xy + 3y2 = 0  2x(x – 3y) – y (x – 3y) = 0
 (x – 3y) (2x – y) = 0  x – 3y = 0 or 2x – y = 0
Thus the given equation represents the lines x – 3y = 0 and 2x – y = 0. The equations of the lines
passing through the origin and perpendicular to the given lines are y – 0 = –3 (x – 0)
1
and y – 0 = – (x – 0) [  (Slope of x – 3 y = 0) is 1/3 and (Slope of 2x – y = 0) is 2]
2
 y + 3x = 0 and 2y + x = 0 Ans.

Solved Example # 37

Find the angle between the pair of straight lines 4x2 + 24xy + 11y2 = 0

Solution.
Given equation is 4x2 + 24xy + 11y2 = 0
Here a = coeff. of x2 = 4, b = coeff. of y2 = 11
and 2h = coeff. of xy = 24  h = 12
2 h 2  ab 2 144  44 4
Now tan  = ab = 4  11 =
3
Where  is the acute angle between the lines.
4
 acute angle betw een the lines is tan –1   and obtuse angle betw een them is
3
 4 
 – tan –1   Ans.
3

Solved Example # 38

Find the equation of the bisectors of the angle between the lines represented by
3x2 – 5xy + y2 = 0
Solution.
Given equation is 3x2 – 5xy + y2 = 0 .......(1)
comparing it with the equation ax + 2hxy + by = 0
2 2
.......(2)
we have a = 3, 2h = –5; and b = 4
x2  y2 xy
Now the equation of the bisectors of the angle between the pair of lines (1) is =
ab h

x2  y2 xy x2  y2 2xy
or = ; or =
34 5 1 5

2
or 5x2 – 2xy – 5y2 = 0 Ans.

RESONANCE TM 41
Self practice problems :

34. Find the area of the triangle formed by the lines y2 – 9xy + 18x 2 = 0 and y = 9.

27
Ans. sq. units
4

35. If the pairs of straight lines x 2 – 2pxy – y2 = 0 and x 2 – 2qxy – y2 = 0 be such that each pair bisects the
angle between the other pair, prove that pq = –1.

2 1 . General equation of second degree representing a pair of Straight


l in e s :

(i) ax² + 2hxy + by² + 2gx + 2fy + c = 0 represents a pair of straight lines if :

a h g

abc + 2fgh  af²  bg²  ch² = 0, i.e. if h b f


= 0.
g f c

(ii) The angle  between the two lines representing by a general equation is the same as that
between the two lines represented by its homogeneous part only.

Solved Example # 39

Prove that the equation 2x2 + 5xy + 3y2 + 6x + 7y + 4 = 0 represents a pair of straight lines. Find
the co-ordinates of their point of intersection and also the angle between them.

Solution.
Given equation is
2x2 + 5xy + 2y2 + 6x + 7y + 4 = 0
Writing the equation (1) as a quadratic equation in x we have
2x2 + (5y + 6) x + 3y2 + 7y + 4 = 0

 (5 y  6)  (5y  6)2  4.2(3y 2  7 y  4)


 x=
4

 (5 y  6)  25y 2  60y  36  24y 2  56 y  32


=
4

 (5 y  6)  y 2  4 y  4 (5 y  6)  ( y  2)
= =
4 4

5 y  6  y  2 5 y  6  y  2
 x= ,
4 4
or 4x + 4y + 4 = 0and 4x + 6y + 8 = 0
or x + y + 1 = 0 and 2x + 3y + 4 = 0
Hence equation (1) represents a pair of straight lines whose equation are x + y + 1 = 0 .....(1)
and 2x + 3y + 4 = 0 .........(2) Ans.
Solving these two equations, the required point of intersection is (1, – 2) Ans.

RESONANCE TM 42
Self practice problem :

36. Find the combined equation of the straight lines passing through the point (1, 1) and parallel to the
lines represented by the equation x2 – 5xy + 4y2 + x + 2y – 2 = 0 and find the angle between them.
3
Ans. x 2 – 5xy + 4y2 + 3x – 3y = 0, tan–1  
5

2 2 . Homogenization :

The equation of a pair of straight lines joining origin to the points of intersection of the line
L  x + my + n = 0 and a second degree curve,
S  ax² + 2hxy + by² + 2gx + 2fy + c = 0

 x  my   x  my   x  my 
2

is ax² + 2hxy + by² + 2gx    2fy    c   = 0.


 n   n   n 
The equation is obtained by homogenizing the equation of curve with the help of equation of line.

NOTE : Equation of any curve passing through the points of intersection of two curves C1 = 0 and
C2 = 0 is given by  C1 +  C2 = 0 where  &  are parameters.

Solved Example # 40

Prove that the angle between the lines joining the origin to the points of intersection of the
straight line y = 3x + 2 with the curve x2 + 2xy + 3y2 + 4x + 8y – 11 = 0 is

2 2
tan–1 .
3
Solution.
Equation of the given curve is x 2 + 2xy + 3y2 + 4x + 8y – 11 = 0
y  3x
and equation of the given straight line is y – 3x = 2;  =1
2
Making equation (1) homogeneous equation of the second degree in x any y with the help of (1), we
have

 y  3x   y  3x   y  3x 
2

x 2 + 2xy + 3y2 + 4x   + 8y   – 11
1   =0
 2   2   2 

1 11 2
or x 2 + 2xy + 3y2 + (4xy + 8y2 – 12x 2 – 24 xy) – (y – 6xy + 9x 2) = 0
2 4
or 4x 2 + 8xy + 12y2 + 2(8y2 – 12x2 – 20xy) – 11 (y2 – 6xy + 9x 2) = 0
or –119x2 + 34xy + 17y2 = 0 or 119x2 – 34xy – 17y2 = 0
or 7x 2 – 2xy – y2 = 0
This is the equation of the lines joining the origin to the points of intersection of (1) and (2).
Comparing equation (3) with the equation ax2 + 2hxy + by2 = 0
we have a = 7, b = –1 and 2h = –2 i.e. h = –1
If  be the acute angle between pair of lines (3), then

2 h 2  ab 2 1 7 2 8 2 2 2 2
tan  = ab = 7 1 = =   = tan–1 Proved
6 3 3

RESONANCE TM 43
Self practice problems :

37. Find the equation of the straight lines joining the origin to the points of intersection of the line
3x + 4y – 5 = 0 and the curve 2x 2 + 3y2 = 5.
Ans. x 2 – y2 – 24xy = 0

38. Find the equation of the straight lines joining the origin to the points of intersection of the line
lx + my + n = 0 and the curve y2 = 4ax. Also, find the condition of their perpendicularity.
Ans. 4alx2 + 4amxy + ny2 = 0; 4al + n = 0

RESONANCE TM 44
Part : (A) Only one correct option

1. The equation of the internal bisector of BAC of ABC with vertices A(5, 2), B(2, 3) and
C(6, 5) is
(A) 2x + y + 12 = 0 (B) x + 2y – 12 = 0 (C) 2x + y – 12 = 0 (D) none of these

2. The lines ax + by + c = 0, where 3a + 2b + 4c = 0 are concurrent at the point :

1 3  3 1
(A)  ,  (B) (1, 3) (C) (3, 1) (D)  , 
2 4 4 2

3. The equation of second degree x2 + 2 2 xy + 2y2 + 4x + 4 2 y + 1 = 0 represents a pair of straight


lines. The distance between them is

4
(A) 4 (B) (C) 2 (D) 2 3
3

4. The straight lines joining the origin to the points of intersection of the line 2x + y = 1 and curve
3x2 + 4xy – 4x + 1 = 0 include an angle :

   
(A) (B) (C) (D)
2 3 4 6

5. Given the points A (0, 4) and B (0,  4), the equation of the locus of the point P (x, y) such that
AP  BP = 6 is :
(A) 9x2  7y2 + 63 = 0 (B) 9x2  7y2  63 = 0
(C) 7x2  9y2 + 63 = 0 (D) 7x2  9y2  63 = 0

6. A triangle ABC with vertices A ( 1, 0),B ( 2, 3/4) & C ( 3,  7/6) has its orthocentre H. Then the
orthocentre of triangle BCH will be :
(A) ( 3,  2) (B) (1, 3) (C) (  1, 2) (D) none of these

7. Equation of a straight line passing through the origin and making with x  axis an angle twice the size
of the angle made by the line y = 0.2 x with the x  axis, is :
(A) y = 0.4 x (B) y = (5/12) x (C) 6y  5x = 0 (D) none of these

8. A variable straight line passes through a fixed point (a, b) intersecting the coordinates axes at A & B.
If 'O' is the origin then the locus of the centroid of the triangle OAB is :
(A) bx + ay  3xy = 0 (B) bx + ay  2xy = 0
(C) ax + by  3xy = 0 (D) ax + by  2xy = 0

9. Area of the quadrilateral formed by the lines x  + y = 2 is :


(A) 8 (B) 6 (C) 4 (D) none

10. The distance of the point (2, 3) from the line 2 x  3 y + 9 = 0 measured along a line x - y + 1 = 0 is :
(A) 5 3 (B) 4 2 (C) 3 2 (D) 2 2
11. The set of values of 'b' for which the origin and the point (1, 1) lie on the same side of the straight line,
a2x + a by + 1 = 0  a  R, b > 0 are :
(A) b  (2, 4) (B) b  (0, 2) (C) b  [0, 2] (D) (2, )

RESONANCE 45
TM
12. Drawn from the origin are two mutually perpendicular straight lines forming an isosceles triangle to-
gether with the straight line, 2x + y = a. Then the area of the triangle is :
a2 a2 a2
(A) (B) (C) (D) none
2 3 5
13. The line joining two points A (2, 0);B (3, 1) is rotated about A in the anticlock wise direction through an
angle of 15º. The equation of the line in the new position is :

(A) x  3y 2=0 (B) x  2y  2 = 0


(C) 3xy2 3 =0 (D) none

14. The line x + 3y  2 = 0 bisects the angle between a pair of straight lines of which one has equation
x  7y + 5 = 0. The equation of the other line is :
(A) 3x + 3y  1 = 0 (B) x  3y + 2 = 0 (C) 5x + 5y  3 = 0 (D) none

15. On the portion of the straight line, x + 2y = 4 intercepted between the axes, a square is constructed on
the side of the line away from the origin. Then the point of intersection of its diagonals has
coordinates :
(A) (2, 3) (B) (3, 2) (C) (3, 3) (D) none

16. A light beam emanating from the point A(3, 10) reflects from the straight line 2x + y  6 = 0 and then
passes through the point B(4, 3). The equation of the reflected beam is :
(A) 3x  y + 1 = 0 (B) x + 3y  13 = 0 (C) 3x + y  15 = 0 (D) x  3y + 5 = 0

17. The equation of the bisector of the angle between two lines 3 x  4 y + 12 = 0 and
12 x  5 y + 7 = 0 which contains the points ( 1, 4) is :
(A) 21x + 27y  121 = 0 (B) 21x  27y + 121 = 0
 3x  4y  12 12x  5y  7
(C) 21x + 27y + 191 = 0 (D) =
5 13

18. The equation of bisectors of two lines L1 & L2 are 2 x  16 y  5 = 0 and 64 x + 8 y + 35 = 0. If the line
L1 passes through (  11, 4), the equation of acute angle bisector of L1 & L2 is :
(A) 2 x  16 y  5 = 0 (B) 64 x + 8 y + 35 = 0 (C) data insufficient (D) none of these

19. The equation of the pair of bisectors of the angles between two straight lines is, 12x 2  7xy  12y2 = 0.
If the equation of one line is 2y  x = 0 then the equation of the other line is :
(A) 41x  38y = 0 (B) 38x  41y = 0 (C) 38x + 41y = 0 (D) 41x + 38y = 0

20. If the straight lines j oining the origin and the point s of int ersect ion of the curve
5x2 + 12xy  6y2 + 4x  2y + 3 = 0 and x + ky  1 = 0 are equally inclined to the x-axis then the value
of k is equal to :
(A) 1 (B)  1 (C) 2 (D) 3

21. If the points of intersection of curves C1 =  x2 + 4 y2  2 x y  9 x + 3 &


C2 = 2 x 2 + 3 y2  4 x y + 3 x 1 subtends a right angle at origin, then the value of  is :
(A) 19 (B) 9 (C)  19 (D)  9

Part : (B) May have more than one options correct

22. The equation of the bisectors of the angle between the two intersecting lines :

x3 y5 x3 y5 x 3 y5 x 3 y5


= and = are = and = then
cos  sin  cos  sin  cos  sin   


(A)  = (B)  = – sin  (C)  = cos  (D)  = sin 
2

RESONANCE 46
TM
23. Equation of a straight line passing through the point of intersection of x – y + 1 = 0 and 3x + y – 5 = 0
are perpendicular to one of them is
(A) x + y + 3 = 0 (B) x + y – 3 = 0 (C) x – 3y – 5 = 0 (D) x – 3y + 5 = 0

24. Three lines px + qy + r = 0, qx + ry + p = 0 and rx + py + q = 0 are concurrent if


(A) p + q + r = 0 (B) p2 + q2 + r 2 = pq + qr + rp
(C) p + p + r = 3 pqr
3 3 3
(D) none of these

25. Equation of a straight line passing through the point (4, 5) and equally inclined to the lines,
3x = 4y + 7 and 5y = 12x + 6 is
(A) 9 x  7 y = 1 (B) 9 x + 7 y = 71 (C) 7 x + 9 y = 73 (D) 7 x  9 y + 17 = 0

26. If the equation, 2x 2 + k xy  3y2  x  4y  1 = 0 represents a pair of lines then the value of k can be:
(A) 1 (B) 5 (C)  1 (D)  5

27. If a2 + 9b2  4c2 = 6 ab then the family of lines ax + by + c = 0 are concurrent at :


(A) (1/2, 3/2) (B) (  1/2,  3/2) (C) ( 1/2, 3/2) (D) (1/2,  3/2)

y2  y3 y 3  y1 y1  y 2
1. If the points (x1, y1), (x 2, y2) and (x3, y3) be collinear, show that x 2 x 3 + x 3 x1 + x1x 2 = 0.

2. Find the length of the perpendicular from the origin upon the straight line joining the two points whose
coordinates are (a cos , a sin ) and (a cos , a sin ).

3. Show that the product of the perpendiculars drawn from the two points (± a 2  b 2 , 0) upon the

x y
straight line cos  + sin  = 1 is b2.
a b

4. Find the equation of the bisector of the acute angle between the lines 3x – 4y + 7 = 0 and
12x + 5y – 2 = 0.

5. Find the equation to the pair of straight lines joining the origin to the intersections of the straight line
y = mx + c and the curve x2 + y2 = a2. Prove that they are at right angles if 2c2 = a2 (1 + m 2).

6. The variable line x cos  + y sin = 2 cuts the x and y axes at A and B respectively. Find the locus of the
vertex P of the rectangle OAPB, O being the origin.

7. If A(x1, y1), B(x 2, y2), C(x 3, y3) are the vertices of the triangle then show that :

x y 1 x y 1
(i) The median through A can be written in the form x1 y1 1  x1 y1 1 = 0.
x2 y2 1 x3 y3 1
x y 1 x y 1
(ii) the line through A & parallel to BC can be written in the form ; x1 y1 1  x1 y1 1 = 0.
x2 y2 1 x3 y3 1

x y 1 x y 1
(iii) equation to the angle bisector through A is b x1 y1 1  c x1 y1 1 = 0.
x2 y2 1 x3 y 3 1

where b = AC & c = AB.


RESONANCE 47
TM
8. Is there a real value of  for which the image of the point ( ,  1) by the line mirror 3 x + y = 6  is
the point (  2 + 1,  ) ? If so, find  .

9. If the straight lines, ax + by + p = 0 & x cos  + y sin  p = 0 enclose an angle /4 between them, and
meet the straight line x sin  y cos = 0 in the same point, then find the value of a2 + b2 .

10. Drive the conditions to be imposed on  so that (0,  ) should lie on or inside the triangle having sides
y + 3x + 2 = 0, 3y  2x  5 = 0 & 4y + x  14 = 0.

11. A straight line L is perpendicular to the line 5x  y = 1. The area of the triangle formed by the line L &
the coordinate axes is 5. Find the equation of the line.

12. Two equal sides of an isosceles triangle are given by the equations 7x – y + 3 = 0 and x + y – 3 = 0 and
its third side passes through the point (1, –10). Determine the equation of the third side.

13. Find the equations of the straight lines passing through the point (1, 1) and parallel to the lines represented
by the equation, x2  5 xy + 4y2 + x + 2 y  2 = 0.

14. Find the coordinates of the vertices of a square inscribed in the triangle with vertices
A (0, 0), B (2, 1), C (3, 0); given that two of its vertices are on the side AC.

15. The equations of perpendiculars of the sides AB & AC of  ABC are x  y  4 = 0 and
2x  y  5 = 0 respectively. If the vertex A is ( 2, 3) and point of intersection of perpendiculars

3 5
bisectors  ,  is, find the equation of medians to the sides AB and AC respectively..
2 2

16. The sides of a triangle are 4x + 3y + 7 = 0, 5x + 12y = 27 and 3x + 4y + 8 = 0. Find the equations of
the internal bisectors of the angles and show that they are concurrent.

17. A ray of light is sent along the line x  2y  3 = 0. Upon reaching the line 3x  2y  5 = 0, the ray is
reflected from it. Find the equation of the line containing the reflected ray.

18. A triangle is formed by the lines whose equations are AB : x + y – 5 = 0, BC : x + 7y – 7 = 0 and CA :


7x + y + 14 = 0. Find the bisector of the interior angle at B and the exterior angle at C. Determine the
nature of the interior angle at A and find the equation of the bisector.

19. Find the equations of the sides of a triangle having (4, –1) as a vertex, if the lines x – 1 = 0 and
x – y – 1 = 0 are the equations of two internal bisectors of its angles.

20. The equations of the altitudes AD, BE, CF of a triangle ABC are x + y = 0, x – 4y = 0 and 2x – y = 0
respectively. Thr coordinates of A are (t, –t). Find coordinates of B and C. Prove that it t varies the locus
of the centroid of the triangle ABC is x + 5y = 0.

21. For points P = (x 1, y1) and Q = (x 2, y2) of the co-ordinate plane, a new distance d(P, Q) is defined by
d(P, Q) = |x 1 – x2| + |y1 – y2|. Let O = (0, 0) and A = (3, 2). Prove that the set of points in the first quadrant
which are equidistant (with respect to the new distance) from O and A consists of the union of a line
segment of finite length and an infinite ray. Sketch this set in a labelled diagram. [IIT -2000, 10]

RESONANCE 48
TM
22. Let ABC and PQR be any two triangles in the same plane. Assume that the prependiculars from the
points A, B, C to the sides QR, RP, PQ respectively are concurrent. Using vector methods or otherwise,
prove that the prependiculars from P, Q, R to BC, CA, AB respectively are also concurrent.
[IIT - 2000, 10]

23. A straight line L through the origin meets the lines x + y = 1 and x + y = 3 at P and Q respectively.
Through P and Q two straight lines L1 and L2 are drawn parallel to 2x – y = 5 and 3x + y = 5 respectively.
Lines L1 and L2 intersect at R. Show that the locus of R, as L varies, is a straight line.
[IIT - 2002, 5]

24. A straight line L with negative slope passes through the point (8, 2) and cuts the positive coordinate
axes at points P and Q. Find the absolute minimum value of OP + OQ, as L varies, where O is the
origin. [IIT - 2002, 5]

25. The area of the triangle formed by the intersection of a line parallel to x-axis and passing through P(h,
k) with the lines y = x and x + y = 2 is 4h2. Find the locus of the point P. [IIT - 2005, 2]

EXERCISE # 1 12. x – 3y – 31 = 0 or 3x + y + 7 = 0
1. C 2. D 3. C 4. A 5. A 6. D 7. B
13.  4 y + 3) (x  y) = 0
(x
8. A 9. A 10. B 11. B 12. C 13. C 14. C
or x2  5 x y + 4 y2 + 3 x  3 y = 0
15. C 16. B 17. A 18. A 19. A 20. B 21. C
3  9   3 3  9 3
22. ABC 23. BD 24. ABC 25. AC 26. AD 14.  , 0   , 0  ,  , ,  , 
2  4  2 4 4 4
27. CD

EXERCISE # 2 15. x + 4y = 4; 5x + 2y = 8

17. 29x  2y = 31
 
2. a cos   4. 11x – 3y + 9 = 0
 2  18. 3x + 6y – 16 = 0; 8x + 8y + 7 = 0; 12x + 6y – 11 = 0

1 1 1 19. 2x – y + 3 = 0, 2x + y – 7 = 0; x – 2y – 6 = 0
6.  2  8. 2 9. 2
x 2
y 4
10. 5/3 7/2  2t t t 
20. B   ,   , C  , t
 3 6  2 
11. x + 5y + 5 2 = 0 or x + 5y 5 2 =0
24. 18 25. y = 2x + 1 or y = –2x + 1

RESONANCE 49
TM
Circle

A circle is a locus of a point whose distance from a fixed point


(called centre) is always constant (called radius).

1. Equation of a Circle in Various Form:


(a) The circle with centre as origin & radius ‘r’ has the
equation; x2 + y2 = r 2.
(b) The circle with centre (h, k) & radius ‘r’ has the

Applied mathematics is (c)


equation; (x  h)2 + (y  k)2 = r 2.
The general equation of a circle is
x 2 + y2 + 2gx + 2fy + c = 0
like the wine, which
with centre as (g, f) & radius = g2 f 2 c . If:
becomes pure in course g² + f²  c > 0  real circle.
g² + f²  c = 0  point circle.
oftime. g² + f²  c < 0  imaginary circle, with real centre,
....... Dantzig that is (– g, – f)
Note : that every second degree equation in x & y, in which
coefficient of x 2 is equal to coefficient of y2 & the coefficient of
xy is zero, always represents a circle.

W henever two (d) The equation of circle with (x 1, y1 ) & (x 2, y2 ) as


extremeties of its diameter is:
unknown magnitudes (x  x 1) (x  x 2) + (y  y1) (y  y2) = 0.
Note that this will be the circle of least radius passing
appear in a final through (x1, y1) & (x2, y2).

equation, we have a Example : Find the equation of the circle whose centre is (1, –2)
locus, the extremity of and radius is 4.
Solution : The equation of the circle is (x – 1) 2 + (y – (–2))2 = 42
one of the unknown  (x – 1) 2 + (y + 2)2 = 16
 x 2 + y2 – 2x + 4y – 11 = 0 Ans.
magnitudes describing
Example : Find the equation of the circle which passes through
a straight line or a the point of intersection of the lines 3x – 2y – 1 = 0
curve. and 4x + y – 27 = 0 and whose centre is (2, – 3).
Solution : Let P be the point of intersection of the lines AB and
LM whose equations are respectively
....... PierreFermat
3x – 2y – 1 = 0 ..........(i)
and 4x + y – 27 = 0 ..........(ii)
Solving (i) and (ii), we get x = 5, y = 7. So, coordinates
of P are (5, 7). Let C(2, –3) be the centre of the circle.
Since the circle passes through P, therefore
CP = radius  (5  2)2  (7  3 )2 = radius
 radius = 109 .

RESONANCE 1
Hence the equation of the required circle is

(x – 2) 2 + (y + 3)2 =  109 2

Example : Find the centre & radius of the circle whose equation is x2 + y2 – 4x + 6y + 12 = 0
Solution : Comparing it with the general equation x 2 + y2 + 2gx + 2fy + c = 0, we have
2g = – 4  g = –2
2f = 6  f=3
& c = 12
 centre is (–g, –f) i.e. (2, –3)

and radius = g2  f 2  c = ( 2)2  (3)2  12 = 1

Example : Find the equation of the circle, the coordinates of the end points of whose diameter are
(–1, 2) and (4, –3)
Solution : We know that the equation of the circle described on the line segment joining (x 1, y1) and
(x 2, y2) as a diameter is (x – x1) (x – x 2) + (y – y1) (y – y2) = 0.
Here, x1 = –1, x2 = 4, y1 = 2 and y2 = –3.
So, the equation of the required circle is
(x + 1) (x – 4) + (y – 2) (y + 3) = 0  x 2 + y2 – 3x + y – 10 = 0.

Self Practice Problems :

1. Find the equation of the circle passing through the point of intersection of the lines x + 3y = 0 and
2x – 7y = 0 and whose centre is the point of intersection of the lines x + y + 1 = 0 and x – 2y + 4 = 0.
Ans. x 2 + y2 + 4x – 2y = 0

2. Find the equation of the circle whose centre is (1, 2) and which passes through the point (4, 6)
Ans. x 2 + y2 – 2x – 4y – 20 = 0

3. Find the equation of a circle whose radius is 6 and the centre is at the origin.
Ans. x 2 + y2 = 36.

2. Intercepts made by a Circle on the Axes:


The intercepts made by the circle x 2 + y2 + 2gx + 2fy + c = 0 on the coordinate axes are 2 g2 c &

2 f c respectively. If
2

g2  c > 0  circle cuts the x axis at two distinct points.

g2 = c  circle touches the xaxis.

g2 < c  circle lies completely above or below the xaxis.

Example : Find the equation to the circle touching the y-axis at a distance – 3 from the origin and
intercepting a length 8 on the x-axis.
Solution : Let the equation of the circle be x 2 + y2 + 2gx + 2fy + c = 0. Since it touches y-axis at (0, –3)
and (0, – 3) lies on the circle.
 c = f 2 ...(i) 9 – 6f + c = 0 .......(ii)
From (i) and (ii), we get 9 – 6f + f 2 = 0  (f – 3) 2 = 0  f = 3.
Putting f = 3 in (i) we obtain c = 9.
It is given that the circle x2 + y2 + 2gx + 2fy + c = 0 intercepts length 8 on x-axis

 2 g2  c = 8  2 g2  9 = 8  g2 – 9 = 16  g=±5
Hence, the required circle is x + y ± 10x + 6y + 9 = 0.
2 2

RESONANCE 2
Self Practice Problems :

1. Find the equation of a circle which touches the axis of y at a distance 3 from the origin and intercepts
a distance 6 on the axis of x.
Ans. x 2 + y2 ± 6 2 x – 6y + 9 = 0

2. Find the equation of a circle which touches y-axis at a distance of 2 units from the origin and cuts an
intercept of 3 units with the positive direction of x-axis.
Ans. x 2 + y2 ± 5x – 4y + 4 = 0

3. Parametric Equations of a Circle:


The parametric equations of (x  h)2 + (y  k) 2 = r 2 are: x = h + r cos  ; y = k + r sin ;   <   
where (h, k) is the centre, r is the radius &  is a parameter.

Example : Find the parametric equations of the circle x2 + y2 – 4x – 2y + 1 = 0


Solution : We have : x2 + y2 – 4x – 2y + 1 = 0  (x 2 – 4x ) + (y2 – 2y) = – 1
 (x – 2) + (y – 1) = 2
2 2 2

So, the parametric equations of this circle are


x = 2 + 2 cos  , y = 1 + 2 sin .

Example : Find the equations of the following curves in cartesian form. Also, find the centre and radius of
the circle x = a + c cos , y = b + c sin 
x a y b
Solution : We have : x = a + c cos , y = b + c sin  cos  = , sin  =
c c

 xa  y b
2 2

   +   = cos 2 + sin2  (x – a)2 + (y – b)2 = c 2


 c   c 
Clearly, it is a circle with centre at (a, b) and radius c.

Self Practice Problems :

1. Find the parametric equations of circle x 2 + y2 – 6x + 4y – 12 = 0


Ans. x = 3 + 5 cos , y = –2 + 5 sin 

2. Find the cartesian equations of the curve x = –2 + 3 cos , y = 3 + 3 sin 


Ans. (x + 2)2 + (y – 3)2 = 9

4. Position of a point with respect to a circle:


The point (x 1, y1) is inside, on or outside the circle S  x 2 + y2 + 2gx + 2fy + c = 0.
according as S1  x 1² + y1² + 2gx1 + 2fy1 + c <, = or > 0.

NOTE : The greatest & the least distance of a point A from a circle with centre C & radius r is AC + r &

AC  r respectively.

Example : Disc uss the position of the points (1, 2) and (6, 0) wit h respect to the c ircle
x 2 + y2 – 4x + 2y – 11 = 0
Solution : We have x2 + y2 – 4x + 2y – 11 = 0 or S = 0, where S = x 2 + y2 – 4x + 2y – 11.
For the point (1, 2), we have S1 = 12 + 22 – 4 × 1 +2 × 2 – 11 < 0
For the point (6, 0), we have S2 = 62 + 02 – 4 × 6 +2 × 0 – 11 > 0
Hence, the point (1, 2) lies inside the circle and the point (6, 0) lies outside the circle.

RESONANCE 3
Self Practice Problem :

1. How are the points (0, 1) (3, 1) and (1, 3) situated with respect to the circle x 2 + y2 – 2x – 4y + 3 = 0?
Ans. (0, 1) lies on the circle ; (3, 1) lies outside the circle ; (1, 3) lies inside the circle.

5. Line and a Circle:


Let L = 0 be a line & S = 0 be a circle. If r is the radius of the circle & p is the length of the perpendicular
from the centre on the line, then:
(i) p>r  the line does not meet the circle i. e. passes out side the circle.
(ii) p=r  the line touches the circle. (It is tangent to the circle)
(iii) p<r  the line is a secant of the circle.
(iv) p=0  the line is a diameter of the circle.
Also, if y = mx + c is line and x2 + y2 = a2 is circle then
(i) c 2 > a2 (1 + m 2) the line is a secant of the circle.
(ii) c 2 = a2 (1 + m 2)  the line touches the circle. (It is tangent to the circle)
(iii) c < a (1 + m ) 
2 2 2
the line does not meet the circle i. e. passes out side the circle.

Example : For what value of c will the line y = 2x + c be a tangent to the circle x 2 + y2 = 5 ?
Solution : We have : y = 2x + c or 2x – y + c = 0 ......(i) and x 2 + y2 = 5 ........(ii)
If the line (i) touches the circle (ii), then
length of the  from the centre (0, 0) = radius of circle (ii)

20  0  c c
 = 5  = 5
2  ( 1)
2 2
5

c
 =± 5  c=±5
5
Hence, the line (i) touches the circle (ii) for c = ± 5

Self Practice Problem :

1. For what value of , does the line 3x + 4y =  touch the circle x 2 + y2 = 10x.
Ans. 40, –10

6. Tangent :
(a) Slope form :
y = mx + c is always a tangent to the circle x2 + y2 = a2 if c 2 = a2 (1 + m 2). Hence, equation

 a 2m a 2 
of tangent is y = mx ± a 1  m2 and the point of contact is   , .
 c c
(b) Point form :
(i) The equation of the tangent to the circle x2 + y2 = a2 at its point (x 1, y1) is,
x x1 + y y1 = a².
(ii) The equation of the tangent to the circle x 2 + y2 + 2gx + 2fy + c = 0 at its point
(x 1, y1) is: xx1 + yy1 + g (x+x 1) + f (y+y1) + c = 0.
NOTE : In general the equation of tangent to any second degree curve at point (x 1, y1) on it can be
x  x1 y  y1
obtained by replacing x2 by x x 1, y2 by yy1, x by , y by ,
2 2
x1y  xy1
xy by and c remains as c.
2

RESONANCE 4
(c) Parametric form :
The equation of a tangent to circle x 2 + y2 = a2 at (a cos , a sin ) is
x cos  + y sin  = a.
NOTE : The point of intersection of the tangents at the points P() & Q() is

 a cos    a sin    
 2 , 2 
  
 cos 
cos 2 
 2

Example : Find the equation of the tangent to the circle x2 + y2 – 30x + 6y + 109 = 0 at (4, –1).
Solution : Equation of tangent is

x4  y  ( 1) 
4x + (–y) – 30   +6   + 109 = 0
 2   2 
or 4x – y – 15x – 60 + 3y – 3 + 109 = 0 or –11x + 2y + 46 = 0
or 11x – 2y – 46 = 0
Hence, the required equation of the tangent is 11x – 2y – 46 = 0

Example : Find the equation of tangents to the circle x 2 + y2 – 6x + 4y – 12 = 0 which are parallel to the
line 4x + 3y + 5 = 0
Solution : Given circle is x2 + y2 – 6x + 4y – 12 = 0 .......(i)
and given line is 4x + 3y + 5 = 0 .......(ii)
Centre of circle (i) is (3, –2) and its radius is 5. Equation of any line
4x + 3y + k = 0 parallel to the line (ii) .......(iii)
If line (iii) is tangent to circle, (i) then

| 4.3  3( 2)  k |
= 5 or |6 + k| = 25
42  32
or 6 + k = ± 25  k = 19, – 31
Hence equation of required tangents are 4x + 3y + 19 = 0 and 4x + 3y – 31 = 0

Self Practice Problem :

1. Find the equation of the tangents to the circle x2 + y2 – 2x – 4y – 4 = 0 which are (i) parallel,
(ii) perpendicular to the line 3x – 4y – 1 = 0
Ans. (i) 3x – 4y + 20 = 0 and 3x – 4y – 10 = 0
(ii) 4x + 3y + 5 = 0 and 4x + 3y – 25 = 0

7. Normal :
If a line is normal / orthogonal to a circle then it must pass through the centre of the circle. Using this
y1  f
fact normal to the circle x 2 + y2 + 2gx + 2fy + c = 0 at (x 1, y1) is; y  y1 = (x  x1).
x1  g

Exercise : Find the equation of the normal to the circle x2 + y2 – 5x + 2y – 48 = 0 at the point (5, 6).
Solution : The equation of the tangent to the circle x2 + y2 – 5x + 2y – 48 = 0 at (5, 6) is

x5 x 6
5x + 6y – 5   +2   – 48 = 0  10x + 12y – 5x – 25 + 2y + 12 – 96 = 0
 2   2 
 5x + 14y – 109 = 0
5 14
 Slope of the tangent = –  Slope of the normal =
14 5
Hence, the equation of the normal at (5, 6) is
y – 6 = (14/5) (x – 5)  14x – 5y – 40 = 0

RESONANCE 5
Self Practice Problem :

1. Find the equation of the normal to the circle x2 + y2 – 2x – 4y + 3 = 0 at the point (2, 3).
Ans. x–y+1=0

8. Pair of Tangents from a Point:


The equation of a pair of tangents drawn from the point A (x1, y1) to the circle
x 2 + y2 + 2gx + 2fy + c = 0 is : SS1 = T².
Where S  x 2 + y2 + 2gx + 2fy + c ; S1  x 1² + y1² + 2gx 1 + 2fy1 + c
T  xx1 + yy1 + g(x + x1) + f(y + y1) + c.

Example : Find the equation of the pair of tangents drawn to the circle x2 + y2 – 2x + 4y = 0 from the point
(0, 1)
Solution : Given circle is S = x2 + y2 – 2x + 4y = 0 .......(i)
Let P  (0, 1)
For point P, S1 = 02 + 12 – 2.0 + 4.1 = 5
Clearly P lies outside the circle
and T  x . 0 + y . 1 – (x + 0) + 2 (y + 1)
i.e. T  –x +3y + 2.
Now equation of pair of tangents from P(0, 1) to circle (1) is SS1 = T 2
or 5 (x2 + y2 – 2x + 4y) = (– x + 3y + 2)2
or 5x 2 + 5y2 – 10x + 20y = x2 + 9y2 + 4 – 6xy – 4x + 12y
or 4x 2 – 4y2 – 6x + 8y + 6xy – 4 = 0
or 2x 2 – 2y2 + 3xy – 3x + 4y – 2 = 0 .......(ii)
Note : Separate equation of pair of tangents : From (ii), 2x2 + 3(y – 1) x – 2(2y2 – 4y + 2) = 0

3( y  1)  9( y  1)2  8(2y 2  4 y  2)
 x=
4

or 4x – 3y + 3 = ± 25 y 2  50 y  25 = ± 5(y – 1)
 Separate equations of tangents are x – 2y + 2 = 0 and 2x + y – 1 = 0

Self Practice Problems :

1. Find the equation of the tangents through (7, 1) to the circle x 2 + y2 = 25.
Ans. 12x2 – 12y2 + 7xy – 175x – 25y + 625 = 0

9. Length of a Tangent and Power of a Point:


The length of a tangent from an external point (x1, y1) to the circle
S  x2 + y2 + 2gx + 2fy + c = 0 is given by L = x 1  y1  2gx1  2f1y  c = S1 .
2 2

Square of length of the tangent from the point P is also called the power of point w.r.t. a circle.
Power of a point w.r.t. a circle remains constant.
Power of a point P is positive, negative or zero according as the point ‘P’ is outside, inside or on the
circle respectively.

Exercise : Find the length of the tangent drawn from the point (5, 1) to the circle x2 + y2 + 6x – 4y – 3 = 0

Solution : Given circle is x2 + y2 + 6x – 4y – 3 = 0 .........(i)


Given point is (5, 1). Let P = (5, 1)

Now length of the tangent from P(5, 1) to circle (i) = 5 2  12  6.5  4.1  3 = 7

RESONANCE 6
Self Practice Problems :

1. Find the area of the quadrilateral formed by a pair of tangents from the point (4, 5) to the circle
x 2 + y2 – 4x – 2y – 11 = 0 and a pair of its radii.
Ans. 8 sq. units

2. If the length of the tangent from a point (f, g) to the circle x 2 + y2 = 4 be four times the length of the
tangent from it to the circle x 2 + y2 = 4x, show that 15f 2 + 15g2 – 64f + 4 = 0

1 0 . Director Circle:
The locus of the point of intersection of two perpendicular tangents is called the director circle of the
given circle. The director circle of a circle is the concentric circle having radius equal to 2 times the
original circle.

Example : Find the equation of director circle of the circle (x – 2)2 + (y + 1)2 = 2.
Solution : Centre & radius of given circle are (2, –1) & 2 respectively..
Centre and radius of the director circle will be (2, –1) & 2 × 2 = 2 respectively..
 equation of director circle is (x – 2) 2 + (y + 1)2 = 4
 x 2 + y2 – 4x + 2y + 1 = 0 Ans.

Self Practice Problems :

1. Find the equation of director circle of the circle whose diameters are 2x – 3y + 12 = 0 and
x + 4y – 5 = 0 and area is 154 square units.
Ans. (x + 3)2 + (y + 2)2 = 98

1 1 . Chord of Contact:
If two tangents PT 1 & PT 2 are drawn from the point P(x 1, y1) to the circle S  x 2 + y2 + 2gx + 2fy + c = 0,
then the equation of the chord of contact T 1T 2 is:
xx 1 + yy1 + g (x + x1) + f (y + y1) + c = 0.

NOTE : Here R = radius; L = length of tangent.

(a) Chord of contact exists only if the point ‘P’ is not inside.

2 LR
(b) Length of chord of contact T 1 T 2 = .
R 2  L2

R L3
(c) Area of the triangle formed by the pair of the tangents & its chord of contact =
R 2  L2

 2RL 
(d) Tangent of the angle between the pair of tangents from (x1, y1) =  2 
2 
L R 
(e) Equation of the circle circumscribing the triangle PT 1 T 2 is:
(x  x 1) (x + g) + (y  y1) (y + f) = 0.

RESONANCE 7
Example : Find the equation of the chord of contact of the tangents drawn from (1, 2) to the circle
x 2 + y2 – 2x + 4y + 7 = 0
Solution : Given circle is x2 + y2 – 2x + 4y + 7 = 0 .......(i)
Let P = (1, 2)
For point P (1, 2), x2 + y2 – 2x + 4y + 7 = 1 + 4 – 2 + 8 + 7 = 18 > 0
Hence point P lies outside the circle
For point P (1, 2), T = x . 1 + y . 2 – (x + 1) + 2(y + 2) + 7
i.e. T = 4y + 10
Now equation of the chord of contact of point P(1, 2) w.r.t. circle (i) will be
4y + 10 = 0 or 2y + 5 = 0

Example : Tangents are drawn to the circle x 2 + y2 = 12 at the points where it is met by the circle
x 2 + y2 – 5x + 3y – 2 = 0; find the point of intersection of these tangents.
Solution : Given circles are S1  x2 + y2 – 12 = 0 ....... (i)
and S2 = x + y – 5x + 3y – 2 = 0
2 2
....... (ii)
Now equation of common chord of circle (i) and (ii) is
S1 – S2 = 0 i.e. 5x – 3y – 10 = 0 ....... (iii)
Let this line meet circle (i) [or (ii)] at A and B
Let the tangents to circle (i) at A and B meet at P(), then AB will be the chord of contact of
the tangents to the circle (i) from P, therefore equation of AB will be

x + y – 12 = 0 ....... (iv)
Now lines (iii) and (iv) are same, therefore, equations (iii) and (iv) are identical
  12 18
 = =   = 6,  = –
5 3  10 5

 18 
Hence P =  6,  
 5 
Self Practice Problems :

1. Find the co-ordinates of the point of intersection of tangents at the points where the line
2x + y + 12 = 0 meets the circle x2 + y2 – 4x + 3y – 1 = 0
Ans. (1, – 2)

2. Find the area of the triangle formed by the tangents drawn from the point (4, 6) to the circle x2 + y2 = 25
and their chord of contact.

405 3
Ans. ; 4x + 6y – 25 = 0
52

1 2 . Pole and Polar:


(i) If through a point P in the plane of the circle, there be drawn any straight line to meet the circle
in Q and R, the locus of the point of intersection of the tangents at Q & R is called the Polar of
the point P; also P is called the Pole of the Polar.

(ii) The equation to the polar of a point P (x 1, y1) w.r.t. the circle x 2 + y2 = a 2 is given by
xx 1 + yy 1 = a 2 , & if the circle is general then the equation of the polar becom es
xx 1 + yy1 + g (x + x 1) + f (y + y1) + c = 0 i.e. T = 0. Note that if the point (x1, y1) be on the circle
then the tangent & polar will be represented by the same equation. Similarly if the point (x1, y1)
be outside the circle then the chord of contact & polar will be represented by the same equation.

RESONANCE 8
 Aa 2 Ba 2 
(iii) Pole of a given line Ax + By + C = 0 w.r.t. circle x2 + y2 = a2 is   , .

 C C 
(iv) If the polar of a point P pass through a point Q, then the polar of Q passes through P.

(v) Two lines L1 & L2 are conjugate of each other if Pole of L1 lies on L2 & vice versa. Similarly two
points P & Q are said to be conjugate of each other if the polar of P passes through Q &
vice-versa.

Example : Find the equation of the polar of the point (2, –1) with respect to the circle
x 2 + y2 – 3x + 4y – 8 = 0
Solution : Given circle is x2 + y2 – 3x + 4y – 8 = 0 ............(i)
Given point is (2, –1) let P = (2, –1). Now equation of the polar of point P
with respect to circle (i)

 x  2  y  1
x.2 + y(–1) – 3   +4   –8=0
 2   2 
or 4x – 2y – 3x – 6 + 4y – 4 – 16 = 0 or x + 2y – 26 = 0

Example : Find the pole of the line 3x + 5y + 17 = 0 with respect to the circle x2 + y2 + 4x + 6y + 9 = 0
Solution : Given circle is x 2 + y2 + 4x + 6y + 9 = 0 ............(i)
and given line is 3x + 5y + 17 = 0 ............(ii)
Let P() be the pole of line (ii) with respect to circle (i)
Now equation of polar of point P() with respect to circle (i) is
x + y + 2(x + ) + 3(y +) + 9 = 0
or ( + 2)x + ( + 3) y + 2 + 3 + 9 = 0 ............(iii)
Now lines (ii) and (iii) are same, therefore,
2 3 2  3  9
= =
3 5 17
(i) (ii) (iii)
From (i) and (ii), we get
5 + 10 = 3 + 9 or 5 – 3 = – 1 ............(iv)
From (i) and (iii), we get
17 + 34 = 6 + 9 + 27 or 11 – 9 = –7 ............(v)
Solving (iv) & (v), we get  = 1,  = 2
Hence required pole is (1, 2).

Self Practice Problems :

1. Find the co-ordinates of the point of intersection of tangents at the points where the line
2x + y + 12 = 0 meets the circle x2 + y2 – 4x + 3y – 1 = 0.
Ans. (1, – 2)

2. Find the pole of the straight line 2x – y + 10 = 0 with respect to the circle x2 + y2 – 7x + 5y – 1 = 0

3 3
Ans.  , 
2 2

1 3 . Equation of the Chord with a given Middle Point:


The equation of the chord of the circle S  x2 + y2 + 2gx + 2fy + c = 0 in terms of its mid point M (x 1, y1)
is xx 1 + yy1 + g (x + x1) + f (y + y1) + c = x 12 + y12 + 2gx 1 + 2fy1 + c which is designated by T = S1.

RESONANCE 9
NOTE :
(i) The shortest chord of a circle passing through a point ‘M’ inside the circle, is one chord whose
middle point is M.
(ii) The chord passing through a point ' M ' inside the circle and which is at a maximum distance
from the centre is a chord with middle point M.

Example : Find the equation of the chord of the circle x2 + y2 + 6x + 8y – 11 = 0, whose middle point is
(1, –1)
Solution : Equation of given circle is S  x 2 + y2 + 6x + 8y – 11 = 0
Let L  (1, –1)
For point L(1, –1), S1 = 12 + (–1)2 + 6.1 + 8(–1) – 11 = –11 and
T  x.1 + y (–1) + 3(x + 1) + 4(y – 1) – 11 i.e. T  4x + 3y – 12
Now equation of the chord of circle (i) whose middle point is L(1, –1) is
T = S1 or 4x + 3y – 12 = –11 or 4x + 3y – 1 = 0
Second Method :
4  1 3
Let C be the centre of the given circle, then C  (–3, –4). L  (1, –1) slope of CL = =
 3 1 4
 Equation of chord of circle whose middle point is L, is
4
 y+1=– (x – 1) [ chord is perpendicular to CL)
3
or 4x + 3y – 1 = 0

Self Practice Problems :

1. Find the equation of that chord of the circle x2 + y2 = 15, which is bisected at (3, 2)
Ans. 3x + 2y – 13 = 0

2. Find the co-ordinates of the middle point of the chord which the circle x 2 + y2 + 4x – 2y – 3 = 0 cuts off
on the line y = x + 2.

 3 1
Ans.  , 
 2 2

1 4 . Equation of the chord joining two points of circle :


The equation of chord PQ to the circle x2 + y2 = a2 joining two points P() and () on it is given by. The
equation of a straight line joining two point  &  on the circle x2 + y2 = a2 is
    
x cos + y sin = a cos .
2 2 2

1 5 . Common Tangents to two Circles:


Case Number of Tangents Condition

(i) 4 common tangents

(2 direct and 2 transverse) r 1 + r 2 < c 1 c 2.

(ii) 3 common tangents. r 1 + r 2 = c 1 c 2.

RESONANCE 10
(iii) 2 common tangents. r1  r 2 < c 1 c2 < r1 + r2

(iv) 1 common tangent. r1  r2 = c 1 c2.

(v) No common tangent. c 1 c 2 < r1  r2.

(Here C1C2 is distance between centres of two circles.)


IMPORTANT NOTE :
(i) The direct common tangents meet at a point which divides the line joining centre of circles
externally in the ratio of their radii.
Transverse common tangents meet at a point which divides the line joining centre of circles
internally in the ratio of their radii.
(ii) Length of an external (or direct) common tangent & internal (or transverse) common tangent to
the two circles are given by: Lext = d2  (r1  r2 )2 & Lint = d2  (r1  r2 )2 ,
where d = distance between the centres of the two circles and r1, r2 are the radii of the two
circles. Note that length of internal common tangent is always less than the length of the
external or direct common tangent.

Example: Examine if the two circles x2 + y2 – 2x – 4y = 0 and x2 + y2 – 8y – 4 = 0 touch each other


externally or internally.
Solution : Given circles are x2 + y2 – 2x – 4y = 0 ...........(i)
and x2 + y2 – 8y – 4 = 0 ...........(ii)
Let A and B be the centres and r1 and r 2 the radii of circles (i) and (ii) respectively, then
A  (1, 2), B  (0, 4), r1 = 5, r2 = 25

Now AB = (1  0)2  (2  4)2 = 5 and r1 + r2 = 3 5 , |r1 – r 2| = 5


Thus AB = |r1 – r 2|, hence the two circles touch each other internally.

Self Practice Problems :

1. Find the position of the circles x 2 + y2 – 2x – 6y + 9 = 0 and x2 + y2 + 6x – 2y + 1 = 0 with respect to


each other.
Ans. One circle lies completely outside the other circle.

1 6 . Orthogonality Of Two Circles:


Two circles S1= 0 & S2= 0 are said to be orthogonal or said to intersect orthogonally if the tangents at
their point of intersection include a right angle. The condition for two circles to be orthogonal is:
2 g1 g 2 + 2 f 1 f 2 = c 1 + c 2.
NOTE :
(a) The centre of a variable circle orthogonal to two fixed circles lies on the radical axis of two
circles.
(b) If two circles are orthogonal, then the polar of a point 'P' on first circle w.r.t. the second circle
passes through the point Q which is the other end of the diameter through P. Hence locus of a
point which moves such that its polars w.r.t. the circles S1 = 0, S2 = 0 & S3 = 0 are concurrent
in a circle which is orthogonal to all the three circles.
(c) The centre of a circle which is orthogonal to three given circles is the radical centre provided
the radical centre lies outside all the three circles.

RESONANCE 11
Example : Obtain the equation of the circle orthogonal to both the circles x 2 + y2 + 3x – 5y+ 6 = 0 and
4x2 + 4y2 – 28x +29 = 0 and whose centre lies on the line 3x + 4y + 1 = 0.
Solution. Given circles are x 2 + y2 + 3x – 5y + 6 = 0 ...........(i)
and 4x2 + 4y2 – 28x + 29 = 0
29
or x 2 + y2 – 7x + = 0. ..........(ii)
4
Let the required circle be x 2 + y2 + 2gx + 2fy + c = 0 ..........(iii)
Since circle (iii) cuts circles (i) and (ii) orthogonally

3  5
 2g   + 2f    = c + 6 or 3g – 5f = c + 6 ...........(iv)
 
2  2

 7 29 29
and 2g    + 2f.0 = c + or – 7g = c + ...........(v)
 2 4 4

5
From (iv) & (v), we get 10g – 5f = –
4
or 40g – 20f = – 5. ..........(vi)
Given line is 3x + 4y = – 1 ..........(vii)
Since centre (– g, – f) of circle (iii) lies on line (vii),
 – 3g – 4g = – 1 .........(viii)
1
Solving (vi) & (viii), we get g = 0, f =
4
29
 from (5), c = –
4
 from (iii), required circle is
1 29
x 2 + y2 + y– =0 or 4(x 2 + y2) + 2y – 29 = 0
2 4

Self Practice Problems :

1. For what value of k the circles x 2 + y2 + 5x + 3y + 7 = 0 and x 2 + y2 – 8x + 6y + k = 0 cut orthogonally.


Ans. – 18

2. Find the equation to the circle which passes through the origin and has its centre on the line
x + y + 4 = 0 and cuts the circle x 2 + y2 – 4x + 2y + 4 = 0 orthogonally.
Ans. 3x2 + 3y2 + 4x + 20y = 0

17. Radical Axis and Radical Centre:


The radical axis of two circles is the locus of points whose powers w.r.t. the two circles are equal. The
equation of radical axis of the two circles S1 = 0 & S2 = 0 is given by
S1  S2 = 0 i.e. 2 (g1  g2) x + 2 (f 1  f 2) y + (c 1  c2) = 0.
The common point of intersection of the radical axes of three circles taken two at a time is called the
radical centre of three circles. Note that the length of tangents from radical centre to the three circles
are equal.
NOTE:
(a) If two circles intersect, then the radical axis is the common chord of the two circles.
(b) If two circles touch each other then the radical axis is the common tangent of the two circles at
the common point of contact.
(c) Radical axis is always perpendicular to the line joining the centres of the two circles.
(d) Radical axis will pass through the mid point of the line joining the centres of the two circles
only if the two circles have equal radii.

RESONANCE 12
(e) Radical axis bisects a common tangent between the two circles.
(f) A system of circles, every two which have the same radical axis, is called a coaxal system.
(g) Pairs of circles which do not have radical axis are concentric.

Example : Find the co-ordinates of the point from which the lengths of the tangents to the following three
circles be equal.
3x2 + 3y2 + 4x – 6y – 1 = 0
2x2 + 2y2 – 3x – 2y – 4 = 0
2x2 + 2y2 – x + y – 1 = 0
Solution : Here we have to find the radical centre of the three circles. First reduce them to standard form
in which coefficients of x 2 and y2 be each unity. Subtracting in pairs the three radical axes are
17 5 3 3
x–y+ =0 ; –x– y– =0
6 3 2 2

11 5 1
– x+ y– = 0.
6 2 6

 16 31 
solving any two, we get the point   ,  which satisfies the third also. This point is called
 21 63 
the radical centre and by definition the length of the tangents from it to the three circles are
equal.

Self Practice Problem :

1. Find the point from which the tangents to the three circles x2 + y2 – 4x + 7 = 0,
2x2 + 2y2 – 3x + 5y + 9 = 0 and x 2 + y2 + y = 0 are equal in length. Find also this length.
Ans. (2, – 1) ; 2.

1 8 . Family of Circles:
(a) The equation of the family of circles passing through the points of intersection of two circles
S1 = 0 & S2 = 0 is : S1 + K S2 = 0
(K  1, provided the coefficient of x2 & y2 in S1 & S2 are same)

(b) The equation of the family of circles passing through the point of intersection of a circle S = 0
& a line L = 0 is given by S + KL = 0.

(c) The equation of a family of circles passing through two given points (x 1, y1) & (x 2, y2) can be
written in the form:

x y 1
(x  x 1) (x  x 2) + (y  y1) (y  y2) + K x1 y1 1 = 0 where K is a parameter..
x2 y2 1

(d) The equation of a family of circles touching a fixed line y  y1 = m (x  x1) at the fixed p o i n t
(x 1, y1) is (x  x 1)2 + (y  y1)2 + K [y  y1  m (x  x 1)] = 0, where K is a parameter.

(e) Family of circles circumscribing a triangle whose sides are given by L1 = 0; L2 = 0 and L3 = 0
is given by; L1L2 +  L2L3 +  L3L1 = 0 provided coefficient of xy = 0 and coefficient of
x 2 = coefficient of y2.

(f) Equation of circle circumscribing a quadrilateral whose side in order are represented by the
lines L1 = 0, L2 = 0, L3 = 0 & L4 = 0 are u L1L3 +  L2L4 = 0 where values of u &  can be found
out by using condition that coefficient of x 2 = coefficient of y2 and coefficient of xy = 0.

RESONANCE 13
Example : Find the equations of the circles passing through the points of intersection of the circles
x 2 + y2 –2x – 4y – 4 = 0 and x 2 + y2 – 10x – 12y + 40 = 0 and whose radius is 4.
Solution : Any circle through the intersection of given circles is S1 + S2 = 0
or (x2 + y2 – 2x – 4y – 4) + l(x2 + y2 – 10x – 12y + 40 ) = 0
(1  5 ) ( 2  6 ) 40  4
or (x2 + y2) – 2 x–2 y+ =0 ...........(i)
1  1  1 

r= g2  f 2  c = 4, given

(1  5 )2 ( 2  6 ) 2 40  4
 16 = + –
(1   )2 (1   )2 1 
16(1 + 2 +  2) = 1 + 10 + 25 2 + 4 + 24 + 36 2 – 40 2 – 40 + 4 + 4
or 16 + 32 + 16 2 = 21 2 – 2 + 9
or 5 2 – 34 – 7 = 0
 ( – 7) (5 + 1) = 0
  = 7, – 1/5
Putting the values of  in (i) the required circles are
2x2 + 2y2 – 18x – 22y + 69 = 0
and x2 + y2 – 2y – 15 = 0

Example : Find the equations of circles which touche 2x – y + 3 = 0 and pass through the points of
intersection of the line x + 2y – 1 = 0 and the circle x 2 + y2 – 2x + 1 = 0.
Solution : The required circle by S + P = 0 is
x 2 + y2 – 2x + 1 +  (x + 2y – 1) = 0
or x 2 + y2 – x (2 – ) + 2y + (1 – ) = 0
centre (– g, – f) is [{2 – )/2, – ]

r= g2  f 2  c

1
= (2   )2 / 4  2  (1   ) =
2 = (/2)
5.
2 5
Since the circle touches the line 2x – y + 3 = 0 therefore perpendicular from centre is equal to
2.[( 2   ) / 2]  (  )  3 
radius = 5.
 5 2


or 5=± .5
2
 =±2
Puttin the values of  in (i) the required circles are
x 2 + y2 + 4y – 1 = 0
x 2 + y2 – 4x – 4y + 3 = 0.

Example : Find the equation of circle pasing through the points A(1, 1) & B(2, 2) and whose radiu is 1.
Solution : Equation of AB is x – y = 0
 equation of circle is
(x – 1) (x – 2) + (y – 1) (y – 2) + (x – y) = 0
or x 2 + y2 + ( – 3)x – ( + 3)y + 4 = 0

(   3) 2 (   3) 2
radius =  4
4 4
But radius = 1 (given)
(   3) 2 (   3) 2
  4 = 1
4 4
or ( – 3) 2 + ( + 3)2 – 16 = 4.
or 2 2 = 2

RESONANCE 14
or =±1
 equation of circle is
x 2 + y2 – 2x – 4y + 4 = 0 Ans.
& x 2 + y2 – 4x – 2y + 4 = 0 Ans.

Example : Find the equation of the circle passing through the point (2, 1) and touching the line
x + 2y – 1 = 0 at the point (3, – 1).
Solution : Equation of circle is
(x – 3) 2 + (y + 1)2 + (x + 2y – 1) = 0
Since it passes through the point (2, 1)
1 + 4 +  (2 + 2 – 1) = 0
5
 =–
3
 circle is
5
(x – 3) 2 + (y + 1)2 – (x + 2y – 1) = 0
3
 3x2 + 3y2 – 23x – 4y + 35 = 0 Ans.

Example : Find the equation of circle circumcscribing the triangle whose sides are 3x – y – 9 = 0,
5x – 3y – 23 = 0 & x + y – 3 = 0.
Solution :

L1L2 + L2L3 + µL1L3 = 0


(3x – y – 9) (5x – 3y – 23) + (5x – 3y – 23) (x + y – 3) + µ (3x – y – 9) (x + y – 3) = 0
(15x2 + 3y2 – 14xy – 114x + 50y + 207) + (5x2 – 3y2 + 2xy – 38x – 14y + 69)
+ µ (3x2 – y2 + 2xy – 18x – 6y + 27) = 0
(5 + 3µ + 15)x + (3 – 3 – µ)y + xy (2 + 2µ – 14) – x (114 + 38 + 18µ) + y(50 – 14 – 6µ)
2 2

+ (207 + 69 + 27µ) = 0 ...........(i)


coefficient of x 2 = coefficient of y2
 5 + 3µ + 15 = 3 – 3 – µ
8 + 4µ + 12 = 0
2 + µ + 3 = 0 ...........(ii)
coefficient of xy = 0
 2 + 2µ – 14 = 0
 + µ – 7 = 0 ..........(iii)
Solving (ii) and (iii), we have
 = – 10, µ = 17
Puting these values of  & µ in equation (i), we get
2x2 + 2y2 – 5x + 11y – 3 = 0

Self Practice Problems :

1. Find the equation of the circle passing through the points of intersection of the circles
x 2 + y2 – 6x + 2y + 4 = 0 and x 2 + y2 + 2x – 4y – 6 = 0 and with its centre on the line y = x.
Ans. 7x2 + 7y2 – 10x – 10y – 12 = 0

2. Find the equation of circle circumcribing the quadrilateral whose sides are 5x + 3y = 9, x = 3y, 2x = y
and x + 4y + 2 = 0.
Ans. 9x2 + 9y2 – 20x + 15y = 0.

RESONANCE 15
Part : (A) Only one correct option

1. If (–3, 2) lies on the circle x2 + y2 + 2gx + 2fy + c = 0, which is concentric with the circle
x 2 + y2 + 6x + 8y – 5 = 0, then c is
(A) 11 (B) –11 (C) 24 (D) none of these

2. The circle x² + y²  6x  10y + c = 0 does not intersect or touch either axis, & the point (1, 4) is inside
the circle. Then the range of possible values of c is given by:
(A) c > 9 (B) c > 25 (C) c > 29 (D) 25 < c < 29

3. The length of the tangent drawn from any point on the circle x² + y² + 2gx + 2fy + p = 0 to the circle
x² + y² + 2gx + 2fy + q = 0 is:
(A) qp (B) pq (C) qp (D) none

4. The angle between the two tangents from the origin to the circle (x  7)² + (y + 1)² = 25 equals
  
(A) (B) (C) (D) none
4 3 2

5. The circumference of the circle x 2 + y2  2x + 8y  q = 0 is bisected by the circle


x 2 + y2 + 4x + 12y + p = 0, then p + q is equal to:
(A) 25 (B) 100 (C) 10 (D) 48

 1  1  1  1
6. If  a,  ,  b ,  ,  c,  &  d ,  are four distinct points on a circle of radius 4 units then, abcd is equal
 a  b  c   d
to:
(A) 4 (B) 16 (C) 1 (D) none

7. The centre of a circle passing through the points (0, 0), (1, 0) & touching the circle x 2 + y2 = 9 is :
 3 1 1 3 1 
(C)  , 
1 1
(A)  ,  (B)  ,  (D)  ,  2 
2 2 2 2  2 2 2 

8. Two thin rods AB & CD of lengths 2a & 2b move along OX & OY respectively, when ‘O’ is the origin. The
equation of the locus of the centre of the circle passing through the extremities of the two rods is:
(A) x² + y² = a² + b² (B) x²  y² = a²  b² (C) x² + y² = a²  b² (D) x²  y² = a² + b²

9. The value of 'c' for which the set,


{(x, y)x 2 + y2 + 2x  1}  {(x, y)x  y + c  0} contains only one point in common is:
(A) ( ,  1]  [3, ) (B) { 1, 3} (C) {3} (D) { 1 }

10. Let x & y be the real numbers satisfying the equation x2  4x + y2 + 3 = 0. If the maximum and minimum
values of x 2 + y2 are M & m respectively, then the numerical value of M  m is:
(A) 2 (B) 8 (C) 15 (D) none of these

11. A line meets the coordinate axes in A & B. A circle is circumscribed about the triangle OAB. If d1 & d2
are the distances of the tangent to the circle at the origin O from the points A and B respectively, the
diameter of the circle is:
2d1  d 2 d1  2d 2 d1d2
(A) (B) (C) d1 + d2 (D) d  d
2 2 1 2

12. The distance between the chords of contact of tangents to the circle;
x² + y² + 2gx + 2fy + c = 0 from the origin & the point (g, f) is:

g2  f 2  c g2  f 2  c g2  f 2  c
(A) g f
2 2 (B) (C) (D)
2 2 g2  f 2 2 g2  f 2

RESONANCE 16
13. If tangent at (1, 2) to the circle c1: x2 + y2 = 5 intersects the circle c 2: x 2 + y2 = 9 at A & B and tangents
at A & B to the second circle meet at point C, then the coordinates of C are:
 9 18   9 18 
(A) (4, 5) (B)  ,  (C) (4,  5) (D)  , 
 15 5  5 5 

14. The locus of the mid points of the chords of the circle x² + y² + 4x  6y  12 = 0 which subtend an angle

of radians at its circumference is:
3
(A) (x  2)² + (y + 3)² = 6.25 (B) (x + 2)² + (y  3)² = 6.25
(C) (x + 2)² + (y  3)² = 18.75 (D) (x + 2)² + (y + 3)² = 18.75

15. If the length of a common internal tangent to two circles is 7, and that of a common external tangent is
11, then the product of the radii of the two circles is:
(A) 36 (B) 9 (C) 18 (D) 4

16. Two circles whose radii are equal to 4 and 8 intersect at right angles. The length of their common chord
is:
16 8 5
(A) (B) 8 (C) 4 6 (D)
5 5

17. A circle touches a straight line lx + my + n = 0 & cuts the circle x² + y² = 9 orthogonally. The locus of
centres of such circles is:
(A) (lx + my + n)² = (l² + m²) (x² + y²  9) (B) (lx + my  n)² = (l² + m²) (x² + y²  9)
(C) (lx + my + n)² = (l² + m²) (x² + y² + 9) (D) none of these

18. If a circle passes through the point (a, b) & cuts the circle x² + y² = K² orthogonally, then the equation
of the locus of its centre is:
(A) 2ax + 2by  (a² + b² + K²) = 0 (B) 2ax + 2by  (a²  b² + K²) = 0
(C) x² + y²  3ax  4by + (a² + b²  K²) = 0 (D) x² + y²  2ax  3by + (a²  b²  K²) = 0

19. The circle x² + y² = 4 cuts the circle x² + y² + 2x + 3y  5 = 0 in A & B. Then the equation of the circle on
AB as a diameter is:
(A) 13(x² + y²)  4x  6y  50 = 0 (B) 9(x² + y²) + 8x  4y + 25 = 0
(C) x² + y²  5x + 2y + 72 = 0 (D) none of these

20. The length of the tangents from any point on the circle 15x2 + 15y2 – 48x + 64y = 0 to the two circles
5x2 + 5y2 – 24x + 32y + 75 = 0 and 5x2 + 5y2 – 48x + 64y + 300 = 0 are in the ratio
(A) 1 : 2 (B) 2 : 3 (C) 3 : 4 (D) none of these

21. The normal at the point (3, 4) on a circle cuts the circle at the point (–1, –2). Then the equation of the
circle is
(A) x 2 + y2 + 2x – 2y – 13 = 0 (B) x2 + y2 – 2x – 2y – 11 = 0
(C) x + y – 2x + 2y + 12 = 0
2 2 (D) x2 + y2 – 2x – 2y + 14 = 0

22. The locus of poles whose polar with respect to x² + y² = a² always passes through (K, 0) is:
(A) Kx  a² = 0 (B) Kx + a² = 0 (C) Ky + a² = 0 (D) Ky  a² = 0

23. If two distinct chords, drawn from the point (p, q) on the circle x 2 + y2 = px + q) (where pq  0) are
bisected by the x-axis, then [IIT - 1999]
(A) p2 = q2 (B) p2 = 8q2 (C) p2 < 8q2 (D) p2 > 8q2

24. The triangle PQR is inscribed in the circle x2 + y2 = 25. If Q and R have co-ordinates (3, 4) and (–4, 3)
respectively, the  QPR is equal to [IIT - 2000]
   
(A) (B) (C) (D)
2 3 4 6

25. Let PQ and RS be tangents at the extremities of diameter PR of a circle of radius r. If PS and RQ
intersect at a point X on the circumference of the circle, then 2r equals [IIT- 2001]

PQ  RS 2PQ  RS PQ 2  RS 2
(A) PQ . RS (B) (C) (D)
2 PQ  RS 2

RESONANCE 17
26. Let AB be a chord of the circle x 2 + y2 = r2 subtending a right angle at the centre. Then, locus of the
centroid of the triangle PAB as P moves on the circles is [IIT- 2001]
(A) a parabola (B) a circle (C) an ellipse (D) a pair of straight line

27. If the tangent at the point P on the circle x 2 + y 2 + 6x + 6y = 2 meets the straight line
5x – 2y + 6 = 0 at a point Q on the y-axis, then the length of PQ is [IIT- 2002]
(A) 4 (B) 2 5 (C) 5 (D) 3 5

28. Tangent to the curve y = x2 + 6 at a point P(1, 7) touches the circle x2 + y2 + 16x + 12y + c =0 at a point
Q. Then, the coordinates of Q are [IIT- 2005]
(A) (– 6, –11) (B) (– 9, – 13) (C) (– 10, – 15) (D) (– 6, – 7)

Part : (B) May have more than one options correct

 7 
29. A circle passes through the point  3,  and touches the line pair x  y  2x + 1 = 0. The
2 2
 2 
coordinates of the centre of the circle are:
(A) (4, 0) (B) (5, 0) (C) (6, 0) (D) (0, 4)

x y
30. The equation of the circle which touches both the axes and the line + = 1 and lies in the first
3 4
quadrant is (x – c)2 + (y – c)2 = c 2 where c is
(A) 1 (B) 2 (C) 4 (D) 6

1. If y = 2x is a chord of the circle x2 + y2 – 10x = 0, find the equation of a circle with this chord as
diameter.

2. Find the points of intersection of the line x – y + 2 = 0 and the circle 3x2 + 3y2 – 29x – 19y + 56 = 0. Also
determine the length of the chord intercepted.

3. Show that two tangents can be drawn from the point (9, 0) to the circle x 2 + y2 = 16; also find the
equation of the pair of tangents and the angle between them.

4. Given the three circles x2 + y2 – 16x + 60 = 0, 3x2 + 3y2 – 36x + 81 = 0 and x2 + y2 – 16x – 12y + 84 = 0,
find (1) the point from which the tangents to them are equal in length, and (2) this length.

5. On the line joining (1, 0) and (3, 0) an equilateral triangle is drawn having its vertex in the first quadrant.
Find the equation to the circles described on its sides as diameter.

6. One of the diameters of the circle circumscribing the rectangle ABCD is 4 y = x + 7. If A & B are the
points (3, 4) & (5, 4) respectively. Then find the area of the rectangle.

7. Let A be the centre of the circle x² + y²  2x  4y  20 = 0. Suppose that the tangents at the points B
(1, 7) & D (4,  2) on the circle meet at the point C. Find the area of the quadrilateral ABCD.
8. Let a circle be given by 2x (x  a) + y (2y  b) = 0, (a  0, b  0). Find the condition on a & b if two
 b
chords, each bisected by the xaxis, can be drawn to the circle from  a ,  .
 2
9. Find the equation of the circle which cuts each of the circles, x² + y² = 4, x² + y²  6x  8y + 10 = 0
& x² + y² + 2x  4y  2 = 0 at the extremities of a diameter.

10. Find the equation and the length of the common chord of the two circles given by the equations,
x 2 + y2 + 2 x + 2 y + 1 = 0 & x 2 + y2 + 4 x + 3 y + 2 = 0.

11. Find the values of a for which the point (2a, a + 1) is an interior point of the larger segment of the circle
x 2 + y2  2x  2y  8 = 0 made by the chord whose equation is x  y + 1 = 0.

12. If 4l²  5m² + 6l + 1 = 0. Prove that lx + my + 1 = 0 touches a definite circle. Find the centre & radius
of the circle.

RESONANCE 18
13. A circle touches the line y = x at a point P such that OP = 4 2 where O is the origin. The circle
contains the point (10, 2) in its interior and the length of its chord on the line x + y = 0 is 6 2 . Find
the equation of the circle.

14. Show that the equation of a straight line meeting the circle x2 + y2 = a2 in two points at equal distances
d2
'd' from a point (x1, y1) on its circumference is xx 1 + yy1  a2 + = 0.
2

15. For each natural number k, let Ck denote the circle with radius k centimetres and centre at the origin.
On the circle Ck, -particle moves k centrimetres in the counter - clockwise direction. After completing
its motion on Ck, the particle moves to Ck + 1 in the radial direction. The motion of the particle continues
in this manner. The particle starts at (1, 0). If the particle crosses the positive direction of the x-axis for
the first time on the circle Cn then n = __________. [IIT 1997]

16.  
Let C be any circle with centre 0, 2 . Prove that at the most two rational point can be there on C.
(A rational point is a point both of whose coordinate are rational numbers). [IIT - 1997]

17. Let T 1, T 2 be two tangents drawn from ( 2, 0) onto the circle C: x2 + y2 = 1. Determine the circles
touching C and having T1, T2 as their pair of tangents. Further, find the equations of all possible common
tangents to these circles, when taken two at a time. [IIT - 1999]

18. Let C1 and C2 be two circles with C2 lying inside C1. A circle C lying inside C1 touches C1 internally C1
internally and C2 externally. Identify the locus of the centre of C. [IIT 2001]

19. Circles with raddi 3, 4 and 5 touch each other externally. If P is the point of intersection of tangents to
these circles at their points of contact, find the distance of P from the points of contact.
[IIT - 2005]

EXERCISE # 1 6. 32 sq. unit 7. 75 sq. units 8. (a² > 2b²)

1. B 2. D 3. A 4. C 5. C 6. C 7. D 9. x² + y²  4x  6y  4 = 0

8. B 9. D 10. B 11. C 12. C 13. D 14. B 2


10. 2 x + y + 1 = 0, 11. a  (0, 9/5)
15. C 16. A 17. A 18. A 19. A 20. A 21. B 5

22. A 23. D 24. C 25. A 26. B 27. C 28. D 12. Centre  (3, 0), (radius) = 5
29. AC 30. AD 13. x 2 + y2 + 18 x  2 y + 32 = 0

EXERCISE # 2 15. 7

 4 1
2

1. x + y – 2x – 4y = 0 2. (1, 3), (5, 7), 4 2 17. c 1: (x  4) 2 + y2 = 9; c 2:  x   + y2 =


 3 9
2 2

 8 65  common tangent between c & c 1: T 1 = 0;


  T 2 = 0 and x  1 = 0; common tangent between
3. 16x – 65y – 288x + 1296 = 0, tan  49 
2 2 –1

  c & c2: T 1 = 0; T 2 = 0 and x + 1 = 0; common


tangent between c1 & c 2: T 1 = 0; T 2 = 0 and
 33  1 5  4
4.  , 2 ;  x   where T : x 
 4  4 y=± 3y+2=0
39  5 1

5. x 2 + y2  3 x  and T 2: x + 3y+2=0
3 y + 2 = 0;

x 2 + y2  5 x  3 y + 6 = 0; 18. ellipse 19. 5


x2 + y2  4x + 3 = 0

RESONANCE 19
Parabola

1. Conic Sections:
A conic section, or conic is the locus of a point which moves in
a plane so that its distance from a fixed point is in a constant

There are things ratio to its perpendicular distance from a fixed straight line.
• The fixed point is called the Focus.
which seem • The fixed straight line is called the Directrix.
• The constant ratio is called the Eccentricity denoted by e.
incredible to most
• The line passing through the focus & perpendicular to the directrix
men who have not is called the Axis.
• A point of intersection of a conic with its axis is called a Vertex.
studied
mathematics. 2. Section of right circular cone by different
planes
D Mac Hal e A right circular cone is as shown in the

The desire to
understand the
world and the
desire to reform it
are the two great (i) Section of a right circular cone
by a plane passing through its
engines of progress. vertex is a pair of straight lines
passing through the vertex as
Marri age and Mor al s shown in the

(ii) Section of a righ t


circular cone by a plane
parallel to its base is a
circle as shown in the
figure  3.

RESONANCE 20
(iii) Section of a right circular cone by a plane parallel to a generator of the cone is a parabola as shown in
the

(iv) Section of a right circular cone by a plane neither parallel to any generator of the cone nor perpendicular
or parallel to the axis of the cone is an ellipse or hyperbola as shown in the figure  5 & 6.

Figure -5 Figure -6

3D View :

3. General equation of a conic: Focal directrix property:


The general equation of a conic with focus (p, q) & directrix lx + my + n = 0 is:
(l2 + m 2) [(x  p)2 + (y  q)2] = e2 (lx + my + n)2  ax2 + 2hxy + by2 + 2gx + 2fy + c = 0

4. Distinguishing various conics :


The nature of the conic section depends upon the position of the focus S w.r.t. the directrix & also upon
the value of the eccentricity e. Two different cases arise.
Case (I) When The Focus Lies On The Directrix.
In this case   abc + 2fgh  af 2  bg2  ch2 = 0 & the general equation of a conic represents a pair of
straight lines if:
e > 1  h2 > ab the lines will be real & distinct intersecting at S.
e = 1  h2 > ab the lines will coincident.
e < 1  h2 < ab the lines will be imaginary.
Case (II) When The Focus Does Not Lie On Directrix.
a parabola an ellipse a hyperbola rectangular hyperbola
e = 1;   0, 0 < e < 1;   0; e > 1;   0; e > 1;   0
h² = ab h² < ab h² > ab h² > ab; a + b = 0

RESONANCE 21
PARABOLA

5. Definition and Terminology


A parabola is the locus of a point, whose distance
from a fixed point (focus) is equal to perpendicular
distance from a fixed straight line (directrix).
Four standard forms of the parabola are
y² = 4ax; y² =  4ax; x² = 4ay; x² =  4ay
For parabola y2 = 4ax:
(i) Vertex is (0, 0) (ii) focus is (a, 0)
(iii) Axis is y = 0 (iv) Directrix is x + a = 0
Focal Distance: The distance of a point on the parabola from the focus.
Focal Chord : A chord of the parabola, which passes through the focus.
Double Ordinate: A chord of the parabola perpendicular to the axis of the symmetry.
Latus Rectum: A double ordinate passing through the focus or a focal chord perpendicular to the axis
of parabola is called the Latus Rectum (L.R.).
For y² = 4ax.  Length of the latus rectum = 4a.
 ends of the latus rectum are L(a, 2a) & L’ (a,  2a).
NOTE :
(i) Perpendicular distance from focus on directrix = half the latus rectum.
(ii) Vertex is middle point of the focus & the point of intersection of directrix & axis.
(iii) Two parabolas are said to be equal if they have the same latus rectum.

Examples :
Find the equation of the parabola whose focus is at (– 1, – 2) and the directrix the line
x – 2y + 3 = 0.
Solution.
Let P(x, y) be any point on the parabola whose focus is S(– 1, – 2) and the directrix
x – 2y + 3 = 0. Draw PM perpendicular to directrix x – 2y + 3 = 0. Then by definition,
SP = PM
 SP2 = PM2

 x  2y  3 
2

 (x + 1) + (y + 2) = 
2 2 
 1 4 
 5 [(x + 1)2 + (y + 2)2] = (x – 2y + 3) 2
 5(x2 + y2 + 2x + 4y + 5) = (x2 + 4y2 + 9 – 4xy + 6x – 12y)
 4x2 + y2 + 4xy + 4x + 32y + 16 = 0
This is the equation of the required parabola.

Example :
Find the vertex, axis, focus, directrix, latusrectum of the parabola, also draw their rough sketches.
4y2 + 12x – 20y + 67 = 0
Solution.
The given equation is
67
4y2 + 12x – 20y + 67 = 0  y2 + 3x – 5y + =0
4

5 5
2 2
67 67
 y2 – 5y = – 3x –  y2 – 5y +   = – 3x – +  
4 2 4 2

 5  5  7
2 2
42
  y   = – 3x –  y   = – 3 x   ....(i)
 2  4  2   2

RESONANCE 22
7 5
Let x=X– ,y=Y+ ....(ii)
2 2
Using these relations, equation (i) reduces to
Y2 = – 3X ....(iii)
This is of the form Y2 = – 4aX. On comparing, we get 4a = 3  a = 3/4.
Vertex - The coordinates of the vertex are (X = 0, Y = 0)
So, the coordinates of the vertex are
 7 5
 ,  [Putting X = 0, Y = 0 in (ii)]
 2 2
Axis
The equation of the axis of the parabola is Y = 0.
So, the equation of the axis is
5
y= [Putting Y = 0 in (ii)]
2
Focus-
The coordinates of the focus are (X = –a, Y = 0)
i.e. (X = – 3/4, Y = 0).
So, the coordinates of the focus are
(–17/4, 5/2) [Putting X = 3/4 in (ii)]
Directrix -
3
The equation of the directrix is X = a i.e. X = .
4
So, the equation of the directrix is
11
x=– [Putting X = 3/4 in (ii)]
4
Latusrectum -
The length of the latusrectum of the given parabola is 4a = 3.

Self Practice Problems

1. Find the equation of the parabola whose focus is the point (0, 0)and whose directrix is the straight line
3x – 4y + 2 = 0.
Ans. 16x2 + 9y2 + 24xy – 12x + 16y – 4 = 0

2. Find the extremities of latus rectum of the parabola y = x2 – 2x + 3.


1 9 3 9
Ans.  ,   , 
2 4 2 4

3. Find the latus rectum & equation of parabola whose vertex is origin & directrix is x + y = 2.
Ans. 4 2 , x2 + y2 – 2xy + 8x + 8y = 0

4. Find the vertex, axis, focus, directrix, latusrectum of the parabola y2 – 8y – x + 19 = 0. Also draw their
roguht sketches.

Ans.

5. Find the equation of the parabola whose focus is (1, – 1) and whose vertex is (2, 1). Also find its axis
and latusrectum.
Ans. (2x – y – 3) 2 = – 20 (x + 2y – 4), Axis 2x – y – 3 = 0. LL = 4 5 .

RESONANCE 23
6. Parametric Representation:
The simplest & the best form of representing the coordinates of a point on the parabola is (at², 2at)
i.e. the equations x = at² & y = 2at together represents the parabola y² = 4ax, t being the parameter.

Example :
Find the parametric equation of the parabola (x – 1)2 = –12 (y – 2)
Solution.
 4a = – 12  a = 3, y – 2 = at2
x – 1 = 2 at  x = 1 – 6t, y = 2 – 3t2

Self Practice Problems

1. Find the parametric equation of the parabola x2 = 4ay


Ans. x = 2at, y = at2.

7. Position of a point Relative to a Parabola:


The point (x1 y1) lies outside, on or inside the parabola y² = 4ax according as the expression y1²  4ax1
is positive, zero or negative.

Example :
Check weather the point (3, 4) lies inside or outside the paabola y2 = 4x.

Solution.
y2 – 4x = 0
 S1 y12 – 4x1 = 16 – 12 = 4 > 0
 (3, 4) lies outside the parabola.

Self Practice Problems

1. Find the set of value's of for which (, – 2 – ) lies inside the parabola y2 + 4x = 0.
Ans. a  (– 4 – 2 3 , – 4 + 2 3 )

8. Line & a Parabola:


The line y = mx + c meets the parabola y² = 4ax in two points real, coincident or imaginary according
as a  c m  condition of tangency is, c = a/m.

Length of the chord intercepted by the parabola on the line y = m x + c is:

 4  a (1  m 2 ) (a  m c) .
 2
m 
NOTE :
1. The equation of a chord joining t1 & t2 is 2x  (t1 + t2) y + 2 at1 t2 = 0.

2. If t1 & t2 are the ends of a focal chord of the parabola y² = 4ax then t1t2 = 1. Hence the

coordinates at the extremities of a focal chord can be taken as (at², 2at) &  a ,  2 a 
 t2 t 

3. Length of the focal chord making an angle  with the x axis is 4acosec² 

RESONANCE 24
Example :
Discuss the position of line y = x + 1 with respect to parabolas y2 = 4x.
Solution.
Solving we get (x + 1) 2 = 4x  (x – 1)2 = 0
so y = x + 1 is tangent to the parabola.

Example :
Prove that focal distance of a point P(at2, 2at) on parabola y2 = 4ax (a > 0) is a(1 + t2).
Solution.
 PS = PM
= a + at2
PS = a (1 + t2).

Example :
If t1, t2 are end points of a focal chord then show that t1 t2 = –1.
Solution.
Let parabola is y2 = 4ax
since P, S & Q are collinear
 mPQ = mPS

2 2t 1
 t1  t 2 =
t1  1
2

 t12 – 1 = t12 + t1t2


 t1 t2 = – 1

Example :
If the endpoint t1, t2 of a chord satisfy the relation t1 t2 = k (const.) then prove that the chord always passes
through a fixed point. Find the point?
Solution.
Equation of chord joining (at12, 2at1) and (at22, 2at2) is
2
y – 2at1 = t  t (x – at12)
1 2

(t1 + t2) y – 2at12 – 2at1t2 = 2x – 2at12


2
y = t  t (x + ak) ( t1t2 = k)
1 2

 This line passes through a fixed point (– ak, 0).

Self Practice Problems

1. If the line y = 3x + intersect the parabola y2 = 4x at two distinct point's then set of value's of '' is
Ans. (– , 1/3)

2. Find the midpoint of the chord x + y = 2 of the parabola y2 = 4x.


Ans. (4, – 2)

3. If one end of focal chord of parabola y2 = 16x is (16, 16) then coordinate of other end is.
Ans. (1, – 4)

4. If PSQ is focal chord of parabola y2 = 4ax (a > 0), where S is focus then prove that
1 1 1
+ = .
PS SQ a

 1
2

5. Find the length of focal chord whose one end point is ‘t’. [Ans. a t   ]
 t

RESONANCE 25
9. Tangents to the Parabola y² = 4ax:
(i) y y1 = 2 a (x + x 1) at the point (x 1, y1) ;

(ii) y = mx +
a  a 2 a 
(m  0) at  2 ,
m m m
(iii) t y = x + a t² at (at², 2at).
NOTE : Point of intersection of the tangents at the point t1 & t2 is [ at1 t2, a(t1 + t2) ].
Example :
a
Prove that the straight line y = mx + c touches the parabola y2 = 4a (x + a) if c = ma +
m
Solution.
Equation of tangent of slope ‘m’ to the parabola y2 = 4a(x + a) is

a  1
y = m(x + a) +  y = mx + a  m  
m  m
but the given tangent is y = mx +c
a
 c = am +
m
Example :
A tangent to the parabola y2 = 8x makes an angel of 45° with the straight line y = 3x + 5. Find its
equation and its point of contact.
Solution.
Slope of required tangent’s are
3 1
m=
1 3

1
m 1 = – 2, m2 =
2
 Equation of tangent of slope m to the parabola y2 = 4ax is
a
y = mx + .
m
1 
 tangent’s y = – 2x – 1 at  ,  2 
2 

1
y= x + 4 at (8, 8)
2
Example :
Find the equation to the tangents to the paabola y2 = 9x which goes through the point (4, 10).
Solution.
Equation of tangent to parabola y2 = 9x is
9
y = mx +
4m
Since it passes through (4, 10)
9
 10 = 4m +  16 m 2 – 40 m + 9 = 0
4m
1 9
m= ,
4 4
x 9
 equation of tangent’s are y= +9 & y= x + 1.
4 4

RESONANCE 26
Example :
Find the equations to the common tangents of the parabolas y2 = 4ax and x 2 = 4by.
Solution.
Equation of tangent to y2 = 4ax is
a
y = mx + ........(i)
m
Equation of tangent to x 2 = 4by is
b
x = m 1y +
m1

1 b
 y= x– ........(ii)
m1 (m1 ) 2
for common tangent, (i) & (ii) must represent same line.
1 a b
 m1 = m & = – 2
m m1

 a
1/ 3
a
 = – bm 2  m =  
m  b
 equation of common tangent is

 a  b
1/ 3 1/ 3

y =   x + a   .
 b  a

Self Practice Problems


1. Find equation tangent to parabola y2 = 4x whose intercept on y–axis is 2.
x
Ans. y 2
2
2. Prove that perpendicular drawn from focus upon any tangent of a parabola lies on the tangent at the vertex.

3. Prove that image of focus in any tangent to parabola lies on its directrix.

4. Prove that the area of triangle formed by three tangents to the parabola y2 = 4ax is half the area of triangle
formed by their points of contacts.

1 0 . Normals to the parabola y² = 4ax :


y1
(i) y  y1 =  (x  x 1) at (x 1, y1) ;
2a
(ii) y = mx  2am  am 3 at (am 2,  2am)
(iii) y + tx = 2at + at3 at (at2, 2at).
NOTE :
(i) Point of intersection of normals at t1 & t2 are, a (t 12 + t 22 + t1t2 + 2);  a t1 t2 (t1 + t2).
(ii) If the normals to the parabola y² = 4ax at the point t1, meets the parabola again at the point

 2
t2, then t2 =   t1   .
 t1 
(iii) If the normals to the parabola y² = 4ax at the points t1 & t2 intersect again on the parabola at the
point 't3' then t1 t2 = 2; t3 =  (t1 + t2) and the line joining t1 & t2 passes through a fixed point
(2a, 0).

RESONANCE 27
Example :
2
If the normal at point ‘t1’ intersects the parabola again at ‘t2’ then show that t2 = –t1 –
t1
Solution.
2
Slope of normal at P = – t1 and slope of chord PQ =
t1  t 2

2
 – t1 =
t1  t 2

2 2
t1 + t2 = –  t2 = – t 1 – .
t1 t1
Example :
If the normals at points t1, t2 meet at the point t3 on the parabola then prove that
(i) t 1 t2 = 2 (ii) t1 + t2 + t3 = 0
Solution.
Since normal at t1 & t2 meet the curve at t3
2
 t3 = – t 1 – .....(i)
t1

2
t3 = – t2 – .....(ii)
t2
 (t12 + 2) t2 = t1 (t22 + 2)
t1t2 (t1 – t2) + 2 (t2 – t1) = 0
 t1  t 2 , t 1 t2 = 2 ......(iii)
Hence (i) t1 t2 = 2
from equation (i) & (iii), we get
t3 = – t1 – t2
Hence (ii) t1 + t2 + t3 = 0
Example :
Find the locus of the point N from which 3 normals are drawn to the parabola y2 = 4ax are such that
(i) Two of them are equally inclined to x-axis
(ii) Two of them are perpendicular to each other
Solution.
Equation of normal to y2 = 4ax is
y = mx – 2am – am3
Let the normal is passes through N(h, k)
 k = mh – 2am – am3  am3 + (2a – h) m + k = 0
For given value’s of (h, k) it is cubic in ‘m’.
Let m1, m2 & m3 are root’s
 m1 + m2 + m3 = 0 ......(i)
2a  h
m 1m 2 + m 2m 3 + m 3m 1 = ......(ii)
a

k
m 1m 2m 3 = – ......(iii)
a
(i) If two nromal are equally inclined to x-axis, then m1 + m2 = 0
 m3 = 0  y=0
(ii) If two normal’s are perpendicular
 m1 m2 = – 1
k
from (3) m3 = .....(iv)
a

RESONANCE 28
k 2a  h
from (2) –1+ (m1 + m2) = .....(v)
a a

k
from (1) m1 + m2 = – .....(vi)
a
from (5) & (6), we get

k2 h
–1– =2–
a a
y2 = a(x – 3a)
Self Practice Problems
1. Find the points of the parabola y2 = 4ax at which the normal is inclined at 30° to the axis.

a 2a   a 2a 
 , ,  , 
Ans. 3   
 3  3 3 

2. If the normal at point P(1, 2) on the parabola y2 = 4x cuts it again at point Q then Q = ?
Ans. (9, – 6)
3. Find the length of normal chord at point ‘t’ to the parabola y2 = 4ax.
3
4a( t 2  1) 2
Ans. 
t2
4. If normal chord at a point 't' on the parabola y2 = 4ax subtends a right angle at the vertex then prove that
t2 = 2
5. Prove that the chord of the parabola y2 = 4ax, whose equation is y – x 2 + 4a 2 = 0, is a normal to
the curve and that its length is 6 3a .
6. If the normals at 3 points P, Q & R are concurrent, then show that
(i) The sum of slopes of normals is zero, (ii) Sum of ordinates of points P, Q, R is zero
(iii) The centroid of PQR lies on the axis of parabola.

1 1 . Pair of Tangents:
The equation to the pair of tangents which can be drawn from any point (x1, y1) to the parabola y² = 4ax
is given by: SS1 = T² where :
S  y²  4ax ; S1 = y1²  4ax1 ; T  y y1  2a(x + x 1).
Example :
Write the equation of pair of tangents to the parabola y2 = 4x drawn from a point P(–1, 2)
Solution.
We know the equation of pair of tangents are given by SS1 = T²
 (y2 – 4x) (4 + 4) = (2y + 2 (x – 1))2
 8y2 – 32x = 4y2 + 4x2 + 4 + 8xy – 8y – 8x
 y2 – x2 – 2xy – 6x + 2y = 1
Example :
Find the focus of the point P from which tangents are drawn to parabola y2 = 4ax having slopes m1, m2 such
that
(i) m1 + m2 = m0 (const) (ii) 1 + 2 = 0 (const)
Sol. Equation of tangent to y2 = 4ax, is
a
y = mx +
m
Let it passes through P(h, k)
 m2h – mk + a = 0

RESONANCE 29
k
(i) m1 + m2 = m0 =  y = m 0x
h

m1  m 2 k /h
(ii) tan0 = 1  m m =
1 2 1 a / h
 y = (x – a) tan0

Self Practice Problem

1. If two tangents to the parabola y2 = 4ax from a point P make angles 1 and 2 with the axis of the parabola,
then find the locus of P in each of the following cases.
(i) tan21 + tan22 =  (a constant)
(ii) cos 1 cos 2 =  (a constant)
Ans. (i) y2 – 2ax = x2 , (ii) x2 = 2 {(x – a)2 + y2}

1 2 . Director Circle:
Locus of the point of intersection of the perpendicular tangents to a curve is called the Director Circle.
For parabola y2 = 4ax it’s equation is x + a = 0 which is parabola’s own directrix.

1 3 . Chord of Contact:
Equation to the chord of contact of tangents drawn from a point P(x1, y1) is
yy1 = 2a (x + x1).
NOTE : The area of the triangle formed by the tangents from the point (x1, y1) & the chord of contact is
(y1²  4ax 1)3/2 ÷ 2a.

Example :
Find the length of chord of contact of the tangents drawn from point (x1, y1) to the parabola y2 = 4ax.
Solution.
Let tangent at P(t1) & Q(t2) meet at (x1, y1)
 at1t2 = x1 & a(t1 + t2) = y1

 PQ = (at 12  at 22 )2  (2a( t1  t 2 ))2

=a (( t1  t 2 )2  4t 1t 2 )((t1  t 2 )2  4)

( y12  4ax 1 )( y12  4a 2 )


=
a2
Example :
If the line x – y – 1 = 0 intersect the parabola y2 = 8x at P & Q, then find the point of intersection of tangents
at P & Q.
Solution.
Let (h, k) be point of intersection of tangents then chord of contact is
yk = 4(x + h)
4x – yk + 4h = 0 .....(i)
But given is
x–y–1=0
4 k 4h
 = =
1 1 1
 h = – 1, k = 4
 point (–1, 4)

RESONANCE 30
Example :
Find the locus of point whose chord of contact w.r.t to the parabola y2 = 4bx is the tangents of the parabola
y2 = 4ax.
Solution.
a
Equation of tangent to y2 = 4ax is y = mx + ......(i)
m
Let it is chord of contact for parabola y2 = 4bx w.r.t. the point P(h, k)
 Equation of chord of contact is
yk = 2b(x + h)
2b 2bh
y= x+ .....(ii)
k k
From (i) & (ii)

2b a 2bh 4b 2h
m= , =  a=
k m k k2
locus of P is

4b 2
y2 = x.
a
Self Practice Problems

1. Prove that locus of a point whose chord of contact w.r.t. parabola passes through focus is directrix

2. If from a variable point ‘P’ on the line x – 2y + 1 = 0 pair of tangent’s are drawn to the parabola
y2 = 8x then prove that chord of contact passes through a fixed point, also find that point.
Ans. (1, 8)

1 4 . Chord with a given middle point:


Equation of the chord of the parabola y² = 4ax whose middle point is
2a
(x 1, y1) is y  y1 = (x  x 1)  T = S1
y1
Example :
Find the locus of middle point of the chord of the parabola y2 = 4ax which pass through a given point (p, q).
Solution.
Let P(h, k) be the mid point of chord of parabola y2 = 4ax,
so equation of chord is yk – 2a(x + h) = k2 – 4ah.
Since it passes through (p, q)
 qk – 2a (p + h) = k2 – 4ah
 Required locus is
y2 – 2ax – qy + 2ap = 0.

Example :
Find the locus of middle point of the chord of the parabola y2 = 4ax whose slope is ‘m’.
Solution.
Let P(h, k) be the mid point of chord of parabola y2 = 4ax,
so equation of chord is yk – 2a(x + h) = k2 – 4ah.
2a
but slope = =m
k
2a
 locus is y =
m

RESONANCE 31
Self Practice Problems

1. Find the equation of chord of parabola y2 = 4x whose mid point is (4, 2).
Ans. x–y–2=0

2. Find the locus of mid - point of chord of parabola y2 = 4ax which touches the parabola x2 = 4by.
Ans. y (2ax – y2) = 4a2b

1 5 . Important Highlights:
(i) If the tangent & normal at any point ‘P’ of the parabola intersect the axis at T & G then
ST = SG = SP where ‘S’ is the focus. In other words the tangent and the normal at a point P on
the parabola are the bisectors of the angle between the focal radius SP & the perpendicular
from P on the directrix. From this we conclude that all rays emanating from S will become
parallel to the axis of theparabola after reflection.
(ii) The portion of a tangent to a parabola cut off between the directrix & the curve subtends a right
angle at the focus.
(iii) The tangents at the extremities of a focal chord intersect at right angles on the directrix, and
hence a circle on any focal chord as diameter touches the directrix. Also a circle on any focal
radii of a point P (at 2, 2at) as diameter touches the tangent at the vertex and intercepts a chord
of length a 1  t on a normal at the point P..
2

(iv) Any tangent to a parabola & the perpendicular on it from the focus meet on the tangent at the
vertex.
(v) If the tangents at P and Q meet in T, then:
 TP and TQ subtend equal angles at the focus S.
 ST 2 = SP. SQ &  The triangles SPT and STQ are similar.
(vi) Semi latus rectum of the parabola y² = 4ax, is the harmonic mean between segments of any
focal chord of the parabola.
(vii) The area of the triangle formed by three points on a parabola is twice the area of the triangle
formed by the tangents at these points.
(viii) If normal are drawn from a point P(h, k) to the parabola y2 = 4ax then
k = mh  2am  am 3 i.e. am 3 + m(2a  h) + k = 0.
2a  h k
m1 + m2 + m3 = 0 ; m 1m 2 + m 2m 3 + m 3m 1 = ; m 1 m2 m3 =  .
a a
Where m 1, m 2, & m 3 are the slopes of the three concurrent normals. Note that
 algebraic sum of the slopes of the three concurrent normals is zero.
 algebraic sum of the ordinates of the three conormal points on the parabola is zero
 Centroid of the  formed by three conormal points lies on the xaxis.
 Condition for three real and distinct normals to be drawn froma point P (h, k) is
4
h > 2a & k 2 <
27a (h – 2a)
3.

(ix) Length of subtangent at any point P(x, y) on the parabola y² = 4ax equals twice the abscissa
of the point P. Note that the subtangent is bisected at the vertex.
(x) Length of subnormal is constant for all points on the parabola & is equal to the semi latus
rectum.
Note : Students must try to proof all the above properties.

RESONANCE 32
Part : (A) Only one correct option

1. If (2, 0) is the vertex & y  axis the directrix of a parabola, then its focus is:
(A) (2, 0) (B) ( 2, 0) (C) (4, 0) (D) ( 4, 0)

2. A parabola is dr awn with its f ocus at (3, 4) and vert ex at the f ocus of t he par abola
y2  12 x  4 y + 4 = 0. The equation of the parabola is:
(A) x 2  6 x  8 y + 25 = 0 (B) y2  8 x  6 y + 25 = 0
(C) x  6 x + 8 y  25 = 0
2
(D) x 2 + 6 x  8 y  25 = 0

3. The length of the chord of the parabola, y2 = 12x passing through the vertex & making an angle of 60º
with the axis of x is:
(A) 8 (B) 4 (C) 16/3 (D) none

4. The length of the side of an equilateral triangle inscribed in the parabola, y2 = 4x so that one of its
angular point is at the vertex is:

(A) 8 3 (B) 6 3 (C) 4 3 (D) 2 3

5. The circles on focal radii of a parabola as diameter touch:


(A) the tangent at the vertex (B) the axis
(C) the directrix (D) none of these

6. The equation of the tangent to the parabola y = (x  3) 2 parallel to the chord joining the points
(3, 0) and (4, 1) is:
(A) 2 x  2 y + 6 = 0 (B) 2 y  2 x + 6 = 0
(C) 4 y  4 x + 11 = 0 (D) 4 x  4 y = 11

7. The angle between the tangents drawn from a point ( – a, 2a) to y2 = 4 ax is


   
(A) (B) (C) (D)
4 2 3 6

8. An equation of a tangent common to the parabolas y2 = 4x and x 2 = 4y is


(A) x – y + 1 = 0 (B) x + y – 1 = 0 (C) x + y + 1 = 0 (D) y = 0

9. The line 4x  7y + 10 = 0 intersects the parabola, y2 = 4x at the points A & B. The co-ordinates of the
point of intersection of the tangents drawn at the points A & B are:

 7 5  5 7  5 7  7 5
(A)  ,  (B)   ,  (C)  ,  (D)   , 
 2 2  2 2  2 2  2 2

10. AP & BP are tangents to the parabola, y2 = 4x at A & B. If the chord AB passes through a fixed point
( 1, 1) then the equation of locus of P is
(A) y = 2 (x  1) (B) y = 2 (x  1) (C) y = 2 x (D) y2 = 2 (x  1)

11. Equation of the normal to the parabola, y2 = 4ax at its point (am 2, 2 am) is:
(A) y =  mx + 2am + am 3 (B) y = mx  2am  am 3
(C) y = mx + 2am + am 3
(D) none

12. At what point on the parabola y2 = 4x the normal makes equal angles with the axes?
(A) (4, 4) (B) (9, 6) (C) (4, – 1) (D) (1, 2)

13. If on a given base, a triangle be described such that the sum of the tangents of the base angles is a
constant, then the locus of the vertex is:
(A) a circle (B) a parabola (C) an ellipse (D) a hyperbola

RESONANCE 33
14. A point moves such that the square of its distance from a straight line is equal to the difference
between the square of its distance from the centre of a circle and the square of the radius of the circle.
The locus of the point is:
(A) a straight line at right angles to the given line
(B) a circle concentric with the given circle
(C) a parabola with its axis parallel to the given line
(D) a parabola with its axis perpendicular to the given line.

15. P is any point on the parabola, y2 = 4ax whose vertex is A. PA is produced to meet the directrix in D &
M is the foot of the perpendicular from P on the directrix. The angle subtended by MD at the focus is:
(A) /4 (B) /3 (C) 5/12 (D) /2

16. If the distances of two points P & Q from the focus of a parabola y2 = 4ax are 4 & 9, then the distance
of the point of intersection of tangents at P & Q from the focus is:
(A) 8 (B) 6 (C) 5 (D) 13

17. Tangents are drawn from the point ( 1, 2) on the parabola y2 = 4 x. The length of intercept made by
these tangents on the line x = 2 is:

(A) 6 (B) 6 2 (C) 2 6 (D) none of these

18. From the point (4, 6) a pair of tangent lines are drawn to the parabola, y2 = 8x. The area of the triangle
formed by these pair of tangent lines & the chord of contact of the point (4, 6) is:
(A) 8 (B) 4 (C) 2 (D) none of these

19. Locus of the intersection of the tangents at the ends of the normal chords of the parabola
y2 = 4ax is
(A)(2a + x) y2 + 4a3 = 0 (B) (2a + x) + y2 = 0
(C) (2a + x) y + 4a = 0
2
(D) none of these

20. If the tangents & normals at the extremities of a focal chord of a parabola intersect at
(x 1, y1) and (x2, y2) respectively, then:
(A) x 1 = x2 (B) x 1 = y2 (C) y1 = y2 (D) x 2 = y1

21. Tangents are drawn from the points on the line x  y + 3 = 0 to parabola y2 = 8x. Then all the chords of
contact passes through a fixed point whose coordinates are:
(A) (3, 2) (B) (2, 4) (C) (3, 4) (D) (4, 1)

22. The distance between a tangent to the parabola y2 = 4 A x (A > 0) and the parallel normal with gradient
1 is:

(A) 4 A (B) 2 2 A (C) 2 A (D) 2 A

23. A variable parabola of latus ractum , touches a fixed equal parabola, then axes of the two curves being
parallel. The locus of the vertex of the moving curve is a parabola, whole latus rectum is:
(A)  (B) 2  (C) 4  (D) none

24. Length of the focal chord of the parabola y2 = 4ax at a distance p from the vertex is:

2a2 a3 4a3 p2
(A) (B) (C) (D)
p p2 p2 a

25. AB is a chord of the parabola y2 = 4ax with vertex at A. BC is drawn perpendicular to AB meeting the
axis at C. The projection of BC on the axis of the parabola is
(A) a (B) 2a (C) 4a (D) 8a

26. The locus of the foot of the perpendiculars drawn from the vertex on a variable tangent to the parabola
y2 = 4ax is:
(A) x (x 2 + y2) + ay2 = 0 (B) y (x2 + y2) + ax 2 = 0
(C) x (x  y ) + ay = 0
2 2 2
(D) none of these

RESONANCE 34
27. T is a point on the tangent to a parabola y2 = 4ax at its point P. TL and TN are the perpendiculars on the
focal radius SP and the directrix of the parabola respectively. Then:
(A) SL = 2 (TN) (B) 3 (SL) = 2 (TN) (C) SL = TN (D) 2 (SL) = 3 (TN)

28. The point of contact of the tangent to the parabola y2 = 9x which passes through the point
(4, 10) and makes an angle  with the axis of the parabola such that tan  > 2 is
(A) (4/9, 2) (B) (36, 18) (C) (4, 6) (D) (1/4, 3/2)

29. If the parabolas y2 = 4x and x2 = 32 y intersect at (16, 8) at an angle , then  is equal to


3 4 
(A) tan–1   (B) tan–1   (C)  (D)
5 5 2

30. From an external point P, pair of tangent lines are drawn to the parabola, y2 = 4x. If 1 & 2 are the

inclinations of these tangents with the axis of x such that, 1 + 2 = , then the locus of P is:
4
(A) x  y + 1 = 0 (B) x + y  1 = 0 (C) x  y  1 = 0 (D) x + y + 1 = 0

31. Locus of the point of intersection of the normals at the ends of parallel chords of gradient m of the
parabola y2 = 4ax is:
(A) 2 xm 2  ym 3 = 4a (2 + m 2) (B) 2 xm 2 + ym 3 = 4a (2 + m 2)
(C) 2 xm + ym = 4a (2 + m)
2
(D) 2 xm 2  ym 3 = 4a (2  m 2)

32. The equation of the other normal to the parabola y2 = 4ax which passes through the intersection of
those at (4a,  4a) & (9a,  6a) is:
(A) 5x  y + 115 a = 0 (B) 5x + y  135 a = 0
(C) 5x  y  115 a = 0 (D) 5x + y + 115 = 0

33. The point(s) on the parabola y2 = 4x which are closest to the circle,

 
x 2 + y2  24y + 128 = 0 is/are:
(A) (0, 0) (B) 2 , 2 2 (C) (4, 4) (D) none

34. If P1 Q1 and P2 Q2 are two focal chords of the parabola y2 = 4ax, then the chords P1P2 and Q1Q2 intersect on
the
(A) directrix (B) axis
(C) tangent at the vertex (D) none of these

35. If x + y = k, is the normal to y2 = 12x, then k is [IIT - 2000]


(A) 3 (B) 9 (C) –9 (D) – 3

36. The equation of the common tangent touching the circle (x – 3)2 + y2 = 9 and the parabola y2 = 4x above the
x-axis is [IIT - 2001]
(A) 3 y = 3x + 1 (B) 3 y = –(x + 3) (C) 3 y =x + 3 (D) 3 y = –(3x + 1)

37. The focal chord to y2 = 16 x is tangent to (x  6) 2 + y2 = 2, then the possible values of the slope of this
chord are: [IIT - 2003]
(A) { 1, 1} (B) { 2, 2} (C) { 2, 1/2} (D) {2,  1/2}

38. The normal drawn at a point (at12, –2at1) of the parabola y2 = 4ax meets it again in the point (at22, 2at2), then
[IIT - 2003]
2 2 2 2
(A) t2 = t1 + t (B) t2 = t1 – (C) t2 = –t1 + (D) t2 – t1 –
1 t1 t1 t1

39. The angle between the tangents drawn from the point (1, 4) to the parabola y2 = 4x is
[IIT - 2004]
   
(A) (B) (C) (D)
2 3 4 6

RESONANCE 35
40. Let P be the point (1, 0) and Q a point of the locus y2 = 8x. The locus of mid point of PQ is
[IIT - 2005]
(A) x2 + 4y + 2 = 0 (B) x2 – 4y + 2 = 0 (C) y2 – 4x + 2 = 0 (D) y2 + 4x + 2 = 0

41. A parabola has its vertex and focus in the first quadrant and axis along the line y = x. If the distances
of the vertex and focus from the origin are respectively 2 and 2 2 , then an equation of the parabola
is [IIT - 2006]
(A) (x + y)2 = x – y + 2 (B) (x – y)2 = x + y – 2
(C) (x – y) 2 = 8(x + y – 2) (D) (x + y) 2 = 8(x – y + 2)

Comprehension [IIT - 2006]

Let ABCD be a square of side length 2 units. C2 is the circle through vertices A, B, C, D and C1 is the circle
touching all the sides of the square ABCD. L is a line through A.
PA 2  PB 2  PC 2  PD 2
42. If P is a point on C1 and Q in another point on C2, is equal to [IIT - 2006 ]
QA 2  QB 2  QC 2  QD 2
(A) 0.75 (B) 1.25 (C) 1 (D) 0.5

43. A circle touch the line L and the circle C1 externally such that both the circles are on the same side of the
line, then the locus of centre of the circle is [IIT - 2006 ]
(A) ellipse (B) hyperbola (C) parabola (D) parts of straight line

44. A line M through A is drawn parallel to BD. Point S moves such that its distances from the line BD and the
vertex A are equal. If locus of S cuts M at T2 and T3 and AC at T1, then area of T1T2T3 is [IIT - 2006)]
1 2
(A) sq. units (B) sq. units (C) 1 sq. units (D) 2 sq. units
2 3

Part : (B) May have more than one options correct

45. If one end of a focal chord of the parabola y2 = 4x is (1, 2), the other end lies on
(A) x 2 y + 2 = 0 (B) xy + 2 = 0 (C) xy – 2 = 0 (D) x2 + xy – y – 1 = 0

46. The tangents at the extremities of a focal chord of a parabola


(A) are perpendicular (B) are parallel
(C) intersect on the directrix (D) intersect at the vertex

47. If from a variable point 'P' pair of perpendicular tangents PA and PB are drawn to any parabola then
(A) P lies on directrix of parabola
(B) chord of contact AB passes through focus
(C) chord of contact AB passes through of fixed point
(D) P lies on director circle

48. A normal chord of the parabola subtending a right angle at the vertex makes an acute angle  with the
x  axis, then  =

(A) arc tan 2 (B) arc sec 3 (C) arc cot 2 (D)  arc cot 2
2
49. Variable chords of the parabola y2 = 4ax subtend a right angle at the vertex. Then:
(A) locus of the feet of the perpendiculars from the vertex on these chords is a circle
(B) locus of the middle points of the chords is a parabola
(C) variable chords passes through a fixed point on the axis of the parabola
(D) none of these

50. Two parabolas have the same focus. If their directrices are the x  axis & the y  axis respectively, then
the slope of their common chord is:
(A) 1 (B)  1 (C) 4/3 (D) 3/4

51. P is a point on the parabola y2 = 4ax (a > 0) whose vertex is A. PA is produced to meet the directrix in
D and M is the foot of the perpendicular from P on the directrix. If a circle is described on MD as a
diameter then it intersects the xaxis at a point whose coordinates are:
(A) ( 3a, 0) (B) ( a, 0) (C) ( 2a, 0) (D) (a, 0)

RESONANCE 36
1. Find the vertex, axis, focus, directrix, latusrectum of the parabola x2 + 2y – 3x + 5 = 0.

2. Find the set of values of  in the interval [/2, 3/2], for which the point (sin, cos) does not lie outside the
parabola 2y2 + x – 2 = 0.

3. Two perpendicular chords are drawn from the origin ‘O’ to the parabola y = x2, which meet the parabola at P
and Q Rectangle POQR is completed. Find the locus of vertex R.

4. Find the equation of tangent & normal at the ends of the latus rectum of the parabola
y2 = 4a (x – a).

5. Prove that the straight line x + my + n = 0 touches the parabola y2 = 4ax if n = am 2.

6. If tangent at P and Q to the parabola y2 = 4ax intersect at R then prove that mid point of R and M lies
on the parabola, where M is the mid point of P and Q.
7. Find the equation of normal to the parabola x2 = 4y at (9, 6).

8. Find the equation of the chord of y2 = 8x which is bisected at (2, – 3)

9. Find the locus of the mid-points of the chords of the parabola y2 = 4ax which subtend a right angle at the
vertex of the parabola.

10. Find the equation of the circle which passes through the focus of the parabola x 2 = 4 y & touches it at
the point (6, 9).
11. Prove that the normals at the points, where the straight line x + my = 1 meets the parabola y2 = 4ax,

 4am 2 4am 
meet on the normal at the point  2 ,  of the parabola.

   

12. If the normals at three points P, Q, and R on parabola y2 = 4ax meet in a point O and S be the focus, prove
that SP. SQ . SR = a. SO2.

13. Show that the locus of the point of intersection of the tangents to y2 = 4ax which intercept a constant length
d on the directrix is (y2 – 4ax) (x + a)2 = d2 x2.

14. Show that the distance between a tangent to the parabola y2 = 4ax and the parallel normal is
a sec2 cosec , where  is the inclination of the either with the axis of the parabola.

15. P and Q are the point of contact of the tangents drawn from a point R to the parabola y2 = 4ax. If PQ be a
normal to the parabola at P, prove that PR is bisected by the directrix.

16. A circle is described whose centre is the vertex and whose diameter is three-quarters of the latus
rectum of the parabola y2 = 4ax. If PQ is the common chord of the circle and the parabola and L1 L2 is

 2  2
the latus rectum, then prove that the area of the trapezium PL1 L2Q is   a2.
 2 

17. If the normals from any point to the parabola x 2 = 4y cuts the line y = 2 in points whose abscissa are
in A.P., then prove that slopes of the tangents at the 3 conormal points are in GP.

RESONANCE 37
18. Prove that the length of the intercept on the normal at the point (at2, 2at) made by the circle which is

described on the focal distance of the given point as diameter is a 1 t .


2

19. A parabola is drawn to pass through A and B, the ends of a diameter of a given circle of radius a, and
to have as directrix a tangent to a concentric circle of radius b; then axes being AB and a perpendicular

x2 y2
diameter, prove that the locus of the focus of the parabola is + =1
b 2
b  a2
2

20. PNP is a double ordinate of the parabola then prove that the locus of the point of intersection of the
norm al at P and the straight line through P parallel to the axis is the equal parabola
y2 = 4a (x – 4a).

21. Find the locus of the point of intersection of those normals to the parabola x2 = 8 y which are at right
angles to each other. [IIT - 1997]

22. Let C1 and C2 be respectively, the parabolas x 2 = y – 1 and y2 = x – 1. Let P be any point on C1 and Q
be any point on C2. Let P1 and Q 1 be the reflections of P and Q, respectively, with respect to the line
y = x. Prove that P1 lies on C2, Q 1 lies on C1 and PQ  min {PP1 , QQ 1}. Hence or otherwise determine
points P0 and Q 0 on the parabolas C1 and C2 respectively such that P0 Q0  PQ for all pairs of points
(P, Q) with P on C1 and Q on C2. [IIT - 2000]

23. Normals are drawn from the point P with slopes m1, m 2, m 3 to the parabola y2 = 4x. If locus of P with
m 2 m 2 =  is a part of the parabola itself then find . [IIT - 2003]

EXERCISE # 1 EXERCISE # 2
1. C 2. A 3. A 4. A 5. A 6. D 7. B 3 29  3 33 
1. vertex   ,   , focus  ,  
2 8  2 8 
8. C 9. C 10. A 11. A 12. D 13. B 14. D
29
15. D 16. B 17. B 18. C 19. A 20. C 21. C axis x = 3, directrix y = – . Latus rectum = 2.
3

22. B 23. B 24. C 25. C 26. A 27. C 28. A 2.   [/2, 5/6]  [, 3/2] 3. y2 = x – 2

29. A 30. C 31. A 32. B 33. C 34. A 35. B 4. Tangent y = x, y = – x,


Normal x + y = 4a, x – y = 4a
36. C 37. A 38. A 39. B 40. C 41. C 42. A 7. 2x + 9y = 72 8. 4x + 3y + 1 = 0

43. C 44. C 45. ABD 46. AC 47. ABCD 9. y2 – 2ax + 8a2 = 0

10. x 2 + y2 + 18x – 28y + 27 = 0


48. BD 49. ABC 50. AB 51. AD
21. x 2  2 y + 12 = 0 23. = 2

RESONANCE 38
Ellipse

1. Definitions

It is a locus of a point which moves in such a way that the ratio


of its distance from a fixed point and a fixed line (not passes
through fixed point and all points and line lies in same plane) is
constant (e) which is less than one.
The fixed point is called - focus

He who can properly


The fixed line is called -directrix.
The constant ratio is called - eccentricity, it is denoted by 'e'.

define and divide is Solved Example # 1


Find the equation to the ellipse whose focus is the point
to be considered a
(– 1, 1), whose directrix is the straight line x – y + 3 = 0 and
god. eccentricity is
1
.
2
Solution.
Let P  (h, k) be moving point,

The ludicrous state of e=


PS
=
1
PM 2
solid geometry made
hk 3
2
1  
me pass over this  (h + 1) + (k – 1) =
2 2
4  2 

branch. Republic,  locus of P(h, k) is


8 {x 2 + y2 + 2x – 2y + 2} = (x 2 + y2 – 2xy + 6x – 6y + 9)
.......... Pl at o 7x2 + 7y2 + 2xy + 10x – 10 y + 7 = 0. Ans.

Note : The general equation of a conic with focus (p, q) & directrix
lx + my + n = 0 is:
(l2 + m 2) [(x  p)2 + (y  q)2] = e2 (lx + my + n) 2
 ax 2 + 2hxy + by2 + 2gx + 2fy + c = 0
represent ellipse if 0 < e < 1;   0, h² < ab

Self Practice Problem :-

1. Find the equation to the ellipse whose focus is (0, 0) directrix is

1
x + y – 1 = 0 and e = .
2
Ans. 3x2 + 3y2 – 2xy + 2x + 2y – 1 = 0.

RESONANCE 1
2. Standard Equation
Standard equation of an ellipse referred
to its principal axes along the coordinate

x2 y2
axes is  = 1,
a2 b2
where a > b & b² = a² (1  e²).

b2
Eccentricity: e = 1  2 , (0 < e < 1)
a
Focii : S  (a e, 0) & S  ( a e, 0).
a a
Equations of Directrices : x = &x= .
e e

Major Axis : The line segment AA in which the focii S & S lie is of length 2a & is called the major axis
(a > b) of the ellipse. Point of intersection of major axis with directrix is called the foot of the directrix
(Z).
Minor Axis : The yaxis intersects the ellipse in the points B  (0,  b) & B  (0, b). The line segment
BB is of length 2b (b < a) is called the minor axis of the ellipse.
Principal Axis : The major & minor axes together are called principal axis of the ellipse.
Vertices : Point of intersection of ellipse with major axis. A ( a, 0) & A  (a, 0) .
Focal Chord : A chord which passes through a focus is called a focal chord.
Double Ordinate : A chord perpendicular to the major axis is called a double ordinate.
Latus Rectum : The focal chord perpendicular to the major axis is called the latus rectum.

Length of latus rectum (LL) =


2b 2 minor axis2
a

major axis
 2a 1 e 2 
= 2 e (distance from focus to the corresponding directrix)
Centre : The point which bisects every chord of the conic drawn through it, is called the centre of the

y2
conic. C  (0, 0) the origin is the centre of the ellipse x 2  2 = 1.
2

a b
NOTE :
y2
If the equation of the ellipse is given as x 2  2 = 1 and nothing is mentioned, then the rule is
2
(i)
a b
to assume that a > b.
(ii) If b > a is given, then the yaxis will become major axis and x-axis will become the minor axis
and all other points and lines will change accordingly.

Solved Example # 2
Find the equation to the ellipse whose centre is origin, axes are the axes of co-ordinate and passes
through the points (2, 2) and (3, 1).
Solution.
Let the equation to the ellipse is

x2 y2
+ =1
a2 b2
Since it passes through the points (2, 2) and (3, 1)
4 4
 + =1 ..........(i)
a 2
b2

RESONANCE 2
9 1
and + =1 .........(ii)
a 2
b2
from (i) – 4 (ii), we get
4  36 32
= 1– 4  a2 =
a 2
3
from (i), we get
1 1 3 83
= – =
b 2
4 32 32

32
b2 =
5
 Ellipse is 3x2 + 5y2 = 32 Ans.

Solved Example # 3
1
Find the equation of the ellipse whose focii are (4, 0) and (– 4, 0) and eccentricity is
3
Solution.
Since both focus lies on x-axis, therefore x-axis is major axis and mid point of focii is origin which is
centre and a line perpendicular to major axis and passes throguh centre is minor axis which is y-axis.
Let equation of ellipse is

x2 y2
+ =1
a2 b2
1
ae = 4 and e= (Given)
3

 a = 12 and b2 = a2 (1 – e2)

 1
 b2 = 144 1  
 9 
b2 = 16 × 8
b=8 2

x2 y2
Equation of ellipse is + =1 Ans.
144 128

Solved Example # 4
If minor-axis of ellipse subtend a right angle at its focus then find the eccentricity of ellipse.
Solution.

x2 y2
Let the equation of ellipse is + =1 (a > b)
a2 b2

BSB =
2

and OB = OB

 BSO =
4
 OS = OB  ae = b

b2
 e2 = = 1 – e2
a2

1
 e= Ans.
2

RESONANCE 3
Solved Example # 5
From a point Q on the circle x2 + y2 = a2, perpendicular QM are drawn to x-axis, find the locus of point
'P' dividing QM in ratio 2 : 1.
Solution.
Let Q  (a cos, a sin)
M  (a cos, 0)
Let P  (h, k)
a sin 
 h = a cos, k =
3

 3k  h
2 2

   +   =1
 a a

x2 y2
 Locus of P is + =1 Ans.
a2 (a / 3 ) 2
Solved Example # 6
Find the equation of axes, directrix, co-ordinate of focii, centre, vertices, length of latus - rectum and

( x  3) 2 ( y  2) 2
eccentricity of an ellipse + = 1.
25 16
Solution.

X2 Y2
Let x – 3 = X, y – 2 = Y, so equation of ellipse becomes as + = 1.
5 2
42
equation of major axis is Y = 0  y = 2.
equation of minor axis is X = 0  x = 3.
centre (X = 0, Y = 0)  x = 3, y = 2
C  (3, 2)
Length of semi-major axis a = 5
Length of major axis 2a = 10
Length of semi-minor axis b = 4
Length of major axis = 2b = 8.
Let 'e' be eccentricity
 b2 = a2 (1 – e2)

a2  b2 25  16 3
e= = = .
a 2 25 5

2b 2 2  16 32
Length of latus rectum = LL = = =
a 5 5
Co-ordinates focii are X = ± ae, Y = 0
 S  (X = 3, Y = 0) & S (X = –3, Y = 0)
 S  (6, 2) & S (0, 2)
Co-ordinate of vertices
Extremities of major axis A  (X = a, Y = 0) & A (X = – a, Y = 0)
 A  (x = 8, y = 2) & A = (x = – 2, 2)
A  (8, 2) & A (– 2, 2)
Extremities of minor axis B (X = 0, Y = b) & B  (X = 0, Y = – b)
B (x = 3, y = 6) & B (x = 3, y = – 2)
B  (3, 6) & B (3, – 2)
a
Equation of directrix X = ±
e
25 34 16
x–3=±  x= & x=–
3 3 3

RESONANCE 4
Self Practice Problem

2. Find the equation to the ellipse whose axes are of lengths 6 and 2 6 and their equations are
x – 3y + 3 = 0 and 3x + y – 1 = 0 respectively.
Ans. 3(x – 3y + 3)2 + 2(3x+ y – 1)2 = 180, 21x 2 – 6xy + 29y2 + 6x – 58y – 151 = 0.

3. Find the eccentricity of ellipse whose minor axis is double the latus rectum.

3
Ans.
2
4. Find the co-ordinates of the focii of the ellipse 4x2 + 9y2 = 1.

 
Ans.   5 , 0
 6 
 

x2 y2 1
5. Find the standard ellipse + = 1 passing through (2, 1) and having eccentricity .
a 2
b 2
2
Ans. 3x + 4y = 16
2 2

6. A point moves so that the sum of the squares of its distances from two intersecting non perpendicular
straight lines is constant. Prove that its locus is an ellipse.

3. Auxiliary Circle / Eccentric Angle :


A circle described on major axis of ellipse as diameter is called the auxiliary circle.
Let Q be a point on the auxiliary circle x² + y² = a² such that line through Q perpendicular to the x  axis
on the way intersects the ellipse at P, then P & Q are called as the Corresponding Points on the
ellipse & the auxiliary circle respectively. ‘’ is called the Eccentric Angle of the point P on the ellipse
( < ). Q  (a cos , a sin)
P  (a cos , b sin)
Note that :
(PN) b Semi minor axis
 
(QN) a Semi major axis

NOTE : I f f r om each point of a c ircle


perpendiculars are drawn upon a fixed diameter
then the locus of the points dividing these
perpendiculars in a given ratio is an ellipse of
which the given circle is the auxiliary circle.

Solved Example # 7
x2 y2
Find the focal distance of a point P() on the ellipse + =1 (a > b)
a2 b2
Solution.
Let 'e' be the eccentricity of ellipse.
 PS = e . PM
a 
= e   a cos  
e 
PS = (a – a e cos)
and PS = e. PM
 a
= e  a cos   
 e
PS = a + ae cos
 focal distance are (a ± ae cos)
Note : PS + PS = 2a
PS + PS = AA

RESONANCE 5
Solvex Eample # 8

x2 y2
Find the distance from centre of the point P on the ellipse + = 1 whose radius makes angle 
a2 b2
with x-axis.
Sol. Let P  (a cos, b sin)

b
 tan = tan
a

a
tan = tan 
b

a 2  b 2 tan 2 
OP = a cos   b sin  =
2 2 2 2
sec 2 

a2
a2  b2  tan2 
a 2  b 2 tan 2  b2
= = a2
1  tan 2  1 tan 2 
b2

ab
OP= Ans.
a sin   b 2 cos 2 
2 2

Self Practice Problem

x2 y2
7. Find the distance from centre of the point P on the ellipse + = 1 whose eccentine angle is 
a2 b2

Ans. r  a 2 cos 2   b 2 sin 2 

x2 y2
8. Find the eccentric angle of a point on the ellipse + = 1 whose distance from the centre is 2.
6 2

 3
Ans. ± ,±
4 4

9. Show that the area of triangle inscribed in an ellipse bears a constant ratio to the area of the triangle
formed by joining points on the auxiliary circle corresponding to the vertices of the first triangle.

4. Parametric Representation:
y2
The equations x = a cos  & y = b sin  together represent the ellipse x 2  2 = 1.
2

a b
Where  is a parameter. Note that if P()  (a cos  b sin ) is on the ellipse then;
Q()  (a cos  a sin ) is on the auxiliary circle.

The equation to the chord of the ellipse joining two points with eccentric angles  &  is given by
x  y   
cos  sin  cos .
a 2 b 2 2

RESONANCE 6
Solved Example # 9
x2 y2   5 
Write the equation of chord of an ellipse + =1 joining two points P   and Q  .
25 16 4  4 
Solution.
Equation of chord is

  5    5    5 
x    y      
cos  4 4  + . sin  4 4  = cos  4 4 
5 4
2 2 2

x  3  y  3 
. cos   + . sin   =0
5  4  4  4 

x y
– + =0  y=x Ans.
5 5
Solved Example 10
If P() and P() are extremities of a focal chord of ellipse then prove that its eccentricity

 
cos  
 2 
e= .

cos 
 2 
Solution.

x2 y2
Let the equation of ellipse is + =1
a 2
b2
 equation of chord is

x   y    


cos   + sin   = cos  
a  2  b  2   2 
Since above chord is focal chord,
 it passes through focus (ae, 0) or (– ae, 0)

   
 ± e cos   = cos  
 2   2 

 
cos  
 2 
 e= Ans.

cos 
 2 
 
cos
2
Note :  ±e=

cos
2

 
1  tan
. tan
2 2
±e=
 
1  tan . tan
2 2
Applying componendo and dividendo

1 e 2
=  
 e 1 2 tan . tan
2 2
  1 e e 1
tan tan = or
2 2 e 1 1 e

RESONANCE 7
Solved Example # 11
x2 y2
Find the angle between two diameters of the ellipse + = 1. Whose extremities have eccentrici
a2 b2

angle  and  =  + .
2
Solution.

x2 y2
Let ellipse is + =1
a2 b2

b sin  b
Slope of OP = m 1 = = tan
a cos  a

b sin  b 
Slope of OQ = m 2 = =– cot given  =  +
a cos  a 2

b
(tan   cot  )
m1  m 2 a 2ab
 tan = = = Ans.
1  m1m 2 b2 (a  b 2 ) sin 2
2
1 2
a

Self Practice Problem

x2 y2
10. Find the sum of squares of two diameters of the ellipse + = 1 whose extremitites have
a2 b2


eccentric angles differ by and show that it is constant.
2
Ans. 4(a2 + b2)

11. Show that the sum of squares of reciprocals of two perpendicular diameters of the ellipse
x2 y2
+ = 1 is constant. Find the constant also.
a 2
b2
1 1 1 
Ans.   
4  a2 b2 
x2 y2
12. Find the locus of the foot of the perpendicular from the centre of the ellipse + = 1 on the chord
a2 b2

joining two points whose eccentric angles differ by .
2
Ans. 2(x2 + y2)2 = a2 x 2 + b2 y2.

5. Position of a Point w.r.t. an Ellipse:


x12 y12
The point P(x 1, y1) lies outside, inside or on the ellipse according as ;   1 > < or = 0.
a2 b2

Solved Example # 12

x2 y2
Check wether the point P(3, 2) lies inside or outside of the ellipse + = 1.
25 16
Solution.
9 4 9 1
S1  + –1= + –1<0
25 16 25 4
 Point P  (3, 2) lies inside the ellipse. Ans.

RESONANCE 8
Solved Example # 13
x2 y2
Find the set of value(s) of '' for which the point P(, – ) lies inside the ellipse + = 1.
16 9
Solution.
If P(, – ) lies inside the ellipse
 S1 < 0

2 2
 + –1<0
16 9
25 144
 . 2 < 1  2 <
144 25
 12 12 
    , . Ans.
 5 5 

6. Line and an Ellipse:


y2
The line y = mx + c meets the ellipse x 2  2 = 1 in two points real, coincident or imaginary according
2

a b
as c² is < = or > a²m² + b².
y2
Hence y = mx + c is tangent to the ellipse x 2  2 = 1 if c² = a²m² + b².
2

a b
Solved Example # 14
x2 y2
Find the set of value(s) of '' for which the line 3x – 4y +  = 0 intersect the ellipse + = 1 at two
16 9
distinct points.
Solution
Solving given line with ellipse, we get

( 4 y   )2 y2
+ =1
9  16 9

2y 2 y 2
– + –1 = 0
9 18 144
Since, line intersect the parabola at two distinct points,
 roots of above equation are real & distinct
 D0
2 8  2 
 –
 
.  144  1 > 0
(18)2 9  
 –12 2 <  < 12 2

Self Practice Problem


x2 y2
13. Find the value of '' for which 2x – y +  = 0 touches the ellipse + =1
25 9
Ans. =± 109

7. Tangents:
x2 y2
(a) Slope form: y = mx ± a m b
2 2 2 is tangent to the ellipse + = 1 for all values of m.
a2 b2
x x1 y y1 x2 y2
(b) Point form :   1 is tangent to the ellipse + = 1 at (x1, y1).
a2 b2 a2 b2

xcos ysin x2 y2
(c) Parametric form:   1 is tangent to the ellipse 2 + 2 = 1 at the point
a b a b
(a cos , b sin ).

RESONANCE 9
NOTE :
(i) There are two tangents to the ellipse having the same m, i.e. there are two tangents parallel to
any given direction.These tangents touches the ellipse at extremities of a diameter.

 cos   sin    
(ii) Point of intersection of the tangents at the point  &  is,  a 2
 
,b 2 
  
 cos 2
cos 2 

(iii) The eccentric angles of the points of contact of two parallel tangents differ by .
Solved Example # 15
Find the equations of the tangents to the ellipse 3x 2 + 4y2 = 12 which are perpendicular to the
line y + 2x = 4.
Solution.
1
Slope of tangent = m =
2
x2 y2
Given ellipse is + =1
4 3
Equation of tangent whose slope is 'm' is
y = mx ± 4m 2  3
1 1
m=  y= x± 1 3
2 2

2y = x ± 4
Solved Example # 16
x2 y2
A tangent to the ellipse +
= 1 touches at the point P on it in the first quadrant and meets the
a2 b2
co-ordinate axes in A and B respectively. If P divides AB in the ratio 3 : 1, find the equation of the
tangent.
Solution.
Let P  ( a cos, b sin)
 equation of tangent is
x y
cos + sin = 1
a b
A  (a sec, 0)
B  (0, b cosec)
 P divide AB internally in the ratio 3 : 1
a sec  1 1
 a cos =  cos2 =  cos =
4 4 2

3b cos ec 3
and b sin  =  sin =
4 2

x 3y
 tangent is + =1  bx + 3 ay = 2ab Ans.
2a 2b
Solved Example # 17
Prove that the locus of the point of intersection of tangents to an ellipse at two points whose eccentric
angle differ by a constant  is an ellipse.
Solution.
Let P (h, k) be the point of intersection of tangents at A() and B() to the ellipse.

 
a cos   b sin  
 2   2 
 h=  & k = 
cos   cos  
 2   2 

RESONANCE 10
h k  
2 2

   +   = sec2  
 
a  
b  2 
but given that  –  = 

x2 y2
 locus is + =1 Ans.
 
a 2 sec 2   b 2 sec 2  
2 2
Solved Example # 18
x2 y2
Find the locus of foot of perpendicular drawn from centre to any tangent to the ellipse is + = 1.
a2 b2
Solution.
Let P(h, k) be the foot of perpendicular to a tangent y = mx + a 2m 2  b 2 .......(i)
from centre
k h
 .m=–1  m=– .......(ii)
h k
 P(h, k) lies on tangent
 k = mh + a 2m 2  b 2 .......(iii)
from equation (ii) & (iii), we get

 
2
k  h
2
 = a h + b2
2 2

 k 
  k2
 locus is (x 2 + y2)2 = a2x 2 + b2y2 Ans.
Self Practice Problem
14. Show that the locus of the point of intersection of the tangents at the extremities of any focal chord of
an ellipse is the directrix corresponding to the focus.

15. Show that the locus of the foot of the perpendicular on a varying tangent to an ellipse from either of its
foci is a concentric circle.
16. Prove that the portion of the tangent to an ellipse intercepted between the ellipse and the directrix
subtends a right angle at the corresponding focus.
17. Find the area of parallelogram formed by tangents at the extremities of latera recta of the ellipse
x2 y2
  1.
a2 b2
2a 3
Ans.
a2  b2

18. If y1 is ordinate of a point P on the ellipse then show that the angle between its focal radius and tangent
 b2 
 
at it, is tan  aey  .
–1

 1

x2 y2
19. Find the eccentric angle of the point P on the ellipse + = 1 tangent at which, is equally
a2 b2
inclined to the axes.

b b b


Ans. = ± tan–1   ,  – tan–1   , –  + tan–1  
a a a

RESONANCE 11
8. Norm als:
x2 y2 a2 x b2 y
(i) Equation of the normal at (x1, y1) to the ellipse + = 1 is  = a²  b².
a2 b2 x1 y1

x2 y2
(ii) Equation of the normal at the point (acos , bsin ) to the ellipse + = 1 is;
a2 b2

a  b m
ax. sec   by. cosec  = (a²  b²).
2 2
(iii) Equation of a normal in terms of its slope 'm' is y = mx  .
a 2  b 2m 2
Solved Example # 19

x2 y2
P and Q are corresponding points on the ellipse + = 1 and the auxiliary circles respectively..
a2 b2
The normal at P to the ellipse meets CQ in R, where C is the centre of the ellipse. Prove that
CR = a + b
Sol. Let P  (acos , b sin)
 Q  (a cos, a sin)
Equation of normal at P is
(a sec) x – (b cosec ) y = a2 – b2 ..........(i)
equation of CQ is y = tan . x .........(ii)
Solving equation (i) & (ii), we get
(a – b) x = (a2 – b2) cos
x = (a + b) cos, & y = (a + b) sin
 R  ((a + b) cos, (a + b) sin)
 CR = a + b Ans.

Solved Example # 20

x2 y2
Find the shortest distance between the line x + y = 10 and the ellipse  =1
16 9
Solution.
Shortest distance occurs between two non-intersecting curve always along common normal.
Let 'P' be a point on ellipse and Q is a point on given line for which PQ is common normal.
 Tangent at 'P' is parallel to given line
 Equation of tangent parallel to given line is (y = mx ± a 2m 2  b 2 )
y=–x±5
 x + y + 5 = 0 or x+y–5=0
 minimum distance = distance between
x + y – 10 = 0 & x + y – 5 = 0
| 10  5 |
 shortest distance =
1 1

5
= Ans.
2
Solved Example # 21

Prove that, in an ellipse, the distance between the centre and any normal does not exceed the difference
between the semi-axes of the ellipse.
Solution.
x2 y2
Let the equation of ellipse is  1
a 2
b2

RESONANCE 12
Equation of normal at P () is
(a sec)x – (bcosec )y – a2 + b2 = 0
distance of normal from centre

| a2  b2 |
= OR =
a 2  b 2  (a tan )2  (b cot )2
| a2  b2 |
=
(a  b)2  (a tan   b cot )2
| a2  b2 |
 (a + b)2 + (a tan – b cot)2  (a + b)2 or 
(a  b ) 2
|OR| (a – b) Ans.
Self Practice Problem
x2 y2
20. Find the value(s) of 'k' for which the line x + y = k is a normal to the ellipse  1
a 2
b2

Ans. k= 
a 2
 b2 
2

a2  b2
x2 y2
21. If the normal at the point P() to the ellipse  = 1 intersects it again at the point Q(2) then
14 5
cos =
2 2 6 6
(A*) – (B) (C) – (D)
3 3 7 7

9. Pair of Tangents:
The equation to the pair of tangents which can be drawn from any point (x 1, y1) to the ellipse

x 2  y = 1 is given by: SS = T² where :


2

a2 b 2 1

x1 y1
2 2
y2 xx1 yy1
S  x2  2 – 1 S1 = 2  2 – 1 ;
2
; T + – 1.
a b a b a 2
b2
Solved Example # 22
x2 y2
How many real tangents can be drawn from the point (4, 3) to the ellipse + = 1. Find the
16 9
equation these tangents & angle between them.
Solution.
Given point P  (4, 3)
x2 y2
ellipse S + –1=0
16 9
16 9
S1  + –1=1>0
16 9

 Point P  (4, 3) lies outside the ellipse.


 Two tangents can be drawn from the point P(4, 3).
Equation of pair of tangents is
SS1 = T 2
 x2 y2 
 1 . 1 =  4 x  3 y  1
2


 
 16 9 
   16 9 
x2 y2 x2 y2 xy x 2y
 + –1= + +1+ – –  – xy + 3x + 4y – 12 = 0
16 9 16 9 6 2 3
 (4 – x) (y – 3) = 0  x=4&y=3

and angle between them = Ans.
2

RESONANCE 13
Solved Example # 23
x2 y2
Find the locus of point of intersection of perpendicular tangents to the ellipse  =1
a2 b2
Solution.
Let P(h, k) be the point of intersection of two perpendicular tangents
equation of pair of tangents is SS1 = T 2

 x2 y2   h2 k 2 
 =  hx  ky  1
2


   1    1
 a2 b2   a2 b 2 
     a2 b2 

x 2  k  1  h2 
2
y2   1 + ........ = 0
  2  +  a2  .........(i)
a2  b  a2  
Since equation (i) represents two perpendicular lines

1  k2   h2 

  1 + 1   1 =0
 b2   a2 
a2   b2  
 k 2 – b2 + h2 – a2 = 0  locus is x2 + y2 = a2 + b2 Ans.

Self Practice Problem

22. Find the locus of point of intersection of the tangents drawn at the extremities of a focal chord of the

x2 y2
ellipse + = 1.
a 2
b2
a
Ans. x=±
e

1 0 . Director Circle:
Locus of the point of intersection of the tangents which meet at right angles is called the Director
Circle. The equation to this locus is x² + y² = a² + b² i.e. a circle whose centre is the centre of the
ellipse & whose radius is the length of the line joining the ends of the major & minor axes.

Solved Example # 24

An ellipse slides between two perpendicular lines. Show that the locus of its centre is a circle.
Solution
Let length of semi-major and semi-minor axis are 'a' and 'b' and centre is C  (h, k)
Since ellipse slides between two perpendicular lines, there for point of intersection of two perpendicular
tangents lies on director circle.
Let us consider two perpendicular lines as x & y axes
 point of intersection is origin O  (0, 0)
 OC = radius of director circle

 h2  k 2 = a2  b2
 locus of C  (h, k) is
 x 2 + y2 = a2 + b2 which is a circle

Self Practice Problem

A tangent to the ellipse x2 + 4y2 = 4 meets the ellipse x 2 + 2y2 = 6 at P and Q. Prove that the tangents
at P and Q of the ellipse x 2 + 2y2 = 6 are at right angles.

RESONANCE 14
1 1 . Chord of Contact:
x2 y2
Equation to the chord of contact of tangents drawn from a point P(x1, y1) to the ellipse + = 1 is
a 2
b2
xx1 yy1
T = 0, where T= + –1
a 2
b2
Solved Example # 25
x2 y2
If tangents to the parabola y2 = 4ax intersect the ellipse + = 1 at A and B, then find the locus
a2 b2
of point of intersection of tangents at A and B.
Solution
Let P  (h, k) be the point of intersection of tangents at A & B
xh yk
 equation of chord of contact AB is 2 + 2 = 1 .............(i)
a b
which touches the parabola equation of tangent to parabola y2 = 4ax
a
y = mx +
m
a
 mx – y = – .............(ii)
m
equation (i) & (ii) as must be same

a
1 
m m
 = =
 h   k  1
 2  2
a  b 

h b
2
ak
 m=– 2 & m =
k a b2

hb 2 ak
 – =
ka 2
b2

b4
 locus of P is y = – 2
.x Ans.
a3
Self Practice Problem

23. Find the locus of point of intersection of tangents at the extremities of normal chords of the

x2 y2
ellipse + = 1.
a2 b2

a6 b6
Ans. + = (a2 – b2)2
x2 y2

24. Find the locus of point of intersection of tangents at the extremities of the chords of the

x2 y2
ellipse + = 1 subtending a right angle at its centre.
a2 b2

x2 y2 1 1
Ans. + = +
a 4
b 4
a 2
b2

RESONANCE 15
1 2 . Chord with a given middle point:

y2
Equation of the chord of the ellipse x 2  2 = 1 whose middle point is (x 1, y1) is T = S1,
2

a b
x1 y1 xx1 yy1
2 2

where S1 =  –1; T + – 1.
a 2
b 2
a 2
b2

Solved Example # 26
x2 y2
Find the locus of the mid - point of focal chords of the ellipse + = 1.
a 2
b2
Solution
Let P  (h, k) be the mid-point

xh yk h2 k2
 equation of chord whose mid-point is given + –1= + –1
a 2
b 2
a2 b2
since it is a focal chord,
 it passes through focus, either (ae, 0) or (–ae, 0)
If it passes trhrough (ae, 0)

ex x2 y2
 locus is = 2 + 2
a a b
If it passes through (–ae, 0)

ex x2 y2
 locus is – = 2 + 2 Ans.
a a b

Solved Example # 27
x2 y2
Find the condition on 'a' and 'b' for which two distinct chords of the ellipse + = 1 passing
2a 2 2b 2
through (a, – b) are bisected by the line x + y = b.
Solution
Let the line x + y = b bisect the chord at P(, b – )
 equation of chord whose mid-point is P(, b – )

x y(b   ) 2 (b   ) 2
+ = +
2a 2
2b 2
2a 2 2b 2
Since it passes through (a, –b)

 (b   ) 2 (b   ) 2
 – = +
2a 2b 2a 2 2b 2

 1 1  1 1  2  1 1   3 1
     – 1 = 2  2  2  – +1  2  2  2  –     + 2 = 0
 a b  a b  b a b  b a
since line bisect two chord
 above quadratic equation in  must have two distinct real roots

3 1  1 1 
2

    –4  2  2 .2>0
b a a b 

9 1 6 8 8 1 7 6
 + + – 2 – 2 >0  – + >0
b 2
a 2
ab a b b 2
a 2
ab
 a – 7b + 6ab > 0
2 2

 a2 > 7b2 – 6ab which is the required condition.

RESONANCE 16
Self Practice Problem
x2 y2
25. Find the equation of the chord + = 1 which is bisected at (2, 1).
36 9
Ans. x + 2y = 4
x2 y2
26. Find the locus of the mid-points of normal chords of the ellipse + = 1.
a2 b2
 x2 y2   a6 b 6 
2
     
Ans.  a2 b2   x 2 y 2  = (a – b )
2 2 2

   
x2 y2  1 2
27. Find the length of the chord of the ellipse + = 1 whose middle point is  , 
25 16 2 5
7
Ans. 41
5

1 3 . Important High Lights :


y2
Refering to the ellipse x 2  2 = 1
2

a b
 If P be any point on the ellipse with S & S as its foci then  (SP) +  (SP) = 2a.
 The tangent & normal at a point P on the ellipse bisect the external & internal angles between the focal
distances of P. This refers to the well known reflection property of the ellipse which states that rays
from one focus are reflected through other focus & viceversa. Hence we can deduce that the straight
lines joining each focus to the foot of the perpendicular from the other focus upon the tangent at any
point P meet on the normal PG and bisects it where G is the point where normal at P meets the major
axis.
 The product of the length’s of the perpendicular segments from the foci on any tangent to the ellipse is
b² and the feet of these perpendiculars lie on its auxiliary circle and the tangents at these feet to the
auxiliary circle meet on the ordinate of P and that the locus of their point of intersection is a similiar
ellipse as that of the original one.
 The portion of the tangent to an ellipse between the point of contact & the directrix subtends a right
angle at the corresponding focus.
 If the normal at any point P on the ellipse with centre C meet the major & minor axes in G & g
respectively, & if CF be perpendicular upon this normal, then
(i) PF. PG = b²
(ii) PF. Pg = a²
(iii) PG. Pg = SP. S P
(iv) CG. CT = CS2
(v) locus of the mid point of Gg is another ellipse having the same eccentricity as that of the
original ellipse.
[where S and S are the focii of the ellipse and T is the point where tangent at P meet the major axis]
 The circle on any focal distance as diameter touches the auxiliary circle. Perpendiculars from the
centre upon all chords which join the ends of any perpendicular diameters of the ellipse are of constant
length.
 If the tangent at the point P of a standard ellipse meets the axis in T and t and CY is the perpendicular
on it from the centre then,
(i) T t. PY = a2  b2 and
(ii) least value of T t is a + b.

RESONANCE 17
Part : (A) Only one correct option

1. The eccentricity of the ellipse 4x2 + 9y2 + 8x + 36y + 4 = 0 is

5 3 2 5
(A) (B) (C) (D)
6 5 3 3

2. The equation of the ellipse with its centre at (1, 2), focus at (6, 2) and passing through the point (4, 6)
is

( x  1)2 ( y  2)2 ( x  1)2 ( y  2) 2


(A)  1 (B) + =1
45 20 20 45

( x  1)2 ( y  2)2 ( x  1)2 ( y  2)2


(C)  1 (D)  1
25 16 16 25

x y
3. The eccentricity of the ellipse which meets the straight line + = 1 on the axis of x and the straight
7 2

x y
line – = 1 on the axis of y and whose axes lie along the axes of coordinates, is
3 5

3 2 2 6 3
(A) (B) (C) (D) none of these
7 7 7

4. The curve represented by x = 3 (cos t + sin t), y = 4 (cos t – sin t), is


(A) ellipse (B) parabola (C) hyperbola (D) circle

x2 y2
5. Minimum area of the triangle by any tangent to the ellipse + = 1 with the coordinate axes is
a2 b2

a2  b2 (a  b ) 2 (a  b) 2
(A) (B) (C) ab (D)
2 2 2

6. A circle has the same centre as an ellipse & passes through the focii F1 & F2 of the ellipse, such that
the two curves intersect in 4 points. Let 'P' be any one of their point of intersection. If the major axis of
the ellipse is 17 & the area of the triangle PF 1F2 is 30, then the distance between the focii is :

(A) 11 (B) 12 (C) 13 (D) 15

y
Q is a point on the auxiliary circle corresponding to the point P of the ellipse x 2  2 = 1. If T is the foot
2 2
7.
a b
of the perpendicular dropped from the focus S onto the tangent to the auxiliaryy circle at Q then the
 SPT is :
(A) isosceles (B) equilateral (C) right angled (D) right isosceles

x2 y2
8. x  2y + 4 = 0 is a common tangent to y2 = 4x &  = 1. Then the value of ‘b’ and the other
4 b2
common tangent are given by :
(A) b = 3 ; x + 2y + 4 = 0 (B) b = 3; x + 2y + 4 = 0

(C) b = 3 ; x + 2y  4 = 0 (D) b = 3 ; x  2y  4 = 0

RESONANCE 18
x2 y2
9. The locus of point of intersection of tangents to an ellipse 2 + 2 = 1 at the points whose the sum
a b
of eccentric angles is constant, is :
(A) a hyperbola (B) an ellipse (C) a circle (D) a straight line

4 x2 y2
10. A tangent having slope of  to the ellipse + = 1 intersects the major & minor axes in points
3 18 32
A & B respectively. If C is the centre of the ellipse, then the area of the triangle ABC is :
(A) 12 sq. units (B) 24 sq. units (C) 36 sq. units (D) 48 sq. units

x2 y2
11. The normal at a variable point P on an ellipse 2  2 = 1 of eccentricity ‘e’ meets the axes of the
a b
ellipse in Q and R then the locus of the mid-point of QR is a conic with an eccentricity e such that :

(A) e  is independent of e (B) e = 1


(C) e = e (D) e = 1/e

x2 y2
12. y = mx + c is a normal to the ellipse, 2  = 1, if c2 is equal to :
a b2
(a 2  b 2 )2 (a 2  b 2 ) 2 (a 2  b 2 ) 2 m 2 (a 2  b 2 ) 2 m 2
(A) 2 2 (B) (C) (D)
a m  b2 a 2m2 a2  b2 m2 a2 m2  b2
13. An arc of a bridge is semi-elliptical with major axis horizontal. The length of the base is 9 meter and the
highest part of the bridge is 3 meter from the horizontal. The best approximation of the Pillar 2 meter
from the centre of the base is :
(A) 11/4 m (B) 8/3 m (C) 7/2 m (D) 2 m

14. Point 'O' is the centre of the ellipse with major axis AB & minor axis CD. Point F is one focus of the
ellipse. If OF = 6 & the diameter of the inscribed circle of triangle OCF is 2, then the product (AB) (CD)
is
(A) 64 (B) 12 (C) 65 (D) 3

15. An ellipse is such that the length of the latus rectum is equal to the sum of the lengths of its semi
principal axes. Then:
(A) Ellipse bulges to a circle
(B) Ellipse becomes a line segment between the two foci
(C) Ellipse becomes a parabola
(D) none of these
16. A line of fixed length (a + b) moves so that its ends are always on two fixed perpendicular straight lines.
The locus of the point which divided this line into portions of lengths a & b is:
(A) an ellipse (B) an hyperbola (C) a circle (D) none of these
17. The line 2x + y = 3 cuts the ellipse 4x 2 + y2 = 5 at P and Q. If  be the angle between the normals at
these points, then tan =
(A) 1/2 (B) 3/4 (C) 3/5 (D) 5
18. The focal chord of y2 = 16x is tangent to (x – 6)2 + y2 = 2, then the possible values of the slope of this chord
are [IIT – 2003]
 1  1
(A) {– 1, 1} (B) {– 2, 2} (C)  2,  (D) 2,  
 2  2

19. A tangent is drawn to ellipse x2 + 2y2 = 2. Then the locus of mid point of portion of the tangent
intercepted between coordinate axes. [IIT - 2004 ]
1 1 1 1 x2 y2 x2 y2
(A)  =1 (B)  =1 (C)  1 (D)  1
2x 2
4y 2
4x 2
2y 2
2 4 4 2

RESONANCE 19
20. The locus of mid point of the intercept of the tangent drawn from an external point to the ellipse
x2 + 2y2 = 2 between the coordinate axes, is [IIT - 2004]
1 1 1 1 1 1 1 1
(A) 2 + 2 = 1 (B) 2 + 2y 2 = 1 (C) 2 + 4y 2 = 1 (D) + 2 =1
x 2 y 4x 2x 2x 2 y
21. An ellipse has OB as semi-minor axis, F and F its foci and the angle FBFis a right angle. Then, the
eccentricity of the ellipse is [IIT - 2005]

1 1 1 1
(A) (B) (C) (D)
4 3 2 2

Part : (B) May have more than one options correct

22. The tangent at any point ‘P’ on the standard ellipse with focii as S & S meets the tangents at the
vertices A & A in the points V & V, then :
(A) (AV) (A V) = b2 (B) (AV) (A V) = a2
(C) V SV = 90º (D) V S VS is a cyclic quadrilateral

23. Identify the statements which are True.


(A) the equation of the director circle of the ellipse, 5x 2 + 9y2 = 45 is x 2 + y2 = 14.
x2 y 2
(B) the sum of the focal distances of the point (0, 6) on the ellipse + = 1 is 10.
25 36
(C) the point of intersection of any tangent to a parabola & the perpendicular to it from the focus
lies on the tangent at the vertex.
(D) the line through focus and (at21, 2 at1) on y2 = 4ax, meets it again in the point (at22, 2 at2) iff
t1t2 =  1.

24. The Cartesian equation of the curve whose parametric equation is x = 2t – 3 and y = 4t2 – 1 is given by
(A) (x + 3)2 – y – 1 = 0 (B) x2 + 6x – y + 8 = 0
(C) (y + 1) + x + 3 = 0
2
(D) y2 + 6x – 2y + 4 = 0

x2 y2
25. If P is a point of the ellipse + = 1, whose focii are S and S. Let PSS =  and PSS = ,
a2 b2
then
(A) PS + PS = 2a, if a > b
(B) PS + PS = 2b, if a < b

  1 e
(C) tan tan =
2 2 1 e

  a2  b2
(D) tan tan = [a – a 2  b 2 ] when a > b
2 2 b2

26. If the distance between the focii of an ellipse is equal to the length of its latus rectum, the eccentricity
of the ellipse is :
5 1 5 1 5 2 2
(A) (B) (C) (D)
2 2 2 5 1

1. Let use consider an ellipse whose major and minor axis are 3x + 4y – 7 = 0 and 4x – 3y – 1 = 0
respectively 'P' be a variable point on the ellipse at any instance, it is given that distance of 'P' from
major and minor axis are 4 and 5 respectively. It is also given that maximum distance of 'P' from minor
axis is 5 2 , then find its eccentricity..

RESONANCE 20
2. Prove that the area of the triangle formed by the three points on an ellipse, whose eccentric angle are
  
  and  is 2 ab sin sin sin .
2 2 2

x2 y2
3. Find the equation of tangents to the ellipse + = 1 which passes through a point (15, – 4).
50 32

x2 y2 25
4. If 'P' be a moving point on the ellipse + = 1 in such a way that tangent at 'P' intersect x =
25 16 3
at Q then circle on PQ as diameter passes through a fixed point. Find that fixed point.

5. Any tangent to an ellipse is cut by the tangents at the ends of major axis in the points T and T . Prove
that the circle, whose diameter is T T  will pass through the foci of the ellipse.

x2 y2
6. If 3x + 4y = 12 intersect the ellipse + = 1 at P and Q, then find the point of intersection of
25 16
tangents at P and Q.

7. Find the equation of the largest circle with centre (1, 0) that can be inscribed in the ellipse
x 2 + 4y2 = 16.

x2 y2
8. If P is a variable point on the ellipse + = 1 whose foci are S and S, then prove that the locus
a 2
b2

2e
of the incentre of PSS is an ellipse whose eccentricity is where e is the eccentricity of the
1  e
given ellipse.

x2 y2
9. The tangent at a point P (a cos, b sin) of an ellipse + = 1, meets its auxiliary circle in two
a2 b2
points, the chord joining which subtends a right angle at the centre. Show that the eccentricity of the
ellipse is (1 + sin2)–1/2.

x2 y2
10. A circle of radius r is concentric with the ellipse + = 1. Prove that the common tangent is
a 2
b2

 r 2  b2 
inclined to the major axis at an angle tan–1  .
 a2  r 2 
 
11. ‘O’ is the origin & also the centre of two concentric circles having radii of the inner & the outer circle as
‘a’ & ‘b’ respectively. A line OPQ is drawn to cut the inner circle in P & the outer circle in Q. PR is drawn
parallel to the y axis & QR is drawn parallel to the x axis. Prove that the locus of R is an ellipse
touching the two circles. If the focii of this ellipse lie on the inner circle, find the ratio of inner : outer
radii & find also the eccentricity of the ellipse.

12. If any two chords be drawn through two points on the major axis of an ellipse equidistant from the
   
centre, show that tan . tan . tan . tan  1 , where , , ,  are the eccentric angles of the extremities
2 2 2 2
of the chords.

x2 y2
13. The tangent at a point P on the ellipse 
 1 intersects the major axis in T & N is the foot of the
a2 b2
perpendicular from P to the same axis. Show that the circle on NT as diameter intersects the auxiliary
circle orthogonally.

RESONANCE 21
x2 y2
14. Show that the equation of the tangents to the ellipse  = 1 at the points of intersection with the
a 2 b2

 x2 y2 
 
line, p x + q y + 1 = 0 is,  2  2  1 (p2 a2 + q2 b2  1) = (p x + q y + 1) 2.
a b 

15. Common tangents are drawn to the parabola y2 = 4x & the ellipse 3x 2 + 8y2 = 48 touching the parabola
at A & B and the ellipse at C & D. Find the area of the quadrilateral.

x 2
y 2
x2 y2
16. A tangent to the ellipse 2  2  1 meets the ellipse   ab
a b a b
in the points P and Q; prove that the tangents at P and Q are at right angles.

x2 y2
17. Let P be a point on the ellipse + = 1 for which the area of the PON is the maximum where O
a2 b2
is the origin and N is the foot of the perpendicular from O to the tangent at P. Find the maximum area
and eccentric angle of point P.

18. Find the equation of the largest circle with centre (1, 0) that can be inscribed in the ellipse
x 2 + 4 y2 = 16. [IIT - 1999]

x2 y2
19. Let P be point on the ellipse 2 + 2 = 1, 0 < b < a. Let the line parallel to yaxis passing through P
a b
meet the circle x 2 + y2 = a2 at the point Q such that P and Q are on the same side of xaxis. For two
positive real numbers r and s. Find the locus of the point R on PQ such that PR : RQ = r : s as P varies
over the ellipse. [IIT - 2001]

20. Prove that in an ellipse, the perpendicular from a focus upon any tangent and the line joining the centre
of the ellipse to the point of contact meet on the corresponding directrix. [IIT - 2002]

EXERCISE # 1 EXERCISE # 2
1. D 2. A 3. B 4. A 5. C 6. C 7. A  3
1. e   3. 4x + 5y = 40, 4x – 35y = 200.
 5
8. A 9. D 10. B 11. C 12. C 13. B 14. C
 25 16  1 1
15. A 16. A 17. B 18. A 19. A 20. B 21. C 4. (3, 0) 6.  ,  11. ,
 4 3  2 2
22. ACD 23. ACD 24. AB 25. ABD 26. BD
11 x2 y 2 ( r  s)2
18. (x  1)2 + y2 = 19. 2 + =1
3 a ( ra  sb) 2

RESONANCE 22
Hyperbola

The Hyperbola is a conic whose eccentricity is greater than


unity. (e > 1).

1. Standard Equation & Definition(s)

* So their combinations
with themselves and
with each other give
rise to endless
complexities, which x2 y2
Standard equation of the hyperbola is 2  2  1 ,
anyone who is to give a b
where b2 = a2 (e2  1).
a likely account of Eccentricity (e) :

reality must survey. b2  C.A


2

e =1+ 2 =1+  
 T.A
2

a
The Ti maeus
Foci : S  (ae, 0) & S  ( ae, 0).

Equations Of Directrices :
* Miracles are not to
a a
be multiplied beyond x= & x= .
e e

necessity. Transverse Axis :


The line segment AA of length 2a in which the foci S & S both
Got t f r i ed Lei bni z lie is called the transverse axis of the hyperbola.

Conjugate Axis :
The line segment BB between the two points B  (0,  b) &
B (0, b) is called as the conjugate axis of the hyperbola.

Principal Axes :
The transverse & conjugate axis together are called Principal
Axes of the hyperbola.

Vertices :
A (a, 0) & A ( a, 0)

RESONANCE 23
Focal Chord :
A chord which passes through a focus is called a focal chord.
Double Ordinate :
A chord perpendicular to the transverse axis is called a double ordinate.
Latus Rectum (  ) :
The focal chord perpendicular to the transverse axis is called the latus rectum.
2 b2

C . A .2

= = 2a (e2  1).


a T . A.
Note : (L.R.) = 2 e (distance from focus to directrix)
Centre :
The point which bisects every chord of the conic drawn through it is called the centre of the conic.
x2 y2
C  (0, 0) the origin is the centre of the hyperbola   1.
a2 b2
General Note :
Since the fundamental equation to the hyperbola only differs from that to the ellipse in having
b2 instead of b2 it will be found that many propositions for the hyperbola are derived from those for the
ellipse by simply changing the sign of b2.
Example :

Find the equation of the hyperbola whose directrix is 2x + y = 1, focus (1, 2) and eccentricity 3.
Solution.
Let P 9x,y) be any point on the hyperbola.
Draw PM perpendicular from P on the directrix.
Then by definition SP = e PM
 (SP) 2 = e2 (PM)2

 2 x  y  1
2

 (x – 1) + (y – 2) = 3 
2 2

 4 1 
 5 (x 2 + y2 – 2x – 4y + 5}
= 3 (4x 2 + y2 + 1 + 4xy – 2y – 4x)
 7x – 2y2 + 12xy – 2x + 14y – 22 = 0
2

which is the required hyperbola.


Example :
Find the eccentricity of the hyperbola whose latus rectum is half of its transverse axis.
Solution.

x2 y2
Let the equation of hyperbola be – =1
a2 b2
2b 2
Then transverse axis = 2a and latus–rectum =
a
2b 2 1
According to question = (2a)
a 2
 2b2 = a2 ( b2 = a2 (e2 – 1))
 2a (e – 1) = a2
2 2
 2e2 – 2 = 1
3
 e=
2
3
 e=
2
3
Hence the required eccentricity is .
2

RESONANCE 24
2. Conjugate Hyperbola :
Two hyperbolas such that transverse & conjugate axes of one hyperbola are respectively the conjugate
& the transverse axes of the other are called Conjugate Hyperbolas of each other.

x2 y2 x2 y2
eg.  2  1 &  2  2  1 are conjugate hyperbolas of each.
a 2
b a b
Note : (a) If e1 & e2 are the eccentrcities of the hyperbola & its conjugate then e12 + e22 = 1.
(b) The foci of a hyperbola and its conjugate are concyclic and form the vertices of a
square.
(c) Two hyperbolas are said to be similiar if they have the same eccentricity.
(d) Two similar hyperbolas are said to be equal if they have same latus rectum.
(e) If a hyperbola is equilateral then the conjugate hyperbola is also equilateral.
Example :
Find the lengths of transverse axis and conjugate axis, eccentricity, the co-ordinates of foci,
vertices, lengths of the latus-rectum and equations of the directrices of the following hyperbolas
16x2 – 9y2 = – 144.
Solution.
The equation 16x2 – 9y2 = –144 can be written as

x2 y2
– =–1
9 16

x2 y2
This is of the form – =–1
a2 b2
 a2 = 9, b2 = 16
 a = 3, b = 4
Length of transverse axis : The length of transverse axis = 2b = 8
Length of conjugate axis : The length of conjugate axis = 2a = 6
 a2   9 
1   5
Eccentricity : e =  b 2  = 1  16  =
  4
Foci : The coªordinates of the foci are (0, + be) i.e., (0, + 5)
Vertices : The co–ordinates of the vertices are (0, + b) i.e., (0, + 4)
2a 2
Length of latus–rectum : The length of latus–rectum =
b
2(3)2 9
=
4 2
Equation of directrices : The equation of directrices are
b
y=+
e
4
y=+
(5 / 4 )
16
y=+
5
Self Practice Problems :

1. Find the equation of the hyperbola whose focis are (6, 4) and (– 4, 4) and eccentricity is 2.
Ans. 12x2 – 4y2 – 24x + 32y – 127 = 0

2. Obtain the equation of a hyperbola with coordinates axes as principal axes given that the distances of
one of its vertices from the foci are 9 and 1 units.

x2 y2 y2 x2
Ans. – = 1, – =1
16 9 16 9

RESONANCE 25
x2 y2
3. The foci of a hyperbola coincide with the foci of the ellipse + = 1. Find the equation of the
25 9
hyperbola if its eccentricity is 2.
Ans. 3x2 – y2 – 12 = 0.

3. Auxiliary Circle :
A circle drawn with centre C & T.A. as a diameter is called the Auxiliary Circle of the hyperbola.
Equation of the auxiliary circle is x2 + y2 = a2.
Note from the figure that P & Q are called the "Corresponding Points" on the hyperbola & the auxiliary
circle.

4. Parametric Representation :
x2 y2
The equations x = a sec & y = b tan  together represents the hyperbola   1 where is a
a2 b2
parameter. The parametric equations; x = a cosh, y = b sinh also represents the same hyperbola.
Note that if P()  (a sec  b tan ) is on the hyperbola then;
Q()  (a cos  a sin ) is on the auxiliary circle.
The equation to the chord of the hyperbola joining two points with eccentric angles  &  is given by

x  y   
cos  sin  cos .
a 2 b 2 2

5. Position Of A Point 'P' w.r.t. A Hyperbola :


2 2
x y
The quantity S1  1  1  1 is positive, zero or negative according as the point (x 1, y1) lies inside,
a2 b2
on or outside the curve.

Example :
Find the position of the point (5, – 4) relative to the hyperbola 9x 2 – y2 = 1.

Solution.
Since 9 (5) 2 – (–4)2 = 1 = 225 – 16 – 1 = 208 > 0,
So the point (5,–4) inside the hyperbola 9x 2 – y2 = 1.

6. Line And A Hyperbola :


x2 y2
The straight line y = mx + c is a secant, a tangent or passes outside the hyperbola  1
a2 b2
according as : c 2 > or = or < a2 m 2  b2, respectively.

RESONANCE 26
7. Tangents :
x2 y2
(i) Slope Form : y = m x  a 2 m2  b 2 can be taken as the tangent to the hyperbola  2  1,
a 2
b
having slope 'm'.
x2 y2
(ii) Point Form : Equation of tangent to the hyperbola   1 at the point (x1, y1) is
a2 b2
xx1 yy1
  1.
2
a b2
x2 y2
(iii) Parametric Form : Equation of the tangent to the hyperbola   1 at the point.
a2 b2
x sec  y tan 
(a sec , b tan ) is   1.
a b

  2
cos 1  1   2 
2
Note : (i) Point of intersection of the tangents at 1 & 2 is x = a , y = b tan  2 
1   2  
cos
2
(ii) If |1 + 2| = , then tangetns at these points (1 & 2) are parallel.

(iii) There are two parallel tangents having the same slope m. These tangents touches the hyperbola
at the extremities of a diameter.
Example :

x2 y2
Prove that the straight line x + my + n = 0 touches the hyperbola – = 1 if a22 – b2 m 2 = n2.
a2 b2
Solution.
The given line is x + my + n = 0
 n
or y=– x–
m m
Comparing this line with y = Mx + c
n


M=–
and c = – ..........(1)
m m
This line (1) will touch the hyperbola

x2 y2
– = 1 if c 2 = a2M2 – b2
a2 b2

n2 a2 l 2
 = – b2
m2 m2
or a 2 2 – b2m 2 = n 2

Example :
Find the equation of the tangent to the hyperbola x 2 – 4y2 = 36 which is perpendicular to the line
x – y + 4 = 0.
Solution.
Let m be the slope of the tangent. Since the tangent is perpendicular to the line x – y = 0
 m × 1 = –1  m=–1
Since x2 – 4y2 = 36

x2 y2
or – =1
36 9

RESONANCE 27
x2 y2
Comparing this with – =1
a2 b2
 a2 = 36 and b2 = 9
So the equation of tangents are

y = (–1) x + 36  ( 1)2  9

y = –x + 27

 x+y+3 3 =0

Example :

x2 y2 y2 x2
Find the equation and the length of the common tangents to hyperbola – = 1 and – = 1.
a2 b2 a2 b2
Solution.
Tangent at (a sec  b tan ) on the 1st hyperbola is
x y
sec  – tan  = 1 .....(1)
a b
Similarly tangent at any point (b tan , a sec ) on 2nd hyperbolas is
y x
sec  – tan  = 1 .....(2)
a b
If (1) and (2) are common tangents then they should be identical. Comparing the co–effecients of x and y
sec  tan 
 = – .....(3)
a b

tan  sec 
and – =
b a

a
or sec  = – tan ......(4)
b
 sec2 – tan2 = 1

a2 b2
 tan2 – sec 2= 1 {from (3) and (4)}
b2 a2

a2 b2
or tan2 – (1+ tan 2) = 1
b2 a2

 a2 b2  b2
  
or  b 2 a 2  tan  = 1 + 2
2

  a

b2
tan2 =
a2  b2

a2
and sec2 = 1 + tan2  =
a2  b2
Hence the point of contanct are

   
 a2 b2   b2 a2 
 ,  and  ,  {from (3) and (4)}
 (a  b )
2 2
(a  b ) 
2 2
 (a  b )
2 2
(a  b ) 
2 2

RESONANCE 28
(a 2  b 2 )
Length of common tangent i.e., the distance between the above points is 2 and equation
(a 2  b 2 )
of common tangent on putting the values of sec and tan in (1) is
x y
+ =1 or x  y=+ (a 2  b 2 )
(a  b ) (a  b 2 )
2 2  2

Self Practice Problems :

x2 y2
1. Show that the line x cos  + y sin  = p touches the hyperbola – =1
a2 b2
if a2 cos 2  – b2 sin2  = p2.
Ans. p2 = a2 cos2 – b2 sin2

2. For what value of  does the line y = 2x + touches the hyperbola 16x 2 – 9y2 = 144 ?
Ans. =±2 5

3. Find the equation of the tangent to the hyperbola x2 – y2 = 1 which is parallel to the line 4y = 5x + 7.
Ans. 4y = 5x ± 3

8. NORMALS :
x2 y2
(a) The equation of the normal to the hyperbola  2  1 at the point P (x1, y1) on it is
a 2
b

a 2 x b2 y
 = a 2 + b 2 = a 2 e 2.
x1 y1

x2 y2
(b) The equation of the normal at the point P (a sec , b tan ) on the hyperbola  2 1
a 2
b

ax by
is  = a 2 + b 2 = a 2 e 2.
sec tan

(c) Equation of normals in terms of its slope 'm' are y = mx 


a 2

 b2 m
.
a 2  b2 m 2
Example :

x2 y2
A normal to the hyperbola – = 1 meets the axes in M and N and lines MP and NP are drawn
a2 b2
perpendicular to the axes meeting at P. Prove that the locus of P is the hyperbola
a2x 2 – b2y2 = (a2 + b2)2.
Solution.

x2 y2
The equation of normal at the point Q (a sec , b tan ) to the hyperbola – = 1 is
a2 b2
ax cos  + by cot  = a2 + b2 ........(1)
The normal (1) meets the x–axis in

 a 2  b2 
M  sec  , 0  and y–axis in

 a 

RESONANCE 29
 a2  b2 

N  0, b tan  

 
 Equation of MP, the line through M and
perpendicular to x–axis, is
 a2  b 2  ax

x= 
 sec  or sec  =
a  ( a  b2 )
........(2)
 
2

and the equation of NP, the line through N and perpendicular to the y–axis is
 a2  b 2
 by

y=  b
 tan  or tan  =
 (a  b 2 )
.........(3)
 
2

The locus of the point of intersection of MP and NP will be obtained by eliminating  from (2) and (3),
we have
sec2 – tan2 = 1

a2 x 2 b2 y 2
 – =1
(a 2  b 2 ) 2 (a 2  b 2 ) 2
or a2x 2 – b2y2 = (a2 + b2)2
is the required locus of P.

Self Practice Problems :


x2 y2
1. Prove that the line lx + my – n = 0 will be a normal to the hyperbola – =1
a2 b2

a2 b2 (a 2  b 2 ) 2
if – = .
2 m2 n2

a2 b2 (a 2  b 2 ) 2
Ans. – = .
2 m2 n2

2. Find the locus of the foot of perpendicular from the centre upon any normal to the hyperbola
x2 y2
– = 1.
a2 b2
Ans. (x 2 + y2)2 (a2y2 – b2x 2) = x2y2 (a2 + b2)

9. Pair of Tangents:
The equation to the pair of tangents which can be drawn from any point (x 1, y1) to the hyperbola
x 2  y = 1 is given by: SS = T² where :
2

a 2
b 2 1

x  y2 x1
2
y1
2
xx1 yy1
S a –1 ; S1 =  –1; T – – 1.
2
2
b2 a 2
b 2
a 2
b2

Example :
x2 y2
How many real tangents can be drawn from the point (4, 3) to the ellipse – =1. Find the
16 9
equation these tangents & angle between them.
Solution.
Given point P  (4, 3)

x2 y2
Hyperbola S – –1=0
16 9

RESONANCE 30
16 9
S1  – –1=–1<0
16 9

 Point P  (4, 3) lies outside the hyperbola.


 Two tangents can be drawn from the point P(4, 3).
Equation of pair of tangents is
SS1 = T 2

 x2 y2 
 1 . (– 1) =  4 x  3 y  1
2


 
 16 9 
   16 9 

x2 y2 x2 y2 xy x 2y
 – + +1= + +1– – +
16 9 16 9 6 2 3
 3x2 – 4xy – 12x + 16y = 0
4
 = tan–1  
3
Example :
x2 y2
Find the locus of point of intersection of perpendicular tangents to the hyperbola  =1
a2 b2
Solution.
Let P(h, k) be the point of intersection of two perpendicular tangents
equation of pair of tangents is SS1 = T 2

 x2 y2   h2 k 2 
 =  hx  ky  1
2


   1    1
 a2 b2   a2 b2 
     a2 b2 

x2  k2   h2 
  – y   + ........ = 0
2
  1  1
 b2   a2  .........(i)
a2   b2  
Since equation (i) represents two perpendicular lines

1  k2  1  h2 

  1 –   1 = 0
 b2   a2 
a2   b2  
 – k 2 – b2 – h2 + a2 = 0  locus is x2 + y2 = a2 – b2 Ans.

10. Director Circle :


The locus of the intersection point of tangents which are at right angles is known as the Director Circle
of the hyperbola. The equation to the director circle is : x 2 + y2 = a2  b2.
If b2 < a2 this circle is real.
If b2 = a2 (rectangular hyperbola) the radius of the circle is zero & it reduces to a point circle at the
origin. In this case the centre is the only point from which the tangents at right angles can be drawn to
the curve.
If b2 > a2, the radius of the circle is imaginary, so that there is no such circle & so no pair of tangents
at right angle can be drawn to the curve.

1 1 . Chord of Contact:
Equation to the chord of contact of tangents drawn from a point P(x 1, y1 ) to the hyperbola

x2 y2
– = 1 is
a2 b2

xx1 yy1
T = 0, where T= – –1
a 2
b2

RESONANCE 31
Example :
x2 y2
If tangents to the parabola y2 = 4ax intersect the hyperbola – = 1 at A and B, then find the
a 2
b2
locus of point of intersection of tangents at A and B.
Solution.
Let P  (h, k) be the point of intersection of tangents at A & B

xh yk
 equation of chord of contact AB is – =1 .............(i)
a2 b2
which touches the parabola
equation of tangent to parabola y2 = 4ax
a a
y = mx –  mx – y = – .............(ii)
m m
equation (i) & (ii) as must be same

a
1 
m m
 = =
 h   k 1
 2  2 
a   b 

h b
2
ak
 m= &m=– 2
k a2 b

hb 2 ak
 =–
ka 2
b2

b4
 locus of P is y2 = – .x Ans.
a3

1 2 . Chord with a given middle point:


x 2  y 2 = 1 whose middle point is (x , y ) is T = S ,
Equation of the chord of the hyperbola a 2 2
b 1 1 1

x1 y1 xx1 yy1
2 2
where S1 =  –1; T – – 1.
a 2
b 2
a 2
b2

Example :
x2 y2
Find the locus of the mid - point of focal chords of the hyperbola – = 1.
a2 b2
Solution.
Let P  (h, k) be the mid-point

xh yk h2 k2
 equation of chord whose mid-point is given – –1= – –1
a2 b2 a2 b2
since it is a focal chord,
 it passes through focus, either (ae, 0) or (–ae, 0)
If it passes trhrough (ae, 0)

ex x2 y2
 locus is = 2 – 2
a a b
If it passes through (–ae, 0)

ex x2 y2
 locus is – = 2 – 2 Ans.
a a b

RESONANCE 32
Example :
x2 y2
Find the condition on 'a' and 'b' for which two distinct chords of the hyperbola – = 1 passing
2a 2 2b 2
through (a, b) are bisected by the line x + y = b.
Solution.
Let the line x + y = b bisect the chord at P(, b – )
 equation of chord whose mid-point is P(, b – )
x y(b   ) 2 (b   ) 2
– = –
2a 2 2b 2 2a 2 2b 2
Since it passes through (a, b)
 (b   ) 2 (b   ) 2
 – = –
2a 2b 2a 2 2b 2
 1 1   1 1
2  2  2  +     = 0
a b  b a
1
 = 0,  =
1 1

a b
 a±b

Example :
x2 y2
Find the locus of the mid point of the chords of the hyperbola – = 1 which subtend a right angle
a2 b2
at the origin.
Solution.
let (h,k) be the mid–point of the chord of the hyperbola. Then its equation is

hx ky h2 k2 hx ky h2 k2
– –1= – –1 or – = – ........(i)
a2 b2 b2 b2 a2 b2 a2 b2
The equation of the lines joining the origin to the points of intersection of the hyperbola and the chored
(1) is obtained by making homogeneous hyperbola with the help of (1)

 hx ky 
2
 2  2
x 2
y 2
a b 
 – =
a2 b2  h2 k 2 
2
  
 a2 b2 
 

1  h2 k 2   h2 k 2 
2 2
1   h2 k2 2hk
 2  2
 2 x2 – 2  y2 = x2 + y2 – xy .......(2)
a a b   b  a2 b 2  a 2b 2
  a b
4 4

The lines represented by (2) will be at right angle if coefficient of x2 + coefficient of y2 = 0

 h2 k 2   h2 k 2 
2 2
1   – h – 1
2
  – k =0
2
  
a2  a2 b2  b2  a2 b2 
  a4   b4

 h2 k 2 
2
   1 1  h2 k2
   2  2 = + 4
 a2 b2  a b 
  a b
4

hence, the locus of (h,k) is

 x2 y2 
2
   1 1  x2 y2
  2  2 = +
 a2 b2  a b 
  a4 b4

RESONANCE 33
Self Practice Problem

x2 y2
1. Find the equation of the chord – = 1 which is bisected at (2, 1).
36 9
Ans. x = 2y

x2 y2
2. Find the point 'P' from which pair of tangents PA & PB are drawn to the hyperbola – = 1 in such
25 16
a way that (5, 2) bisect AB

 375 
Ans.  , 12 
 4 
3. From the points on the circle x 2 + y2 = a2, tangent are drawn to the hyperbola x2 – y2 = a2, prove that the
locus of the middle points of the chords of contact is the curve (x 2 – y2)2 = a2 (x 2 + y2).
Ans. (x 2 – y2)2 = a2 (x 2 + y2).

13. Diameter :
The locus of the middle points of a system of parallel chords with slope 'm' of an hyperbola is called its
diameter. It is a straight line passing through the centre of the hyperbola and has the equation

y =
b 2 x.
a 2m
NOTE : All diameters of the hyperbola passes through its centre.

14. Asymptotes :
Definition : If the length of the perpendicular let fall from a point on a hyperbola to a straight line tends
to zero as the point on the hyperbola moves to infinity along the hyperbola, then the straight line is
called the Asymptote of the hyperbola.

Equations of Asymptote :
x y x y
 0 and   0.
a b a b

x2 y2
Pair of asymptotes :  2  0.
a2 b
NOTE : (i) A hyperbola and its conjugate have the same asymptote.
(ii) The equation of the pair of asymptotes differs from the equation of hyperbola
(or conjugate hyperbola) by the constant term only.
(iii) The asymptotes pass through the centre of the hyperbola & are equally inclined
to the transverse axis of the hyperbola. Hence the bisectors of the angles between the
asymptotes are the principle axes of the hyperbola.
(iv) The asymptotes of a hyperbola are the diagonals of the rectangle formed by the lines
drawn through the extremities of each axis parallel to the other axis.
(v) Asymptotes are the tangent to the hyperbola from the centre.
(vi) A sim ple m ethod to find the co-ordinates of the centre of the hyperbola
expressed as a general equation of degree 2 should be remembered as:
Let f (x , y) = 0 represents a hyperbola.
f f f f
Find & .Then the point of intersection of =0& = 0 gives the centre of the hyperbola.
x y x y

RESONANCE 34
Example :
Find the asymptotes xy – 3y – 2x = 0.
Solution.
Since equation of a hyperbola and its asymptotes differ in constant terms only,
 Pair of asymptotes is given by xy – 3y – 2x +  = 0
where  is any constant such that it represents two straight lines.
 abc + 2fgh – af 2 – bg2 – ch2 = 0

 1
2
3 1
 0+2×– ×–1× –0–0–   =0
2 2 2
 =6
From (1), the asymptotes of given hyperbola are given by
xy – 3y – 2x + 6 = 0
or (y – 2) (x – 3) = 0
 Asymptotes are x – 3 = 0 and y – 2 = 0
Example :
The asymptotes of a hyperbola having centre at the point (1, 2) are parallel to the lines 2x + 3y = 0 and
3x + 2y = 0. If the hyperbola passes through the point (5, 3), show that its equation is
(2x + 3y – 8) (3x + 2y + 7) = 154
Solution.
Let the asymptotes be 2x + 3y +  = 0 and 3x + 2y + µ = 0. Since asymptotes passes through (1,2),
then  = – 8 and  = – 7
Thus the equation of asympotes are
2x + 3y – 8 = 0 and 3x + 2y – 7 = 0
Let the equation of hyperbola be
(2x + 3y – 8) (3x + 2y – 7) + v = 0 ......(1)
It passes through (5,3), then
(10 + 9 – 8) (15 + 6 – 7) + v = 0
 11 × 14 + v = 0
 v = – 154
putting the value of v in (1) we obtain
(2x + 3y – 8) (3x + 2y – 7) – 154 = 0
which is the equation of required hyperbola.

Self Practice Problems :

1. Show that the tangent at any point of a hyperbola cuts off a triangle of constant area from the asymptotes
and that the portion of it intercepted between the asymptotes is bisected at the point of contact.

15. Rectangular Or Equilateral Hyperbola :


The particular kind of hyperbola in which the lengths of the transverse & conjugate axis are equal is
called an Equilateral Hyperbola. Note that the eccentricity of the rectangular hyperbola is 2 .

Rectangular Hyperbola (xy = c2) :


It is referred to its asymptotes as axes of coordinates.

Vertices : (c , c) & ( c,  c);

Foci :   
2 c, 2 c &  2 c, 2 c , 
Directrices : x + y =  2 c
Latus Rectum (l ) :

 = 2 2 c = T.A. = C.A.

RESONANCE 35
Parametric equation x = ct, y = c/t, t  R – {0}
Equation of a chord joining the points P (t1) & Q(t2) is x + t1 t2 y = c (t1 + t2).
x y x
Equation of the tangent at P (x 1, y1) is  = 2 & at P (t) is + t y = 2 c.
x1 y 1 t
Equation of the normal at P (t) is x t  y t = c (t  1).
3 4

Chord with a given middle point as (h, k) is kx + hy = 2hk.


Example :
A triangle has its vertices on a rectangle hyperbola. Prove that the orthocentre of the triangle also lies
on the same hyperbola.
Solution.
Let "t1", "t2" and "t3" are the vertices of the triangle ABC, described on the rectangular hyperbola
xy = c2.

 c  c  
 Co–ordinates of A,B and C are  ct1,  ,  ct 2 ,  and  ct 3 , c  respectively
t2   
 t1    t3 
t3  t2 1
Now lope of BC is ct  ct = – t t
3 2 2 3
 Slope of AD is t2t3
Equation of Altitude AD is
c
y – t = t2t3(x – ct1)
1

or t1y – c = x t1t2t3 – ct12t2t3 .....(1)


Similarly equation of altitude BE is
t2y – c = x t1t2t3 – ct1t22t3 ......(2)
 c 
Solving (1) and (2), we get the orthocentre   t t t ,ct1t 2 t 3 
 12 3 
Which lies on xy = c 2.

Example :
A, B, C are three points on the rectangular hyperbola xy = c2, find
(i) The area of the triangle ABC
(ii) The area of the triangle formed by the tangents A, B and C.
Solution.

 c  c  c 
Let co–ordinates of A,B and C on the hyperbola xy = c2 are  ct1,  ,  ct 2 ,  and  ct 3 ,  respectively..

 t1   t2   t3 

c c c
ct1 ct 2 ct 3
1 t1 t2 t3
(i)  Area of triangle ABC = c + c + c
2 ct ct 3 ct1
t2 t3 t1
2

c 2 t1 t 2 t2 t3 t 3 t1
=     
2 t 2 t1 t3 t2 t1 t 3

c2
= t 2t  t 2t  t t 2  t2t  t t 2  t2t
2t1t 2 t 3 3 3 2 3 1 2 3 1 2 3 1 2

c2
= | (t1 – t2) (t2 – t3) (t3 – t1) |
2t1t 2 t 3

RESONANCE 36
(ii) Equations of tangents at A,B,C are
x + t12 – 2ct1 = 0
x + yt22 – 2ct2 = 0
and x + yt32 – 2ct3 = 0

2
1 t 12  2ct1
1
 Required Area = 2 | C C C | 1 t 2
2
 2ct 2 .........(1)
1 t 32  2ct 3
1 2 3

1 t12 1 t12 1 t12


where C1 = 2 , C2 = – 2 and C 3 =
1 t3 1 t3 1 t 22
 C1 = t32 – t22, C2 = t12 – t32 and C3 = t22 – t12

1
From (1) = 4c 2.(t1 – t2)2 (t2 – t3)2 (t3 – t1)2
2 ( t 23  t 22 ) ( t12  t 32 ) ( t 22  t12 )

( t1  t 2 ) ( t 2  t 3 ) ( t 3  t 1 )
= 2c 2 ( t  t ) ( t  t ) ( t  t
1 2 2 3 3 1)

 t1  t 2 ) ( t 2  t 3 ) ( t 3  t 1 )
 Required area is, 2c2 ( t  t ) ( t  t ) ( t  t )
1 2 2 3 3 1

Example :
Prove that the perpendicular focal chords of a rectangular hyperbola are equal.
Solution.
Let rectangular hyperbola is x2 – y2 = a2
Let equations of PQ and DE are
y = mx + c ......(1)
and y = m 1x + c 1 .......(2)
respectively.
Be any two focal chords of any rectangular hyperbola x2 – y2 = a2 through its focus. We have to prove
PQ = DE. Since PQ  DE.
 mm 1 = –1 ......(3)
Also PQ passes through S (a 2 ,0) then from (1),

0 = ma 2 +c
or c 2 = 2a2m 2 ......(4)
Let (x1,y1) and (x 2,y2) be the co–ordinates of P and Q then
(PQ)2 = (x 1 – x 22) + (y1 – y2)2 ......(5)
Since (x1,y1) and (x2,y2) lie on (1)
 y1 = mx 1 + c and y2 = mx 2 + c
 (y1 – y2) = m (x 1 – x2) .......(6)
From (5) and (6)
(PQ)2 = (x 1 – x2)2 (1 + m 2) .......(7)
Now solving y = mx + c and x 2 – y2 = a2 then
x 2 – (mx + c)2 = a2
or (m 2 – 1) x 2 + 2mcx + (a2 + c 2) = 0

2mc a2  c 2
 x1 + x2 = and x 1x 2 =
m2  1 m2  1
 (x 1 – x2)2 = (x 1 + x 2)2 – 4x 1x 2

4m 2c 2 4(a 2  c 2 )
= –
(m 2  1)2 (m 2  1)

RESONANCE 37
4 {a 2  c 2  a 2 m 2 }
=
(m 2  1)2

4a 2 (m 2  1)
= { c 2 = 2a2m 2}
(m 2  1)2

 m2  1

(PQ) = 4a  2

From (7),
 
2 2

 m 1 

 m12  1 
2

Similarly, (DE) = 4a  2
2 2 

 m1  1 

 
2
 1  1   1 
2

 m 
 
= 4a2   1  2  ( mm 1 = – 1)
  m  1 
   

 m2  1

= 4a  2


 m  1
2

= (PQ)2
Thus (PQ) 2 = (DE)2  PQ = DE.
Hence perpendicular focal chords of a rectangular hyperbola are equal.

15. Important Results :


x2 y2
 Locus of the feet of the perpendicular drawn from focus of the hyperbola  2  1 upon any tangent
a 2
b
is its auxiliary circle i.e. x 2 + y2 = a2 & the product of these perpendiculars is b2.
 The portion of the tangent between the point of contact & the directrix subtends a right angle at the
corresponding focus.
 The tangent & normal at any point of a hyperbola bisect the angle between the focal radii. This spells
the reflection property of the hyperbola as "An incoming light ray " aimed towards one focus is
reflected from the outer surface of the hyperbola towards the other focus. It follows that if an ellipse and
a hyperbola have the same foci, they cut at right angles at any of their common point.

x2 y2 x y 2 2
Note that the ellipse   1 & the hyperbola 2  2 = 1 (a > k > b > 0) are confocal
a 2
b 2
a k 2
k  b2
and therefore orthogonal.

RESONANCE 38
 The foci of the hyperbola and the points P and Q in which any tangent meets the tangents at the
vertices are concyclic with PQ as diameter of the circle.

 If from any point on the asymptote a straight line be drawn perpendicular to the transverse axis, the
product of the segments of this line, intercepted between the point & the curve is always equal to the
square of the semi conjugate axis.

 Perpendicular from the foci on either asymptote meet it in the same points as the corresponding
directrix & the common points of intersection lie on the auxiliary circle.
x2 y2
 The tangent at any point P on a hyperbola 2  2  1 with centre C, meets the asymptotes in Q and
a b
R and cuts off a  CQR of constant area equal to ab from the asymptotes & the portion of the tangent
intercepted between the asymptote is bisected at the point of contact. This implies that locus of the
centre of the circle circumscribing the  CQR in case of a rectangular hyperbola is the hyperbola itself
& for a standard hyperbola the locus would be the curve, 4 (a2x 2  b2y2) = (a2 + b2)2.
x2 y2
 If the angle between the asymptote of a hyperbola 2  2  1 is 2  then the eccentricity of the
a b
hyperbola is sec .
 A rectangular hyperbola circumscribing a triangle also passes through the orthocentre of this triangle.
   c 
If  c t i , c  i = 1, 2, 3 be the angular points P, Q, R then orthocentre is  t t t , ct1 t 2 t 3  .
 t i  1 2 3 
 If a circle and the rectangular hyperbola xy = c 2 meet in the four points t1, t2, t3 & t4, then
(a) t1 t2 t3 t4 = 1
(b) the centre of the mean position of the four points bisects the distance between the
centres of the two curves.
(c) the centre of the circle through the points t1, t2 & t3 is :
c  1 c1 1 1 
 2  t1  t 2  t 3  t t t , 2  t  t  t  t1  t 2  t 3 
  1 2 3   1 2 3 

Example :
A ray emanating from the point (5, 0) is incident on the hyperbola 9x 2 – 16y2 = 144 at the point P with
abscissa 8. Find the equation of the reflected ray after first reflection and point P lies in first quadrant.
Solution.
Given hyperbola is
9x2 – 16y2 = 144. This equation can be

x2 y2
rewritten as – =1 ......(1)
16 9
Since x co–ordinate of P is 8. Let y
co–ordinate of P ia .
 (8,) lies on (1)

64 2
 – =1
16 9
 2 = 27
 a=3 3 ( P lies in first quadrant)

Hence coªordinate of point P is (8,3 3 ).

 Equation of reflected ray passing through P (8,3 3 ) and S(–5,0)

03 3
 Its equation is y – 3 3 = (x – 8)
58

or 13y – 39 3 = 3 3 x – 24 3

or 3 3 x – 13y + 15 3 = 0.

RESONANCE 39
Part : (A) Only one correct option

1. An ellipse and a hyperbola have the same centre origin, the same foci and the minor-axis of the one is

1 1
the same as the conjugate axis of the other. If e1, e2 be their eccentricities respectively, then  2=
e1 e 2
2

(A) 1 (B) 2 (C) 4 (D) none

2. The line 5x + 12y = 9 touches the hyperbola x2 – 9y2 = 9 at the point


(A) (– 5, 4/3) (B) (5, – 4/3) (C) (3, – 1/2) (D) none of these

x2 y2 x 2 y2 1
3. If the foci of the ellipse  2 = 1 & the hyperbola  = coincide then the value of b2 is :
25 b 144 81 25
(A) 4 (B) 9 (C) 16 (D) none

4. The tangents from (1, 2 2 ) to the hyperbola 16x 2 – 25y2 = 400 include between them an angle equal
to:
   
(A) (B) (C) (D)
6 4 3 2

5. If P(x1, y1), Q(x 2, y2), R(x 3, y3) and S(x 4, y4) are four concyclic points on the rectangular hyperbola
xy = c 2, the coordinates of orthocentre of the PQR are
(A) (x 4, y4) (B) (x4, – y4) (C) (–x 4, – x 4) (D) (– x 4, – y4)

6. The asymptotes of the hyperbola xy = hx + ky are :


(A) x  k = 0 & y  h = 0 (B) x + h = 0 & y + k = 0
(C) x  k = 0 & y + h = 0 (D) x + k = 0 & y  h = 0

7. The combined equation of the asymptotes of the hyperbola 2x 2 + 5xy + 2y2 + 4x + 5y = 0 is


(A) 2x 2 + 5xy + 2y2 + 4x +5y + 2 = 0 (B) 2x 2 + 5xy + 2y2 + 4x +5y – 2 = 0
(C) 2x + 5xy + 2y = 0
2 2
(D) none of these

8. If the hyperbolas, x 2 + 3 x y + 2 y2 + 2 x + 3 y + 2 = 0 and x2 + 3 x y + 2 y2 + 2 x + 3 y + c = 0 are


conjugate of each other, then the value of ‘c‘ is equal to :
(A)  2 (B) 4 (C) 0 (D) 1

x2 y2
9. P is a point on the hyperbola 2  2 = 1, N is the foot of the perpendicular from P on the transverse
a b
axis. The tangent to the hyperbola at P meets the transverse axis at T. If O is the centre of the
hyperbola, then OT. ON is equal to :
(A) e2 (B) a2 (C) b2 (D)b2/a2

10. The locus of the foot of the perpendicular from the centre of the hyperbola xy = c2 on a variable tangent
is :
(A) (x 2  y2)2 = 4c2 xy (B) (x2 + y2)2 = 2c 2 xy (C) (x 2 + y2) = 4x 2 xy (D) (x 2 + y2)2 = 4c2 xy

x2 y2
11. If the chords of contact of tangents from two points (x1, y1) and (x2, y2) to the hyperbola – = 1 are
a2 b2
x1 x 2
at right angles, then is equal to
y1 y 2
a2 b2 b4 a4
(A) – (B) – (C) – (D) –
b2 a2 a4 b4

RESONANCE 40
12. The equations of the transverse and conjugate axes of a hyperbola are respectively x + 2y – 3 = 0,
2x – y + 4 = 0, and their respective lengths are 2 and 2/ 3 . The equation of the hyperbola is

2 3 2 3
(A) (x + 2y – 3)2 – (2x – y + 4)2 = 1 (B) (2x – y + 4)2 – (x + 2y – 3)2 = 1
5 5 5 5
(C) 2(2x – y + 4)2 – 3 (x + 2y – 3) 2 = 1 (D) 2(x + 2y – 3)2 – 3 (2x – y + 4) 2 = 1

13. The chord PQ of the rectangular hyperbola xy = a2 meets the x-axis at A; C is the mid point of PQ & 'O'
is the origin. Then the  ACO is :
(A) equilateral (B) isosceles (C) right angled (D) right isosceles.

14. The number those triangles that can be inscribed in the rectangular hyperbola xy= c 2 whose all sides
touch the parabola y2 = 4ax is :
(A) 0 (B) 1 (C) 2 (D) Infinite
15. The number of points from where a pair of perpendicular tangents can be drawn to the hyperbola,
x 2 sec 2   y2 cosec 2  = 1,  (0, /4), is :
(A) 0 (B) 1 (C) 2 (D) infinite

x2 y2 x2 y2
16. If hyperbola 2 – 2 = 1 passes through the focus of ellipse 2 + 2 = 1 then eccentricity of hyperbola is
b a a b
2
(A) 2 (B) (C) 3 (D) None of these
3

17. The transverse axis of a hyperbola is of length 2a and a vertex divides the segment of the axis between
the centre and the corresponding focus in the ratio 2 : 1, the equation of the hyperbola is :
(A) 4x2 – 5y2 = 4a2 (B) 4x2 – 5y2 = 5a2 (C) 5x2 – 4y2 = 4a2 (D) 5x2 – 4y2 = 5a2

x2 y2
18. If AB is a double ordinate of the hyperbola – = 1 such that OAB (O is the origin) is an
a2 b2
equilateral triangle, then the eccentricity ‘e’ of the hyperbola satisfies
2 2 2
(A) e > 3 (B) 1 < e < 2 (C) e = (D) e >
3 3 3

x2 y2
19. If x cos  + y sin  = p, a variable chord of the hyperbola – = 1 subtends a right angle at the
a2 2a 2
centre of the hyperbola, then the chords touch a fixed circle whose radius is equal to

(A) 2a (B) 3a (C) 2 a (D) 5a

x2 y2 1
20. Two conics – =1 and x2 = – y intersect if
a 2
b 2 b

1 1
(A) 0 < b  (B) 0 < a  (C) a2 < b2 (D) a2 > b2
2 2
x2 y2
21. Number of points on hyperbola – = 1 from where mutually perpendicular tangents can be drawn
a2 b2
to circle x2 + y2 = a2 (a > b) is
(A) 2 (B) 3 (C) infinite (D) 4

22. The normal to the rectangular hyperbola xy = c 2 at the point ‘t1’ meets the curve again at the point ‘t2’.
The value of t13t2 is
(A) –1 (B) –|c| (C) |c| (D) 1

RESONANCE 41
23. If the tangent and the normal to a rectangular hyperbola cut off intercepts x1 and x2 on one axis and
y1 and y2 on the other axis, then
(A) x 1y1 + x2y2 = 0 (B) x1y2 + x2y1 = 0 (C) x 1x 2 + y1y2 = 0 (D) none of these

24. If x = 9 is the chord of contact of the hyperbola x2 – y2 = 9, then the equation of the corresponding pair
of tangents is [IIT - 1999]
(A) 9x – 8y + 18x – 9 = 0
2 2
(B) 9x – 8y + 18x + 9 = 0
2 2

(C) 9x2 – 8y2 – 18x – 9 = 0 (D) 9x 2 – 8y2 + 18x + 9 = 0

Part : (B) May have more than one options correct

x2 y2
25. The value of m for which y = mx + 6 is a tangent to the hyperbola – = 1 is
100 49

 17   17   20   20 
(A)   (B) –   (C)   (D) –  
 20   20   17   17 

x2 y2
26. If (a sec , b tan ) and (a sec, b tan ) are the ends of a focal chord of – = 1, then
a2 b2
 
tan tan equals to
2 2

e 1 1 e 1 e e 1
(A) (B) (C) (D)
e 1 1 e 1 e e 1

27. A common tangent to 9x2 – 16y2 = 144 and x 2 + y2 = 9 is


3 15 2 15
(A) y = x+ (B) y = 3 x+
7 7 7 7

3 2 15
(C) y = 2 x + 15 7 (D) y = 3 x–
7 7 7

28. The equation of a hyperbola with co-ordinate axes as principal axes, if the distances of one of its
vertices from the foci are 3 & 1 can be :
(A) 3x2  y2 = 3 (B) x2  3y2 + 3 = 0 (C) x 2  3y2  3 = 0 (D) none

29. If (5, 12) and (24, 7) are the foci of a conic passing through the origin then the eccentricity of conic is

(A) 386 /12 (B) 386 /13 (C) 386 /25 (D) 386 /38
30. If the normal at P to the rectangular hyperbola x2  y2 = 4 meets the axes in G and g and C is the centre
of the hyperbola, then
(A) PG = PC (B) Pg = PC (C) PG = Pg (D) Gg = PC

31. The tangent to the hyperbola, x2  3y2 = 3 at the point  


3 , 0 when associated with two asymptotes
constitutes :
(A) isosceles triangle (B) an equilateral triangle

(C) a triangles whose area is 3 sq. units (D) a right isosceles triangle.

32. Which of the following equations in parametric form can represent a hyperbolic profile, where 't' is a
parameter.

a  1 b  1 tx y x ty
(A) x = t   & y = t   (B)  +t=0& + 1=0
2  t 2  t a b a b

t
(C) x = et + et & y = et  et (D) x 2  6 = 2 cos t & y2 + 2 = 4 cos2
2

RESONANCE 42
x2 y2
33. If a hyperbola passes through the focii of the ellipse + = 1. Its transverse and conjugate axes
25 16
coincide respectively with the major and minor axes of the ellipse and if the product of eccentricities of
hyperbola and ellipse is 1, then [IIT - JEE ]

x2 y2 x2 y2
(A) the equation of hyperbola is – =1 (B) the equation of hyperbola is – =1
9 16 9 25

(C) focus of hyperbola is (5, 0) (D) focus of hyperbola is (5 3 , 0)

1. For the hyperbola x2/100  y2/25 = 1, prove that

(i) eccentricity = 5 / 2
(ii) SA . SA = 25, where S & S are the foci & A is the vertex .

2. Chords of the hyperbola, x2  y2 = a2 touch the parabola, y2 = 4 a x. Prove that the locus of their middle
points is the curve, y2 (x  a) = x3.

3. Find the asymptotes of the hyperbola 2 x2  3 xy  2 y2 + 3 x  y + 8 = 0 . Also find the equation to the
conjugate hyperbola & the equation of the principal axes of the curve .

4. Given the base of a triangle and the ratio of the tangent of half the base angles. Show that the vertex
moves on a hyperbola whose foci are the extremities of the base.

x2 y2
5. If p1 and p2 are the perpendiculars from any point on the hyperbola  = 1 on its asymptotes, then
a 2 b2

1 1 1
prove that,  2  2.
p1 p2 a b

6. If two points P & Q on the hyperbola x2/a2  y2/b2 = 1 whose centre is C be such that CP is perpendicular

1 1 1 1
to CQ & a < b, then prove that   2  2 .
C P2 C Q2 a b

7. If the normal at a point P to the hyperbola x2/a2  y2/b2 = 1 meets the xaxis at G, show that
SG = e . SP, S being the focus of the hyperbola .

8. A transversal cuts the same branch of a hyperbola x 2/a2  y2/b2 = 1 in P, Pandthe asymptotes in Q, Q.
Prove that (i) PQ = PQ & (ii) PQ = PQ

9. If PSP & QSQ are two perpendicular focal chords of the hyperbola x 2/a2  y2/b2 = 1 then prove that

1 1
 is a constant .
 (PS) .  (SP )  (QS) .  (SQ )

RESONANCE 43
10. A line through the origin meets the circle x 2 + y2 = a2 at P & the hyperbola x 2  y2 = a2 at Q . Prove that
the locus of the point of intersection of the tangent at P to the circle and the tangent at Q to the
hyperbola is curve a4 (x2  a2) + 4 x 2 y4 = 0 .

11. Prove that the part of the tangent at any point of the hyperbola x 2/a2  y2/b2 = 1 intercepted between the
point of contact and the transverse axis is a harmonic mean between the lengths of the perpendiculars
drawn from the foci on the normal at the same point .

12. Let 'p' be the perpendicular distance from the centre C of the hyperbola x2/a2  y2/b2 = 1 to the tangent
drawn at a point R on the hyperbola . If S & S are the two foci of the hyperbola, then show that

 b2 
(RS + RS)2 = 4 a2  1   .
 p2 

13. Chords of the hyperbola x2/a2  y2/b2 = 1 are tangents to the circle drawn on the line joining the foci as
diameter . Find the locus of the point of intersection of tangents at the extremities of the chords .

14. A point P divides the focal length of the hyperbola 9x²  16y² = 144 in the ratio
SP : PS = 2 : 3 where S & S are the foci of the hyperbola. Through P a straight line is drawn at an
angle of 135° to the axis OX. Find the points of intersection of this line with the asymptotes of the
hyperbola.

15. The angle between a pair of tangents drawn from a point P to the parabola y2 = 4ax is 45º. Show that the
locus of the point P is a hyperbola. [IIT - 1998]

x2 y2
16. Tangents are drawn from any point on the hyperbola – = 1 to the circle x2 + y2 = 9. Find the locus of
9 4
mid-point of the chord of constant. [IIT - 2005]

EXERCISE # 1 EXERCISE # 2
1. A 2. B 3. C 4. D 5. D
3. x  2y + 1 = 0 ; 2x + y + 1 = 0 ;
2x2  3xy  2y2 + 3x  y  6 = 0 ;
6. A 7. A 8. C 9. B 10. D
3x  y + 2 = 0 ; x + 3y = 0
11. D 12. B 13. B 14. D 15. D
x2 y2 1  4 3
16. C 17. D 18. D 19. A 20. B 13.  4  2 14. ( 4, 3) &   ,  
a 4
b a  b2 7 7

21. D 22. A 23. C 24. B 25. AB

 x2  y2 
2
26. BC 27. BD 28. AB 29. AD 30. ABC x2 y2
16.  =  

9 4  9 
31. BC 32. ACD 33. AC

RESONANCE 44
Vector

1. Vectors & Their Representation:

Measurement does not


Vector quantities are specified by definite magnitude and definite
directions. A vector is generally represented by a directed line
segment, say AB . A is called the initial point & B is called the
necessarily mean
terminal point. The magnitude of vector AB is expressed by
progress.Failingthepos-
 AB .
sibilityofmeasuringthat
which you desire, the Zero Vector:
lust for measurement A vector of zero magnitude is a zero vector. i.e. which has the
may, for example, same initial & terminal point, is called a Zero Vector. It is denoted
by O. The direction of zero vector is indeterminate.
merely result in your
measuringsomethingelse Unit Vector:
A vector of unit magnitude in the direction of a vector a is called

- and perhaps forget-
ting the difference- or 
a
unit vector along a and is denoted by â symbolically,, â   .

in your ignoring some |a|
things because they Equal Vectors:
cannot be measured.
Two vectors are said to be equal if they have the same magnitude,
.....Geor ge Udny Yul e direction & represent the same physical quantity.

This,therefore,ismath- Collinear Vectors:


Two vectors are said to be collinear if their directed line segments
ematics: she reminds
are parallel irrespective of their directions. Collinear vectors are
youoftheinvisibleform also called parallel vectors. If they have the same direction they
of the soul; she gives are named as like vectors otherwise unlike vectors.

Symbolically, two non zero vectors a and b are collinear if and
light to her own discov- 

only if, a  Kb , where K  R

eries; she awakens the
mind and purifies the Vectors a = a1 î + a 2 ĵ + a 3k̂ and b = b1 î + b 2 ĵ + b 3k̂ are
 
intellect;shebringslight
a1 a2 a3
to our intrinsic ideas; collinear if b = b = b
1 2 3
she abolishes oblivion
and ignorance which are
Coplanar Vectors:
ours by birth
A given number of vectors are called coplanar if their line segments
........Proclus
are all parallel to the same plane. Note that “T WO VECTORS ARE
ALWAYS COPLANAR”.

RESONANCE 1
Solved Example
Find unit vector of î  2 ĵ  3k̂
Solution

a = î  2 ĵ  3k̂

if a = a x î + a y ĵ + a zk̂

ax  ay  az

then |a| =
2 2 2



|a| = 14

a 1 2 3
â = | a | = 14 î
– ĵ +
14 k̂
14

Solved Example
Find values of x & y for which the vectors

a = (x + 2) î – (x – y) ĵ + k̂

b = (x – 1) î + (2x + y) ĵ + 2 k̂ are parallel.
Solution
x2 yx 1
a and b are parallel if x  1 = 2x  y = 2
 

x = – 5, y = – 20

2. Angle Between two Vectors


It is the smaller angle formed when the initial points or the terminal points of the two vectors are
brought together. It should be noted that 0º    180º .

3. Addition Of Vectors:
 
& b are represented by OA & OB , then their sum a  b is a vector represented
  
If two vectors a



by OC , where OC is the diagonal of the parallelogram OACB.

a  b  b  a (commutative) (a  b)  c  a  ( b  c) (associativity)
         
 

a0  a  0a
    
a  ( a )  0  (  a )  a
    
 

| a  b | ||a|  |b||
  
|ab||a| |b|

 

 
a  b = | a |2  | b |2 2 | a || b | cos  where  is the angle between the vectors
  

 
a b
A vector in the direction of the bisector of the angle between the two vectors a & b is  
 
  . Hence
a b

 
bisector of the angle between the two vectors a and b is  a  b , where  R+. Bisector of the exterior
 


angle between a & b is  a  b ,   R+. 
 

RESONANCE 2
4. Multiplication Of A Vector By A Scalar:
If a is a vector & m is a scalar, then m a is a vector parallel to a whose modulus is m times that of
  
  
a . This multiplication is called SCALAR M ULTIPLICATION. If a and b are vectors & m, n are scalars, then:
m (a )  (a ) m  m a m (na )  n(m a )  (mn )a
     
 
(m  n ) a  m a  n a m (a  b )  m a  m b
   

Solved Example

If a  î  2 ĵ  3k̂ and b  2 î  4 ĵ  5k̂ represent two adjacent sides of a parallelogram, find unit vectors

parallel to the diagonals of the parallelogram.


Solution.
 
Let ABCD be a parallelogram such that AB = a and BC = b .

Then, AB + BC = AC


 
AC = a  b = 3 î  6 ĵ  2k̂

and AB + BD = AD
 AD + AD = AB
 
 BD = AD  AB = b  a
Now, AC = 3 î  6 ĵ  2k̂

 | AC | = 9  36  4 = 7

and, BD = î  2 ĵ  8k̂

 | BD | = 1  4  64 = 69

 
AC 1
 Unit vector along AC = = 3 î  6 ĵ  2k̂
| AC | 7

î  2 ĵ  8k̂ 
BD 1
and, Unit vector along BD = =
| BD | 69
Solved Example

ABCDE is a pentagon. Prove that the resultant of the forces AB , AE , BC , DC , ED and AC is 3 AC .


Solution.
Let R be the resultant force

 R = AB + AE + BC + DC + ED + AC

 R = ( AB + BC ) + ( AE + ED + DC ) + AC

= AC + AC + AC

= 3 AC . Hence proved.

Self Practice Problems :

1. Express :
(i) The vectors BC CA and AB in terms of the vectors OA , OB and OC

(ii) The vectors OA , OB and in terms of the vectors OC , OB and OC .

Ans. (i) BC  OC  OB , CA  OA  OC , AB  OB  OA

RESONANCE 3
2. Given a regular hexagon ABCDEF with centre O, show that
(i) OB – OA = OC – OD (ii) OD + OA = 2 OB + OF

(iii) AD + EB + PC = 4 AB
3. The vector  î  ĵ  k̂ bisects the angle between the vectors c and 3 î  4 ĵ . Determine the unit vector


along c .
1 2 14
Ans.  î  ĵ  k̂
3 15 15
4. The sum of the two unit vectors is a unit vector. Show that the magnitude of the their difference is 3.

5. Position Vector Of A Point:


 
let O be a fixed origin, then the position vector of a point P is the vector OP . If a and b are position
vectors of two points A and B, then,
 
AB = b  a = pv of B  pv of A.

DISTANCE FORMULA
  
Distance between the two points A (a) and B (b) is AB = a  b

SECTION FORMULA
 
If a and b are the position vectors of two points A & B then the p.v. of

 na  m b

a point which divides AB in the ratio m: n is given by: r  .
mn
 
ab
Note p.v. of mid point of AB = .
2
Solved Example
ABCD is a parallelogram. If L, M be the middle point of BC and CD, express AL and AM in terms of
3
AB and AD , also show that AL + AM = AC .
2
Solution.
Let the position vectors of points B and D be respectively b and a referred to A as origin of reference.
 

Then AC = AD + DC = AD + AB [ DC = AB ]
   
= d + b  AB = b , AD = d
 
i.e. position vector of C referred to A is d + b

 AL = p.v. of L, the mid point of BC .

=
1
2
[p.v. of D + p.v. of C] = 
1   
2

b  d  b = AB +
1
2 AD
1   
 1

AM = 2 d  d  b = AD + 2 AB

1  1 
 AL + AM = b +
 
d + d+ b
2 2
3  3 
= b + d
2 2
3   3
= (b + d ) = AC .
2 2

RESONANCE 4
Solved Example
If ABCD is a parallelogram and E is the mid point of AB, show by vector method that DE trisects and is
trisected by AC.
Solution.
 
Let AB = a and AD = b
  
Then BC = AD = b and AC = AB + AD = a + b
Also let K be a point on AC, such that AK : AC = 1 : 3

1 1  
or, AK = AC  AK = (a + b ) .........(i)
3 3
Again E being the mid point of AB, we have
1 
AE = a
2
Let M be the point on DE such that DM : ME = 2 : 1
 
AD  2AE ba
 AM = = ..........(ii)
1 2 3
From (i) and (ii) we find that
1  
AK = ( a + b ) = AM , and so we conclude that K and M coincide. i.e. DE trisect AC and is
3
trisected by AC. Hence proved.

Self Practice Problems

 
1. If a, b are position vectors of the points (1, –1), (–2, m), find the value of m for which a and b are
 

collinear.
Ans. m=2

2. The position vectors of the points A, B, C, D are î  ĵ  k̂ , 2 î  5 ĵ , 3 î  2 ĵ  3k̂ , î  6 ĵ  k̂ respectively..


Show that the lines AB and CD are parallel and find the ratio of their lengths.
Ans. 1:2

3. The vertices P, Q and S of a triangle PQS have position vectors p, q and s respectively..
  

(i) Find m , the position vector of M, the mid-point of PQ, in terms of p and q .
  


(ii) Find t , the position vector of T on SM such that ST : TM = 2 : 1, in terms of p, q and s .
  

(iii) If the parallelogram PQRS is now completed. Express r , the position vector of the point R in

 
terms of p, q and s

Prove that P, T and R are collinear.


1   1    1   
m = 2 (p  q) , t = 2 (p  q  s ) , r = 2 qps

Ans.
 

4. D, E, F are the mid-points of the sides BC, CA, AB respectively of a triangle. Show FE = 1/2 BC and

that the sum of the vectors AD , BE , CF is zero.

RESONANCE 5
5. The median AD of a triangle ABC is bisected at E and BE is produced to meet the side AC in F; show
that AF = 1/3 AC and EF = 1/4 BF.

6. Point L, M, N divide the sides BC, CA, AB of ABC in the ratios 1 : 4, 3 : 2, 3 : 7 respectively. Prove
that AL + BM + CN is a vector parallel to CK , when K divides AB in the ratio 1 : 3.

6. Scalar Product Of Two Vectors:


Geometrical interpretation of Scalar Product
Let a and b be vectors represented by OA and OB respectively. Let  be the angle between OA and
 

OB . Draw BL  OA and AM  OB.


From s OBL and OAM, we have OL = OB cos  and OM = OA cos . Here OL and OM are known as
   
projections of b on a and a on b respectively..
= | a | | b | cos 
  
Now, a.b

= | a | (OB cos  )

= | a | (OL)

  
= (Magnitude of a ) (Projection of b on a ) ........(i)
= | a | | b | cos  = | b | (| a | cos  )
   
Again a . b
 

= | b | (OA cos )


= | b | (OM)
  
= (magnitude of b ) (Projection of a on b ) ........(ii)
Thus geometrically interpreted, the scalar product of two vectors is the product of modulus of either
vector and the projection of the other in its direction.

 a.b
1. i.i = j.j = k.k = 1; i.j = j.k = k.i = 0  projection of a on b  

|b|
  
if a = a1i + a2j + a3k & b = b1i + b2j + b3k then a . b = a1b1 + a2b2 + a3b3

2.

b  b12  b 2 2  b 32

a  a1  a 2  a 3
 2 2 2
,

a.b
the angle  between a & b is given by cos   
   0 
3.
|a| |b|
   
4. a . b  a b cos  (0    ) ,

   
note that if  is acute then a . b > 0 & if  is obtuse then a . b < 0
2   
a . a  a  a 2 , a.b  b.a (commutative) 
      
a . (b  c )  a . b  a . c (distributive)
 
5.
    
6. a.b  0  a  b (a  0 b  0 )

(m a ) . b (m b) = m (a . b) (associative) where m is scalar..


    
7. =a .

Note :
  
Maximum value of a . b is  a  b 

(i)
  
Minimum value of a . b is –  a  b 

(ii)

(iii)
  
     
Any vector a can be written as, a = a . i i  a . j j  a . k k .
 

RESONANCE 6
Solved Example

Find the value of p for which the vectors a  3 î  2 ĵ  9k̂ and b  î  p ĵ  3k̂ are

(i) perpendicular (ii) parallel

3 î  2 ĵ  9k̂  . î  pĵ  3k̂  = 0


Solution.
 
 
(i) a.b = 0
 
ab
 3 + 2p + 27 = 0  p = – 15
We know that the vectors a = a1î  a 2 ĵ  a 3 k̂ and b = b1î  b 2 ĵ  b 3k̂ are parallel iff

(ii)

 
   
a  b  a1î  a 2 ĵ  a 3 k̂ =  b1î  b 2 ĵ  b 3k̂  a1 = b1, a2 = b2, a3 = b3

a1 a2 a3
 b = b = b ( =)
1 2 3

So, vectors a = 3 î  2 ĵ  9k̂ and b = î  p ĵ  3k̂ are parallel iff


 

3 2 9 2
= =  3=  p=2/3
1 p 3 p
Solved Example

If a + b + c = 0 , | a | = 3, | b | = 5 and | c | = 7, find the angle between a and b .
       

Solution.
We have, a  b  c  0
   

 ab = –c
  
 a  b  . a  b  =  c  .  c 
  2  2
 ab 
2   2
+ b
  
= | c |2 a + 2a . b = c

 2 
 b cos  = c
2  2
+ b +2 a
 
a

1 
 9 + 25 + 2 (3) (5) cos  = 49  cos  =  = .
2 3
Solved Example

Find the values of x for which the angle between the vectors a = 2x 2 î + 4x ĵ + k̂ and b = 7 î – 2 ĵ +
 

x k̂ is obtuse.
Solution.
 
a.b
The angle q between vectors a and b is given by cos  =  
 
|a||b|
Now,  is obtuse  cos  < 0
 
a.b
   <0
|a||b|


  
a.b < 0 [, | a |, | b |  0 ]

 14x2 – 8x + x < 0
1
 17x (2x – 1) < 0  x(2x – 1) < 0  0 < x <
2
Hence, the angle between the given vectors is obtuse if x  (0, 1/2)

RESONANCE 7
Solved Example
D is the mid point of the side BC of a triangle ABC, show that AB2 + AC2 = 2 (AD2 + BD2)
Solution.
We have

AB = AD + DB
 AB2 = ( AD  DB )2

= AD2 + DB + 2 AD . DB .............(i)
Also we have

AC = AD + DC

 AC2 = ( AD  DC)
2

= AD2 + DC2 + 2 AD . DC .............(ii)


Adding (i) and (ii), we get

AB2 + AC2 = 2AD2 + 2BD2 + 2 AD . (DB  DC )

= 2(DA2 + DB2), for DB + DC = 0

Solved Example
 
If a = î + ĵ + k̂ and b = 2 î – ĵ + 3 k̂ , then find
 
(i) Component of b along a .
 
(ii) Component of b perpendicular to along a .
Solution.
 
(i) Component of b along a is

 a.b  
 
   a
 | a |2 
 
 
Here a . b = 2 – 1 + 3 = 4
| a |2 = 3

 a.b  
 
   4  4
Hence  2  a = a = ( î + ĵ + k̂ )
| a |  3 3

 
 a.b  
 
     1
2 î  7 ĵ  5k̂

(ii) Component of b perpendicular to along a is b –  2  a. =
| a |  3
Self Practice Problems :
 
  |ab|
If a and b are unit vectors and  is angle between them, prove that tan

1. =   .
2 |ab|

2. Find the values of x and y is the vectors a = 3 î  xĵ  k̂ and b = 2 î  ĵ  yk̂ are mutually perpendicular
 

vectors of equal magnitude.


31 41
Ans. x=– , y=
12 12
Let a = x 2 î  2 ĵ  2k̂ , b = î  ĵ  k̂ and c = x 2 î  5 ĵ  4k̂ be three vectors. Find the values of x for which

3.

the angle between a and b is acute and the angle between b and c is obtuse.
   

Ans. (–3, –2)  (2, 3)

RESONANCE 8
4. The points O, A, B, C, D, are such that OA  a , OB  b , OC  2a  3b , OD  a  2b . Give that the
     

length of OA is three times the length of OB show that BD and AC are perpendicular..

5. ABCD is a tetrahedron and G is the centroid of the base BCD. Prove that
AB2 + AC2 + AD2 = GB2 + GC2 + GD2 + 3GA2

7. Vector Product Of Two Vectors:


 a b sin  n , where n is the unit
   
If a & b are two vectors &  is the angle between them then a x b
   
1.
 
vector perpendicular to both a & b such that a , b & n forms a right handed screw system.
  
  
a x b = area of the parallelogram whose two adjacent sides are represented by a & b .

2. Geometrically


3. î  î  ĵ  ĵ  k̂  k̂  0 ; î  ĵ  k̂, ĵ  k̂  î, k̂  î  ĵ

 î ĵ k̂
If a = a1 î +a2 ĵ + a3 k̂

& b = b1 î + b2 ĵ + b3 k̂ then a  b  a1 a 2 a 3
 
4.
b1 b 2 b3

a x b  b x a (not commutative)
   
5.

( ) ( )
(m a )  b = a  m b = m a  b (associative) where m is a scalar..
   
6.

a x ( b  c)  (a x b )  (a x c) (distributive)
      
7.

a x b  0  a & b are parallel (collinear) (a  0 , b  0 ) i.e. a  K b , where K is a scalar..


       
8.
 
axb
Unit vector perpendicular to the plane of a & b is n    
 
9.
axb


 
r axb
A vector of magnitude ‘r’ & perpendicular to the palne of a & b is   
   
axb

 
axb
If  is the angle between a & b then sin    
 

a b

 
If a , b & c are the pv’s of 3 points A, B & C then the vector area of triangle ABC =


1  
 
a x b  bx c  cx a . The points A, B & C are collinear if a x b  b x c  c x a  0
          
2
  1  
 Area of any quadrilateral whose diagonal vectors are d1 & d 2 is given by d1 x d 2
2
   
    2  2  2   2 a .a a . b
 Lagrange's Identity: for any two vectors a & b ;(a x b)  a b  (a . b)     
a .b b.b

RESONANCE 9
Solved Example
Find a vector of magnitude 9, which is perpendicular to both the vectors 4 î  ĵ  3k̂ and  2 î  ĵ  2k̂ .
Solution.
Let a = 4 î  ĵ  3k̂ and b =  2 î  ĵ  2k̂ . Then,
 

î ĵ k̂
4 1 3 = (2 – 3) î – (–8 + 6) ĵ + (4 – 2) k̂ =  î  2 ĵ  2k̂
ab =
 
2 1 2


 
| ab | = ( 1)2  2 2  2 2 = 3

 ab 
 
    9
 Required vector = 9   = 3 (  î  2 ĵ  2k̂ ) =  3 î  6 ĵ  6k̂
 | a  b | 
Solved Example
            
For any three vectors a, b, c . Show that a  (b  c )  b  (c  a)  c  (a  b)  0 .
Solution.
We have, a × (b  c ) + b × (c  a) + c × (a  b)
        

= a  b  a  c  b  c  b  a  c  a  c  b [Using distributive law]


           
   
= ab  ac  bc  ab  ac bc [ a  b  b  a etc]
           

Solved Example
| a  î |2 + | a  ĵ |2 + | a  k̂ |2 = 2 | a |2

For any vector a , prove that
   

Solution.
Let a = a1î  a 2 ĵ  a 3 k̂ . Then

a  î = (a1î  a 2 ĵ  a 3k̂ ) × î = a1 ( î  î ) + a2 ( ĵ  î ) + a3 (k̂  î ) = –a2 k̂  a 3 î


 | a  î |2 = a22 + a32

a  ĵ = (a1î  a 2 ĵ  a 3k̂ ) × ĵ = a1k̂  a 3 î


 | a  ĵ |2 = a21 + a32

 | a  k̂ |2 = a12 + a22

 | a  î |2 + | a  ĵ |2 + | a  k̂ |2 = a22 + a33 + a12 + a32 + a12 + a22


  


2 (d12 + a22 + a32) = 2 | a |2
Solved Example
  
Let OA = a , OB = 10 a + 2b and OC = b where O is origin. Let p denote the area of the quadrilateral

OABC and q denote the area of the parallelogram with OA and OC as adjacent sides. Prove that p = 6q.
Solution.
We have,
p = Area of the quadrilateral OABC
1
= | OB  AC |
2
1
= | OB  (OC  OA ) |
2
1   
= | (10a  2b)  (b  a) |

2

RESONANCE 10
1      
= | 10(a  b  10(a  a)  2(b  b)  2(b  a ) |
 
2
1    
= | 10(a  b)  0  0  2(a  b) |
2
and, q = Area of the parallelogram with OA and OC as adjacent sides
 
= | OA  OC | = | a  b | ........(ii)
From (i) and (ii), we get p = 6q

Self Practice Problems :

1. If p and q are unit vectors forming an angle of 30º; find the area of the parallelogram having a  p  2q
    


and b  2p  q as its diagonals.
 

Ans. 3/4 sq. units

      
Show that {( a + b + c ) × ( c – b )} . a = 2 [ a b c ].
 
2.

  
3. Prove that the normal to the plane containing the three points whose position vectors are a, b, c lies in
     
the direction b  c  c  a  a  b

4. ABC is a triangle and EF is any straight line parallel to BC meeting AC, AB in E, F respectively. If BR
and CQ be drawn parallel to AC, AB respectively to meet EF in R and Q respectively, prove that
 ARB = ACQ.

8. Scalar Triple Product:


a x b . c  a b c sin  cos  where
       
The scalar triple product of three vectors a , b & c is defined as:


     
 is the angle between a & b &  is the angle between a x b & c . It is also written as [ a b c ] and
 
spelled as box product.

 Scalar triple product geometrically represents the volume


of the parallelopiped whose three coterminous edges are
represented by a , b & c i. e. V  [ a b c ]
     

 In a scalar triple product the position of dot & cross can be interchanged i.e.

a . ( b x c )  (a x b). c OR [ a b c ]  [ b c a ]  [ c a b ]
            

a . (b x c)   a .( cx b) i. e. [ a b c ]   [ a c b ]
       

1 2 3 a a a

If a = a1i+a2j+a3k; b = b1i+b2j+b3k & c = c 1i+c 2j+c3k then [a b c]  b1 b 2 b 3 .
  

c1 c2 c3

In general, if a  a 1 l  a 2 m  a 3 n ; b  b1 l  b 2 m  b 3 n & c  c1 l  c2 m  c3 n
           

   l m n  ; where  , m & n are non coplanar vectors.


a1 a2 a3
then a b c  b1
  
b2 b3
c1 c2 c3

RESONANCE 11
If a , b , c are coplanar  [ a b c ]  0 .
   

Scalar product of three vectors, two of which are equal or parallel is 0 i.e. [ a b c ]  0 ,


If a , b , c are non  coplanar then [ a b c ]  0 for right handed system & [ a b c ]  0 for left handed
    

system.

 [ Ka b c ]  K[ a b c ]  [(a  b) c d ]  [ a c d ]  [ b c d ]
      
 [i j k] = 1
 
The volume of the tetrahedron OABC with O as origin & the pv’s of A, B and C being a , b & c respectively

1 
is given by V  [a b c]
6
   
 The positon vector of the centroid of a tetrahedron if the pv’s of its vertices are a , b , c & d are given by

1    
[a  b  c  d] .
4
Note that this is also the point of concurrency of the lines joining the vertices to the centroids of the
opposite faces and is also called the centre of the tetrahedron. In case the tetrahedron is regular it is

     
equidistant from the vertices and the four faces of the tetrahedron.

Remember that: a  b b c ca = 0 & a  b b c ca = 2 a b c .
          

Solved Example
Find the volume of a parallelopiped whose sides are given by  3 î  7 ĵ  5k̂ ,  5 î  7 ĵ  3k̂ and 7 î  5 ĵ  3k̂
Solution.

Let a  3 î  7 ĵ  5k̂ , b  5 î  7 ĵ  3k̂ and c  7 î  5 ĵ  3k̂ .
 

  
We know that the volume of a parallelopiped whose three adjacent edges are a, b, c is [a, b, c ] .
  

3 7 5
 5 7 3
Now, [a b c ] = = –3 (–21 – 15) – 7 (15 + 21) + 5 (25 – 49)
 
7 5 3
= 108 – 252 – 120 = –264
  
So, required volume of the parallelopiped = [a, b, c ] = | – 264 | = 264 cubic units
Solved Example
   
Simplify [a  b b  c c  a]
Solution.
We have :
[a  b b  c c  a] = {(a  b)  (b  c )} . (c  a)
       
[by def.]
 

= (a  b  a  c  b  b  b  c ) . ( c  a )
       
[by dist. law]
 

= (a  b  c  a  b  c ) . ( c  a )
       
[  bb  0 ]
 

= (a  b) . c – (a  b) . a + (c  a) . c – (c  a) . a + (b  c ) . c – (b  c ) . a
                 

[by dist. law]


       
= [a b c ] – [a b a] + [c a c ] – [c a a] + [b c c ] – [b c a]
 
= [a b c ] – [b c a]
[ scalar triple product when any two vectors are equal is zero ]
   
= [a b c ] – [a b c ] =0 [ [b c a] = [a b c ] ]

RESONANCE 12
Solved Example
   
Find the volume of the tetrahedron whose four vertices have position vectors a b c and d .
Solution.
   
Let four vertices be A, B, C, D with p. v. a b c and d . respectively..

 
DA = (a – d)
 
DB = (b – d)
 
DC = (c – d)

1      
Hence volume = [a – d b – d c – d]
6

1      
= ( a – d ) . [( b – d ) × ( c – d )]
6

1        
= (a – d ) . [b × c – b × d + c × d ]
6

1            
= {[ a b c ] – [ a b d ] + [ a c d ] – [ d b c ]}
6

1            
= {[ a b c ] – [ a b d ] + [ a c d ] – [ b c d ]}
6
Solved Example
         
Show that the vectors a  2 i  4 j  2k , b  4 i  2 j  2k and c  2 i  2 j  4k are coplanar..
 

Solution
2 4 2
4 2 2 =0

The vectors are coplanar since [a b c ] =
2 2 4
Self Practice Problems :
     
1. Show that a . (b  c )  (a  b  c )  0

2. One vertex of a parallelopiped is at the point A (1, –1, –2) in the rectangular cartesian co-ordinate. If three
adjacent vertices are at B(–1, 0, 2), C(2, –2, 3) and D(4, 2, 1), then find the volume of the parallelopiped.
Ans. 72

3. Find the value of m such that the vectors 2 î  ĵ  k̂ , î  2 ĵ  3k̂ and 3 î  m ĵ  5k̂ are coplanar..
Ans. –4
  
4. Show that the vector a, b, c , are coplanar if and only if b  c , c  a , a  b are coplanar..
     

9. Vector Triple Product:


   
Let a , b , c be any three vectors, then the expression a x ( b x c ) is a vector & is called a vector
 
triple product.

Geometrical Interpretation of a x ( b x c )
 

Consider the expression a x ( b x c ) which itself is a vector, since it is a cross product of two
 
    
vectors a & ( b x c ) . Now a x ( b x c ) is a vector perpendicular to the plane containing a & ( b x c )
   
  
but b x c is a vector perpendicular to the plane containing b & c , therefore a x ( b x c ) is a vector
   

which lies in the plane of b & c and perpendicular to a . Hence we can express a x ( b x c ) in terms
     

of b & c i.e. a x ( b x c ) = xb  yc where x & y are scalars.


      

RESONANCE 13
(a x b) x c = (a . c) b  ( b . c) a
     
(a . c) b  (a . b) c
          
a x ( b x c) =

 

(a x b) x c 
     
 a x (b x c) , in general

Solved Example
 
For any vector a , prove that î  (a  î )  ĵ  (a  ĵ )  k̂  (a  k ) = 2a
   

Solution.
Let a  a1î  a 2 ĵ  a 3 k̂ . Then, î  (a  î )  ĵ  (â  ĵ)  k̂  (a  k̂ )
  

= {( î . î )a  ( î . a) î } + {( ĵ . ĵ )a  ( ĵ . a) ĵ} + {(k̂ . k̂ )a  (k̂ . a)k̂ }


     

= {(a  ( î . a) î }  {a  ( ĵ . a) ĵ} + {a  (k̂ . a)k̂ }


     

= 3a  {( î . a) î  ( ĵ . a) ĵ  (k̂ . a)k̂
   

= 3a  (a1î  a 2 ĵ  a 3k̂ )

= 3a  a  2a
  

Solved Example
           
Prove that a  {b  (c  a )} = (b . d)(a  c ) – (b. c ) (a  d)
Solution.
We have,
         
a  {b  (c  a )} = a  {(b . d) c  (b . c ) d}

      
= a  {(b . d) c }  a  {(b . c ) d} [by dist. law]

         
= (b . d) (a  c )  (b . c )  (b . c ) (a  d)

Solved Example

Let a =  î + 2 ĵ – 3k̂ , b = î + 2 ĵ – 2k̂ and c = 2 î –  ĵ + k̂ . Find the value(s) of , if any,, such


  

that
 

ab  bc
 
  

×  c  a  = 0. Find the vector product when  = 0.

  a  b   b  c   ×  c  a 
Solution.

= a b c  b ×  c  a 
     

= a b c   a . b  c  b . c  a 
        

which vanishes if (i) a . b  c = b . c  a (ii) a b c  = 0


        

(i) a . b  c = b . c  a leads to the equation 2  + 10  + 12 = 0, 


      3 2
+ 6 = 0 and 6 – 6 = 0, which do

 
not have a common solution.
  
(ii) a b c = 0

 2 3
1 2 2 2
 =0  3 = 2  =
2  1 3

   
 
when  = 0, a b c = – 10, a . b = 6, b . c = 0 and the vector product is – 60 2 î  k̂ .
    

RESONANCE 14
Solved Example

 
  
ab  a
   
b  a  a | a |2 1  
If A  B = a , A . a = 1 and A  B = b , then prove that A =
       
 and B =  .
| a |2 | a |2
Solution.
  
Given A  B  a .....(i)


  
 
a. A B = a.a
 


   
a.Aa . B = a.a
 

 

 2
1 + a.B = |a|


   2   
a.B = |a| – 1 Given A  B  b

 


  
a A B = a × b

  
  
      
a .B A – a . A B  ab

  

| a |2 1 A  B = a  b
   
[using equation (2)]
solving equation (1) and (5), simultaneously, we get


  
ab  a 
 
 
b  a  a | a |2 1

A =  and B = 
| a |2 | a |2
Solved Example
Solve for r , the simultaneous equations r  b  c  b , r . a  0 provided a is not perpnedicular to b .
        

Solution
(r  c) × b = 0 
  
r  c and b are collinear
  

  r = c  kb ........(i)
 
r  c  kb
  



(c  kb ) . a = 0

r .a = 0
  

 
a.c  
  a.c 
 k=–   putting in (i) we get r c   b
a.b a.b
Solved Example
   
If x  a  k x  b , where k is a scalar and a, b are any two vectors, then determine x in terms of a, b
  

and k.
Solution
x  a  k x  b ..........(i)
   

Premultiple the given equation vectorially by a


a  ( x  a ) + k (a  x ) = a  b
      


 
(a . a) x  (a . x ) a  k(a  x )  a  b ..........(ii)
       

Premultiply (i) scalarly by a



 
[a x a] + k ( a . x ) = a . b
    

 
k(a . x )  a . b .......(iii)
 

Substituting x  a from (i) and a . x from (iii) in (ii) we get


   

    (a . b) 
 
1
x = a2  k 2 kb  (a  b)  a

 k 

RESONANCE 15
Self Practice Problems :
        
1. Prove that a  (b  c )  b  (c  a)  c  (a  b)  0 .

2. Find the unit vector coplanar with î + ĵ + 2 k̂ and î + 2 ĵ + k̂ and perpendicular to î + ĵ + k̂ .

1 1
Ans. ( – ĵ + k̂ ) or, ( ĵ – k̂ )
2 2

       
3. Prove that a  {a  (a  b)}  (a . a) (b  a) .

 1       1 
Given that x   2 (p . x ) p  q , show that p . x  p . q and find x in terms of p and q .
 
4.

p 2

  
5. If x . a = 0, x . b = 0 and x . c = 0 for some non-zero vector x , then show that [a b c ] = 0
    

           
( r . a ) (b  c ) ( r . b ) (c  a ) ( r . c ) (a  b )
6. Prove that r = + +

[abc ] [abc ] [abc ]
  
where a, b, c are three non-coplanar vectors

1 0 . Reciprocal System Of Vectors:


       
If a, b, c & a' , b' , c' are two sets of non coplanar vectors such that a.a' = b. b' = c. c' = 1 then the two
  
systems are called Reciprocal System of vectors.

     
     
 bxc  cxa  axb
Note: a=   b   c    
 
abc abc abc

Solved Example
If a b c and a, b, c  be the reciprocal system of vectors, prove that
   

(i) a . a  b . b  c . c   3 (ii) a  a  b  b  c  c   0
            

Solution.
a . a  = b . b = c . c  = 1
 
(i) We have :
   

a . a  + b . b + c . c  = 1 + 1 + 1 = 3
     

1
a =  (b  c ) , b =  (c  a) and c  =  ( (a  b) , where  = [a b c ]
    
(ii) We have :
    

a  a  a  (b  c )   {a  (b  c )}   {(a . c ) b  (a . b) c }
             

b  b  b  (c  a)   {b  (c  a )}   {(b . a) c  (b . c ) a}
             

c  c   c  (a  b)   {c  (a  b )}   {(c . b) a  (c . a ) b}
          
and
  

 a  a  b  b  c  c 
     

                 
=  {(a . c ) b  (a . b) c }   {(b . a) c  (b . c ) a}   {(c . b) a  (c . a) b}

=  [(a . c ) b  (a . b) c  (b . a ) c  (b . c ) a  (c . b ) a  (c . a) b]
                 

=  [(a . c ) b  (a . b) c  (a . b ) c  (b . c ) a  (b . c ) a  (a . c ) b]
                 

 
= 0  0

RESONANCE 16
1 1 . Linear Combinations:

Given a finite set of vectors a , b , c ,...... then the vector r  xa  yb  zc  ........ is called a linear
      
  
combination of a , b , c ,...... for any x, y, z..... R. We have the following results:

If a , b are non zero, noncollinear vectors then xa  yb  x' a  y' b  x  x' ; y  y'
     
(a)
 
FundamentalTheorem: Let a , b be non zero, non collinear vectors. Then any vector r coplanar

(b)
   
with a , b can be expressed uniquely as a linear combination of a , b

i.e. There exist some uniquly x, y  R such that xa  yb  r .


  

  
(c) If a , b , c are nonzero, noncoplanar vectors then:

xa  yb  zc  x' a  y' b  z' c  x  x' , y  y' , z  z'


     
  
(d) Fundamental Theorem In Space: Let a , b , c be nonzero, noncoplanar vectors in space. Then any
  
vector r , can be uniquly expressed as a linear combination of a , b , c i.e. There exist some unique x,y

 R such that xa  yb  zc  r .
   

If x1 , x 2 ,...... x n are n non zero vecto rs, & k 1 , k 2 ,. ... .k n are n scalars & if the linear
  
(e)
com bination k1x1  k 2 x 2 ........ k n x n  0  k1  0, k 2  0..... k n  0
  
then we sa y that
vectors x1 , x 2 ,...... x n are LINEARLY INDEPENDENT VECTORS.
  

If x1 , x 2 ,...... x n are not LINEARLY INDEPENDENT then they are said to be LINEARLY DEPENDENT vectors. i.e.
  
(f)
if k 1x1  k 2 x 2  ........  k n x n  0 & if there exists at least one k r  0 then x1 , x 2 ,...... x n are said to
     
be

L I N E A RLY D E P E N DE N T .

k1x1  k 2 x 2  k 3x 3  .......  k r x r  ......  k n x n  0


    
Note 1: If k r  0;

 k r x r  k 1x1  k 2 x 2  .......  k r 1 . x r 1  k r 1 . x r 1 ...... k n x n


     

1  1  1  1  1 
k r x r  k1 x1  k 2 x 2  ..... k r 1. x r 1 ..... k n xn
kr kr kr kr kr
x r  c1x1  c 2 x 2 ...... c r 1x r 1  c r x r 1 ...... c n x n
     

i.e. x r is expressed as a linear combination of vectors.


x1 , x 2 ,.......... x r 1 , x r 1 ,........... x n
    
     
Hence x r with x1 , x 2 ,.... x r 1 , x r 1 .... x n forms a linearly dependent set of vectors.
Note 2:

If a = 3 î + 2 ĵ + 5 k̂ then a is expressed as a LINEAR COMBINATION of vectors î , ĵ , k̂ Also, a , î , ĵ , k̂


  

form a linearly dependent set of vectors. In general, every set of four vectors is a linearly dependent
system.

 î , ĵ , k̂ are Linearly Independent set of vectors. For

K1 î + K2 ĵ + K3 k̂ = 0 K1= K2= K3 = 0

RESONANCE 17
Two vectors a & b are linearly dependent  a is parallel to b i.e. a x b  0   linear dependence
     

of a & b . Conversely if a x b  0 then a & b are linearly independent.


     

  
If three vectors a, b, c are linearly dependent, then they are coplanar i.e. [ a , b, c ]  0 , conversely,,
  

if [ a , b, c ]  0 , then the vectors are linearly independent.


  

Solved Example
     
Given A that the points a – 2 b + 3 c , 2 a + 3 b – 4 c , – 7 b + 10 c , A, B, C have position vector prove
 

that vectors AB and AC are linearly dependent.


Solution.
Let A, B, C be the given points and O be the point of reference then
     
OA = a – 2 b + 3 c , OB = 2 a + 3 b – 4 c OC = – 7 b + 10 c
 
and

Now AB = p.v. of B – p.v. of A


 
= OB – OA = ( a + 5 b – 7 c ) = – AB

 AC =  AB where  = – 1. Hence AB and AC are linearly dependent

Solved Example
     
Prove that the vectors 5 a + 6 b + 7 c , 7 a – 8 b + 9 c and 3 a + 20 b + 5 c are linearly dependent
  
  
a , b , c being linearly independent vectors.
Solution.
We know that if these vectors are linearly dependent , then we can express one of them as a linear
combination of the other two.
Now let us assume that the given vector are coplanar, then we can write
     
5 a + 6 b + 7 c = ( 7 a – 8 b + 9 c ) + m (3 a + 20 b + 5 c )
  

where , m are scalars


  
Comparing the coefficients of a , b and c on both sides of the equation
5 = 7 + 3 ..........(i)
6 = – 8 + 20 m ..........(ii)
7 = 9 + 5m ..........(iii)
From (i) and (iii) we get
1
4 = 8  = = m which evidently satisfies (ii) equation too.
2
Hence the given vectors are linearly dependent .

Self Practice Problems :

       
1. Does there exist scalars u, v, w such that ue1  ve 2  we 3  i where e1  k , e 2  j  k , e 3   j  2k ?
   

Ans. No

 
Consider a base a, b, c and a vector  2a  3b  c . Compute the co-ordinates of this vector relatively to
   
2.
     
the base p, q, r where p  2a  3b , q  a  2b  c , r  3a  b  2c .
    

Ans. (0, –7/5, 1/5)

  
If a and b are non-collinear vectors and A =(x + 4y) a + (2x + y + 1) b and B = (y – 2x + 2) a +

3.
  
 
(2x – 3y – 1) b , find x and y such that 3 A  2B .

Ans. x = 2, y = –1

RESONANCE 18
 
4. If vectors a, b,c be linearly independent, then show that

(i) a  2b  3c ,  2a  3b  4c ,  b  2c are linearly dependent


       

(ii) a  3b  2c ,  2a  4b  c , 3a  2b  c are linearly independent.


       

5. Given that î  ĵ , î  2 ĵ are two vectors. Find a unit vector coplanar with these vectors and perpendicular

to the first vector î  ĵ . Find also the unit vector which is perpendicular to the plane of the two given
vectors. Do you thus obtain an orthonormal triad?
1
Ans. ( î  ĵ ) ; k; Yes
es
2

 
6. If with reference to a right handed system of mutually perpendicular unit vectors î, ĵ, k̂   3 i  j ,

     
  2 i  j  3k express  in the form   1   2 where 1 is parallel to  and 2 is perpendicular to
    

.

3 1  1 3 
1  i  j ,  2  i  j  3k

Ans.
2 2 2 2

7. Prove that a vector r in space can be expressed linearly in terms of three non-coplanar, non-null

     


 [ r b c ] a  [ r c a] b  [ r a b] c
vectors a, b, c in the form r 
  

[a b c ]

Note: Test Of Collinearity:


  
Three points A,B,C with position vectors a, b, c respectively are collinear, if & only if there exist scalars

x , y, z not all zero simultaneously such that; xa  yb  zc  0 , where x + y + z = 0.


  

Note: Test Of Coplanarity:


   
Four points A, B, C, D with position vectors a, b, c, d respectively are coplanar if and only if there
 
exist scalars x, y, z, w not all zero simultaneously such that xa + yb + zc + wd = 0 where, x + y + z +
 
w = 0.

Solved Example
 
Show that the vectors 2a  b  3c , a  b  2c and a  b  3c are non-coplanar vectors.
      

Solution.
Let, the given vectors be coplanar.

   
Then one of the given vectors is expressible in terms of the other two.
   
Let 2a  b  3c = x a  b  2c + y a  b  3c , for some scalars x and y..
    

 2a  b  3c = (x + y) a (x + y) b + (–2x – 3y) c
     

 2 = x + y, –1 = x + y and 3 = 2x – 3y.
Solving, first and third of these equations, we get x = 9 and y = –7.
Clearly, these values do not satisfy the thrid equation.
Hence, the given vectors are not coplanar.

RESONANCE 19
Solved Example
  
Prove that four points 2a  3b  c , a  2b  3c , 3a  4b  2c and a  6b  6c are coplanar..
        

Solution.
Let the given four points be P, Q, R and S respectively. These points are coplanar if the vectors PQ ,

PR and PS are coplanar. These vectors are coplanar iff one of them can be expressed as a linear
combination of other two. So, let
PQ = x PR + y PS

    
  


 a  5b  4c = x a  b  c + y  a  9b  7c
 

  a  5b  4c = (x – y) a + (x – 9y) b + (–x + 7y) c
     

 x – y = –1, x – 9y = –5, –x + 7y = 4
  
[Equating coeff. of a, b, c on both sides]
1 1
Solving the first of these three equations, we get x = – ,y= .
2 2
These values also satisfy the third equation. Hence, the given four points are coplanar.

Self Practice Problems :

   
1. If, a, b, c, d are any four vectors in 3-dimensional space with the same initial point and such that

3a  2b  c  2d  0 , show that the terminal A, B, C, D of these vectors are coplanar. Find the point at
    

which AC and BD meet. Find the ratio in which P divides AC and BD.
 
2. Show that the vector a  b  c , b  c  a and 2a  3b  4c are non-coplanar, where a, b, c , are any non-
        

coplanar vectors.

3. Find the value of  for which the four points with position vectors  ĵ  k̂ , 4 î  5 ĵ  k̂ . 3 î  9 ĵ  4k̂ and

 4 î  4 ĵ  4k̂ are coplanar..


Ans. =1

1 2 . Application Of Vectors:

Work done against a constant force F over a displacement s is defined as W  F . s
   
(a)

The tangential velocity V of a body moving in a circle is given by V  w x r where r is the pv of the
   
(b)
point P.

The moment of F about ’O’ is defined as M  r x F where r is the pv of P wrt ’O’. The direction of M
     
(c)
  
is along the normal to the plane OPN such that r , F & M form a right handed system.

RESONANCE 20
Moment of the couple = ( r1  r2 ) x F

where r1 & r2 are pv’s of the point of the application of the forces
   
(d)

F & F .
 

Solved Example
Forces of magnitudes 5 and 3 units acting in the directions 6 î  2 ĵ  3k̂ and 3 î  2 ĵ  6k̂ respectively
act on a particle which is displaced from the point (2, 2, –1) to (4, 3, 1). Find the work done by the
forces.
Solution.
 
Let F be the resultant force and d be the displacement vector. Then,

(6 î  2 ĵ  3k̂ ) (3 î  2 ĵ  6k̂ ) 1
5 (39 î  4 ĵ  33k̂ )
F = +3 =

36  4  9 9  4  36 7

and, d = ( 4 î  3 ĵ  k̂ ) – (2 î  2 ĵ  k̂ ) = 2 î  ĵ  2k̂

1
 Total work done = F . d = (39 î  4 ĵ  33k̂ ) . (2 î  ĵ  2k̂ )
 
7
1 148
= (78 + 4 + 66) = units.
7 7
Self Practice Problems :

1. A point describes a circle uniformly in the î , ĵ plane taking 12 seconds to complete one revolution. If

its initial position vector relative to the centre is î , and the rotation is from î to ĵ , find the position

   
vector at the end of 7 seconds. Also find the velocity vector.
Ans. 1/2 ĵ  3 î , p/12 î  3 ĵ

2. The force represented by 3 î  2k̂ is acting through the point 5 î  4 ĵ  3k̂ . Find its moment about the

point î  3 ĵ  k̂ .

Ans. 2 î  20 ĵ  3k̂

3. Find the moment of the comple formed by the forces 5 î  k̂ and  5 î  k̂ acting at the points
(9, –1, 2) and (3, –2, 1) respectively
Ans. î  ĵ  5k̂

Miscellaneous Solved Examples


Solved Example
Show that the points A, B, C with posit ion vec tors 2 î  ĵ  k̂ , î  3 ĵ  5k̂ and 3 î  4 ĵ  4k̂
respectively, are the vertices of a right angled triangle. Also find the remaining angles of the triangle.
Solution.
We have,

AB = Position vector of B – Position vector of A

= ( î  3 ĵ  5k̂ ) – (2 î  ĵ  k̂ ) =  î  2 ĵ  6k̂

BC = Position vector of C – Position vector of B

= (3 î  4 ĵ  4k̂ ) – ( î  3 ĵ  5k̂ ) = 2 î  ĵ  k̂

RESONANCE 21
and, CA = Position vector of A – Position vector of C

= (2 î  ĵ  k̂ ) – (3 î  4 ĵ  4k̂ ) =  î  3 ĵ  5k̂

Since AB + BC + CA = (  î  2 ĵ  6k̂ ) + (2 î  ĵ  k̂ ) + (  î  3 ĵ  5k̂ ) = 0
So, A, B and C are the vertices of a triangle.
Now, BC . CA = (2 î  ĵ  k̂ ) . (  î  3 ĵ  5k̂ ) = –2 – 3 + 5 = 0

 BC  CA  BCA =
2
Hence, ABC is a right angled triangle.
Since a is the angle between the vectors AB and AC . Therefore

AB . AC (  î  2 ĵ  6k̂ ) . ( î  3 ĵ  5k̂ )
cos A = =
| AB | | AC | ( 1)2  ( 2)2  ( 6)2 12  ( 3)2  ( 5)2

1  6  30 35 35
= = =
1  4  36 1  9  25 41 35 41

35
A = cos –1
41

BA ( î  2 ĵ  6k̂ ) (2 î  ĵ  k̂ )
cos B = =
| BA | | BC | 1  2 2  6 2 2 2  ( 1)2  (1)2
2

226 6 6
 cos B = =  B = cos –1
41 6 41 41

Solved Example
  
If a, b, c are three mutually perpendicular vectors of equal magnitude, prove that a  b  c is equally
  

 
inclined with vectors a, b and c .

Solution.
Let | a | = | b | = | c | =  (say). Since a, b, c are mutually
     

     
perpendicular vectors, therefore a . b = b . c = c . a = 0 ..............(i)
   2
Now, abc
     
= a . a + b . b + c . c + 2a . b + 2b . c + 2c . a
     

 2 
= | a | | + | b | 2 + | c |2

[Using (i) ]

[ | a | = | b | = | c | = ]

= 3 2
 


  
| a  b  c | = 3 ..............(ii)
  
Suppose a  b  c makes angles 1, 2, 3 with a, b and c respectively. Then,
  

         
a . (a  b  c ) a.aa.ba.c
cos1 =     =    
|a||a bc | | a || a b  c |


 
| a |2 |a| 1
=    =   = = [Using (ii)]
|abc |

|a||a bc | 3

3

RESONANCE 22
 1 
 1 cos –1  

 3

 1   1 
Similarly, 2 = cos–1   and  = cos–1





 3 3
 3
  1 =  2 =  3.
    
Hence, a  b  c is equally inclineded with a, b and c

Solved Example
Prove using vectors : If two medians of a triangle are equal, then it is isosceles.

Solution.
Let ABC be a triangle and let BE and CF be two equal medians. Taking A as the origin, let the position
vectors of B and C be b and c respectively. Then,
 

1  1 
P.V. of E = c and, P.V. of F =
2 2 b
1 


BE = 2 (c  2b)

1 
CF = 2 (b  2c )

Now, BE = CF  | BE | = | CF |
2 2
1   1 
(c  2b) (b  2c )

 | BE | = | CF | 
2 2 =
2 2

1  1 
 | b  2c |2  | c  2b |2 = | b  2c |2
  
| c  2b |2 =
  
4 4

   
(c  2b) . (c  2b) = (b  2c ) . (b  2c )
   


       
c . c – 4b . c + 4b . b = b . b – 4b . c + 4c . c
   


       
| c |2 – 4b . c + 4 | b |2 = | b |2 – 4b . c + 4 | c |2

 
 
3 | b |2 = 3 | c |2 | b |2 = | c |2
 AB = AC Hence, triangle ABC is an isosceles triangle.

Solved Example
Using vectors : Prove that cos (A + B) = cos A cos B – sin A sin B
Solution.
Let OX and OY be the coordinate axes and let î and ĵ be unit vectors along OX and OY respectively..
Let XOP = A and XOQ = B. Drawn PL  OX and QM  OX.
Clearly angle between OP and OQ is A + B

In OLP, OL = OP cos A and LP = OP sin A. Therefore OL = (OP cos A) î and LP = (OP sin A)  ĵ  
Now. OL + LP = OP
 OP = OP [(cos A ) î – (sin A) ĵ ]
In OMQ, OM = OQ cos B and MQ = OQ sin B.
Therefore,

OM = (OQ cos B) î , MQ = (OQ sin B) ĵ

RESONANCE 23
Now, OM + MQ = OQ
From (i) and (ii), we get

OP . OQ = OP [(cos A) î – (sin A) ĵ ] . OQ [(cos B) î + (sin B) ĵ ]


= OP . OQ [cos A cos B – sin A sin B]
But, OP . OQ = | OP | | OQ | cos (A + B) = OP . OQ cos (A + B)
 OP . OQ cos (A + B) = OP . OQ [cos A cos B – sin A sin B]
 cos (A + B) = cos A cos B – sin A sin B

Solved Example
Prove that in any triangle ABC
(i) c 2 = a2 + b2 – 2ab cos C (ii) c = bcosA + acosB.
Solution.
(i) In ABC, AB + BC + CA = 0

or, BC + CA = – AB ......(i)
Squaring both sides
( BC )2 + ( CA )2 + ( BC ). CA + ( AB )2

 a2 + b2 + 2 ( BC . CA ) = c 2
 c 2 = a2 + b2 = 2 ab cos ( – C)
 c 2 = a2 + b2 – 2ab cosC
(ii) ( BC + CA ). AB = – AB . AB

BC . AB + CA . AB = – c 2
– ac cosB – bc cos A = – c 2
acosB + bcosA = c.

Solved Example
If D, E, F are the mid-points of the sides of a triangle ABC, prove by vector method that area of
1
DEF = (area of ABC)
4
Solution.

Taking A as the origin, let the position vectors of B and C be b and c respectively. Then, the position

1   1  1 
vectors of D, E and F are (b  c ) , c and 2 b respectively..
2 2

1  1   b
Now, DE =
2
c – 2 (b  c ) =
2
c

1  1  
and DF = – ( (b  c ) =
2 b 2 2

1   b   c 
 
1
 Vector area of DEF (DE  DF ) =
2 2  2 2 

1   1  1 ( AB  AC )
= (b  c ) =  
8 4 2 

1
= (vector area of ABC)
4
1
Hence, area of DEF = area of ABC.
4

RESONANCE 24
Solved Example
P, Q are the mid-points of the non-parallel sides BC and AD of a trapezium ABCD. Show that
APD = CQB.
Solution.
Let AB = b and AD = d
 

Now DC is parallel to AB  there exists a acalar t sush that DC = t DB = t b



 
AC = AD + DC = d  t b
1    1 
The position vectors of P and Q are (b  d  t b) and respectively..
2 2 d
Now 2 APD = AP × AD
1     1  
= (b  d  t b ) × d = (1 + t) (b  d)
2 2
1   
2 CQB = CQ × CB =  d  (d  tb × [b  (d  t b)]
  
Also
 2 

=  d  t b   d  (1  t ) b
 1 
 2 

1    
=  (1  t ) d  b  t b  d
2
1  
= (1  t  2t ) b  d
2
1  
= (1  t ) b  d = 2 APD
2
Hence the result.
Solved Example
Let u and v are unit vectors and w is a vector such that u  v  u = w and w  u  v then find the
         

value of u v w  .
  

Solution.
Given u  v  u = w and w  u  v
      

 u  v  u  × u  w  u
 u  v  × u  u  u = v (as, w  u  v )
      

 u . u  v  v . u  u  u  u  v (using u . u = 1 and u  u = 0, since unit vector)


     

 v  ( v . u) u  v
    

 u . v  u  0

 (as; u  0)
  
u.v = 0 .............(i)
 u . (v  w )
  

 u . ( v  (u  v  u)) (given w  u  v + u)
       

 u . ( v  (u  v )  v  u)
     

 u . (( v . v ) u  ( v . u) v  v  u)
        

 u . (| v |2 u  0  v  u) (as; u . v  0 from (i))


     

 | v |2 (u . u) – u . ( v  u)
     

 | v |2 | u |2 – 0 (as, u v u = 0)
    

1 (as; | u | = | v | = 1)
 

 u v w  = 1

RESONANCE 25
Solved Example
In any triangle, show that the perpendicular bisectors of the sides are concurrent.
Solution.
Let ABC be the triangle and D, E and F are respectively middle points of sides BC, CA and AB.
Let the perpendicular of D and E meet at O join OF. We are required to prove that OF is  to AB. Let the
  
position vectors of A, B, C with O as origin of reference be a , b and c respectively..
1   1  1  
 OD =

( b + c ), OE = ( c + a ) and OF = (a + b )
2 2 2
     
Also BC = c – b , CA = a – c and AB = b – a
1  
Since OD  BC,
 
(b + c ) . (c – b ) = 0
2
 b2 = c 2 ............(i)
1    
Similarly (c + a) . (a + c ) = 0
2
 a2 = c 2 ............(ii)
from (i) and (ii) we have a2 – b2 = 0
 
 (a + b ) . (b + a ) = 0
 

1  

 
(b + a) . (b – a) = 0
2

Solved Example
A, B, C, D are four points in space. using vector methods, prove that
AC2 + BD2 + AD2 + BC2  AB2 + CD2 what is the implication of the sign of equaility.
Solution.
   
Let the position vector of A, B, C, D be a , b , c and d respectively then
   

 
 
 
 
 
 
AC2 + BD2 + AD2 + BC2 = c  a  . c  a  + d  b . d  b + d  a . d  a + c  b .  
   
 c  b 
 2   
= | c |2 + | a | – 2 a . c + | d | 2 + | b | 2 – 2 d . b + | d | 2 + | a | – 2 a . d + | c | +
  2       2  
| b |2 – 2b . c
 

 2     2      2 
= | a | + | b |2 – 2 a . b + | c | + | d |2 – 2 c . d + | a | + | b |2 + | c | + | d |2
2

         
+ 2 a . b + 2c . d – 2a . c – 2b . d – 2a . d – 2b . c
 

        
= a  b . a  b + c  d . c  d + a  b  c  d  AB2 + CD2
           

   
= AB2 + CD2 + a  b  c  d . a  b  c  d  AB2 + CD2
       

 AC + BD + AD + BC  AB + CD
2 2 2 2 2 2

   
for the sign of equality to hold, a  b  c  d = 0
   
ac  db

 AC and BD are collinear the four points A, B, C, D are collinear

RESONANCE 26
Part : (A) Only one correct option

The lengths of the diagonals of a parallelogram constructed on the vectors p  2 a  b & q  a  2 b ,


     
1.
 
where a & b are unit vectors forming an angle of 60º are:

(A) 3 & 4 (B) 7 & 13 (C) 5 & 11 (D) none

   
2

2.  a  b  =
 a 2 2
 b

   a a  b b 
2   2
ab
(A) a  b (B)    
(C)  
2 2
(D) none
a b  a b 
 
   
   
3. A, B, C & D are four points in a plane with pv's a , b , c & d respectively such that

    
a  d . b  c  b  d .  c  a  = 0. Then for the triangle ABC, D is its:
       

(A) incentre (B) circumcentre (C) orthocentre (D) centroid

2
a  3 b x 3a  b
     2
Vectors a & b make an angle  = . If a = 1, b = 2 then

4. =
3
(A) 225 (B) 250 (C) 275 (D) 300
   
5. Consider a tetrahedron with faces f 1, f 2, f 3, f 4. Let a 1 , a 2 , a 3 , a 4 be the vectors whose magnitudes are
respectively equal to the areas of f 1, f 2, f 3, f 4 & whose directions are perpendicular to these faces in the
outward direction. Then,
(A) a 1  a 2  a 3  a 4 = 0 (B) a 1  a 3  a 2  a 4
       

(C) a 1  a 2  a 3  a 4
   
(D) none

        
6. For nonzero vectors a , b , c , a x b . c = a b c holds if and only if;
     
(A) a . b = 0, b . c = 0 (B) c . a = 0, a . b = 0
 
       
(C) a . c = 0, b . c = 0 (D) a . b = b . c = c . a = 0
 

     
a .a a .b a.c
If a  i  j  k , b  i  j  k , c  i  2 j  k , then the value of b . a b . b
        
7. b.c =
     
c.a c.b c.c
(A) 2 (B) 4 (C) 16 (D) 64

 (b  c)  (a  b)  c is true if:
     
If a , b & c are any three vectors, then a
  
8.
  
(A) b & c are collinear (B) a & c are collinear


(C) a & b are collinear

(D) none of these

 r . i i  r   r . j  j  r   r . k k  r =
  
9.
  
(A) 0 (B) r (C) 2 r (D) 3 r

RESONANCE 27
10. A point taken on each median of a triangle divides the median in the ratio 1: 3, reckoning from the vertex.
Then the ratio of the area of the triangle with vertices at these points to that of the original triangle is:
(A) 5: 13 (B) 25: 64 (C) 13: 32 (D) none

    
11. Given a parallelogram ABCD. If AB = a, AD = b & AC = c, then DB . AB has the value:

3 a 2  b 2  c2 a 2  3 b 2  c2 a 2  b 2  3 c2
(A) (B) (C) (D) none
2 2 2

12. The points whose position vectors are p i  q j  r k ; q i  r j  p k & r i  p j  q k are collinear if:
(A) p + q + r = 0 (B) p2 + q2 + r2  pq  qr  rp = 0
(C) p3 + q3 + r3  3 pqr = 0 (D) none of these

If p & s are not perpendicular to each other and r x p  q x p & r . s = 0, then r =


        
13.

  q . s    q . p 
   
(B) q      p (C) q      p (D) q   p for all scalars 
   
(A) p . s
 p . s  p . s

14. If a, b, c are pth, qth, rth terms of an H.P. and


  
  i j k
u = (q  r) i + (r  p) j + (p  q) k ,  =   , then:
  
a b c
(A) u ,  are parallel vectors (B) u ,  are orthogonal vectors
   

(C) u .  = 1 (D) u   = i + j + k
      

If p , q are two noncollinear and nonzero vectors such that b  c p  q  c  a p  a  b q  0 ,
 
15.
where a, b, c are the length of the sides of a triangle, then the triangle is
(A) right angled (B) obtuse angled (C) equilateral (D) isoceles

16. If cos  i  j  k , i  cos  j  k & i  j  cos  k (      2 n ) are coplanar then the value of

 2 2 2
cos ec 2  cos ec 2  cos ec 2  =
 
(A) 1 (B)  (C) 3 (D) none of these

If r x b  cx b & r . a  0 where a  2 i  3 j  k , b  3 i  j  k & c  i  j  3 k , then r is equal to:


         
17.


(A) 2 i  j  k  
(B) 2 i  j  k  
(C) 2  i  j  k  
(D) 2 i  j  k 
         
The value of d b c a  d c a b  d a b c  d a b c is equal to:
       
18.

(B) 2 a b c d (C) – 2 a b c d
   
(A) 0 (D) none of these

19. The three vectors i  j , j  k , k  i taken two at a time form three planes. The three unit vectors
drawn perpendicular to these three planes form a parallelopiped of volume:

1 3 3 4
(A) (B) 4 (C) (D)
3 4 3 3

RESONANCE 28
     
20. For any four points P, Q, R, S, P Q  R S  Q R  P S  R P  Q S is equal to 4 times the area of the
triangle:
(A) PQR (B) QRS (C) PRS (D) PQS
  
If a , b , c are three non  coplanar & p , q , r are reciprocal vectors, then:
  
21.

   
    

  a  m b  n c .   p  mq  n r is equal to:
   
(A) 2 + m 2 + n2 (B)  m + m n + n  (C) 0 (D) none of these

   2
| | | |
    
22. In a quadrilateral ABCD, A C is the bisector of the  A B A D which is , 15 AC = 3 A B
  3

| |     

= 5 A D then cos  BA C D is:
 

14 21 2 2 7
(A)  (B)  (C) (D)
7 2 7 3 7 14

23. | | | |
 
In the isosceles triangle ABC A B = BC = 8, a point E divides AB internally in the ratio 1: 3, then the

| |
  
cosine of the angle between C E & C A is (where CA = 12)

3 7 3 8 3 7 3 8
(A)  (B) (C) (D)
8 17 8 17

If p  3 a  5 b ; q  3 a  b ; r  a  4 b ; s   a  b are four vectors such that


           
24.


  
  
q = 1 and r  s = 1 then cos a  b is:  
   
sin p

19 19
(A)  (B) 0 (C) (D) 1
5 43 5 43
  
25. If p, q, r be three mutually perpendicular vectors of the same magnitude. If a vector x satisfies the

equation p ×  x  q  p  + q ×  x  r   q  + r  x  p   r  = 0 , then x is given by [IIT - 1997]


             

p  q  2r  p  q  r  p  q  r  2p  q  r 
1   1    1    1   
(A) (B) (C) (D)

2 2 3 3

 
Let a & b be two noncollinear unit vectors. If u  a  a . b b & v  a x b , then v is [IIT - 1999]
         
26.

(C) u  u . b
 
 
(D) u  u . a  b

(B) u  u . a
      
(A) u

   
     b.a    b.a 
27. If a , b , c are three non-zero, non coplanar vectors and b1 = b –  2 a , b 2 = b +  2 a ,
|a| |a|
 
b1.c 
       
c.a  b.c  c . a b .c 
 
 c.a  
 2 a +  2 b1 , c 2 = c –  2 a –  2 b1 , c 3 = c –  2 a +  2 b1 ,
   
c1 = c –

|a| |c| |a| | b1 | |c| |c|
   
 c.a  b.c 
c4

= c –  2 a –  2 b1 , then the set of orthogonal vectors is [IIT - 2005]
|c| |b|
           
(A) (a, b1, c 3 ) (B) (a, b1, c 2 ) (C) (a, b1, c 1 ) (D) (a, b 2 , c 2 )

RESONANCE 29

28. Let A be vector parallel to line of intersection of planes P1 and P2 through origin, P1 is parallel to the vectors
2 î + 3 k̂ and 4 ĵ – 3 k̂ and P2 is parallel to ĵ – k̂ and 3 î + 3 ĵ , then the angle between vector

A and 2 î + ĵ – 2 k̂ is [IIT - 2006]

   3
(A) (B) (C) (D)
2 4 6 4

Part : (B) May have more than one options correct

If a , b , c & d are linearly independent set of vectors & K1 a  K 2 b  K 3 c  K 4 d = 0 then:


       
29.
(A) K1 + K2 + K3 + K4 = 0 (B) K1 + K3 = K2 + K4 = 0
(C) K1 + K4 = K2 + K3 = 0 (D) none of these

  
30. Given three vectors a , b , c such that they are non  zero, non  coplanar vectors, then which of the
following are coplanar.

(A) a  b , b  c , c  a (B) a  b , b  c , c  a
           

(C) a  b , b  c , c  a (D) a  b , b  c , c  a
           

Let p  2 i  3 j  a k , q  b i  5 j  k & r  i  j  3 k . If p , q , r are coplanar and p . q = 20, a & b


       
31.
have the values:
(A) 1, 3 (B) 9, 7 (C) 5, 5 (D) 13, 9

If z1  a i  b j & z 2  c i  d j are two vectors in i & j system where z1  z 2


     
32. = r & z1 . z 2 = 0

then w1  a i  c j & w 2  b i  d j satisfy:


 

 
(A) w1 = r (B) w 2 = r
 
(C) w1 . w 2 = 0 (D) none of these

   
If a & b are two non colinear unit vectors & a , b , x a  y b form a triangle, then:
 
33.

   
   a b
(A) x =  1; y = 1 &  a + b  = 2 cos  
 2 
 

(B)
   

 

    


x =  1; y = 1 & cos  a b +  a + b  cos a ,  a  b  =  1 

       
   a b  a b
(C)  a + b  =  2 cot   cos   & x =  1, y = 1
 2   2 
   

(D) none

The value(s) of  [0, 2] for which vector a  i  3 j  sin 2  k makes an obtuse angle with the

34.

k and c   tan   i   tan   j  3 cos ec k are


  
Z-axis and the vectors b  (tan  ) i  j  2 sin
 
2 2
orthogonal, is/are:
(A) tan 1 3 (B)  tan 1 2 (C) + tan 1 3 (D) 2  tan 1 2

RESONANCE 30
A parallelogram is constructed on the vectors p & q . A vector which coincides with the altitude of the
 
35.

parallelogram & perpendicular to the side p expressed in terms of the vectors p & q is:
  

 p x q x p p x  p x q
     
 q.p 
 
q.p  
 

 p
(A) q   2 p (B) (C)  2 p  q (D)
p2 p p2
 

   a  b a 
36. Identify the statement(s) which is/are incorrect?

a x a ab
     2
(A) =

If a , b , c are non coplanar vectors and v . a  v . b  v . c = 0 then v must be a null vector


         
(B)
   
If a and b lie in a plane nor m al t o the plane c ontaining the vec tors c and d

a  b x c  d = 0
(C)
 
then

  
If a , b , c and a  , b  , c are reciprocal system of vectors then a . b   b . c  c . a  = 3
        
(D)


If a  i  2 j  4 k , b  2 i  3 j  k , c  i  4 j  4 k , then the vector a  b  c is orthogonal to: 
    
37.

(D) a  b  c
     
(A) a (B) b (C) c

   
38. If a , b , c are non-zero, non-collinear vectors such that a vector p = a b

   
cos 2  a   c and a vector q = a c cos   a  c    b then p + q is
   
b

 
(A) parallel to a (B) perpendicular to a

(C) coplanar with b & c

(D) none of these

  =0
39. Which of the following statement(s) is/are true?
  
If n . a = 0, n . b = 0 & n . c = 0 for some non zero vector n , then a b c
    
(A)
(B) there exist a vector having direction angles  = 30º &  = 45º
(C) locus of point for which x = 3 & y = 4 is a line parallel to the z - axis whose distance from the
z axis is 5
(D) the vertices of a regular tetrahedron are OABC where ' O ' is the origin. The vector
  
OA  OB  OC is perpendicular to the plane ABC.

40. In a  ABC, let M be the mid point of segment AB and let D be the foot of the bisector of  C. Then the
Area  CDM
ratio is:
Area  ABC
1 ab 1 ab
(A) (B)
4 ab 2 ab

1 AB AB 1 AB AB


(C) tan cot (D) cot tan
2 2 2 4 2 2

The vectors a , b , c are of the same length & pairwise form equal angles. If a  i  j & b  j  k , the
    
41.

pv's of c can be:
 4 1 4 1 4 1  1 4 1
(A) (1, 0, 1) (B)   , ,  (C)  , ,  (D)   , , 
 3 3 3 3 3 3  3 3 3

RESONANCE 31
1. Through the middle point M of the side AD of a parallelogram ABCD the straight line BM is drawn
cutting AC at R and CD produced at Q prove that QR = 2RB
 
Show that the perpendicular distance of the point c from the line joining a & b is,

2.

b c  ca  a b
     
.
ba
 

 
  
If   i  2 j  3 k ;   2 i  j  k ;   3 i  2 j  k and       p   q   r  then find the values
     
3.
of p, q, r

If a = 2 i  3 j  k , b = i  j  2 k
 , c = 2 i  j  k & d = 3 i  j  2 k , then find the value of
  
4.
 
( a x b ) x ( a x c ). d
  

Show that a  q  c  p  b   


 b   p  c   q  a   c   p  a   q  b  
              
5.
   
 bx c  cx a  axb
 
It is given that x     ; y     ; z     where a , b , c are non  coplanar vectors. Show
  
6.
[a b c] [a b c] [a b c]
that x , y , z also forms a non  coplanar system. Find the value of
  

x .(a  b )  y .( b  c )  z .( c  a ) .
        

7. The median AD of a triangle ABC is bisected at E and BE is produced to meet the side AC in F. Prove
that AF = (1/3) AC and EF = (1/4) BF.
8. Points X and Y are taken on the sides QR and RS, respectively of a parallelogram PQRS, so that
QX = 4XR and RY = 4YS. The line XY cuts the line PR at Z. Find the ratio PZ: ZR.
  
9. Forces P , Q act at O & have a resultant R . If any transversal cuts their line of action at A,B,C respectively,,
P Q R
then show that   .
OA OB OC
10. In a tetrahedron, if two pairs of opposite edges are perpendicular, then show that the third pair of
opposite edges is also perpendicular & in this case the sum of the squares of two opposite edges is
the same for each pair. Also show that the segment joining the mid points of opposite edges bisect one
another.
11. Use vectors to prove that the diagonals of a trapezium having equal non parallel sides are equal &
conversely.

     
Given four non zero vectors a , b , c and d . The vectors a , b & c are coplanar but not collinear pair by

12.
       
pair and vector d is not coplanar with vectors a , b & c and ( a b )  ( b c ) 

, (d a )  , (d b)  ,
3

prove that ( d c)  cos 1 (cos   cos  ) .
  
13. If p , q & r are three non-coplanar vectors, prove that,
  
p q r
1
ab =
 
q  r , r  p , p  q
     
      p . a q . a r .a
   
p.b q.b r.b

RESONANCE 32
   
Consider the non zero vectors a , b , c & d such that no three of which are coplanar then prove

            
14.
   
that a b cd  c a b d  b a c d  d a b c . Hence prove that a , b , c & d represent the position vectors
      

 b cd   a b d
  

a cd  a b c


 
of the vertices of a plane quadrilateral if and only if      1.

Solve the following equation for the vector p ; p x a   p . b c  b x c where a , b , c are non zero non coplanar
          
15.

    a b c  

vectors and a is neither perpendicular to b nor to c , hence show that  p x a    c is perpendicular
  
 a .c 
 
to b  c .
 

     
16. If a, b, c & a', b', c' are reciprocal system of vectors then prove that:
  
    a  b c
(ii) (a' x b' )(b' x c' )  (c' x a' ) 
       
(i) [a b c] [a' b' c' ]  1  .
[abc ]
   
17. Let A = 2i + k; B = i + j + k & C = 4i  3j + 7k. Determine a vector R satisfying
     
R xB  CxB&R.A 0

For any two vectors u & v , prove that


 
18. [IIT - 1998]

(a) (u. v) 2  | u  v|2  | u|2 | v|2 & (1  | u|2 )(1 | v|2 )  (1  u. v)2  | u  v  (u  v)|2
            
(b)

19. Let ABC and PQR be any two triangles in the same plane. Assume that the perpendiculars from the
points A, B, C to the sides QR, RP, PQ respectively are concurrent. Using vector methods or otherwise,
prove that the perpendiculars from P, Q, R to BC, CA, AB respectively are also concurrent.
[IIT - 2000]

20. Find 3 – dimensional vectors v 1, v 2 , v 3 satisfying


 
           
v 1 . v 1 = 4, v 1 . v 2 = –2, v 1 . v 3 = 6, v 2 . v 2 = 2, v 2 . v 3 = –5, v 3 . v 3 = 29. [IIT - 2001]

21. If û , v̂ , ŵ be three non-coplanar unit vectors with angles between û & v̂ is, between v̂ & ŵ is 
and between ŵ & û is . If a , b , c are the unit vectors along angle bisectors of ,  ,  respectively,,
  

then prove that,  a  b , b  c , c  a  = 161  û v̂ ŵ 2 sec  2  sec  2  sec  2  .
2 2 2
[IIT - 2003]

EXERCISE # 1 35. BD 36. ACD 37. AD 38. BC 39. ACD

1. B 2. B 3. C 4. D 5. A 6. D 40. BC 41. AD

7. C 8. B 9. A 10. B 11. A 12. B


EXERCISE # 2
13. B 14. B 15. C 16. B 17. C 18. A
3. p = 0; q = 10; r =  3
19. D 20. B 21. A 22. C 23. C 24. C
4. – 98 6. 3
25. B 26. A 27. B 28. D 29. ABC
30. BCD 31. AD 32. ABC 33. AB 34. BD 20. v 1  2 î , v 2   î  ĵ , v 3  3 î  2 ĵ  4k̂ are some
  

possible values

RESONANCE 33
THREE DIMENSIONAL
GEOMETRY

Coordinate of a point in space


There are infinite number of points in space. We want to identify each
and every point of space with the help of three mutually perpendicular

But
coordinate axes OX, OY and OZ.
when gr eat and
Three mutually perpendicular lines OX, OY, OZ are considered as the
i ngeni ous ar t i st s three axes.

behol d t hei r so i nept The plane formed with the help of x and y axes is called x-y plane,
similarly y & z axes form y-z plane and z and x axes form z - x plane.
per f or mances , not
Consider any point P in the space, Drop a perpendicular from that
undeser vedl y do t hey
point to x -y plane, then the algebraic length of this perpendicular is
r i di cul e t he bl i ndness of considered as z-coordinate and from foot of the perpendicular drop
such men; si nce sane
perpendiculars to x and y axes. These algebraic lengths of
perpendiculars are considered as y and x coordinates respectively.
j udgment abhor s not hi ng so

much as a pi ct ur e

per pet r at ed wi t h no

t echni cal knowl edge,

al t hough wi t h pl ent y of

car e and di l i gence. Now t he

sol e r eason why pai nt er s of

t hi s sor t ar e not awar e of 1. Vector representation of a point in space


t hei r own er r or i s t hat t hey If coordinate of a point P in space is (x, y, z) then the position
have not l ear nt Geomet r y , vector of the point P with respect to the same origin is
wi t hout whi ch no one can x î + y ĵ + z k̂ .
ei t her be or become an
2. Distance formula
absol ut e ar t i st ; but t he
Distance between any two points (x1, y1, z1) and (x 2, y2, z2) is
bl ame f or t hi s shoul d be

l ai d upon t hei r mast er s, given as ( x 1  x 2 ) 2  ( y 1  y 2 ) 2  ( z1  z 2 ) 2

who ar e t hemsel ves Vector method

i gnor ant of t hi s ar t . We know that if position vector of two points A and B are

Al br echt Dür er given as OA and OB then

AB = | OB – OA |
 AB = |(x2i + y2 j + z2k) – (x1i + y1j + z1k)|

 AB = ( x 2  x 1 ) 2  ( y 2  y 1 ) 2  ( z 2  z1 ) 2

RESONANCE 34
3. Distance of a point P from coordinate axes
Let PA, PB and PC are distances of the point P(x, y, z) from the coordinate axes OX, OY and OZ
respectively then

PA = y 2  z 2 , PB = z 2  x 2 , PC = x2  y2

Example :
Show that the points (0, 7, 10), (– 1, 6, 6) and (– 4, 9, 6) form a right angled isosceles triangle.
Solution
Let A  (0, 7, 10), B  (–1, 6, 6), C  (– 4, 9, 6)
AB2 = (0 + 1)2 + (7 – 6)2 + (10 – 6)2 = 18  AB = 3 2
Similarly  BC = 3 2 ,
& AC = 6
Clearly AB2 +BC2 = AC2   ABC = 90°
Also AB = BC
Hence ABC is right angled isosceles.
Example :
Show by using distance formula that the points (4, 5, –5), (0, –11, 3) and (2, –3, –1) are collinear.
Solution
Let A  (4, 5, –5), B  (0, –11, 3), C  (2, –3, –1).

AB = ( 4  0 )2  (5  11)2  ( 5  3)2  336  4  84  2 84

BC = (0  2)2  ( 11  3)2  (3  1)2  84

AC = ( 4  2)2  (5  3)2  ( 5  1)2  84


BC + AC = AB
Hence points A, B, C are collinear and C lies between A and B.
Example :
Find the locus of a point which moves such that the sum of its distances from points A(0, 0, –) and
B(0, 0, ) is constant.
Solution.
Let the variable point whose locus is required be P(x, y, z)
Given PA + PB = constant = 2a (say)

 ( x  0 ) 2  ( y  0 )2  ( z   ) 2 + ( x  0 )2  ( y  0)2  ( z   )2 = 2a


1
2
= 2a – x 2  y 2  ( z   )2
 x 2 + y2 + z2 + 2 + 2z = 4a2 + x 2 + y2 + z2 + 2 – 2z

– 4a x 2  y 2  ( z   )2

 4z– 4a2 = – 4a x 2  y 2  ( z   )2

z 2 2
 + a2 – 2z = x 2 + y2 + z2 + 2 – 2z
a2

 2  x2  y 2 z2
or, 2 2 2
x + y + z 1  2  = a2 – 2 or,  =1
 a  a2   2 a2
This is the required locus.

RESONANCE 35
Self practice problems :
1. One of the vertices of a cuboid is (1, 2, 3) and the edges from this vertex are along the +ve x-axis,
+ve y-axis and +z axis respectively and are of length 2, 3, 2 respectively find out the vertices.
Ans. (1, 2, 5), (3, 2, 5), (3, 2, 3), (1, – 1, 3), (1, – 1, 5), (3, – 1, 5), (3, – 1, 3).

2. Show that the points (0, 4, 1), (2, 3, –1), (4, 5, 0) and (2, 6, 2) are the vertices of a square.

3. Find the locus of point P if AP2 – BP2 = 18, where A  (1, 2, – 3) and B  (3, – 2, 1)
Ans. 2x – 4y + 4z – 9 = 0

4. Section Formula
If point P divides the distance between the points A (x 1, y1, z1) and B (x2, y2, z2) in the ratio of m : n, then
coordinates of P are given as

 mx 2  nx 1 my 2  ny 1 mz 2  nz1 
 , , 
 mn mn mn 
Note :- Mid point

 x1  x 2 y1  y 2 z1  z 2 
 , , 
 2 2 2 

5. Centroid of a triangle

 x 1  x 2  x 3 y1  y 2  y 3 z1  z 2  z 3 
G  , , 
 3 3 3 

6. Incentre of triangle ABC


 ax 1  bx 2  cx 3 ay 1  by 2  cy 3 az1  bz 2  cz 3 
 , , 
 abc abc abc 
Where AB = c, BC = a, CA = b

7. Centroid of a tetrahedron
A (x1, y1, z1) B (x2, y2, z2) C (x3, y3, z3) and D (x 4, y4, z4) are the vertices of a tetrahedron then coordinate
of its centroid (G) is given as


  x ,  y ,  z 
i i i
 4 4 4 
 

Example :
Show that the points A(2, 3, 4), B(–1, 2, –3) and C(–4, 1, –10) are collinear. Also find the ratio in which
C divides AB.
Solution
Given A  (2, 3, 4), B  (–1, 2, –3), C  (– 4, 1, –10).

A (2, 3, 4) B (–1, 2, –3)

RESONANCE 36
Let C divide AB internally in the ratio k : 1, then

  k  2 2k  3  3k  4 
C  , , 
 k 1 k 1 k 1 

k  2
 =–4 3k = – 6 k = –2
k 1

2k  3 3k  4
For this value of k, = 1, and = –10
k 1 k 1
Since k < 0, therefore k divides AB externally in the ratio 2 : 1 and points A, B, C are collinear.
Example :

The vertices of a triangle are A(5, 4, 6), B(1, –1, 3) and C(4, 3, 2). The internal bisector of  BAC meetss
BC in D. Find AD.
A(5, 4, 6)
Solution
AB = 42  52  32  5 2

AC = 12  12  4 2  3 2

Since AD is the internal bisector of  BAC B D C


BD AB 5 (1, –1, 3) (4, 3, 2)
  
DC AC 3
 D divides BC internally in the ratio 5 : 3

 5  4  3  1 5  3  3( 1) 5  2  3  3 
 D  , , 
 53 53 53 

 23 12 19 
or, D=  , , 
 8 8 8 

 23   12   19 
2 2 2

 AD = 5    4    6  
 8   8   8 

1530
= unit
8

Example :
If the points P, Q, R, S are (4, 7, 8), (– 1, – 2, 1), (2, 3, 4) and (1, 2, 5) respectively, show that PQ and
RS intersect. Also find the point of intersection.
Solution
Let the lines PQ and RS intersect at point A.
Let A divide PQ in the ratio  : 1, then P(4, 7, 8) S(1, 2, 5)

    4  2  7   8  l
A  , , . .... (1)
  1  1  1
A
1
Let A divide RS in the ratio k : 1, then K
R(2, 3, 4) Q(–1, –2, 1)
 k  2 2k  3 5k  4 
A   , ,  ..... (2)
 k 1 k 1 k 1 
From (1) and (2), we have,
  4 k  2
 ..... (3)
 1 k 1

RESONANCE 37
2  7 2k  3
 ..... (4)
 1 k 1

  8 5k  4
 ..... (5)
 1 k 1
From (3), –k –  + 4k + 4 = k + 2 + k + 2
or 2k + 3 – 3k – 2 = 0 ..... (6)
From (4), –2k – 2 + 7k + 7 = 2k + 3 + 2k + 3
or 4k + 5 – 5k – 4 = 0 ..... (7)
Multiplying equation (6) by 2, and subtracting from equation (7), we get
–+k=0 or , =k
Putting  = k in equation (6), we get
2 2 + 3 – 3 – 2 = 0
or,  = ± 1.
But  –1, as the co-ordinates of P would then be underfined and in this case
PQ || RS, which is not true.
  = 1 = k.
Clearly  k = 1 satisfies eqn. (5).
Hence our assumption is correct

  1 4  2  7 1 8  3 5 9
 A  , ,  or, A   , ,  .
 2 2 2  2 2 2
Self practice problems:
1. Find the ratio in which xy plane divides the line joining the points A (1, 2, 3) and B (2, 3, 6).
Ans. –1:2

2. Find the co-ordinates of the foot of perpendicular drawn from the point A(1, 2, 1) to the line joining the
point B(1, 4, 6) and C(5, 4, 4).
Ans. (3, 4, 5)

8 
3. Two vertices of a triangle are (4, –6, 3) and (2, –2, 1) and its centroid is  ,  1, 2  . Find the third vertex.
 3 
Ans. (2, 5, 2)

4. If centroid of the tetrahedron OABC, where co-ordinates of A, B, C are (a, 2, 3), (1, b, 2) and (2, 1, c)
respectively be (1, 2, 3), then find the distance of point (a, b, c) from the origin.
Ans. 107

 1 
5. Show that   , 2, 0  is the circumcentre of the triangle whose vertices are A (1, 1, 0), B (1, 2, 1) and
 2 
C (– 2, 2, –1) and hence find its orthocentre.
Ans. (1, 11, 0)

8. Direction Cosines And Direction Ratios


(i) Direction cosines: Let  be the angles which a directed
line makes with the positive directions of the axes of x, y and
z respectively, then cos , cos  cos  are called the direction
cosines of the line. The direction cosines are usually denoted
by (, m, n).
Thus  = cos , m = cos  , n = cos .
(ii) If , m, n be the direction cosines of a line, then 2 + m 2 + n2 = 1
(iii) Direction ratios: Let a, b, c be proportional to the direction cosines , m, n then a, b, c are
called the direction ratios.

RESONANCE 38
If a, b, c, are the direction ratios of any line L then a î  b ĵ  ck̂ will be a vector parallel to the line L.

If , m, n are direction cosines of line L then  î + m ĵ + n k̂ is a unit vector parallel to the line L.

(iv) If , m, n be the direction cosines and a, b, c be the direction ratios of a vector, then

 a b c 
  ,m  ,n  
 2 
 a2  b2  c 2 a2  b2  c 2 a b c 
2 2

a b c
or = ,m= ,n=
a2  b2  c 2 a2  b2  c 2 a2  b2  c 2

(v) If OP = r, when O is the origin and the direction cosines of OP are , m, n then the coordinates
of P are (r, mr, nr).
If direction cosines of the line AB are , m,n, |AB| = r, and the coordinates of A is (x1, y1, z1)
then the coordinates of B is given as (x1 + r , y1 + rm, z1 + rn)
(vi) If the coordinates P and Q are (x 1, y1, z1) and (x2, y2, z2) then the direction ratios of line PQ are,
x 2  x1
a = x2  x1, b = y2  y1 & c = z2  z1 and the direction cosines of line PQ are  = | PQ | ,

y 2  y1 z 2  z1
m= and n =
| PQ | | PQ |
(vii) Direction cosines of axes: Since the positive x axis makes angles 0º, 90º, 90º with axes of x,
y and z respectively. Therefore
Direction cosines of x axis are (1, 0, 0)
Direction cosines of yaxis are (0, 1, 0)
Direction cosines of zaxis are (0, 0, 1)
Example :
If a line makes angles , , with the co-ordinate axes, prove that sin2+ sin2+ sin2 = 2.
Solution
Since a line makes angles , , with the co-ordinate axes,
hence cos, cos, cosare its direction cosines
 cos2+ cos2+ cos2= 1
 (1 – sin2) + (1 – sin2) + (1 – sin2) = 1
 sin2+ sin2+ sin2= 2.
Example :
Find the direction cosines , m, n of a line which are connected by the relations  + m + n = 0,
2mn + 2m– n= 0
Solution
Given,  + m + n = 0 ..... (1)
2mn + 2m – n= 0 ..... (2)
From (1), n = – ( + m).
Putting n = – (+ m) in equation (2), we get,
– 2m(+ m) + 2m+ (+ m) = 0
or, – 2m– 2m 2 + 2m+ 2 + m= 0
or, 2 + m– 2m 2 = 0

 
2

or,     – 2 = 0 [dividing by m 2]
m m

  1 1 8  1 3
or   = 1, –2
m 2 2

RESONANCE 39
Case I. when

= 1 : In this case m = 
m
From (1), 2+ n = 0  n = – 2
 : m : n = 1 : 1 : – 2
 Direction ratios of the line are 1, 1, – 2
 Direction cosines are

1 1 2
± ,± ,±
1  1  ( 2 )
2 2 2
1  1  ( 2 )
2 2 2
1  12  ( 2)2
2

1 1 2 1 1 2
or, , , or – ,– ,
6 6 6 6 6 6

Case II. When



= – 2 : In this case  = – 2m
m
From (1), – 2m + m + n = 0  n=m
  : m : n = – 2m : m : m
=–2:1:1
 Direction ratios of the line are – 2, 1, 1.
 Direction cosines are

2 1 1 2 1 1
, , or, , , .
( 2 )  1  1
2 2 2
( 2 )  1  1
2 2 2
( 2 )  1  1
2 2 2 6 6 6

Self practice problems:

1. Find the direction cosine of a line lying in the xy plane and making angle 30° with x-axis.

1 3
Ans. m=± ,= ,n=0
2 2

2. A line makes an angle of 60° with each of x and y axes, find the angle which this line makes with
z-axis.
Ans. 45°

3. A plane intersects the co-ordinates axes at point A(a, 0, 0), B(0, b, 0), C(0, 0, c) O is origin. Find the
direction ratio of the line joining the vertex B to the centroid of face AOC.
a c
Ans. , – b,
3 3
4. A line makes angles , , ,  with the four diagonals of a cube, prove that
4
cos2 + cos 2+ cos2+ cos2= .
3

9. Angle Between Two Line Segments:


If two lines have direction ratios a1, b1, c1 and a2, b2, c2 respectively then we can consider two vectors
parallel to the lines as a1i + b1j + c 1k and a2i + b2j + c 2k and angle between them can be given as.
a 1a 2  b1b 2  c1c 2
cos = .
a12  b12  c12 a 22  b 22  c 22
(i) The line will be perpendicular if a1a2 + b1b2 + c1c 2 = 0

a1 b c1
(ii) The lines will be parallel if = 1 =
a2 b 2 c2
(iii) Two parallel lines have same direction cosines i.e. 1 = 2, m 1 = m 2, n1 = n2

RESONANCE 40
Example :

3 1 3 3 1 3
What is the angle between the lines whose direction cosines are  , , ; , , ?
4 4 2 4 4 2
Solution
Let  be the required angle, then
cos = 12 + m 1m 2 + n1n2

 3   3   1   1   3   3 

=  4   4    4   4    2  .  2 
          Y
3 1 3 1 G(0, a, 0) F(a, a, 0)
=      = 120°,
16 16 4 2
Example :
Find the angle between any two diagonals of a cube. E (a, a, a)
D
Solution (0, a, a)
The cube has four diagonals
OE, AD, CF and GB
The direction ratios of OE are O A X
(0, 0, 0) (a, 0, 0)
a, a, a or, 1, 1, 1
1 1 1 C(0, 0, a) B(a, 0, a)
 its direction cosines are , , .
3 3 3
Z’
Direction ratios of AD are – a, a, a. or, – 1, 1, 1.
1 1 1
 its direction cosines are , , .
3 3 3
Similarly, direction cosines of CF and GB respectively are
1 1 1 1 1 1
, , and , , .
3 3 3 3 3 3
We take any two diagonals, say OE and AD
Let  be the acute angle between them, then

 1   1  1   1   1   1  1
cos =  
 

 
. 
 

 
.
 
 
 3
 3 3  3  3  3  3

 1
or,  = cos–1   .
3

Example :
If two pairs of opposite edges of a tetrahedron are mutually perpendicular, show that the third pair will
also be mutually perpendicular.
Solution
Let OABC be the tetrahedron where O is the origin and co-ordinates of A, B, C be (x 1, y1, z1),
(x 2, y2, z2), (x3, y3, x3) respectively.

A (x1, y1, z1)

O (0, 0, 0)

B C
(x2, y2, z2) (x3, y3, z3)

RESONANCE 41
Let OA  BC and OB  CA .
We have to prove that
OC  BA .
Now, direction ratios of OA are x1 – 0, y1 – 0, z1 – 0 or, x1, y1, z1
direction ratios of BC are (x3 – x 2), (y3 – y2), (z3 – z2).
 OA  BC .
 x 1(x 3 – x 2) + y1(y3 – y2) + z1(z3 – z2) = 0 ..... (1)
Similarly,
 OB  CA
 x 2(x 1 – x 3) + y2(y1 – y3) + z2(z1 – z3) = 0 ..... (2)
Adding equations (1) and (2), we get
x 3(x 1 – x 2) + y3(y1 – y2) + z3(z1 – z2) = 0
 OC  BA [ direction ratios of OC are x3, y3, z3 and that of BA are (x 1 – x2), (y1 – y2), (z1 – z2)]

Self practice problems:


1. Find the angle between the lines whose direction cosines are given by  + m + n = 0 and
2 + m 2 – n2 = 0
Ans. 60°

2. P (6, 3, 2)
Q (5, 1, 4)
R (3, 3, 5)
are vertices of a  find Q.
Ans. 90°

3. Show that the direction cosines of a line which is perpendicular to the lines having directions cosines
1 m 1 n1 and 2 m 2 n2 respectively are proportional to
m 1n 2 – m 2n 1 , n 1 2 – n2 1,  1m 2 –  2m 1

1 0 . Projection of a line segment on a line


(i) If the coordinates P and Q are (x1, y1, z1) and (x2, y2, z2) then the projection of the line segments
PQ on a line having direction cosines , m, n is (x 2  x1 )  m(y 2  y1 )  n(z2  z1 )
 
  a.b
Vector form: projection of a vector a on another vector b is a . b̂ = 

(ii)
|b|

In the above case we can consider PQ as (x 2 – x1) î + (y2 – y1) ĵ + (z2 – z1) k̂ in place of a and


 î + m ĵ + n k̂ in place of b .
 | r |, m | r | & n | r | are the projection of r in OX, OY & OZ axes.

(iii)

r = | r | ( î + m ĵ + n k̂ )
 
(iv)

Solved Example :
Find the projection of the line joining (1, 2, 3) and (–1, 4, 2) on the line having direction ratios 2, 3, – 6.
Solution
Let A  (1, 2, 3), B  (–1, 4, 2)

B
A

90° 90°
P L M Q

RESONANCE 42
Direction ratios of the given line PQ are 2, 3, – 6

2 2  3 2  ( 6 ) 2 = 7  direction cosines of PQ are

2 3 6
, ,–
7 7 7
Projection of AB on PQ
=  (x 2 – x1) + m(y2 – y1) + n(z2 – z1)
2 3 6
= (–1 – 1) + (4 – 2) – (2 – 3)
7 7 7

4  6  6 8
= 
7 7
Self practice problems:

1. A (6, 3, 2), B (5, 1, 1,), C(3, –1, 3) D (0, 2, 5)


Find the projection of line segment AB on CD line.
Ans. 5/7
2. The projections of a directed line segment on co-ordinate axes are – 2, 3, – 6. Find its length and
direction cosines.
12 4 3
Ans. 13 ; , ,
13 13 13

3. Find the projection of the line segment joining (2, – 1, 3) and (4, 2, 5) on a line which makes equal
acute angles with co-ordinate axes.
7
Ans.
3

A PLANE
If line joining any two points on a surface lies completely on it then the surface is a plane.
OR
If line joining any two points on a surface is perpendicular to some fixed straight line. Then this surface
is called a plane. This fixed line is called the normal to the plane.

1 1 . Equation Of A Plane
(i) Normal form of the equation of a plane is x + my + nz = p, where, ,m n are the direction
cosines of the normal to the plane and p is the distance of the plane from the origin.

(ii) General form: ax + by + cz + d = 0 is the equation of a plane, where a, b, c are the


direction ratios of the normal to the plane.

(iii) The equation of a plane passing through the point (x 1, y1, z1) is given by
a (x  x 1) + b( y  y1) + c (z  z1) = 0 where a, b, c are the direction ratios of the normal
to the plane.

(iv) Plane through three points: The equation of the plane through three noncollinear points

x y z 1
x 1 y 1 z1 1
(x 1, y1, z1), (x 2, y2, z2), (x3, y3, z3) is =0
x 2 y2 z2 1
x 3 y3 z3 1

RESONANCE 43
x y z
(v)   1
Intercept Form: The equation of a plane cutting intercept a, b, c on the axes is
a b c
Vector form: The equation of a plane passing through a point having position vector a &

(vi)
normal to vector n is ( r  a ). n = 0 or r . n = a . n
       

Note: (a) Vector equation of a plane normal to unitvector n̂ and at a distance d from the origin is
. =d
r n
(b) Coordinate planes
(i) Equation of yzplane is x = 0
(ii) Equation of xzplane is y = 0
(iii) Equation of xyplane is z = 0
(c) Planes parallel to the axes:
If a = 0, the plane is parallel to x axis i.e. equation of the plane parallel to the xaxis is
by + cz + d = 0.
Similarly, equation of planes parallel to yaxis and parallel to zaxis are ax + cz +d = 0
and ax + by + d = 0 respectively.
(d) Plane through origin: Equation of plane passing through origin is ax + by + cz = 0.
(e) Transformation of the equation of a plane to the normal form: To reduce any equation
ax + by + cz  d = 0 to the normal form, first write the constant term on the right hand

side and make it positive, then divide each term by a 2  b2  c2 , where a, b, c are
coefficients of x, y and z respectively e.g.

ax by cz d
+ + =
 a 2  b2  c2  a 2  b2  c2  a 2  b2  c2  a 2  b2  c2
Where (+) sign is to be taken if d > 0 and () sign is to be taken if d < 0.

(f) Any plane parallel to the given plane ax + by + cz + d = 0 is ax + by + cz +  = 0.


Distance between two parallel planes ax + by + cz + d1 = 0 and ax + by + cz + d2 = 0 is
| d1  d2 |
given as
a2  b2  c 2

(g) Equation of a plane passing through a given point & parallel to the given vectors:
The equation of a plane passing through a point having position vector a and parallel to


b & c is r = a + b +  c (parametric form) where  &  are scalars.
    

r . ( b  c) = a . ( b  c) (non parametric form)


     
or

(h) A plane ax + by + cz + d = 0 divides the line segment joining (x 1, y1, z1) and (x 2, y2, z2). in the

 ax1  by1  cz1  d 


ratio  
 ax2  by2  cz2  d 

(i) The xyplane divides the line segment joining the points (x 1, y1, z1) and (x2, y2, z2) in the ratio

z1 x1 y1
 z . Similarly yzplane in  x and zxplane in  y
2 2 2

RESONANCE 44
(j) Coplanarity of four points
The points A(x1 y1 z1), B(x2 y2 z2) C(x 3 y3 z3) and D(x 4 y4 z4) are coplaner then

x 2  x 1 y 2  y 1 z 2  z1
x 3  x 1 y 3  y 1 z 3  z1
=0
x 4  x 1 y 4  y 1 z 4  z1
   
very similar in vector method the points A ( r1 ), B( r2 ), C( r3 ) and D( r4 ) are coplanar if
     
[ r4 – r1 , r4 – r2 , r4 – r3 ] = 0

Example :
Find the equation of the plane upon which the length of normal from origin is 10 and direction ratios of
this normal are 3, 2, 6.
Solution
If p be the length of perpendicular from origin to the plane and , m, n be the direction cosines of this
normal, then its equation is
x + my + nz = p ..... (1)
Here p = 10
Direction ratios of normal to the plane are 3, 2, 6

32  22  62 = 7
 Direction cosines of normal to the required plane are
3 2 6
= ,m= ,n=
7 7 7
Putting the values of , m, n, p in (1), equation of required plane is
3 2 6
x  y  z = 10
7 7 7
or, 3x + 2y + 6z = 70

Example :
Show that the points (0, – 1, 0), (2, 1, – 1), (1, 1, 1), (3, 3, 0) are coplanar.
Solution
Let A  (0, – 1, 0), B  (2, 1, – 1), C  (1, 1, 1) and D  (3, 3, 0)
Equation of a plane through A (0, – 1, 0) is
a (x – 0) + b (y + 1) + c (z – 0) = 0
or, ax + by + cz + b = 0 ..... (1)
If plane (1) passes through B (2, 1, – 1) and C (1, 1, 1)
Then 2a + 2b – c = 0 ..... (2)
and a + 2b + c = 0 ..... (3)
From (2) and (3), we have
a b c
 
2  2  1 2 4  2

a b c
or,   = k (say)
4 3 2
Putting the value of a, b, c, in (1), equation of required plane is
4kx – 3k(y + 1) + 2kz = 0
or, 4x – 3y + 2z – 3 = 0 ..... (2)
Clearly point D (3, 3, 0) lies on plane (2)
Thus points D lies on the plane passing through A, B, C and hence points A, B, C and D are coplanar.

RESONANCE 45
Example :
If P be any point on the plane x + my + nz = p and Q be a point on the line OP such that
OP . OQ = p2, show that the locus of the point Q is p(x + my + nz) = x2 + y2 + z2.
Solution
Let P  (, , ), Q  (x 1, y1, z1)
Direction ratios of OP are , ,  and direction ratios of OQ are x 1, y1, z1.
Since O, Q, P are collinear, we have
  
 
x1 y1 z1 = k (say) ..... (1)

As P (, , ) lies on the plane x + my + nz = p,


 + m + n = p or k(x 1 + my1 + nz1) = p ..... (2)
Given, OP . OQ = p 2

  2  2   2 x12  y12  z12 = p2

or, k 2 ( x12  y12  z12 ) x12  y12  z12 = p2

or, k ( x12  y12  z12 ) = p2 ..... (3)


On dividing (2) by (3), we get,
x 1  my 1  nz 1 1

x 12  y 12  z12 p

or, p (x 1 + my1 + nz1) = x12  y12  z12


Hence the locus of point Q is
p (x + my + nz) = x2 + y2 + z2.

Example :
x y z
A point P moves on a plane   = 1. A plane through P and perpendicular to OP meets the
a b c
co-ordinate axes in A, B and C. If the planes through A, B and C parallel to the planes x = 0, y = 0,
z = 0 intersect in Q, find the locus of Q.
Solution
Given plane is
x y z
  1 ..... (1)
a b c
Let P  (h, k, )
h k 
Then   =1 ..... (2)
a b c

OP = h2  k 2   2

h k
, ,

Direction cosines of OP are
h2  k 2   2 h2  k 2   2 h2  k 2   2
 Equation of the plane through P and normal to OP is

h k
x y  h2  k 2   2

h k 
2 2 2
h k 
2 2 2
h k 
2 2 2

or, hx + ky + z = (h + k +  ) 2 2 2

RESONANCE 46
 h2  k 2   2   h2  k 2   2 
 A   , 0, 0  , B   0, , 0 ,

h  k
   

 h2  k 2   2 

C  0, 0,

  
Let Q  (, , ), then

h2  k 2   2 h2  k 2   2 h2  k 2   2
= ,= ,= ..... (3)
h k 

1 1 1 h2  k 2   2 1
Now     ..... (4)
2 2 2 (h 2  k 2   2 )2 (h2  k 2   2 )

h2  k 2   2
From (3), h =

h h2  k 2   2
 
a a

k h2  k 2   2  h2  k 2   2
Similarly  and 
b b c c

h2  k 2   2 h2  k 2   2 h2  k 2   2 h k 
      = 1 [from (2)]
a b c a b c

1 1 1 1 1 1 1
or,    2  2  2  2 [from (4)]
a b c h  k  
2 2
  
 Required locus of Q (, , ) is
1 1 1 1 1 1
   2  2  2 .
ax by cz x y z

Self practice problems :

1. Check wether this point are coplanar if yes find the equation of plane containing them
A  (1, 1, 1)
B (0, – 1, 0)
C (2, 1, –1)
D (3, 3, 0)
Ans. yes, 4x – 3y + 2z = 3

2. Find the plane passing through point (– 3, – 3, 1) and perpendicular to the line joining the points
(2, 6, 1) and (1, 3, 0).
Ans. x + 3y + z + 11 = 0

3. Find the equation of plane parallel to x + 5y – 4z + 5 = 0 and cutting intercepts on the axes whose rum
is 150.
3000
Ans. x + 5y – 4z =
19

4. Find the equation of plane passing through (2, 2, 1) and (9, 3, 6) and perpendicular to the plane
x + 3y + 3z = 8.
Ans. 3x + 4y – 5z = 9

RESONANCE 47
5. Find the equation of the plane | | to î  ĵ  k̂ and î  ĵ and passing through (1, 1, 2).
Ans. x + y – 2z + 2 = 0

6. Find the equation of the plane passing through the point (1, 1, – 1) and perpendicular to the planes
x + 2y + 3z – 7 = 0 and 2x – 3y + 4z = 0.
Ans. 17x + 2y – 7z = 26

1 2 . Sides of a plane:
A plane divides the three dimensional space in two equal parts. Two points A (x 1 y 1 z 1 )
and B (x 2 y2 z2) are on the same side of the plane ax + by + cz + d = 0 if ax1 + by1 + cz1 + d and
ax2 + by2 + cz2 + d are both positive or both negative and are opposite side of plane if both of these
values are in opposite sign.

Example :
Show that the points (1, 2, 3) and (2, – 1, 4) lie on opposite sides of the plane x + 4y + z – 3 = 0.
Solution
Since the numbers 1+ 4 × 2 + 3 – 3 = 9 and 2 – 4 + 4 – 3 = – 1 are of opposite sign., the points are on
opposite sides of the plane.

1 3 . A Plane & A Point


ax' by' cz'd
(i) Distance of the point (x , y, z) from the plane ax + by + cz+ d = 0 is given by .
a 2  b2  c2
The length of the perpendicular from a point having position vector a to plane r . n = d is
  
(ii)

| a.n  d |
 
given by p = .
|n|

(iii) The coordinates of the foot of perpendicular from the point (x, y, z) to the plane
x '  x 1 y '  y 1 z'  z1 (ax1  by1  cz1  d)
ax + by + cz + d = 0 aregain by   =–
a b c a 2  b2  c 2
(iv) To find image of a point w.r.t. a plane.
Let P (x 1, y1, z1) is a given point and ax + by + cz + d = 0 is given plane Let (x, y, z) is the
image point. then
(a) x – x1 = a, y – y1 = b, z – z1 = c
 x = a + x1, y = b + y1, z = c + z1

 x  x 1   y  y 1   z  z1 
(b) a  + b  + c  =0
 2   2   2 
from (i) put the values of x, y, z in (ii) and get the values of  and resubtitute in (i) to get
(x y z).
The coordinate of the image of point (x 1 , y1 , z1) w.r.t the plane ax + by + cz + d = 0 are given
x '  x 1 y '  y 1 z'  z1 (ax1  by1  cz1  d)
by   =–2
a b c a 2  b2  c 2

(v) The distance between two parallel planes ax + by + cx + d = 0 and ax + by + cx + d’ = 0 is


| d  d' |
given by
a  b2  c 2
2

RESONANCE 48
Example :
Find the image of the point P (3, 5, 7) in the plane 2x + y + z = 16.
Solution
Given plane is 2x + y + z = 16 ..... (1)
P  (3, 5, 7)
Direction ratios of normal to plane (1) are 2, 1, 1
Let Q be the image of point P in plane (1). Let PQ meet plane (1) in R
then PQ  plane (1)
Let R  (2r + 3, r + 5, r + 7)
Since R lies on plane (1)
 2(2r + 3) + r + 5 + r + 7 = 0
or, 6r + 18 = 0  r=–3
 R  (– 3, 2, 4)
Let Q  (, , )
Since R is the middle point of PQ
3
 –3=  = – 9
2
5
2=  = – 1
2
7
4=  = 1
2
 Q = (– 9, – 1, 1).
Example :
Find the distance between the planes
2x – y + 2z = 4 and 6x – 3y + 6z = 2.
Solution
Given planes are
2x – y + 2z – 4 = 0 ..... (1)
and 6x – 3y + 6z – 2 = 0 ..... (2)
a1 b1 c 1
We find that a  b  c
2 2 2

Hence planes (1) and (2) are parallel.


2
Plane (2) may be written as 2x – y + 2z – =0 ..... (3)
3
 Required distance between the planes

2
4
3 10 10
=  
2  ( 1)  2
2 2 2 3.3 9

Example :
A plane passes through a fixed point (a, b, c). Show that the locus of the foot of perpendicular to it from
the origin is the sphere x2 + y2 + z2 – ax – by – cz = 0
Solution
Let the equation of the variable plane be

O(0, 0, 0)

P(, , )

x + my + nz + d = 0 ..... (1)

RESONANCE 49
Plane passes through the fixed point (a, b, c)
 a + mb + nc + d = 0 ..... (2)
Let P (, , ) be the foot of perpendicular from origin to plane (1).
Direction ratios of OP are
– 0, – 0, – 0 i.e. , , 
From equation (1), it is clear that the direction ratios of normal to the plane i.e. OP are , m, n ;
, , and , m, n are the direction ratios of the same line OP
   1
 = = = (say)
 m n k
  = k, m = k, n = k ..... (3)
Putting the values of
, m, n in equation (2), we get
ka + kb + kc + d = 0 ..... (4)
Since , , lies in plane (1)
  + m + n + d = 0 ..... (5)
Putting the values of , m, n from (3) in (5), we get
k2 + k 2 + k2 + d = 0 ..... (6)
or k2 + k 2 + k2 – ka – kb – kc = 0
[putting the value of d from (4) in (6)]
or  +  +  – a – b – c = 0
2 2 2

Therefore, locus of foot of perpendicular P (, , ) is


x 2 + y2 + z2 – ax – by – cz = 0 ..... (7)

Self practice problems:

1. Find the intercepts of the plane 3x + 4y – 7z = 84 on the axes. Also find the length of perpendicular
from origin to this line and direction cosines of this normal.
1 3 4 7
Ans. a = 28, b = 21, c = – 12, p = ; , ,
74 74 74 74
2. Find : (i) perpendicular distance
(ii) foot of perpendicular
(iii) image of (1, 0, 2) in the plane 2x + y + z = 5

1  4 1 13  5 1 7
Ans. (i) (ii)  , ,  (iii)  , , 
6 3 6 6  3 3 3

1 4 . Angle Between Two Planes:


(i) Consider two planes ax + by + cz + d = 0 and ax + by + c z + d = 0. Angle between these
planes is the angle between their normals. Since direction ratios of their normals are (a, b, c)
and (a, b, c ) respectively, hence , the angle between them, is given by

aa' bb' cc'


cos =
a 2  b2  c2 a ' 2  b ' 2  c' 2

a b c
Planes are perpendicular if aa + bb + cc  = 0 and planes are parallel if = =
a' b' c'

n1 . n2
 
The angle  between the planes r . n = d1 and r . n 2 = d2 is given by, cos  = 
   
(ii)
| n1 | . | n 2 |

Planes are perpendicular if n1 . n 2 = 0 & planes are parallel if n1 =  n 2 .


   

RESONANCE 50
1 5 . Angle Bisectors
(i) The equations of the planes bisecting the angle between two given planes
a1x + b1y + c 1z + d1 = 0 and a2x + b2y + c2z + d2 = 0 are

a1x  b1y  c1z  d1 a 2 x  b2 y  c2 z  d 2



a12  b12  c12 a 22  b22  c22

(ii) Equation of bisector of the angle containing origin: First make both the constant terms positive.

a1x  b1y  c1z  d1 a x  b2 y  c2 z  d 2 gives the bisector of


Then the positive sign in =± 2
a12  b12  c12 a 22  b22  c22
the angle which contains the origin.

(iii) Bisector of acute/obtuse angle: First make both the constant terms positive. Then
a 1a 2 + b 1b 2 + c 1c 2 > 0  origin lies on obtuse angle
a 1a 2 + b 1b 2 + c 1c 2 < 0  origin lies in acute angle

1 6 . Family of Planes
(i) Any plane passing through the line of intersection of nonparallel planes or equation of
the plane through the given line in serval form.
a 1x + b1y + c 1z + d 1 = 0 & a2x + b2y + c 2z + d2 = 0 is
a 1x + b 1y + c 1z + d 1 +  (a2x + b2y + c2z + d2) = 0

The equation of plane passing through the intersection of the planes r . n1 = d 1 &
 
(ii)

r . n 2 = d2 is r . (n1 +  n 2 ) = d1  d2 where  is arbitrary scalar


   

Example :
The plane x – y – z = 4 is rotated through 90° about its line of intersection with the plane
x + y + 2z = 4. Find its equation in the new position.
Solution
Given planes are
x–y–z=4 ..... (1)
and x + y + 2z = 4 ..... (2)
Since the required plane passes through the line of intersection of planes (1) and (2)
 its equation may be taken as
x + y + 2z – 4 + k (x – y – z – 4) = 0
or (1 + k)x + (1 – k)y + (2 – k)z – 4 – 4k = 0 ..... (3)
Since planes (1) and (3) are mutually perpendicular,
 (1 + k) – (1 – k) – (2 – k) = 0

2
or, 1+k–1+k–2+k=0 or, k=
3

2
Putting k = in equation (3), we get,
3
5x + y + 4z = 20
This is the equation of the required plane.

RESONANCE 51
Example :
Find the equation of the plane through the point (1, 1, 1) which passes through the line of intersection
of the planes x + y + z = 6 and 2x + 3y + 4z + 5 = 0.
Solution
Given planes are
x+y+z–6=0 ..... (1)
and 2x + 3y + 4z + 5 = 0 ..... (2)
Given point is P (1, 1, 1).
Equation of any plane through the line of intersection of planes (1) and (2) is
x + y + z – 6 + k (2x + 3y + 4z + 5) = 0 ..... (3)
If plane (3) passes through point P, then
1 + 1 + 1 – 6 + k (2 + 3 + 4 + 5) = 0
3
or, k=
14
From (1) required plane is
20x + 23y + 26z – 69 = 0

Example :
Find the planes bisecting the angles between planes
2x + y + 2z = 9 and 3x – 4y + 12z + 13 = 0.
Which of these bisector planes bisects the acute angle between the given planes. Does origin lie in the
acute angle or obtuse angle between the given planes ?
Solution
Given planes are
– 2x – y – 2z + 9 = 0 ..... (1)
and 3x – 4y + 12z + 13 = 0 ..... (2)
Equations of bisecting planes are

2 x  y  2 z  9 3 x  4 y  12z  13

( 2)  ( 1)  ( 2)
2 2 2
3 2  ( 4)2  (12)2
or, 13 [– 2x – y – 2z + 9] = ± 3 (3x – 4y + 12z + 13)
or, 35x + y + 62z = 78, ..... (3) [Taking +ve sign]
and 17x + 25y – 10z = 156 ..... (4) [Taking – ve sign]
Now a1a2 + b1b2 + c 1c 2 = (– 2) (3) + (– 1) (– 4) + (– 2) (12)
= – 6 + 4 – 24 = – 26 < 0
 Bisector of acute angle is given by 35x + y + 62z = 78
 a1a2 + b1b2 + c 1c 2 < 0, origin lies in the acute angle between the planes.

Example :
If the planes x – cy – bz = 0, cx – y + az = 0 and bx + ay – z = 0 pass through a straight line, then find
the value of a2 + b2 + c 2 + 2abc.
Solution
Given planes are
x – cy – bz = 0 ..... (1)
cx – y + az = 0 ..... (2)
bx + ay – z = 0 ..... (3)
Equation of any plane passing through the line of intersection of planes (1) and (2) may be taken as
x – cy – bz +  (cx – y + az) = 0
or, x (1 + c) – y (c + ) + z (– b + a) = 0 ..... (4)
If planes (3) and (4) are the same, then equations (3) and (4) will be identical.
1  c (c   ) b  a
  
b a 1
(i) (ii) (iii)

RESONANCE 52
From (i) and (ii), a + ac = – bc – b
(a  bc )
or, =– ..... (5)
(ac  b)
From (ii) and (iii),
(ab  c )
c +  = – ab + a2 or = ..... (6)
1  a2
From (5) and (6), we have,
(a  bc ) (ab  c )
 .
ac  b (1  a 2 )
or, a – a3 + bc – a2bc = a2bc + ac 2 + ab2 + bc
or, a2bc + ac 2 + ab2 + a3 + a2bc – a = 0
or, a2 + b2 + c2 + 2abc = 1.

Self practice problems:


1. A tetrahedron has vertices at O(0, 0, 0), A(1, 2, 1), B(2, 1, 3) and C(–1, 1, 2). Prove that the angle

 19 
between the faces OAB and ABC will be cos –1  .
 35 

2. Find the equation of plane passing through the line of intersection of the planes 4x – 5y – 4z = 1 and
2x + y + 2z = 8 and the point (2, 1, 3).
10
Ans. 32x – 5y + 8z – 83 = 0,  =
3

3. Find the equations of the planes bisecting the angles between the planes
x + 2y + 2z – 3 = 0, 3x + 4y + 12z + 1 = 0 and sepecify the plane which bisects the acute angle
between them.
Ans. 2x + 7y – 5z = 21, 11x + 19y + 31z = 18; 2x + 7y – 5z = 21

4. Show that the origin lies in the acute angle between the planes
x + 2y + 2z – 9 = 0 and 4x – 3y + 12z + 13 = 0

5. Prove that the planes 12x – 15y + 16z – 28 = 0, 6x + 6y – 7z – 8 = 0 and 2x + 35y – 39z + 12 = 0 have
a common line of intersection.

17. Area of a triangle:

Let A (x1, y1, z1), B (x2, y2, z2), C (x3, y3, z3) be the vertices of a triangle, then  = (2x  2y  2z )

y1 z1 1 z1 x1 1 x1 y1 1
1 1
where x = y2 z2 1 , y = z x 2 1 and z = x 2 y2 1
2 2 2
y3 z3 1 z3 x3 1 x3 y3 1

Vector Method  From two vector AB and AC . Then area is given by


 

i j k
1  1
x 2  x1 y 2  y1 z2  z1

| AB x AC | =
2 2
x 3  x1 y 3  y1 z3  z1

RESONANCE 53
Example :
Through a point P (h, k, ) a plane is drawn at right angles to OP to meet the co-ordinate axes in A, B

p5
and C. If OP = p, show that the area of ABC is .
2hk 
Solution
OP = h2  k 2   2 = p
Direction cosines of OP are
h k
, ,

h2  k 2   2 h2  k 2   2 h2  k 2   2
Since OP is normal to the plane, therefore, equation of the plane will be,
h k
x y z  h2  k 2   2

h k 
2 2 2
h k 
2 2 2
h k 
2 2 2

or, hx + ky + z = h + k +  = p 2 2 2 2
..... (1)

 p2   p2   p 2 

    
A   h , 0, 0  , B   0, k , 0  , C   0, 0,  
     
Now area of ABC,  = A2xy + A2yz + A2zx
Now Axy = area of projection of ABC on xy-plane = area of AOB

p2
0 1
h
1 p2 1 p4
= Mod of 0 1 =
2 k 2 | hk |
0 0 1

1 p4 1 p4
Similarly, Ayz = and Azx =
2 | k | 2 | h |

1 p8 1 p8 1 p8
 2 =  
4 h 2k 2 4 k 2  2 4  2h 2

p8 p10
= (2 + k 2 + h2) =
4h 2k 2  2 4h 2k 2  2

p5
or, = .
2hk 

1 8 . Volume Of A Tetrahedron:
Volume of a tetrahedron with vertices A (x 1, y1, z1), B( x 2, y2, z2), C (x 3, y3, z3) and

x1 y1 z1 1
x y2 z2 1
1 2
D (x4, y4, z4) is given by V =
6 x3 y3 z3 1
x4 y4 z4 1

RESONANCE 54
A LINE
1 9 . Equation Of A Line
(i) A straight line in space is characterised by the intersection of two planes which are not parallel
and therefore, the equation of a straight line is a solution of the system constituted by the
equations of the two planes, a1x + b1y + c 1z + d1 = 0 and a2x + b2y + +c2z + d2 =0.
This form is also known as nonsymmetrical form.

(ii) The equation of a line passing through the point (x 1, y1, z1) and having direction ratios a, b, c

x  x 1 y  y 1 z  z1
is = = = r. This form is called symmetric form. A general point on the line
a b c
is given by (x  + ar, y + br, z + cr).

(iii) Vector equation: Vector equation of a straight line passing through a fixed point with position
vector a and parallel to a given vector b is r = a +  b where  is a scalar..
    

(iv) The equation of the line passing through the points (x 1, y1, z1) and (x2, y2, z2) is

x  x1 y  y1 z  z1
= =
x2  x1 y2  y1 z 2  z1
 
(v) Vector equation of a straight line passing through two points with position vectors a &b
 
 ( b  a ).
 
is r = a +

(vi) Reduction of cartesion form of equation of a line to vector form & vice versa
x  x1 y  y1 z  z1
= =   = (x + y + z ) +
r 1 î 1 ĵ 1 k̂
 (a î + b ĵ + c k̂ ).
a b c

Note: Straight lines parallel to co-ordinate axes:

Straight lines Equation Straight lines Equation


(i) Through origin y = mx, z = nx (v) Parallel to xaxis y = p, z = q
(ii) x axis y = 0, z = 0 (vi) Parallel to yaxis x = h, z = q
(iii) y axis x = 0, z = 0 (vii) Parallel to zaxis x = h, y = p
(iv) z axis x = 0, y = 0

Example :
Find the equation of the line through the points (3, 4, –7) and (1, – 1, 6) in vector form as well as in
cartesian form.
Solution
Let A  (3, 4, – 7), B  (1, – 1, 6)
   

Now a = OA = 3 i + 4 j – 7 k ,
    
= b = OB = i – j + 6 k
 
Equation of the line through A( a ) and B( b )
   
is r = a +t(b – a )

RESONANCE 55
      
or r = 3 i + 4 j – 7 k + t (–2 i – 5 j + 13 k ) ..... (1)
Equation in cartesian form :
x3 y4 z7
Equation of AB is  
3 1 4 1  7  6

x3 y4 z7


or,  
2 5  13
Example :
x 1 y  2 z  3
Find the co-ordinates of those points on the line   which is at a distance of 3 units
2 3 6
from point (1, –2, 3).
Solution
x 1 y  2 z  3
Given line is   ..... (1)
2 3 6
Let P  (1, –2, 3)
Direction ratios of line (1) are 2, 3, 6
2 3 6
 Direction cosines of line (1) are , ,
7 7 7
Equation of line (1) may be written as

x 1 y  2 z  3
  ..... (2)
2 3 6
7 7 7
Co-ordinates of any point on line (2) may be taken as

2 3 6 
 r  1, r  2, r  3 
 7 7 7 

2 3 6 
Let Q   r  1, r  2, r  3 
 7 7 7 
Distance of Q from P = | r |
According to question | r | = 3  r=±3
Putting the value of r, we have

 1 5 39   13 23 3 
Q   ,  ,  or Q   , , 
 7 7 7   7 7 7

Example :
Find the equation of the line drawn through point (1, 0, 2) to meet at right angles the line
x 1 y  2 z 1
 
3 2 1
Solution
Given line is
x 1 y  2 z 1
  ..... (1)
3 2 1
Let P  (1, 0, 2)
Co-ordinates of any point on line (1) may be taken as
Q  (3r – 1, – 2r + 2, – r – 1)
Direction ratios of PQ are 3r – 2, – 2r + 2, – r – 3
Direction ratios of line AB are 3, – 2, – 1

RESONANCE 56
Since PQ  AB
 3 (3r – 2) – 2 (– 2r + 2) – 1 (– r – 3) = 0
 9r – 6 + 4r – 4 + r + 3 = 0
 14r = 7
1
 r=
2
Therefore, direction ratios of PQ are
1 7
– , 1, – or, – 1, 2, – 7
2 2
Equation of line PQ is
x 1 y  0 z  2 x 1 y z2
  or,  
1 2 7 1 2 7

Example :
x 1 y  2 z  3 x  4 y 1
Show that the two lines   and  = z intersect. Find also the point of
2 3 4 5 2
intersection of these lines.
Solution
x 1 y  2 z  3
Given lines are   ..... (1)
2 3 4

x  4 y 1 z  0
and   ..... (2)
5 2 1
Any point on line (1) is P (2r + 1, 3r + 2, 4r +3)
and any point on line (2) is Q (5 + 4, 2 + 1, )
Lines (1) and (2) will intersect if P and Q coincide for some value of  and r.
 2r + 1 = 5 + 4  2r – 5 = 3 ..... (1)
3r + 2 + 2 + 1  3r – 2 = – 1 ..... (2)
4r + 3 =   4r –  = – 3 ..... (3)
Solving (1) and (2), we get r = – 1,  = – 1
Clearly these values of r and  satisfy eqn. (3)
Now P  (– 1, – 1, – 1)
Hence lines (1) and (2) intersect at (– 1, – 1, – 1).

Self practice problems:

1. Find the equation of the line passing through point (1, 0, 2) having direction ratio 3, – 1, 5. Prove that
this line passes through (4, – 1, 7).
x 1 y z2
Ans.  
3 1 5

x  2 y 1 z 7
2. Find the equation of the line parallel to line   and passing through the point (3, 0, 5).
3 1 9

x3 y z5
Ans.  
3 1 3

3. Find the coordinates of the point when the line through (3, 4, 1) and (5, 1, 6) crosses the xy plane.

 13 23 
Ans.  , , 0
 5 5 

RESONANCE 57
2 0 . Reduction Of Non-Symmetrical Form To Symmetrical Form:
Let equation of the line in nonsymmetrical form be a1x + b1y + c 1z + d1 = 0, a2x + b2y + c 2z + d2 = 0.
To find the equation of the line in symmetrical form, we must know (i) its direction ratios (ii) coordinate
of any point on it.

(i) Direction ratios: Let , m, n be the direction ratios of the line. Since the line lies in both the
planes, it must be perpendicular to normals of both planes. So a 1 + b 1m + c 1n = 0,
a2 + b2m + c 2n = 0. From these equations, proportional values of , m, n can be found by

m n
cross multiplication as

= =
b1c2  b2 c1 c1a 2  c2a1 a1b2  a 2 b1

Alternative method

i j k
The vector a1 b1 c 1 = i (b1c 2 – b2c 1) + j (c1a2 – c2a1) + k (a1b2 – a2b1) will be parallel to the line of
a2 b2 c 2

intersection of the two given planes. hence : m: n = (b1c 2 – b2c 1): (c1a2 – c 2a1): (a1b2 – a2b1)
(ii) Point on the line  Note that as , m, n cannot be zero simultaneously, so at least one must
be nonzero. Let a1b2  a2b1  0, then the line cannot be parallel to xy plane, so it intersect it.
Let it intersect xyplane in (x1, y1, 0). Then a1x 1 + b1y1 + d1 = 0 and a2x 1 + b2y1 + d2 = 0.Solving
these,we get a point on the line. Then its equation becomes.

b1d 2  b2 d1 d a  d 2a1
x y 1 2
x  x1 y  y1 z0 a1b2  a 2 b1 a1b2  a 2 b1 z0
= = or = =
b1c2  b2 c1 c1a 2  c2a1 a1b2  a 2 b1 b1c2  b2 c1 c1a 2  c2a1 a1b2  a 2 b1

Note: If   0, take a point on yzplane as (0, y1, z1) and if m  0, take a point on xzplane as (x1, 0, z1).
Alternative method
a1 b1
If a  b Put z = 0 in both the equations and solve the equations a1x + b1y + d1 = 0, a2x + b2y + d2 =0
2 2

otherwise Put y = 0 and solve the equations a1x + c 1z + d1 = 0 and a2x + c 2z + d2 = 0

Example :
Find the equation of the line of intersection of planes 4x + 4y – 5z = 12, 8x + 12y – 13z = 32 in the
symmetric form.
Solution
Given planes are 4x + 4y – 5z – 12 = 0 ..... (1)
and 8x + 12y – 13z – 32 = 0 ..... (2)
Let , m, n be the direction ratios of the line of intersection :
then 4+ 4m – 5n = 0 ..... (3)
and 8+ 12m – 13n = 0
m n
  

 52  60  40  52 48  32

 m n  m n
or,   or,  
8 12 16 2 3 4
Hence direction ratios of line of intersection are 2, 3, 4.
Here 4  0, therefore line of intersection is not parallel to xy-plane.

RESONANCE 58
Let the line of intersection meet the xy-plane at P (, , 0).
Then P lies on planes (1) and (2)
 4+ 4+ 12 = 0
or,  + – 3 = 0 ..... (5)
and 8+ 12– 32 = 0
or, 2+ 3– 8 = 0 ..... (6)
Solving (5) and (6), we get
  1
 
89 68 32

  1
or,  
1 2 1
  = 1,  = 2
x 1 y  2 z  0
Hence equation of line of intersection in symmetrical form is   .
2 3 4
Example :
Find the angle between the lines x – 3y – 4 = 0, 4y – z + 5 = 0 and x + 3y – 11 = 0, 2y – z + 6 = 0.
Solution
Given lines are
x  3 y  4  0
 ..... (1)
4y  z  5  0

x  3 y  11  0
and  ..... (2)
2y  z  6  0 
Let 1, m 1, n1 and 1, m 2, n2 be the direction cosines of lines (1) and (2) respectively
 line (1) is perpendicular to the normals of each of the planes
x – 3y – 4 = 0 and 4y – z + 5 = 0
 1 – 3m 1 + 0.n1 = 0 ..... (3)
and 01 + 4m 1 – n1 = 0 ..... (4)
Solving equations (3) and (4), we get,
m1 n
 1
1
=
30 0  ( 1) 4  0

 1 m1 n1
or,   = k (let).
3 1 4
Since line (2) is perpendicular to the normals of each of the planes
x + 3y – 11 = 0 and 2y – z + 6 = 0,
 2 + 3m 2 = 0 ..... (5)
and 2m 2 – n2 = 0 ..... (6)


2
2 = – 3m 2 or, = m2
3

n2
and n2 = 2m 2 or, = m 2.
2

m n
  3  2 = t (let).
2
3 1 2
If  be the angle between lines (1) and (2), then
cos = 12 + m 1m 2 + n1n2
= (3k) (– 3t) + (k) (t) + (4k) (2t)
= – 9kt + kt + 8kt = 0
  = 90°.

RESONANCE 59
Self practice problems:

1. Find the equation of the line of intersection of the plane


4x + 4y – 5z = 12
8x + 12y – 13z = 32
x 1 y2 z0
Ans. = =
2 3 4

2. Show that the angle between the two lines defined by the equations x = y and xy + yz + zx = 0 is

 1
cos–1  
3

3. Prove that the three planes 2x + y – 4z – 17 = 0, 3x + 2y – 2z – 25 = 0, 2x – 4y + 3z + 25 = 0 intersect


at a point and find its co-ordinates.
Ans. (3, 7, – 1)

2 1 . Foot, Length And Equation Of Perpendicular From A Point To A Line:

x a y  b z c
(i) Cartesian form: Let equation of the line be = = = r (say) ..........(i)
 m n
and A ( ) be the point.
Any point on line (i) is P (r + a, mr + b, nr + c) ......... (ii)

If it is the foot of the perpendicular from A on the line, then AP is perpendicular to the line. So
 (r + a  ) + m (mr + b  ) + n (nr + c ) = 0 i.e. r = ( a)  + ( b) m + ( c)n since
2 + m 2 + n2 = 1. Putting this value of r in (ii), we get the foot of perpendicular from point A on
the given line. Since foot of perpendicular P is known, then the length of perpendicular is given

by AP = (r  a   )  ( mr  b  )  ( nr  c   ) the equation of perpendicular is given


2 2 2

x  y z
by = =
r  a   mr  b   nr  c  
(ii) Vector Form: Equation of a line passing through a point having position vector  and

perpendicular to the lines r = a1 +  b1 and r = a 2 +  b 2 is parallel to b1 x b 2 . So the vector


       

equation of such a line is r =  +  ( b1 x b 2 ). Position vector  of the image of a point  in


     

  ) . b   
  
2 (a
a straight line r = a +  b is given by  = 2 a    b   . Position vector of the
      2
 | b | 

   (a ) . b  
  
foot of the perpendicular on line is f = a    b . The equation of the perpendicular
2
 |b| 

    (a     
) . b 
is r =  +   ( a   )  
  .
b

 |b|
 
   
2

RESONANCE 60
22. To find image of a point w. r. t a line
x  x2 y  y2 z  z2
Let L  = = is a given line
a b c
Let (x, y, z) is the image of the point P (x 1, y1, z1) with respect to the line L. Then

(i) a (x 1 – x) + b (y1 – y) + c (z1 – z) = 0

x1  x y1  y z1  z
 x2  y2  z2
(ii) 2 = 2 = 2 =
a b c
from (ii) get the value of x, y, z in terms of  as
x = 2a + 2x2 – x 1, y = 2b – 2y2 – y1,
z = 2c + 2z2 – z1
now put the values of x, y, zin (i) get  and resubtitute the value of  to to get (x y z).
Example :

x 1 y 3 z  2
Find the length of the perpendicular from P (2, – 3, 1) to the line   .
2 3 1
Solution
x 1 y 3 z  2
Given line is   ..... (1)
2 3 1
P  (2, – 3, 1)
Co-ordinates of any point on line (1) may be taken as
Q  (2r – 1, 3r + 3, – r – 2)
Direction ratios of PQ are 2r – 3, 3r + 6, – r – 3
Direction ratios of AB are 2, 3, – 1
Since PQ  AB
 2 (2r – 3) + 3 (3r + 6) – 1 (– r – 3) = 0
15
or, 14r + 15 = 0  r=
14

  22  3  13 
 Q  , , 
 7 14 14 

531
 PQ = units.
14
Second method : Given line is
x 1 y 3 z  2
 
2 3 1
P  (2, – 3, 1) P (2, –3, 1)
2 3 1
Direction ratios of line (1) are , ,–
14 14 14
RQ = length of projection of RP on AB A R Q B
(–1, 3, –2)
2 3 1 15
= (2  1)  ( 3  3)  (1  2) 
14 14 4 14
PR = 3 + 6 + 3 = 54
2 2 2 2

225 531
 PQ = PR 2  RQ 2 = 54  
14 14

RESONANCE 61
Self practice problems:
x  11 y  2 z  8
1. Find the length and foot of perpendicular drawn from point (2, –1, 5) to the line   .
10 4  11
Also find the image of the point in the line.
Ans. 14 , N  (1, 2, 3),  (0, 5, 1)

x y 1 z  2
2. Find the image of the point (1, 6, 3) in the line   .
1 2 3
Ans. (1, 0 , 7)
x  15 y  29 z  5
3. Find the foot and hence the length of perpendicular from (5, 7, 3) to the line   .
3 8 5
Find also the equation of the perpendicular.
x5 y7 z3
Ans. (9, 13, 15) ; 14 ;  
2 3 6

2 3 . Angle Between A Plane And A Line:


x  x1 y  y1 z  z1
(i) If  is the angle between line = = and the plane ax + by + cz + d = 0, then
 m n

 
a   bm  cn
sin  = .

 (a 2  b2  c2 )  2  m 2  n 2 

 b.n 
 
Vector form: If  is the angle between a line r = ( a +  b ) and r . n = d then sin =     .

(ii)

| b | | n |

 m n  
(iii) Condition for perpendicularity = = b xn = 0
a b c
 
(iv) Condition for parallel a + bm + cn = 0 b.n = 0

24. Condition For A Line To Lie In A Plane


x  x1 y  y1 z  z1
(i) Cartesian form: Line = = would lie in a plane
 m n
ax + by + cz + d = 0, if ax1 + by1 + cz1 + d = 0 & a + bm + cn = 0.
 b would lie in the plane r . n = d if b . n = 0 & a . n = d
    
Vector form: Line r = a +
   
(ii)

2 5 . Coplanar Lines:
x  y  z   x  ' y  ' z   '
(i) If the given lines are = = and = = , then condition
 m n ' m' n'

   '   '    '


for intersection/coplanarity is  m n = 0 & plane containing the above
' m' n'

RESONANCE 62
x  y  z 
two lines is  m n =0
' m' n'
(ii) Condition of coplanarity if both the lines are in general form Let the lines be
ax + by + cz + d = 0 = ax + by + c z + d &
x +  y + z +  = 0 = x + y + z + 

a b c d
a' b' c' d'
They are coplanar if =0
   
' ' ' '

Alternative method
get vector along the line of shortest distance as

i j k
 m n
 m n

Now get unit vector along this vector


û = i + mj + nk
Let v = ( – ) î + (B – B) ĵ + (y – y)

S. D. = u. v
Example :
Find the distance of the point (1, 0, – 3) from the plane x – y – z = 9 measured parallel to the line
x2 y2 z6
  .
2 3 6
Solution
Given plane is x – y – z = 9 ..... (1)
x2 y2 z6
Given line AB is   ..... (2)
2 3 6
Equation of a line passing through the point Q(1, 0, – 3) and parallel to line (2) is
x 1 y z  3
  = r.. ..... (3)
2 3 6
Co-ordinates of point on line (3) may be taken as
P (2r + 1, 3r, – 6r – 3)
If P is the point of intersection of line (3) and plane (1), then P lies on plane (1), B
 (2r + 1) – (3r) – (– 6r – 3) = 9
r=1 Q (1, 0, – 3)
or, P  (3, 3, – 9)
A
Distance between points Q (1, 0, – 3) and P (3, 3, – 9)

PQ = (3  1)2  (3  0 )2  ( 9  (3))2 = 4  9  36 = 7. P

RESONANCE 63
Example :
x  3 y 1 z  2
Find the equation of the plane passing through (1, 2, 0) which contains the line   .
3 4 2
Solution
Equation of any plane passing through (1, 2, 0) may be taken as
a (x – 1) + b (y – 2) + c (z – 0) = 0 ..... (1)
where a, b, c are the direction ratios of the normal to the plane. Given line is
x  3 y 1 z  2
  ..... (2)
3 4 2
If plane (1) contains the given line, then
3a + 4b – 2c = 0 ..... (3)
Also point (– 3, 1, 2) on line (2) lies in plane (1)
 a (– 3 – 1) + b (1 – 2) + c (2 – 0) = 0
or, – 4a – b + 2c = 0 ..... (4)
Solving equations (3) and (4), we get,
a b c
 
8  2 8  6  3  16

a b c
or,   = k (say). ..... (5)
6 2 13
Substituting the values of a, b and c in equation (1), we get,
6 (x – 1) + 2 (y – 2) + 13 (z – 0) = 0.
or, 6x + 2y + 13z – 10 = 0. This is the required equation.

Example :

x 1 y 1 z  3
Find the equation of the projection of the line   on the plane x + 2y + z = 9.
2 1 4
Solution B
A
Let the given line AB be
x 1 y 1 z  3
  ..... (1)
2 1 4
Given plane is
x + 2y + z = 9 ..... (2) D C
Let DC be the projection of AB on plane (2)
Clearly plane ABCD is perpendicular to plane (2).
Equation of any plane through AB may be taken as (this plane passes through the point (1, – 1, 3) on
line AB)
a (x – 1) + b (y + 1) + c (z – 3) = 0 ..... (3)
where 2a – b + 4c = 0 ..... (4)
[ normal to plane (3) is perpendicular to line (1)]
Since plane (3) is perpendicular to plane (2),
 a + 2b + c = 0 ..... (5)
Solving equations (4) & (5), we get,
a b c
  .
9 2 5
Substituting these values of a, b and c in equation (3), we get
9 (x – 1) – 2 (y + 1) – 5 (z – 3) = 0
or, 9x – 2y – 5z + 4 = 0 ...... (6)
Since projection DC of AB on plane (2) is the line of intersection of plane ABCD and plane (2), therefore
equation of DC will be

RESONANCE 64
9x  2y  5z  4  0 .....(i) 
and  ..... (7)
x  2y  z  9  0 .....(ii)
Let , m, n be the direction ratios of the line of intersection of planes (i) and (ii)
 9– 2m – 5n = 0 ..... (8)
and + 2m + n = 0 ..... (9)
m n
  

 2  10  5  9 18  2

Example :

x 3 y 1 z  2 x7 y z2
Show that the lines   and   are coplanar. Also find the equation of
2 3 1 3 1 2
the plane containing them.
Solution
Given lines are
x 3 y 1 z  2
  = r (say) ..... (1)
2 3 1

x7 y z7
and   = R (say) ..... (2)
3 1 2
If possible, let lines (1) and (2) intersect at P.
Any point on line (1) may be taken as
(2r + 3, – 3r – 1, r – 2) = P (let).
Any point on line (2) may be taken as
(– 3R + 7, R, 2R – 7) = P (let).
 2r + 3 = – 3R + 7
or, 2r + 3R = 4 ..... (3)
Also – 3r – 1 = R
or, – 3r – R = 1 ..... (4)
and r – 2 = 2R – 7
or, r – 2R = – 5. ..... (5)
Solving equations (3) and (4), we get,
r = – 1, R = 2
Clearly r = – 1, R = 2 satisfies equation (5).
Hence lines (1) and (2) intersect.
 lines (1) and (2) are coplanar.
Equation of the plane containing lines (1) and (2) is

x  3 y 1 z  2
2 3 1
=0
3 1 2

or, (x – 3) (– 6 – 1) – (y + 1) (4 + 3) + (z + 2) (2 – 9) = 0
or, – 7 (x – 3) – 7 (y + 1) – 7 (z + 2) = 0
or, x–3+y+1+z+2=0 or, x + y + z = 0.

Self practice problems:


x2 y3 z6
1. Find the values of a and b for which the line   is perpendicular to the plane
a 4 2
3x – 2y + bz + 10 = 0.
Ans. a = 3, b = – 2

RESONANCE 65
x 1 y  2 z  3 x2 y3 z4
2. Prove that the lines   and   are coplanar. Also find the equation
2 3 3 3 4 5
of the plane in which they lie.
Ans. x – 2y + z = 0

x2 y 3 z4
3. Find the plane containing the line = = and parallel to the line
2 3 5

x 1 y 1 z  1
= =
1 2 1
Ans. 13x + 3y – 72 – 7 = 0

x 1 y2 z3 x4 y 1


4. Show that the line = = & = = z are intersecting each other. Find thire
2 3 4 5 2
intersection and the plane containing the line.
Ans. (– 1, – 1, – 1) & 5x – 8y + 11z – 2 = 0

Show that the lines r = (– î – 3 ĵ – 5 k̂ ) +  (–3 î – 5 ĵ – 7 k̂ ) & r (2 î + 4 ĵ + 6 k̂ ) + µ ( î +4 ĵ + 7 k̂ ) are



5.

 
coplanar and find the plane containing the line.
Ans. r . î  2 ĵ  k̂ = 0

2 6 . Skew Lines:
(i) The straight lines which are not parallel and noncoplanar i.e. nonintersecting are called

' '  ' 


skew lines. If =  m n  0, then lines are skew..
' m' n'

(ii) Shortest distance: Suppose the equation of the lines are

x  y  z   x  ' y  ' z   '


= = and  
 m n ' m' n'

(  ' ) (mn'm' n)  (  ' ) (n  n' )  (   ' ) (m'' m)


 (mn'm' n)
S.D. =
2

' '  ' 


=
 m n   (mn  mn) 2

' m' n'

Vector Form: For lines a 1 +  b1 & a 2 +  b 2 to be skew


 
(iii)
 

( b1 x b 2 ). ( a 2  a1 )  0 [ b1 b 2 ( a 2  a1 )]  0.
       
or

(iv) Shortest distance between the two parallel lines r = a 1 + b &

(a 2  a1 ) x b
  
r = a 2 +  b is d =
  
 .
|b|

RESONANCE 66
Example :
Find the shortest distance and the vector equation of the line of shortest distance between the lines
given by

          
r  3 r  8 j  3 k    3 r  j  k      
and r  3 r  7 j  6 k     3 i  2 j  4 k 
   
Solution
Given lines are

       
r  3 r  8 j  3 k    3 i  j  k  ..... (1)
 

        A L B
and r  3 r  7 j  6 k     3 i  2 j  4 k  ..... (2)
  90°
Equation of lines (1) and (2) in cartesian form is
90°
x3 y8 z3
AB :   = C M D
3 1 1

 1 
and CD :   
 3 
Let L  (3 + 3, –  + 8,  + 3)
and M  (– 3 – 3, 2 – 7, 4 + 6)
Direction ratios of LM are
3 + 3 + 6, –  – 2 + 15,  – 4 – 3.
Since LM  AB
 3 (3 + 3 + 6) – 1 (– – 2 + 15) + 1 ( – 4 – 3) = 0
or, 11 + 7 = 0 ..... (5)
Again LM  CD
 – 3 (3 + 3 + 6) + 2 (–  – 2 + 15) + 4 ( – 4 – 3) = 0
or, – 7 – 29 – 0 ..... (6)
Solving (5) and (6), we get
 = 0,  = 0
 L  (3, 8, 3), M  (– 3, – 7, 6)

Hence shortest distance LM = ( 3  3 ) 2  ( 8  7 ) 2  (3  6 ) 2

= 270 = 3 30 units
Vector equation of LM is

       
r  3 i  8 j  3 k  t  6 i  15 j  3 k 
 

x3 y8 z3


Note : Cartesian equation of LM is   .
6 15 3

Example :
Prove that the shortest distance between any two opposite edges of a tetrahedron formed by the

planes y + z = 0, x + z = 0, x + y = 0, x + y + z = 3 a is 2 a.
Solution
Given planes are y+z=0 ..... (i) x+z=0 ..... (ii)

x+y=0 ..... (iii) x+y+z= 3a ..... (iv)

RESONANCE 67
Clearly planes (i), (ii) and (iii) meet at O(0, 0, 0)
Let the tetrahedron be OABC
Let the equation to one of the pair of opposite edges OA and BC be P A
, 0)
y + z = 0, x + z = 0 ..... (1) O (0, 0

x + y = 0, x + y + z = 3 a ..... (2)
equation (1) and (2) can be expressed in symmetrical form as
x0 y0 z0 C
  ..... (3) (0, 0, Q D
1 1 1 3 a)
x0 y0 z 3a
and,   ..... (4)
1 1 0
d. r. of OA and BC are (1, – 1) and (1, – 1, 0).
Let PQ be the shortest distance between OA and BC having direction cosine (, m, n)
 PQ is perpendicular to both OA and BC.
 +m–n=0
and –m=0
Solving (5) and (6), we get,
 m n
  = k (say)
1 1 2
also, 2 + m 2 + n2 = 1
O
A
1
 k 2 + k 2 + 4k 2 = 1  k =
6

1 1 2
 = ,m= ,n= B C
6 6 6
Shortest distance between OA and BC
i.e. PQ = The length of projection of OC on PQ O P A
= | (x 2 – x 1)  + (y2 – y1) m + (z2 – z1) n | 90°
1 1 2
= 0. 0.  3a. = 2 a. 90°
6 6 6 C Q B

Self practice problems:


x 1 y  2 z  3 x2 y4 z5
1. Find the shortest distance between the lines   and   . Find also
2 3 4 3 4 5
its equation.
1
Ans. , 6x – y = 10 – 3y = 6z – 25
6
2. Prove that the shortest distance between the diagonals of a rectangular parallelopiped whose sides are
a, b, c and the edges not meeting it are
bc ca ab
, ,
b2  c 2 c 2  a2 a2  b2

27. Sphere
General equation of a sphere is given by x 2 + y2 + z2 + 2ux + 2vy + 2wz + d = 0 ( u, v, w) is the
centre and u 2  v 2  w 2  d is the radius of the sphere.

RESONANCE 68
Example :
Find the equation of the sphere having centre at (1, 2, 3) and touching the plane x + 2y + 3z = 0.
Solution
Given plane is x + 2y + 3z = 0 ..... (1)
Let H be the centre of the required sphere.
Given H  (1, 2, 3) H
Radius of the sphere,
HP = length of perpendicular from H to plane (1) P

| 1 2  2  3  3 |
=
14

=
14
Equation of the required sphere is
(x – 1)2 + (y – 2)2 + (z – 3)2 = 14
or x 2 + y2 + z2 – 2x – 4y – 6z = 0

Example :
   
Find the equation of the sphere if it touches the plane r .(2 i  2 j  k ) = 0 and the position vector of its

  
centre is 3 i  6 j  4 k
Solution
   
Given plane is r .(2 i  2 j  k ) = 0 ..... (1)
Let H be the centre of the sphere, then
    
OH  3 i  6 j  4 k  c (say)
Radius of the sphere = length of perpendicular from H ot plane (1)
   
| c .( 2 i  2 j  k ) |
=   
|2 i 2 j k |

     
| (3 i  6 j  4 k ).(2 i  2 j  k ) |
=   
|2 i2 jk |

| 6  12  4 | 2
=  = a (say)
3 3
Equation of the required sphere is
 
| rc| =a

      2
or | x i  y j  z k  (3 i  6 j  4 k ) | 
3

   4
or | (x – 3) i + (y – 6) j + (z + 4) k |2 =
9

4
or (x – 3)2 + (y – 6)2 + (z + 4)2 =
9
or 9 (x2 + y2 + z2 – 6x – 12y + 8z + 61) = 4
or 9x 2 + 9y2 + 9z2 – 54x – 108y + 72z + 545 = 0

RESONANCE 69
Example :
Find the equation of the sphere passing through the points (3, 0, 0), (0, – 1, 0), (0, 0, – 2) and whose
centre lies on the plane 3x + 2y + 4z = 1
Solution
Let the equation of the sphere be
x 2 + y2 + z2 + 2ux + 2vy + 2wz + d = 0 ..... (1)
Let A  (3, 0, 0), B  (0, – 1, 0), C  (0, 0, – 2)
Since sphere (1) passes through A, B and C,
 9 + 6u + d = 0 ..... (2)
1 – 2v + d = 0 ..... (3)
4 – 4w + d = 0 ..... (4)
Since centre (– u, – v, – w) of the sphere lies on plane
3x + 2y + 4z = 1
 – 3u – 2v – 4w = 1 ..... (5)
(2) – (3)  6u + 2v = – 8 ..... (6)
(3) – (4)  – 2v + 4w = 3 ..... (7)
2v  8
From (6), u = ..... (8)
6
From (7), 4w = 3 + 2v ..... (9)
Putting the values of u, v and w in (5), we get
2v  8
 2v – 3 – 2v  1
2
 2v + 8 – 4v – 6 – 4v = 2  v=0
08 4
From (8), u= 
6 3

3
From (9), 4w = 3  w=
4
From (3), d = 2v – 1 = 0 – 1 = – 1
From (1), equation of required sphere is
08 8 3
x 2 + y2 + z2 –  x+ z–1=0
6 3 2
or 6x 2 + 6y2 + 6z2 – 16x + 9z – 6 = 0

Example :
Find the equation of the sphere with the points (1, 2, 2) and (2, 3, 4) as the extremities of a diameter.
Find the co-ordinates of its centre.
Solution
Let A  (1, 2, 2), B  (2, 3, 4)
Equation of the sphere having (x1, y1, z1) and (x 2, y2, z2) as the extremities of a diameter is
(x – x 1) (x – x 2) + (y – y1) (y – y2) + (z – z1) (z – z2) = 0
Here x1 = 1, x2 = 2, y1 = 2, y2 = 3, z1 = 2, z2 = 4
 required equation of the sphere is
(x – 1) (x – 2) + (y – 2) (y – 3) + (z – 2) (z – 4) = 0
or x 2 + y2 + z2 – 3x – 5y – 6z + 16 = 0
Centre of the sphere is middle point of AB

3 5 
 Centre is  , , 3 
2 2 

RESONANCE 70
Self practice problems:

1. Find the value of k for which the plane x + y + z = 3 k touches the sphere
x 2 + y2 + z2 – 2x – 2y – 2z – 6 = 0.
Ans. 3 ±3

2. Find the equation to the sphere passing through (1, – 3, 4), (1, – 5, 2) and (1, – 3, 0) which has its
centre in the plane x + y + z = 0
Ans. x 2 + y2 + z2 – 2x + 6y – 4z + 10 = 0

3. Find the equation of the sphere having centre on the line 2x – 3y = 0, 5y + 2z = 0 and passing through
the points (0, – 2, – 4) and (2, – 1, – 1).
Ans. x 2 + y2 + z2 – 6x – 4y + 10 z + 12 = 0

4. Find the centre and radius of the circle in which the plane 3x + 2y – z – 7 14 = 0 intersects the sphere
x 2 + y2 + z2 = 81.
Ans. 4 2 units

5. A plane passes through a fixed point (a, b, c) and cuts the axes in A, B, C. Show that the locus of the
centre of the sphere OABC is
a b c
 
x y z = 2.

RESONANCE 71
Part : (A) Only one correct option

1. The locus of a point P which moves such that PA2 – PB2 = 2k 2 where A and B are (3, 4, 5) and
(– 1, 3 – 7) respectively is
(A) 8x + 2y + 24z – 9 + 2k 2 = 0 (B) 8x + 2y + 24z – 2k 2 = 0
(C) 8x + 2y + 24z + 9 + 2k = 0
2
(D) none of these

2. The position vectors of three points A, B, C are i  2 j  3 k , 2 i  3 j  k & 3 i  j  2 k . A unit vector


perpendicular to the plane of the triangle ABC is:


(A)  


1    
 i jk
3
 (B) 
 3

 1    
 i jk 

(C) 
 3

 1    
 i jk  (D) none

3. The square of the perpendicular distance of a point P (p, q, r) from a line through A(a, b, c) and whose
direction cosine are , m, n is
(A)  {(q – b) n – (r – c) m} 2 (B)  {(q + b) n – (r + c) m} 2
(C)  {(q – b) n + (r – c) m} 2
(D) none of these

4. A variable plane passes through a fixed point (1, 2, 3). The locus of the foot of the perpendicular drawn
from origin to this plane is:
(A) x 2 + y2 + z2  x  2y  3z = 0 (B) x 2 + 2y2 + 3z2  x  2y  3z = 0
(C) x + 4y + 9z + x + 2y + 3 = 0
2 2 2
(D) x2 + y2 + z2 + x + 2y + 3z = 0

5. The equation of the plane which bisects the angle between the planes 3x  6y + 2z + 5 = 0 and
4x  12y + 3z  3 = 0 which contains the origin is
(A) 33x  13y + 32z + 45 = 0 (B) x  3y + z  5 = 0
(C) 33x + 13y + 32z + 45 = 0 (D) None of these

6. The distance of the point of intersection of the line x – 3 = (1/2) (y–4) = (1/2) (z–5) and the plane
x + y + z = 17 from the point (3, 4, 5)
(A) 2 (B) 3 (C) 1/3 (D) 1/2

7. The lines x = ay + b, z = cy + d and x = a’y + b’, z = c’y + d’ will be mutually perpendicular provided
(A) (a + a’)(b + b’) (c + c’) (B) aa’ + cc’ + 1 = 0
(C) aa’ + bb’ + cc’ + 1 = 0 (D) (a + a’) (b + b’) (c + c’) + 1 = 0

a +  b meets the plane r . n̂ = p in the point P whose position vector is
 
A straight line r =

8.

  a . n̂     p  a . n̂     a . n̂     p  a . n̂  
   
(A) a +    b (B) a +    b (C) a    b (D) a     b
 b . n̂   b . n̂   b . n̂   b . n̂ 

x 1 y  2 z3 x 1 y  2 z3
9. Equation of the angle bisector of the angle between the lines = = & = = is
1 1 1 1 1 1

x 1 y2 x 1 y2 z3


(A) = ;z –3=0 (B) = =
2 2 1 2 3

y2 z3
(C) x – 1 = 0 ; = (D) None of these
1 1

RESONANCE 72
x  2 y 1 z  2
10. The distance of the point, ( 1,  5,  10) from the point of intersection of the line,  
3 4 12
and the plane, x  y + z = 5, is:
(A) 10 (B) 11 (C) 12 (D) 13

11. If a plane cuts off intercepts OA = a, OB = b, OC = c from the coordinate axes, then the area of the
triangle ABC =

1 1
(A)
2 b 2 c2  c2a 2  a 2 b2 (B)
2
(bc + ca + ab)

1 1
(C) abc (D) (b  c)2  (c  a)2  (a  b)2
2 2

12. The angle between the lines whose direction cosines satisfy the equations  + m + n = 0 and
2 = m 2 + n2 is
   
(A) (B) (C) (D)
6 2 3 4

13. If a1, b1, c 1 and a2 , b2, c 2 are the direction ratios of two lines and  is the angle between the lines then
tan  is equal to

(b1c 2  b 2c 1 )2 (b1c 2  b 2c 1 )2
(A) (B)
a1b1  a 2b 2  c 1c 2 a1a 2  b1b 2  c 1c 2

(b1c 2  b 2c1 )2
(C) (D) none of these
a1a 2  b1b 2  c 1c 2

14. A point moves so that the sum of the squares of its distances from the six faces of a cube given by
x = ± 1, y = ± 1, z = ± 1 is 10 units. The locus of the point is
(A) x 2 + y2 + z2 = 1 (B) x 2 + y2 + z2 = 2 (C) x + y + z = 1 (D) x + y + z = 2

15. In the adjacent figure ‘P’ is any arbitrary interior point of the triangle ABC
such that the lines AA1, BB1 and CC1 are concurrent at P. Value of
PA 1 PB1 PC1
AA 1 + BB1 + CC1 is always equal to .

(A) 1 (B) 2
(C) 3 (D) None of these

16. The plane ax + by + cz = d, meets the coordinate axes at the points A, B and C respectively. Area of triangle
ABC is equal to

d2 a 2  b 2  c 2 d2 a 2  b 2  c 2 d2 a 2  b 2  c 2
(A) (B) (C) (D) None of these
| abc | 2 | abc | 4 | abc |

17. The length of projection, of the line segment joining the points (1, –1, 0) and (–1, 0, 1), to the plane
2x + y + 6z = 1, is equal to

255 237 137 155


(A) (B) (C) (D)
61 61 61 61

RESONANCE 73
18. Two systems of rectangular axes have the same origin. If a plane cuts them at distances a, b, c and
a1, b1, c1 from the origin, then

1 1 1 1 1 1 1 1 1 1 1 1
(A)   =   (B)   =  
a 2
b 2
c 2 a12 b12 c 12 a 2
b 2
c 2 a12 b12 c 12

(C) a2 + b2 + c2 = a1  b1  c 1 (D) a2 – b2 + c2 = a12  b12  c 12


2 2 2

19. The angle between the plane 2x – y + z = 6 and a plane perpendicular to the planes x + y + 2z = 7 and
x – y = 3 is :
   
(A) (B) (C) (D)
4 3 6 2

20. The non zero value of ‘a’ for which the lines 2x – y + 3z + 4 = 0 = ax + y – z + 2 and
x – 3y + z = 0 = x + 2y + z + 1 are co-planar is :
(A) – 2 (B) 4 (C) 6 (D) 0

21. The equation of the plane through the point (–1, 2 , 0) and parallel to the lines

x y 1 z  2 x  1 2y  1 z  1
= = and = = is -
3 0 1 1 2 1
(A) x + 2y + 3z - 1 = 0 (B) x – 2y + 3z + 5 = 0
(C) x + y – 3z + 1 = 0 (D) x + y + 3z – 1 = 0

22. The equation of the plane bisecting the acute angle between the planes 2x + y + 2z = 9 and
3x – 4y + 12z + 13 = 0 is :
(A) 11x + 33y – 34z – 172 = 0 (B) 11x + 33y – 34z – 182 = 0
(C) 41x – 7y + 86z – 52 = 0 (D) 41x – 7y + 86z – 62 = 0

23. The base of the pyramid AOBC is an equilateral triangle OBA with each side equal to 4 2 , ' O ' is the

| |

origin of reference, AO is perpendicular to the plane of  OBC and AO = 2 . Then the cosine of the
angle between the skew straight lines one passing through A and the mid point of OB and the other
passing through O and the mid point of BC is :
1 1 1
(A)  (B) 0 (C) (D)
2 6 2

24. The coplanar points A , B , C , D are (2  x , 2 , 2) , (2 , 2  y , 2) , (2 , 2 , 2  z) and (1 , 1 , 1)


respectively . Then :

1 1 1
(A)   =1 (B) x + y + z = 1
x y z

1 1 1
(C) + + =1 (D) none of these
1 x 1 y 1 z

 
25. Let the centre of the parallelopiped formed by PA  i  2 j  2 k ; PB  4 i  3 j  k ;


PC  3 i  5 j  k is given by the position vector (7, 6, 2). Then the position vector of the point P is:
(A) (3, 4, 1) (B) (6, 8, 2) (C) (1, 3, 4) (D) (2, 6, 8)

RESONANCE 74
  1 
26. Taken on side A C of a triangle ABC, a point M such that A M = A C . A point N is taken on the
3
    
side CB such that BN = CB then, for the point of intersection X of A B & MN which of the following
holds good?
 1   1   3   
(A) X B = AB (B) A X = AB (C) XN = MN (D) X M = 3 XN
3 3 4
27. If the acute angle that the vector,  i   j   k makes with the plane of the two vectors

2 i  3 j  k & i  j  2 k is cot 1 2 then:


(A)  ( + ) =  (B)  ( + ) = 
(C)  ( + ) =  (D)  +  +  = 0

 1
28. Locus of the point P, for which OP represents a vector with direction cosine cos  =
2
( ' O ' is the origin) is:
(A) A circle parallel to y z plane with centre on the x  axis
(B) a cone concentric with positive x  axis having vertex at the origin and the slant
height equal to the magnitude of the vector
(C) a ray emanating from the origin and making an angle of 60º with x  axis

(D) a disc parallel to y z plane with centre on x  axis & radius equal to O P sin 60º | | 

Part : (B) May have more than one options correct

x  x2 y  y2 z  z2
29. Equation of the plane passing through A(x1, y1, z1) and containing the line = = is
d1 d2 d3

x  x1 y  y1 z  z1 x  x2 y  y2 z  z2
x 2  x1 y 2  y1 z 2  z1 x1  x 2 y1  y 2 z1  z 2
(A) =0 (B) =0
d1 d2 d3 d1 d2 d3

x  d1 y  d2 z  d3 x y z
x1 y1 z1 x1  x 2 y1  y 2 z1  z 2
(C) =0 (D) =0
x2 y2 z2 d1 d2 d3

30. The equations of the line of shortest distance between the lines
x y z x2 y 1 z2
  and = = are
2 3 1 3 5 2

x  ( 62 / 3) y  31 z  (31/ 3)
(A) 3(x – 21) = 3y + 92 = 3z – 32 (B) = =
1/ 3 1/ 3 1/ 3

x  21 y  (92 / 3) z  (32 / 3) x2 y3 z 1


(C) = = (D) = =
1/ 3 1/ 3 1/ 3 1/ 3 1/ 3 1/ 3

31. A line passes through a point A with p.v. 3 i  j  k & is parallel to the vector 2 i  j  2 k . If P is a point on
this line such that AP = 15 units, then the p.v. of the point P is:

(A) 13 i  4 j  9 k (B) 13 i  4 j  9 k (C) 7 i  6 j  11 k (D)  7 i  6 j  11 k

RESONANCE 75
32. The equations of the planes through the origin which are parallel to the line
x 1 y3 z 1 5
= = and distant from it are
2 1 2 3
(A) 2x + 2y + z = 0 (B) x + 2y + 2z = 0 (C) 2x – 2y + z = 0 (D) x – 2y + 2z = 0

33. The value(s) of k for which the equation x 2 + 2y2 – 5z2 + 2kyz + 2zx + 4xy = 0 represents a pair of
planes passing through origin is/are
(A) 2 (B) – 2 (C) 6 (D) – 6

x y 2
34. The equation of lines AB is = = . Through a point P(1, 2, 5), line PN is drawn perpendicular
2 3 6
to AB and line PQ is drawn parallel to the plane 3x + 4y + 5z = 0 to meet AB is Q. Then

 52 78 156 
(A) coordinate of N is  ,  , 
 49 49 49 

 9 
(B) the coordinates of Q is  3,  , 9 
 2 

x 1 y2 z5
(C) the equation of PN is = =
3  176  89

x 1 y2 z5
(D) the equation of PQ is = =
4  13 8

x  15 y  29 z5
35. Let a perpendicular PQ be drawn from P (5, 7, 3) to the line = = when Q is the
3 8 5
foot. Then
(A) Q is (9, 13, – 15)
(B) PQ = 14
(C) the equation of plane containing PQ and the given line is 9x – 4y – z – 14 = 0
(D) none of these

1. Find the equation of the plane which contains the origin and the line of intersection of the
 
planes r . a = p and r . b = q
 

x a y b zc x  a  y  b z  c
2. If the lines = = and = = intersect at a point then the coordinate of the
a b c a b c
point of intersection.

3. The locus of a point which is a equidistant from the two given points with position vectors

 1   
a and b is the plane  r  (a  b ) . ( a – b ) = 0 bisecting the line joining the points normally..
   
 2 

4. The foot of the perpendicular from (a, b, c) on the line x = y = z is the point (r, r, r) where
3r = a + b + c.

RESONANCE 76
5. Match the following :
Column A Column B
(a) Sum of the square of the direction (P) 0
cosines of line is

(b) All the points on the z-axis have (Q) 1


their x and y coordinate equal to

(c) Distance between the points (1, 3, 2) (R) 9


and (2, 3, 1) is
(d) Shortest distance between the lines (S) 2

x6 y2 z2 x4 y z 1


= = and = = is
1 2 2 3 2 2

6. Show that the angle between the straight lines whose direction cosines are given by the equations

 1 1 1
 + m + n = 0 and amn + bn + cm = 0 is if + + = 0.
3 a b c

7. Prove that the two lines whose direction cosines are given by the relations.p + qm + rn = 0 &
a2 + bm 2 + cn2 = 0 are perpendicular if, p2(b + c) + q2 (c + a) + r2 (a + b) = 0 and parallel if

p2 q 2 r 2
  0.
a b c

8. Find the plane  passing through the points of intersection of the planes 2x  3y z +1= 0 and
x  y  2z + 3 = 0 and is perpendicular to the plane 3x  y  2z = 4. Find the image of point (1, 1, 1) in
plane 

Given parallel planes r . (2 î  ĵ + k̂ ) = 3 and r . (4 î  ĵ   k̂ ) = 5 for what values of , planes


 
9.

r . ( î  ĵ + 3 k̂ ) = 0 & r . ( î  ĵ + 2 k̂ ) = 0 would be perpendicular..


 

10. The edges of a rectangular parallelepiped are a, b, c; show that the angles between the four diagonals

a 2  b2  c2
are given by cos .
a 2  b2  c2
1

Prove that the line of intersection of the planes r . ( î   ĵ + 3 k̂ ) = 0 and r . (3 î   ĵ + k̂ ) = 0 is


 
11.

r = t( î   ĵ + k̂ ). Show that the line is equally inclined to î and k̂ and makes an angle

(1/2) sec 1 3 with. ĵ .

12. Find the shortest distance between the lines

x 1 y 1 x 1
= =z& = (y  2); z = 2
2 3 3

Show that the line L whose equation is, r = (2 î  2 ĵ  3 k̂ ) +  ( î  ĵ  4 k̂ ) is parallel to the plane 

13.

whose vector r . ( î  5 ĵ  k̂ ) = 5. Find the distance between them.


RESONANCE 77

r b
 
A sphere has an equation r  a b = 2 î  4 ĵ  2 k̂ . Find:
  2
= 72 where a = î  3 ĵ  6 k̂ and
2 
14. +

(i) the centre of the sphere


(ii) the radius of the sphere

 
(iii) perpendicular distance from the centre of the sphere to the plane

r . 2 î  2 ĵ  k̂ =  3.

15. Find the equation of the sphere which is tangential to the plane x  2y  2z = 7 at (3, 1, 1) and
passes through the point (1, 1, 3).

16. P1 and P2 are planes passing through origin. L1 and L2 also passes through origin. L1 lies on P1 not on P2 and
L2 lies on P2 but not on P1. Show that there exists points A, B, C and whose permutation A .B.C can be
chosen such that
(i) A is on L1, B on P1 but not on L1 and C not on P1.
(ii) A in on L2, B on P2 but not on L2 and C not on P2. [IIT - 2004]

17. A parallelopiped ‘S’ has base points A, B, C and D and upper face points A, B, C and D. This parallelopiped
is compressed by upper face ABCD to form a new parallelopiped ‘T’ having upper face points A, B, C
and D. Volume of parallelpiped T is 90 percent of the volume of parallelopiped S. Prove that the locus of ‘A’
is a plane. [IIT - 2004]

EXERCISE # 1 EXERCISE # 2
1. C 2. A 3. A
r . (aq  pb) = 0
  
1.
4. A 5. D 6. B
7. B 8. B 9. A 2. ( a  a, b  b, c  c  )
10. D 11. A 12. C 3. True 4. True
13. B 14. B 15. A 5. (a)  (Q), (b)  (P), (c)  (S), (d)  (R)
16. B 17. B 18. A
 12  78 57 
19. D 20. A 21. A 8. 7x + 13y + 4z – 9 = 0 ;  , , 
 117 117 117 
22. C 23. D 24. A
3 10
25. A 26. C 27. A 9.  = + 3 12. 13.
59 3 3
28. B 29. AB 30. ABC
31. AB 32. AD 33. BC 8
14. (i) (0, 5, 5) (ii) 9 (iii)
3
34. ABCD 35. BC
15. (x – 2)2 + (y – 1)2 + (z – 1)2 = 5

RESONANCE 78
Permutation and Combination

Permutations are arrangements and combinations are selections.


In this chapter we discuss the m ethods of counting of
arrangements and selections. The basic results and formulas
are as follows:

1. Fundamental Principle of Counting :


A man has one pair
(i) Principle of Multiplication:
of rabbits at a certain
If an event can occur in ‘m’ different ways, following which another
place entirely event can occur in ‘n’ different ways, then total number of different
ways of simultaneous occurrence of both the events in a definite
surrounded by a wall. order is m  n.
We wish to know how
(ii) Principle of Addition:
many pairs will be If an event can occur in ‘m’ different ways, and another event can
occur in ‘n’ different ways, then exactly one of the events can
bred from it in one happen in m + n ways.
year, if the nature of Example # 1
these rabbits is such There are 8 buses running from Kota to Jaipur and 10 buses
running from Jaipur to Delhi. In how many ways a person can
that they breed every
travel from Kota to Delhi via Jaipur by bus.
month one other pair Solution.
Let E1 be the event of travelling from Kota to Jaipur & E2 be the
and begin to breed in event of travelling from Jaipur to Delhi by the person.
E1 can happen in 8 ways and E2 can happen in 10 ways.
the second month after
Since both the events E1 and E2 are to be happened in order,
simultaneously, the number of ways = 8 × 10 = 80.
theirbirth....
Example # 2
How many numbers between 10 and 10,000 can be formed by
using the digits 1, 2, 3, 4, 5 if
Fi bonacci (i) No digit is repeated in any number.
(ii) Digits can be repeated.
Solution.
(i) Number of two digit numbers = 5 × 4 = 20
Number of three digit numbers = 5 × 4 × 3 = 60
Number of four digit numbers = 5 × 4 × 3 × 2 = 120
Total = 200

(ii) Number of two digit numbers = 5 × 5 = 25


Number of three digit numbers = 5 × 5 × 5 = 125
Number of four digit numbers = 5 × 5 × 5 × 5 = 625
Total = 775

RESONANCE 1
Self Practice Problems :

1. How many 4 digit numbers are there, without repetition of digits, if each number is divisible by 5.
Ans. 952

2. Using 6 different flags, how many different signals can be made by using atleast three flags, arranging
one above the other.
Ans. 1920

2. Arrangement :

If nPr denotes the number of permutations of n different things, taking r at a time, then

n!
n
Pr = n (n  1) (n  2)..... (n  r + 1) =
(n  r )!
NOTE : (i) factorials of negative integers are not defined.
(ii) 0 ! = 1 ! = 1 ;
(iii) nPn = n ! = n. (n  1) !
(iv) (2n) ! = 2n. n ! [1. 3. 5. 7... (2n  1)]

Example # 3
How many numbers of three digits can be formed using the digits 1, 2, 3, 4, 5, without repetition of
digits. How many of these are even.
Solution.
Three places are to be filled with 5 different objects.
 Number of ways = 5P3 = 5 × 4 × 3 = 60
For the 2nd part, unit digit can be filled in two ways & the remaining two digits can be filled in 4P2 ways.
 Number of even numbers = 2 × 4P2 = 24.

Example # 4
If all the letters of the word 'QUEST' are arranged in all possible ways and put in dictionary order, then
find the rank of the given word.
Solution.
Number of words beginning with E = 4P4 = 24
Number of wards beginning with QE = 3P3 = 6
Number of words beginning with QS = 6
Number of words beginning withQT = 6.
Next word is 'QUEST'
 its rank is 24 + 6 + 6 + 6 + 1 = 43.

Self Practice Problems :

3. Find the sum of all four digit numbers (without repetition of digits) formed using the digits
1, 2, 3, 4, 5.
Ans. 399960

4. Find 'n', if n–1


P3 : nP4 = 1 : 9.
Ans. 9

5. Six horses take part in a race. In how many ways can these horses come in the first, second and third
place, if a particular horse is among the three winners (Assume No Ties).
Ans. 60

RESONANCE 2
3. Circular Permutation :
The number of circular permutations of n different things taken all at a time is; (n  1) !.
(n  1)!
If clockwise & anticlockwise circular permutations are considered to be same, then it is .
2
Note: Number of circular permutations of n things when p alike and the rest different taken all at a time
(n 1)!
distinguishing clockwise and anticlockwise arrangement is .
p!
Example # 5
In how many ways can we arrange 6 different flowers in a circle. In how many ways we can form a
garland using these flowers.
Solution.
The number of circular arrangements of 6 different flowers = (6 – 1)! = 120
When we form a garland, clockwise and anticlockwise arrangements are similar. Therefore, the number
1
of ways of forming garland = (6 – 1) ! = 60.
2
Example # 6
In how many ways 6 persons can sit at a round table, if two of them prefer to sit together.
Solution.
Let P1, P2, P3, P4, P5, P6 be the persons, where P1, P2 want to sit together.
Regard these person as 5 objects. They can be arranged in a circle in (5 – 1)! = 24. Now P1P2 can be
arranged in 2! ways. Thus the total number of ways = 24 × 2 = 48.

Self Practice Problems :

6. In how many ways the letters of the word 'MONDAY' can be written around a circle if the vowels are to
be separated in any arrangement.
Ans. 72

7. In how many ways we can form a garland using 3 different red flowers, 5 different yellow flowers and 4
different blue flowers, if flowers of same colour must be together.
Ans. 17280.

4. Selection :
If nCr denotes the number of combinations of n different things taken r at a time, then

n! n
P
Cr = = r where r  n ; n  N and r  W..
r! (n  r )!
n
r!
NOTE : (i) nCr = nCn – r
(ii) nCr + nCr – 1 = n + 1Cr
(iii) nCr = 0 if r  {0, 1, 2, 3........, n}

Example # 7
Fifteen players are selected for a cricket match.
(i) In how many ways the playing 11 can be selected
(ii) In how many ways the playing 11 can be selected including a particular player.
(iii) In how many ways the playing 11 can be selected excluding two particular players.
Solution.
(i) 11 players are to be selected from 15
Number of ways = 15C11 = 1365.
(ii) Since one player is already included, we have to select 10 from the remaining 14
Number of ways = 14C10 = 1001.
(iii) Since two players are to be excluded, we have to select 11 from the remaining 13.
Number of ways = 13C11 = 78.

RESONANCE 3
Example # 8
If 49C3r – 2 = 49C2r + 1, find 'r'.
Solution.
n
Cr = nCs if either r = s or r + s = n.
Thus 3r – 2 = 2r + 1  r=3
or 3r – 2 + 2r + 1 = 49  5r – 1 = 49  r = 10
 r = 3, 10

Example # 9
A regular polygon has 20 sides. How many triangles can be drawn by using the vertices, but not using
the sides.
Solution.
The first vertex can be selected in 20 ways. The remaining two are to be selected from 17 vertices so
that they are not consecutive. This can be done in 17C2 – 16 ways.
 The total number of ways = 20 × (17C2 – 16)
But in this method, each selection is repeated thrice.

20  (17 C 2  16 )
 Number of triangles = = 800.
3

Example # 10
10 persons are sitting in a row. In how many ways we can select three of them if adjacent persons are
not selected.
Solution.
Let P1, P2, P3, P4, P5, P6, P7, P8, P9, P10 be the persons sitting in this order.
If three are selected (non consecutive) then 7 are left out.
Let PPPPPPP be the left out & q, q, q be the selected. The number of ways in which these 3 q's can
be placed into the 8 positions between the P's (including extremes) is the number ways of required
selection.
Thus number of ways = 8C3 = 56.

Example # 11
In how many ways we can select 4 letters from the letters of the word MSSSSPP.
Solution.
M

SSSS
PP
Number of ways of selecting 4 alike letters = 2C1 = 2.
Number of ways of selecting 3 alike and 1 different letters = 2C1 × 3C1 = 6
Number of ways of selecting 2 alike and 2 alike letters = 3C2 = 3
Number of ways of selecting 2 alike & 2 different = 3C1 × 3C2 = 9
Number of ways of selecting 4 different = 4C4 = 1
Total = 21

Self Practice Problems :

8. In how many ways 7 persons can be selected from among 5 Indian, 4 British & 2 Chinese, if atleast two
are to be selected from each country.
Ans. 100

9. 10 points lie in a plane, of which 4 points are collinear. Barring these 4 points no three of the 10 points
are collinear. How many quadrilaterals can be drawn.
Ans. 185.

10. In how many ways 5 boys & 5 girls can sit at a round table so that girls & boys sit alternate.
Ans. 2880

RESONANCE 4
11. In how many ways 4 persons can occupy 10 chairs in a row, if no two sit on adjacent chairs.
Ans. 840.

12. In how many ways we can select 3 letters of the word PROPORTION.
Ans. 36

5. The number of permutations of 'n' things, taken all at a time, when 'p' of them are similar & of one type,
q of them are similar & of another type, 'r' of them are similar & of a third type & the remaining

n!
n  (p + q + r) are all different is .
p! q! r !

Example # 12
In how many ways we can arrange 3 red flowers, 4 yellow flowers and 5 white flowers in a row. In how
many ways this is possible if the white flowers are to be separated in any arrangement (Flowers of
same colour are identical).
Solution.
Total we have 12 flowers 3 red, 4 yellow and 5 white.
12 !
Number of arrangements = 3 ! 4 ! 5 ! = 27720.

For the second part, first arrange 3 red & 4 yellow


7!
This can be done in 3 ! 4 ! = 35 ways

Now select 5 places from among 8 places (including extremes) & put the white flowers there.
This can be done in 8C5 = 56.
 The number of ways for the 2nd part = 35 × 56 = 1960.

Example # 13
In how many ways the letters of the word "ARRANGE" can be arranged without altering the relative
positions of vowels & consonants.
Solution.

4!
The consonants in their positions can be arranged in 2 ! = 12 ways.

3!
The vowels in their positions can be arranged in 2 ! = 3 ways

 Total number of arrangements = 12 × 3 = 26

Self Practice Problems :

13. How many words can be formed using the letters of the word ASSESSMENT if each word begin with A
and end with T.
Ans. 840

14. If all the letters of the word ARRANGE are arranged in all possible ways, in how many of words we will
have the A's not together and also the R's not together.
Ans. 660

15. How many arrangements can be made by taking four letters of the word MISSISSIPPI.
Ans. 176.

RESONANCE 5
6. Formation of Groups :
Number of ways in which (m + n + p) different things can be divided into three different groups containing
m  n  p !
m, n & p things respectively is ,
m!n! p!

(3n)!
If m = n = p and the groups have identical qualitative characteristic then the number of groups = .
n! n! n! 3!

(3n)!
n!3
However, if 3n things are to be divided equally among three people then the number of ways = .

Example # 14
12 different toys are to be distributed to three children equally. In how many ways this can be done.
Solution.
The problem is to divide 12 different things into three different groups.
12 !
Number of ways = = 34650.
4! 4! 4!

Example # 15
In how many ways 10 persons can be divided into 5 pairs.
Solution.
We have each group having 2 persons and the qualitative characteristic are same (Since there is no
purpose mentioned or names for each pair).
10 !
Thus the number of ways = = 945.
( 2 ! )5 5 !

Self Practice Problems :

16. 9 persons enter a lift from ground floor of a building which stops in 10 floors (excluding ground floor). If
is known that persons will leave the lift in groups of 2, 3, & 4 in different floors. In how many ways this
can happen.
Ans. 907200

17. In how many ways one can make four equal heaps using a pack of 52 playing cards.
52 !
Ans.
(13 ! ) 4 4 !

18. In how many ways 11 different books can be parcelled into four packets so that three of the packets
contain 3 books each and one of 2 books, if all packets have the same destination.
11 !
Ans.
(3 ! ) 4 2

7. Selection of one or more objects


(a) Number of ways in which atleast one object be selected out of 'n' distinct objects is
n
C1 + nC2 + nC3 +...............+ nCn = 2n – 1
(b) Number of ways in which atleast one object may be selected out of 'p' alike objects of one type
'q' alike objects of second type and 'r' alike of third type is
(p + 1) (q + 1) (r + 1) – 1
(c) Number of ways in which atleast one object may be selected from 'n' objects where 'p' alike of
one type 'q' alike of second type and 'r' alike of third type and rest
n – (p + q + r) are different, is
(p + 1) (q + 1) (r + 1) 2n – (p + q + r) – 1

RESONANCE 6
Example # 16
There are 12 different books on a shelf. In how many ways we can select atleast one of them.
Solution.
We may select 1 book, 2 books,........, 12 books.
 The number of ways = 12C1 + 12C2 + ....... + 12C12 = 212 – 1. = 4095

Example # 17
There are 12 fruits in a basket of which 5 are apples, 4 mangoes and 3 bananas (fruits of same species
are identical). How many ways are there to select atleast one fruit.
Solution.
Let x be the number of apples being selected
y be the number of mangoes being selected and
z be the number of bananas being selected.
Then x = 0, 1, 2, 3, 4, 5
y = 0, 1, 2, 3, 4
z = 0, 1, 2, 3
Total number of triplets (x, y, z) is 6 × 5 × 4 = 120
Exclude (0, 0, 0)
 Number of combinations = 120 – 1 = 119.

Self Practice Problems

19. In a shelf there are 5 physics, 4 chemistry and 3 mathematics books. How many combinations are
there if (i) books of same subject are different (ii) books of same subject are identical.
Ans. (i) 4095 (ii) 119

20. From 5 apples, 4 mangoes & 3 bananas in how many ways we can select atleast two fruits of each
variety if (i) fruits of same species are identical (ii) fruits of same species are different.
Ans. (i) 24 (ii) 1144

8. Multinomial Theorem:
Coefficient of x r in expansion of (1  x)n = n+r1Cr (n  N)
Number of ways in which it is possible to make a selection from m + n + p = N things, where p are alike
of one kind, m alike of second kind & n alike of third kind taken r at a time is given by coefficient of
x r in the expansion of
(1 + x + x2 +...... + xp) (1 + x + x2 +...... + xm) (1 + x + x2 +...... + x n).

(i) For example the number of ways in which a selection of four letters can be made from the
letters of the word PROPORTION is given by coefficient of x4 in
(1 + x + x2 + x3) (1 + x + x 2) (1 + x + x 2) (1 + x) (1 + x) (1 + x).

(ii) Method of fictious partition :


Number of ways in which n identical things may be distributed among p persons if each person
may receive none, one or more things is; n+p1Cn.

Example # 18
Find the number of solutions of the equation x + y + z = 6, where x, y, z  W.
Solution.
Number of solutions = coefficient of x6 in (1 + x + x2 + ....... x 6)3
= coefficient of x 6 in (1 – x7)3 (1 – x) –3
= coefficient of x 6 in (1 – x) –3

 3  6  1
=   = 8C6 = 28.
 6 

RESONANCE 7
Example # 19
In a bakery four types of biscuits are available. In how many ways a person can buy 10 biscuits if he
decide to take atleast one biscuit of each variety.
Solution.
Let x be the number of biscuits the person select from first variety, y from the second, z from the third
and w from the fourth variety. Then the number of ways = number of solutions of the equation
x + y + z + w = 10.
where x = 1, 2, .........,7
y = 1, 2, .........,7
z = 1, 2, .........,7
w = 1, 2, .........,7
This is equal to = coefficient of x 10 in (x + x 2 + ...... + x 7)4
= coefficient of x6 in (1 + x + ....... + x6)4
= coefficient of x 6 in (1 – x7)4 (1 – x) –4
= coefficient x 6 in (1 – x)–4

 4  6  1
=   = 84.
 6 

Self Practice Problems:

21. Three distinguishable dice are rolled. In how many ways we can get a total 15.
Ans. 10.

22. In how many ways we can give 5 apples, 4 mangoes and 3 oranges (fruits of same species are similar)
to three persons if each may receive none, one or more.
Ans. 3150

9. Let N = pa. qb. rc...... where p, q, r...... are distinct primes & a, b, c..... are natural numbers then :
(a) The total numbers of divisors of N including 1 & N is = (a + 1) (b + 1) (c + 1)........
(b) The sum of these divisors is =
(p0 + p1 + p2 +.... + pa) (q0 + q1 + q2 +.... + qb) (r 0 + r 1 + r 2 +.... + r c)........

(c) Number of ways in which N can be resolved as a product of two factors is

1 (a  1)(b  1)(c  1).... if N is not a perfect square


(a  1)(b  1)(c  1)....1
= 2
1
2
if N is a perfect square

(d) Number of ways in which a composite number N can be resolved into two factors which are
relatively prime (or coprime) to each other is equal to 2n1 where n is the number of different
prime factors in N.

Example # 20
Find the number of divisors of 1350. Also find the sum of all divisors.
Solution.
1350 = 2 × 33 × 52
 Number of divisors = (1+ 1) (3 + 1) (2 + 1) = 24
sum of divisors = (1 + 2) (1 + 3 + 32 + 33) (1 + 5 + 52) = 3720.

Example # 21
In how many ways 8100 can be resolved into product of two factors.
Solution.
8100 = 22 × 34 × 52
1
Number of ways = ((2 + 1) (4 + 1) (2 + 1) + 1) = 23
2

RESONANCE 8
Self Practice Problems :

23. How many divisors of 9000 are even but not divisible by 4. Also find the sum of all such divisors.
Ans. 12, 4056.

24. In how many ways the number 8100 can be written as product of two coprime factors.
Ans. 4

10. Let there be 'n' types of objects, with each type containing atleast r objects. Then the number of ways
of arranging r objects in a row is nr.

Example # 22
How many 3 digit numbers can be formed by using the digits 0, 1, 2, 3, 4, 5. In how many of these we
have atleast one digit repeated.
Solution.
We have to fill three places using 6 objects (repeatation allowed), 0 cannot be at 100th place. The

number of numbers = 180.

Number of numbers in which no digit is repeated = 100


 Number of numbers in which atleast one digit is repeated = 180 – 100 = 80

Example # 23
How many functions can be defined from a set A containing 5 elements to a set B having 3 elements.
How many these are surjective functions.
Solution.
Image of each element of A can be taken in 3 ways.
 Number of functions from A to B = 35 = 243.
Number of into functions from A to B = 25 + 25 + 25 – 3 = 93.
 Number of onto functions = 150.

Self Practice Problems :

25. Find the sum of all three digit numbers those can be formed by using the digits. 0, 1, 2, 3, 4.
Ans. 27200.

26. How many functions can be defined from a set A containing 4 elements to a set B containing 5 elements.
How many of these are injective functions.
Ans. 625, 120

27. In how many ways 5 persons can enter into a auditorium having 4 entries.
Ans. 1024.

1 1 . Dearrangement :
Number of ways in which 'n' letters can be put in 'n' corresponding envelopes such that no letter goes
to correct envelope is

 1 1 1 1 n 1
n ! 1     .......... ..  ( 1) 
 1! 2! 3 ! 4 ! n! 

RESONANCE 9
Example # 24
In how many ways we can put 5 writings into 5 corresponding envelopes so that no writing go to the
corresponding envelope.
Solution.
The problem is the number of dearragements of 5 digits.

 1 1 1 1
This is equal to 5!      = 44.
 2 ! 3 ! 4 ! 5! 

Example # 25
Four slip of papers with the numbers 1, 2, 3, 4 written on them are put in a box. They are drawn one by
one (without replacement) at random. In how many ways it can happen that the ordinal number of
atleast one slip coincide with its own number.
Solution.
Total number of ways = 4 ! = 24.
The number of ways in which ordinal number of any slip does not coincide with its own number is the

 1 1 1
number of dearrangements of 4 objects = 4 !  2 !  3 !  4 !  = 9
 
Thus the required number of ways. = 24 – 9 = 15

Self Practice Problems:

28. In a match column question, Column  contain 10 questions and Column II contain 10 answers written
in some arbitrary order. In how many ways a student can answer this question so that exactly 6 of his
matchings are correct.
Ans. 1890

29. In how many ways we can put 5 letters into 5 corresponding envelopes so that atleast one letter go to
wrong envelope.
Ans. 119

RESONANCE 10
Part : (A) Only one correct option
1. There are 2 identical white balls, 3 identical red balls and 4 green balls of different shades. The number
of ways in which they can be arranged in a row so that atleast one ball is separated from the balls of the
same colour, is:
(A) 6 (7 !  4 !) (B) 7 (6 !  4 !) (C) 8 !  5 ! (D) none
2. The number of permutations that can be formed by arranging all the letters of the word ‘NINETEEN’ in
which no two E’s occur together is
8! 5! 5! 8!
(A) 3! 3! (B) (C) 3 ! × 6C3 (D) 5 ! × 6C3.
3!  6 C 2
3. The number of ways in which n different things can be given to r persons when there is no restriction as
to the number of things each may receive is:
(A) nCr (B) nPr (C) nr (D) rn
4. The number of divisors of apbqc rds where a, b, c, d are primes & p, q, r, s  N, excluding 1 and the
number itself is:
(A) p q r s (B) (p + 1) (q + 1) (r + 1) (s + 1)  4
(C) p q r s  2 (D) (p + 1) (q + 1) (r + 1) (s + 1)  2
5. The number of ordered triplets of positive integers which are solutions of the equation x + y + z = 100
is:
(A) 3125 (B) 5081 (C) 6005 (D) 4851
6. Number of ways in which 7 people can occupy six seats, 3 seats on each side in a first class railway
compartment if two specified persons are to be always included and occupy adjacent seats on the
same side, is (k). 5 ! then k has the value equal to:
(A) 2 (B) 4 (C) 8 (D) none
7. Number of different words that can be formed using all the letters of the word "DEEPMALA" if two
vowels are together and the other two are also together but separated from the first two is:
(A) 960 (B) 1200 (C) 2160 (D) 1440
8. Six persons A, B, C, D, E and F are to be seated at a circular table. The number of ways this can be
done if A must have either B or C on his right and B must have either C or D on his right is:
(A) 36 (B) 12 (C) 24 (D) 18
9. The number of ways in which 15 apples & 10 oranges can be distributed among three persons, each
receiving none, one or more is:
(A) 5670 (B) 7200 (C) 8976 (D) none of these
10. The number of permutations which can be formed out of the letters of the word "SERIES" taking three
letters together is:
(A) 120 (B) 60 (C) 42 (D) none
11. Seven different coins are to be divided amongst three persons. If no two of the persons receive the
same number of coins but each receives atleast one coin & none is left over, then the number of ways
in which the division may be made is:
(A) 420 (B) 630 (C) 710 (D) none
12. The streets of a city are arranged like the lines of a chess board. There are m streets running North to
South & 'n' streets running East to West. The number of ways in which a man can travel from NW to SE
corner going the shortest possible distance is:
( m  n) ! ( m  n  2) !
(A) m2  n 2 (B) (m  1)2 . (n  1)2 (C) (D)
m! . n! ( m  1) ! . ( n  1) !
13. In a conference 10 speakers are present. If S1 wants to speak before S2 & S2 wants to speak after
S3, then the number of ways all the 10 speakers can give their speeches with the above restriction if the
remaining seven speakers have no objection to speak at any number is:

(A) 10C3 (B) 10P8 (C) 10P3 (D)


10 !
3

RESONANCE 11
14. Two variants of a test paper are distributed among 12 students. Number of ways of seating of the
students in two rows so that the students sitting side by side do not have identical papers & those
sitting in the same column have the same paper is:
12! (12)!
(A) (B) (C) (6 !) 2. 2 (D) 12 ! × 2
6! 6! 2 5 . 6!
15. Sum of all the numbers that can be formed using all the digits 2, 3, 3, 4, 4, 4 is:
(A) 22222200 (B) 11111100 (C) 55555500 (D) 20333280
16. There are m apples and n oranges to be placed in a line such that the two extreme fruits being both
oranges. Let P denotes the number of arrangements if the fruits of the same species are different and
Q the corresponding figure when the fruits of the same species are alike, then the ratio P/Q has the
value equal to:
(A) nP2. mPm. (n  2) ! (B) mP2. nPn. (n  2) ! (C) nP2. nPn. (m  2) ! (D) none
17. The number of integers which lie between 1 and 106 and which have the sum of the digits equal to 12 is:
(A) 8550 (B) 5382 (C) 6062 (D) 8055
18. Number of ways in which a pack of 52 playing cards be distributed equally among four players so that
each may have the Ace, King, Queen and Jack of the same suit is:
36 ! 36 ! . 4 ! 36 !
9 ! 9 ! 9 !4
(A) (B) (C) (D) none
. 4!
4 4

19. A five letter word is to be formed such that the letters appearing in the odd numbered positions are
taken from the letters which appear without repetition in the word "MATHEMATICS". Further the letters
appearing in the even numbered positions are taken from the letters which appear with repetition in the
same word "MATHEMATICS". The number of ways in which the five letter word can be formed is:
(A) 720 (B) 540 (C) 360 (D) none
20. Number of ways of selecting 5 coins from coins three each of Rs. 1, Rs. 2 and Rs. 5 if coins of the
same denomination are alike, is:
(A) 9 (B) 12 (C) 21 (D) none
21. Number of ways in which all the letters of the word " ALASKA " can be arranged in a circle distinguishing
between the clockwise and anticlockwise arrangement, is:
(A) 60 (B) 40 (C) 20 (D) none of these
22. If r, s, t are prime numbers and p, q are the positive integers such that the LCM of p, q is r2 t4s2, then the
number of ordered pair (p, q) is [IIT – 2006]
(A) 252 (B) 254 (C) 225 (D) 224
Part : (B) May have more than one options correct
23. n+1
C6 + nC4 > n + 2C5  nC5 for all ' n ' greater than:
(A) 8 (B) 9 (C) 10 (D) 11
24. In an examination, a candidate is required to pass in all the four subjects he is studying. The number
of ways in which he can fail is
(A) 4P1 + 4P2 + 4P3 + 4P4 (B) 44 – 1
(C) 24 – 1 (D) 4C1 + 4C2 + 4C3 + 4C4
25. The kindergarten teacher has 25 kids in her class. She takes 5 of them at a time, to zoological garden
as often as she can, without taking the same 5 kids more than once. Then the number of visits, the
teacher makes to the garden exceeds that of a kid by:
(A) 25C5  24C4 (B) 24C5 (C) 25C5  24C5 (D) 24C4
26. The number of ways of arranging the letters AAAAA, BBB, CCC, D, EE & F in a row if the letter C are
separated from one another is:
12 ! 13 ! 14 ! 13 !
(A) 13C3. 5 ! 3 ! 2! (B) 5 ! 3 ! 3 ! 2 ! (C) 3 ! 3 ! 2 ! (D) 11. 6 !

27. There are 10 points P1, P2,...., P10 in a plane, no three of which are collinear. Number of straight lines
which can be determined by these points which do not pass through the points P1 or P2 is:
(A) 10C2  2. 9C1 (B) 27 (C) 8C2 (D) 10C2  2. 9C1 + 1

RESONANCE 12
28. Number of quadrilaterals which can be constructed by joining the vertices of a convex polygon of
20 sides if none of the side of the polygon is also the side of the quadrilateral is:
15
C 3 . 20
(A) 17C4  15C2 (B) (C) 2275 (D) 2125
4
29. You are given 8 balls of different colour (black, white,...). The number of ways in which these balls can
be arranged in a row so that the two balls of particular colour (say red & white) may never come
together is:
(A) 8 !  2.7 ! (B) 6. 7 ! (C) 2. 6 !. 7C2 (D) none

30. A man is dealt a poker hand (consisting of 5 cards) from an ordinary pack of 52 playing cards. The
number of ways in which he can be dealt a "straight" (a straight is five consecutive values not of the
same suit, eg. {Ace, 2, 3, 4, 5}, {2, 3, 4, 5, 6}.......................... & {10, J, Q, K, Ace}) is
(A) 10 (45  4) (B) 4 ! . 210 (C) 10. 210 (D) 10200

31. Number of ways in which 3 numbers in A.P. can be selected from 1, 2, 3,...... n is:
 n  1 n  n  2
2

(A)   if n is even (B) if n is odd


 2  4
 n 12 n  n  2
(C) if n is odd (D) if n is even
4 4
32. Consider the expansion, (a1 + a2 + a3 +....... + ap)n where n  N and n  p. The correct statement(s) is/
are:
(A) number of different terms in the expansion is , n + p  1C n
(B) co-efficient of any term in which none of the variables a1, a2 ..., ap occur more than once is ' n '
(C) co-efficient of any term in which none of the variables a1, a2,..., ap occur more than once is n ! if
n=p
 p
(D) Number of terms in which none of the variables a1, a2,......, ap occur more than once is   .
 n

1. In a telegraph communication how many words can be communicated by using atmost 5 symbols.
(only dot and dash are used as symbols)

2. If all the letters of the word 'AGAIN' are arranged in all possible ways & put in dictionary order, what is
the 50th word.

3. A committee of 6 is to be chosen from 10 persons with the condition that if a particular person 'A' is
chosen, then another particular person B must be chosen.

4. A family consists of a grandfather, m sons and daughters and 2n grand children. They are to be seated
in a row for dinner. The grand children wish to occupy the n seats at each end and the grandfather
refuses to have a grand children on either side of him. In how many ways can the family be made to sit?

5. The sides AB, BC & CA of a triangle ABC have 3, 4 & 5 interior points respectively on them. Find the
number of triangles that can be constructed using these interior points as vertices.

6. How many five digits numbers divisible by 3 can be formed using the digits 0, 1, 2, 3, 4, 7 and 8 if, each
digit is to be used atmost one.

7. In how many other ways can the letters of the word MULTIPLE be arranged ; (i) without changing the
order of the vowels (ii) keeping the position of each vowel fixed (iii) without changing the relative order/
position of vowels & consonants.

8. There are p intermediate stations on a railway line from one terminus to another. In how many ways can
a train stop at 3 of these intermediate stations if no 2 of these stopping stations are to be consecutive?

RESONANCE 13
9. Find the number of positive integral solutions of x + y + z + w = 20 under the following conditions:
(i) Zero values of x, y, z, w are include
(ii) Zero values are excluded
(iii) No variable may exceed 10; Zero values excluded
(iv) Each variable is an odd number
(v) x, y, z, w have different values (zero excluded).
10. Find the number of words each consisting of 3 consonants & 3 vowels that can be formed from the
letters of the word “CIRCUMFERENCE”. In how many of these C’s will be together.
11. If ' n ' distinct things are arranged in a circle, show that the number of ways of selecting three of these
1
things so that no two of them are next to each other is, n (n  4) (n  5).
6
12. In maths paper there is a question on "Match the column" in which column A contains 6 entries & each
entry of column A corresponds to exactly one of the 6 entries given in column B written randomly.
2 marks are awarded for each correct matching & 1 mark is deducted from each incorrect matching.
A student having no subjective knowledge decides to match all the 6 entries randomly. Find the number
of ways in which he can answer, to get atleast 25 % marks in this question.
13. Show that the number of combinations of n letters together out of 3n letters of which n are a and n are
b and the rest unlike is, (n + 2). 2n  1.
14. Find the number of positive integral solutions of, (i) x 2  y2 = 352706 (ii) xyz = 21600
15. There are ' n ' straight line in a plane, no two of which are parallel and no three pass through the same
point. Their points of intersection are joined. Show that the number of fresh lines thus introduced is,
1
8 n (n  1) (n  2) (n  3).
16. A forecast is to be made of the results of five cricket matches, each of which can be a win or a draw or
a loss for Indian team. Find
(i) number of forecasts with exactly 1 error
(ii) number of forecasts with exactly 3 errors
(iii) number of forecasts with all five errors

n !
2
17. Prove by permutation or otherwise
n !n is an integer (n  I+). [IIT – 2004]

 n  1
18. If total number of runs scored in n matches is   (2n+1 – n – 2) where n > 1, and the rund scored
 4 
in the k th match are given by k. 2n+1–k, where 1  k  n. Find n [IIT – 2005]

EXERCISE # 1 EXERCISE # 2
1. A 2. C 3. D 4. D 5. D 6. C 7. D 1. 62 2. NAAIG 3. 154

8. D 9. C 10. C 11. B 12. D 13. D 14. D 4. (2n)! m! (m  1) 5. 205 6. 744

15. A 16. A 17. C 18. B 19. B 20. B 21. C 7. (i) 3359 (ii) 59 (iii) 359

22. C 23. BCD 24. CD 25. AB 26. AD 8. p–2


C3

27. CD 28. AB 29. ABC 30. AD 31. CD 32. ACD 9. (i) 23C3 (ii) 19C3 (iii) 19C3 – 4.9C3 (iv) 11C8 (v) 552

10. 22100, 52 12. 56 ways

14. (i) Zero (ii) 1260 16. (i) 10 (ii) 80 (iii) 32

18. 7

RESONANCE 14
Probability

There are various phenomena in nature, leading to an outcome,


which cannot be predicted apriori e.g. in tossing of a coin,
a head or a tail may result. Probability theory aims at measuring
As long as algebra the uncertainties of such outcomes.

(I) Important terminology:


is taught in school,
(i) Random Experiment :
there will be prayer It is a process which results in an outcome which is one of the
various possible outcomes that are known to us before hand e.g.
inschool. throwing of a die is a random experiment as it leads to fall of one
of the outcome from {1, 2, 3, 4, 5, 6}. Similarly taking a card from
Coki eRobert s
a pack of 52 cards is also a random experiment.

Probabilityisexpec-
(ii) Sample Space :

It is the set of all possible outcomes of a random experiment e.g.


{H, T} is the sample space associated with tossing of a coin.
tation founded upon In set notation it can be interpreted as the universal set.

partial knowledge. A
perfect acquaintance Solved Example # 1
Write the sample space of the experiment ‘A coin is tossed and
with all the circum- a die is thrown’.
Solution
stances affecting the The sample space S = {H1, H2, H3, H4, H5, H6, T1, T2, T3, T4,
T5, T6}.
occurrence of an
Solved Example # 2
event would change Write the sample space of the experiment ‘A coin is tossed, if it
shows head a coin tossed again else a die is thrown.
expectation into
Solution
certainty, and leave The sample space S = {HH, HT, T1, T2, T3, T4, T5, T6}

nether room nor Solved Example # 3


Find the sample space associated with the experiment of rolling a
demand for a theory pair of dice (plural of die) once. Also find the number of elements of
the sample space.
ofprobabilities. Sol. Let one die be blue and the other be grey. Suppose ‘1’ appears on
blue die and ‘2’ appears on grey die. We denote this outcome by an
ordered pair (1, 2). Similarly, if ‘3’ appears on blue die and ‘5’ appears
George Bool e on grey die, we denote this outcome by (3, 5) and so on. Thus, each
outcome can be denoted by an ordered pair (x, y), where x is the
number appeared on the first die (blue die) and y appeared on the

RESONANCE 15
second die (grey die). Thus, the sample space is given by
S = {(x, y) x is the number on blue die and y is the number on grey die}
We now list all the possible outcomes (figure)

1 2 3 4 5 6
1 (1, 1) (1, 2) (1, 3) (1, 4) (1, 5) (1, 6)
2 (2, 1) (2, 2) (2, 3) (2, 4) (2, 5) (2, 6)
3 (3, 1) (3, 2) (3, 3) (3, 4) (3, 5) (3, 6)
4 (4, 1) (4, 2) (4, 3) (4, 4) (4, 5) (4, 6)
5 (5, 1) (5, 2) (5, 3) (5, 4) (5, 5) (5, 6)
6 (6, 1) (6, 2) (6, 3) (6, 4) (6, 5) (6, 6)

Figure

Number of elements (outcomes) of the above sample space is 6 × 6 i.e., 36

Self Practice Problems :


1. A coin is tossed twice, if the second throw results in head, a die is thrown.
Answer {HT, TT, HH1, HH2, HH3, HH4, HH5, HH6, TH1, TH2, TH3, TH4, TH5, TH6}.
2. An urn contains 3 red balls and 2 blue balls. Write sample space of the experiment ‘Selection of a
ball from the urn at random’.
Answer {R1, R2, R3, B1, B2 }.
Note : -
Here the balls are distinguished from one and other by naming red balls as R1, R2 and R3 and the blue
balls as B1 and B2.
(iii) Event :
It is subset of sample space. e.g. getting a head in tossing a coin or getting a prime number is
throwing a die. In general if a sample space consists ‘n’ elements, then a maximum of 2n events
can be associated with it.

(iv) Complement of event :


The complement of an event ‘A’ with respect to a sample space S is the set of all elements of ‘S’ which
are not in A. It is usually denoted by A, A or AC.

(v) Simple Event :


If an event covers only one point of sample space, then it is called a simple event e.g. getting a head
followed by a tail in throwing of a coin 2 times is a simple event.

(vi) Compound Event :


When two or more than two events occur simultaneously, the event is said to be a compound event.
Symbolically A  B or AB represent the occurrence of both A & B simultaneously.

Note : “A B” or A + B represent the occurrence of either A or B.

RESONANCE 16
Solved Example # 4
Write down all the events of the experiment ‘tossing of a coin’.
Solution
S = {H, T}
the events are  , {H}, {T}, {H, T}

Solved Example # 5
A die is thrown. Let A be the event ‘ an odd number turns up’ and B be the event ‘a number divisible
by 3 turns up’. Write the events (a) A or B (b) A and B
Solution
A = {1, 3, 5}, B = {3, 6}
 A or B = A  B = {1, 3, 5, 6}
A and B = A  B = {3}

Self Practice Problems :

3. A coin is tossed and a die is thrown. Let A be the event ‘H turns up on the coin and odd number turns
up on the die’ and B be the event ‘ T turns up on the coin and an even number turns up on the die’.
Write the events (a) A or B (b) A and B.
Answer (a) {H1, H3, H5, T2, T4, T6} (b) 

4. In tossing of two coins, let A = {HH, HT} and B = {HT, TT}. Then write the events (a) A or B
(b) A and B.
Answer (a) {HH, HT, TT} (b) {HT}

(vii) Equally likely Events :


If events have same chance of occurrence, then they are said to be equally likely.
e.g
(i) In a single toss of a fair coin, the events {H} and {T} are equally likely.
(ii) In a single throw of an unbiased die the events {1}, {2}, {3} and {4}, are equally likely.
(iii) In tossing a biased coin the events {H} and {T} are not equally likely.

(viii) Mutually Exclusive / Disjoint / Incompatible Events :

Two events are said to be mutually exclusive if occurrence of one of them rejects the possibility of
occurrence of the other i.e. both cannot occur simultaneously.
In the vein diagram the events A and B are mutually exclusive. Mathematically, we write
A  B = 

Solved Example # 6
In a single toss of a coin find whether the events {H}, {T} are mutually exclusive or not.
Solution
Since {H}  {T} = ,
 the events are mutually exclusive.

RESONANCE 17
Solved Example # 
In a single throw of a die, find whether the events {1, 2}, {2, 3} are mutually exclusive or not.
Solution
Since {1, 2}  {2, 3} = {2}  
 the events are not mutually exclusive.

Self Practice Problems :

5. In throwing of a die write whether the events ‘Coming up of an odd number’ and ‘Coming up of an even
number’ are mutually exclusive or not.
Answer Yes

6. An experiment involves rolling a pair of dice and recording the numbers that come up. Describe the following
events :
A : the sum is greater than 8.
B : 2 occurs on either die.
C : the sum is at least 7 and a multiple of 3.
Also, find A  B, B  C and A  C.
Are (i) A and B mutually exclusive ?
(ii) B and C mutually exclusive ?
(iii) A and C mutually exclusive ?
Ans. A = {(3, 6), (4, 5), (5, 4), (6, 3), (4, 6), (5, 5), (6, 4), (5, 6), (6, 5), (6, 6)}
B = {(1, 2), (2, 2), (3, 2), (4, 2), (5, 2), (6, 2), (2, 1), (2, 3), (2, 4). (2, 5), (2, 6)}
C = {(3, 6), (6, 3), (5, 4), (4, 5), (6, 6)}
A  B = , B  C = , A  C = {(3, 6), (6, 3), (5, 4), (4, 5), (6, 6)}
(i) Yes (ii) Yes (iii) No.

(ix) Exhaustive System of Events :


If each outcome of an experiment is associated with at least one of the events E1, E2, E3, .........En,
then collectively the events are said to be exhaustive. Mathematically we write
E1 E2 E3.........En = S. (Sample space)

Solved Example # 8
In throwing of a die, let A be the event ‘even number turns up’, B be the event ‘an odd prime turns
up’ and C be the event ‘a numbers less than 4 turns up’. Find whether the events A, B and C form
an exhaustive system or not.
Solution
A  {2, 4, 6}, B  {3, 5} and C  {1, 2, 3}.
Clearly A B C = {1, 2, 3, 4, 5, 6} = S. Hence the system of events is exhaustive.

Solved Example # 9
Three coins are tossed. Describe
(i) two events A and B which are mutually exclusive
(ii) three events A, B and C which are mutually exclusive and exhaustive.
(iii) two events A and B which are not mutually exclusive.
(iv) two events A and B which are mutually exclusive but not exhaustive.
(v) three events A, B and C which are mutually exclusive but not exhaustive.
Ans. (i) A : “getting at least two heads” B : “getting at least two tails”
(ii) A : “getting at most one heads” B : “getting exactly two heads”
C : “getting exactly three heads”
(iii) A : “getting at most two tails” B : “getting exactly two heads”
(iv) A : “getting exactly one head” B : “getting exactly two heads”
(v) A : “ getting exactly one tail” B : “getting exactly two tails”
C : “getting exactly three tails”
[Note : There may be other cases also]

RESONANCE 18
Self Practice Problems :

7. In throwing of a die which of the following pair of events are mutually exclusive ?
(a) the events ‘coming up of an odd number’ and ‘coming up of an even number’
(b) the events ‘coming up of an odd number’ and ‘coming up of a number  4’
Answer (a)

8. In throwing of a die which of the following system of events are exhaustive ?


(a) the events ‘an odd number turns up’, ‘a number  4 turns up’ and ‘the number 5 turns up’.
(b) the events ‘a number  4 turns up’, ‘a number > 4 turns up’.
(c) the events ‘an even number turns up’, ‘a number divisible by 3 turns up’, ‘number 1 or 2
turns up’ and ‘the number 6 turns up’.
Answer (b)

(I I ) Classical (A priori) Definition of Probability :

If an experiment results in a total of (m + n) outcomes which are equally likely and mutually exclusive
with one another and if ‘m ’ outcom es are f avorable to an even t ‘A’ while ‘n’ ar e
unfa vorable, then the pr obability of occ urrenc e of t he event ‘A’, denot ed by

m number of favourable outcomes


P(A), is defined by =
mn total number of outcomes

m
i.e. P(A) = .
mn
We say that odds in favour of ‘A’ are m : n, while odds against ‘A’ are n : m.

n
Note that P( A ) or P(A) or P(AC), i.e. probability of non-occurrence of A = = 1 – P(A)
mn
In the above we shall denote the number of out comes favourable to the event A by n(A) and the total
number of out comes in the sample space S by n(S).

n( A )
 P(A) = .
n(S)
Solved Example # 10
In throwing of a fair die find the probability of the event ‘ a number  4 turns up’.
Solution
Sample space S = {1, 2, 3, 4, 5, 6} ; event A = {1, 2, 3, 4}
 n(A) = 4 and n(S) = 6

n( A ) 4 2
 P(A) = = = .
n(S) 6 3
Solved Example # 11
In throwing of a fair die, find the probability of turning up of an odd number  4.
Solution
S = {1, 2, 3, 4, 5, 6}
Let E be the event ‘turning up of an odd number  4’
then E = {5}

n (E) 1
 P(E) = n (S) = .
6

RESONANCE 19
Solved Example # 12
In throwing a pair of fair dice, find the probability of getting a total of 8.
Solution.
When a pair of dice is thrown the sample space consists
{(1, 1) (1, 2) .......... (1, 6)
(2, 1,) (2, 2,)......... (2, 6)
.... ..... .... ...
.... ... ... ...
(6, 1), (6, 2) ........ (6, 6)}
Note that (1, 2) and (2, 1) are considered as separate points to make each outcome as equally likely.
To get a total of ‘8’, favourable outcomes are, (2, 6) (3, 5) (4, 4) (5, 3) and (6, 2).
5
Hence required probability =
36

Solved Example # 13
A four digit number is formed using the digits 0, 1, 2, 3, 4 without repetition. Find the probability that it is
divisible by 4
Solution

Total 4 digit numbers formed

Each of these 96 numbers are equally likely & mutually exclusive of each other.
Now, A number is divisible by 4, if last two digits of the number is divisible by 4
Hence we can have  first two places can be filled in 3 × 2 = 6 ways

 first two places can be filled in 2 × 2 = 4 ways

 6 ways

 4 ways

 4 ways

 6 ways
__________
Total number of ways 30 ways
favorable outcomes 30 5
probability = Total outcomes = = Ans.
96 16

Self Practice Problems :

9. A bag contains 4 white, 3 red and 2 blue balls. A ball is drawn at random. Find the probability of the
event (a) the ball drawn is white or red (b) the ball drawn is white as well as red.
Answer (a) 7/9 (b) 0

10. In throwing a pair of fair dice find the probability of the events ‘ a total of of less than or equal to 9”’.
Answer 5/36.

RESONANCE 20
(III) Addition theorem of probability :

If ‘A’ and ‘B’ are any two events associated with an experiment, then

P(AB) = P(A) + P(B) – P(AB)

De Morgan’s Laws : If A & B are two subsets of a universal set U, then


(a) (A  B)c = Ac  Bc
(b) (A  B)c = Ac  Bc
Distributive Laws : (a) A  (B C) = (A B)  (A C)
(b) A  (B C) = (A B)  (A C)

For any three events A, B and C we have the figure

(i) P(A or B or C) = P(A) + P(B) + P(C) – P(A B) – P(B C) – P(C A) + P(A B C)
(ii) P (at least two of A, B, C occur) = P(B  C) + P(C  A) + P(A  B) – 2P(A B C)
(iii) P(exactly two of A, B, C occur) = P(B C) + P(C A) + P(A B) – 3P(A B  C)
(iv) P(exactly one of A, B, C occur) =
P(A) + P(B) + P(C) – 2P(B C) – 2P(C A) – 2P(A B) + 3P(A B C)

Note : If three events A, B and C are pair wise mutually exclusive then they must be mutually exclusive,
i.e. P(A  B) = P(B C) = P(C A) = 0  P(A B C) = 0.
However the converse of this is not true.

Solved Example # 14
A bag contains 4 white, 3red and 4 green balls. A ball is drawn at random. Find the probability of the
event ‘the ball drawn is white or green’.
Solution
Let A be the event ‘the ball drawn is white’ and B be the event ‘the ball drawn is green’.

8
P(The ball drawn is white or green) = P (A  B) = P(A) + P(B) – P(A  B) =
11

Solved Example # 15
In throwing of a die, let A be the event ‘an odd number turns up’, B be the event ‘a number divisible
by 3 turns up’ and C be the event ‘a number  4 turns up’. Then find the probability that exactly two
of A, B and C occur.
Solution
Event A = {1, 3, 5}, event B = {3, 6} and event C = {1, 2, 3, 4}
 A  B = {3}, B  C = {3}, A  C = {1, 3} and A  B  C = {3}.

RESONANCE 21
Thus P(exactly two of A, B and C occur)
= P(A  B) + P(B  C) + P(C  A) – 3P(A  B  C)

1 1 2 1 1
= + + – 3 × =
6 6 6 6 6

Self Practice Problems :


11. In throwing of a die, let A be the event ‘an odd number turns up’, B be the event ‘a number divisible
by 3 turns up’ and C be the event ‘a number  4 turns up’. Then find the probability that atleast two
of A, B and C occur.

1
Answer
3

12. In the problem number 11, find the probability that exactly one of A, B and C occurs.

2
Answer
3

(IV) Conditional Probability

P(A  B)
If A and B are two events, then P(A/B) = .
P(B)
Note that for mutually exclusive events P(A/B) = 0.

Solved Example # 16
If P(A/B) = 0.2 and P(B) = 0.5 and P(A) = 0.2. Find P(A  B ).
Solution.
P(A  B ) = P(A) – P(A  B)

P( A  B)
Also P(A/B) =
P(B)
 P(A  B) = 0.1
From given data,
P(A  B ) = 0.1

Solved Example # 17

 
If P(A) = 0.25, P(B) = 0.5 and P(A  B) = 0.14, find probability that neither ‘A’ nor ‘B’ occurs. Also find
P AB

 
Solution
We have to find P A  B = 1 – P(A  B) (by De-Morgan’s law)
Also, P(A B) = P(A) + P(B) – P(A  B)
putting data we get, P A  B  = 0.39

The shaded region denotes the simultaneous occurrence of A and B

 
Hence P A  B = P(A) – P(A  B) = 0.11

RESONANCE 22
Self Practice Problem:-

13. If P( A / B ) = 0.2, P(A  B) = 0.9, then find P(A  B ) ?


Ans. 0.4

5. Independent and dependent events


If two events are such that occurence or non-occurence of one does not affect the chances of occurence
or non-occurence of the other event, then the events are said to be independent. Mathematically : if
P(A  B) = P(A) P(B), then A and B are independent.

Note: (i) If A and B are independent, then (a) A and B are independent, (b) A and B are independent
and (c) A and B are independent.
(ii) If A and B are independent, then P(A / B) = P(A).

If events are not independent then they are said to be dependent.

Independency of three or more events


Three events A, B & C are independent if & only if all the following conditions hold :
P(A  B) = P(A) . P(B) ; P(B  C) = P(B) . P(C)
P (C  A) = P(C) . P(A) ; P(A  B  C) = P(A) . P(B) . P(C)
i.e. they must be independent in pairs as well as mutually independent.
Similarly for n events A1, A2, A3, ........ An to be independent, the number of these conditions is equal
to nC2 + nC3 + ....... + cCn = 2n – n – 1.

Solved Example # 18
In drawing two balls from a box containing 6 red and 4 white balls without replacement, which of the
following pairs is independent ?
(a) Red on first draw and red on second draw
(b) Red on first draw and white on second draw
Solution
Let E be the event ‘Red on first draw’, F be the event ‘Red on second draw’ and G be the event ‘white
on second draw’.

6 6 4
P(E) = , P(F) = , P(G) =
10 10 10

6
P2 1
(a) P(E  F) = =
10
P2 3

3 3 9 1
P(E) . P(F) = × = 
5 5 25 3
 E and F are not independent

6 4 6
(b) P(E) . P(G) = × =
10 10 25

6
P1  4 P1 4
P(E  G) = =
10
P2 15

 P(E) . P(G)  P(E  G)


 E and G are not independent

RESONANCE 23
Solved Example # 19
If two switches S1 and S2 have respectively 90% and 80% chances of working. Find the probabilities that
each of the following circuits will work.

Solution
Consider the following events :
A = Switch S1 works,
B = Switch S2 works,
We have,
90 9 80 8
P(A) = = and P(B) = =
100 10 100 10
(i) The circuit will work if the current flows in the circuit. This is possible only when both the switches work
together. Therefore,
Required probability
= P(A  B) = P(A) P (B) [ A and B are independent events]
9 8 72 18
= × = =
10 10 100 25
(ii) The circuit will work if the current flows in the circuit. This is possible only when at least one of the two
switches S1, S2 works. Therefore,
Required Probability
= P(A  B) = 1 – P ( A ) P( B ) [ A, Bare independent events]

 9   8 
= 1 – 1   1  
 10   10 

1 2 49
= 1– × =
10 10 50

Solved Example # 20
A speaks truth in 60% of the cases and b in 90% of the cases. In what percentage of cases are they likely
to contradict each other in stating the same fact?
Solution
Let E be the event that A speaks truth and F be the event that B speaks truth. Then E and F are independent
events such that
60 3 90 9
P(E) = = and P(F) = =
100 5 100 10
A and B will contradict each other in narrating the same fact in the following mutually exclusive ways :
(i) A speaks truth and B tells a lie i.e. E  F
(ii) A tells a lie and B speaks truth lie i.e. E  F

 P(A and B contradict each other)

= P(I or II) = (I  II)

RESONANCE 24
= P[(E  F )  ( E  F)]
= P(E  F ) + P ( E  F) [ E  F and E  F are mutually exclusive]
= P(E) P( F ) + P( E ) P(F) [ E and F are in dep.]

3  9   3 9 3 1 2 9 21
= × 1   + 1   × = × + × =
5  10   5  10 5 10 5 10 50

Solved Example # 21
An urn contains 7 red and 4 blue balls. Two balls are drawn at random with replacement. Find the probability
of getting
(i) 2 red balls (ii) 2 blue balls (iii) one red and one blue ball
49 16 56
Ans. (i) (ii) (iii)
121 121 121

Solved Example # 22
1 1
Probabilities of solving a specific problem independently by A and B are and respectively. If both try to
2 3
solve the problem independently, find the probability that
(i) the problem is solved (ii) exactly one of them solves the problem.
2 1
Ans. (i) (ii)
3 2

Solved Example # 23
A box contains 5 bulbs of which two are defective. Test is carried on bulbs one by one untill the two defective
bulbs are found out. Find the probability that the process stops after
(i) Second test (ii) Third test
Solution
(i) Process will stop after second test. Only if the first and second bulb are both found to be defective
2 1 1
probability = × = (Obviously the bulbs drawn are not kept back.)
5 4 10
(ii) Process will stop after third test when either
2 3 1 1
(a) DND  × × = Here ‘D’ stands for defective
5 4 3 10
3 2 1 1
or (b) NDD  × × = and ‘N’ is for not defective.
5 4 3 10
3 2 1 1
or (c) NNN  × × =
5 4 3 10

3
hence required probability =
10
Solved Example # 24

1 1  E1 
If E1 and E2 are two events such that P(E1) = ; P(E2) = ; P   = 1 , then choose the correct options.
4 2  E2  4
(i) E1 and E2 are independent (ii) E1 and E2 are exhaustive
(iii) E1 and E2 are mutually exclusive (iv) E1 & E2 are dependent
 E1   E2 
Also find P  E  and  

 2  E1 

RESONANCE 25
Solution
 E2 
Since  E  = P(E1)  E1 and E2 are independent of each other
 1
Also since P(E1  E2) = P(E1) + P(E2) – P(E1) . P(E2)  1
Hence events are not exhaustive. Independent events can’t be mutually exclusive.
Hence only (i) is correct
Further since E1 & E2 are independent; E1 and E2 or E1 , E2 are E1 , E2 are also independent.

E 
Hence P 1  = P E1 =
E 
3
4
  and
E 
P 2  = P (E2) =
1
2
 2  E1 

Solved Example # 25
If cards are drawn one by one from a well shuffled pack of 52 cards without replacement,
until an ace appears, find the probability that the fourth card is the first ace to appear.
Solution
48
C 3  4 C1
Probability of selecting 3 non-Ace and 1 Ace out of 52 cards is equal to 52
C4
Since we want 4th card to be first ace, we will also have to consider the arrangement, Now 4 cards in sample
space can be arranged in 4! ways and, favorable they can be arranged in 3 ! ways as we want 4th position to
be occupied by ace
48
C 3  4 C1 3!
Hence required probability = × 4!
52
C4
Aliter :
‘NNNA’ is the arrangement than we desire in taking out cards, one by one
48 47 46 4
Hence required chance is × × ×
52 51 50 49

Self Practice Problems :

14. In throwing a pair of dies find the probability of getting an odd number on the first die and a total of
7 on both the dies.

1
Answer
12

15. In throwing of a pair of dies, find the probability of getting a boublet or a total of 4.

2
Answer
9

16. A bag contains 8 marbles of which 3 are blue and 5 are red. One marble is drawn at random, its colour is
noted and the marble is replaced in the bag. A marble is again drawn from the bag and its colour is noted.
Find the probability that the marbles will be
(i) blue followed by red (ii) blue and red in any order
(iii) of the same colour.
15 15 17
Ans. (i) (ii) (iii)
64 32 32

RESONANCE 26
17. A coin is tossed thrice. In which of the following cases are the events E and F independent ?
(i) E : “the first throw results in head”.
F : “the last throw result in tail”.
(ii) E : “the number of heads is two”.
F : “the last throw result in head”.
(iii) E : “the number of heads is odd ”.
F : “the number of tails is odd”.
Ans. (i)

6. Binomial Probability Theorem

If an experiment is such that the probability of success or failure does not change with trials, then
the probability of getting exactly r success in n trials of an experiment is nCr pr qn – r, where ‘p’ is the
probability of a success and q is the probability of a failure. Note that p + q = 1.

Solved Example 26
A pair of dice is thrown 5 times. Find the probability of getting a doublet twice.
Solution

1
In a single throw of a pair of dice probability of getting a doublet is
6

1
con sidering it to be a success, p =
6

1 5
 q = 1 – =
6 6
number of success r = 2

 1 5
2 3
625
 P(r = 2) = 5C2 p2 q3 = 10 .   .   =
6 6 3888

Solved Example # 27
A pair of dice is thrown 4 times. If getting ‘a total of 9’ in a single throw is considered as a success
then find the probability of getting ‘a total of 9’ thrice.
Solution

4 1
p = probability of getting ‘a total of 9’ = =
36 9

1 8
 q = 1 – =
9 9
r = 3, n = 4

 1
3
8 32
 P(r = 3) = C3 p q = 4 ×  
4 3
. =
9 9 6561

Solved Example # 28
In an examination of 10 multiple choice questions (1 or more can be correct out of 4 options). A student
decides to mark the answers at random. Find the probability that he gets exactly two questions correct.
Solution
A student can mark 15 different answers to a MCQ with 4 option i.e. 4C1 + 4C2 + 4C3 + 4C4 = 15
1
Hence if he marks the answer at random, chance that his answer is correct = and being incorrecting
15

RESONANCE 27
14 1 14
. Thus p= ,q= .
15 15 15

 1   14 
2 8

P (2 success) = C2 ×   ×  
10
 
15  15 

Solved Example # 29
A family has three children. Event ‘A’ is that family has at most one boy, Event ‘B’ is that family has at least
one boy and one girl, Event ‘C’ is that the family has at most one girl. Find whether events ‘A’ and ‘B’ are
independent. Also find whether A, B, C are independent or not.
Solution
A family of three children can have
(i) All 3 boys (ii) 2 boys + 1 girl (iii) 1 boy + 2 girls (iv) 3 girls

 1
3
1
(i) P (3 boys) = C0   =
3
(Since each child is equally likely to be a boy or a girl)
 
2 8

 1
2
1 3
(ii) P (2 boys +1girl) = C1 ×   × = (Note that there are three cases BBG, BGB, GBB)
3
 
2 2 8

 1  1
1 2
3
(iii) P (1 boy + 2 girls) = C2 ×   ×   =
3
2 2 8

1
(iv) P (3 girls) =
8
1
Event ‘A’ is associated with (iii) & (iv). Hence P(A) =
2
3
Event ‘B’ is associated with (ii) & (iii). Hence P(B) =
4
1
Event ‘C’ is associated with (i) & (ii). Hence P(C) =
2
3
P(A B) = P(iii) = = P(A) . P(B) . Hence A and B are independent of each other
8
P(A C) = 0  P(A) . P(C) . Hence A, B, C are not independent

Self Practice Problems :

18. A box contains 2 red and 3 blue balls. Two balls are drawn successively without replacement. If getting
‘a red ball on first draw and a blue ball on second draw’ is considered a success, then find the probability
of 2 successes in 3 performances.
Answer .189

19. Probability that a bulb produced by a factory will fuse after an year of use is 0.2. Find the probability
that out of 5 such bulbs not more than 1 bulb will fuse after an year of use.

2304
Answer
3125

7. Expectation :
If a value Mi is associated with a probability of pi , then the expectation is given by  piMi.

RESONANCE 28
Solved Example # 30
There are 100 tickets in a raffle (Lottery). There is 1 prize each of Rs. 1000/-, Rs. 500/- and
Rs. 200/-. Remaining tickets are blank. Find the expected price of one such ticket.
Solution
Expectation =  piMi outcome of a ticket can be
pi Mi piMi

1
(i) I prize 1000 10
100
1
(ii) II prize 500 5
100
1
(iii) III prize 200 2
100
97
(iv) Blank 0 0
100
________________
 piMi = 17
________________
Hence expected price of one such ticket Rs. 17

Solved Example # 31
A purse contains four coins each of which is either a rupee or two rupees coin. Find the expected value of a
coin in that purse.
Solution
Various possibilities of coins in the purse can be
pi Mi piMi

1 4
(i) 4 1 rupee coins 4
16 16
4 20
(ii) 3 one Rs. + 1 two Rs. 5
16 16
6 36
(iii) 2 one Rs. + 2 two Rs. 6
16 16
4 28
(iv) 1 one Rs. + 3 two Rs. 7
16 16
1 8
(iv) 4 two Rs. 8
16 16
________________
6/-
________________
Note that since each coin is equally likely to be one Rs. or two Rs. coin, the probability is determined using
Binomial probability; unlike the case when the purse contained the coins with all possibility being equally
1
likely, where we take pi = for each.
5
Hence expected value is Rs. 6/-

Self Practice Problems :

20. From a bag containing 2 one rupee and 3 two rupee coins a person is allowed to draw 2 coins indiscrimi-
nately; find the value of his expectation.
Ans. Rs. 3.20

RESONANCE 29
8. Total Probability Theorem
If an event A can occur with one of the n mutually exclusive and exhaustive events B1, B2, ....., Bn
and the probabilities P(A/B1), P(A/B2) .... P(A/Bn) are known, then

 P(B ) . P( A / B )
n
P(A) = i i
i1

Solved Example # 32

Box -  contains 5 red and 4 white balls whilst box -  contains 4 red and 2 white balls. A fair die is
thrown. If it turns up a multiple of 3, a ball is drawn from box -  else a ball is drawn from box - .
Find the probability that the ball drawn is white.
Solution
Let A be the event ‘a multiple of 3 turns up on the die’ and R be the event ‘the ball drawn is white’
then P (ball drawn is white)
= P(A) . P(R / A) + P ( A ) P(R / A )

2 4  2 2 10
= × + 1   =
6 9  6 6 27

Solved Example # 33

Cards of an ordinary deck of playing cards are placed into two heaps. Heap - consists of all the red
cards and heap - consists of all the black cards. A heap is chosen at random and a card is drawn,
find the probability that the card drawn is a king.
Solution
Let  and be the events that heap -  and heap - are choosen respectively. Then

1
P() = P() =
2
Let K be the event ‘the card drawn is a king’

2 2
 P (K / ) = and P(K / ) =
26 26

1 2 1 2 1
 P(K) = P () P(K / ) + P() P(K / ) = × + × = .
2 26 2 26 13

Self Practice Problems :

21. Box -  contains 3 red and 2 blue balls whilest box - II contains 2 red and 3 blue balls. A fair coin is
toss ed. If it t urns u p head, a ball is dr awn f r om box - , else a ball is dr awn f r om
box -  . Find the probability that the ball drawn is red.

1
Answer
2

22. There are 5 brilliant students in class XI and 8 brilliant students in class XII. Each class has 50
students. The odds in favour of choosing the class XI are 2 : 3. If the class XI is not chosen then the
class XII is chosen. Find the probability of selecting a brilliant student.

17
Answer .
125

RESONANCE 30
9. Bayes’ Theorem :
If an event A can occur with one of the n mutually exclusive and exhaustive events B1, B2 , ....., Bn and
the probabilities P(A/B1), P(A/B2) .... P(A/Bn) are known, then

P(B i ) . P( A / Bi )

 P(B ) . P(A / B )
P(Bi / A) = n

i i
i 1

Proof :
The event A occurs with one of the n mutually exclusive and exhaustive events
B1, B 2, B 3,........,Bn
A = (A  B1)  (A  B2)  (A  B3)  ........  (A  Bn)

 P( A  B )
n

P(A) = P(A  B1) + P(A  B2) + ....... + P(A Bn) = i


i 1

Note: A  event what we have ; Bi = event what we want ;


Now,
P(A  Bi) = P(A) . P(Bi/A) = P(Bi) . P(A/Bi)

P(Bi ) . P( A / Bi ) P(Bi ) . P( A / B i )


P (Bi/A) = =
P( A ) n
P( A  Bi )
i 1

P(Bi ) . P( A / Bi )
 P(B ) . P(A / B )
P(Bi/A) =
i i

Solved Example # 34
2
Pal’s gardener is not dependable, the probability that he will forget to water the rose bush is . The rose
3
1
bush is in questionable condition any how, if watered the probability of its withering is , if not watered, the
2
3
probability of its withering is. Pal went out of station and upon returning, he finds that the rose bush has
4
withered, what is the probability that the gardener did not water the bush.
[Here result is known that the rose bush has withered, therefore. Bayes’s theorem should be used]
Solution
Let A = the event that the rose bush has withered
Let A1 = the event that the gardener did not water.
A2 = the event that the gardener watered.
By Bayes’s theorem required probability,
P( A 1 ) . P( A / A 1 )
P(A1/A) = P( A ) . P( A / A )  P( A ) . P( A / A ) .....(i)
1 1 2 2

2 1
Given, P(A1) =  P(A2) =
3 3

RESONANCE 31
3 1
P(A/A1) = , P(A/A2) =
4 2
2 3
.
3 4 6 3
From (1), P(A1/A) = 2 3 1 1 = =
.  . 62 4
3 4 3 2

Solved Example # 35
There are 5 brilliant students in class XI and 8 brilliant students in class XII. Each class has 50
students. The odds in favour of choosing the class XI are 2 : 3. If the class XI is not chosen then the
class XII is chosen. A student is a chosen and is found to be brilliant, find the probability that the
chosen student is from class XI.
Solution
Let E and F be the events ‘Class XI is chosen’ and ‘Class XII is chosen’ respectively.

2 3
Then P(E) = , P(F) =
5 5
Let A be the event ‘Student chosen is brilliant’.

5 8
Then P(A / E) = and P(A / F) = .
50 50

2 5 3 8 34
 P(A) = P(E) . P(A / E) + P(F) . P(A / F) = . + . = .
5 50 5 50 250

P(E) . P( A / E) 5
 P(E / A) = P(E ) . P( A / E)  P(F) . P( A / F) = .
17

Solved Example # 36
A pack of cards is counted with face downwards. It is found that one card is missing. One card is drawn and
is found to be red. Find the probability that the missing card is red.
Solution
Let A be the event of drawing a red card when one card is drawn out of 51 cards (excluding missing card.) Let
A1 be the event that the missing card is red and A2 be the event that the missing card is black.
Now by Bayes’s theorem, required probability,
P( A 1 ) . (P( A / A 1 )
P(A1/A) = P( A ) . P( A / A )  P( A ) . P( A / A ) ..........(i)
1 1 2 2
In a pack of 52 cards 26 are red and 26 are black.
26
C1 26 1
Now P(A1) = probability that the missing card is red = = =
52
C1 52 2

26 1
P(A2) = probability that the missing card is black = =
52 2
P(A/A1) = probability of drawing a red card when the missing card is red.
25
=
51
[ Total number of cards left is 51 out of which 25 are red and 26 are black as the missing card is red]
26
Again P(A/A2) = Probability of drawing a red card when the missing card is black =
51
Now from (i), required probability,

RESONANCE 32
1 25
.
2 51 25
P(A1/A) = 1 25 1 26 =
.  . 51
2 51 2 51

Solved Example # 37
A bag contains 6 white and an unknown number of black balls ( 3). Balls are drawn one by one with
replacement from this bag twice and is found to be white on both occassion. Find the probability that the bag
had exactly ‘3’ Black balls.
Solution
Apriori, we can think of the following possibilies
(i) E1 6W , 0B
(ii) E2 6W , 1B
(iii) E3 6W , 2B
(iv) E4 6W , 3B
1
Clearly P(E1) = P(E2) = P(E3) = P(E4) =
4
Let ‘A’ be the event that two balls drawn one by one with replacement are both white therefore we have to find
 E4 
P  
 A 

 A

P  P(E 4 )
 E4   E4

By Baye’s theorem P   =
 A  A  A   A   A 
P   P(E1 )  P  . P(E 2 )  P  . P(E3 )  P  . P(E 4 )
E
 1 E
 2 E
 3  E4 
 A  6 6  A  6 6  A  6 6
P  E  = × ; P  E  = × ; P  E  = × ;
 4 9 9  3 8 8  2 7 7

A 6 6
P  E  = × ;
 1 6 6

1
 E4  81
Putting values P   =
 A  1 1 1 1
  
81 64 49 36

Self Practice Problems :

23. Box- contains 3 red and 2 blue balls whilest box- contains 2 red and 3 blue balls. A fair coin is
tossed. If it turns up head, a ball is drawn from box-, else a ball is drawn from box-. If the ball drawn
is red, then find the probability that the ball is drawn from box-.

3
Answer
5

24. Cards of an ordinary deck of playing cards are placed into two heaps. Heap - consists of all the red
cards and heap - consists of all the black cards. A heap is chosen at random and a card is drawn,
if the card drawn is found to be a king, find the probability that the card drawn is from the heap - .

1
Answer
2

RESONANCE 33
10. Value of Testimony

If p1 and p2 are the probabilities of speaking the truth of two independent witnesses A and B then P(their

p1 p 2
combined statement is true) = .
p1 p 2  (1  p1 )(1  p 2 )

In this case it has been assumed that we have no knowledge of the event except the statement made
by A and B.
However if p is the probability of the happening of the event before their statement, then

p p1 p 2
P(their combined statement is true) = .
p p1 p 2  (1  p) (1  p1 )(1  p 2 )

Here it has been assumed that the statement given by all the independent witnesses can be given in
two ways only, so that if all the witnesses tell falsehoods they agree in telling the same falsehood.
If this is not the case and c is the chance of their coincidence testimony then the
Probability that the statement is true = P p1 p2
Probability that the statement is false = (1 – p). c (1 – p1) (1 – p2)
However chance of coincidence testimony is taken only if the joint statement is not contradicted by
any witness.

Solved Example # 38
A die is thrown, a man C gets a prize of Rs. 5 if the die turns up 1 and gets a prize of Rs. 3 if the

1
die turns up 2, else he gets nothing. A man A whose probability of speaking the truth is tells C that
2

2
the die has turned up 1 and another man B whose probability of speaking the truth is tells C that
3
the die has turned up 2. Find the expectation value of C.
Solution

1
Let A and B be the events ‘A speaks the truth’ and ‘B speaks the truth’ respectively. Then P(A) =
2

2
and P(B) = .
3
The experiment consists of three hypothesis
(i) the die turns up 1
(ii) the die turns up 2
(iii) the die turns up 3, 4, 5 or 6

1 4
Let these be the events E 1, E 2 and E 3 respectively then P(E 1) = P(E 2) = and P(E 3) = .
6 6
Let E be the event that the statements made by A and B agree to the same conclusion.

1 1 1
 P(E / E1) = P(A) . P ( B ) = × =
2 3 6

1 2 2
P(E / E2) = P( A ) . P(B) = × =
2 3 6

1 1 1
P(E / E3) = P( A ) . P ( B ) = × =
2 3 6
 P(E) = P(E1) P(E / E1) + P(E2) P(E / E2) + P(E3) P(E / E3)

RESONANCE 34
1 1 1 2 4 1 7
= . + . + . =
6 6 6 6 6 6 36

P(E1 )P(E / E1 ) 1
Thus P(E1 / E) = =
P(E) 7

P(E 2 )P(E / E 2 ) 2
P(E2 / E) = =
P(E) 7

P(E3 )P(E / E 3 ) 4
P(E3 / E) = =
P(E) 7

1 2 11
 expectation of C = ×5 + × 3 + 0 = Rs.
7 7 7

Solved Example #39


A speaks the truth ‘3 times out of 4’ and B speaks the truth ‘2 times out of 3’. A die is thrown. Both
assert that the number turned up is 2. Find the probability of the truth of their assertion.
Solution
Let A and B be the events ‘A speaks the truth’ and ‘B speaks the truth’ repectively. Let C be the event
‘the number turned up is not 2 but both agree to the same conclustion that the die has turned up 2’.

3 2 1 1
Then P(A) = , P(B) = and P(C) = ×
4 3 5 5
There are two hypotheses
(i) the die turns up 2
(ii) the die does not turns up 2
Let these be the events E1 and E2 respectively, then

1 5
P(E1) = , P(E2) = (a priori probabilities)
6 6
Now let E be the event ‘the statement made by A and B agree to the same conclusion.

3 2 1
then P(E / E1) = P(A) . P(B) = . =
4 3 2

1 1 1 1
P(E / E2) = P( A ) . P( B ) . P(C) = . . =
4 3 25 300
Thus P(E) = P(E1) P(E / E1) + P(E2) P(E / E2)

1 1 5 1 31
= × + × =
6 2 6 300 360

P(E1 ) P(E / E1 ) 30
 P(E1 / E) = =
P(E) 31

RESONANCE 35
Self Practice Problems :

25. A ball is drawn from an urn containing 5 balls of different colours including white. Two men A and B

1 2
whose probability of speaking the truth are and respectively assert that the ball drawn is white.
3 5
Find the probability of the truth of their assertion.

4
Answer
7

11. Binomial Probability Distribution :

(i) A probability distribution spells out how a total probability of 1 is distributed over several values of a
random variable.
(ii) Mean of any probability distribution of a random variable is given by :

 pi x i
µ = =  pi x i (Since  pi = 1)
 pi

(iii) Variance of a random variable is given by, 2 = (x i – µ)2 . pi


 2 = pi x i2 – µ2 (Note that SD = + 2 )
(iv) The probability distribution for a binomial variate ‘X’ is given by :
P(X = r) = nCr pr qn – r where P(X = r) is the probability of r successes.

P(r  1) nr p
The recurrence formula = . , is very helpful for quickly computing P(1) . P(2) . P(3)
P(r ) r 1 q
etc. if P(0) is known.
(v) Mean of BPD = np ; variance of BPD = npq.
(vi) If p represents a person’s chance of success in any venture and ‘M’ the sum of money which he will
receive in case of success, then his expectations or probable value = pM

Solved Example # 40
A random variable X has the following probability distribution :

X 0 1 2 3 4 5 6 7
P(X) 0 k 2k 2k 3k k2 2k2 7k2 + k
Determine
(i) k (ii) P(X < 3) (iii) P(X > 6) (iv) P(0 < X < 3)
[Hint : Use  P(X) = 1 to determine k, P(X < 3) = P(0) + P(1) + P(2), P(X > 6) = P(7) etc.]

Solved Example # 41
A pair of dice is thrown 5 times. If getting a doublet is considered as a success, then find the mean
and variance of successes.
Solution

1
In a single throw of a pair of dice, probability of getting a doublet =
6

1
con sidering it to be a success, p =
6

1 5
 q = 1 – =
6 6

RESONANCE 36
1 5
mean = 5 × =
6 6

1 5 25
variance = 5 × . =
6 6 36

Solved Example # 42
A pair of dice is thrown 4 times. If getting a total of 9 in a single throw is considered as a success
then find the mean and variance of successes.
Solution

4 1
p = probability of getting a total of 9 = =
36 9

1 8
 q = 1 – =
9 9

1 4
 mean = np = 4 × =
9 9

1 8 32
variance = npq = 4 × × =
9 9 81

Solved Example # 43
Difference between mean and variance of a Binomial variate is ‘1’ and difference between their squares is ‘11’.
Find the probability of getting exactly three success
Solution
Mean = np & variance = npq
therefore, np – npq = 1 ..........(i)
n2p2 – n2p2q2 = 11 ..........(ii)
Also, we know that p + q = 1 ..........(iii)
5 1
Divide equation (ii) by square of (i) and solve, we get, q = ,p= & n = 36
6 6

 1 5
3 33

Hence probability of ‘3’ success = 36C3 ×   ×   Ans.


6 6

Self Practice Problems :

26. A box contains 2 red and 3 blue balls. Two balls are drawn successively without replacement. If getting
‘a red ball on first draw and a blue ball on second draw’ is considered a success, then find the mean
and variance of successes.
Answer mean = 2.1, 2 = .63

27. Probability that a bulb produced by a factory will fuse after an year of use is 0.2. If fusing of a bulb
is considered an failure, find the mean and variance of successes for a sample of 10 bulbs.
Answer mean = 8 and variance = 1.6

RESONANCE 37
28. A random variable X is specified by the following distribution law :

X 2 3 4
P(X = x) 0.3 0.4 0.3
Then the variance of this distribution is :
(A*) 0.6 (B) 0.7 (C) 0.77 (D) 1.55

12. Geometrical Applications:


The following statements are axiomatic :

(i) If a point is taken at random on a given straight line segment AB, the chance that it falls on a particular
segment PQ of the line segment is PQ/AB.
(ii) If a point is taken at random on the area S which includes an area , the chance that the point falls
on  is /S.

Solved Example # 44
A sphere is circumscribed over a cube. Find the probability that a point lies inside the sphere, lies outside the
cube.
Solution

favorable volume
Required probability =
total volume

a 3
Clearly if edge length of cube is a radius of sphere will be
2

4  a 3 
3
a 3 3
Thus, volume of sphere =   =
3  2  2

1 2
Hence P = 1 – =1–
3  3

2

Solved Example # 45
A given line segment is divided at random into three parts. What is the probability that they form sides
of a possible triangle ?
Solution
Let AB be the line segment of length .
Let C and D be the points which divide AB into three parts.
Let AC = x, CD = y. Then DB =  – x – y.
Clearly x + y < 
 the sample space is given by
the region enclosed by  OPQ, where OP = OQ = 

2
Area of OPQ =
2

RESONANCE 38
Now if the parts AC, CD and DB form a triangle, then


x + y >  – x – y i.e. x + y > ...........(i)
2


x +  – x – y > y i.e. y < ...........(ii)
2


y+  – x – y > x i.e. x < ...........(iii)
2
from (i), (ii) and (iii), we get
the event is given by the region closed in RST

1  
ar ( RST ) . .
2 2 2 1
 Probability of the event = ar ( OPQ ) = =
2 4
2

Solved Example # 46
On a line segment of length L two points are taken at random, find the probability that the distance
between them is  , where  < 1
Solution
Let AB be the line segment
Let C and D be any two points on AB so that AC = x and CD = y. Then x + y < L, y > 
 sample space is represented by the region enclosed by OPQ.

1 2
Area of OPQ = L
2

The event is represented by the region, bounded by the RSQ

1
Area of RSQ = (L – )2
2

L  
2

 probability of the event =  


 L 

Self Practice Problems :

29. A line segment of length a is divided in two parts at random by taking a point on it, find the probability
that no part is greater than b, where 2b > a

2b  a
Answer
a
30. A cloth of length 10 meters is to be randomly distributed among three brothers, find the probability that
no one gets more than 4 meters of cloth.

1
Answer
25

RESONANCE 39
Part : (A) Only one correct option
1. If A, B, C are 3 events, then the probability that exactly 2 of them occur is given by:
(A) P(A  B) + P(B  C) + P(C  A)  2P(A  B  C)
(B) P(A  B) + P(B  C) + P(C  A)  3P(A  B  C)
(C) P(A) + P(B) + P(C)  P(A  B)  P(B  C) P(C  A) + P(A  B  C)
(D) none of these

2. In a series of 3 independent trials the probability of exactly 2 success is 12 times as large as the
probability of 3 successes. The probability of a success in each trial is:
(A) 1/5 (B) 2/5 (C) 3/5 (D) 4/5

3. There are two urns. There are m white & n black balls in the first urn and p white & q black balls in the
second urn. One ball is taken from the first urn & placed into the second. Now, the probability of
drawing a white ball from the second urn is:
pm  (p  1)n (p  1)m  pn qm  (q  1)n (q  1)m  qn
(A) (m  n)(p  q  1) (B) (m  n)(p  q  1) (C) (m  n)(p  q  1) (D) (m  n)(p  q  1)

4. Box– contains 3 red and 2 blue balls whilst box- contains 2 red and 6 blue balls. A fair coin is tossed.
If it turns up head, a ball is drawn from Box-, else a ball is drawn from Box–. Find the probabiliy of
event ‘ball drawn is from Box-, if it is red’.
12 10 17 3
(A) (B) (C) (D)
17 17 20 5

5. A local post office is to send M telegrams which are distributed at random over N communication
channels, (N > M). Each telegram is sent over any channel with equal probability. Chance that not
more than one telegram will be sent over each channel is:
N
CM . M ! N
CM . N ! N
CM . M ! N
CM . N !
(A) (B) (C) 1  (D) 1 
N M
M N
M
N
NM
6. A mapping is selected at random from all the mappings defined on the set A consisting of three
distinct elements. The probability that the mapping selected is one to one is:
(A) 1/9 (B) 1/3 (C) 1/4 (D) 2/9

7. A bag contains 7 tickets marked with the numbers 0, 1, 2, 3, 4, 5, 6 respectively. A ticket is drawn &
replaced. Then the chance that after 4 drawings the sum of the numbers drawn is 8 is:
(A) 165/2401 (B) 149/2401 (C) 3/49 (D) none

8. A biased coin with probability p, 0 < p < 1 of heads is tossed until a head appears for the first time. If
the probability that the number of tosses required is even is 2/5, then p equals
(A) 1/3 (B) 2/3 (C) 2/5 (D) 3/5

9. If 4 whole numbers taken at random are multiplied together, then the chance that the last digit in the
product is 1, 3, 7 or 9 is:
(A) 16/625 (B) 4/125 (C) 8/81 (D) none

10. A letter is known to have come either from "KRISHNAGIRI" or "DHARMAPURI". On the post mark only
the two consecutive letters "RI" are visible. Then the chance that it came from Krishnagiri is:
(A) 3/5 (B) 2/3 (C) 9/14 (D) none

(1 + 3 p) (1 – p) (1 – 2 p)
11. If , & are the probabilities of three mutually exclusive events then the set of all
3 4 2
values of p is.

1 2 1 1 1 1 1 2
(A)  , (B)  , (C)  , (D)  ,
2 3  3 2  4 2  3 3 

RESONANCE 40
12. Let p be the probability that a man aged x years will die in a year time. The probability that out of 'n'
men A1, A2, A3,......, An each aged 'x' years. A1 will die & will be the first to die is:

1  pn p p (1  p)n1 1  (1  p)n
(A) (B) (C) (D)
n n n n
13. 5 girls and 10 boys sit at random in a row having 15 chairs numbered as 1 to 15, then the probability
that end seats are occupied by the girls and between any two girls an odd number of boys sit is:

20  10!  5! 10  10!  5! 20  10 !  30 10  10!  5!


(A) (B) (C) (D)
15! 15! 15 ! 25!
14. Two dice are rolled simultaneously. The probability that the sum of the two numbers on the top faces
will be atleast 10 is:
(A) 1/6 (B) 1/12 (C) 1/18 (D) none
15. There are 4 urns. The first urn contains 1 white & 1 black ball, the second urn contains 2 white & 3
black balls, the third urn contains 3 white & 5 black balls & the fourth urn contains 4 white & 7 black
i2 1
balls. The selection of each urn is not equally likely. The probability of selecting ith urn is
34
(i = 1, 2, 3, 4). If we randomly select one of the urns & draw a ball, then the probability of ball being
white is :
569 27 8
(A) (B) (C) (D) none of these
1496 56 73

16. 2/3rd of the students in a class are boys & the rest girls. It is known that probability of a girl getting a
first class is 0.25 & that of a boy is 0.28. The probability that a student chosen at random will get a first
class is:
(A) 0.26 (B) 0.265 (C) 0.27 (D) 0.275
17. The contents of urn I and II are as follows,
Urn I: 4 white and 5 black balls
Urn II: 3 white and 6 black balls
One urn is chosen at random and a ball is drawn and its colour is noted and replaced back to the urn.
Again a ball is drawn from the same urn, colour is noted and replaced. The process is repeated 4 times
and as a result one ball of white colour and 3 of black colour are noted. Find the probability the chosen
urn was I.
125 64 25 79
(A) (B) (C) (D)
287 127 287 192

18. The sides of a rectangle are chosen at random, each less than 10 cm, all such lengths being equally
likely. The chance that the diagonal of the rectangle is less than 10 cm is
(A) 1/10 (B) 1/20 (C) /4 (D) /8
19. The sum of two positive quantities is equal to 2n. The probability that their product is not less than
3/4 times their greatest product is
3 1 1
(A) (B) (C) (D) none of these
4 2 4

20. The probability that 4th power of a positive integer ends in the digit 6 is:
(A) 10 % (B) 20 % (C) 25 % (D) 40 %
21. Posteriory probability of the occurrance of the event ‘The coin turns head and the die turns up an odd
number’ is
1 2 3 4
(A) (B) (C) (D)
7 7 7 7

22. Expectation of D is
225 150 200 300
(A) (B) (C) (D)
7 7 7 7

RESONANCE 41
23. For the three events A, B & C, P(exactly one of the events A or B occurs) = P(exactly one of the events
B or C occurs) = P(exactly one of the events C or A occurs) = p & P (all the three events occur
simultaneously) = p2, where 0 < p < 1/2. Then the probability of at least one of the three events A, B &
C occurring is: [IIT -1996]
3p  2p 2 p  3p 2 p  3p 2 3p  2p 2
(A) (B) (C) (D)
2 4 2 4
Part : (B) May have more than one options correct
24. In throwing a die let A be the event ‘coming up of an odd number’, B be the event ‘coming up of an even
number’, C be the event ‘coming up of a number  4’ and D be the event ‘coming up of a number < 3’,
then
(A) A and B are mutually exclusive and exhautive
(B) A and C are mutually exclusive and exhautive
(C) A, C and D form an exhautive system
(D) B, C and D form an exhautive system
25. Let 0 < P(A) < 1, 0 < P(B) < 1 & P(A  B) = P(A) + P(B)  P(A). P(B), then:
(A) P(B/A) = P(B)  P(A) (B) P(AC  BC) = P(AC) + P(BC)
(C) P((A  B) ) = P(A ). P(B )
C C C
(D) P(A/B) = P(A)
26. For any two events A & B defined on a sample space,

P A B 
P (A)  P (B)  1
(A) , P (B)  0 is always true

 
P (B)
(B) P A  B = P (A) - P (A  B)
(C) P (A  B) = 1 - P (Ac). P (Bc), if A & B are independent
(D) P (A  B) = 1 - P (Ac). P (Bc), if A & B are disjoint
27. If A, B & C are three events, then the probability that none of them occurs is given by:

(A)  
P A + P(B) + P(C)  P(A  B)  P(B  C)  P(A  C) + P(A  B  C)

(B) P  A  + P  B + P  C 

(C) P  A   P(B)  P(C) + P(A  B) + P(B  C) + P(C  A)  P(A  B  C)

(D) P A  B  C   P(A)  P(B) – P(C) + P(A  B) + P(B  C) + P(C  A)

28. A student appears for tests I, II & III. The student is successful if he passes either in tests I & II or tests
I & III. The probabilities of the student passing in the tests I, II & III are p, q &
1/2 respectively. If the probability that the student is successful is 1/2, then:
(A) p = 1, q = 0 (B) p = 2/3, q = 1/2
(C) p = 3/5, q = 2/3 (D) there are infinitely many values of p & q.

29. If the integers m and n are chosen at random between 1 and 100, then the probability that a number of
the form 7m + 7n is divisible by 5 equals [IIT - 1999]
(A) 1/4 (B) 1/7 (C) 1/8 (D) 1/49

1. A letter is known to have come either from London or Clifton; on the postmark only the two consecutive
letters ON are legible; what is the chance that it came from London?
2. A speaks the truth 3 out of 4 times, and B 5 out of 6 times; what is the probability that they will
contradict each other in starting the same fact?
3. A pair of dice is thrown 5 times. Find the mean and variance of the probability distribution of appearance
of doublets on the throws.
4. If on a straight line 10 cm. two length of 6 cm and 4 cm are measured at random, find the probability
that their common part does not exceed 3 cms.

RESONANCE 42
5. Let p be the probability that a man aged x years will die in a year time. Find the probability that out of
'n' men A1, A2, A3,......, An each aged 'x' years. A1 will die & will be the second to die.
6. A car is parked by an owner amongst 25 cars in a row, not at either end. On his return he finds that
exactly 15 placed are still occupied. Find the probability that both the neighbouring places are empty.
7. A gambler has one rupee in his pocket. He tosses an unbiased normal coin unless either he is ruined
or unless the coin has been tossed for a maximum of five times. If for each head he wins a rupee and
for each tail he looses a rupee, then find the probability that the gambler is ruined.
8. Mr. Dupont is a professional wine taster. When given a French wine, he will identify it with probability 0.9
correctly as French, and will mistake it for a Californian wine with probability 0.1. When given a Californian
wine, he will identify it with probability 0.8 correctly as Californian, and will mistake it for a French wine with
probability 0.2. Suppose that Mr. Dupont is given ten unlabelled glasses of wine, three with French and seven
with Californian wines. He randomly picks a glass, tries the wine and solemnly says. “French”. Find the
probability that the wine he tasted was Californian.
9. In ten trials of an experiment, if the probability of getting '4 successes’ is maximum, then show that
3
probability of failure in each trial can be equal to .
5

10. Mean and variance of a Binomial variate are in the ratio of 3 : 2. Find the most probable number of
happening of the variable in 10 trials of the experiment.
11. In a Nigerian hotel, among the english speaking people 40% are English & 60% Americans. The English
& American spellings are "RIGOUR" & "RIGOR" respectively. An English speaking person in the hotel
writes this word. A letter from this word is chosen at random & found to be a vowel. Find the probability
that the writer is an Englishman.
12. There is a group of k targets, each of which independently of the other targets, can be detected by a
radar unit with probability p. Each of 'm' radar units detects the targets independently of other units.
Find the probability that not all the targets in the group will be detected.
13. 2 positive real numbers x and y satisfy x  1 and y  1 are chosen at random. Find the probability that
1
x + y  1, given that x2 + y2  .
4

14. There are two lots of identical articles with different amounts of standard & defective articles. There are
N articles in the first lot, n of which are defective & M articles in the second lot, m of which are
defective. K articles are selected from the first lot & L articles from the second & a new lot results. Find
the probability that an article selected at random from the new lot is defective.
15. The odds that a book will be favorably reviewed by three independent crities are 5 to 2, 4 to 3, and 3 to 4
respectively : what is the probability that of the three reviews a majority will be favourable?

16. Find the chance of throwing 10 exactly in one throw with 3 dice.
17. If 12 tickets numbered 0, 1, 2, .......11 are placed in a bag, and three are drawn out, show that the chance
3n 3
that the sum of the numbers on them is equal to 12 is =
(6n  1)(6n  2) 55
18. A man has 10 coins and one of them is known to have two heads. He takes one at random and tosses it
5 times and it always falls head : what is the chance that it is the coins with two heads?
19. A purse contains five coins, each of which may be a rupees coin or a 50 ps coin ; two are drawn and found
to be shillings : find the probable value of the remaining coins.
20. One of a pack of 52 cards has been lost; from the remainder of the pack two cards are drawn and are found
to be spades; find the chance that the missing card is a spade.

1 1
21. A, B are two inaccurate arithmeticians whose chance of solving a given question correctly are and
8 12
respectively; if they obtain the same result, and if it is 1000 to 1 against their making the same mistake, find
the chance that the result is correct.

RESONANCE 43
22. If n integers taken at random are multiplied together, show that the chance that the last digit of the product
2n 4 n  2n 5n  4n
is 1, 3, 7, or 9 is ; the chance of its being 2, 4, 6 or 8 is ; of its being 5 is ; and of its being
5n 5n 10 n
10  8  5 n  4 n
n n
0 is .
10 n
23. A player tosses a coin and is to score one point for every head and 2 points if every tail turned up.
He is to play until he reaches ‘n’. If pn is the chance of obtaining exactly ‘n’ crores, find pn for
1
n = 1, 2, 3, 4. Also show that pn = (pn – 1 + pn – 2).
2
24. A lot contains 50 defective & 50 non defective bulbs. Two bulbs are drawn at random, one at a time, with
replacement. The events A, B, C are defined as: [IIT - 1992]
A = { the first bulb is defective} ; B = { the second bulb is non defective}
C = { the two bulbs are both defective or both non defective}
Determine whether (i) A,B,C are pair wise independent (ii) A,B,C are independent
25. Eight players P1, P2, P3,............P8 play a knock-out tournament. It is known that whenever the players
Pi and Pj play, the player Pi will win if i < j. Assuming that the players are paired at random in each
round, what is the probability that the players P4 reaches the final. [IIT - 1999]
26. A box contains N coins, m of which are fair and the rest are biased. The probability of getting a head
when a fair coin is tossed is 1/2, while it is 2/3 when a biased coin is tossed. A coin is drawn from the
box at random and is tossed twice. The first time it shows head and the second time it shows tail.
What is the probability that the coin drawn is fair? [IIT - 2002 ]
27. A person has to go through three successive tests. Probability of his passing first exam is P. Probability
of passing successive test is P or P/2 according as he passed the last test or not. He is selected if he
passes atleast two tests. Find the probability of his selection. [IIT - 2003]

28.  
Prove that P(A U B) P A  B  P (C) where A and B are independent events and P(C) is the probability of
exactly one of A or B occurs. [IIT - 2004]
29. A person goes to office either by car, scooter, bus or train, the probability of which being
1 3 2 1
, , and respectively. Probability that he reaches office late, if he takes car, scooter, bus or
7 7 7 7
2 1 4 1
train is , , and respectively. Given that he reached office in time, then what is the probability
9 9 9 9
that he travelled by a car. [IIT - 2005]

EXERCISE # 1 15 11 14 5
6. 7. 8. 10. 3 11.
92 16 41 11
1. B 2. A 3. B 4. A 5. A 6. D 7. B
8. A 9. A 10. C 11. B 12. D 13. A 14. A 8 KnM  LmN
12. 1  {1  (1  p)m}k 13. 16   14. MN (K  L )
15. A 16. C 17. A 18. C 19. B 20. D 21. B
22. A 23. A 24. AC 25. CD 26. AC 27. CD 15. 209/343 16. 1/5 18. (32/41) 19. 2.25 Rs
28. ABCD 29. AD 20. 11/50 21. (13/14)

EXERCISE # 2 23. p1 =
1 3 5
, p2 = , p3 = , p4 =
11
.
2 4 8 16

12 1 5 25 24. (i) A, B, C are pairwise independent


1. 2. 3. mean = , variance =
77 3 6 36 (ii) A, B, C are not independent

9 1 9m
4. 5. [1 – (1 – p)n – np (1 – p) n–1] 25. 4/35 26. 27. 2 P2 - P3 29. 1/7
24 n 8N m

RESONANCE 44
Binomial Theorem

1. Binomial Expression :
Any algebraic expression which contains two dissimilar terms is
called binomial expression.

He who loves For example : x + y, x2y +


1
, 3 – x, x2  1 +
1
etc.
xy 2
( x  1)1/ 3
3

practice without
theory is like the
2. Statement of Binomial theorem :
If a, b  R and n  N, then ;
sailor who boards (a + b)n = nC0 anb0 + nC1 an–1 b1 + nC2 an–2 b2 +...+ nCr an–r br +...+ nCn
ship without a a0 bn


rudder and compass n

or (a + b)n =
n
C r a n r b r
and never knows r0

where he may cast Now, putting a = 1 and b = x in the binomial theorem -


or (1 + x) n = nC0 + nC1 x + nC2 x 2 +... + nCr x r +...+ nCn xn
.....LeonardodaVi nci

n

(1 + x) = n
n
Cr x r
r 0

Solved Example # 1
Expand the following binomials :

Number is the  
4
 1  3x
2
(i) (x – 3) (ii) 
 
5

 2 
ruler of forms and Solution.
ideas, and the (i) (x – 3)5 = 5C0x 5 + 5C1x 4 (– 3)1 + 5C2 x3 (– 3)2 + 5C3 x2 (–3)3
+ 5C4 x (– 3)4 + 5C5 (– 3)5
cause of gods and = x – 15x + 90x – 270x 2 + 405x – 243
5 4 3

demons.
 3x 2   3x 2   3x 2 
4 2
....... Pythagoras 1   = 4C + 4C   4  
(ii)  2   2  + C2  2 

0 1
  

 3x 2   3x 2 
3 4

+ C3  
4  + 4C





 2 
4
 2 

27 4 27 6 81 8
= 1 – 6x2 + x – x + x
2 2 16

RESONANCE 1
Solved Example # 2

 2x 3 y 
20
Expand the binomial    up to four terms
 3 2 
Solution.

 2x 3y   2x   2x   3y   2x   3y 
20 20 19 18 2
   = C0 
20  + C1  
20   + 20C    
 3 2   3   3   2  2
 3   2 

 2x   3y 
17 3

+ C3  
20   + ....
 3   2 

 2x  2 2 2


20 18 16 14

=   + 20.   x19y + 190 .   x18 y2 + 1140   x17 y3 + .....


 3  3 3 3

Self practice problems

 y
6

1. Write the first three terms in the expansion of  2   .


 3 

 x2 3 
5

2. Expand the binomial    .



 3 x
80 2
Ans. (1) 64 – 64y + y
3

x10 5 7 10 4 135 243


(2) + x + x + 30x + 2 + .
243 27 3 x x5

3. Properties of Binomial Theorem :


(i) The number of terms in the expansion is n + 1.
(ii) The sum of the indices of x and y in each term is n.
(iii) The binomial coefficients ( nC0, nC1 ..........nCn) of the terms equidistant from the beginning and
the end are equal, i.e. nC0 = nCn, nC1 = nCn–1 etc. { nCr = nCn–r}

Solved Example # 3
The number of dissimilar terms in the expansion of (1 – 3x + 3x2 – x3)20 is
(A) 21 (B) 31 (C) 41 (D) 61
Solution.
(1 – 3x + 3x 2 – x3)20 = [(1 – x)3]20 = (1 – x)60
Therefore number of dissimilar terms in the expansion of (1 – 3x + 3x2 – x 3)20 is 61.

4. Some important terms in the expansion of (x + y) n :


(i) General term :
(x + y)n = nC0 xn y0 + nC1 xn–1 y1 + ...........+ nCr xn–r yr + ..........+ nCn x0 yn
(r + 1)th term is called general term.
T r+1 = nCr xn–r yr

RESONANCE 2
Solved Example # 4
Find

 4x 5 
9
(i) 28th term of (5x + 8y) 30
(ii) 7th term of   
 5 2x 
Solution.
(i) T27 + 1 = 30C27 (5x) 30– 27 (8y)27

30 !
= (5x)3 . (8y) 27 Ans.
3 ! 27 !

 4x 5 
9

(ii) 7th term of   


 5 2x 

96
 4x   5 
6

T6 + 1 = C6 
9   
 5   2x 

9!  4x   5 
3 6

= 3!6!    
 5   2x 

10500
= Ans.
x3

Solved Example # 5
Find the number of rational terms in the expansion of (91/4 + 81/6)1000.
Solution.

The general term in the expansion of 91/ 4  81/ 6  1000


is

1000 r
 1  1
r
 4 8 6 
Tr+1 = 1000Cr  9   
   

1000 r r
= 1000
Cr 3 2
22
The above term will be rational if exponent of 3 and 2 are integres

1000  r r
It means and must be integers
2 2
The possible set of values of r is {0, 2, 4, ............, 1000}
Hence, number of rational terms is 501 Ans.

(ii) Middle term (s) :

n 2
(a) If n is even, there is only one middle term, which is   th term.
 2 

 n  1 n1 
(b) If n is odd, there are two middle terms, which are   th and   1 th terms.
 2   2 

RESONANCE 3
Solved Example # 6

Find the middle term(s) in the expansion of

 2   3 
14 9

(i) 1 x  (ii)  3a  a 


 2   6 
 
Solution.

 2 
14

(i) 1  x 
 2 

 14  2 
Here, n is even, therefore middle term is   th term.
 2 
It means T 8 is middle term

 x2 
 = – 429 x14. Ans.
7

T 8 = 14C7   
 2  16

 
9
 3a  a
3
(ii) 
 6 
 

 9  1 9 1 
Here, n is odd therefore, middle terms are   th &   1 th.
 2   2 
It means T 5 & T 6 are middle terms

 a3 
4

T 5 = 9C4 (3a) 9 – 4   = 189 a17 Ans.


 6 
  8

 a3 
5

T 6 = 9C5 (3a)9 – 5   = – 21 a19. Ans.


 6  16
 

  b 
n
(iii) Term containing specified powers of x in  ax   
 x 

Solved Example # 7

 1 
15
Find the coefficient of x32 and x–17 in  x 4  3  .
 x 
Solution.
Let (r + 1)th term contains xm

 1 
r
Tr + 1 = 15Cr (x 4)15 – r   3 
 x 
= 15Cr x60 – 7r (– 1)r
(i) for x32 , 60 – 7r = 32
 7r = 28
 r = 4. (T5)
T 5 = C4 x32 (– 1) 4
15

Hence, coefficient of x32 is 1365Ans.

RESONANCE 4
(ii) for x –17, 60 – 7r = – 17
 r = 11 (T12)
T12= 15C11 x–17 (– 1)11
Hence, coefficient of x–17 is – 1365 Ans.
(iv) Numerically greatest term in the expansion of (x + y)n, n  N
Let T r and T r+1 be the rth and (r + 1)th terms respectively
Tr = nCr–1 x n–(r–1) yr–1
Tr+1 = nCr x n–r yr

Tr 1 n
Cr x n r y r nr 1 y
Now, Tr = = .
n
Cr 1 x nr 1y r 1 r x

Tr 1
Consider Tr 1

 n  r  1 y
  1
 r  x

n 1 x
–1
r y

n 1
r
x
1
y

Case - 

n 1
When is an integer (say m), then
x
1
y

(i) T r+1 > T r when r < m (r = 1, 2, 3 ...., m – 1)


i.e. T 2 > T 1, T 3 > T 2, ......., T m > T m–1
(ii) T r+1 = T r when r = m
i.e. T m+1 = Tm
(iii) T r+1 < T r when r > m (r = m + 1, m + 2, ..........n )
i.e. T m+2 < T m+1 , T m+3 < T m+2 , ..........T n+1 < T n
Conclusion :

n 1
When is an integer, equal to m, then Tm and Tm+1 will be numerically greatest terms (both terms
x
1
y

are equal in magnitude)


Case - 

n 1
When is not an integer (Let its integral part be m), then
x
1
y

n 1
(i) T r+1 > T r when r< (r = 1, 2, 3,........, m–1, m)
x
1
y

i.e. T 2 > T 1 , T 3 > T 2, .............., T m+1 > T m

RESONANCE 5
n 1
(ii) T r+1 < T r when r > (r = m + 1, m + 2, ..............n)
x
1
y

i.e. T m+2 < T m+1 , T m+3 < T m+2 , .............., T n +1 < T n

Conclusion :

n 1
When is not an integer and its integral part is m, then T m+1 will be the numerically greatest
x
1
y

term.

Solved Example # 8
1
Find the numerically greatest term in the expansion of (3 – 5x)15 when x = .
5
Solution.
Let r th and (r + 1)th be two consecutive terms in the expansion of (3 – 5x)15
Tr + 1  Tr
15
Cr 315 – r (| – 5x|)r  15Cr – 1 315 – (r – 1) (|– 5x|)r – 1

15 )! 3. 15 )!
|– 5x | 
(15  r ) ! r ! (16  r ) ! (r  1) !

1
5. (16 – r) 3r
5
16 – r  3r
4r  16
r4
Explanation:

For r  4, T r + 1  T r
 T2 > T1
T3 > T2
T4 > T3
T5 = T4
For r > 5, T r + 1 < T r
T6 < T5
T7 < T6
and so on
Hence, T 4 and T 5 are numerically greatest terms and both are equal.

Self practice problems :

 2 3
9

3. Find the term independent of x in  x  


 x

4. The sum of all rational terms in the expansion of (31/5 + 21/3)15 is


(A) 60 (B) 59 (C) 95 (D) 105

RESONANCE 6
 1
8

5. Find the coefficient of x–1 in (1 + 3x2 + x 4) 1  


 x

6. Find the middle term(s) in the expansion of (1 + 3x + 3x2 + x 3)2n

2
7. Find the numerically greatest term in the expansion of (7 – 5x) 11 where x = .
3
Ans. (3) 28.37 (4) B (5) 232

440
(6) 6n
C3n . x3n (7) T4 = × 7 8 × 5 3.
9

5. Multinomial Theorem

As we know the Binomial Theorem –


n

(x + y) =
n
n
Cr xn–r yr
r 0

 (n  r )! r!
n!
n

= xn–r yr
r0

putting n – r = r1 , r = r2


n!
therefore, (x + y)n = x r1 . y r2
r1  r2
r ! r2 !
n 1

Total number of terms in the expansion of (x + y) n is equal to number of non-negative integral solution
of r1 + r2 = n i.e. n+2–1C2–1 = n+1C1 = n + 1

In the same fashion we can write the multinomial theorem


n!
(x 1 + x 2 + x3 + ........... xk)n = x1r1 . x r22 ...x rkk
r1  r2 ...rk n
r1! r2 !... rk !

Here total number of terms in the expansion of (x 1 + x 2 + .......... + x k)n is equal to number of non-
negative integral solution of r1 + r2 + ........ + r k = n i.e. n+k–1Ck–1

Solved Example # 9
Find the coeff. of a2 b3 c4 d in the expansion of (a – b – c + d) 10
Solution.


(10 )!
(a – b – c + d) 10 = r ! r ! r ! r ! (a) ( b) ( c ) (d)
r1 r2 r3 r4
r1 r2  r3  r4 10 1 2 3 4

we want to get a2 b3 c4 d this implies that r 1 = 2, r2 = 3, r 3 = 4, r4 = 1


 coeff. of a2 b3 c4 d is

(10 )!
2! 3! 4! 1! (–1) (–1) = – 12600 Ans.
3 4

RESONANCE 7
Solved Example # 10

 7
11
In the expansion of  1  x   find the term independent of x.
 x
Solution.


 7
11 (11)! 73
r
1  x   = r !r !r ! (1)r1 ( x )r2  
 x r1  r2 r3 11 1 2 3 x

7
The exponent 11 is to be divided among the base variables 1, x and in such a way so that we get x0.
x
Therefore, possible set of values of (r1, r2, r 3) are (11, 0, 0), (9, 1, 1), (7, 2, 2), (5, 3, 3), (3, 4, 4),
(1, 5, 5)
Hence the required term is

(11)! (11)! (11)! (11)! (11)! (11)!


(70) + 9! 1 !1 ! 71 + 7! 2 ! 2 ! 72 + 5! 3 ! 3 ! 73 + 3! 4 ! 4 ! 74 + 1 ! 5 ! 5 ! 75
(11)!

(11)! 2! (11)! 4! (11) ! 6!


= 1 + 9 ! 2 ! . 1 ! 1 ! 71 + 7 ! 4 ! . 2 ! 2 ! 72 + 5 ! 6 ! . 3 ! 3 ! 73

(11) ! 8! (11) ! (10) !


+ 3 ! 8 ! . 4 ! 4 ! 74 + 1 ! 10 ! . 5 ! 5 ! 75

= 1 + 11C2 . 2C1 . 71 + 11C4 . 4C2 . 72 + 11C6 . 6C3 . 73 + 11C8 . 8C4 . 74 + 11C10 . 10C5 . 75

1+ 
5

=
11
C 2r . 2rC . 7r Ans.
r
r 1

Self practice problems :

8. The number of terms in the expansion of (a + b + c + d + e + f) n is


(A) n+4C4 (B) n+3Cn (C) n+5Cn (D) n + 1

9. Find the coefficient of x 3 y4 z2 in the expansion of (2x – 3y + 4z) 9

10. Find the coefficient of x 4 in (1 + x – 2x2)7

9!
Ans. (8) C (9) 3! 4! 2! (10) 23 34 42 – 91

6. Application of Binomial Theorem :


(i) I f ( A  B)n =  + f w h e re  an d n are p ositi v e integ ers, n being odd and
0 < f < 1 then ( + f) f = k where A – B2 = k > 0 and
n
A – B < 1.
If n is an even integer, then ( + f) (1 – f) = kn

Solved Example # 11

If n is positive integer, then prove that the integral part of (7 + 4 3 )n is an odd number..
Solution.

Let (7 + 4 3 )n =  + f .............(i)
where  & f are its integral and fractional parts respectively.
It means 0 < f < 1

Now, 0<7–4 3 <1

RESONANCE 8
0 < (7 – 4 3 )n < 1

Let (7 – 4 3 )n = f .............(ii)
 0 < f < 1
Adding (i) and (ii)

 + f + f = (7 + 4 3 )n + (7 – 4 3 )n

= 2 [nC0 7n + nC2 7n – 2 (4 3 )2 + ..........]


 + f + f = even integer(f + f must be an integer)
0 < f + f < 2  f + f = 1
 + 1 = even integer
therefore  is an odd integer.

Solved Example # 12

Show that the integer just above ( 3 + 1)2n is divisible by 2n + 1 for all n  N.
Solution.

Let ( 3 + 1)2n = (4 + 2 3 )n = 2n (2 + 3 )n =  + f ..........(i)


where  and f are its integral & fractional parts respectively
0 < f < 1.

Now 0< 3 –1<1

0 < ( 3 – 1)2n < 1

Let ( 3 – 1)2n = (4 – 2 3 )n = 2n (2 – 3 )n = f. ........(ii)


0 < f < 1
adding (i) and (ii)

 + f + f = ( 3 + 1)2n + ( 3 – 1) 2n

= 2n [(2 + 3 )n + (2 – 3 ) n]

= 2.2n [nC0 2n + nC2 2n – 2 ( 3 )2 + ........]


 + f + f =2n + 1 k (where k is a positive integer)
0 < f + f < 2  f + f= 1
 + 1 = 2n + 1 k.

 + 1 is the integer just above ( 3 + 1)2n and which is divisible by 2n + 1.


(ii) Cheking divisibility
Solved Example # 13
Show that 9n + 7 is divisible by 8, where n is a positive integer.
Solution.
9n + 7 = (1 + 8)n + 7
= nC0 + nC1 . 8 + nC2 . 82 + ....... + nCn 8n + 7.
= 8. C1 + 82. C2 + ....... + Cn . 8n + 8.
= 8 where,  is a positive integer
Hence, 9n + 7 is divisible by 8.

(iii) Finding remainder

RESONANCE 9
Solved Example # 14

What is the remainder when 599 is divided by 13.


Solution.
599 = 5.598 = 5. (25)49
= 5 (26 – 1)49
= 5 [49C0 (26)49 – 49C1 (26)48 + .......... + 49C48 (26)1 – 49C49 (26)0]
= 5 [49C0 (26)49 – 49C1 (26)48 + ...........+ 49C48 (26) 1 – 1]
= 5 [49C0 (26)49 – 49C1(26)48 + .......... + 49C48 (26)1 – 13] + 60
= 13 (k) + 52 + 8 (where k is a positive integer)
= 13 (k + 4) + 8
Hence, remainder is 8. Ans.
(iv) Finding last digit, last two digits and last there digits of the given number.

Solved Example # 15
Find the last two digits of the number (17)10.
Solution.
(17)10 = (289)5
= (290 – 1) 5
= 5C0 (290)5 – 5C1 (290)4 + ........ + 5C4 (290)1 – 5C5 (290)0
= 5C0 (290)5 – 5C1 . (290) 4 + .........5C3 (290)2 + 5 × 290 – 1
= A multiple of 1000 + 1449
Hence, last two digits are 49 Ans.

Note : We can also conclude that last three digits are 449.
(v) Comparison between two numbers

Solved Example # 16
Which number is larger (1.01)1000000 or 10,000 ?
Solution.
By Binomial Theorem
(1.01)1000000 = (1 + 0.01)1000000
= 1 + 1000000C1 (0.01) + other positive terms
= 1 + 1000000 × 0.01 + other positive terms
= 1 + 10000 + other positive terms
Hence (1.01)1000000 > 10,000
Self practice problems :

11. If n is positive integer, prove that the integral part of (5 5 + 11)2n + 1 is an even number..

12. If (7 + 4 3 )n =  + , where  is a positive integer and  is a proper fraction then prove that
(1 – ) ( + ) = 1.

13. If n is a positive integer then show that 32n + 1 + 2n + 2 is divisible by 7.

14. What is the remainder when 7103 is divided by 25 .

15. Find the last digit, last two digits and last three digits of the number (81)25.

16. Which number is larger (1.2) 4000 or 800

Ans. (14) 18 (15) 1, 01, 001 (16) (1.2)4000.

RESONANCE 10
7. Properties of Binomial Coefficients :
(1 + x) n = C0 + C1x + C2x 2 + ......... + Cr x r + .......... + Cnx n ......(1)
(1) The sum of the binomial coefficients in the expansion of (1 + x)n is 2n
Putting x = 1 in (1)
n
C0 + nC1 + nC2 + ........+ nCn = 2n ......(2)


n

or
n
Cr  2n
r 0

(2) Again putting x = –1 in (1), we get


n
C0 – nC1 + nC2 – nC3 + ............. + (–1)n nCn = 0 ......(3)

 (1)
n

or
r n
Cr  0
r0

(3) The sum of the binomial coefficients at odd position is equal to the sum of the binomial coefficients
at even position and each is equal to 2n–1.
from (2) and (3)
n
C0 + nC2 + nC4 + ................ = 2n–1
n
C1 + nC3 + nC5 + ................ = 2n–1

(4) Sum of two consecutive binomial coefficients


n
Cr + nCr–1 = n+1Cr
n! n!
L.H.S. = nCr + nCr–1 = +
(n  r )! r! (n  r  1)! (r  1)!

n! 1 1 
= (n  r )! (r  1)!   
 r n  r  1

n! (n  1)
= (n  r )! (r  1)! r(n  r  1)

(n  1)!
= (n  r  1)! r! = n+1
Cr = R.H.S.

(5) Ratio of two consecutive binomial coefficients


n
Cr nr 1
=
n
Cr 1 r

n n(n  1) n(n  1)(n  2).........(n  (r  1))


(6) Cr = Cr–1 = Cr–2 = ............. = r (r  1)(r  2).......2 .1
r(r  1)
n n–1 n–2
r

Solved Example # 17

If (1 + x) n = C0 + C1x + C2x2 + ............. + cnxn, then show that


(i) C0 + 3C1 + 32C2 + .......... + 3n Cn = 4n.

(ii) C0 + 2C1 + 3. C2 + ........ + (n + 1) Cn = 2n – 1 (n + 2).


C1 C2 C3 Cn 1
(iii) C0 – + – + ......... + ( –1) n = .
2 3 4 n1 n1
Solution.
(i) (1 + x)n = C0 + C1 x + C2x 2 + ........... + Cnx n
put x = 3
C0 + 3 . C1 + 32 . C2 + .......... + 3n . Cn = 4n

RESONANCE 11
(ii)  Method : By Summation

L.H.S. = nC0 + 2. nC1 + 3 . nC2 + ........ + (n + 1). nCn.

 (r  1) .
n
= n
Cr
r 0

 
n n

= r. nCr + n
Cr
r 0 r 0

 
n n
n1
=n Cr 1 + n
Cr
r 0 r 0

= n . 2n – 1 + 2n = 2n – 1 (n + 2). RHS

Method : By Differentiation

(1 + x)n = C0 + Cxx + C2x 2 + ........... + Cnx n


Multiplying both sides by x,
x(1 + x)n = C0x + C1x 2 + C2x 3 + ........ + Cn xn + 1.
Differentiating both sides
(1 + x)n + x n (1 + x)n – 1 = C0 + 2. C1 + 3 . C2x 2 + ....... + (n + 1)Cnx n.
putting x = 1, we get
C0 + 2.C1 + 3 . C2 + ...... + (n + 1) Cn = 2n + n . 2n – 1
C0 + 2.C1 + 3 . C2 + ...... + (n + 1) Cn = 2n – 1 (n + 2) Proved

(iii)  Method : By Summation


C1 C2 C3 Cn
L.H.S. = C0 – + – + ........ + (– 1)n.
2 3 4 n 1

 (1)
n
r n
Cr
= .
r 0 r 1

 (1) n  1 n 
n
1 r
. Cr 

n 1
= . Cr + 1 Cr 1 
r 1
n +1
n 1 r 0 

1
= [ n + 1C 1 – n+1
C2 + n+1
C3 – .............+ (– 1)n . n+1
C n + 1]
n 1
1
= [– n+1
C0 + n+1
C1 – n+1
C2 + ......... + (– 1) n . n+1
Cn + 1 + n+1
C 0]
n 1

=
1
n 1
= R.H.S.  n1
C0  n1 C1  n1 C2  ...  ( 1)n n1
Cn1  0 
Method : By Integration

(1 + x)n = C0 + C1x + C2x 2 + ...... + Cn x n.


Integrating both sides, with in the limits – 1 to 0.

 (1  x )n  1   x n1 
0 0
x2 x3
  = C 0 x  C1  C2  .....  Cn 
 n  1  1  2 3 n  1
1

1  C1 C 2 C 
– 0 = 0 –  C 0    .....  ( 1)n1 n 
n 1  2 3 n  1

C1 C2 Cn 1
C0 – + – .......... + (– 1) n = Proved
2 3 n 1 n 1

RESONANCE 12
Solved Example # 18

If (1 + x)n = C0 + C1x + C2x2 + ........+ Cnxn, then prove that


(i) C02 + C12 + C22 + ...... + Cn2 = 2nCn
(ii) C0C2 + C1C3 + C2C4 + .......... + Cn – 2 Cn = 2nCn – 2 or 2nCn + 2
(iii) 1. C02 + 3 . C12 + 5. C22 + ......... + (2n + 1) . Cn2 . = 2n. 2n – 1Cn + 2nCn.

Solution.
(i) (1 + x)n = C0 + C1x + C2x 2 + ......... + Cn xn. ........(i)
(x + 1) n = C0x n + C1x n – 1+ C2x n – 2 + ....... + Cn x 0 ........(ii)
Multiplying (i) and (ii)
(C0 + C1x + C2x 2 + ......... + Cnx n) (C0x n + C1x n – 1 + ......... + Cnx 0) = (1 + x) 2n
Comparing coefficient of xn,
C02 + C12 + C22 + ........ + Cn2 = 2nCn

(ii) From the product of (i) and (ii) comparing coefficients of xn – 2 or xn + 2 both sides,
C0C2 + C1C3 + C2C4 + ........ + Cn – 2 Cn = 2nCn – 2 or 2nCn + 2.

(iii)  Method : By Summation

L.H.S. = 1. C02 + 3. C12 + 5. C22 + .......... + (2n + 1) Cn2.

 (2r  1)
n
= n
C r2
r0

 (
n n
= 2.r . (nCr)2 +
n
Cr )2
r 0 r0

. n .
n
=2 n–1
Cr – 1 nCr + 2nCn
r 1

(1 + x)n = nC1 + nC4 x + nC2 x 2 + .............nCn x n ..........(i)


(x + 1) n – 1 = n – 1C0 xn – 1 + n – 1C1 xn – 2 + .........+n – 1Cn – 1x 0 .........(ii)
Multiplying (i) and (ii) and comparing coeffcients of xn.
n–1
C0 . nC1 + n – 1C1 . nC2 + ........... + n – 1Cn – 1 . nCn = 2n – 1Cn


n
n1
Cr 1 . nCr = 2n – 1Cn
r 0

Hence, required summation is


2n. 2n – 1Cn + 2nCn = R.H.S.
 Method : By Differentiation
(1 + x 2)n = C0 + C1x 2 + C2x 4 + C3x 6 + ..............+ Cn x 2n
Multiplying both sides by x
x(1 + x2)n = C0x + C1x 3 + C2x 5 + ............. + Cnx 2n + 1.
Differentiating both sides
x . n (1 + x 2)n – 1 . 2x + (1 + x2)n = C0 + 3. C1x 2 + 5. C2 x 4 + ....... + (2n + 1) Cn x 2n ........(i)
(x + 1) = C0 x + C1 x
2 n 2n 2n – 2
+ C2 x 2n – 4
+ ......... + Cn ........(ii)
Multiplying (i) & (ii)
(C0 + 3C1x 2 + 5C2x 4 + ......... + (2n + 1) Cn x 2n) (C0 x 2n + C1x 2n – 2 + ........... + Cn)
= 2n x2 (1 + x2)2n – 1 + (1 + x 2)2n
comparing coefficient of x2n,
C02 + 3C12 + 5C22 + .........+ (2n + 1) Cn2 = 2n . 2n – 1Cn – 1 + 2nCn.

C02 + 3C12 + 5C22 + .........+ (2n + 1) Cn2 = 2n . 2n–1Cn + 2nCn. Proved

RESONANCE 13
Solved Example # 19
Find the summation of the following series –
(i) m
Cm + m+1Cm + m+2Cm + .............. + nCm
(ii) n
C3 + 2 . n+1C3 + 3. n+2C3 + ......... + n . 2n–1C3
Solution.
(i) Method : Using property, nCr +nCr–1 = n+1Cr
m
Cm + m+1Cm + m+2Cm + .............. + nCm
m 1
=  Cm1  m1 Cm + m+2
Cm + .............. + nCm { mCm = m+1Cm+1}
 

m 2
Cm1  m 2 Cm
=   + .................. + nC
m

= m+3Cm+1 + ............. + nCm


= nCm+1 + nCm = n+1Cm+1 Ans.
 Method
m
Cm + m+1Cm + m+2Cm + .......... + nCm
The above series can be obtained by writing the coefficient of xm in
(1 + x) m + (1 + x)m+1 + ......... + (1 + x) n
Let S = (1 + x) m + (1 + x)m+1 +.............. + (1 + x) n

=

(1  x )m 1  x 
n m 1

1
x

=
1  x n1  1  x m
x
x : S (coefficient of x m in S)
m

(1  x )n1  (1  x )m
xm :
x
Hence, required summation of the series is n+1Cm+1 Ans.
(ii)
n
C3 + 2 . n+1C3 + 3 . n+2C3 + .......... + n . 2n–1C3
The above series can be obatined by writing the coefficient of x3 in
(1 + x) n + 2 . (1 + x) n+1 + 3 . (1 + x)n+2 + ........... + n . (1 + x) 2n–1
Let S = (1 + x)n + 2 . (1 + x)n+1 + 3. (1 + x) n+2 + ........... + n (1 + x)2n–1 ........(i)
(1 + x)S = (1 + x)n+1 + 2 (1 + x) n+2 + ............. + (n – 1) (1 + x)2n–1
+ n(1 + x)2n ........(ii)
Subtracting (ii) from (i)
– xS = (1 + x)n + (1 + x)n+1 + (1 + x)n+2 + .............. + (1 + x)2n–1 – n(1 + x) 2n

=

(1  x )n (1  x )n  1  – n (1 + x) 2n
x

 (1  x )2n  (1  x )n n(1  x )2n


S = +
x2 x
x3 : S (coefficient of x 3 in S)

 (1  x )2n  (1  x )n n(1  x )2n


x3 : +
x 2
x
Hence, required summation of the series is – 2nC5 + nC5 + n . 2nC4 Ans.

RESONANCE 14
Self practice problems :

17. Prove the following


(i) C0 + 3C1 + 5C2 + ............. + (2n + 1) Cn = 2n (n + 1)

42 43 4n1 5n1  1
(ii) 4C0 + . C1 + C2 + .............. + Cn =
2 3 n 1 n 1
(iii) n
C0 . n+1Cn + nC1 . nCn–1 + nC2 . n–1
Cn–2 + ........... + nCn . 1C0 = 2n–1 (n + 2)
(iv) 2
C2 + 3C2 + ......... + nC2 = n+1
C3

8. Binomial Theorem For Negative Integer Or Fractional Indices


If n  R then,

n(n  1) n(n  1)(n  2)


(1 + x) n = 1 + nx + 2! x2 + 3! x3 + ................

n(n  1)(n  2).......(n  r  1)


.................. + r! x r + .................... .

Remarks

(i) The above expansion is valid for any rational number other than a whole number if | x | < 1.

(ii) When the index is a negative integer or a fraction then number of terms in the expansion of
(1 + x)n is infinite, and the symbol nCr cannot be used to denote the coefficient of the general term.
(iii) The first terms must be unity in the expansion, when index ‘n’ is a negative integer or fraction

   y n (n  1)  y  
 x n 1    x n 1  n . 
n 2
y y
   .....  if 1
  x  x 2!  x  x

(x + y)n = 
  
 y n 1  x   y n 1  n . x  n (n  1)  x   .....  if
n 2
x
      1
  y y 2! y y
  

n(n  1)(n  2)......... (n  r  1)


(iv) The general term in the expansion of (1 + x) n is T r+1 = r! xr

(v) When ‘n’ is any rational number other than whole number then approximate value of (1 + x) n is
1 + nx (x2 and higher powers of x can be neglected)
(vi) Expansions to be remembered (|x| < 1)
(a) (1 + x)–1 = 1 – x + x 2 – x 3 + .......... + (–1) r xr + .........
(b) (1 – x)–1 = 1 + x + x 2 + x 5 + .......... + xr + .........
(c) (1 + x)–2 = 1 – 2x + 3x 2 – 4x 3 + .......... + (–1) r (r + 1) x r + ...........
(d) (1 – x)–2 = 1 + 2x + 3x2 + 4x3 + ............. + (r + 1)x r + ........... 
Solved Example # 20
Prove that the coefficient of xr in (1 – x)–n is n+r–1
Cr
Soltion.
(r + 1)th term in the expansion of (1 – x) –n can be written as

n( n  1)( n  2)......( n  r  1)


T r +1 = (–x)r
r!

RESONANCE 15
n(n  1)(n  2)......(n  r  1)
= (–1)r (–x)r
r!

n(n  1)(n  2)......(n  r  1) r


= x
r!

(n  1)! n(n  1)......(n  r  1) r


= x
(n  1) ! r !

(n  r  1)!
Hence, coefficient of x r is = Cr Proved
(n  1)! r!
n+r–1

Solved Example # 21
If x is so small such that its square and higher powers may be neglected then find the value of

(1  3x)1/ 2  (1  x) 5 / 3
( 4  x )1 / 2
Solution.

(1  3x )1/ 2  (1  x )5 / 3
( 4  x )1/ 2

3 5x
1 x  1
1  2  19 x  
1 / 2
2 3 x
= =   1  
 x 2  6   4
1/ 2
21  
 4

1  2  19 x  1  x  1  2  x  19 x 
=     =  
2  6   8 2  4 6 

x 19 41
=1– – x =1– x Ans.
8 12 24

Self practice problems :

18. Find the possible set of values of x for which expansion of (3 – 2x)1/2 is valid in ascending powers of x.

1.3  2  1.3.5  2 
2 3
3
19. If y = + 2 !   + 3 !   + ............., then find the value of y2 + 2y
5  
5 5

3  5x
20. The coefficient of x100 in is
(1  x )2
(A) 100 (B) –57 (C) –197 (D) 53

 3 3
Ans. (18) x   ,  (19) 4 (20) C
 2 2

RESONANCE 16
Part : (A) Only one correct option

 
15
17
1. In the expansion of  3   3 2  the 11th term is a:
 4 
,

(A) positive integer (B) positive irrational number


(C) negative integer (D) negative irrational number.

 a 
n n
C3
2. If the second term of the expansion a1/ 13   is 14a5/2 then the value of n is:
 a 1  C2
(A) 4 (B) 3 (C) 12 (D) 6

18 3  7 3  3. 18 . 7 . 25
3. The value of, is :
3 6  6 . 243 . 2  15 . 81. 4  20 . 27 . 8  15 . 9 . 16  6 . 3 . 32  64
(A) 1 (B) 2 (C) 3 (D) none

P  Q
5
4. Let the co-efficients of x n in (1 + x)2n & (1 + x) 2n  1 be P & Q respectively, then   =
 Q 
(A) 9 (B) 27 (C) 81 (D) none of these
5. If the sum of the co-efficients in the expansion of (1 + 2 x)n is 6561 , then the greatest term in the
expansion for x = 1/2 is :
(A) 4th (B) 5th (C) 6th (D) none of these
6. Find numerically the greatest term in the expansion of (2 + 3 x)9, when x = 3/2.
(A) 9C6. 29. (3/2)12 (B) 9C3. 29. (3/2)6 (C) 9C5. 29. (3/2) 10 (D) 9C4. 29. (3/2) 8

 
100
1
7. The numbers of terms in the expansion of  a 3  3  1 is
 a 
(A) 201 (B) 300 (C) 200 (D) 100C3
8. The coefficient of x 10 in the expansion of (1 + x2  x3)8 is
(A) 476 (B) 496 (C) 506 (D) 528
9. (1 + x) (1 + x + x ) (1 + x + x + x )...... (1 + x + x +...... + x
2 2 3 2 100
) when written in the ascending power
of x then the highest exponent of x is
(A) 505 (B) 5050 (C) 100 (D) 50

10. 
If x = 7  4 3 
2n
= [x] + f, then x (1  f) =
(A) 2 (B) 0 (C) 1 (D) 2520
11. The remainder when 22003 is divided by 17 is
(A) 1 (B) 2 (C) 8 (D) none of these
12. The last two digits of the number 3 400
are:
(A) 81 (B) 43 (C) 29 (D) 01

 50   50   50   50   50   50  n
13. The value of     +     +...........+     is, where nCr =  
0  1  1 2  49   50  r 

100  100   50   50 
2
(A)   (B)   (C)   (D)  
 50   51   25   25 

 
 10 10   10 CK 
10
 Cr   ( 1)K
14. The value of the expression    2K  is –
 r 0   K 0
(A) 210 (B) 220 (C) 1 (D) 25
15. If |x| < 1, then the co-efficient of xn in the expansion of (1 + x + x2 + x 3 +.......) 2 is
(A) n (B) n  1 (C) n + 2 (D) n + 1

RESONANCE 17
16. The number of values of ' r ' satisfying the equation, 39 C3r 1 39C = 39 Cr 2 1 39 C3r is :
r2
(A) 1 (B) 2 (C) 3 (D) 4
17. Number of elements in set of value of r for which, r  2 + 2.
18C 18C
r1 + 18C  20C13 is satisfied
r
(A) 4 elements (B) 5 elements (C) 7 elements (D) 10 elements
18. The co-efficient of x 5 in the expansion of, (1 + x)21 + (1 + x)22 +....... + (1 + x)30 is :
(A) 51C5 (B) 9C5 (C) 31C6  21C6 (D) 30C5 + 20C5
19. If (1 + x) 10 = a0 + a1x + a2x 2 +......+ a10x 10, then (a0 – a2 + a4 + a6 + a8 – a10)2 + (a1 – a3 + a5 – a7 + a9)2
is equal to
(A) 310 (B) 210 (C) 29 (D) none of these

r .
Cr
10 n
20. The value of is equal to
r 1
n
Cr 1
(A) 5 (2n – 9) (B) 10 n (C) 9 (n – 4) (D) none of these
21. If C0, C1, C2,..........Cn are the Binomial coefficients in the expansion of (1 + x) n. n being even, then
C0 + (C0 + C1) + (C0 + C1 + C2) +......... + (C0 + C1 + C2 +......... + Cn–1) is equal to
(A) n. 2n (B) n. 2n–1 (C) n. 2n–2 (D) n. 2n–3
22. If (1 + x + 2x2)20 = a0 + a1x + a2x 2 +......... + a40x 40, then a0 + a2 + a4.......+ a38 equals
(A) 219 (230 + 1) (B) 219(220 – 1) (C) 220 (219 – 1) (D) none of these
23. Co-efficient of x 15 in (1 + x + x 3 + x 4)n is :

   
5 5 5 3
(A) n
C 5  r . nC 3 r (B) n
C5 r (C) n
C3 r (D) n
C 3  r . nC 5 r
r0 r0 r0

 
r0
40
24. The sum of the coefficients of all the integral powers of x in the expansion of 1 2 x is

1 40 1 40
(A) 340 + 1 (B) 340 – 1 (C) (3 – 1) (D) (3 + 1)
2 2

 3 1001 

25. If { x } denotes the fractional part of ' x ', then   =
 82 
(A) 9/82 (B) 81/82 (C) 3/82 (D) 1/82

 
10
 x 1 x 1 
  
26. The coefficient of the term independent of x in the expansion of  2 1 1
 is
 x  x 1 x  x
3 3 2 
(A) 70 (B) 112 (C) 105 (D) 210
27. The coefficient of xn in polynomial (x + 2n+1C0) (x + 2n+1C1) (x + 2n+1C2).......(x + 2n+1Cn) is
(A) 2n+1 (B) 22n+1 – 1 (C) 22n (D) none of these
28. In the expansion of (1 + x) n (1 + y) n (1 + z)n, the sum of the co-efficients of the terms of degree ' r ' is :

(A) n C r (B) n C
3
(C) 3nCr (D) 3. 2nCr
r3

 
 r 1 
n
 n
C r Cp 2p 
r
29.   is equal to
r 1  p 0 
(A) 4n – 3n + 1 (B) 4n – 3n – 1 (C) 4n – 3n + 2 (D) 4n – 3n
30. If (1 + x)n = C0 + C1x + C2x² +.... + Cn x n, then show that the sum of the products of the Ci’s taken two at
  Ci C j
a time, represented by is equal to
0i j n
2n ! 2 n! 2n ! 2 n!
(A) 22n  1  2 (n ! ) (B) 22n  1  (C) 22n1  2(n ! ) (D) 22n 1 
2 (n !) 2 2 (n !) 2

RESONANCE 18
Part : (B) May have more than one options correct
31. In the expansion of (x + y + z)25
(A) every term is of the form 25Cr. rCk. x 25 – r. yr – k. zk
(B) the coefficient of x8 y9 z9 is 0
(C) the number of terms is 325
(D) none of these
32. 79 + 97 is divisible by
(A) 16 (B) 24 (C) 64 (D) 72

 2 
8
 log5 4 x  44 1 
1. Find the value of ' x ' for which the fourth term in the expansion,  5 5   is 336.
 3 2 x1  7 
 5log5 
n
3 1 
2. In the binomial expansion of  2  3  , the ratio of the 7th term from the begining to the 7th term
 3
from the end is 1 : 6 ; find n.

3 2 1 
9

3. Find the terms independent of 'x' in the expansion of the expression,(1 + x + 2 x 3) 


 2 x  3 x  .
 
4. If in the expansion of (1  x) 2n  1 ,the co-efficient of x is denoted by ar, then prove that ar  1 + a2n  r = 0.
r


5
 6
10
5. Show that the term independent of x in the expansion of 1  x   is, 1 + 10C 2r C r 6 r.
 x
2r
r 1

6. Find the coefficient of a5 b4 c7 in the expansion of (bc + ca  ab) 8.


7. If (1 + 2x + 3x 2)10 = a0 + a1x + a2x 2 +.... + a20x 20, then calculate a1, a2, a4.

8.  
If 3 3  5 = p+ f, where p is an integer and f is a proper fraction then find the value of
n

3 
3  5 , n N.
n

9. Write down the binomial expansion of (1 + x) n + 1, when x = 8. Deduce that 9n + 1 – 8n – 9 is divisible by


64, whenever n is a positive integer.
10. Prove that 5353 – 3333 is divisible by 10.
11. Which is larger : (9950 + 10050) or (101)50.
If C0, C1, C2,........, Cn are the combinatorial co-efficients in the expansion of (1 + x)n, n  N, then prove
the followings: (Q. No. 12 - 14)

2 2.C1 23.C 2 2 4.C 3 2n  1.Cn 3n  1 1


12. 2. Co +    ...... 
2 3 4 n1 n 1

C1 C C Cn n (n  1)
13. + 2. 2 + 3 3 +........ + n =
C0 C1 C2 Cn  1 2
14. 1². C0 + 2². C1 + 3². C 2 + 4². C3 +.... + (n+1)² Cn = 2n2 (n+1) (n+4).

 1  34 x 
15. Assuming ' x ' to be so small that x 2 and higher powers of ' x ' can be neglected, show that,

(16  3 x)1/ 2
4
305
is approximately equal to, 1  x.
(8  x ) 2/3 96

 1 

n 3r 7r
16. If ( 1) r. nC     ........ to m terms  = k 1  1  , then find the value of k.
 2 2 2   2m n 
r r 2r 3r
r0
17. Find the coefficient of x 50 in the expression:
(1 + x) 1000 + 2x. (1 + x)999 + 3x² (1 + x)998 +..... + 1001 x 1000

RESONANCE 19
q  1  q  1  q  1
2 n

18. Given sn= 1 + q + q² +..... + & Sn = 1 + +


qn  +.... +   , q  1,
2  2   2 
prove that n+1C1 + n+1C2.s 1 + n+1C3.s 2 +....+ n+1Cn+1.s n = 2n. Sn.

19. Show that if the greatest term in the expansion of (1 + x)2n has also the greatest co-efficient, then ' x '
n n1
lies between, & .
n1 n
32
20. Find the remainder when 3232 is divided by 7.

21. If (1 + x + x² +... + xp)n = a0 + a1x + a2x²+...+anp. x np, then find the value of :
a1 + 2a2 + 3a3 +.... + np. anp.

( 4n  1) !
22. Prove that, (2nC1)²+ 2. (2nC2)² + 3. ( 2nC3)² +... + 2n. (2nC2n)² =
{(2n  1) ! } 2

23. If (1+x)n = C0 + C1x + C2x² +..... + Cn x n, then show that:


C2 C3 1
C1 (1x)  (1  x)² + (1  x)3 ........ + ( 1)n1 (1  x)n
2 3 n
1 1 1
= (1  x) + (1  x²) + (1  x 3) +........ + (1  x n)
2 3 n

r
n
24. Prove that 2 n
C r pr qn – r = npq + n2p2 if p + q = 1.
r 0

  b ( x  3)
2n 2n
25. If a r ( x  2)r  r
r
& ak = 1 for all k  n, then show that bn = 2n+1C
n+1.
r 0 r 0

26. If a0, a1, a2,..... be the coefficients in the expansion of (1 + x + x²)n in ascending powers of x, then prove
that :
(i) a0 a1  a1 a2 + a2 a3 .... = 0
(ii) a0a2  a1a3 + a2a4 ..... + a2n  2 a2n = an + 1
(iii) E1 = E2 = E3 = 3n1; where E1= a0 + a3 + a6 +...; E2 = a1 + a4 + a7 +...& E3 = a2 + a5 + a8 +...

27. If (1 + x) n = p0 + p1 x + p2 x 2 + p3 x 3 +......., then prove that :


n n
(a) p0  p2 + p4 ....... = 2n/2 cos (b) p1  p3 + p5 ....... = 2n/2 sin
4 4
28. If (1+x) n = C0 + C1x + C2x² +.... + Cn x n, then show that the sum of the products of the Ci’s taken two at
  Ci C j 2n !
a time, represented by is equal to 22n1  .
0i jn 2 (n ! ) 2

EXERCISE # 1 EXERCISE # 2
17
1. B 2. A 3. A 4. D 5. B 6. A 7. A 1. x = 0 or 1 2. n = 9 3.
54
8. A 9. B 10. C 11. C 12. D 13. B 14. C 6. 280 7. a1 = 20, a2 = 210, a4 = 8085
15. D 16. B 17. C 18. C 19. B 20. A 21. B 8. 1 – f, if n is even and f, if n is odd

22. B 23. A 24. D 25. C 26. D 27. C 28. C 1


11. 10150 16. 17. 1002C
2 1
n 50

29. D 30. B 31. AB 32. AC


np
20. 4 21. (p + 1)n
2

RESONANCE 20
Complex Numbers

1. The complex number system


There is no real number x which satisfies the polynomial equation
x 2 + 1 = 0. To permit solutions of this and similar equations, the
set of complex numbers is introduced.
We can consider a complex number as having the form a + bi

Theshortest path where a and b are real number and i, which is called the imaginary
unit, has the property that i2 = – 1.
It is denoted by z i.e. z = a + ib. ‘a’ is called as real part of z
between two truths in which is denoted by (Re z) and ‘b’ is called as imaginary part of
z which is denoted by (Im z).
the real domain passes Any complex number is :
through the (i) Purely real, if b = 0 ;
(ii) Purely imaginary, if a = 0
complex domain. (iii) Imaginary, if b  0.

NOTE :
......Jacques Hadamard (a) The set R of real numbers is a proper subset of the
Complex Numbers. Hence the complete number system
is N  W  I  Q  R  C.

Mathematics is the
(b) Zero is purely real as well as purely imaginary but not
imaginary.
(c) i = 1 is called the imaginary unit.
queen of the sciences
Also i² =  1; i3 =  i ; i4 = 1 etc.
and
(d) a b = a b only if atleast one of a or b is non -
number theory is the
negative.
queen of mathematics (e) is z = a + ib, then a – ib is called complex conjugate of
z and written as z = a – ib

.....CarlFriedrichGauss
Self Practice Problems

1. Write the following as complex number


(i)  16 (ii) x , (x > 0)

(iii) –b +  4ac , (a, c> 0)

Ans. (i) 0 + i 16 (ii) x + 0i (iii) –b + i 4ac

2. Write the following as complex number


(i) x (x < 0) (ii) roots of x 2 – (2 cos)x + 1 = 0

RESONANCE 21
2. Algebraic Operations:
Fundamental operations with complex numbers
In performing operations with complex numbers we can proceed as in the algebra of real numbers,
replacing i2 by – 1 when it occurs.
1. Addition (a + bi) + (c + di) = a + bi + c + di = (a + c) + (b + d) i
2. Subtraction (a + bi) – c + di) = a + bi – c – di = (a – c) + (b – d) i
3. Multiplication (a + bi) (c + di) = ac + adi + bci + bdi2 = (ac – bd) + (ad+ bc)i

a  bi a  bi c  bi ac  adi  bci  bdi 2


4. Division = . =
c  di c  di c  di c 2  d2i2

ac  bd  (bc  ad)i ac  bd bc  ad
= = + i
c d
2 2
c d
2 2
c 2  d2

Inequalities in complex numbers are not defined. There is no validity if we say that complex number is
positive or negative.
e.g. z > 0, 4 + 2i < 2 + 4 i are meaningless.
In real numbers if a2 + b2 = 0 then a = 0 = b however in complex numbers,
z12 + z22 = 0 does not imply z1 = z2 = 0.

Example : Find multiplicative inverse of 3 + 2i.


Solution Let z be the multiplicative inverse of 3 + 2i. then
 z . (3 + 2i) = 1
1 3  2i
 z= = 3  2i 3  2i
3  2i
3 2
 z= – i
13 13
 3 2 
  i Ans.
 13 13 

Self Practice Problem

1. Simplify in+100 + in+50 + in+48 + in+46 , n  .


Ans. 0

3. Equality In Complex Number:


Two complex numbers z1 = a1 + ib1 & z2 = a2 + ib2 are equal if and only if their real and imaginary parts
are equal respectively
i.e. z 1 = z2  Re(z1) = Re(z2) and m (z1) = m (z2).

Example: Find the value of x and y for which (2 + 3i) x 2 – (3 – 2i) y = 2x – 3y + 5i where x, y  R.
Solution (z + 3i)x2 – (3 – 2i)y = 2x – 3y + 5i
 2x2 – 3y = 2x – 3y
 x2 – x = 0
 x = 0, 1 and 3x2 + 2y = 5
5
 if x = 0,y = and if x = 1, y = 1
2
5
 x = 0, y = and x = 1, y = 1
2
 5
are two solutions of the given equation which can also be represented as  0,  & (1, 1)
 2
 5
 0,  , (1, 1) Ans.
 2

RESONANCE 22
Example: Find the value of expression x 4 – 4x3 + 3x 2 – 2x + 1 when x = 1 + i is a factor of expression.
Solution. x=1+i
 x–1=i
 (x – 1)2 = –1
 x 2 – 2x + 2 = 0
Now x 4 – 4x3 + 3x2 – 2x + 1
= (x2 – 2x + 2) (x2 – 3x – 3) – 4x + 7
 when x = 1 + i i.e. x2 – 2x + 2 = 0
x – 4x + 3x – 2x + 1 = 0 – 4 (1 + i) + 7
4 3 2

= –4 + 7 – 4i
= 3 – 4i
3 – 4i Ans.

Example: Solve for z if z2 + |z| = 0


Solution. Let z= x + iy

 (x + iy) 2 + x2  y2 = 0

 x 2 – y2 + x 2  y 2 = 0 and 2xy = 0
 x = 0 or y = 0
when x = 0 – y2 + | y | = 0
 y = 0, 1, –1
 z = 0, i, –i
when y = 0 x2 + | x | = 0
 x=0  z=0
Ans. z = 0, z = i, z = – i

Example: Find square root of 9 + 40i


Solution. Let (x + iy)2 = 9 + 40i
 x 2 – y2 = 9 ...............(i)
and xy = 20 ...............(ii)
squing (i) and adding with 4 times the square of (ii)
we get x 4 + y4 – 2x2 y2 + 4x 2 y2 = 81 + 1600
 (x 2 + y2)2 = 168
 x 2 + y2 = 4 ...............(iii)
from (i) + (iii) we get x2 = 25  x=±5
and y = 16  y=±4
from equation (ii) we can see that
x & y are of same sign
 x + iy = +(5 + 4i) or = (5 + 4i)
 sq. roots of a + 40i = ± (5 + 4i)
Ans. ± (5 + 4i)

Self Practice Problem

1. Solve for z : z = i z2

3 1
Ans. ± – i, 0, i
2 2

RESONANCE 23
4. Representation Of A Complex Number:
(a) Cartesian Form (Geometric Representation) :
Every complex number z = x + i y can be represented by a point on the Cartesian plane
known as complex plane (Argand diagram) by the ordered pair (x, y).

Length OP is called modulus of the complex number which is denoted by z &  is called the
argument or amplitude.
y
z = x 2  y 2 &  = tan1 (angle made by OP with positive xaxis)
x
NOTE :
(i) Argument of a complex number is a many valued function. If is the argument of a complex
number then 2 n + ; n  I will also be the argument of that complex number. Any two
arguments of a complex number differ by 2n

(ii) The unique value of  such that   <   is called the principal value of the argument.
Unless otherwise stated, amp z implies principal value of the argument.
(iii) By specifying the modulus & argument a complex number is defined completely. For the complex
number 0 + 0 i the argument is not defined and this is the only complex number which is only
given by its modulus.

(b) Trignometric/Polar Representation :


z = r (cos  + i sin ) where z = r; arg z = ; z = r (cos  i sin )

NOTE : cos  + i sin  is also written as CiS or ei .


e ix e ix e ix e ix
Also cos x = & sin x = are known as Euler's identities.
2 2
(c) Euler's Representation :
z = rei; z = r; arg z = ; z = re i

(d) Vectorial Representation :


Every complex number can be considered as if it is the position vector of a point. If the point

P represents the complex number z then, OP = z &  OP  = z


 

Example: Express the complex number z = – 1 + 2 i in polar form.


Solution. z = –1 + i 2

|z|= ( 1)2   2
2
= 1 2 = 3

 2
 
Arg z =  – tan–1  1  =  – tan–1 2 =  (say)
 

 z= 3 (cos  + i sin  ) where  =  – tan–1 2

RESONANCE 24
Self Practice Problems

1. Find the principal argument and |z|


1 (9  i)
z=
2i

17 82
Ans. – tan–1 ,
11 5

2. Find the |z| and principal argument of the complex number z = 6(cos 310º – i sin 310°)
Ans. 6, 50°

5. Modulus of a Complex Number :


If z = a + ib, then it's modulus is denoted and defined by |z| = a 2  b 2 . Infact |z| is the distance
of z from origin. Hence |z1 – z2| is the distance between the points represented by z1 and z2.

Properties of modulus
(i) |z1z2| = |z1| . |z2|

z1 z1
(ii) z2 = z (provided z2  0)
2

(iii) |z1 + z2|  |z1| + |z2|


(iv) |z1 – z2|  ||z1| – |z2||

(Equality in (iii) and (iv) holds if and only if origin, z1 and z2 are collinear with z1 and z2 on the same side
of origin).

Example: If |z – 5 – 7i| = 9, then find the greatest and least values of |z – 2 – 3i|.
Solution. We have 9 = |z – (5 + 7i)| = distance between z and 5 + 7i.
Thus locus of z is the circle of radius 9 and centre at 5 + 7i. For such a z (on the circle), we
have to find its greatest and least distance as from 2 + 3i, which obviously 14 and 4.

Example: Find the minimum value of |1 + z| + |1 – z|.


Solution |1 + z| + |1 – z|  |1 + z + 1 – z| (triangle inequality)
 |1 + z | + |1 – z|  2
 minimum value of (|1 + z| + |1 – z|) = 2
Geometrically |z + 1| + |1 – 2| = |z + 1| + |z – 1| which represents sum of distances of z from
1 and – 1
it can be seen easily that minimu (PA + PB) = AB = 2
 
1  n 
Ans. 21/ 4
e 8 

2
Example: z = 1 then find the maximum and minimum value of |z|
z

2 2 2 2
Solution. z =1 |z|  z |z|+ 
z z 2 z
Let | z | = r

2 2
 r 1  r+
r r

RESONANCE 25
2
r+ 1  r  R+ ..............(i)
r

2 2
and r  1 –1  r – 1
r r
 r  (1, 2) ..............(ii)
 from (i) and (ii) r  (1, 2)
Ans. r  (1, 2)

Self Practice Problem

1. |z – 3| < 1 and |z – 4i| > M then find the positive real value of M for which these exist at least one
complex number z satisfy both the equation.
Ans. M  (0, 6)

6. Agrument of a Complex Number :


Argument of a non-zero complex number P(z) is denoted and defined by arg(z) = angle which OP
makes with the positive direction of real axis.
If OP = |z| = r and arg(z) = , then obviously z = r(cos + isin), called the polar form of z. In what
follows, 'argument of z' would mean principal argument of z(i.e. argument lying in (–, ] unless the
context requires otherwise. Thus argument of a complex number z = a + ib = r(cos + isin) is the value
of  satisfying rcos = a and rsin = b.

b
Thus the argument of z = ,  – , –  + , – ,  = tan–1 , according as z = a + ib lies in , , 
a
or Vth quadrant.

Properties of arguments
(i) arg(z1z2) = arg(z1) + arg(z2) + 2m for some integer m.
(ii) arg(z1/z2) = arg (z1) – arg(z2) + 2m for some integer m.
(iii) arg (z2) = 2arg(z) + 2m for some integer m.
(iv) arg(z) = 0  z is real, for any complex number z  0
(v) arg(z) = ± /2  z is purely imaginary, for any complex number z  0
(vi) arg(z2 – z1) = angle of the line segment
PQ || PQ, where P lies on real axis, with the real axis.

 2
Example: Solve for z, which satisfy Arg (z – 3 – 2i) = and Arg (z – 3 – 4i) = .
6 3
Solution From the figure, it is clear that there is no z, which satisfy both ray

RESONANCE 26
Example: Sketch the region given by
(i) Arg (z – 1 – i) /3
(ii) |z| =  5 & Arg (z – i – 1) >/3

Solution (i) (ii)

Self Practice Problems

1. Sketch the region given by


(i) |Arg (z – i – 2)| < /4 (ii) Arg (z + 1 – i) /6

2. Consider the region |z – 15i|  10. Find the point in the region which has
(i) max |z| (ii) min |z|
(iii) max arg (z) (iv) min arg (z)

7. Conjugate of a complex Number

Conjugate of a complex number z = a + b is denoted and defined by z = a – ib.


In a complex number if we replace i by – i, we get conjugate of the complex number. z is the mirror
image of z about real axis on Argand's Plane.

Properties of conjugate

(i) |z| = | z |
(ii) z z = |z|2
(iii) ( z1  z 2 ) = ( z1 ) + ( z 2 )

(iv) ( z1  z 2 ) = ( z1 ) – ( z 2 )

(v) ( z1 z 2 ) = z1 z 2

 z1  ( z1 )
  =
(vi)
 z2  ( z 2 ) (z2  0)

(vii) |z1 + z2|2 = (z1 + z2) ( z1  z 2 ) = |z1|2 + |z2|2 + z1 z 2 + z1 z2

(viii) ( z1 ) = z
(ix) If w = f(z), then w = f( z )
(x) arg(z) + arg( z ) = 0

RESONANCE 27
z 1
Example: If is purely imaginary, then prove that | z | = 1
z 1
 z  1
Solution. Re   =0
 z  1
z 1  z  1
 +   =0

z 1  z  1
z 1 z 1
 + =0
z 1 z 1
 zz – z + z – 1 + zz – z + z – 1 = 0
 zz = 1
 | z |2 = 1
 |z|=1 Hence proved

Self Practice Problem


z1  2 z 2
1. If is unmodulus and z2 is not unimodulus then find |z1|.
2  z1z 2
Ans. |z1| = 2

8. Rotation theorem
(i) If P(z1) and Q(zz) are two complex numbers such that |z1| = |z2|, then z2 = z1 eiwhere  = POQ
(ii) If P(z1), Q(z2) and R(z3) are three complex numbers and PQR = , then

 z3  z2  z3  z2
  =
z1  z 2 e
i
 z1  z 2 

(iii) If P(z1), Q(z2), R(z3) and S(z4) are four complex numbers and STQ = , then

z3  z 2 z3  z 4
z1  z 2 = z1  z 2 e
i

 z  1 
Example: If arg   = then interrupter the locus.
 z  i  3

 z  1 
Solution arg   =
 z  i  3

 1 z  
 arg   =
  1  z  3
 1 z 
Here arg   represents the angle between lines joining –1 and z and 1 + z. As this angle
  1 z 
is constant, the locus of z will be a of a circle segment. (angle in a segment is count). It can be
 1 z  2
seen that locus is not the complete side as in the major are arg   will be equal to – .
  1 z  3
Now try to geometrically find out radius and centre of this circle.

 1  2
centre   0, 
 Radius  Ans.
 3 3

RESONANCE 28
Example: If A(z + 3i) and B(3 + 4i) are two vertices of a square ABCD (take in anticlock wise order) then
find C and D.
Solution. Let affix of C and D are z3 + z4 respectively
Considering DAB = 90º + AD = AB

z 4  (2  3 i) (3  4 i)  (2  3 i) i
we get = e
AD AB 2
 z4 – (2 + 3i) = (1 + i) i
 Z4 = 2 + 3i+ i – 1 = 1 + zi

z 3  (3  4i) ( z  3i)  (3  4i) i


and = e–
CB AB 2
 z3 = 3 + 4i – (1 + i) (–i)
z3 = 3 + 4i + i – 1 = z + 5i

Self Practice Problems

1. z1, z2, z3, z4 are the vertices of a square taken in anticlockwise order then prove that
2z2 = (1 + i) z1 + (1 – i) z3
Ans. (1 + i) z1 + (1 – i)z3

2. Check that z1z2 and z3z4 are parallel or, not


where, z1 = 1 + i z3 = 4 + 2i
z2 = 2 – i z4 = 1 – i
Ans. Hence, z1z2 and z3z4 are not parallel.

3. P is a point on the argand diagram on the circle with OP as diameter “two point Q and R are taken such
that POQ = QOR
If O is the origin and P, Q, R are represented by complex z1, z2, z3 respectively then show that
z22 cos 2 = z1z3cos2
Ans. z1z3 cos 2

9. Demoivre’s Theorem:
Case 

Statement :
If n is any integer then
(i) (cos  + i sin  )n = cos n + i sin n
(ii) (cos 1 + i sin 1) (cos 2) + i sin 2) (cos3 + i sin 2) (cos 3 + i sin 3) .....(cos n + i sin n)
= cos (1 + 2 + 3 + ......... n) + i sin (1 + 2 + 3 + ....... + n)

Case 
Statement : If p, q  Z and q  0 then

 2k  p   2k  p 
(cos  + i sin )p/q = cos   + i sin  
 q   q 
where k = 0, 1, 2, 3, ......, q – 1

NOTE : Continued product of the roots of a complex quantity should be determined using theory of equations.

RESONANCE 29
1 0 . Cube Root Of Unity :
(i) The cube roots of unity are 1,  1  i 3 ,  1  i 3 .
2 2
(ii) If  is one of the imaginary cube roots of unity then 1 +  + ² = 0. In general 1 + r + 2r = 0;
where r  I but is not the multiple of 3.
(iii) In polar form the cube roots of unity are :
2 2 4 4
cos 0 + i sin 0; cos + i sin , cos + i sin
3 3 3 3
(iv) The three cube roots of unity when plotted on the argand plane constitute the verties of an
equilateral triangle.
(v) The following factorisation should be remembered :
(a, b, c  R &  is the cube root of unity)
a3  b3 = (a  b) (a  b) (a  ²b) ; x2 + x + 1 = (x  ) (x  2) ;
a3 + b3 = (a + b) (a + b) (a +  2b) ; a2 + ab + b2 = (a – bw) (a – bw2)
a3 + b3 + c3  3abc = (a + b + c) (a + b + ²c) (a + ²b + c)
Example: Find the value of 192 + 194
Solution. 192 + 194
= 1 + 2 =–
Ans. –
Example: If 1, , 2 are cube roots of unity prove
(i) (1 –  + 2) (1 +  – 2) = 4
(ii) (1 –  + 2)5 + (1 +  – 2)5 = 32
(iii) (1 – ) (1 – 2) (1 –4) (1 – 8) = 9
(iv) (1 –  + 2) (1 – 2 + 4) (1 – 4 + 8) .......... to 2n factors = 22n
Solution. (i) (1 –  + 2) (1 +  – 2)
= ( – 2) ( – 22)
=4
Self Practice Problem

 (1  r  2r )
10
1. Find
r 0

Ans. 12

1 1 . n th Roots of Unity :
If 1, 1, 2, 3..... n  1 are the n, nth root of unity then :

(i) They are in G.P. with common ratio ei(2/n) &

 1 +  2 +.... + n  1 = 0 if p is not an integral multiple of n


p p p
(ii) 1p +
= n if p is an integral multiple of n

(iii) (1  1) (1  2)...... (1  n  1) = n &


(1 + 1) (1 + 2)....... (1 + n  1) = 0 if n is even and 1 if n is odd.

(iv) 1. 1. 2. 3......... n  1 = 1 or 1 according as n is odd or even.

RESONANCE 30
Example: Find the roots of the equation z6 + 64 = 0 where real part is positive.
Solution. z6 = – 64
z6 = z6 . e+ i(2n + 1) xz

i( 2n1)
 z=z e 6

   5 7 3 11
i i i i i i i
 z=2 e 6 , 2e 2 , ze 2 , ze 6
= e 6
, ze 2 , ze 2

i 11
i
 roots with +ve real part are = e 6 + e 6

 
i  
2e  6
Ans.

  sin
6
 2k 2k 
Example: Find the value  cos 
k 1 7 7 

   cos
 2k   2k 
6 6

Solution.  sin  – 
k 1  7  k 1
7 

  cos
2k 2k
6 6

= sin
k 0
7 – k 0
7 +1


6

= (Sum of imaginary part of seven seventh roots of unity)


k 0


6

– (Sum of real part of seven seventh roots of unity) + 1


k 0

= 0–0+1=1
i Ans.
Self Practice Problems

1. Resolve z7 – 1 into linear and quadratic factor with real coefficient.

 2 2   4   6 
Ans. (z – 1)  z  2 cos z  1 .  z 2  2 cos z  1 .  z 2  2 cos z  1
 7   7   7 

2 4 6
2. Find the value of cos + cos + cos .
7 7 7

1
Ans. –
2

1 2 . The Sum Of The Following Series Should Be Remembered :

sin  n / 2  n  1
(i) cos
 
 + cos 2  + cos 3  +..... + cos n  = sin  / 2 cos   
2 

sin  n / 2  n  1
(ii) sin
 
 + sin 2  + sin 3  +..... + sin n  = sin  / 2 sin   
2 

NOTE : If  = (2/n) then the sum of the above series vanishes.

RESONANCE 31
1 3 . Logarithm Of A Complex Quantity :
1  1  
(i) Loge (+ i  ) = Loge (² +  ²) + i  2 n   tan  where n  .
2  

, n  .
 
 2 n   
(ii) ii represents a set of positive real numbers given by e  2

Example: Find the value of



(i) log (1 + 3 i) Ans. log2 + i(2n + )
3
(ii) log(–1) Ans. i
(iii) zi Ans. cos(ln2) + i sin(ln2) = ei(ln2)

( 4n 1).
(iv) ii Ans. e 2


( 8n 1).
(v) |(1 + i) i | Ans. e 4

1
(vi) arg ((1 + i)i) Ans.
2
n(2).

 i    2n  
Solution. (i) log (1 + 3 i) = log  2 e  3 

 
 

 
= log 2 + i   2n 
3 
(iii) 2i = ein 2 = cos (n 2) cos (n 2) + i sin (n 2) ]

Self Practice Problem

1. Find the real part of cos (1 + i)

1 e2
Ans.
2ei

1 4 . Geometrical Properties :
Distance formula :
If z1 and z2 are affixies of the two points P and Q respectively then distance between P + Q is given
by |z1 – z2|.
Section formula
If z1 and z2 are affixes of the two points P and Q respectively and point C devides the line joining P and
Q internally in the ratio m : n then affix z of C is given by
mz 2  nz1
z=
mn
If C devides PQ in the ratio m : n externally then
mz 2  nz1
z=
mn
(b) If a, b, c are three real numbers such that az1 + bz2 + cz3 = 0 ; where a + b + c = 0 and a,b,c
are not all simultaneously zero, then the complex numbers z1, z2 & z3 are collinear.

RESONANCE 32
(1) If the vertices A, B, C of a  represent the complex nos. z1, z2, z3 respectively and
a, b, c are the length of sides then,

z1  z 2  z 3
(i) Centroid of the  ABC = :
3
(ii) Orthocentre of the  ABC =
asec A z1  b sec B z 2  csecCz 3 z1tan A  z 2 tanB  z 3 tan C
asec A  bsec B  c secC or tanA  tan B  tanC
(iii) Incentre of the  ABC = (az1 + bz2 + cz3)  (a + b + c).

(iv) Circumcentre of the  ABC = :


(Z1 sin 2A + Z 2 sin 2B + Z 3 sin 2C)  (sin 2A + sin 2B + sin 2C).

(2) amp(z) =  is a ray emanating from the origin inclined at an angle  to the x axis.

(3) z  a = z  b is the perpendicular bisector of the line joining a to b.

(4) The equation of a line joining z1 & z2 is given by, z = z1 + t (z1  z2) where t is a real parameter.

(5) z = z1 (1 + it) where t is a real parameter is a line through the point z1 & perpendicular to the
line joining z1 to the origin.

(6) The equation of a line passing through z1 & z2 can be expressed in the determinant form as

z z 1
z1 z1 1 = 0. This is also the condition for three complex numbers to be collinear.. The above
z 2 z2 1
equation on manipulating, takes the form  z   z  r = 0 where r is real and  is a non zero
complex constant.

NOTE : If we replace z by zei and z by ze – i then we get equation of a straight line which. Passes through the
foot of the perpendicular from origin to given straight line and makes an angle with the given straightl
line.

(7) The equation of circle having centre z0 & radius  is :


z  z0 =  or z z  z0 z  z 0 z + z 0 z0  ² = 0 which is of the form

z z   z   z + k = 0, k is real. Centre is  & radius =    k .


Circle will be real if    k  0..
(8) The equation of the circle described on the line segment joining z1 & z2 as diameter is
z  z2 
arg =± or (z  z1) ( z  z 2) + (z  z2) ( z  z 1) = 0.
z  z1 2
(9) Condition for four given points z1, z2, z3 & z4 to be concyclic is the number
z 3  z1 z 4  z 2
. should be real. Hence the equation of a circle through 3 non collinear
z 3  z 2 z 4  z1

 z  z 2   z 3  z1 
 z  z1   z 3  z 2 
points z1, z2 & z3 can be taken as is real

 z  z 2   z 3  z1   z  z2   z3  z1 

 z  z1   z 3  z 2   z  z1   z3  z 2 
= .

RESONANCE 33
 z  z1 
(10) Arg  z  z  = represent (i) a line segment if  = 
 2

(ii) Pair of ray if  = 0 (iii) a part of circle, if 0 <  < 

z1 z1 1
1
(11) Area of triangle formed by the points z1, z2 & z3 is z2 z2 1
4i
z3 z3 1

|  z 0  z0  r |
(12) Perpendicular distance of a point z0 from the line  z   z  r  0 is 2||


(13) (i) Complex slope of a line  z   z  r  0 is  = – .

z1  z 2
(ii) Complex slope of a line joining by the points z1 & z2 is  = z  z
1 2

(iii) Complex slope of a line making  angle with real axis = e2i

(14) 1 & 2 are the compelx slopes of two lines.


(i) If lines are parallel then 1 = 2
(ii) If lines are perpendicular then 1 + 2 = 0

(15) If |z – z1| + |z – z2| = K > |z1 – z2| then locus of z is an ellipse whose focii are z1 & z2

 z  z  r
(16) If |z – z0| = 2|| then locus of z is parabola whose focus is z0 and directrix is the

line  z 0 +  z 0 + r = 0

z  z1
(17) If z  z2 = k  1, 0, then locus of z is circle.

(18) If z – z1  – z – z2 = K < z1 – z2 then locus of z is a hyperbola, whose focii are

z 1 & z2 .

Match the following columns :


Column -  Column - 
(i) If | z – 3+2i | – | z + i | = 0, (i) circle
then locus of z represents ..........

 z  1 
(ii) If arg   = , (ii) Straight line
 z  1 4
then locus of z represents...
(iii) if | z – 8 – 2i | + | z – 5 – 6i | = 5 (iii) Ellipse
then locus of z represents .......

 z  3  4i  5
(iv) If arg  z  2  5i  = , (iv) Hyperbola
  6
then locus of z represents .......

RESONANCE 34
(v) If | z – 1 | + | z + i | = 10 (v) Major Arc
then locus of z represents ........
(vi) |z–3+i|–|z+2–i|=1 (vi) Minor arc
then locus of z represents .....
(vii) | z – 3i | = 25 (vii) Perpendicular bisector of a line segment

 z  3  5i 
(viii) arg  z  i  =  (viii) Line segment
 
Ans.  (i) (ii) (iii) (iv) (v) (vi) (vii) (viii)
 (vii) (v) (viii) (vi) (iii) (iv) (i) (ii)

15. (a) Reflection points for a straight line :


Two given points P & Q are the reflection points for a given straight line if the given line is the
right bisector of the segment PQ. Note that the two points denoted by the complex
numbers z1 & z2 will be the reflection points for the straight line  z   z  r  0 if and only if;

 z1   z 2  r  0 , where r is real and is non zero complex constant.

(b) Inverse points w.r.t. a circle :


Two points P & Q are said to be inverse w.r.t. a circle with centre 'O' and radius , if:
(i) the point O, P, Q are collinear and P, Q are on the same side of O.
(ii) OP. OQ = 2.

Note : that the two points z1 & z2 will be the inverse points w.r.t. the circle z z   z   z  r  0 if and only

if z1 z 2   z1   z 2  r  0 .

1 6 . Ptolemy’s Theorem:
It states that the product of the lengths of the diagonals of a convex quadrilateral inscribed in a circle
is equal to the sum of the products of lengths of the two pairs of its opposite sides.
i.e. z1  z3 z2  z4 = z1  z2 z3  z4 + z1  z4 z2  z3

Example: If cos  + cos  + cos  = 0 and also sin  + sin  + sin  = 0, then prove that
(i) cos 2 + cos2 + cos2 = sin 2 + sin 2 + sin 2 = 0
(ii) sin 3 + sin 3 + sin 3 = 3 sin ()
(iii) cos 3 + cos 3 + cos 3 = 3 cos ()
Solution. Let z1 = cos  + i sin , z2 = cos  + i sin ,
z3 = cos + i sin 
 z 1 + z2 + z3 = (cos  + cos  + cos ) + i (sin  + sin  + sin )
=0+i.0=0 (1)
1
(i) Also z1 = (cos  + i sin ) = cos  – i sin 
–1

1 1
z2 = cos  – i sin , z 3 – cos  – sin 

1 1 1
 z1 + z 2 + z 3 = (cos  + cos  + cos ) – i (sin  + sin  + sin ) (2)

= 0–i.0=0

RESONANCE 35
Now z12 + z22 + z33 = (z1 + z2 + z3)2 – 2 (z1z2 + z2z3 + z3z1 )

 1 1 1 
= 0 – 2z1z2z3  z  z  z 
 3 1 2 

= 0 – 2z1 z2 z3. 0 = 0, using (1) and (2)


or (cos  + i sin )2 + (cos  + i sin )2 + (cos  + i sin )2 = 0
or cos 2 + i sin 2)2 + cos 2 + i sin 2 + cos 2 + i sin 2 = 0 + i.0
Equation real and imaginary parts on both sides, cos 2 + cos 2 + cos 2 = 0 and
sin 2 + sin 2 + sin 2 = 0
(ii) z 1 3 + z 2 3 + z3 3 = (z1 + z2)3 – 3z1z2(z1 + z2) + z33
= (–z3)3 – 3z1z2 (– z3) + z33, using (1)
= 3z1z2z3
 (cos  + i sin )3 + (cos  + i sin )3 + (cos  + i sin )3
= 3 (cos  + i sin ) (cos  + i sin ) (cos  + i sin )
or cos 3 + i sin 3 + cos 3 + i sin 3 + cos 3 + i sin 3
= 3{cos( +  + ) + i sin ( +  + )
Equation imaginary parts on both sides, sin 3 + sin 3 + sin 3 = 3 sin ( +  + )
Alternative method
Let C  cos  + cos  + cos  = 0
S  sin  + sin  + sin  = 0
C + iS = ei + ei + ei = 0 (1)
C – iS = e–i + e–i + e–i = 0 (2)
From (1)  (e–i)2 + (e–i )2 + (e–i)2 = (ei) (ei ) + (ei ) (ei) + (ei) (ei)
 ei2 + ei2 + ei2 = ei ei ei (e–2 + e–i + ei )
 ei(2) + ei2 + ei2 = 0 (from 2)
Comparing the real and imaginary parts we
cos 2 + cos 2 + cos 2 – sin 2 + sin 2 + sin 2 = 0
Also from (1) (ei)3 + (ei )3 + (ei)3 = 3ei ei ei
 ei3 + ei3 + ei3 = 3ei()
Comparing the real and imaginary parts we obtain the results.

Example: If z1 and z2 are two complex numbers and c > 0, then prove that
|z1 + z2|2  (I + C) |z1|2 + (I +C–1) |z2|2
Solution. We have to prove :
|z1 + z2|2  (1 + c) |z1|2 + (1 + c –1) |z2|2
i.e. |z1|3 + |z2|2 + z1 z 2 + z 2z2  (1 + c) |z1|2 + (1 +c–1) |z2|3
1
or z1 z 2 + z 2z2  c|z1|2 + c –1|z2|2 or c|z1|2 + |z |2 – z1 z 2 – z 2 z2  0
c 2
(using Re (z1 z 2)  |z1 z 2|)

 
2
1
or  c z1  | z2 |   0 which is always true.

 c 

Example: If ,  [/6, /3], i = 1, 2, 3, 4, 5, and z4 cos 1 + z3 cos 2 + z3 cos 3. + z cos 4 + cos5 = 2 3 ,

3
then show that |z| >
4
Solution. Given that
cos1 . z4 + cos2 . z3 + cos3 . z2 + cos4 . z + cos5 = 23
or |cos1 . z4 + cos2 . z3 + cos3 . z2 + cos4 . z + cos5| = 23
23  |cos1 . z4 | + |cos2 . z3 | + |cos3 . z2 | + cos4 . z| + |cos5 |
 i  [/6, /3]

RESONANCE 36
1 3
  cosi 
2 2

3 3 3 3 2 3 3
2 3  |z|4 + |z| + |z| + |z| +
2 2 2 2 2
3  |z|4 + |z|3 + |z|2 + |z|
3 < |z| + |z|2 + |z|3 + |z|4 +|z|5 + .........
|z|
3 < 1 | z | 3 – e |z| < |z|

3
4|z| > 3  |z| >
4

Example: Two different non parallel lines cut the circle |z| = r in point a, b, c, d respectively. Prove that

a 1  b 1  c 1  d 1
these lines meet in the point z given by z =
a 1b 1  c 1d 1
Solution. Since point P, A, B are collinear

z z 1


a a 1
=0    
z a  b – z (a – b) + a b  a b = 0  (i)
b b 1

Similarlym, since points P, C, D are collinear


      
z a  b (c – d) – z c  d (a – b) = c d  cd (a – b) – a b  a b (c – d)   (iii)

k k k
 zz = r = k (say)
2
 a = a , b = b , c = c etc.

From equation (iii) we get

k k k k   ck kd   ak bk 
z    (c – d) – z    (a – b) =    (a – b) –    (c – d)
a b c d  d c   b a 

a 1  b 1  c 1  d 1
 z=
a 1b 1  c 1d 1

RESONANCE 37
Part : (A) Only one correct option
z 1
1. If |z| = 1 and  = (where z  –1), the Re() is [IIT – 2003, 3]
z 1
1 z 1 2
(A) 0 (B)  (C) z  1 . (D)
| z  1 |2 | z  1 |2 | z  1 |2
2. The locus of z which lies in shaded region (excluding the boundaries) is best represented by

[IIT – 2005, 3]

(A) z : |z + 1| > 2 and |arg (z + 1)| < /4 (B) z : |z – 1| > 2 and |arg (z – 1)| < /4
(C) z : |z + 1| < 2 and |arg (z + 1)| < /2 (D) z : |z – 1| < 2 and |arg (z + 1)| < /2

 w  wz 
3. If w = , + i, where   0 and z  1, satisfies the condition that   is purely real, then the set of
 1 z 
values of z is [IIT – 2006, (3, –1)]
(A) {z : |z| = 1} (B) {z : z = z } (C) {z : z 1} (D) {z : |z| = 1, z 1}

4. If ( 3 + i)100 = 299 (a + ib), then b is equal to


(A) 3 (B) 2 (C) 1 (D) none of these

 z  8i 
5. If Re   = 0, then z lies on the curve
 z6
(A) x2 + y2 + 6x – 8y = 0 (B) 4x – 3y + 24 = 0 (C) 4ab (D) none of these

6. If n1, n2 are positive integers then : (1  i)n1 + (1  i3 )n1 + (1  i5 )n 2 + (1  i7 )n 2 is a real number if and only if
(A) n1 = n2 + 1 (B) n1 + 1 = n2
(C) n1 = n2 (D) n1, n2 are any two positive integers
7. The three vertices of a triangle are represented by the complex numbers, 0, z1 and z2. If the triangle is
equilateral, then
(A) z12 – z22 = z1z2 (B) z22 – z12 = z1 z2 (C) z12 + z22 = z1z2 (D) z12 + z22 + z1z2 = 0


2
 n 1 
5
8. If x2 – x + 1 = 0 then the value of  x  n  is
n 1  x 
(A) 8 (B) 10 (C) 12 (D) none of these

9. If  is nonreal and  = 5
1 then the value of 2|1     2   2   1| is equal to
(A) 4 (B) 2 (C) 1 (D) none of these

10. If z = x + iy and z1/3 = a  ib then


x y
 
  k a 2  b 2 where k =
a b
(A) 1 (B) 2 (C) 3 (D) 4

 1  i 3   1  i 3   1  i 3   1  i 3 
6 6 5 5

11.         is equal to :
 2   2   2   2 
(A) 1 (B)  1 (C) 2 (D) none

RESONANCE 38
12. Expressed in the form r (cos  + i sin ),  2 + 2i becomes :
        3   3 
(A) 2 2  cos     i sin     (B) 2 2  cos    i sin   
   4   4   4  4 

  3   3       
(C) 2 2  cos     i sin     (D) 2  cos     i sin    
  4  4    4   4 

13. The number of solutions of the equation in z, z z - (3 + i) z - (3 - i) z - 6 = 0 is :


(A) 0 (B) 1 (C) 2 (D) infinite
14. If |z| = max {|z – 1|, |z + 1|} then
1
(A) |z + z | = (B) z + z = 1 (C) |z + z | = 1 (D) none of these
2
15. If P, P represent the complex number z1 and its additive inverse respectively then the complex equation of
the circle with PPas a diameter is
z  z1 
(A) z =   (B) z z + z1 z1 = 0 (C) z z1 + z z1 = 0 (D) none of these
1  z

16. The points z1 = 3 + 3 i and z2 = 2 3 + 6 i are given on a complex plane. The complex number lying
on the bisector of the angle formed by the vectors z1 and z2 is :
(3  2 3 ) 3 2
(A) z =  i (B) z = 5 + 5 i
2 2
(C) z =  1  i (D) none

 1  i tan   1  i tan n 
n

17. The expression    when simplified reduces to :


 1  i tan   1  i tan n 
(A) zero (B) 2 sin n  (C) 2 cos n  (D) none
18. All roots of the equation, (1 + z)6 + z6 = 0 :
(A) lie on a unit circle with centre at the origin
(B) lie on a unit circle with centre at ( 1, 0)
(C) lie on the vertices of a regular polygon with centre at the origin
(D) are collinear
19. Points z1 & z2 are adjacent vertices of a regular octagon. The vertex z3 adjacent to z2 (z3  z1) is
represented by :
1 1
(A) z2 + (1 ± i) (z1 + z2) (B) z2 + (1 ± i) (z1  z2)
2 2
1
(C) z2 + (1 ± i) (z2  z1) (D) none of these
2
20. If z = x + i y then the equation of a straight line Ax + By + C = 0 where A, B, C  R, can be written on
the complex plane in the form a z  a z  2 C = 0 where 'a' is equal to :

A  i B A  iB
(A) (B) (C) A + i B (D) none
2 2
21. The points of intersection of the two curves z  3 = 2 and z = 2 in an argand plane are:

(A)
1
2

7i 3  (B)
1
2

3i 7  (C)
3
2
±i
7
2
(D)
7
2
±i
3
2

22. The equation of the radical axis of the two circles represented by the equations,
z  2 = 3 and z  2  3 i = 4 on the complex plane is :
(A) 3iz – 3i z – 2 = 0 (B) 3iz – 3i z + 2 = 0 (C) iz – i z + 1 = 0 (D) 2iz – 2i z + 3 = 0

RESONANCE 39
r
23. If  eip = 1 where  denotes the continued product, then the most general value of  is :
p1

2n  2n  4n  4n 
(A) (B) (C) (D)
r ( r  1) r ( r  1) r ( r  1) r ( r  1)

24. The set of values of a  R for which x2 + i(a – 1) x + 5 = 0 will have a pair of conjugate imaginary roots is
(A) R (B) {1} (C) |a| a2 – 2a + 21 > 0} (D) none of these

25. If |z1 – 1| < 1, |z2 – 2| < 2, |z3 – 3| < 3 then |z1 + z2 + z3|
(A) is less than 6 (B) is more than 3
(C) is less than 12 (D) lies between 6 and 12

26. If z1, z2, z3, ........., zn lie on the circle |z| = 2, then the value of
1 1 1
E = |z1 + z2 + ..... + zn| – 4 z  z  .......  z is
1 2 n
(A) 0 (B) n (C) –n (D) none of these

Part : (B) May have more than one options correct

27. If z1 lies on |z| = 1 and z2 lies on |z| = 2, then


(A) 3  |z1 – 2z2|  5 (B) 1  |z1 + z2|  3
(C) |z1 – 3z2|  5 (D) |z1 – z2|  1
28. If z1, z2, z3, z4 are root of the equation a0z4 + z1z3 + z2z2 + z3z + z4 = 0, where a0, a1, a2, a3 and a4 are real,
then
(A) z1 , z 2 , z3 , z 4 are also roots of the equation
(B) z1 is equal to at least one of z1 , z 2 , z3 , z 4
(C) – z1 ,– z 2 , – z3 , – z 4 are also roots of the equation
(D) none of these
29. If a3 + b3 + 6 abc = 8 c 3 &  is a cube root of unity then :
(A) a, c, b are in A.P. (B) a, c, b are in H.P.
(C) a + b  2 c2 = 0 (D) a + b2  2 c = 0
30. The points z1, z2, z3 on the complex plane are the vertices of an equilateral triangle if and only if :
(A)  (z1  z2) (z2  z3) = 0 (B) z12 + z22 + z32 = 2 (z1 z2 + z2 z3 + z3 z1)
(C) z1 + z2 + z3 = z1 z2 + z2 z3 + z3 z1
2 2 2 (D) 2 (z12 + z22 + z32) = z1 z2 + z2 z3 + z3 z1
31. If |z1 + z2| = |z1 – z2| then

(A) |amp z1 – amp z2| = (B) | amp z1 – amp2| = 
2
z1 z1
(C) z is purely real (D) z is purely imaginary
2 2

1. Given that x, y  R, solve : 4x² + 3xy + (2xy  3x²)i = 4y²  (x2/2) + (3xy  2y²)i
2. If  &  are any two complex numbers, prove that :
  2  2     2  2         

3. If ,  are the numbers between 0 and 1, such that the points z1 =  + i, z2 = 1 + i and z3 = 0 form an
equilateral triangle, then find  and .
4. ABCD is a rhombus. Its diagonals AC and BD intersect at the point M and satisfy BD = 2AC. If the points D
and M represent the complex numbers 1 + i and 2 - i respectively, then find the complex number corresponding
to A.

RESONANCE 40
5. Show that the sum of the pth powers of nth roots of unity :
(a) is zero, when p is not a multiple of n.
(b) is equal to n, when p is a multiple of n.

6. If (1 + x) n = p0 + p1 x + p2 x 2 + p3 x 3 +......., then prove that :


n n
(a) p0  p2 + p4 ....... = 2n/2 cos (b) p1  p3 + p5 ....... = 2n/2 sin
4 4

 1  1    
7. Prove that, loge   = loge  cosec  + i   

 1  ei   2 2  2 2

i ....... 
8. If i i = A + i B, principal values only being considered, prove that
1 B
(a) tan A = (b) A2 + B2 = e B
2 A

1  r 
9. Prove that the roots of the equation, (x - 1) n = xn are 1  i cot  , where
2  r 
r = 0, 1, 2,....... (n  1) & n  N.
10. If cos (   ) + cos (  ) + cos (  ) =  3/2 then prove that :
(a)  cos 2 = 0 =  sin 2 (b)  sin ( +  ) = 0 =  cos ( +  )
(c)  sin 3 = 3 sin ( +  + ) (d)  cos 3  = 3 cos ( +  + )
(e)  sin2  =  cos2  = 3/2
(f) cos 3 () + cos3 ( ) + cos3 () = 3 cos ( + ). cos ( +  ). cos ( + )
where  R.

11. If ,  ,  are roots of x 3  3 x 2 + 3 x + 7 = 0 (and  is imaginary cube root of unity), then find the value
 1 1  1
of + + .
 1  1  1
z2
12. Given that, z  1 = 1, where ' z ' is a point on the argand plane. Show that = i tan (arg z).
z
13. P is a point on the Argand diagram. On the circle with OP as diameter two points Q & R are taken such
that  POQ =  QOR = . If ‘O’ is the origin & P, Q & R are represented by the complex numbers
Z1, Z2 & Z3 respectively, show that : Z22. cos 2  = Z1. Z 3 cos²

14. Find an expression for tan 7  in terms of tan  , using complex numbers. By considering
tan 7= 0, show that x = tan2 (3 /7) satisfies the cubic equation x 3  21x2 + 35x  7 = 0.

 n 1 n
If (1 + x)n = C0 + C1x + C2x² +.... + Cn x n (n  N), prove that : C2 + C6 + C10 +..... =
1
15. 2  2 n / 2 cos
2  4 

 2   4   6   2n 
16. Prove that : cos   + cos   + cos   +..... + cos 
1
 =  When n  N.
 2 n  1  2 n  1  2 n  1  2 n  1 2
17. Show that all the roots of the equation a1z3 + a2z2 + a3z + a4 = 3, where |ai|  1, i = 1, 2, 3, 4 lie outside the
circle with centre origin and radius 2/3.

 (n  k ) cos 2nk = – n2 , where n  3 is an integer


n 1

18. Prove that


k 1

A1 A2 An
2 2 2
19. Show that the equation   ......  = k has no imaginary root, given that :
x  a1 x  a 2 x  an
a1, a2, a3.... an & A1, A2, A3..... An, k are all real numbers.

RESONANCE 41
20. Let z1, z2, z3 be three distinct complex numbers satisfying, ½z1-1½ = ½z2-1½ = ½z3-1½. Let A, B & C
be the points represented in the Argand plane corresponding to z1, z2 and z3 resp. Prove that z1 + z2 +
z3 = 3 if and only if D ABC is an equilateral triangle.

21. Let ,  be fixed complex numbers and z is a variable complex number such that,

z   + z   = k.
2 2

Find out the limits for 'k' such that the locus of z is a circle. Find also the centre and radius of the
circle.

22. If 1, 1, 2, 3,......., n  1 are the n, nth roots of unity, then prove that
(1 1) (1 2) (1 3)........ (1  n  1) = n.
 2 3 (n  1)  n
Hence prove that sin . sin . sin ........ sin = n 1 .
n n n n 2
23. Find the real values of the parameter ‘a’ for which at least one complex number
z = x + iy satisfies both the equality z  ai = a + 4 and the inequality z  2 < 1.

24. Prove that, with regard to the quadratic equation z2 + (p + ip) z + q + iq = 0; where p, p, q, q are all
real.
(a) if the equation has one real root then q 2  pp  q + qp 2 = 0.
(b) if the equation has two equal roots then p2  p2 = 4q & pp = 2q .
State whether these equal roots are real or complex.
25. The points A, B, C depict the complex numbers z1, z2, z3 respectively on a complex plane & the angle
1
B & C of the triangle ABC are each equal to (   ) . Show that
2

(z2  z3)² = 4 (z3  z1) (z1  z2) sin2 .
2
26. If z1 , z2 & z3 are the affixes of three points A, B & C respectively and satisfy the condition
|z1 – z2| = |z1| + |z2| and |(2 - i) z1 + iz3 | = |z1| + |(1 – i) z1 + iz3| then prove that  ABC in a right angled.

27. If 1, 1, 2, 3, 4 be the roots of x5  1 = 0, then prove that


  1 .    2 .    3 .    4 = .
2  1    2    3    4
2 2 2

28. If one the vertices of the square circumscribing the circle |z – 1| = 2 is 2 + 3 i. Find the other vertices of
the square. [IIT – 2005, 4]

EXERCISE # 1 EXERCISE # 2
1. A 2. C 3. D 4. A

5. A 6. D 7. C 8. A 3K
1. x = K, y = KR 3. 2  3, 2  3
2
9. A 11. D 12. A 13. B
i 3
14. D 15. D 16. A 17. B 4. 3– or 1 – i 11. 3 2
2 2
18. A 19. D 20. C 21. C
1
  21 5
 ,  
2
22. B 23. B 24. D 25. B 21. k> 23.
2  10 6
26. C 27. A 28. ABCD 29. AB
28. –i 3,1– 3 + i, 1 + 3 –i
30. ACD 31. AC 10. AD

RESONANCE 42
Trigonometric Ratios
& Identities

1. Basic Trigonometric Identities:

There being, then, three


(a) sin²  + cos²  = 1; 1  sin   1; 1  cos   1    R

(b) sec²  tan² = 1 ; sec   1    R – 2n  1 , n   


  
 2 
figures which of (c) cosec²  cot² = 1 ; cosec   1    R – n , n  
themselves can fill up
Solved Example # 1
space round a point, viz.
Prove that
the triangle, the square (i) cos 4A – sin4A + 1 = 2 cos 2A
tan A  sec A  1 1 sin A
and the hexagon, the (ii) =
tan A  sec A  1 cos A
bees have wisely selected Solution
(i) cos 4A – sin4A + 1
for their structure = (cos 2A – sin2A) (cos2A + sin2A) + 1
= cos 2A – sin2A + 1 [ cos 2A + sin2A = 1]
t h a t w h i c h = 2 cos A
2

contains most angles,


tan A  sec A  1
suspecting indeed that it (ii)
tan A  sec A  1

could hold more honey tan A  sec A  (sec 2 A  tan 2 A )


=
tan A  sec A  1
than either of the other
(tan A  sec A )(1  sec A  tan A )
two. =
tan A  sec A  1

....... Pappus 1 sin A


= tan A + sec A =
cos A

Solved Example # 2
If sin x + sin2x = 1, then find the value of
cos12x + 3 cos10x + 3 cos8x + cos6x – 1
Solution
cos12x + 3 cos10x + 3 cos8x + cos6x – 1
= (cos 4x + cos2x)3 – 1
= (sin2x + sinx)3 – 1 [ cos 2x = sin x]
=1–1=0

RESONANCE 1
Solved Example # 3
1 1
If tan  = m – , then show that sec  – tan  = – 2m or
4m 2m
Solution

4m 2  1
Depending on quadrant in which  falls, sec  can be ±
4m

4m 2  1 1
So, if sec  = =m+
4m 4m

1  1 
 sec  – tan  = and if sec  = –  m  
2m  4m
 sec  – tan  = – 2m

Self Practice Problem

1. Prove the followings :


(i) cos 6A + sin6A + 3 sin2A cos 2A = 1
(ii) sec2A + cosec 2A = (tan A + cot A) 2
(iii) sec2A cosec 2A = tan2A + cot2A + 2
(iv) (tan  + cosec )2 – (cot  – sec )2 = 2 tan  cot  (cosec  + sec )

 1 1  1  sin2  cos 2 
(v)    cos  sin  =
 sec 2   cos 2  cos ec 2   sin 2  
2 2
2  sin2  cos 2 

m 2  2mn m 2  2mn
2. If sin  = , then prove that tan  =
m 2  2mn  2n 2 2mn  2n 2

2. Circular Definition Of Trigonometric Functions:

PM OM
sin  = cos  =
OP OP

sin 
tan  = cos  , cos  0

cos 
cot  = sin  , sin  0

1 1
sec  = , cos  0 cosec  = , sin  0
cos  sin 

3. Trigonometric Functions Of Allied Angles:


If  is any angle, then  90 ± , 180 ± , 270 ± , 360 ±  etc. are called ALLIED ANGLES .
(a) sin ( ) =  sin  ; cos ( ) = cos 
(b) sin (90°  ) = cos  ; cos (90°  ) = sin 
(c) sin (90° + ) = cos  ; cos (90° + ) =  sin 
(d) sin (180°  ) = sin  ; cos (180°  ) =  cos 
(e) sin (180° + ) =  sin  ; cos (180° + ) =  cos 
(f) sin (270°  ) =  cos  ; cos (270°  ) =  sin 
(g) sin (270° + ) =  cos  ; cos (270° + ) = sin 
(h) tan (90° ) = cot  ; cot (90° ) = tan 

RESONANCE 2
Solved Example # 4
Prove that
(i) cot A + tan (180º + A) + tan (90º + A) + tan (360º – A) = 0
(ii) sec (270º – A) sec (90º – A) – tan (270º – A) tan (90º + A) + 1 = 0
Solution
(i) cot A + tan (180º + A) + tan (90º + A) + tan (360º – A)
= cot A + tan A – cot A – tan A = 0
(ii) sec (270º – A) sec (90º – A) – tan (270º – A) tan (90º + A) + 1
= – cosec 2A + cot2A + 1 = 0

Self Practice Problem

3. Prove that
(i) sin 420º cos 390º + cos (–300º) sin (–330º) = 1
(ii) tan 225º cot 405º + tan 765º cot 675º = 0

4. Graphs of Trigonometric functions:


(a) y = sin x x  R; y  [–1, 1]

(b) y = cos x x  R; y  [ – 1, 1]

(c) y = tan x x  R – (2n + 1)/2, n  ; y  R

RESONANCE 3
(d) y = cot x x  R – n , n ; y  R

(e) y = cosec x x  R – n , n  ; y  (,  1]  [1, )

(f) y = sec x x R – (2n + 1)/2, n  ; y  (,  1]  [1, )

Solved Example # 5
Find number of solutions of the equation cos x = |x|
Solution

Clearly graph of cos x & |x| intersect at two points. Hence no. of solutions is 2

RESONANCE 4
Solved Example # 6
Find range of y = sin2x + 2 sin x + 3  x  R
Solution
We know – 1  sin x  1
 0  sin x +1  2
 2  (sin x +1)2 + 2  6
Hence range is y  [2, 6]
Self Practice Problem
4 xy
4. Show that the equation sec2 = is only possible when x = y  0
( x  y )2
5. Find range of the followings.
(i) y = 2 sin2x + 5 sin x +1 x  R Answer [–2, 8]

3 
(ii) y = cos2x – cos x + 1  x  R Answer  4 , 3
 

 2   3
6. Find range of y = sin x, x   2  Answer  1, 
3   2 

5. Trigonometric Functions of Sum or Difference of Two Angles:


(a) sin (A ± B) = sinA cosB ± cosA sinB
(b) cos (A ± B) = cosA cosB  sinA sinB
(c) sin²A  sin²B = cos²B  cos²A = sin (A+B). sin (A B)
(d) cos²A  sin²B = cos²B  sin²A = cos (A+B). cos (A  B)
tan A  tan B
(e) tan (A ± B) = 1  tan A tan B

cot A cot B  1
(f) cot (A ± B) = cot B  cot A

tan A  tan B  tanCtan A tan B tan C


(g) tan (A + B + C) = 1  tan A tan B  tan B tan C tan C tan A .

Solved Example # 7
Prove that
(i) sin (45º + A) cos (45º – B) + cos (45º + A) sin (45º – B) = cos (A – B)

   3 
(ii) tan     tan     = –1
4   4 
Solution
(i) Clearly sin (45º + A) cos (45º – B) + cos (45º + A) sin (45º – B)
= sin (45º + A + 45º – B)
= sin (90º + A – B)
= cos (A – B)

   3 
(ii) tan     × tan   
4   4 

1  tan  1  tan 
= × =–1
1  tan  1  tan 

RESONANCE 5
Self Practice Problem

3 5
7. If sin  = , cos  = , then find sin ( + )
5 13

33 63
Answer – ,
65 65

8. Find the value of sin 105º

3 1
Answer
2 2

A A
9. Prove that 1 + tan A tan = tan A cot – 1 = sec A
2 2

6. Factorisation of the Sum or Difference of Two Sines or Cosines:

CD CD CD CD


(a) sinC + sinD = 2 sin cos (b) sinC  sinD = 2 cos sin
2 2 2 2

CD CD CD CD


(c) cosC + cosD = 2 cos cos (d) cosC  cosD =  2 sin sin
2 2 2 2

Solved Example # 8

Prove that sin 5A + sin 3A = 2sin 4A cos A

Solution
L.H.S. sin 5A + sin 3A = 2sin 4A cos A = R.H.S.
CD C D
[ sin C + sin D = 2 sin cos ]
2 2
Solved Example # 9

Find the value of 2 sin 3 cos  – sin 4 – sin 2


Solution
2 sin 3 cos  – sin 4 – sin 2 = 2 sin 3 cos  – [2 sin 3 cos  ] = 0

Self Practice Problem

10. Proved that


13 x 3x
(i) cos 8x – cos 5x = – 2 sin sin
2 2

sin A  sin 2A A
(ii) = cot
cos A  cos 2A 2

sin A  sin 3 A  sin 5 A  sin 7 A


(iii) = tan 4A
cos A  cos 3 A  cos 5 A  cos 7 A

sin A  2 sin 3 A  sin 5 A sin 3 A


(iv) =
sin 3 A  2 sin 5 A  sin 7 A sin 5 A

sin A  sin 5 A  sin 9 A  sin 13 A


(v) = cot 4A
cos A  cos 5 A  cos 9 A  cos 13 A

RESONANCE 6
7. Transforma tion of Products into Sum or Dif ferenc e of Sines &
C osines:
(a) 2 sinA cosB = sin(A+B) + sin(AB) (b) 2 cosA sinB = sin(A+B)  sin(AB)

(c) 2 cosA cosB = cos(A+B) + cos(AB) (d) 2 sinA sinB = cos(AB)  cos(A+B)

Solved Example # 10

Prove that
sin 8 cos   sin 6 cos 3
(i) = tan 2
cos 2 cos   sin 3 sin 4

tan 5  tan 3
(ii) = 4 cos 2 cos 4
tan 5  tan 3

Solution
2 sin 8 cos   2 sin 6 cos 3
(i)
2 cos 2 cos   2 sin 3 sin 4

sin 9  sin 7  sin 9  sin 3 2 sin 2 cos 5


= = = tan 2
cos 3  cos   cos   cos 7 2 cos 5 cos 2

tan 5  tan 3 sin 5 cos 3  sin 3 cos 5 sin 8


(ii) = = = 4 cos2 cos 4
tan 5  tan 3 sin 5 cos 3  sin 3 cos 5 sin 2

Self Practice Problem

 7 3 11
11. Prove that sin sin + sin sin = sin 2 sin 5
2 2 2 2

12. Prove that cos A sin (B – C) + cos B sin (C – A) + cos C sin (A – B) = 0


 9 3 5
13. Prove that 2 cos cos + cos + cos =0
13 13 13 13

8. Multiple and Sub-multiple Angles :


 
(a) sin 2A = 2 sinA cosA ; sin  = 2 sin cos
2 2

 
(b) cos 2A = cos²A  sin²A = 2cos²A  1 = 1  2 sin²A; 2 cos² = 1 + cos , 2 sin² = 1  cos .
2 2

2 tan A 2 tan 2
(c) tan 2A = ; tan  =
1  tan 2 A 1  tan 2 2

2 tan A 1tan 2 A
(d) sin 2A = , cos 2A =
1  tan 2 A 1 tan 2 A
(e) sin 3A = 3 sinA  4 sin3A
(f) cos 3A = 4 cos3A  3 cosA

3 tan A  tan 3 A
(g) tan 3A =
1  3 tan 2 A

RESONANCE 7
Solved Example # 11

Prove that
sin 2A
(i) = tan A
1  cos 2A

(ii) tan A + cot A = 2 cosec 2 A

1  cos A  cos B  cos( A  B) A B


(iii) = tan cot
1  cos A  cos B  cos( A  B) 2 2
Solution

sin 2A 2 sin A cos A


(i) L.H.S. = = tan A
1  cos 2A 2 cos 2 A

1  tan 2 A  1  tan 2 A  2
(ii) L.H.S. tan A + cot A =

= 2  2 tan A  =
 = 2 cosec 2 A
tan A   sin 2 A

1  cos A  cos B  cos( A  B)


(iii) L.H.S.
1  cos A  cos B  cos( A  B)

A A A 
2 sin2  2 sin sin  B 
2 2  2 
=
2 A A A 
2 cos  2 cos cos  B 
2 2  2 

 A A   A B  B 
A 
sin  sin  B    2 sin 2 cos 2  
 2 2   = tan A   
= tan
2  A A  2  A B B 
 cos 2  cos 2  B    2 sin 2 sin 2  
     

A B
= tan cot
2 2

Self Practice Problem

sin   sin 2
14. Prove that = tan 
1  cos   cos 2

3
15. Prove that sin 20º sin 40º sin 60º sin 80º =
16

16. Prove that tan 3A tan 2A tan A = tan 3A – tan 2A – tan A

 A
17. Prove that tan  45 º   = sec A + tan A
 2

RESONANCE 8
9. Important Trigonometric Ratios:
(a) sin n  = 0 ; cos n  = (1)n ; tan n  = 0, where n  

 3 1 5
(b) sin 15° or sin = = cos 75° or cos ;
12 2 2 12

 3 1 5
cos 15° or cos = = sin 75° or sin ;
12 2 2 12

3 1 3 1
tan 15° = = 2 3 = cot 75° ; tan 75° = = 2 3 = cot 15°
3 1 3 1

 5 1  5 1
(c) sin or sin 18° = & cos 36° or cos =
10 4 5 4

1 0 . Conditional Identities:
If A + B + C =  then :

(i) sin2A + sin2B + sin2C = 4 sinA sinB sinC


A B C
(ii) sinA + sinB + sinC = 4 cos cos cos
2 2 2

(iii) cos 2 A + cos 2 B + cos 2 C =  1  4 cos A cos B cos C


A B C
(iv) cos A + cos B + cos C = 1 + 4 sin sin sin
2 2 2

(v) tanA + tanB + tanC = tanA tanB tanC


A B B C C A
(vi) tan tan + tan tan + tan tan =1
2 2 2 2 2 2

A B C A B C
(vii) cot + cot + cot = cot . cot . cot
2 2 2 2 2 2
(viii) cot A cot B + cot B cot C + cot C cot A = 1

(ix) A+B+C= then tan A tan B + tan B tan C + tan C tan A = 1
2

Solved Example # 12
If A + B + C = 180°, Prove that, sin2A + sin2B + sin2C = 2 + 2cosA cosB cosC.
Solution.
Let S = sin2A + sin2B + sin2C
so that 2S = 2sin2A + 1 – cos2B +1 – cos2C
= 2 sin2A + 2 – 2cos(B + C) cos(B – C)
= 2 – 2 cos2A + 2 – 2cos(B + C) cos(B – C)
 S = 2 + cosA [cos(B – C) + cos(B+ C)]
since cosA = – cos(B+C)
 S = 2 + 2 cos A cos B cos C

RESONANCE 9
Solved Example # 13

2x 2y 2z 2x 2y 2z
If x + y + z = xyz, Prove that + + = . . .
1 x 2 1 y2 1 z2 1 x 2
1 y2 1 z2
Solution.
Put x = tanA, y = tanB and z = tanC,
so that we have
tanA + tanB + tanC = tanA tanB tanC  A + B + C = n where n  
Hence
L.H.S.

2x 2y 2z 2 tan A 2 tan B 2 tan C


 + + = + + .
1 x 2 1 y 2
1 z 2
1  tan A 2
1  tan B 2
1  tan2 C
= tan2A + tan2B + tan2C [ A + B + C = n ]
= tan2A tan2B tan2C

2x 2y 2z
= . .
1 x 2 1 y2 1 z2

Self Practice Problem

18. If A + B + C = 180°, prove that


BC CA A B
(i) sin(B + 2C) + sin(C + 2A) + sin(A + 2B) = 4sin sin sin
2 2 2

sin 2A  sin 2B  sin 2C A B C


(ii) = 8 sin sin sin .
sin A  sin B  sin C 2 2 2

19. If A + B + C = 2S, prove that


(i) sin(S – A) sin(S – B) + sinS sin (S – C) = sinA sinB.
A B C
(ii) sin(S – A) + sin (S – B) + sin(S – C) – sin S = 4sin sin sin .
2 2 2

1 1 . Range of Trigonometric Expression:


E = a sin  + b cos 
b
E = a 2  b 2 sin ( + ), where tan  =
a

a
= a 2  b 2 cos (), where tan  =
b

Hence for any real value of ,  a 2  b 2  E  a2  b2

Solved Example # 14

Find maximum and minimum values of following :


(i) 3sinx + 4cosx
(ii) 1 + 2sinx + 3cos2x
Solution.
(i) We know

– 3 2  4 2  3sinx + 4cosx  32  42
– 5  3sinx + 4cosx  5

RESONANCE 10
(ii) 1+ 2sinx + 3cos2x
= – 3sin2x + 2sinx + 4

 2 2 sin x 
= – 3  sin x   +4
 3 

 1
2
13
= – 3  sin x   +
 3 3

 1
2
16
Now 0   sin x   
 3 9

 1
2
16
 –  – 3  sin x    0
3  3 

 1
2
13 13
– 1  – 3  sin x   + 
 3  3 3

Self Practice Problem


20. Find maximum and minimum values of following
(i) 3 + (sinx – 2)2 Answer max = 12, min = 4.
(ii) 10cos 2x – 6sinx cosx + 2sin2x Answer max = 11, min = 1.

 
(iii) cos + 3 2 sin     + 6 Answer max = 11, min = 1
 4 

12 . Sine and Cosine Series:

 
n
sin 2  n1 
sin  + sin (+) + sin ( + 2 ) +...... + sin   n 1 =   
 sin  2 
sin 2

 
n
sin 2  n 1 
cos  + cos (+) + cos ( + 2 ) +...... + cos   n  1 =   
 cos  2 
sin 2

Solved Example # 15
Find the summation of the following
2 4 6
(i) cos + cos + cos
7 7 7

 2 3 4 5 6
(ii) cos + cos + cos + cos + cos + cos
7 7 7 7 7 7

 3 5 7 9
(iii) cos + cos + cos + cos + cos
11 11 11 11 11
Solution.

 2 6  
  
 7 7  3
cos sin
2 4 6 2 7
(i) cos + cos + cos =
7 7 7 
sin
7

RESONANCE 11
4 3
cos sin
7 7
=

sin
7

3 3
 cos sin
7 7
=

sin
7

6
sin
=– 7 =– 1
 2
2 sin
7

 2 3 4 5 6
(ii) cos + cos + cos + cos + cos + cos
7 7 7 7 7 7

  6 
   6
cos  7 7  sin
 2  14  6
  cos sin
  2 14
=  = =0

sin sin
14 14

 3 5 7 9
(iii) cos + cos + cos + cos + cos
11 11 11 11 11

10  5 10 
cos sin sin
=
22 11
= 11 = 1
  2
sin 2 sin
11 11

Self Practice Problem

Find sum of the following series :

 3 5 1
21. cos + cos + cos + ...... + to n terms. Answer
2n  1 2n  1 2n  1 2

22. sin2 + sin3 + sin4 + ..... + sin n, where (n + 2) = 2 Answer 0.

RESONANCE 12
Part : (A) Only one correct option

 
tan x  2 .cos 32  x  sin3 72  x 
 2 .tan 32  x 
1. when simplified reduces to:
cos x 

(A) sin x cos x (B)  sin2 x (C)  sin x cos x (D) sin2x

 4  3    6   
The expression 3 sin  2     sin (3    ) – 2 sin  2     sin (5   ) is equal to
4 6
2.
       
(A) 0 (B) 1 (C) 3 (D) sin 4 + sin 6

3. If tan A & tan B are the roots of the quadratic equation x2  ax + b = 0, then the value of sin2 (A + B).

a2 a2 a2 a2
(A) (B) (C) (D)
a (1b)
2 2
a b
2 2
(bc ) 2
b (1a)2
2

4. The value of log2 [cos 2 ( + ) + cos 2 ()  cos 2. cos 2] :
(A) depends on  &  both (B) depends on  but not on 
(C) depends on  but not on  (D) independent of both  & .

cos208sin70sin50sin10
5. is equal to:
sin 2 80
(A) 1 (B) 2 (C) 3/4 (D) none

6. If cos A = 3/4, then the value of 16cos2 (A/2) – 32 sin (A/2) sin (5A/2) is
(A) – 4 (B) – 3 (C) 3 (D) 4

7. If y = cos2 (45º + x) + (sin x  cos x)2 then the maximum & minimum values of y are:
(A) 2 & 0 (B) 3 & 0 (C) 3 & 1 (D) none

 3 5 17
8. The value of cos + cos + cos +...... + cos is equal to:
19 19 19 19
(A) 1/2 (B) 0 (C) 1 (D) none

9. The greatest and least value of log 2


sin x  cos x  3 2  are respectively:
(A) 2 & 1 (B) & 3 (C) 7 & 5 (D) 9 & 7

10. In a right angled triangle the hypotenuse is 2 2 times the perpendicular drawn from the opposite
vertex. Then the other acute angles of the triangle are
   3    3
(A) & (B) & (C) & (D) &
3 6 8 8 4 4 5 10

1 1
11. cos290  + 3 sin250 =

2 3 4 3
(A) (B) (C) 3 (D) none
3 3

RESONANCE 13
3 1
12. If <  < , then 2 cot   is equal to
4 sin2 

(A) 1 + cot  (B) – 1 – cot  (C) 1 – cot  (D) – 1 + cot 

 3   x
13. If x   ,  then 4 cos 2    + 4 sin 4 x  sin 2 2x is always equal to
 2  4 2

(A) 1 (B) 2 (C) – 2 (D) none of these

14. If 2 cos x + sin x = 1, then value of 7 cos x + 6 sin x is equal to


(A) 2 or 6 (B) 1 or 3 (C) 2 or 3 (D) none of these

11
15. If cosec A + cot A = , then tan A is
2

21 15 44 117
(A) (B) (C) (D)
22 16 117 43

1
16. If cot  + tan  = m and – cos  = n, then
cos 
(A) m (mn2)1/3 – n(nm 2)1/3 = 1 (B) m(m 2n)1/3 – n(nm 2)1/3 = 1
(C) n(mn2)1/3 – m(nm 2)1/3 = 1 (D) n(m 2n)1/3 m(mn2)1/3 = 1

cos 6 x  6 cos 4 x  15 cos 2x  10


17. The expression is equal to
cos 5 x  5 cos 3 x  10 cos x
(A) cos 2x (B) 2 cos x (C) cos2 x (D) 1 + cos x

sin A 3 cos A 5
18. If = and = , 0 < A, B </2, then tan A + tan B is equal to
sin B 2 cos B 2

(A) 3/ 5 (B) 5/ 3 (C) 1 (D) ( 5  3 ) / 5

tan 3  cot 3 
19. If sin 2 = k, then the value of + is equal to
1  tan 2  1  cot 2 

1 k 2 2  k2
(A) (B) (C) k 2 + 1 (D) 2 – k 2
k k

Part : (B) May have more than one options correct

20. Which of the following is correct ?


(A) sin 1° > sin 1 (B) sin 1° < sin 1 (C) cos 1° > cos 1 (D) cos 1° < cos 1

21. If 3 sin  = sin (2 + ), then tan (+ ) – 2 tan  is


(A) independent of  (B) independent of 
(C) dependent of both  and  (D) independent of  but dependent of 

RESONANCE 14
3 sin(  )  2 cos (   )
4 cos 6
22. It is known that sin  = & 0 <  <  then the value of is:
5 sin

5
(A) independent of  for all  in (0, ) (B) for tan  > 0
3

3 (7  24 cot  )
(C) for tan  < 0 (D) none
15

23. If the sides of a right angled triangle are {cos2cos2 + 2cos( + )} and
{sin2sin2 + 2sin( + )}, then the length of the hypotenuse is:
  
(A) 2[1+cos()] (B) 2[1  cos()] (C) 4 cos2 (D) 4sin2
2 2

24. If x = sec  tan  & y = cosec + cot  then:

y1 1 x y 1
(A) x = y  1 (B) y = 1  x (C) x = y  1 (D) xy + x  y + 1 = 0

25. (a + 2) sin  + (2a – 1) cos  = (2a + 1) if tan  =

3 4 2a 2a
(A) (B) (C) (D)
4 3 a 1
2
a 1
2

2b
26. If tan x = , (a  c)
ac
y = a cos 2x + 2b sin x cos x + c sin2x
z = a sin2x – 2b sin x cos x + c cos 2x, then
(A) y = z (B) y + z = a + c (C) y – z = a – c (D) y – z = (a – c)2 + 4b2

 cos A  cos B   sin A  sin B 


n n

27.   +  
 sin A  sin B   cos A  cos B 

A B A B
(A) 2 tann (B) 2 cotn : n is even
2 2
(C) 0 : n is odd (D) none

28. The equation sin6x + cos6x = a2 has real solution if

 1  1 1 1 
(A) a  (–1, 1) (B) a    1,   (C) a     (D) a   , 1
 2  2 2 2 

1. The minute hand of a watch is 1.5 cm long. How far does its tip move in 50 minutes?
(Use  = 3.14).

2. If the arcs of the same length in two circles subtend angles 75° and 120° at the centre, find the ratio of
their radii.

RESONANCE 15
3. Sketch the following graphs :
x
(i) y = 3 sin 2x (ii) y = 2 tan x (iii) y = sin
2

 3    3  
4. Prove that cos     cos (2 + ) cot      cot (2  ) = 1.
 2    2  

 9 5
5. Prove that cos 2  cos – cos 3  cos = sin 5  sin .
2 2 2

3 3 x x
6. If tan x = ,<x< , find the value of sin and cos .
4 2 2 2

 2    
 1  cot  4   
    cos cot 4  9
7. prove that   sec = cosec 4.
1  cot 2     2  2
  4  

8. Prove that, sin 3 x. sin3 x + cos 3 x. cos 3 x = cos3 2 x.

p 1
9. If tan  = where  = 6   being an acute angle, prove that; (p cosec 2  q sec 2 ) = p 2  q2 .
q 2

tan   tan  sin 2  sin 2


10. If tan  = 1  tan . tan  , prove that sin 2 = 1  sin 2. sin 2 .

11. Show that:

(i) cot 7
1
2
or tan 82
1
2
=  3  2  2 1 or 2 3 4 6

1
(ii) tan 142 = 2 + 2 3 6 .
2

(iii) 
4 sin 27° = 5  5   3  5 
1/ 2 1/ 2

12. Prove that, tan  + 2 tan 2 + 4 tan 4 + 8 cot 8  = cot .

3
13. If cos () + cos () + cos () = , prove that
2
cos  + cos  + cos  = 0, sin  + sin  + sin  = 0.

sin 4  cos 4  1 sin8  cos 8  1


   
a  b3
14. Prove that from the equality follows the relation
a b ab a 3
b 3

15. Prove that: cosec  + cosec 2  + cosec 22 +... + cosec 2 n  1 = cot (/2)  cot 2n  1 . Hence or
4 8 16 32
otherwise prove that cosec + cosec + cosec + cosec =0
15 15 15 15

RESONANCE 16
1 1 1
16. Let A1, A2,......, An be the vertices of an nsided regular polygon such that; A A  A A  A A .
1 2 1 3 1 4

Find the value of n.

17. If A + B + C = , then prove that


A B C
(i) tan² + tan² + tan² 1
2 2 2

A B C 1
(ii) sin . sin . sin  .
2 2 2 8

3
(iii) cos A + cos B + cos C 
2

ax by ax sin  by cos 
18. If + = a 2 – b2, – = 0. Show that (ax)2/3 + (by)2/3 = (a2 – b2)2/3
cos  sin  cos 
2
sin2 

19. If Pn = cos n + sinn and Q n = cos n – sinn, then show that


Pn – Pn – 2 = – sin2 cos 2 Pn – 4
Q n – Q n – 2 = – sin2 cos2 Q n – 4
and hence show that
P4 = 1 – 2 sin2 cos 2
Q 4 = cos2 – sin2

20. If sin () = a & sin () = b (0 <, ,</2) then find the value of
cos2 ()  4 ab cos()

21. If A + B + C = , prove that


tan B tan C + tan C tan A + tan A tan B = 1 + sec A. sec B. sec C.

22. If tan2 + 2tan. tan2 = tan2 + 2tan. tan2, then prove that each side is equal to 1 or
tan  = ± tan .

EXERCISE # 1 EXERCISE # 2

1. D 2. B 3. A 4. D 5. B 6. C 7. B 1. 7.85 cm 2. r1 : r 2 = 8 : 5

8. A 9. B 10. B 11. B 12. B 13. B 14. A x 3 x 1


6. sin = and cos =–
2 10 2 10
15. C 16. A 17. B 18. D 19. B 20. BC

21. AB 22. BC 23. AC 24. BCD 25. BD 26. BC 16. n = 7 20. 1  2a2  2b2

27. BC 28. BD

RESONANCE 17
Trigonometric Equation

1. Trigonometric Equation :

An equation involving one or more trigonometric ratios of an


unknown angle is called a trigonometric equation.

2. Solution of Trigonometric Equation :

A solution of trigonometric equation is the value of the unknown


angle that satisfies the equation.
A man can do all things 1  3 9 11
e.g. if sin =  = , , , , ...........
2 4 4 4 4
if he but wills them.
Thus, the trigonometric equation may have infinite number of
Al bert us solutions (because of their periodic nature) and can be classified
as :
(i) Principal solution (ii) General solution.
Euclid taught me that
2.1 Principal solutions:
without assumptions
The solutions of a trigonometric equation which lie in the interval
there is no proof. [0, 2) are called Principal solutions.

Therefore, in any 1
e.g Find the Principal solutions of the equation sinx = .
2
argument, examine the Solution.
assumptions.

Eri c Templ e Bel l

1
sinx =
2

 there exists two values

 5 1
i.e. and which lie in [0, 2) and whose sine is
6 6 2

1 
 Principal solutions of the equation sinx = are ,
2 6

5
Ans.
6

RESONANCE 18
2.2 General Solution :

The expression involving an integer 'n' which gives all solutions of a trigonometric equation is called
General solution.

General solution of some standard trigonometric equations are given below.

3. General Solution of Some Standard Trigonometric Equations :

  
(i) If sin  = sin    = n  + (1)n  where    ,  , n  .
 2 2
(ii) If cos  = cos    = 2n ±  where   [0, ], n  .

  
(iii) If tan  = tan    = n +  where    ,  , n  .
 2 2
(iv) If sin²  = sin²    = n  ± , n  .

(v) If cos²  = cos²    = n  ± , n  .

(vi) If tan²  = tan²    = n  ± , n  . [ Note:  is called the principal angle ]

Some Important deductions :

(i) sin = 0   = n, n 


(ii) sin = 1   = (4n + 1) , n
2


(iii) sin = – 1   = (4n – 1) , n 
2


(iv) cos = 0   = (2n + 1) , n
2

(v) cos = 1   = 2n, n 


(vi) cos = – 1  = (2n + 1), n
(vii) tan = 0   = n, n 

Solved Example # 1

3
Solve sin  = .
2
Solution.

3
 sin  =
2


 sin = sin
3


  = n + (– 1)n , n  Ans.
3

RESONANCE 19
Solved Example # 2

2
Solve sec 2 = –
3
Solution.

2
 sec 2 = –
3

3 5
 cos2 = –  cos2 = cos
2 6

5
 2 = 2n ± , n 
6

5
  = n ± , n  Ans.
12

Solved Example # 3

Solve tan = 2

Solution.
 tan = 2 ............(i)
Let 2 = tan
 tan = tan
  = n + , where  = tan –1(2), n

Self Practice Problems:

1. Solve cot = – 1

1
2. Solve cos3 = –
2

 2n 2
Ans. (1)  = n – , n (2) ± , n 
4 3 9

Solved Example # 4

1
Solve cos2 =
2
Solution.

1
cos2 =
2

 1 
2

 cos  = 
2 
 2


 cos2 = cos2
4


  = n ± , n  Ans.
4

RESONANCE 20
Solved Example # 5

Solve 4 tan2 = 3sec2

Solution.
 4 tan2 = 3sec2 .............(i)

For equation (i) to be defined  (2n + 1) , n 
2
 equation (i) can be written as:

4 sin2  3 
= (2n + 1) , n 
cos  cos 2  2
2 

cos2 0
 4 sin2 = 3
 3
2

 sin2 =  

 2 

 sin2 = sin2
3

  = n ± , n  Ans.
3

Self Practice Problems :

1. Solve 7cos 2 + 3 sin2 = 4.

2. Solve 2 sin2x + sin22x = 2


  
Ans. (1) n ± , n  (2) (2n + 1) , n  or n ± , n
3 2 4

Types of Trigonometric Equations :


Type -1

Trigonometric equations which can be solved by use of factorization.

Solved Example # 6

Solve (2sinx – cosx) (1 + cosx) = sin2x.

Solution.
 (2sinx – cosx) (1 + cosx) = sin2x
 (2sinx – cosx) (1 + cosx) – sin2x = 0
 (2sinx – cosx) (1 + cosx) – (1 – cosx) (1 + cosx) = 0
 (1 + cosx) (2sinx – 1) = 0
 1 + cosx = 0 or 2sinx – 1 = 0
1
 cosx = – 1 or sinx =
2

 
 x = (2n + 1), n or sin x = sin  x = n + (– 1) n , n 
6 6
 Solution of given equation is

(2n + 1), n   or n + (–1)n , n  Ans.
6

RESONANCE 21
Self Practice Problems :

x
1. Solve cos 3x + cos2x – 4cos 2 =0
2

2. Solve cot2 + 3cosec + 3 = 0

Ans. (1) (2n + 1), n 


 
(2) 2n – , n  or n + (–1) n + 1 , n 
2 6
Type - 2

Trigonometric equations which can be solved by reducing them in quadratic equations.

Solved Example # 7

Solve 2 cos2x + 4cosx = 3sin2x

Solution.
 2cos2x + 4cosx – 3sin2x = 0
 2cos2x + 4cosx – 3(1– cos 2x) = 0
 5cos2x + 4cosx – 3 = 0

   2  19    
cos x    cos x    2  19 
     = 0 ........(ii)
  5    5 
 cosx  [– 1, 1]  x  R

 2  19
 cosx 
5
 equation (ii) will be true if

 2  19
cosx =
5

 2  19
 cosx = cos, where cos =
5

  2  19 
 x = 2n ±  where  = cos–1   , n 
 Ans.
 5 

Self Practice Problems :

 1 
1. Solve cos2 – ( 2 + 1)  cos    = 0
 2

2. Solve 4cos – 3sec = tan


 
Ans. (1) 2n ± , n  or 2n ± , n 
3 4

  1  17 
(2) n + (– 1)n  where  = sin–1   , n 

 8 

  1  17 
or n + (–1)n  where  = sin–1   , n 

 8 

RESONANCE 22
Type - 3

Trigonometric equations which can be solved by transforming a sum or difference of trigonometric


ratios into their product.

Solved Example # 8
Solve cos3x + sin2x – sin4x = 0

Solution.
cos3x + sin2x – sin4x = 0  cos3x + 2cos3x.sin(– x) = 0
 cos3x – 2cos3x.sinx = 0  cos3x (1 – 2sinx) = 0
 cos3x = 0 or 1 – 2sinx = 0
 1
 3x = (2n + 1) , n  or sinx =
2 2
 
 x = (2n + 1) , n  or x = n + (–1)n , n 
6 6
 solution of given equation is
 
(2n + 1) , n  or n + (–1)n , n Ans.
6 6
Self Practice Problems :

1. Solve sin7= sin3 + sin

2. Solve 5sinx + 6sin2x +5sin3x + sin4x = 0

3. Solve cos – sin3 = cos2


n n 
Ans. (1) , n  or ± , n 
3 2 12
n 2
(2) , n  or 2n ± , n
2 3
2n  
(3) , n  or 2n – , n or n + , n 
3 2 4

Type - 4
Trigonometric equations which can be solved by transforming a product of trigonometric ratios into their
sum or difference.

Solved Example # 9
Solve sin5x.cos3x = sin6x.cos2x
Solution.
 sin5x.cos3x = sin6x.cos2x  2sin5x.cos3x = 2sin6x.cos2x
 sin8x + sin2x = sin8x + sin4x  sin4x – sin2x = 0
 2sin2x.cos2x – sin2x = 0  sin2x (2cos2x – 1) = 0
 sin2x = 0 or 2cos2x – 1 = 0
1
 2x = n, n  or cos2x =
2
n 
 x= , n  or 2x = 2n ± , n 
2 3


 x = n ± , n
6
 Solution of given equation is
n 
, n   or n ± , n   Ans.
2 6

RESONANCE 23
Type - 5

Trigonometric Equations of the form a sinx + b cosx = c, where a, b, c  R, can be solved by dividing

both sides of the equation by a2  b2 .

Solved Example # 10

Solve sinx + cosx = 2

Solution.

 sinx + cosx = 2 ..........(i)


Here a = 1, b = 1.

 divide both sides of equation (i) by 2 , we get

1 1
sinx . + cosx. =1
2 2

 
 sinx.sin + cosx.cos =1
4 4

 
 cos  x   = 1
 4


 x– = 2n, n 
4


 x = 2n + , n 
4
 Solution of given equation is

2n + , n   Ans.
4

Note : Trigonometric equation of the form a sinx + b cosx = c can also be solved by changing sinx and cosx
into their corresponding tangent of half the angle.

Solved Example # 11

Solve 3cosx + 4sinx = 5

Solution.
 3cosx + 4sinx = 5 .........(i)

x x
1  tan 2 2 tan
2 2
cosx = & sinx =
2 x x

1  tan 1  tan 2
2 2

 equation (i) becomes

 x  x 
 1  tan
2
  2 tan 
2 
+4 
2
 3  =5 ........(ii)
 x  2 x 
 1  tan
2
  1  tan 
 2  2

RESONANCE 24
x
Let tan =t
2
 equation (ii) becomes

 1 t2 
 + 4 
2t 
3  2 
 =5
 1 t   1  t2 

 4t2 – 4t + 1 = 0
 (2t – 1)2 = 0
1 x
 t= t = tan
2 2

x 1
 tan =
2 2

x 1
 tan = tan, where tan =
2 2

x
 = n + 
2

 1
 x = 2n + 2 where  = tan–1   , n   Ans.
2

Self Practice Problems :

1. Solve 3 cosx + sinx = 2

x
2. Solve sinx + tan =0
2


Ans. (1) 2n + , n  (2) x = 2n, n 
6

Type - 6

Trigonometric equations of the form P(sinx ± cosx, sinx cosx) = 0, where p(y, z) is a polynomial, can
be solved by using the substitution sinx ± cosx = t.

Solved Example # 12

Solve sinx + cosx = 1 + sinx.cosx

Solution.
 sinx + cosx = 1 + sinx.cosx ........(i)
Let sinx + cosx = t
 sin2x + cos2x + 2 sinx.cosx = t2

t2  1
 sinx.cosx =
2

t2  1
Now put sinx + cosx = t and sinx.cosx = in (i), we get
2

t2  1
t=1+
2

RESONANCE 25
 t2 – 2t + 1 = 0
 t=1  t = sinx + cosx
 sinx + cosx = 1 .........(ii)
divide both sides of equation (ii) by 2 , we get

1 1 1
 sinx. + cosx. =
2 2 2

  
 cos  x   = cos
 4 4

 
 x– = 2n ±
4 4
(i) if we take positive sign, we get

x = 2n + , n   Ans.
2
(ii) if we take negative sign, we get
x = 2n, n   Ans.

Self Practice Problems:

1. Solve sin2x + 5sinx + 1 + 5cosx = 0

2. Solve 3cosx + 3sinx + sin3x – cos3x = 0

3. Solve (1 – sin2x) (cosx – sinx) = 1 – 2sin2x.


 
Ans. (1) n – , n  (2) n – , n
4 4

 
(3) 2n + , n  or 2n, n  or n + , n
2 4
Type - 7

Trigonometric equations which can be solved by the use of boundness of the trigonometric ratios
sinx and cosx.

Solved Example # 13

 x   x 
Solve sinx  cos  2 sin x  +  1  sin  2 cos x  cos x = 0
 4   4 
Solution.

 x   x 
sinx  cos  2 sin x  + 1  sin  2 cos x  cos x = 0 .......(i)
 4 

 4 

x x
 sinx.cos – 2sin2x + cosx + sin .cosx – 2cos2x = 0
4 4

 x x 
  sin x. cos  sin . cos x  – 2 (sin2x + cos2x) + cosx = 0
 4 4 

5x
 sin + cosx = 2 ........(ii)
4

RESONANCE 26
Now equation (ii) will be true if
5x
sin =1 and cosx = 1
4

5x 
 = 2n + , n  and x = 2m, m
4 2

(8n  2)
 x = , n  ........(iii) and x = 2m, m  ........(iv)
5
Now to find general solution of equation (i)
(8n  2)
= 2m
5
 8n + 2 = 10m
5m  1
 n=
4
if m=1 then n=1
if m=5 then n=6
......... ......... .........
......... ......... .........
if m = 4p – 3, p  then n = 5p – 4, p

 general solution of given equation can be obtained by substituting either m = 4p – 3 in


equation (iv) or n = 5p – 4 in equation (iii)

 general solution of equation (i) is


(8p – 6), p   Ans.

Self Practice Problems :

1. Solve sin3x + cos2x = – 2

2. Solve 3 sin 5 x  cos 2 x  3 = 1 – sinx

 
Ans. (1) (4p – 3) , p  (2) 2m + , m
2 2

RESONANCE 27
Part : (A) Only one correct option

1. The solution set of the equation 4sin.cos – 2cos – 2 3 sin + 3 = 0 in the interval (0, 2) is

 3 7    5   3  5    5  11 
(A)  ,  (B)  ,  (C)  , , ,  (D)  , , 
4 4 3 3   4 3 3  6 6 6 

2. All solutions of the equation, 2 sin + tan = 0 are obtained by taking all integral values of m and n in:
2 2
(A) 2n + , n  (B) n or 2m  ± where n, m 
3 3

 
(C) n or m  ± where n, m  (D) n or 2m  ± where n, m 
3 3

7 
3. If 20 sin2  + 21 cos  24 = 0 & <  < 2 then the values of cot is:
4 2

15 15
(A) 3 (B) (C)  (D)  3
3 3

4. The general solution of sinx + sin5x = sin2x + sin4x is:


(A) 2 n ; n  (B) n ; n   (C) n/3 ; n   (D) 2 n/3 ; n  

1
5. A triangle ABC is such that sin(2A + B) = . If A, B, C are in A.P. then the angle A, B, C are
2
respectively.

5     5   5  5 
(A) , , (B) , , (C) , , (D) , ,
12 4 3 4 3 12 3 4 12 3 12 4

6. The maximum value of 3sinx + 4cosx is


(A) 3 (B) 4 (C) 5 (D) 7

7. If sin  + 7 cos  = 5, then tan (/2) is a root of the equation


(A) x 2  6x + 1 = 0 (B) 6x2  x  1 = 0 (C) 6x 2 + x + 1 = 0 (D) x 2  x + 6 = 0

sin 3   cos 3  cos 


8.   2 tan  cot  =  1 if:
sin   cos  1  cot 2 

     3   3 
(A)   0 ,  (B)   ,  (C)    ,  (D)   , 2 
 2 2   2  2 

9. The number of integral values of a for which the equation cos 2x + a sin x = 2a  7 possesses a solution
is
(A) 2 (B) 3 (C) 4 (D) 5

10. The principal solution set of the equation, 2 cos x = 2  2 sin 2 x is

  13     13     13     13  
(A)  ,  (B)  ,  (C)  ,  (D)  , 
8 8  4 8   4 10   8 10 

RESONANCE 28
11. The number of all possible triplets (a1, a2, a3) such that : a1 + a2 cos 2x + a3 sin2x = 0 for all x is
(A) 0 (B) 1 (C) 2 (D) infinite

 n 
If 2tan2x – 5 secx – 1 = 0 has 7 different roots in 0, , n  N, then greatest value of n is
2 
12.

(A) 8 (B) 10 (C) 13 (D) 15

13. The solution of |cosx| = cosx – 2sinx is



(A) x = n, n  (B) x = n + , n 
4

 
(C) x = n + (–1) n , n  (D) (2n + 1) + , n 
4 4

14. The arithmetic mean of the roots of the equation 4cos3x – 4cos2x – cos( + x) – 1 = 0 in the interval
[0, 315] is equal to
(A) 49 (B) 50 (C) 51 (D) 100

15. Number of solutions of the equation cos 6x + tan2 x + cos 6x . tan2 x = 1 in the interval [0, 2] is :
(A) 4 (B) 5 (C) 6 (D) 7

Part : (B) May have more than one options correct

16. sinx  cos 2x 1 assumes the least value for the set of values of x given by:
(A) x = n + (1)n+1 (/6) , n  (B) x = n + (1)n (/6) , n 
(C) x = n + (1)n (/3), n  (D) x = n (1)n (/6) , n 

17. cos4x cos8x  cos5x cos9x = 0 if


(A) cos12x = cos 14 x (B) sin13 x = 0
(C) sinx = 0 (D) cosx = 0

x x
18. The equation 2sin . cos2x + sin2x = 2 sin . sin2x + cos 2x has a root for which
2 2
1 1
(A) sin2x = 1 (B) sin2x = – 1 (C) cosx = (D) cos2x = –
2 2
19. sin2x + 2 sin x cos x  3cos 2x = 0 if
(A) tan x =  (B) tanx =  1
(C) x = n + /4, n  (D) x = n + tan1 (3), n 

20. sin2x  cos 2x = 2  sin 2x if


(A) x = n/2, n  (B) x = n  /2, n 
(C) x = (2n + 1) /2, n  (D) x = n + (1)n sin1 (2/3), n 

1. Solve cot = tan8

x x
2. Solve cot   – cosec   = cotx
 
2 2

 1 
3. Solve cot2 +  3   cot + 1 = 0.

 3

4. Solve cos2 + 3 cos = 0.

RESONANCE 29
5. Solve the equation: sin 6x = sin 4x  sin 2x .

6. Solve: cos  + sin = cos 2 + sin 2.

7. Solve 4 sin x . sin 2x . sin 4x = sin 3x .

8. Solve sin2n – sin2(n – 1) = sin2, where n is constant and n  0, 1

9. Solve tan + tan2 + 3 tan tan2 = 3.

10. Solve: sin3 x cos 3 x + cos3 x sin 3 x + 0.375 = 0

sin 3 x  cos 3 x cos x


11. Solve the equation, 2 2 = .
2  sin x 3

12. Solve the equation: sin 5x = 16 sin5 x .

3 7
13. If tan  + sin = & tan² + cos² = then find the general value of  & .
2 4

14. Solve for x, the equation 13  18 tanx = 6 tan x  3, where  2  < x < 2  .

15. Find the general solution of sec 4  sec 2 = 2 .

3
16. Solve the equation sin x  cos x = cos² x .
2

 
17. Solve for x: 2 sin  3 x    1  8 sin 2 x . cos 2 2 x .
 4

 
 
Solve the equation for 0  2 ; sin 2  3 cos2  5 = cos   2  .
2
18.
 6 

19. Solve: tan2 x . tan2 3 x . tan 4 x = tan2 x  tan2 3 x + tan 4 x .

3x x
20. Find the values of x, between 0 & 2  satisfying the equation; cos 3x + cos 2x = sin + sin .
2 2

2x
21. Solve: cos cos 6 x =  1 .
3

22. Solve the equation, sin2 4 x + cos2 x = 2 sin 4 x cos4 x .

RESONANCE 30
EXERCISE # 1 n 
10. x = + ( 1)n + 1 , n
4 24
1. D 2. B 3. D 4. C 5. B 6. C 7. B

8. B 9. D 10. A 11. D 12. D 13. D 14. C 


11. x = (4 n + 1) ,n
2
15. D 16. AD 17. ABC 18. ABCD 19. CD


20. BC 12. x = n  ; x = n  ± ,n
6

EXERCISE # 2
 
13.  = n + ,  = n + (1)n , n  I
4 6
 1 
1.  n   , n 
 2  9
2
14.   2    , ,  + , where tan  =
3
2
2. x = 4n ± , n 
3
2n  
15. ± or 2n ± , n 
5 10 2
 
3.  = n – , n  or n – , n 
3 6

16. x = (2 n + 1), , n   or 2 n  ± , n 
3

 17  3   
4. 2n ±  where  = cos –1   , n 
,   or x = (24k – 7) , k 
 4  17. (24 + 1)
  12 12

n  7  19 
5. , n  or n ± , n   18.  = ,
4 6 12 12

(2 n  1) 
19. k , where n, k 
2n  4
6. 2 n , n or  + , n
3 6

 5 9  13 
20. , ,, ,
7 7 7 7
n 
7. x = n , n  or ± , n  
3 9
21. 

m  1 
8. m, m  or , m  or  m   , m  
n 1  2 n 22. x = (2 n + 1) , nI
2

 1 
9. n   , n 
 3 3

RESONANCE 31
Properties & Solution of Triangle

1. Sine Rule:
In any triangle ABC, the sines of the angles are proportional
a b c
According to most to the opposite sides i.e.   .
sin A sinB sin C
accounts, geometry
was first discovered
 A B
among the Egyptians, cos  
ab  2 
taking its origin from Example : In any ABC, prove that = C .
c sin
the measurement of 2
areas. For they found
it necessary by
 A B
reason of the cos  
ab  2 
flooding of the Nile, Solution. We have to prove = C .
c

which wiped out sin
2
everybody's proper
boundaries. Nor is  From sine rule, we know that
there anything a b c
= = = k (let)
surprising in that the sin A sin B sin C
discovery both of this  a = k sinA, b = k sinB and c = k sinC
and of the other
ab
sciences should have L.H.S. =
c

had its origin in a
practical need, since k(sin A  sin B )
everything which is =
k sin C
in process of
becoming progresses  A B  A B
sin   cos  
from the imperfect to  2   2 
= C C
the perfect.
sin cos
2 2

Proclus C  A B
cos cos  
2  2 
= C C
sin cos
2 2

 A B
cos  
 2 
= C
sin
2
= R.H.S.
Hence L.H.S. = R.H.S. Proved

RESONANCE 32
Example : In any ABC, prove that
(b2 – c2) cot A + (c 2 – a2) cot B + (a2 – b2) cot C = 0
Solution.  We have to prove that
(b2 – c2) cot A + (c 2 – a2) cot B + (a2 – b2) cot C = 0
 from sine rule, we know that
a = k sinA, b = k sinB and c = k sinC
 (b2 – c2) cot A = k 2 (sin2B – sin2C) cot A
 sin2B – sin2C = sin (B + C) sin (B – C)
 (b2 – c 2) cot A = k 2 sin (B + C) sin (B – C) cotA  B+C=–A
cos A
 (b2 – c 2) cot A = k 2 sin A sin (B – C) cosA = – cos(B + C)
sin A

= – k 2 sin (B – C) cos (B + C)

k2
=– [2sin (B – C) cos (B + C)]
2
k2
 (b2 – c2) cot A = – [sin 2B – sin 2C] ..........(i)
2
k2
Similarly (c 2 – a2) cot B = –
[sin 2C – sin 2A] ..........(ii)
2
k2
and (a2 – b2) cot C = – [sin 2A – sin 2B] ..........(iii)
2
adding equations (i), (ii) and (iii), we get
(b2 – c2) cot A + (c 2 – a2) cot B + (a2 – b2) cot C = 0 Hence Proved
Self Practice Problems

In any ABC, prove that


A  A
1. a sin   B  = (b + c) sin   .
 2  2

a 2 sin(B  C) b 2 sin(C  A ) c 2 sin( A  B)


2. + + =0
sin B  sin C sin C  sin A sin A  sin B

A B
c tan  tan
2 2
3. = .
ab A B
tan  tan
2 2

2. Cosine Formula:
b 2  c2  a 2
(i) cos A = or a² = b² + c²  2bc cos A = b2 + c 2 + 2bc cos (B + C)
2bc

c2  a 2  b 2 a 2  b 2  c2
(ii) cos B = (iii) cos C =
2 ca 2a b

Example : In a triangle ABC if a = 13, b = 8 and c = 7, then find sin A.

b2  c 2  a2 64  49  169
Solution. cosA = =
2bc 2 .8 .7

1 2
 cosA = –  A=
2 3

2 3
 sinA = sin = Ans.
3 2

RESONANCE 33
*Example : In a ABC, prove that a(b cos C – c cos B) = b2 – c2

Solution.  We have to prove a (b cosC – c cosB) = b2 – c 2.


 from cosine rule we know that

a2  b2  c 2 a2  c 2  b2
cosC = & cos B =
2ab 2ac

  a 2  b 2  c 2   2 
 c a c b
2 2
 b  

    
L.H.S. = a 2ab 2ac
  

a2  b2  c 2 (a 2  c 2  b 2 )
= –
2 2
= (b2 – c2)
= R.H.S.
Hence L.H.S. = R.H.S. Proved

 a b  c a
Example : If in aABC, A = 60° then find the value of 1    1    .
 c c  b b
Solution.  A = 60°

 a b  c a c ab bc a


1    1    =    
 c c   b b   c   b 

(b  c )2  a 2
=
bc

(b 2  c 2  a 2 )  2bc
=
bc

b2  c 2  a2
= +2
bc

 b2  c 2  a 2 

=2 
 +2
2bc 
 

1
= 2cosA + 2  A = 60°  cos A =
2

 a b  c a
 1    1    = 3 Ans.
 c c  b b

Self Practice Problems :

1. The sides of a triangle ABC are a, b, a 2  ab  b 2 , then prove that the greatest angle is 120°.

A
2. In a triangle ABC prove that a(cosB + cosC) = 2(b + c) sin2 .
2
3. Projection Formula:

(i) a = b cosC + c cosB (ii) b = c cosA + a cosC


(iii) c = a cosB + b cosA

RESONANCE 34
Example : In a triangle ABC prove that a(b cosC – c cosB) = b2 – c2

Solution.  L.H.S. = a (b cosC – c cosB)


= b (a cosC) – c (a cosB) ............(i)
 From projection rule, we know that
b = a cosC + c cosA  a cosC = b – c cosA
& c = a cosB + b cosA  a cosB = c – b cosA
Put values of a cosC and a cosB in equation (i), we get
L.H.S. = b (b – ccos A) – c(c – b cos A)
= b2 – bc cos A – c 2 + bc cos A
= b2 – c2
= R.H.S.
Hence L.H.S. = R.H.S. Proved

Note: We have also proved a (b cosC – ccosB) = b2 – c2 by using cosine – rule in solved *Example.

Example : In a ABC prove that (b + c) cos A + (c + a) cos B + (a + b) cos C = a + b + c.

Solution.  L.H.S. = (b + c) cos A (c + a) cos B + (a + B) cos C


= b cos A + c cos A + c cos B + a cos B + a cos C + b cos C
= (b cos A + a cos B) + (c cos A + a cos C) + (c cos B + b cos C)
= a+b+c
= R.H.S.
Hence L.H.S. = R.H.S. Proved

Self Practice Problems

In a ABC, prove that

 2 C B
1. 2  b cos  c cos 2  = a + b + c.
 2 2

cos B c  b cos A
2. = .
cos C b  c cos A

cos A cos B cos C a2  b2  c 2


3. + + = .
c cos B  b cos C a cos C  c cos A a cos B  b cos A 2abc

4. Napier’s Analogy - tangent rule:


BC bc A CA c a B
(i) tan = cot (ii) tan = cot
2 bc 2 2 c a 2

AB ab C
(iii) tan = cot
2 ab 2
Example : Find the unknown elements of the ABC in which a = 3 + 1, b = 3 – 1, C = 60°.

Solution.  a= 3 + 1, b = 3 – 1, C = 60°
 A + B + C = 180°
 A + B = 120° .......(i)
 From law of tangent, we know that

 A B  ab C
tan   = cot
 2  a  b 2

RESONANCE 35
( 3  1)  ( 3  1)
= cot 30°
( 3  1)  ( 3  1)

2
= cot 30°
2 3

 A B 
 tan   =1
 2 
A B 
 = = 45°
2 4
 A – B = 90° .......(ii)
From equation (i) and (ii), we get
A = 105° and B = 15°
Now,
a b c
From sine-rule, we know that = =
sin A sin B sin C

a sin C ( 3  1) sin 60


 c= =
sin A sin105

3
( 3  1) 3 1
2
= sin105° =
3 1 2 2

2 2

 c= 6
 c= 6 , A = 105°, B = 15° Ans.

Self Practice Problem

7 A
1. In a ABC if b = 3, c = 5 and cos (B – C) = , then find the value of tan .
25 2

1
Ans.
3

B C A CA B  A B  C
2. If in a ABC, we define x = tan   tan , y = tan   tan and z = tan   tan
 2  2  2  2  2  2
then show that x + y + z = – xyz.

5. Trigonometric Functions of Half Angles:


A (s  b) (s  c) B (s  c) (s  a ) C (s  a ) (s  b)
(i) sin = ; sin = ; sin =
2 bc 2 ca 2 ab

A s (s  a ) B s (s  b ) C s (s  c)
(ii) cos = ; cos = ; cos =
2 b c 2 ca 2 ab

A (s  b) (s  c)  abc
(iii) tan = = where s = is semi perimetre of triangle.
2 s (s  a ) s (s  a ) 2

2 2
(iv) sin A = s(s  a )(s  b)(s  c) =
bc bc

RESONANCE 36
6. Area of Triangle()

1 1 1
= ab sin C = bc sin A = ca sin B = s (s  a ) (s  b) (s  c)
2 2 2

A C
Example : In a ABC if a, b, c are in A.P. then find the value of tan . tan .
2 2

A  C 
Solution. tan = and tan =
2 s(s  a) 2 s(s  c )

A C 2
 tan . tan = 2 2 = s (s – a) (s – b) (s – c)
2 2 s (s  a)(s  c )

A C sb b
 tan . tan = =1– ........(i)
2 2 s s
 it is given that a, b, c are in A.P.
 2b = a + c
abc 3b
s= =
2 2

b 2
 = put in equation (i)
s 3

A C 2
 tan . tan =1–
2 2 3

A C 1
 tan . tan = Ans.
2 2 3

Example : In a ABC if b sinC(b cosC + c cosB) = 42, then find the area of the ABC.

Solution.  b sinC (b cosC + c cosB) = 42 ........(i) given


 From projection rule, we know that
a = b cosC + c cosB put in (i), we get
ab sinC = 42 ........(ii)
1
= ab sinC
2

  = 21 sq. unit Ans.

 A B C
Example : In any ABC prove that (a + b + c)  tan  tan  = 2c cot .
 2 2  2

 A B
Solution. L.H.S. = (a + b + c)  tan  tan 
 2

2

A (s  b)(s  c ) B (s  a)(s  c )
tan = and tan =
2 s(s  a) 2 s(s  b)

 (s  b)(s  c ) (s  a)(s  c ) 
 L.H.S. = (a + b + c)   
 s(s  a) s(s  b) 

RESONANCE 37
sc  sb sa
= 2s   
s  s  a s  b 

 sbsa 
=2 s( s  c )   2s= a + b + c
 (s  a)(s  b) 

 2s – b – a = c

 c 
=2 s( s  c )  
 (s  a)(s  b) 

s(s  c ) C s(s  c )
= 2c cot =
(s  a)(s  b) 2 (s  a)(s  b)

C
= 2c cot
2
= R.H.S.
Hence L.H.S. = R.H.S. Proved

7. m - n Rule:

(m + n) cot   m cot   n cot 


 n cot B  m cot C

Example : If the median AD of a triangle ABC is perpendicular to AB, prove that tan A + 2tan B = 0.

Solution. From the figure, we see that  = 90° + B (as  is external angle of ABD)

Now if we apply m-n rule in ABC, we get


(1 + 1) cot (90 + B) = 1. cot 90° – 1.cot (A – 90°)
 – 2 tan B = cot (90° – A)
 – 2 tan B = tan A
 tan A + 2 tan B = 0 Hence proved.

Example : The base of a triangle is divided into three equal parts. If t1, t2, t3 be the tangents of the angles
subtended by these parts at the opposite vertex, prove that

 1 1 1 1 1
4 1  2  =      .
t 
 t2   t1 t 2   2 t3 

Solution. Let point D and E divides the base BC into three equal parts i.e. BD = DE = DC = d (Let) and
let ,  and  be the angles subtended by BD, DE and EC respectively at their opposite vertex.

RESONANCE 38
 t1 = tan, t2 = tan and t3 = tan
Now in ABC
 BE : EC = 2d : d = 2 : 1
 from m-n rule, we get
(2 + 1) cot = 2 cot ( + ) – cot
 3cot = 2 cot ( + ) – cot .........(i)
again
 in ADC
 DE : EC = x : x = 1 : 1
 if we apply m-n rule in ADC, we get
(1 + 1) cot = 1. cot – 1 cot
2cot = cot – cot .........(ii)
from (i) and (ii), we get

3 cot  2 cot(  )  cot 


=
2 cot  cot   cot 
 3cot – 3cot = 4cot ( + ) – 2 cot
 3cot – cot = 4 cot ( + )

 cot . cot   1
 3cot – cot = 4  
 cot   cot  
 3cot2+ 3cot cot – cot cot – cot cot = 4 cot cot – 4
 4 + 3cot2 = cot cot + cot cot + cot cot
 4 + 4cot2 = cot cot + cot cot + cot cot + cot2
 4(1 + cot2) = (cot + cot) (cot + cot)

 1   1 1   1 1 
 4 1   
2  =  tan 
    
 tan    tan    tan  tan  

 1 1 1 1 1
 4 1  2  =       Hence proved
 t2   t1 t 2   t 2 t 3 

Self Practice Problems :


1
1. In a ABC, the median to the side BC is of length and it divides angle A into the angles of
11  6 3
30° and 45°. Prove that the side BC is of length 2 units.

8. Radius of Circumcirlce :
a b c a bc
R=   =
2 sinA 2 sinB 2 sinC 4

s
Example : In a ABC prove that sinA + sinB + sinC =
R
Solution. In a ABC, we know that
a b c
= = = 2R
sin A sin B sin C

a b c
 sin A = , sinB = and sinC = .
2R 2R 2R

abc
 sinA + sinB + sinC = a + b + c = 2s
2R

2s s
=  sinA + sinB + sinC = .
2R R

RESONANCE 39
Example : In a ABC if a = 13 cm, b = 14 cm and c = 15 cm, then find its circumradius.
abc
Solution. R= .......(i)
4

 = s( s  a)(s  b)(s  c )

abc
s= = 21 cm
2

 = 21.8.7.6 = 7 2.4 2.3 2


  = 84 cm 2
13.14.15 65
 R= = cm
4.84 8
65
 R= cm.
8
A B C
Example : In a ABC prove that s = 4R cos . cos . cos .
2 2 2
Solution. In a ABC,

A s( s  a) B s( s  b) C s( s  c ) abc
cos = , cos = and cos = and R =
2 bc 2 ca 2 ab 4

A B C
R.H.S. = 4R cos . cos . cos .
2 2 2

abc s(s  a)(s  b)(s  c )


= .s = s( s  a)(s  b)(s  c )
 (abc )2

= s
= L.H.S.
Hence R.H.L = L.H.S. proved
1 1 1 1 4R
Example : In a ABC, prove that + + – = .
sa sb sc s 

1 1 1 1 4R
Solution. + + – =
sa sb sc s 

 1 1   1 1
L.H.S. =    +   
sa sb sc s

2s  a  b (s  s  c )
= + 2s = a + b + c
(s  a)(s  b) s(s  c )

c c
= +
(s  a)(s  b) s(s  c )

 s(s  c )  (s  a)(s  b)   2s 2  s(a  b  c )  ab 


=c   =c  
 s(s  a)(s  b)(s  c )   2 

 2s 2  s(2s)  ab  abc 4R 4R abc


 L.H.S. = c   = 2 = = R=
    4

 
2 2

 abc = 4R
4R
 L.H.S. =

RESONANCE 40
Self Practice Problems :

In a ABC, prove the followings :

1. a cot A + b cotB + cos C = 2(R + r).

s  s  s  r
2. 4   1   1   1 = .
 a   b   c  R

3. If , , are the distances of the vertices of a triangle from the corresponding points of contact with the
 y
incircle, then prove that = r2
 y

9. Radius of The Incircle :


 A B C
(i) r = (ii) r = (s  a) tan = (s  b) tan = (s  c) tan
s 2 2 2
a sin B2 sin C2 A B C
(iii) r = & so on (iv) r = 4R sin sin sin
cos A2 2 2 2

10. Radius of The Ex- Circles :


 ;  ;  A B C
(i) r1 = r2 = r3 = (ii) r 1 = s tan ;
r 2 = s tan ;
r 3 = s tan
sa sb sc 2 2 2

a cos B2 cos C2 A B C
(iii) r 1 = & so on (iv) r1 = 4 R sin . cos . cos
cos A2 2 2 2
Example : In a ABC, prove that r1 + r2 + r 3 – r = 4R = 2a cosecA

Solution.  L.H.S = r1 + r2 + r3 – r
   
= + + –
sa sb sc s

 1 1   1 1
=    +   
sa sb sc s

 s  b  s  a   s  s  c 
=   (s  a)(s  b)    s(s  c ) 
   

 c c 
=    
 ( s  a )( s  b ) s( s  c) 

 s(s  c )  (s  a)(s  b) 
= c  
 s(s  a)(s  b)(s  c ) 
 2s 2  s(a  b  c )  ab 
= c   a + b + c = 2s
 2 

abc abc
= R=
 4

a
= 4R = 2acosecA = 2R = acosecA
sin A

= R.H.S.
Hence L.H.S. = R.H.S. proved

RESONANCE 41
Example : If the area of a ABC is 96 sq. unit and the radius of the escribed circles are respectively
8, 12 and 24. Find the perimeter of ABC.

Solution.   = 96 sq. unit


r1 = 8, r 2 = 12 and r 3 = 24

r1 =  s – a = 12 .........(i)
sa


r2 =  s–b=8 .........(ii)
sb


r3 =  s–c=4 .........(iii)
sc

 adding equations (i), (ii) & (iii), we get


3s – (a + b + c) = 24
s = 24
 perimeter of ABC = 2s = 48 unit.

Self Practice Problems

In a ABC prove that

1. r 1r 2 + r 2r 3 + r 3r 1 = s 2

2. rr1 + rr2 + rr3 = ab + bc + ca – s2

3. If A, A1, A2 and A3 are the areas of the inscribed and escribed circles respectively of a ABC, then prove

1 1 1 1
that = + + .
A A1 A2 A3

r1  r r2  r c
4. + = r .
a b 3

11. Length of Angle Bisectors, Medians & Altitudes :

2 bc cos A
2
(i) Length of an angle bisector from the angle A =  a = ;
bc

1
(ii) Length of median from the angle A = m a = 2 b2  2 c2  a 2
2

2
& (iii) Length of altitude from the angle A = Aa =
a

3 2
NOTE : ma  m b  m c = (a + b2 + c 2)
2 2 2
4

RESONANCE 42
Example : AD is a median of the ABC. If AE and AF are medians of the triangles ABD and ADC
a2
respectively, and AD = m 1, AE = m 2 , AF = m 3 , then prove that m 22 + m 32 – 2m 12 = .
8
Solution.  In ABC
1
AD2 = (2b2 + 2c2 – a2) = m 12 .........(i)
4
1 a2
In ABD, AE2 = m 22 = (2c2 + 2AD2 – ) .........(ii)
4

4
1  2AD 2  2b 2  a 
2

Similarly in ADC, AF = m 3 =
2

2
4  ........(iii)
4 
by adding equations (ii) and (iii), we get
 2 
m 22 + m 32 =
1  4AD 2  2b 2  2c 2  a 

4  2 

 2 2 
1  2b  2c 2  a 
= AD +  2 
2
4 
 2 2 
1  2b  2c 2  a 2  a 
= AD +  2 
2
4 
1 a2
= AD2 + (2b2 + 2c 2 – a2) +
4 8
a2
= AD2 + AD2 +
8
a2
= 2AD2 + AD2 = m 12
8

a2
= 2m 12 +
8
a2
 m 22 + m 32 – 2m 12 = Hence Proved
8

Self Practice Problem :

3. In a ABC a = 5, b = 4, c = 3. ‘G’ is the centroid of triangle, then find circumradius of GAB.


5
Ans. 13
12

12. The Distances of The Special Points from Vertices and Sides of
Triangle:
(i) Circumcentre (O) : OA = R & O a = R cos A

A
(ii) Incentre (I) : IA = r cosec & Ia = r
2
A
(iii) Excentre (I1) : I1 A = r 1 cosec & I1a = r 1
2

(iv) Orthocentre (H) : HA = 2R cos A & Ha = 2R cos B cos C

1 2
(v) Centroid (G) : GA = 2b2 2c2 a 2 & Ga =
3 3a

RESONANCE 43
Example : If x, y and z are respectively the distances of the vertices of the ABC from its orthocentre,
then prove that

a b c abc
(i) + + = (ii) x y + z = 2(R + r)
x y z xyz
Solution.  x = 2R cosA, y = 2R cosB, z = 2R cosC and
and a = 2R sinA, b = 2R sinB, c = 2R sinC
a b c
 + + = tanA + tan B + tan C .........(i)
x y z

abc
& xyz = tanA. tanB. tanC ........(ii)

 We know that in a ABC tanA =  tanA


 From equations (i) and (ii), we get

a b c abc
+ + =
x y z xyz

 x + y + z = 2R (cosA + cosB + cosC)

A B C
in a ABC cosA + cosB + cosC = 1 + 4sin sin sin
2 2 2

 A B C
 x + y + z = 2R 1  4 sin . sin . sin 
 2 2 2

 A B C A B C
= 2  R  4R sin . sin . sin  r = 4R sin sin sin
 2 2 2

2 2 2

 x + y + z = 2(R + r)

Self Practice Problems

A B C
1. If be the incentre of ABC, then prove that A .B .C = abc tan tan tan .
2 2 2

2. If x, y, z are respectively be the perpendiculars from the circumcentre to the sides of ABC, then prove

a b c abc
that + + = .
x y z 4 xyz

13. Orthocentre and Pedal Triangle: A

M L
The triangle KLM which is formed by joining the feet of the altitudes is
called the Pedal Triangle. P
(i) Its angles are  2A,  2B and  2C.

B C
(ii) Its sides are a cosA = R sin 2A, K
b cosB = R sin 2B and
c cosC = R sin 2C

(iii) Circumradii of the triangles PBC, PCA, PAB and ABC are equal.

RESONANCE 44
14. Excentral Triangle:
The triangle formed by joining the three excentres 1, 2 and 3 of  ABC is called
the excentral or excentric triangle.
(i)  ABC is the pedal triangle of the  1 2 3.
(ii) Its angles are
 A  B  C
&  .
I2
 , 
I3 A
2 2 2 2 2 2
A
(iii) Its sides are 4 R cos ,
2
I
B C
4 R cos & 4 R cos .
2 2
B ) C
A )   B 900
(iv) 1 = 4 R sin ; 2 2
2
B C
2 = 4 R sin ; 3 = 4 R sin .
2 2 I1
(v) Incentre  of  ABC is the
orthocentre of the excentral
  1  2  3.

15. Distance Between Special Points :


(i) Distance between circumcentre and orthocentre

OH2 = R2 (1 – 8 cosA cos B cos C)


(ii) Distance between circumcentre and incentre
A B C
O2 = R2 (1 – 8 sinsin sin ) = R2 – 2Rr
2 2 2
(iii) Distance between circumcentre and centroid
1 2
OG2 = R2 – (a + b2 + c2)
9

Example : In  is the incentre and 1, 2, 3 are the centres of escribed circles of the ABC, prove that
(i) 1. 2 . 3 = 16R2r
(ii) 12 + 232 = 22 + 312 = 32 + 122
Solution. (i)  We know that
A B C
1 = a sec , 2 = b sec and 3 = c sec
2 2 2

C A B
12 = c. cosec , 2 3 = a cosec and 31 = b cosec
2 2 2

A B C
1 . 2 . 3 = abc secsec .sec ........(i)
2 2 2

 a = 2R sin A, b = 2R sinB and c = 2R sinC

 equation (i) becomes


A B C
1. 2 . 3 = (2R sin A) (2R sin B) (2R sinC) sec sec sec
2 2 2

RESONANCE 45
 A A B B  C C
 2 sin cos   2 sin cos   2 sin cos 
 2 2  2 2  2 2
= 8R3 .
A B C
cos . cos . cos
2 2 2

A B C A B C
= 64R3 sin sin sin r = 4R sin sin sin
2 2 2 2 2 2

 1 . 2 . 3 = 16R r Hence Proved


2

(ii) 1 + 23 = 2 + 31 = 3 + 12


2 2 2 2 2 2

A A a2
1 + 23 = a2 sec2 + a2 cosec2 =
2 2

2 2 A A
sin 2 cos 2
2 2

A A
a = 2 R sinA = 4R sin cos
2 2

A A
16 R 2 sin2 . cos 2
2 2
 1 + 23 = = 16R
2 2 2
A A
sin 2
. cos 2
2 2
Similarly we can prove 22 + 312 = 32 + 122 = 16R
2

Hence 12 + 232 = 22 + 312 = 32 + 122

Self Practice Problem :


1. In a ABC, if b = 2 cm, c = 3 cm and A = , then find distance between its circumcentre and
6
incentre.

Ans. 2  3 cm

RESONANCE 46
Part : (A) Only one correct option

1. In a triangle ABC, (a + b + c) (b + c  a) = k. b c, if :
(A) k < 0 (B) k > 6 (C) 0 < k < 4 (D) k > 4

2 9 3
2. In a ABC, A = , b – c = 3 3 cm and ar (ABC) = cm 2. Then a is
3 2

(A) 6 3 cm (B) 9 cm (C) 18 cm (D) none of these

b2  c 2
3. If R denotes circumradius, then in ABC, is equal to
2a R
(A) cos (B – C) (B) sin (B – C) (C) cos B – cos C (D) none of these

4. If the radius of the circumcircle of an isosceles triangle PQR is equal to PQ (= PR), then the angle P is
   2
(A) (B) (C) (D)
6 3 2 3

acosA  bcosB  ccosC


5. In a  ABC, the value of is equal to:
abc
r R R 2r
(A) (B) (C) (D)
R 2r r R

6. In a right angled triangle R is equal to


sr s r sr
(A) (B) (C) s – r (D)
2 2 a

7. In a ABC, the inradius and three exradii are r, r 1, r 2 and r 3 respectively. In usual notations the value of
r. r1. r2. r3 is equal to
abc
(A) 2 (B) 2 (C) (D) none of these
4R

8. In a triangle if r 1 > r 2 > r3, then


(A) a > b > c (B) a < b < c (C) a > b and b < c (D) a < b and b > c

1 1 1 1  1 1 KR 3
With usual notation in a  ABC  r  r        =
9. r  r  ,
 1 2  2 r3   3 r1  a2b2c 2
where 'K' has the value equal to:
(A) 1 (B) 16 (C) 64 (D) 128

10. The product of the arithmetic mean of the lengths of the sides of a triangle and harmonic mean of the
lengths of the altitudes of the triangle is equal to:
(A)  (B) 2  (C) 3  (D) 4 

11. In a triangle ABC, right angled at B, the inradius is:


AB  BC  AC AB  AC  BC AB  BC  AC
(A) (B) (C) (D) None
2 2 2

RESONANCE 47
12. The distance between the middle point of BC and the foot of the perpendicular from A is :

 a2  b2  c 2 b2  c 2 b2  c 2
(A) (B) (C) (D) none of these
2a 2a bc

sin BAD
13. In a triangle ABC, B = 60° and C = 45°. Let D divides BC internally in the ratio 1 : 3, then, =
sin CAD

2 1 1 1
(A) (B) (C) (D)
3 3 6 3

14. Let f, g, h be the lengths of the perpendiculars from the circumcentre of the  ABC on the sides a, b and
a b c abc
c respectively. If   =  then the value of  is:
f g h f gh
(A) 1/4 (B) 1/2 (C) 1 (D) 2

15. A triangle is inscribed in a circle. The vertices of the triangle divide the circle into three arcs of length
3, 4 and 5 units. Then area of the triangle is equal to:

9 3 (1  3 ) 9 3 ( 3  1) 9 3 (1  3 ) 9 3 ( 3  1)
(A) (B) (C) (D)
 2
 2 2 2
2 2

16. If in a triangle ABC, the line joining the circumcentre and incentre is parallel to BC, then
cos B + cos C is equal to:
(A) 0 (B) 1 (C) 2 (D) none of these

17. If the incircle of the  ABC touches its sides respectively at L, M and N
and if x, y, z be the circumradii of the triangles MIN, NIL and LIM where
I is the incentre then the product xyz is equal to:

1 1
(A) R r2 (B) r R2 (C) R r2 (D) r R2
2 2

r 1 A  tan B  tan C 
18. If in a ABC, r = , then the value of tan   is equal to :
1 2 2  2 2

1
(A) 2 (B) (C) 1 (D) None of these
2

r1 r2 r3
19. In any ABC, then minimum value of is equal to
r3
(A) 3 (B) 9 (C) 27 (D) None of these

20. In a acute angled triangle ABC, AP is the altitude. Circle drawn with AP as its diameter cuts the sides
AB and AC at D and E respectively, then length DE is equal to
   
(A) (B) (C) (D)
2R 3R 4R R

21. AA1, BB1 and CC1 are the medians of triangle ABC whose centroid is G. If the concyclic, then points
A, C1, G and B1 are
(A) 2b2 = a2 + c 2 (B) 2c2 = a2 + b2 (C) 2a2 = b2 + c 2 (D) None of these

RESONANCE 48
22. In a ABC, a, b, A are given and c 1, c 2 are two values of the third side c. The sum of the areas of two
triangles with sides a, b, c 1 and a, b, c 2 is
1 2 1
(A) b sin 2A (B) a2 sin 2A (C) b2 sin 2A (D) none of these
2 2


23. In a triangle ABC, let C = . If r is the inradius and R is the circumradius of the triangle, then 2(r + R)
2
is equal to [IIT - 2000]
(A) a + b – c (B) b + c (C) c + a (D) a + b + c

24. Which of the following pieces of data does NOT uniquely determine an acute - angled triangle
ABC (R being the radius of the circumcircle )? [IIT - 2002]
(A) a , sin A, sin B (B) a, b, c (C) a, sin B, R (D) a, sin A, R

25. If the angles of a triangle are in the ratio 4 : 1 : 1, then the ratio of the longest side to the perimeter is
(A) 3 : (2 + 3) (B) 1 : 6 (C) 1 : 2 + 3 (D) 2 : 3 [IIT - 2003]

26. The sides of a triangle are in the ratio 1 : 3 : 2, then the angle of the triangle are in the ratio
[IIT - 2004]
(A) 1 : 3 : 5 (B) 2 : 3 : 4 (C) 3 : 2 : 1 (D) 1 : 2 : 3

27. In an equilateral triangle, 3 coincs of radii 1 unit each are kept so that they touche each other and also
the sides of the triangle. Area of the triangle is [IIT - 2005]

7 3 7 3
(A) 4 + 2 3 (B) 6 + 4 3 (C) 12 + (D) 3 +
4 4

PA 2  PB 2  PC 2  PD 2
28. If P is a point on C1 and Q is a point on C2, then equals
QA 2  QB 2  QC 2  QD 2
(A) 1/2 (B) 3/4 (C) 5/6 (D) 7/8

29. A circle C touches a line L and circle C1 externally. If C and C1 are on the same side of the line L, then
locus of the centre of circle C is
(A) an ellipse (B) a circle (C) a parabola (D) a hyperbola

30. Let  be a line through A and parallel to BD. A point S moves such that its distance from the line BD and
the vertex A are equal. If the locus of S meets AC in A1, and  in A2 and A3, then area of A1 A2A3 is
(A) 0.5 (unit)2 (B) 0.75 (unit)2 (C) 1 (unit)2 (D) (2/3) (unit)2

Part : (B) May have more than one options correct

31. In a ABC, following relations hold good. In which case(s) the triangle is a right angled triangle?
(A) r2 + r 3 = r 1  r (B) a2 + b2 + c 2 = 8 R2 (C) r1 = s (D) 2 R = r1  r

32. In a triangle ABC, with usual notations the length of the bisector of angle A is :

A
2 bc cos A 2 bc sin A abc cos ec 2 . A
2 2 2
(A) (B) (C) (D) b  c cos ec 2
bc bc 2R (b  c )

RESONANCE 49
33. AD, BE and CF are the perpendiculars from the angular points of a  ABC upon the opposite sides,
then :
Perimeter of DEF r
(A)  (B) Area of DEF = 2  cosA cosB cosC
Perimeter of ABC R
R
(C) Area of AEF =  cos2A (D) Circum radius of DEF =
2

34. The product of the distances of the incentre from the angular points of a  ABC is:
abc  R abc  r
(A) 4 R2 r (B) 4 Rr2 (C) (D)
s s

35. In a triangle ABC, points D and E are taken on side BC such that BD = DE = EC. If angle
ADE = angle AED = , then:
(A) tan = 3 tan B (B) 3 tan = tanC
6 tan 
(C) = tan A (D) angle B = angle C
tan2   9

36. With usual notation, in a  ABC the value of  (r1  r) can be simplified as:

A  a b c 2
R a  b  c
(A) abc  tan (B) 4 r R 2
(C) (D) 4 R r2
2
2

cos A  2 cos C sin B


1. If in a triangle ABC, = , prove that the triangle ABC is either isosceles or right
cos A  2 cos B sin C
angled.

 A  B
2. In a triangle ABC, if a tan A + b tan B = (a + b) tan   , prove that triangle is isosceles.
 2 

 r  r 
3. If  1  1   1  1  = 2 then prove that the triangle is the right triangle.
 r2   r3 

4. In a  ABC,  C = 60° &  A = 75°. If D is a point on AC such that the area of the BAD is 3 times
the area of the BCD, find the  ABD.

5. The radii r1, r2, r3 of escribed circles of a triangle ABC are in harmonic progression. If its area is 24 sq.
cm and its perimeter is 24 cm, find the lengths of its sides.

6. ABC is a triangle. D is the middle point of BC. If AD is perpendicular to AC, then prove that

cos A. cos C =

2 c 2 a 2
.

3ac

7. Two circles, of radii a and b, cut each other at an angle  Prove that the length of the common chord is
2ab sin 
.
a 2  b 2  2ab cos 

RESONANCE 50
8. In the triangle ABC, lines OA, OB and OC are drawn so that the angles OAB, OBC and OCA are each
equal to , prove that
(i) cot  = cot A + cot B + cot C
(ii) cosec2  = cosec2 A + cosec2 B + cosec2 C

9. In a plane of the given triangle ABC with sides a, b, c the points A, B, C are taken so that the
 A BC,  ABC and  ABC are equilateral triangles with their circum radii Ra, Rb, Rc; inradii r a, r b, r c
& exradii r a , rb & rc respectively. Prove that;

(i)  ra:  Ra:  ra = 1: 8: 27 (ii) r1 r2 r3 =


 3R 6r 2r   tan A
a a a
3

648 3 2

 3  2  . Find the acute


10. The triangle ABC is a right angled triangle, right angle at A. The ratio of the radius of the circle

circumscribed to the radius of the circle escribed to the hypotenuse is, 2 :


angles B & C. Also find the ratio of the two sides of the triangle other than the hypotenuse.

 3  2  . Find the acute


11. The triangle ABC is a right angled triangle, right angle at A. The ratio of the radius of the circle

circumscribed to the radius of the circle escribed to the hypotenuse is, 2 :


angles B & C. Also find the ratio of the two sides of the triangle other than the hypotenuse.

12. If the circumcentre of the  ABC lies on its incircle then prove that,

cosA + cosB + cosC = 2

13. Three circles, whose radii area a, b and c, touch one another externally and the tangents at their points
of contact meet in a point; prove that the distance of this point from either of their points of contacts
1
 abc  2
is   .
abc 

EXERCISE # 1 EXERCISE # 2
1. C 2. B 3. B 4. D 5. A 6. B 7. B 4.  ABD = 30° 5. 6, 8, 10 cms

8. A 9. C 10. B 11. A 12. B 13. C 14. A


5  b
10. B = ,C= , = 2 3
15. A 16. B 17. C 18. B 19. C 20. D 21. C 12 12 c

22. A 23. A 24. D 25. A 26. A 27. B 28. B


5  b
11. B = ,C= , = 2 3
29. C 30. C 31. ABCD 32. ACD 33. ABCD 12 12 c

34. BD 35. ACD 36. ACD

RESONANCE 51

You might also like